Sunteți pe pagina 1din 758

VESTIBULAR 97

EDITORA MODERNA
1

NDICE GERAL

BIOLOGIA ....................................... 3
CESGRANRIO .............................................. 3 FUVEST ....................................................... 9 UFBA........................................................ 18 UFMG ....................................................... 30 UFPA ........................................................ 54 UFPE ........................................................ 58 UFRGS ...................................................... 72 UnB ......................................................... 83 UNESP ...................................................... 91

INGLS........................................ 393
CESGRANRIO .......................................... 393 FUVEST ................................................... 399 UFBA...................................................... 403 UFMG ..................................................... 414 UFPA ...................................................... 425 UFPE ...................................................... 428 UFRGS .................................................... 434 UnB ....................................................... 442 UNESP .................................................... 448

FSICA ........................................... 96
CESGRANRIO ............................................ 96 FUVEST ................................................... 102 UFBA...................................................... 113 UFMG ..................................................... 120 UFPA ...................................................... 143 UFPE ...................................................... 151 UFRGS .................................................... 167 UnB ....................................................... 177 UNESP .................................................... 184

MATEMTICA .............................. 452


CESGRANRIO .......................................... 452 FUVEST ................................................... 457 UFBA...................................................... 462 UFMG ..................................................... 473 UFPA ...................................................... 485 UFPE ...................................................... 488 UFRGS .................................................... 502 UnB ....................................................... 509 UNESP .................................................... 518

GEOGRAFIA................................. 193
CESGRANRIO .......................................... 193 FUVEST ................................................... 201 UFBA...................................................... 213 UFMG ..................................................... 225 UFPA ...................................................... 251 UFPE ...................................................... 257 UFRGS .................................................... 271 UnB ....................................................... 284 UNESP .................................................... 290

PORTUGUS ................................ 525


CESGRANRIO .......................................... 525 FUVEST ................................................... 534 UFBA...................................................... 545 UFMG ..................................................... 572 UFPA ...................................................... 591 UFPE ...................................................... 603 UFRGS .................................................... 626 UnB ....................................................... 653 UNESP .................................................... 661

HISTRIA .................................... 299


CESGRANRIO .......................................... 299 FUVEST ................................................... 306 UFBA...................................................... 313 UFMG ..................................................... 330 UFPA ...................................................... 347 UFPE ...................................................... 352 UFRGS .................................................... 364 UnB ....................................................... 379 UNESP .................................................... 387

QUMICA..................................... 671
CESGRANRIO .......................................... 671 FUVEST ................................................... 676 UFBA...................................................... 687 UFMG ..................................................... 696 UFPA ...................................................... 716 UFPE ...................................................... 720 UFRGS .................................................... 732 UnB ....................................................... 745 UNESP .................................................... 752

BIOLOGIA

CESGRANRIO

1. Nos laboratrios qumicos, a maneira mais freqente de ativar uma reao fornecendo calor, que funciona como energia de ativao. Nos seres vivos, isso no possvel, pois corre-se o risco de as protenas serem desnaturadas. A estratgia desenvolvida pelos seres vivos para superar a barreira inicial das reaes foi a utilizao de: a) ATP. b) enzimas. c) hormnios. 2.
Glicose + O2 CO2 + H2O

d) glicose. e) clorofila.

Aminocidos energia

Protenas

Sobre o esquema acima, so feitas as seguintes afirmativas: I - a formao de molculas de protenas uma reao de degradao; II - atravs de reaes de sntese que o ser vivo consegue energia para a sua vida; III - o conjunto das reaes de sntese e degradao constituem o metabolismo. A(s) afirmativa(s) correta(s) (so): a) apenas a I. b) apenas a II. c) apenas a III. d) apenas a I e a II. e) apenas a II e a III.

3. Observe o esquema ao lado e analise as seguintes afirmaes: I - a transferncia de eltrons para os aceptores permite a transformao de energia luminosa em energia qumica; II - na ausncia de aceptores de eltrons, poderia haver a ocorrncia do fenmeno conhecido como fluorescncia; III - quando excitada pela luz, a clorofila absorve principalmente luz verde. A(s) afirmao(es) corretas(s) (so): a) apenas a I. b) apenas a II. c) apenas a I e a II. d) apenas a I e a III. e) apenas a II e a III.

BIOLOGIA
4. Comparando o esquema dos dois processos metablicos representados ao lado, podemos afirmar que o(a): a) aceptor final de hidrognios, na fermentao, o oxignio. b) molcula de glicose totalmente degradada, na fermentao. c) fermentao encontrada na maioria dos seres vivos unicelulares. d) formao de ATP na cadeia respiratria s ocorre na respirao. e) formao de cido pirvico uma exclusividade da respirao. 5.

CESGRANRIO

Observando o esquema acima, que representa um neurnio em repouso, podemos afirmar que, nestas condies: a) se a membrana do neurnio for atingida por um estmulo, as quantidades de ons Na e K dentro e fora da membrana se igualam. b) devido diferena de cargas entre as faces externa e interna, o neurnio est polarizado. c) a ocorrncia do impulso nervoso depende de estmulos de natureza eltrica. + d) a quantidade de ons K menor na parte interna do neurnio devido sua sada por osmose. + + e) as concentraes dos ons Na e K se fazem sem gasto de energia, sendo exemplo de transporte ativo. 6. Eliminao das clulas sangneas que esto velhas demais. Formao de uria. Armazenamento de energia para qualquer eventualidade. Maior glndula do corpo humano.
+ +

Todas essas caractersticas tpicas de um sper-rgo esto relacionadas ao(): a) bao. b) rim. c) pncreas. d) fgado. e) hipfise.

BIOLOGIA

CESGRANRIO

7. A presena de oprculo, estrutura que recobre as brnquias em peixes sseos, permite eficincia nas trocas gasosas mesmo com o peixe parado. Isto porque o oprculo possibilita melhor captao de oxignio devido (ao): a) b) c) d) e) quebra das molculas de gua. entrada de gua pelas brnquias retirada de gases da bexiga natatria. transporte ativo realizado por esta estrutura. maior contato da gua com as brnquias.

8. Lixo de Proveta Aproximadamente 3.300 embries humanos congelados foram dissolvidos em gua e lcool na Inglaterra.
(Veja: agosto/96)

Os chamados bebs de proveta, apesar de serem fecundados em frascos de vidro, so mais tarde transferidos para o tero da mulher. A estrutura embrionria que funcionar como rgo de respirao e excreo do embrio o(a): a) alantide. b) mnio. c) crion. d) saco vitelnico. e) placenta.

9. O sufocamento por alimento responsvel por quase 3.000 mortes, anualmente, nos USA, mais do que os acidentes com armas de fogo ou avies. O sufocamento ocorre quando uma poro do alimento bloqueia o(a): a) brnquio. b) esfago. c) glote. d) laringe. e) faringe.
Rainha (fmea frtil diplide) Zango (macho frtil haplide)

10. Pela observao do esquema ao lado, podemos concluir que o zango foi originado por: a) gametognese. b) partenognese. c) gemulao. d) conjugao. e) brotamento.

BIOLOGIA
11. No grfico ao lado temos representado um exemplo de doena ligada ao sexo onde apenas o indivduo IV afetado. Feita a anlise desse grfico, podemos afirmar corretamente que: a) b) c) d) e) a anomalia causada por um gen dominante. o indivduo I obrigatoriamente homozigoto. o cromossoma y paterno pode apresentar o gen para a doena. o casal apresenta 50% de probabilidade de ter filhas afetadas. os indivduos III e V podem ser homozigotos ou heterozigotos.

CESGRANRIO

12. O esquema abaixo evidencia que a formao de melanina no depende apenas da ao de um gen. No processo ali representado, est ocorrendo a ao conjunta de dois gens.
gene 1 Fenilalanina enzima 1

gene 2 Tirosina enzima 2

Melanina

Situaes como essa so conhecidas como: a) polialelia. b) poligens. c) norma de reao. 13. d) interao gnica. e) alelos mltiplos.

O esquema acima representa quatro populaes de uma mesma espcie que vivem em uma mesma rea num determinado perodo de tempo. A sua anlise nos permite tirar as concluses abaixo, EXCETO uma. Assinale-a. a) Em 1 as quatro populaes podem reproduzir-se entre si. b) Em 2 a migrao de parte da populao b pode iniciar um processo de especiao. c) O isolamento geogrfico ocorrido em 2 submete a populao b a presses seletivas diferenciadas das populaes em 1. d) Se as diferenas presentes em b* atingirem o aspecto reprodutivo, verifica-se em 3 a impossibilidade de troca gnica com as outras populaes. e) Tendo ocorrido isolamento reprodutivo em 3, b* ser uma subespcie de b.

BIOLOGIA
14. Analise as afirmativas abaixo, a respeito das mutaes.

CESGRANRIO

I - Sempre que o ambiente se torna desfavorvel, o ser vivo reage sofrendo uma mutao gnica. II - As mutaes transmitidas s geraes futuras so aquelas que ocorrem em clulas germinativas. III - As mutaes ocorridas em clulas somticas so de grande valor adaptativo para a perpetuao da espcie. Est(o) correta(s): a) I apenas. b) II apenas. c) III apenas. d) I e II apenas. e) II e III apenas.

15. Cientistas americanos descobrem num meteorito de Marte, que caiu sobre a Antrtida, fortes indcios de vida fora da Terra. Entre as certezas e dvidas levantadas por tal fato, ainda sob a luz das teorias atuais, podemos afirmar que as primeiras formas de vida surgidas no nosso planeta eram: a) b) c) d) e) todas auttrofas devido escassez de alimentos nos oceanos primitivos. fermentadoras que utilizavam a energia radiante para produzir suas molculas orgnicas. hetertrofas que utilizavam substncias formadas na atmosfera e acumuladas nos mares primitivos. fungos primitivos com capacidade de atividade fotossinttica. aerbias graas abundncia de tomos de oxignio existente nas guas do oceano.

16. Urubu-rei, urubu-preto, condor-chileno, cabea-vermelha so nomes de uma mesma ave que desempenha o mesmo papel ecolgico dos abutres e das hienas: livrar a natureza de animais em estado de putrefao. As relaes ecolgicas estabelecidas por esses animais na natureza podem ser classificadas como: a) canibalismo. b) comensalismo. c) mutualismo. 17. Bactrias Faxineiras Uma das mais ativas participantes da comunidade que habita o andar de cima dos oceanos a P. aeroginosa, bactria que consegue degradar o leo despejado no mar.
(Superinteressante: agosto/96)

d) amensalismo. e) protocooperao.

A atividade dessa bactria importante para os ecossistemas porque: a) b) c) d) e) aumenta a entrada de luz do Sol para a fotossntese. aumenta a oxigenao das guas, garantindo o trabalho das bactrias anaerbias. contribui para o ciclo do carbono, por meio da degradao de hidrocarbonetos. permite maior produo de carbono oriundo do leo degradado. facilita a ao dos predadores marinhos atravs da quebra do leo em partculas menores.

BIOLOGIA
18. Um Dia da Terra Sombrio

CESGRANRIO

A comemorao do Dia da Terra, amanh, tem tudo para ser sombria. Os ecossistemas no planeta continuam se deteriorando em quase todas as categorias, segundo um novo relatrio publicado para a ocasio pelas Naes Unidas. Pelas estimativas, entre 5% e 20% das espcies de plantas e animais esto ameaados de extino no futuro prximo.
(Jornal do Brasil, 21/04/96)

Aps a anlise da notcia em destaque, podemos concluir que todos os processos abaixo relacionados esto ocorrendo, EXCETO um. Assinale-o. a) b) c) d) e) Intensificao da eroso. Intensificao do efeito estufa. Diminuio da camada de oznio. Reduo da magnificao trfica. Reduo da biodiversidade.

19. Certos fungos so empregados na produo de queijos, sendo responsveis por sabores caractersticos. Os fungos Penicillium roquefortii e Penicillium camembertii, por exemplo, so utilizados na fabricao de queijos tipos roquefort e camembert, respectivamente. Pela anlise dos nomes cientficos acima citados, podemos concluir que esses seres NO pertencem ao() mesmo(a): a) gnero. b) classe. c) famlia. d) ordem. e) espcie.

20. Alterao gentica de transmissor pode acabar com a dengue. A engenharia gentica est prometendo uma soluo para um dos flagelos dos pases tropicais, como o Brasil. A idia impedir a implantao do agente causal no transmissor.
(Folha de S.Paulo, 12/05/96)

O transmissor e o agente causal da dengue so, respectivamente, um: a) b) c) d) e) mosquito e um vrus. mosquito e uma bactria. percevejo e um vrus. percevejo e uma bactria. barbeiro e um protozorio.

BIOLOGIA
1. Examine a rvore filogentica abaixo:

FUVEST
F A S E

Esperamos encontrar maior semelhana entre genes de a) bactria e protozorio. b) peixe e baleia. c) baleia e pssaro. d) estrela-do-mar e ostra. e) ostra e coral.

2. Um estudante levantou algumas hipteses para explicar porque em alguns rios de caverna os peixes so cegos. Qual delas est de acordo com a teoria sinttica da evoluo ? a) No ambiente escuro das cavernas, os olhos se atrofiaram como conseqncia da falta de uso. b) Os olhos, sem utilidade na escurido das cavernas, se transformaram ao longo do tempo em rgos tteis. c) No ambiente escuro das cavernas, os peixes cegos apresentaram vantagens adaptativas em relao aos no cegos. d) A falta de luz nas cavernas induziu mutao deletria drstica que levou regresso dos olhos num curto espao de tempo. e) A falta de luz nas cavernas induziu mutaes sucessivas que ao longo de muitas geraes levaram regresso dos olhos. 3. De uma populao de 100 camundongos foi retirado ao acaso um indivduo com deficincia da enzima E, carter condicionado por um alelo recessivo a. correto afirmar que a) b) c) d) e) seus pais podem ser fenotipicamente normais. seus pais so certamente heterozigotos. a freqncia do alelo a de 0,1. a freqncia do alelo a de 0,2. 1% dos indivduos da populao tm deficincia da enzima E.

BIOLOGIA

FUVEST
F A S E

4. Uma mulher normal, casada com um portador de doena gentica de herana autossmica dominante, est grvida de um par de gmeos. Qual a probabilidade de que pelo menos um dos gmeos venha a ser afetado pela doena no caso de serem, respectivamente, gmeos monozigticos ou dizigticos ? a) 25% a 50% b) 25% a 75% c) 50% a 25% d) 50% a 50% e) 50% a 75%

5. Um surfista que se expunha muito ao sol sofreu danos em seu DNA em conseqncia de radiaes UV, o que resultou em pequenos tumores na pele. Caso ele venha a ser pai de uma criana, ela a) b) c) d) e) s herdar os tumores se tiver ocorrido dano em um gene dominante. s herdar os tumores se tiver ocorrido dano em dois genes recessivos. s herdar os tumores se for do sexo masculino. herdar os tumores, pois houve dano no material gentico. no herdar os tumores.

6. Uma mulher de sangue tipo A, casada com um homem de sangue tipo B, teve um filho de sangue tipo O. Se o casal vier a ter outros 5 filhos, a chance deles nascerem todos com sangue do tipo O a) b) c) d) e) igual chance de nascerem todos com sangue do tipo AB. menor que a chance de nascerem todos com sangue do tipo AB. maior que chance de nascerem todos com sangue do tipo AB. menor que a chance de nascerem sucessivamente com sangue do tipo AB, A, B, A e B. maior que a chance de nascerem sucessivamente com sangue do tipo AB, B, B, A e A.

7. Enzimas de restrio so fundamentais Engenharia Gentica porque permitem a) b) c) d) e) a passagem do DNA atravs da membrana celular. inibir a sntese de RNA a partir de DNA. inibir a sntese de DNA a partir de RNA. cortar DNA onde ocorrem seqncias especficas de bases. modificar seqncias de bases do DNA.

8. Um pesquisador fez o seguinte desenho de uma clula observada ao microscpio ptico. Pode tratar-se de uma clula de a) ovrio. b) sangue. c) linfa. d) medula ssea. e) pele.

10

BIOLOGIA
9. Analise os eventos mitticos relacionados abaixo:

FUVEST
F A S E

I II III IV

Desaparecimento da membrana nuclear. Diviso dos centrmeros. Migrao dos cromossomos para os plos do fuso. Posicionamento dos cromossomos na regio mediana do fuso.

Qual das alternativas indica corretamente sua ordem temporal ? a) IV - I - II - III. b) I - IV - III - II. c) I - II - IV - III. d) I - IV - II - III. e) IV - I - III - II.

10. Que doenas poderiam ser evitadas com a eliminao de reservatrios de gua parada onde se reproduzem insetos vetores ? a) b) c) d) e) Clera, dengue e esquistossomose Clera, dengue e malria Clera, esquistossomose e febre amarela Dengue, febre amarela e malria Esquistossomose, febre amarela e malria

11. Examine a seguinte lista de eventos que ocorrem durante a propagao de um impulso nervoso:

I II III IV V

Neurotransmissores atingem os dendritos. Neurotransmissores so liberados pelas extremidades do axnio. O impulso se propaga pelo axnio. O impulso se propaga pelos dendritos. O impulso chega ao corpo celular.

Que alternativa apresenta a seqncia temporal correta desses eventos ? a) V - III - I - IV - II b) I - IV - V - III - II c) I - IV - III - II - V d) II - I - IV - III - V e) II - III - I - IV - V

11

BIOLOGIA

FUVEST
F A S E

12. Na genealogia abaixo, os smbolos cheios representam pessoas afetadas por uma doena gentica rara.

O padro de herana que melhor explica o heredograma a) b) c) d) e) autossmico dominante, porque a doena afeta os dois sexos. autossmico dominante, porque a doena aparece em todas as geraes. autossmico dominante, porque aproximadamente 50% da prole afetada. dominante ligado ao sexo, porque todas as filhas de homens afetados so afetadas. recessivo ligado ao sexo, porque no h transmisso de homem para homem.

13. Em caso de hipertenso, recomenda-se uma dieta sem sal porque esta atua a) b) c) d) e) diminuindo o volume de sangue circulante. aumentando o volume de sangue circulante. reduzindo o calibre dos vasos sangneos. dilatando o calibre dos vasos sangneos. obstruindo os capilares arteriais com placas de heteroma.

14. Em um ambiente com temperatura mantida constante em 18C, qual dos animais abaixo necessitar maior consumo de alimento em relao ao tamanho de seu corpo ? a) Sapo . b) Jacar. c) Sabi. d) Tubaro. e) Jararaca.

15. Uma das causas de dor e sensao de queimao nos msculos, decorrentes de esforo fsico intenso, a presena de muito cido lctico nas clulas musculares. Isso ocorre quando essas clulas a) b) c) d) e) realizam intensa respirao celular, com produo de cido lctico. recebem suprimento insuficiente de gs oxignio e realizam fermentao. realizam intensa respirao celular produzindo excesso de ATP. recebem estmulos nervosos sucessivos e acumulam neurotransmissores. utilizam o acar lactose como fonte de energia.

12

BIOLOGIA

FUVEST
F A S E

16. Em qual das alternativas abaixo as trs funes mencionadas so realizadas pelo fgado ? a) b) c) d) e) Regular o nvel de glicose no sangue, transformar amnia em uria, produzir bile. Regular o nvel de glicose no sangue, transformar amnia em uria, secretar quimotripsina. Regular o nvel de glicose no sangue, produzir cido clordrico, secretar quimotripsina. Produzir bile, transformar amnia em uria, produzir cido clordrico. Produzir bile, produzir cido clordrico, secretar quimotripsina.

17. Mediu-se a taxa de fotossntese em plantas submetidas a diferentes condies de temperatura e de luz. Foram utilizadas duas intensidades luminosas: uma baixa, prxima ao ponto de compensao ftico (representada nos grficos por linha interrompida) e outra alta, bem acima do ponto de compensao ftico (representada nos grficos por linha contnua). Qual dos grficos representa melhor os resultados obtidos ?

18. Analise os seguintes eventos que podem ocorrer em conseqncia do despejo de esgoto urbano em lagos:

I II III IV V

Morte de organismos aerbicos, tanto auttrofos quanto hetertrofos. Diminuio do teor de gs oxignio na gua. Aumento de nutrientes nitrogenados e fosfatados. Proliferao de microrganismos aerbicos. Proliferao de algas e bactrias fotossintetizantes.

Indique a alternativa que apresenta a seqncia temporal correta desses eventos. a) I - II - III - IV - V. b) III - II - I - V - IV. c) III - V - IV - II - I. d) IV - II - I - III - V. e) IV - II - III - V - I.

13

BIOLOGIA
19. Considere o diagrama abaixo, representativo do ciclo de vida de uma planta.

FUVEST
F A S E

Qual alternativa completa corretamente o diagrama ? X esporo esporo gameta gameta zigoto Y zigoto gameta esporo zigoto esporo Z gameta zigoto zigoto esporo gameta

a) b) c) d) e)

20. Um pesquisador dividiu um lote de plantas jovens em quatro grupos, dos quais trs receberam os tratamentos indicados abaixo e o quarto foi usado como controle.

As plantas foram ento iluminadas unilateralmente. Quais plantas se curvam em direo fonte de luz, tal como os controles ? a) Nenhuma delas. II. b) Somente as plantas do grupo I. c) Somente as plantas do grupo II. d) Somente as plantas do grupo I e e) As plantas dos grupos I, II e III.

14

BIOLOGIA

FUVEST
F A S E

1. a) Escreva a equao do processo bioqumico da respirao aerbica. Qual a funo desse processo e onde ele ocorre no corpo humano ? b) Complete os esquemas dos percursos do gs oxignio e do gs carbnico, participantes da respirao, preenchendo os espaos com as letras correspondentes s estruturas abaixo. A = alvolos pulmonares B = artria pulmonar C = artrias do corpo D = trio direito E = trio esquerdo F = clulas do corpo G = veia pulmonar H = veias do corpo I = ventrculo direito J = ventrculo esquerdo

2. Explique, de acordo com a teoria sinttica da evoluo, as adaptaes mencionadas nos textos abaixo. a) Durante o longo inverno da regio rtica, a plumagem de certas aves e a pelagem de certos mamferos tornam-se brancas, voltando a adquirir colorao escura no incio da primavera. b) Algumas espcies de anfbio e de inseto apresentam cores e desenhos marcantes que, ao invs de escond-las, as destacam do ambiente e chamam a ateno de possveis predadores. 3. Um pesquisador forneceu a uma cultura de algas gs carbnico marcado com o istopo O do oxignio. A uma segunda cultura de algas foi fornecida gua com esse mesmo istopo. As culturas foram mantidas iluminadas por um certo tempo, aps o que as substncias qumicas presentes no meio e nas clulas das algas foram analisadas. a) Alm de gs carbnico, que outras substncias apresentaro o istopo O na primeira cultura? Justifique sua resposta. 18 b) Alm da gua, que outras substncias apresentaro o istopo O na segunda cultura ? Justifique a resposta. 4. Uma conquista recente no campo da biotecnologia o uso de bactrias para a produo de protena animal de interesse comercial. Por exemplo, hoje j esto sendo comercializadas insulina e somatotrofina (ou somatropina) humanas produzidas por bactrias. a) Em que locais do corpo humano so produzidas essas protenas e qual a principal funo de cada uma delas no organismo ? b) Explique sucintamente o processo por meio do qual se modificam bactrias para que elas passem a produzir protenas humanas.
18 18

15

BIOLOGIA

FUVEST
F A S E

5. a) Relacione estrutural e funcionalmente os seguintes componentes de uma planta: vulo, ovrio, semente e fruto. b) Que grupos de plantas produzem sementes? Qual foi a importncia das sementes na adaptao das plantas ao ambiente terrestre? 6. a) comum ouvirmos a frase: J tomei este antibitico tantas vezes que agora j no faz mais efeito. Esta afirmao pode ser verdadeira? Por qu? b) Costuma-se usar dois antibiticos diferentes no tratamento de certas doenas comuns, como a tuberculose, cujo agente causador j bem conhecido. Qual seria a forma biologicamente mais eficiente de administr-los: simultaneamente ou separadamente com um intervalo de 1 ms entre eles? Justifique sua resposta. 7. Em que clulas do corpo humano podemos encontrar as estruturas abaixo e quais so suas funes?

ESTRUTURA

CLULA ONDE PODE SER ENCONTRADA

FUNO

a b c d

microvilosidades clios flagelos pseudpodos

8. Um tratamento utilizado para certos tipos de doenas do sangue a destruio completa da medula ssea do paciente e implante de clulas medulares sadias provenientes de um doador. Eugnio, cujo grupo sangneo A, recebeu um transplante de medula ssea de seu irmo Valentim, cujo grupo sangneo B, e a operao foi bem sucedida. a) Qual ser o grupo sangneo de Eugnio aps o transplante ? Por qu ? b) Sabendo-se que a me e a esposa de Eugnio tm sangue do tipo O, qual ser a probabilidade de um futuro filho do casal ter sangue do tipo A ? E do tipo B ?

16

BIOLOGIA

FUVEST
F A S E

9. O comportamento da hemoglobina humana em relao ao gs oxignio muito diferente no adulto e no feto. a) Qual essa diferena ? b) Por que essa diferena necessria em mamferos placentrios ? 10. Uma mulher clinicamente normal casou-se duas vezes, em ambos os casos com indivduos normais e que no eram seus consangneos. Conforme mostrado na genealogia abaixo, ela teve quatro filhos, dois de cada casamento, todos afetados por uma doena gentica muito rara. a) Que padro de herana explica melhor a genealogia ? Justifique sua resposta. b) Se os dois maridos da mulher fossem irmos isto poderia alterar sua resposta anterior ? Por qu ?

17

BIOLOGIA
QUESTES DE 1 A 7

UFBA
F A S E

INSTRUO:Assinale as proposies verdadeiras, some os nmeros a elas associados e marque o resultado na Folha de Respostas.

Questo 1 O diagrama abaixo ilustra aspectos da estrutura de uma clula vegetal, destacando nveis hierrquicos na organizao de alguns componentes.

Em relao s propriedades que emergem a cada nvel hierrquico, pode-se afirmar:

(1) As propriedades dos tomos so as mesmas expressas por todos os nveis de organizao molecular. (2) A funcionalidade da celulose dependente das propriedades moleculares da glicose. (4) A funo informativa do DNA caracteriza-se como propriedade emergente de um heteropolmero. (8) A organizao da cromatina define a funo codificante do DNA. (16) A versatilidade em integrar diferentes estruturas celulares assegura s protenas maior participao nas funes celulares. (32) A vida de uma clula a expresso particular das propriedades de seus componentes estruturais. (64) A interdependncia das diferentes partes de um sistema biolgico estabelece a sua identidade funcional.

18

BIOLOGIA
Questo 2

UFBA
F A S E

As investigaes que se seguiram proposio da teoria celular vm confirmando a concepo da clula como a unidade anatmica e fisiolgica dos seres vivos e desvendando, cada vez mais, a sua organizao. Com base no conhecimento atual da clula, pode-se afirmar: (1) O interior da clula distinto do meio externo, pela presena de uma membrana que a isola do ambiente. (2) A localizao do material gentico em um compartimento prprio, delimitado por membrana, uma caracterstica universal da estrutura celular. (4) A compartimentao do citoplasma, em decorrncia do desenvolvimento de endomembranas, favoreceu a especializao celular. (8) A dependncia das mitocndrias para a realizao da respirao aerbica uma caracterstica da clula eucaritica. (16) Os ribossomos so as estruturas celulares fundamentais na converso da informao gentica em molculas prprias da clula. (32) Os lisossomos atuam na nutrio e remodelamento celulares, pela presena de enzimas digestivas. (64) A simples presena do DNA assegura a um sistema molecular o nvel de organizao celular. Questo 3 Plantas e animais, embora preservando uma herana comum de mecanismos celulares bsicos para obter e utilizar energia, evoluram, explorando, diferentemente, os recursos ambientais. Disso resultou um acmulo de especializaes que contriburam para a sobrevivncia desses grupos, estabelecendo, entre eles, as suas grandes diferenas. Essa relao entre a tendncia evolutiva das plantas e dos animais e as aquisies morfolgicas e funcionais est expressa em: (1) A divergncia entre plantas e animais estabelecida com a utilizao, pelas plantas, de molculas especilizadas em captar energia do meio abitico. (2) As diferenas biolgicas entre plantas e animais so mantidas por mecanismos adaptativos que proporcionam uma independncia ecolgica entre esses grupos. (4) A relao entre superfcie e volume, nas folhas, reflete uma adaptao para maior eficincia no intercmbio gasoso com o meio. (8) A localizao do pigmento, utilizado para transporte de oxignio, em clulas especializadas favoreceu a evoluo dos vertebrados. (16) A atividade metablica est subordinada eficincia dos sistemas de transporte de nutrientes. (32) A diversidade biolgica se estabelece atravs de alteraes genticas eficazes, exigidas pelo ambiente.

19

BIOLOGIA
Questo 4

UFBA
F A S E

A figura ao lado representa indivduos maduros de sete diferentes tipos de Achillea (A, B ,C, D, E, F, G), que foram cultivados a grande altitude (3 050 m), a altitude mdia (1 400 m) e a pequena altitude (30 m). As mudas de cada tipo foram obtidas por reproduo vegetativa de uma mesma planta parental.

Comparando-se o desenvolvimento de cada tipo de Achillea, nas diferentes altitudes, pode-se dizer: (1) O desenvolvimento do organismo determinado pela interao de seus genes com o ambiente. (2) A reproduo vegetativa garante a expresso de fentipos diversificados. (4) Diferenas no desenvolvimento das plantas cultivadas no mesmo ambiente podem ser atribudas a influncias genticas. (8) O gentipo da planta do tipo A est mais bem adaptado a ambientes de altitude mdia. (16) A variabilidade gentica da Achillea permite a sua adaptao a diferentes ambientes. (32) O mesmo gentipo, em diferentes ambientes, pode produzir diferentes fentipos. (64) Os indivduos herdam da gerao parental as caractersticas fenotpicas. Questo 5
Embora aparentemente homogneo, o ecossistema lacustre apresenta-se bem diferenciado estrutural e funcionalmente. A regio litornea caracterizada por grande diversidade de espcies animais e vegetais e de menor profundidade. A regio pelgica, que a zona mais interior do lago, caracteriza-se pela presena do plncton, na superfcie, e dos peixes, que fazem parte do ncton e podem servir de alimento para aves aquticas. No fundo do lago, vivem principalmente invertebrados e bactrias, que compem o bentos. (ESTEVES, p. 58 texto adaptado)

Com base nas informaes acima e em conhecimentos sobre a dinmica dos ecossistemas, pode-se afirmar: (1) Entre as diferentes regies de um lago, h constantes interaes atravs da troca de energia e matria. (2) O componente fotossinttico do plncton contribui para a entrada de energia no ecossistema lacustre. (4) A produtividade primria do lago depende do ncton. (8) O fundo do lago o principal stio de reciclagem dos nutrientes. (16) Os peixes servem de elo entre o ecossistema lacustre e o terrestre. (32) A grande diversidade de espcies animais e vegetais sugere um pequeno nmero de nichos ecolgicos, na regio litornea. (64) A circulao da gua, levando nutrientes para as regies iluminadas, uma condio que garante a produtividade do ecossistema lacustre.

20

BIOLOGIA
Questo 6 Milhares de anos podem decorrer, at que um lago apresente as condies ideais para a proliferao de peixes e plantas aquticas, num processo de eutrofizao, que pode ser desencadeado naturalmente pelo aporte de nutrientes, ou acelerado pela ao do homem. O diagrama ao lado registra as variaes de alguns fatores presentes num ecossistema lacustre, em decorrncia de um processo artificial de eutrofizao. Com base nas informaes dadas e na anlise do diagrama, pode-se concluir:

UFBA
F A S E

(1) O maior aporte de nutrientes proporciona a diminuio da biomassa de algas. (2) Os nutrientes fosfato e nitrato so fatores limitantes, que contribuem para manter o equilbrio do ecossistema. (4) Ao longo do tempo, a eutrofizao artificial restabelece o equilbrio entre a produo e a decomposio de matria orgnica. (8) A reduo nas populaes de peixes est associada ao incremento dos processos anaerbicos de obteno de energia. (16) A diminuio de oxignio, no fundo do lago, reflete um aumento da atividade de microorganismos decompositores. (32) O despejo de esgotos domsticos em ambientes lacustres contribui para o processo de eutrofizao. (64) A poluio por detritos orgnicos tem efeitos nocivos imediatos sobre as populaes de consumidores. Questo 7
Como se teriam aperfeioado todas essas adaptaes maravilhosas que vo modificando o organismo parte por parte, em funo das condies de vida que ele suporta, acabando por transform-lo num ser diferente de seus ancestrais? (DARWIN, p. 85)

Os estudos do prprio Darwin e as investigaes cientficas neodarwinista respondem a essa indagao, afirmando: (1) As adaptaes originam-se da ao direta do ambiente sobre o organismo. (2) A matria-prima do processo evolutivo tem sua origem em alteraes do material gentico. (4) Presses seletivas favorecem a manuteno de caractersticas que permitem ao indivduo explorar melhor o ambiente. (8) As modificaes preservadas pela seleo natural so mantidas na populao, atravs da hereditariedade. (16) A reproduo sexuada tem um papel crucial na evoluo, por proporcionar a variabilidade gentica das populaes. (32) A competio intra-especfica dificulta uma melhor adaptao da populao ao ambiente. (64) O pool gnico mantido inalterado no processo de especiao.

21

BIOLOGIA
QUESTES DE 1 A 10

UFBA
F A S E

INSTRUO: Assinale as proposies verdadeiras, some os nmeros a elas associadas e marque o resultado na Folha de Respostas.

2 B

Questo 1 A variao na atividade cataltica de duas enzimas, em funo do pH, est registrada no grfico abaixo. P R O V A

Com base no grfico e em caractersticas e propriedades das biomolculas envolvidas, podese dizer:

(1) Pepsina e tripsina so enzimas que atuam no processo digestivo. (2) In vivo essas duas enzimas devem atuar em um mesmo ambiente fisiolgico. (4) A pepsina e a tripsina, bem como os substratos sobre os quais atuam, pertencem mesma classe de macromolculas. (8) Essas enzimas mantm uma atividade tima numa larga escala de pH. (16) O efeito do pH sobre a atividade dessas enzimas revela que a pepsina atua no meio citosslico. (32) A reao catalisada pela tripsina reversvel em funo do pH. (64) A atividade enzimtica uma expresso funcional da informao gentica.

22

BIOLOGIA
Questes 2 E 3

UFBA
F A S E

2 B

O diagrama ao lado ilustra o experimento que possibilitou levedura Saccharomyces cerevisi utilizar o amido como fonte de glicose para a produo de etanol. Neste experimento, transplantou-se o gene que codifica uma enzima pancretica do camundongo para um pequeno DNA circular (plasmdio), que foi, ento, introduzido na levedura. Como o amido no pode ser transportado para dentro da levedura, acrescentou-se ao gene uma seqncia de nucleotdeos que assegura a exportao da enzima. Constatou-se que a nova linhagem recombinante, em meio contendo exclusivamente glicose, continuou crescendo to bem quanto o tipo selvagem. (SCHENBERG, p. 6 texto adaptado)

P R O V A

Questo 2 Com base em processos relacionados ao fluxo e transmisso da informao gentica, a anlise dos dados permite afirmar: (1) A obteno do DNA complementar efetivada atravs de um processo de transcrio reversa. (2) As seqncias de RNAm que codificam a -amilase no camundongo e na levedura so diferentes entre si. (4) A insero do gene exgeno no cromossomo da levedura garante a herdabilidade dessa informao gentica. (8) A reproduo assexuada das leveduras permite a formao de clones com a informao nova inserida. (16) A traduo correta da protena do camundongo em clulas de levedura est na dependncia de um mesmo cdigo gentico para essas espcies. (32) A sntese de DNA complementar faz-se por um mecanismo molecular especfico da levedura.

23

BIOLOGIA
Questo 3 Em relao a aspectos da estrutura e dinmica da clula da levedura, correto dizer:

UFBA
F A S E

(1) A secreo da -amilase est subordinada ao seu processamento no complexo de Golgi. (2) As molculas da glicose, resultantes da hidrlise do amido, entram na clula da levedura por difuso simples. (4) A -amilase pancretica uma enzima de ao extracelular. (8) O destino final da -amilase permite levedura recombinante manter a sua fisiologia normal. (16) A realizao de processo fermentativo pelas leveduras evidencia a inexistncia de mitocndrias nessas clulas. (32) A incapacidade da levedura selvagem para utilizar o amido como fonte de nutrientes reflete a imperfeita adaptao dessa espcie. Questo 4 O diagrama abaixo evidencia propriedades de biomolculas em relao sua afinidade pelo oxignio.

2 B

P R O V A

Com base nos dados apresentados e em aspectos da biologia da respirao, pode-se concluir: (1) A curva da mioglobina expressa a adaptao dessa molcula a clulas com alta presso de oxignio. (2) O aumento da concentrao de oxignio se reflete mais significativamente na capacidade de transporte desse gs hemoglobina do adulto. (4) A hemoglobina e a mioglobina combinam-se com o oxignio atravs de reaes irreversveis. (8) O grau de afinidade da hemoglobina fetal pelo oxignio favorece o feto na competio com a me, para obter energia. (16) A propriedade de saturao da mioglobina adequada demanda energtica da fibra muscular em atividade normal. (32) As trocas gasosas, efetuadas atravs da hemoglobina, so garantidas pela diferena entre a concentrao do oxignio nos tecidos e no sangue. (64) A grandes altitudes, o suprimento eficaz de oxignio garantido por adaptaes que dificultam a sua liberao, nos tecidos.

24

BIOLOGIA
Questo 5

UFBA
F A S E

A ilustrao abaixo esquematiza os principais estgios do desenvolvimento da r sul-africana Xenopus lvis, associando este processo com a variao nas taxas plasmticas dos hormnios T3 e T4.

2 B

P R O V A

Com base na anlise da ilustrao, pode-se afirmar:

(1) A sobrevivncia do indivduo favorecida pelo aumento da competio entre jovens e adultos de uma mesma espcie. (2) A metamorfose envolve uma remodelao orgnica que adapta o organismo a novas relaes ecolgicas. (4) A metamorfose libera o indivduo da dependncia do meio aqutico, para cumprir todo o seu ciclo biolgico. (8) O clmax da metamorfose coincide com nveis elevados do hormnio T3 no sangue. (16) A inibio da atividade secretora da tireide bloquearia a evoluo do desenvolvimento de uma r. (32) O desencadeamento da metamorfose est subordinado a mecanismos de regulao gnica.

25

BIOLOGIA
Questo 6

UFBA
F A S E

As primeiras abelhas melferas foram trazidas de Portugal para o Brasil, no sculo XIX. Posteriormente, foram introduzidas abelhas por colonizadores alemes e italianos e, no final da dcada de 50 do sculo XX, foram trazidas raas de abelhas africanas. A partir da, desenvolveu-se um processo de expanso e hibridao dessas abelhas, que resultou no quadro de distribuio espacial das abelhas africanizadas no Brasil, conforme ilustrao abaixo. (DINIZ FILHO, p. 75 texto adaptado)

2 B

Propores da mistura racial NORDESTE 85,5% africano 14,5%europeu SUL 73,0% africano 27,0% europeu

P R O V A

Com base nas informaes e na anlise do grfico acima, pode-se afirmar:

(1) A distribuio geogrfica e as caractersticas da espcie Apis mellifera foram influenciadas por aes do homem. (2) A variao no pool gnico de populaes de abelhas africanizadas resulta de interaes com diferentes ambientes. (4) Os graus relativos do componente africano e do europeu, nas populaes, decorrem de mecanismos de seleo natural. (8) O processo de expanso das populaes de abelhas africanizadas favoreceu o estabelecimento do isolamento reprodutivo entre elas. (16) O componente gentico africano assegura estratgias de sobrevivncia mais adequadas s condies do Brasil, do que o componente europeu. (32) Os dados apontam para um aumento progressivo do componente europeu, nas regies do sul do Brasil. (64) A africanizao de populaes de Apis mellifera, no Brasil, expressa um processo evolutivo.

26

BIOLOGIA
Questo 7

UFBA
F A S E

Em um experimento para estudo da dinmica populacional, foi observada, por um determinado perodo, a variao mdia do nmero de espcies de aranhas, em trs grupos de ilhas com as seguintes condies: sem lagartos; com lagartos experimentalmente introduzidos; e com lagartos naturalmente presentes. O grfico abaixo registra essa variao nos trs tipos de ilhas, ao longo do experimento.

2 B

Lagartos ausentes Lagartos introduzidos Lagartos naturalmente presentes

P R O V A

Analisando-se os dados apresentados e com base em aspectos da dinmica de populaes, pode-se afirmar: (1) A introduo do predador marcou os dois primeiros anos do experimento com uma reduo local na diversidade de espcies de aranha. (2) Em ilhas sem lagartos, o potencial bitico das aranhas torna-se mais elevado do que nas ilhas habitadas por lagartos. (4) Na ausncia de predadores, a diversidade entre as aranhas limitada pela competio intra-especfica. (8) A presena do predador se constitui um fator de resistncia ambiental para as populaes de aranhas. (16) A flutuao no nmero de espcies, registrada no grfico, evidencia a disponibilidade de nichos ecolgicos para as aranhas. (32) Os resultados do experimento evidenciam um desequilbrio na relao presa-predador. (64) Um processo de recolonizao por espcies de aranhas, em ilhas com lagartos naturalmente presentes, caracterizou o ltimo ano do experimento.

27

BIOLOGIA
QUESTES 8 E 9

UFBA
F A S E

Nas figueiras, as flores masculinas e femininas, embora em uma mesma inflorescncia, amadurecem em tempos diferentes. Nessas plantas, a polinizao feita por um nico agente polinizador a vespa do figo.

2 B

P R O V A

O diagrama acima esquematiza as fases (de A a E) do florescimento e frutificao da figueira. Na fase floral (feminina), a fmea da vespa introduz seus ovos no ovrio da flor e a poliniza. No ovrio, os ovos se desenvolvem em larvas. Na fase D, as vespas adultas eclodem machos e fmeas e logo se acasalam dentro do prprio figo. As fmeas, que so aladas, em seguida deixam o figo, carregando plen, e vo pr os seus ovos em outra figueira. Os figos maduros so utilizados como alimento por animais frugvoros, por exemplo: macacos, morcegos e aves, e os figos cados no cho podem ser transportados por formigas savas para seus ninhos, onde so utilizados na plantao de fungos que a elas servem de alimento.

Questo 8 Com relao aos processos envolvidos na reproduo da vespa e da figueira, pode-se afirmar: (1) O ciclo de desenvolvimento da figueira envolve uma fase gametoftica e outra, esporoftica. (2) A maturao do gineceu e do androceu, em pocas diferentes, um mecanismo que proporciona maior variabilidade gentica nas populaes. (4) O desenvolvimento da vespa ocorre por crescimento contnuo. (8) Os processos de polinizao e oviposio identificam-se como mecanismos de transporte de gametas. (16) A disseminao das sementes da figueira se d pela interao com a vespa. (32) O ciclo biolgico da vespa depende do ciclo de florescimento da figueira. (64) A complexa interao da vespa com a figueira se deve a um processo de co-evoluo.

28

BIOLOGIA
Questao 9 As interaes ecolgicas descritas permitem afirmar:

UFBA
F A S E

(1) Os fungos, alimentando-se do produto da decomposio do figo, estabelecem-se, nos formigueiros, como produtores. (2) A produtividade primria da figueira pode subsidiar diferentes teias alimentares. (4) A relao da vespa com o figo se caracteriza como uma interao mutualista. (8) Os animais que se alimentam de figos ocupados por vespas funcionam como consumidores de primeira e segunda ordem. (16) A reduo do nmero de figueiras ameaa a sobrevivncia da vespa, por alterar a oferta de flores ao longo do ano. (32) A extino das populaes de vespas contribui para um aumento da biomassa de figueiras. (64) A energia introduzida no ecossistema pelas figueiras gradativamente reduzida ao longo das transferncias alimentares. Questo 10

2 B

P R O V A

O Schistosoma mansoni o verme trematdeo causador da esquistossomose mansnica, uma das mais graves endemias brasileiras. O diagrama ao lado ilustra o ciclo evolutivo desse parasita, representando as suas etapas fundamentais. Considerando-se aspectos biolgicos, ecolgicos e culturais dessa endemia, pode-se afirmar:

(1) O miracdio um estgio larvar que pode sobreviver em vida livre. (2) A passagem pelo hospedeiro intermedirio proporciona a formao de um grande nmero de larvas, favorecendo a continuao do ciclo. (4) A maior atividade diria das cercrias, durante o perodo de iluminao e calor mais intensos, aumenta os riscos de contaminao do homem. (8) A localizao do verme no sistema sangneo venoso um fator desfavorvel obteno de alimento. (16) A migrao do verme adulto para as paredes do intestino grosso uma estratgia favorvel preservao da espcie. (32) A ocorrncia de autofecundao, nos caramujos, uma caracterstica biolgica que favorece a disseminao do Schistosoma mansoni. (64) As reas de plantio, irrigadas por drenagem de rios, se constituem barreiras expanso da endemia.

29

BIOLOGIA

UFMG
F A S E

1. O filme O leo de Lorenzo conta a histria de um menino afetado por uma doena chamada leucodistrofia, que leva a deficincias auditivas, visuais e motoras. Essas deficincias devemse destruio da bainha de mielina das clulas nervosas. Analise a figura abaixo, referente a uma clula nervosa na qual alguns componentes foram numeradas de 1 a 4. Assinale a alternativa que contm o nmero correspondente bainha de mielina.

1 A

P R O V A

a) 1

b) 2

c) 3

d) 4

2. Sabe-se que o crack uma droga inalvel que vicia e pode levar morte. Entre seus possveis efeitos esto hiperatividade motora e sensorial, taquicardia, hipertenso arterial, convulses e derrame cerebral. A figura ao lado representa alguns componentes de sistemas orgnicos humanos, dos quais alguns foram numerados de 1 a 5. Com base na figura e em seus conhecimentos correto afirmar-se, em relao ao crack, que: a) distrbios motores e sensoriais ocorrem pela sua ao em 5 b) absorvido em 1 e atinge os diversos rgos pela ao de 2 c) metabolizado em 4 d) sua excreo ocorre em 3

30

BIOLOGIA

UFMG
F A S E

3. Um estudante colocou 5 kg de sementes de feijo numa estufa a 70 C. Durante uma semana, ele fez pesagens dirias dessas sementes e concluiu que o peso seco poderia ser determinado a partir do terceiro dia. Com base nessas informaes, assinale a alternativa que contm o tipo de curva que representa os dados obtidos pelo estudante.

1 A

P R O V A

4. No Brasil, so conhecidas vrias espcies de aranhas venenosas e de insetos vetores de doenas. Esses animais pertencem ao grupo dos artrpodes, que constituem mais de um milho de espcies, das quais cerca de novecentos mil so de insetos. O grande sucesso evolutivo dos insetos, quando comparados aos demais artrpodes, pode ser explicado pela seguinte adaptao: a) b) c) d) hbitos alimentares diversificados. pequeno forte. presena do exoesqueleto. presena de patas articuladas.

5. Este esquema refere-se a um experimento realizado para estudar a florao em trs plantas da mesma espcie que foram submetidas ao mesmo tempo de exposio luz (fotoperodo). Com base nos resultados observados, todas as concluses so possveis, EXCETO a) O tratamento fotoperidico de uma nica folha ou de toda a planta produz o mesmo efeito. b) A florao da planta depende da retirada de algumas folhas. c) A planta sem folhas no apresenta fotoperiodismo. d) A planta intacta floresce aps um fotoperodo adequado.

31

BIOLOGIA

UFMG
F A S E

6. Nos pomares, freqente o uso da pulverizao foliar com hormnios vegetais sintticos. Assinale a alternativa que NO constitui um efeito desse procedimento. a) Estimular a florao simultnea de vrias plantas da mesma espcie. b) Evitar a queda prematura dos frutos. c) Favorecer a formao de frutos partenocrpicos. d) Induzir a reproduo vegetativa.

1 A

7. O corte da haste de flores a dois centmetros da ponta e dentro dgua, prolonga a conservao das flores em jarros. Essa prtica, muito adotada em floriculturas, tem como objetivo impedir. a) a abertura dos estmatos, essencial ao transporte de gua. b) a formao de bolhas de ar para favorecer a capilaridade. c) a perda de gua pelas flores, que resfria a planta. d) o funcionamento dos vasos do floema como tubos condutores.

P R O V A

8. O diagrama abaixo representa o ciclo de vida de uma clula somtica humana, onde X representa o contedo de DNA.

Com base nas informaes do diagrama e em seus conhecimentos, INCORRETO afirmar que a) a fase de menor durao do ciclo a mitose. b) a fase F do ciclo corresponde interfase. c) em G1 a clula haplide. d) em S ocorre a duplicao dos cromossomos.

32

BIOLOGIA

UFMG
F A S E

9. Este grfico representa o nmero de indivduos por espcie versus o nmero de espcies em quatro reas (I, II, III e IV) do mesmo tamanho, num ecossistema de cerrado.

1 A

P R O V A

Com base nos dados, todas as alternativas so corretas, EXCETO a) b) c) d) A rea I apresenta maior nmero de indivduos. A rea II pode estar em processo de sucesso. A rea III pode possuir espcies com risco de extino. A rea IV apresenta maior biodiversidade.

10. Ao fazer um exame de vista para tirar carteira de motorista, um rapaz descobriu que era daltnico. Assinale, entre as alternativas abaixo, aquela que contm o heredograma que NO poderia representar a famlia desse rapaz.

a) 1

b) 2

c) 3

d) 4

33

BIOLOGIA

UFMG
F A S E

11. Este grfico refere-se taxa metablica de cinco animais de diferentes pesos corporais em um determinado intervalo de tempo.

1 A

P R O V A

Com base no grfico e em seus conhecimentos, correto afirmar que a) b) c) d) a digesto mais eficiente no cavalo do que no coelho. a taxa de consumo de oxignio maior no coelho do que no rato. a taxa metablica depende da relao entre a superfcie e o volume corpreo. a taxa metablica diretamente proporcional ao peso corporal.

12. Sabe-se que populaes de regies do Brasil Central tm, como principal fonte de iodo, o sal de cozinha. Amostras de sal refinado, analisadas recentemente pelo Instituto Adolfo Lutz de So Paulo, mostraram ndices de iodo muito inferiores aos exigidos pela legislao brasileira. Entre os distrbios provocados pela utilizao prolongada desse tipo de sal pela populao NO se inclui a) b) c) d) a deficincia mental nas crianas. o aumento do metabolismo. o atraso do crescimento das crianas. o crescimento excessivo da tireide.

13. Uma receita de po caseiro utiliza farinha, leite, manteiga, ovos, sal, acar e fermento. Esses ingredientes so misturados e sovados e formam a massa que colocada para descansar. A seguir, uma bolinha dessa massa colocada num copo com gua e vai ao fundo. Depois de algum tempo, a bolinha sobe superfcie do copo, indicando que a massa est pronta para ser levada ao forno. Com relao receita correto afirmar que a) b) c) d) a farinha constituda de polissacardeos, utilizados diretamente na fermentao. a manteiga e os ovos so os principais alimentos para os microrganismos do fermento. a subida da bolinha superfcie do copo se deve respirao anaerbica. os microrganismos do fermento so protozorios aerbicos.

34

BIOLOGIA

UFMG
F A S E

14. O esquema abaixo representa uma clula animal vista ao microscpio eletrnico, na qual algumas estruturas foram numeradas de 1 a 9.

1 A

P R O V A

Com relao s estruturas indicadas no esquema, INCORRETO afirmar que a) b) c) d) 1, 5 e 6 sofrem intensas modificaes na diviso celular. 2, 3 e 7 sintetizam e/ou armazenam substncias orgnicas. 4 e 8 realizam digesto celular com produo de energia e liberao de CO2. 5 e 9 so desprovidos de membrana lipoprotica.

15. Este quadro refere-se ao nmero de clulas sangneas, expresso em clulas/mm3 de sangue, encontradas nos exames de sangue de um indivduo normal e de um indivduo doente.

CLULAS SANGNEAS HEMCIAS PLAQUETAS

INDIVDUO NORMAL

INDIVDUO DOENTE

4.500.000 250.000 8.000 4.800 (60%) 80 (1%) 240 (3%) 2.400 (29%) 560 (7%)

4.800.000 100.000 15.000 12.000 (80%) 150 (1%) 1.050 (7%) 1.500 (10%) 300 (2%)

Totais
LEUCCITOS

Neutrfilos(%) Basfilos(%) Eosinfilos(%) Linfcitos (%) Moncitos (%)

Entre as possveis alteraes apresentadas pelo indivduo doente, NO se inclui a) alergia. b) anemia. c) distrbio de coagulao. d) infeco.

35

BIOLOGIA

UFMG
F A S E

16. Os animais abaixo representados so bastante diferentes na sua aparncia, mas apresentam vrias caractersticas comuns.

1 A

P R O V A

Entre essas caractersticas NO se inclui a) fecundao interna. b) homeotermia. c) oviparidade. d) respirao pulmonar.

17. Esta tabela refere-se ao teor de minerais e vitaminas, expressos em mg por 100g de parte comestvel de alguns alimentos.

MINERAIS ALIMENTO

VITAMINAS

Ca Abacate Couve Goiaba Gro de bico 13 203 22 68

P 47 63 26 53

Fe 0,7 1,0 0,7 7,0

A 20 650 26 0

B1 0,07 0,20 0,04 0,46

C 12 92 218 5

Com base nos dados dessa tabela, assinale a alternativa que contm uma recomendao alimentar INADEQUADA. a) b) c) d) Abacate para pessoas que sofrem de bri-bri. Couve para algum com osteoporose e xeroftalmia. Goiaba para quem sofre de escorbuto. Gro-de-bico para pessoas anmicas.

36

BIOLOGIA

UFMG
F A S E

18. As figuras abaixo foram extradas da bula de um medicamento e representam procedimentos que podem ser adotados na preveno de algumas doenas.

1 A

Das protozooses abaixo a nica que pode ser evitada por esses procedimentos a a) amebase. b) doena de Chagas. c) leishmaniose. d) malria.

P R O V A

19. Os cromossomos de uma clula somtica de drosfila podem ser representados como no desenho abaixo.

Suponha que a drosfila da qual foi retirada essa clula seja heterozigota para cor do corpo (Aa) e tamanho da asa (Bb) e que esses genes estejam nos cromossomos 1 e 3, respectivamente. Considerando apenas os cromossomos 1 e 3 dessa clula, escolha a alternativa que contm a configurao observada na metfase mittica.

20. Dona Margarida observou que uma lata de sardinha estava estufada e resolveu no consumir o seu contedo. Assinale a alternativa que apresenta uma justificativa INCORRETA para a atitude de dona Margarida. a) b) c) d) O alimento pode conter toxinas produzidas por microrganismos. O alimento pode estar em processo de decomposio. Os gases responsveis pelo estufamento da lata so txicos. Pode ter ocorrido falta de higiene durante o processo de embalagem.

37

BIOLOGIA

UFMG
F A S E

1. A proibio do fumo em bares e restaurantes, adotada em So Paulo em 1995, com o intuito de proteger o no-fumante (fumante passivo), gerou grande polmica, inclusive jurdica. Todas as alternativas contm argumentaes sobre as aes da fumaa do tabaco que so comprovadamente aceitas, EXCETO a) b) c) d) causa problemas respiratrios, principalmente em crianas. contm monxido de carbono, que bloqueia a funo de certas clulas sangneas. contm nicotina que libera a adrenalina que despigmenta a pele. tem ao cancergena tanto para o fumante ativo quanto para o passivo.

1 B

2. Considere que uma determinada floresta possua 200 espcies de aves. O grfico abaixo apresenta a relao entre o tamanho de algumas reas dessa floresta e a proporo de aves extintas.

P R O V A

Com base nos dados do grfico, todas as afirmativas so corretas, EXCETO a) b) c) d) abaixo de 54 ha ocorre perda de mais de 50% das espcies de aves. a diversidade de espcies de aves depende de matas extensas. em reas de 25 ha, possvel a preservao de 40% das espcies de aves. fragmentos de mata de 10 ha apresentam em torno de 10 espcies.

3. Um criador fez cruzamentos entre porquinhos-da-ndia para estudar as caractersticas: cor e tamanho do plo. Observou os seguintes resultados: Gerao P: Plo preto curto x Plo marrom longo Gerao F1: Plo preto curto Gerao F2: 81 plo preto curto 27 plo preto longo 27 plo marrom curto 09 plo marrom longo Com base nesses resultados, INCORRETO afirmar que a) b) c) d) as caractersticas plo preto e curto so dominantes. as duas caractersticas segregam-se independentemente. os indivduos da gerao F1 so homozigotos. os indivduos da gerao P formam um s tipo de gameta.

38

BIOLOGIA

UFMG
F A S E

4. Esta figura representa o cruzamento de drosfilas de asas normais com indivduos de asas enroladas, mutantes, criadas em temperatura de 25C.

1 B

P R O V A As moscas de gentipo igual ao do tipo III, quando criadas em temperatura de 16C, apresentam asas normais. O resultado desse fenmeno ilustra a) b) c) d) a adaptao dos mutantes. a influncia do ambiente na expresso do gentipo. a transmisso dos caracteres adquiridos. o processo de seleo natural.

5. As figuras abaixo representam animais, numerados de 1 a 4

Assinale a alternativa que contm o animal pertencente ao mesmo grupo das guas vivas, freqentes causadoras de queimaduras em banhistas no litoral brasileiro. a) 1 b) 2 c) 3 d) 4

39

BIOLOGIA

UFMG
F A S E

6. A figura abaixo representa o cultivo de tomates em um sistema hidropnico I e em um canteiro adubado II

1 B

P R O V A

Verificou-se que a produo de tomates em I e II no apresentou diferenas quantitativas nem qualitativas. Com relao aos cultivos I e II , INCORRETO afirmar que a) a absoro da matria orgnica e de sais minerais resulta na mesma produtividade. b) a produtividade, tanto em I quanto em II , resultante da fotossntese. c) o hmus contm substncias que, aps decomposio, liberam nutrientes para o solo. d) o solo e a soluo nutritiva atendem, qualitativamente, s necessidades da planta.

7. Esta figura representa um fenmeno observado aps a poda anual das videiras. A alternativa que melhor explica o fenmeno a) a abertura dos estmatos provocando a eliminao da seiva. b) a suco de gua pelas clulas dos ramos da planta. c) o deslocamento da seiva devido presso de raiz. d) o transporte ativo da seiva pelas clulas das razes da videira.

40

BIOLOGIA

UFMG
F A S E

8. As figuras abaixo ilustram as etapas de um experimento referente ao desenvolvimento de morangos.

1 B

P R O V A Com base nos resultados obtidos e em seus conhecimentos, INCORRETO afirmar que a) b) c) d) a aplicao de hormnios sintticos poderia substituir a presena das sementes. a forma do morango independe do padro de distribuio das sementes a retirada das sementes paralisa o crescimento dos morangos. as sementes produzem substncias indutoras do crescimento.

9. Esta tabela contm os valores de alguns hormnios, dosados no sangue de uma mulher, e os intervalos de valores considerados normais, expressos em unidades internacionais.
HORMNIO Antidiurtico Tireotrfico Folculo estimulante Luteinizante MULHER 0,9 0,06 1,5 0,1 VALORES NORMAIS 1,3 - 4,1 0,3 - 5,0 2,5 - 47,9 0,2 - 65,0

Entre os possveis distrbios que a mulher pode apresentar NO se inclui a) b) c) d) a alterao do ciclo menstrual. a disfuno hipofisria. o aumento da diurese. o aumento do metabolismo basal.

10. Na fabricao de iogurtes e coalhadas, utilizam-se iscas, isto , colnias de microrganismos que realizam a fermentao do leite. Em relao a esse processo, correto afirmar que a) b) c) d) consiste em respirao aerbica. realizado por vrus anaerbicos lticos. resulta da liberao de cido ltico e energia. resulta na formao de cido actico e CO2.

41

BIOLOGIA

UFMG
F A S E

11. Conforme noticiado na imprensa em abril de 1996, as mortes de pacientes submetidos hemodilise em um hospital de Caruaru, Pernambuco, foram devidas presena de algas azuis na gua utilizada nos aparelhos de hemodilise. A provvel ao das algas azuis foi a a) b) c) d) competio pelo O2 livre no sangue levando cianose. formao de colnias levando obstruo de vasos sangneos. liberao de toxinas na gua provocando leses hepticas. utilizao do nitrognio das protenas acarretando deficincia nutricional.

1 B

12. Uma pancada na cabea leva freqentemente formao de um galo que pode ser explicado por a) b) c) d) extravasamento de plasma. formao de tecido cicatricial. formao de um calo sseo. proliferao de clulas do tecido epitelial.

P R O V A

13. A figura abaixo representa uma clula vegetal, vista ao microscpio eletrnico, na qual algumas estruturas foram numeradas de 1 a 8.

Todas as alternativas apresentam relaes funcionais corretas entre as estruturas celulares, EXCETO a) 1 e 3 sntese protica e formao de ribossomos. b) 2 e 5 quebra e sntese de glicose, respectivamente. c) 4 e 7 diviso celular. d) 6 e 8 sntese e armazenamento de material gentico.

42

BIOLOGIA

UFMG
F A S E

14. A doena de Tay-Sachs hereditria e provoca retardamento mental grave e morte do paciente na infncia. Essa doena devida incapacidade das clulas de digerir uma substncias cujo acmulo responsvel pelas leses no sistema nervoso central. Com base nessas informaes, pode-se afirmar que a organela celular cuja funo est alterada nessa doena a) a mitocndria. b) o complexo de Golgi. c) o lisossomo. d) o retculo endoplsmtico rugoso. 15. Dois estudantes, nadando numa lagoa, observaram alguns organismos na gua e estabeleceram o seguinte dilogo:
- Olha os girinos, filhotes de sapos. - No so girinos, so alevinos. - Mas... O que so alevinos? - So filhotes de peixe, animais diicos.

1 B

P R O V A

Com base nesse dilogo e em seus conhecimentos biolgicos, INCORRETO afirmar que a) os alevinos so etapas do desenvolvimento dos peixes. b) os girinos so etapas de metamorfose. c) os peixes realizam fecundao cruzada. d) os sapos realizam fecundao interna. 16. A estrofe abaixo foi extrada do poema, Jogos Frutais, de Joo Cabral de Melo Neto.
Est desenhada a lpis de ponta fina, tal como a cana-de-acar que pura linha.

O termo pura linha a que se refere o poeta corresponde ao tecido vegetal a) colnquima. b) esclernquima. c) meristema. d) parnquima.

43

BIOLOGIA

UFMG
F A S E

17. O grfico abaixo mostra a atividade fotossinttica de plantas cultivadas em diferentes concentraes de CO2.

1 B

P R O V A

Com base no grfico e em seus conhecimentos sobre o assunto, INCORRETO afirmar que a) a eficincia fotossinttica maior nos horrios entre 8 e 10 horas e entre 14 e 16 horas. b) a maior taxa de utilizao de CO2 ocorre entre 12 e 14 horas. c) o uso de estufas com alta concentrao de CO2 pode ser um recurso para o aumento da produtividade. d) os estmatos devem estar fechados entre 10 e 14 horas. 18. Esta tabela mostra o teor de protenas, carboidratos e lpides em alguns alimentos, expresso em gramas por 100g de peso seco.

ALIMENTO Carne seca Farinha de mandioca Arroz Toucinho

PROTENAS 48,0 1,3 8,0 9,7

CARBOIDRATOS 0 80,8 76,5 0

LPIDES 11,0 0,5 1,4 64,0

Com base nos dados da tabela, assinale a alternativa que contm a dieta mais adequada para um jogador de futebol antes de uma competio. a) Arroz com farinha de mandioca. b) Arroz com toucinho. c) Carne seca com farinha de mandioca. d) Carne seca com toucinho.

44

BIOLOGIA

UFMG
F A S E

19. As figuras abaixo foram extradas da bula de um medicamento e representam procedimentos que podem ser adotados na preveno de algumas doenas.

1 B

P R O V A Assinale, a alternativa que contm uma verminose que NO pode ser evitada por qualquer dos procedimentos apresentados nas figuras. a) Ancilostomase b) Esquitossomose c) Filariose d) Tenase

20. Entre os peixes e os primeiros anfbios foram necessrios 40 milhes de anos de lenta e constante evoluo. Todas as alternativas contm adaptaes surgidas durante essa evoluo, EXCETO. a) b) c) d) manuteno da pele mida. membros articulados. respirao pulmonar. termorregulao.

QUESTO 01 Conforme reportagem publicada no jornal Folha de So Paulo (maro de 1996), a Ilha de Lenis, situada no Maranho e com uma populao de 400 habitantes, apresenta um ndice de albinismo de 1,5%. Tal ndice 150 vezes maior do que o apresentado em outras populaes. Lenis est desprovida de condies bsicas de saneamento, de energia eltrica, telefone e assistncia mdica. O povoado mais prximo da ilha localiza-se a 7 horas de viagem de barco. 1. A reportagem mencionada traz o seguinte depoimento de um morador: Para evitar o sol que machuca a pele como queimadura de fogo, pesco camares cor-de-rosa que, como eu, s sai noite. a) EXPLIQUE o motivo da vulnerabilidade da pele do albino ao sol. b) CITE uma doena que pode ser evitada pelo comportamento relatado pelo morador. 2. CITE dois fatores que podem ter favorecido o alto ndice de albinismo nessa ilha. 3. Segundo o relato dos moradores, o ndice de albinismo na ilha j foi maior. CITE um fator social que pode ter contribudo para a diminuio desse ndice.

F A S E

45

BIOLOGIA
4. O heredograma abaixo refere-se herana do albinismo em uma famlia da ilha.

UFMG
F A S E

Com relao ao heredograma, DETERMINE o tipo de herana e CITE os gentipos de I-1; I-2; II-2 e II-8. a) Tipo de herana: b) Gentipos: I-1: QUESTO 02 Estes esquemas representam dois experimentos (I e II) realizados para estudar o efeito de um inseticida na regenerao de um determinado animal. ; I-2: ; II-2: e II-8:

Com base nos dados e em seus conhecimentos, responda ao que se pede: 1. CITE o erro cometido na montagem dos experimentos considerando o problema proposto. 2. CITE o nome do animal representado e a justificativa para sua utilizao nos experimentos. a) Animal: b) Justificativa:

46

BIOLOGIA

UFMG
F A S E

3. CITE dois mecanismos em que um inseticida, usado em ambiente terrestre, poderia atingir organismos aquticos. a) Mecanismo 1: b) Mecanismo 2: 4. PREENCHA o quadro abaixo, escolhendo dois parasitas humanos pertencentes ao mesmo filo do animal utilizado no experimento.
PARASITA DIMORFISMO SEXUAL MODO DE CONTGIO

QUESTO 03 Observe a figura que representa o ciclo do nitrognio na natureza.

Com base nessa figura e em seus conhecimentos, responda ao que se pede: 1. EXPLIQUE as relaes ecolgicas que se estabelecem entre os seres vivos e que favorecem a obteno de nitrognio. a) Na fase 1 b) Entre os seres a e b

47

BIOLOGIA
2. NOMEIE os fenmenos que esto ocorrendo em 2 e 3. a) 2: b) 3:

UFMG
F A S E

3. reas de monocultura como as de plantao de cana-de-acar tm provocado grande desgaste do solo. Para solucionar esse problema, os tcnicos sugerem a adubao verde. CONCEITUE adubao verde e EXPLIQUE de que forma ela contribui para a melhoria do solo. a) Conceito: b) Explicao:

4. EXPLIQUE um fenmeno que poder ocorrer num ambiente aqutico quando houver excesso de nitrato.

QUESTO 04 Aps um acidente automobilstico, o motorista do veculo, pertencente ao grupo sangneo A, Rh +, gravemente ferido, foi socorrido e submetido, entre outros, aos seguintes procedimentos: infuso venosa de 3 litros de sangue, massagem cardaca e traqueostomia (abertura cirrgica e canalizao da traquia). Com base nessas informaes e em seus conhecimentos, responda ao que se pede. 1. CITE uma justificativa para a realizao dos trs procedimentos mencionados. a) Infuso venosa: b) Massagem cardaca: c) Traqueostomia:

2. NOMEIE o principal tipo de tecido existente em cada um dos componentes orgnicos envolvidos nos procedimentos descritos. a) Veia: b) Corao: c) Traquia:

3. CITE os gentipos possveis do motorista para os sistemas sangneos ABO e Rh. a) ABO: b) Rh:

48

BIOLOGIA

UFMG
F A S E

4. CITE dois tipos de sangue que o motorista poderia receber e os respectivos aglutingenos e aglutininas.

TIPO SANGNEO

AGLUTINGENO

AGLUTININA

5. CITE o percentual aproximado do volume sangneo recebido pelo motorista, considerando que o seu peso corporal de 70kg.

QUESTO 05 Estas figuras representam os sistemas digestivos de dois vertebrados, A e B , nos quais alguns componentes foram nomeados ou numerados de 1 a 3

Com relao aos sistemas digestivos desses animais, responda ao que se pede: 1. CITE os hbitos alimentares de A e B . 2. O rmen e o barrete (retculo) so componentes do sistema digestivo de B que contm microrganismos que participam da digesto. CITE o nmero do componente que representa o rmen e a importncia dos microrganismos na digesto. a) Nmero do componente: b) Importncia dos microrganismos:

49

BIOLOGIA

UFMG
F A S E

3. CITE o nmero do componente do sistema digestivo de A que corresponde moela e justifique a sua importncia nesse grupo animal. a) Nmero do componente: b) Justificativa: 4. COMPARE os tipos de digesto que ocorrem na moela e no rmen. QUESTO 06 Estas figuras referem-se a dois ecossistemas brasileiros diferentes.

Com base nas figuras e em seus conhecimentos, responda o que se pede.

1. PREENCHA o quadro, associando uma adaptao da planta s caractersticas do ambiente.

CARACTERSTICAS DO AMBIENTE DISPONIBILIDADE DE GUA NO SOLO DISPONIBILIDADE DE LUZ

ADAPTAO DA PLANTA DO ECOSSISTEMA I

ADAPTAO DA PLANTA DO ECOSSISTEMA II

50

BIOLOGIA

UFMG
F A S E

2. Observe a tabela que representa o nmero de espcies por grupo de vegetais amostradas em um hectare dos ecossistemas I e II .

GRUPOS DE VEGETAIS Brifitas Pteridfitas Angiospermas

ECOSSISTEMA I 1 2 29

ECOSSISTEMA II 6 10 60

a) JUSTIFIQUE os nmeros obtidos para as pteridfitas e para as brifitas no ambiente I : b) Nos dois ecossistemas, so encontradas vrias espcies de orqudeas. Dos grupos citados na tabela, NOMEIE aquele em que se incluem as orqudeas: c) CITE o nmero do ecossistema que apresenta maior riqueza de espcies: 3. As figuras abaixo representam cortes de caule de dois vegetais V1 e V2 , coletados nos ecossistemas I e II .

Com base na figura e em seus conhecimentos, CITE o vegetal ( V ou V ) que pertence ao 1 2 ecossistema I e JUSTIFIQUE sua resposta. a) Vegetal: b) Justificativa:

51

BIOLOGIA
QUESTO 07 Esta figura representa uma rvore evolutiva dos vertebrados.

UFMG
F A S E

Com base nessa figura e em seus conhecimentos, responda ao que se pede. 1. De acordo com as informaes da figura, ESCREVA dentro dos quadros acima, na seqncia evolutiva, os nomes dos seguintes animais: baleia, camundongo, gamb, lagartixa e sapo.

2. CITE duas divergncias adaptativas apresentadas pelos mamferos da figura.

52

BIOLOGIA

UFMG
F A S E

3. NOMEIE o fenmeno que explica a semelhana entre a baleia e o tubaro com base na teoria evolutiva. 4. Preencha as lacunas do seguinte quadro:

SEQNCIA DE OCORRNCIA DOS ANIMAIS aves mamferos anfbios rpteis peixes anfbios

SISTEMA FUNCIONAL ENVOLVIDO reprodutor

ADAPTAO

pele com escamas respiratrio circulatrio homeotermia

5. CITE o sistema funcional cuja adaptao permitiu a diversificao dos peixes, tanto na ocupao de ambientes de gua salgada como nos de gua doce e JUSTIFIQUE a sua resposta. a) Sistema: b) Justificativa:

53

BIOLOGIA

UFPA
F A S E

1. Alm do DNA cromossmico, algumas bactrias podem apresentar pequenos crculos de DNA de grande importncia para a engenharia gentica. Estas estruturas so conhecidas como a) nucleide b) genonema c) plasmdeos d) mesossomo e) capsmeros

2. A administrao de grandes quantidades de drogas a um animal resulta num aumento da atividade enzimtica relacionada detoxificao, bem como a de outras enzimas, e numa considervel hipertrofia do a) Retculo Endoplasmtico agranular b) Retculo Endoplasmtico granular c) Complexo de Golgi d) Lisossomas e) Ribossomas

3. Os pulges so conhecidos insetos parasitas de determinadas plantas, de onde extraem a seiva elaborada que lhes serve de alimento. O excesso da seiva sugada eliminada atravs de poros anais, sendo utilizado por determinadas formigas como fonte de alimento. Esse tipo de relao conhecida como a) comensalismo b) parasitismo c) simbiose d) mutualismo e) predatismo

4. O tipo de adaptao em que um ser vivo imita a forma e at a prpria cor de elementos do meio ambiente, confundindo-se com o meio em que vivem, causando dificuldades ao predador para encontr-los, denomina-se a) competio b) mutualismo c) predatismo
-

d) parasitismo e) mimetismo

5. Quando uma mulher Rh tem um filho com um homem Rh , ocorrem duas possibilidades dependendo de o homem ser puro ou hbrido. No primeiro caso, os filhos do casal sero a) todos Rh b) todos Rh + c) 50% Rh e 50% Rh
+

d) 25% Rh e 75% Rh +e) 25% Rh e 75% Rh

6. Em uma reao imunolgica, os antgenos e anticorpos recebem denominaes diferentes, dependendo do efeito causando pela reao. Quando ocorre aglutinao, o antgeno e anticorpo so chamados, respectivamente de a) lisgeno e lisina b) precipitognio e precipitina c) toxina e antitoxina d) aglutinognio e aglutinina e) precipitognio e aglutinognio

54

BIOLOGIA

UFPA
F A S E

7. Entre os seres vivos, existem uns que possuem grande variedade de enzimas que permitem que eles ataquem praticamente qualquer tipo de material, como madeiras, frutas, conservas, causando grandes prejuzos ao homem. Esses seres pertencem ao reino a) Monera b) Fungi c) Animalia d) Protista e) Plantas

8. Os rgos que constituem estruturas atrofiadas em determinadas grupos, mas que aparecem desenvolvidas e funcionais em outros, revelando a existncia de um parentesco evolutivo entre eles, so denominados de a) anlogos b) homlogos c) heterlogos d) complementares e) vestigiais

1. Desde os tempos mais remotos que a utilizao do NaCl (sal comum) tem sido indicada para evitar a putrefao dos alimentos. Como exemplo prtico temos o salgamento de carnes e pescados, visando mant-los viveis para o consumo por tempo mais prolongado. Explique o mecanismo de preservao dos alimentos, atravs da adoo desta prtica.

F A S E

2. MALRIA INFESTA MUNICPIOS DO RIO TAPAJS


O panorama de sade pblica nos trs municpios da regio do rio Tapajs - Santarm, Itaituba e Aveiro - bastante preocupante. Segundo o chefe do distrito da Fundao Nacional de Sade (FNS), dados registrados no ano de 1993 apontam nestes municpios, mais de 17.230 casos de malria. Este nmero diminuiu em 1994 para 13.495, e at agosto deste ano j somam mais de 5.013 casos da doena. O municpio de Itaituba o responsvel por 97,5% desses casos, e a diminuio da casustica devida provavelmente, reduo das atividades dos garimpos, onde a ocorrncia da doena mais acentuada. Para um dos pesquisadores do Ncleo de Medicina Tropical da UFPa, que estuda o impacto da contaminao mercurial sobre a sade humana nestas reas de doenas tropicais, a Amaznia, dadas as suas condies ambientais e ao acelerado processo de ocupao a que vem sendo submetida, experimenta as conseqncias causadas em relao disperso e recrudescimento de endemias. O crescimento demogrfico acelerado, a expanso das fronteiras de ocupao e as condies de trabalho nestas reas, contribuem para alterar o ecossistema, facilitando a disseminao de doenas como a malria. (O Liberal, 14/10/95).

Considerando o ciclo evolutivo do Plasmodium, responda: a) Qual a forma infectante do parasita no homem? b) Qual(is) a(s) causa(s) dos acessos recorrentes de calafrios e febre que acometem o indivduo doente?

55

BIOLOGIA

UFPA
F A S E

3. As plantas vasculares ou traquefitas possuem tecidos de conduo e so representadas pelas Pteridfitas e Fanergamas. Nestes vegetais o transporte de gua, nutrientes minerais e compostos orgnicos, realizado por clulas especializadas que formam os tecidos vasculares: o lenho ou xilema e o lber ou floema. Baseado nesta afirmao explique: a) O que acontecer com uma rvore frutfera se retirarmos um anel da casca do seu tronco? Justifique. b) Se o mesmo cintamento for realizado num dos seus galhos, o que suceder com os frutos produzidos pelo mesmo? Justifique. 4. Na mulher, as gonadotrofinas hipofisrias hormnio folculo estimulante e hormnio luteinizante interagem com os hormnios produzidos pelos ovrios estrgeno e progesterona , de modo que uns controlam a produo dos outros. A interao desses hormnios determina uma srie de modificaes no sistema reprodutor feminino, dando origem ao ciclo menstrual, com durao mdia de 28 dias. Baseado nesta interao hormonal, explique de que maneira a plula atua como um dos anticoncepcionais mais comumente utilizados.

5. Na maioria dos zigotos animais, ocorrem variaes quanto distribuio de substncias citoplasmticas responsveis pela diferenciao celular, fase da embriognese onde acontecem as alteraes morfolgicas e fisiolgicas das clulas que daro origem aos vrios rgos. Em alguns organismos, os componentes citoplasmticos se distribuem de modo heterogneo entre as clulas resultantes da clivagem da clula-ovo (caso dos anfbios e dos moluscos), enquanto que em outros (caso dos mamferos), essa distribuio homognea. Qual a importncia dessa distribuio citoplasmtica diferencial entre os blastmeros, nos moluscos e nos mamferos?

6. Dois amigos agricultores, possuem, cada um, 13 hectares de terra de mata virgem, que abrigam um grande nmero de insetos predadores. Um deles decide derrubar 100% da mata e plantar soja. O outro derruba apenas 60% da mata, preserva os 40% restantes na forma de vrias ilhas de vegetao nativa, e planta soja, milho e feijo na rea desmatada. Em ambas as situaes no so usados defensivos agrcolas de qualquer natureza. Em qual das duas reas existe maior probabilidade de sucesso no plantio? Justifique.

56

BIOLOGIA

UFPA
F A S E

7. Na genealogia abaixo esto indicados os grupos sangneos dos sistemas ABO, MN e Rh, do casal Caio e Vera. Sabe-se que apenas a filha do casal pertence ao grupo sangneo O, e que os smbolos s e q representam, respectivamente, homens e mulheres Rh negativos.

Com base nestas informaes, determine: a) O(s) provvel(is) gentipo(s) dos trs filhos do casal, para os trs sistemas sangneos; b) Qual(is) deles apresenta(m) chances de sofrer a eritroblastose fetal? Justifique. 8. De modo geral os cientistas acreditam que a clorofila, pigmento vegetal rico em magnsio, e a hemoglobina, molcula animal rica em ferro, possuem uma origem comum. Em que fato(s) est baseada esta afirmativa?

57

BIOLOGIA
1. Observe as sentenas abaixo: 1. Corpsculo nuclear visto somente na interfase. 2. Estrutura esponjosa semimembranosa sem membrana limitante. 3. Corpsculo rico em RNA que se origina de certas regies de cromossomos. As proposies acima referem-se a (ao): a) Cromatina b) Complexo de Golgi c) Nuclolo 2. Assinale a alternativa INCORRETA: d) Cromonema e) Constrio primria

UFPE
F A S E

a) A difuso simples um tipo de transporte passivo atravs da membrana plasmtica que ocorre quando existem condies de gradiente de concentrao, sem haver gasto de energia. b) A difuso facilitada utiliza protenas carreadoras para o transporte de acares simples e aminocidos atravs de membrana constituindo, por essa razo, um processo de transporte ativo. c) A membrana plasmtica formada por uma camada bimolecular de fosfolipdeos onde esto dispersas molculas de protenas globulares, dispostas como um mosaico. d) Qualquer processo de captura por meio do envolvimento de partculas chamado endocitose. e) Na fagocitose a clula engloba partculas slidas atravs da emisso de pseudpodes que as englobam formando um vacolo alimentar denominado fagossomo. 3. Faa a correspondncia entre as colunas: 1. Messossomo 2. Citosol 3. Parede celular 4. Membrana plasmtica ( ) projeo da membrana de clulas procariticas que contm enzimas respiratrias. ( ) possui permeabilidade seletiva devido a sua constituio lipoproteica. ( ) constitui-se de polissardeos formando cadeias entrelaadas com peptdeos. ( ) apresenta grnulos de ribossomos responsveis pela sntese proteica e polissacardeos que podem atuar como combustvel no metabolismo. d) 2, 1, 3, 4 e) 2, 3, 4, 1

a) 1, 4, 3, 2 b) 1, 3, 2, 4 c) 1, 3, 4, 2

58

BIOLOGIA
4. Observe as sentenas abaixo:

UFPE
F A S E

1. O tecido conjuntivo formado por clulas mesenquimais que se separam do mesoderma e formam um tecido que apesar de se diferenciar possui a funo de ligar e sustentar os rgos do organismo. 2. O msculo liso possui fibras plurinucleadas com miofibrilas de contrao involuntria. 3. As protenas da membrana do neurnio, quando atingidas por algum estmulo, sofrem + deformao que provoca o surgimento de canais de K . 4. O tecido epitelial do estmago desempenha duas funes simultaneamente: glandular e de revestimento. 5. Hipfise, tireide e pncreas so exemplos de glndulas endcrinas. Esto corretas apenas: a) 1 e 2 b) 1 e 4 c) 2 e 4 d) 1, 2 e 3 e) 1 e 3

6. Organismos aclorofilados, hetertrofos, unicelulares ou pluricelulares utilizados em indstrias farmacuticas, de bebidas fermentadas e em panificadoras. Esta descrio refere-se s (aos). a) Bactrias b) Fungos c) Rodofceas d) Diatomceas e) Protozorios

7. Pontes citiplasmticas que estabelecem continuidade entre clulas vegetais, conforme se ilustra na figura abaixo, so denominadas:

a) Plasmodesmos. b) Microtbulos. c) Desmossomos.

d) Interdigitaes. e) Microvilosidades.

59

BIOLOGIA
8. A partir dos capilares venosos o sangue circula para: a) b) c) d) e) Vnulas-veias-corao- artrias-arterolas - capilares arteriais. Capilares arteriais-arterolas-artrias-corao-veias-vnulas. Arterolas-artrias-corao-veias-vnulas-capilares arteriais. Corao-arterolas-artrias-capilares arteriais-veias-vnulas. Capilares arterias-vnulas-arterolas-veias-artrias-corao.

UFPE
F A S E

9. Relacione os tecidos vegetais da coluna 1 com as respectivas funes, mostradas na coluna 2. COLUNA 1 1. Floema ou lber 2. Xilema ou lenho 3. Meristema 4. Colnquima 5. Sber A seqncia correta : a) 3, 1, 5, 4 e 2 b) 3, 5, 1, 2 e 4 c) 2, 4, 5, 1 e 3 d) 1, 2, 4, 5 e 3 e) 1, 3, 5, 2 e 4 COLUNA 2 ( ) Transporte da seiva mineral ( ) Sustentao ( ) Proteo ( ) Transporte da seiva orgnica ( ) Crescimento do vegetal

10. Na figura ao lado mostra-se o cruzamento entre plantas de flores vermelhas e plantas de flores brancas de maravilha (Mirabilis sp.). Analise o resultado obtido nas geraes F1 e F2 e assinale a alternativa correta quanto ao tipo de herana envolvido neste caso. a) b) c) d) e) Dominncia completa. Herana ligada ao sexo. Herana parcialmente ligada ao sexo. Sem dominncia. Herana holndrica.

60

BIOLOGIA

UFPE
F A S E

10. Foi realizada uma experincia com duas linhagens (A e B) do vrus do mosaico do tabaco (T.M.V) em que foram isoladas as capas proticas e o RNA (cido ribonuclico) de cada linhagem. A seguir, foi reconstitudo um vrus hbrido o qual possuia o RNA da linhagem A e a protena da linhagem B. Esse vrus hbrido foi utilizado para a infeco de folhas sadias de fumo. Com relao aos novos vrus produzidos, aps a infeco, pode-se afirmar que estes apresentam: a) b) c) d) e) RNA e protena da linhagem B. RNA e protena da linhagem A. RNA da linhagem A e protena da linhagem B. RNA da linhagem B e a protena da linhagem A. RNA e protena hbridos entre a A e B.

11. rgos respiratrio dos (as) o (a) possui um sistema de tubos que se abrem na parte lateral do corpo e se ramificam at os tecidos, onde o oxignio fornecido diretamente s clulas sem passar pelo sangue. Indique a alternativa que completa adequadamente a sentena acima: a) Aneldeos, aparelho circulatrio. b) Aranhas, fitotraquia. c) Caracis, pulmes primitivos. 12. Assinale a alternativa incorreta: d) Insetos, traquia. e) Miripodes, pulmes foliceos.

VITAMINAS

AO mantm os epitlios em boas condies. co-fator da descarboxilao do cido pirvico. atua na sntese do colgeno transportadora de eltrons na cadeia respiratria catalisadora da sntese de protombina no fgado

A CARNCIA PRODUZ cegueira noturna beribri escorbuto distrofia muscular hemorragia

a) b) c) d) e)

A B2 C E K

61

BIOLOGIA

UFPE
F A S E

13. Para distinguir a influncia dos fatores ambientais sobre uma populao faz-se necessrio distinguir a ao dos mesmos sobre os fatores que determinam a densidade populacional. Relacione e assinale a alternativa correta. 1. taxa de mortalidade 2. taxa de natalidade 3. taxa de imigrao 4. taxa de emigrao a) freqncia de nascimentos b) freqncia de mortes c) proporo de indivduos que entram em uma populao em uma determinada unidade de tempo d) proporo de indivduos que saem de uma populao em uma determinada unidade de tempo d) 1-a, 2-b, 3-d, 4-c e) 1-c, 2-d, 3-a, 4-b

a) 1-a, 2-b, 3-c, 4-d b) 1-b, 2-a, 3-c, 4-d c) 1-b, 2-a, 3-d, 4-c

14. As mutaes, recombinao gnica, seleo natural, migrao e oscilao gnica podem originar novas espcies por: 1. Especiao devido ao isolamento reprodutivo. 2. Radiao adaptativa que constitui um fenmeno similar conveno adaptativa. 3. Radiao adaptativa devido adaptao de espcies diferentes que se adaptam a certas particularidades do ambiente e que possuem uma srie de caractersticas semelhantes. 4. Convergncia adaptativa, como por exemplo a evoluo dos marsupiais na Austrlia. a) 1 b) 3 c) 2 e 3 d) 4 e 5 e) 2 e 4

15. Analise as afirmativas abaixo: 1. A clera uma doena produzida por uma bactria transmitida ao homem atravs da gua e de alimentos contaminados pelo vibrio colrico. 2. O ttano, causado pelo vrus Clostridium tetani, pode ser prevenido atravs da vacinao especfica. 3. Tuberculose, difteria e hansenase so doenas cuja profilaxia feita atravs de programas de vacinao. 4. A doena de Chagas, causada pelo Trypanossoma cruzi, e a giardase, causada pela Giardia lamblia, so exemplos de doenas acarretadas por parasitas flagelados. Esto corretas apenas: a) 1, 2 e 4 b) 2 e 3 c) 1 e 4 d) 1, 3 e 4 e) 2, 3 e 4

62

BIOLOGIA

UFPE
F A S E

16. O HIV o vrus responsvel pela doena conhecida como AIDS (ou SIDA), que causa diminuio da imunidade por reduo do nmero de linfcitos T. A este respeito, analise as afirmativas abaixo: 1. Mes portadoras do HIV no devem amamentar seus filhos, pois esta uma das formas de transmisso do vrus. 2. O uso de preservativos (camisinha) um meio eficaz para evitar a transmisso do vrus da AIDS (e outras doenas) atravs de relaes sexuais. 3. O fato do HIV ser um vrus bastante mutagnico facilitar o desenvolvimento de vacinas em curto prazo. 4. O indivduo contaminado com o HIV pode permanecer meses sem desenvolver os sintomas da doena, e apresentar resultados negativos nos testes para detectar o vrus. 5. O perodo de incubao da doena de 1 a 3 meses e neste perodo o indivduo no pode transmitir o vrus. Esto corretas: a) 1, 2 e 5 b) 2, 3 e 4 c) 3, 4 e 5 d) 1, 3 e 4 e) 1, 2 e 4

PARA CADA UMA DAS QUESTES A SEGUIR, ASSINALE NA COLUNA I A(S) PROPOSIO(ES) VERDADEIRA(S) E NA COLUNA II, A(S) FALSA(S). 1. As sentenas abaixo mostram o desenvolvimento das idias que podem explicar a existncia de melhores espcies animais e vegetais. 0-0) Enquanto Lamarck acreditava que a evoluo baseava-se nas alteraes provocadas pelo ambiente e transmitidas a seus descendentes, a teoria de Darwin defendia que a seleo dos indivduos mais adaptados ao ambiente determinava o processo evolutivo. 1-1) Na teoria moderna da evoluo, a seleo natural atua sobre a variabilidade geneticamente determinada, fonte de mutaes e recombinao promovida pela reproduo sexuada. 2-2) So evidncias a favor da teoria evolucionista, o documentrio fssil que demonstra a existncia de espcies diferentes das atuais, os estudos da anatomia comparada que revelam a existncia de indivduos de espcies diferentes organizados segundo o mesmo plano estrutural, e as evidncias bioqumicas que demonstram a similaridade entre protenas de diferentes espcies. 3-3) Embora as mutaes, a recombinao gentica e a seleo natural sejam mecanismos bsicos para o condicionamento das alteraes de um conjunto gnico de uma populao, mecanismos como a imigrao e a hibridao, que independem da seleo natural, podem atuar de forma complementar nesse processo. 4-4) A origem de novas espcies envolve mecanismos de isolamento (geogrfico ou reprodutivo), que impedem a troca de genes entre dois grupos de animais da mesma populao.

F A S E

63

BIOLOGIA
2. A respeito das caractersticas gerais das clulas podemos afirmar que:

UFPE
F A S E

0-0) A bactria Escherichia coli um exemplo de clula eucarionte. 1-1) A membrana plasmtica das clulas procariontes lipoprotica, possui permeabilidade seletica e projees, denominadas mesossomos, que contm enzimas respiratrias. 2-2) A membrana plasmtica das clulas eucariontes pode apresentar especializaes com a funo de aumentar a superfcie de absoro (microvilosidades e invaginaes) ou aumentar a adeso entre as clulas de certos tecidos (desmossomos e interdigitaes). 3-3) O hialoplasma das clulas procariontes formado por gua, sais minerais, aminocidos, nucleotdeos, RNA-t, RNA-m, protenas, filamentos, partculas de glicognio e gotculas de lipdeos. 4-4) O complexo de Golgi participa da formao da cobertura externa das clulas vegetais pois atua como sistema de transporte de substncias pcticas, tanto para membrana plasmtica quanto para lamela mdia. 3. Sobre o metabolismo energtico celular, podemos afirmar que: 0-0) Todos os plastos, orgnulos exclusivos dos vegetais, so dotados de pigmentos fotossintetizantes que esto relacionados bioconverso de energia luminosa em qumica. 1-1) So similaridades entre os cloroplastos e mitocndrias: capacidade de duplicao pelo fato de possurem DNA prprio, presena de ADP, ATP, NADP e RNA. 2-2) Na etapa fotoqumica da fotossntese, a energia transferida atravs de quantias de comprimento de onda entre 400 e 700 nm absorvida pelas clorofilas e transferida, atravs dos eltrons, aos seguintes aceptores: plastoquinona, citocromos b e f, plastocianina, ferredoxina e NADP. 3-3) A respirao aerbia compreende trs fases que ocorrem em diferentes regies da mitocndria: a gliclise, responsvel pela decomposio da glicose a cido pirvico; o ciclo de Krebs, responsvel pelo transporte de hidrognio atravs das molculas de NAD e FAD; e a cadeia respiratria, que atravs da degradao do cido ctrico e ao de vrios citocromos transporta eltrons para o oxignio. 4-4) As reaes envolvidas na respirao anaerbia so as mesmas da glicose, porm, na fermentao ocorre a produo de somente 2 molculas de ATP por molcula da glicose, pois o substrato no totalmente oxidado, restando energia nas molculas de lcool etlico. 4. Observe a figura ao lado e avalie as proposies. 0-0) Representao de cromossomos de uma clula haplide durante a metfase. 1-1) Estruturas constitudas de DNA, protenas, RNA e ons. 2-2) Os cromossomos do grupo C possuem constrices primrias e secundrias, constitudas de heterocromatina. 3-3) O cromossomo Y e os cromossomos dos grupos A e B so, respectivamente, telocntrico, metacntricos, e sub-metacntricos. 4-4) Representao do genoma de uma clula sexual.

64

BIOLOGIA

UFPE
F A S E

5. Alguns mecanismos de troca entre a clula e o meio ambiente esto representados na figura abaixo. Avalie as proposies a seguir.

2 A

P R O V A 0-0) A letra a representa o processo de difuso simples no qual ocorre a passagem de solutos pequenos (O2 CO2, gua e ons) do local de maior para o de menor concentrao. 1-1) Se a molcula a ser transportada atravs da membrana celular for a gua, a letra a indicar o processo de osmose. 2-2) No processo de difuso facilitada (letra b) ocorre uma mudana conformacional nas protenas de membrana que cria um canal para passagem de solutos especficos (aminocidos, lipdeos e aucares), do meio mais concentrado para o menos concentrado. 3-3) A difuso simples (letra a) e a facilitada (letra b) so exemplos de transporte passivo, pois em ambos os mecanismos no h gasto de energia para a clula. 4-4) O processo de migrao de substncias do local de sua menor concentrao para o de maior concentrao, que depende da energia fornecida pelas mitocndrias e da presena de protenas carreadoras especficas na membrana, denomina-se transporte ativo (letra c). 6. A respeito do processo respiratrio, observe as proposies abaixo: 0-0) Nos porferos, cnidrios, platelmintos e asquelmintos, a superfcie do corpo suficientemente grande e o nmero de camadas celulares suficientemente pequeno para permitir a livre difuso do oxignio e do gs carbnico atravs da pele. 1-1) O aparelho respiratrio sempre uma estrutura ramificada, com grande superfcie relativa, que absorve oxignio dissolvido na gua ou diretamente no ar. 2-2) Os animais aquticos possuem dobras da pele muito vascularizadas, responsveis pela respirao, chamadas brnquias. 3-3) As aves possuem, alm do pulmo muito mais ramificado que o dos anfbios e rpteis, sacos areos que se comunicam com o mesmo e com cavidades cheias de ar nos ossos. Esses sacos areos atuam como reservatrio complementar, diminuem a densidade do corpo e atuam como meio de refrigerao. 4-4) O transporte de oxignio pelo sangue depende de pigmentos respiratrios constitudos de protenas associadas a um metal. Em todos os animais que possuem os referidos pigmentos, com exceo dos crustceos, alguns aracndeos e moluscos, esse metal o ferro.

65

BIOLOGIA
7. Observe as proposies sobre o processo digestivo dos animais:

UFPE
F A S E

0-0) Os artrpodes e moluscos so os primeiros animais na escala evolutiva a apresentarem glndulas como o fgado e o pncreas, que secretam grandes quantidades de enzimas digestivas. 1-1) O incio da digesto dos alimentos nos seres humanos se d na boca, onde o amido transformado em fragmentos menores e maltose, por ao da amilase salivar. 2-2) No estmago dos seres humanos, o cido clordrico ativa o tripsinognio, transformandoo em tripsina que degrada protenas em peptdeos e aminocidos. 3-3) Nos seres humanos, o quimo, quando entra em contato com a parede intestinal, estimula a produo de colecistocinina e secretina que agem sobre o pncreas e a vescula biliar, estimulando-os a lanar no duodeno o suco pancretico e a bile, respectivamente. 4-4) A finalizao do processo digestivo ocorre, nos seres humanos, no intestino grosso por ao do suco intestinal que contm maltase, sacarase, lactase, erepsina, aminopeptidases e lipases. 8. No que se refere fisiologia animal, avalie as seguintes proposies: 0-0) As contraes do corao so controladas pelo sistema nervoso autnomo simptico, pela acetilcolina, noradrenalina e por uma estrutura prpria chamada rede de Purkinge, que determina o ritmo das batidas. 1-1) Nos invertebrados, com exceo dos artrpodes e cefalpodes, a circulao aberta, pois o sangue sai dos vasos e os rgos ficam mergulhados em lacunas cheias de sangue denominadas hemoceles. 2-2) Os anfbios e rpteis possuem circulao dupla e incompleta, pois a existncia de um s ventrculo no corao, mesmo que parcialmente dividido pelo formen de Paniza, permite a mistura do sangue venoso com o arterial. 3-3) Nos insetos a excreo ocorre atravs dos tubos de Malpighi, que retiram excretas do celoma e os liberam para fora, atravs de aberturas na pele. 4-4) O funcionamento dos rins dos seres humanos ocorre em duas etapas: a filtrao, que consiste na passagem de gua, sais, glicose, aminocidos e uria do sangue do glomrulo para a cpsula de Bowman, e a reabsoro, na qual parte dos sais e molculas orgnicas (por transporte ativo) e gua (por osmose) retornam ao sangue. As substncias txicas, ou em excesso so eliminadas na urina. 9. Analise as proposies a respeito das figuras ao lado: 0-0) Na figura 1 observa-se o tecido que reveste internamente o corao e que se caracteriza por apresentar o mnimo de substncia intercelular. 1-1) Na figura 2 est representada uma clula com funo condutora, formada a partir dos neuroblastos. 2-2) A clula da micrglia (figura 3) origina-se dos espongioblastos e tem funo de nutrir os neurnios. 3-3) As fibras musculares estriadas (4) apresentam contrao rpida e involuntria, enquanto as lisas (5) apresentam contrao lenta e voluntria. 4-4) Na figura 6 encontramos algumas clulas sangneas responsveis pelo processo de defesa do organismo.

66

BIOLOGIA

UFPE
F A S E

10. A partir da figura que mostra o processo de diferenciao do meristema, avalie as proposies a seguir:

0-0) O meristema primrio, com o desenvolvimento do embrio, diferencia-se em 3 partes fundamentais, indicadas pelas letras a (pleroma), b (periblema) e c (dermatognio). 1-1) A letra d indica o parnquima, tecido com funo de preenchimento que pode participar da fotossntese e do armazenamento de substncias. 2-2) A letra e indica o tecido composto por clulas desprovidas de cloroplastos, impregnadas com cutina e que apresentam ncleo grande e citoplasma delgado com grande vacolo. 3-3) O tecido condutor de seiva bruta denominado lenho ou xilema (letra f) tem como principal componente a traquia. 4-4) A letra g representa o lber ou floema, cujo principal elemento de conduo da seiva elaborada o vaso liberiano (letra h), composto por clulas mortas, impregnadas de lignina. 11. Observe as proposies a seguir: 0-0) Bactrias so seres unicelulares desprovidos de carioteca, que podem ser utilizados na fabricao de vinagre e coalhada. 1-1) As cianofcias so seres procariontes que fixam nitrognio atmosfrico e se reproduzem, exclusivamente, de forma assexuada. 2-2) Algas verdes, pardas, vermelhas e douradas, alm de fungos e protozorios, pertencem ao reino Protista, pois so seres eucariontes uni ou pluricelulares, cujas clulas no formam tecidos especializados. 3-3) Enquanto as carapaas das algas vermelhas so utilizadas para fabricao de abrasivos, certas algas pardas fornecem gar-gar, material utilizado como meio nutritivo em pesquisas biolgicas. 4-4) Os protozorios so seres uni e pluricelulares que se reproduzem, exclusivamente, de forma sexuada e podem viver como parasitas em organismos animais, causando doenas como a malria, disenteria e doena de Chagas.

67

BIOLOGIA

UFPE
F A S E

12. A partir da anamnese de um paciente com anemia, perturbaes digestivas e aumento do volume do corao, foram levantadas algumas hipteses sobre o mal que o afligia. 0-0) Como nas casas vizinhas do paciente agentes de sade coletaram vrios exemplares do inseto Triatoma infestans, a cardiomegalia poderia ser explicada por uma infestao causada pelo protozorio Trypanossoma cruzi. 1-1) Os mesmos agentes de sade tambm coletaram o inseto Phebotomus intermedius, porm, como o paciente no apresentava ulceraes decorrentes da instalao e reproduo da leishmnia no tecido conjuntivo, a hiptese de leishimaniose foi descartada. 2-2) Embora a Doena de Chagas, causada pela injeo da saliva do barbeiro infectado no momento da picada, explicasse as perturbaes cardacas do paciente, esta no poderia explicar a anemia e as perturbaes digestivas apresentadas. 3-3) As condies de higiene descritas pelo paciente e as perturbaes digestivas apresentadas pelo mesmo embasaram a hiptese de uma verminose por Ascaris ou Necatur, associada Doena de Chagas. 4-4) A ascaridase foi detectada atravs do exame que revelou a presena de proglotes nas fezes do paciente. 13. Os seres vivos em seu ambiente e as relaes entre eles so objeto de estudo da ecologia. Analise as proposies abaixo: 0-0) O mutualismo, comensalismo e inquilinismo so exemplos de relaes harmoniosas interespecficas exemplificadas, respectivamente, pela relao entre algas/fungos formando os lquens, paguro-eremita /anmonas do mar e rmora / tubaro. 1-1) Nas cadeias alimentares h um fluxo de energia unidirecional e decrescente dos produtores em direo aos consumidores. 2-2) A partir da anlise de pirmides de biomassa conclui-se que quando se passa de um nvel trfico para outro superior (no sentido produtor-consumidor) h aumento do tamanho dos indivduos e, portanto, tambm da biomassa. 3-3) O nitrognio um elemento qumico que faz parte da composio de vrias biomolculas importantes, porm, apesar de sua abundncia na atmosfera (78%), a grande maioria dos seres vivos no consegue usar o nitrognio na forma molecular em qualquer processo biolgico. 4-4) Eutroficao consiste na superadubao de um lago ou rio de guas calmas, com conseqente florao e posterior morte dos animais. 14. Numa populao de 1000 indivduos, 160 apresentam uma determinada anomalia. Sabendose que esta anomalia determinada por um par de alelos recessivos, autossmicos e de segregao independente, qual das alternativas abaixo expressa corretamente o nmero provvel de indivduos que, embora no manifeste a doena, so portadores deste alelo recessivo? 0-0) 840 1-1) 420 2-2) 240 3-3) 480 4-4) 280

68

BIOLOGIA

UFPE
F A S E

15. Foram analisados dados de famlias nas quais ocorriam casos de uma determinada anomalia e verificou-se que, em todas, os casamentos entre homens afetados e mulheres normais culminavam, sempre, em proles nas quais todas as fmeas eram afetadas e todos os machos eram normais. De possa destes dados pode-se concluir que a referida molstia condicionada por um alelo: 0-0) Dominante autossmico. 1-1) Autossmico recessivo. 2-2) Dominante ligado ao Y. 3-3) Recessivo ligado ao X. 4-4) Dominante ligado ao X.

16. Se em um determinado mapa gentico a distncia entre os locos A e B de 16 unidades de mapa (morgandeos), qual das alternativas abaixo indica as freqncias relativas dos gametas AB - Ab - aB e ab formados por um indivduo de gentipo AB ? ab
GAMETAS ALTERNATIVAS AB Ab aB ab

(0-0) (1-1) (2-2) (3-3) (4-4)

36% 34% 42% 08% 44%

14% 16% 08% 42% 06%

14% 16% 08% 42% 06%

36% 34% 42% 08% 44%

17. Analise a veracidade das proposies abaixo, considerando a sntese de uma protena a partir de um segmento de DNA com a seguinte seqncia de bases: CCC - CAT - AAA - CGG. 0-0) No processo de transcrio ocorrer a sntese do RNA-m, de forma complementar aos cdons da fita de DNA, que conter a seqncia: GGG - GUA - UUU - GCC. 1-1) A traduo da seqncia de bases do RNA-m ser realizada no ncleo pelos ribossomos. 2-2) No citoplasma, os aminocidos iro ligar-se aos anticdons das molculas de RNA-t. 3-3) A seqncia para ligao dos diferentes aminocidos s molculas de RNA-t (CCC, CAU, AAA, CGG) diferente e especfica para cada aminocido. 4-4) A protena formada a partir do segmento de DNA considerado poder ter, no mximo, 4 aminocidos. 18. Analise as sentenas abaixo: 0-0) Endemias so doenas que aparecem repentina e inesperadamente em regies restritas. 1-1) Parasitismo a relao entre seres e espcies diferentes, na qual um indivduo (hospedeiro) fornece ao outro (parasita) energia e habitat. 2-2) A forma mais comum de profilaxia de protozooses a vacinao. 3-3) Vrios parasitas, principalmente protozorios, possuem um hospedeiro intermedirio no qual habitam as formas adultas que posteriormente infectam o hospedeiro definitivo. 4-4) O processo teraputico mais indicado para combater infeces causadas por vrus e bactrias consiste no uso de antibiticos que podem matar ou inibir o crescimento e multiplicao microorganismos.

69

BIOLOGIA
19. O heredograma ao lado refere-se a uma caracterstica controlada por um nico par de alelos. Analise-o cuidadosamente antes de responder s proposies apresentadas.

UFPE
F A S E

II 0-0) Trata-se de uma anomalia rara na populao, causada por um alelo ligado ao cromossomo X. III 1-1) III-2, IV-2 e IV-4 tm gentipo homozigtico recessivo (aa). IV 2-2) Se IV-2 casar-se com uma mulher de mesmo fentipo, todos os seus filhos tero o mesmo gentipo. 3-3) Para a descendncia do casamento entre III-3 x III-4 espera-se que a metade dos filhos seja homozigtica e a outra metade seja heterozigtica. 4-4) VI-1 e IV-3 so genotipicamente iguais a IV-6 e IV-8, a saber, AA. 20. Na figura a seguir est mostrada, de forma esquemtica, a germinao do plen em angiospermas. Com relao ao assunto correto afirmar que:

0-0) O gro de plen germina emitindo um prolongamento, denominado tubo polnico, o qual cresce atravs do estilete, em direo micrpila do vulo. 1-1) Na ponta do tubo polnico, orientando o seu crescimento, situa-se o ncleo reprodutivo, e, logo atrs deste, encontra-se o ncleo vegetativo. 2-2) Antes de atingir o vulo, o ncleo reprodutivo divide-se e origina dois ncleos espermticos haplides. 3-3) O tubo polnico penetra no vulo pela micrpila e atinge o saco embrionrio no interior do qual ocorre a unio de um dos ncleos espermticos com a oosfera e a fuso do outro ncleo espermtico com os ncleos polares. 4-4) O ncleo vegetativo funde-se com as antpodas, localizadas na regio central do saco embrionrio, formando um tecido triplide, rico em reservas nutritivas, chamado endosperma.

70

BIOLOGIA
21. Analise o ciclo de vida de uma samambaia, dado ao lado. Em seguida responda ao que se pede. A meiose ocorre em: 0-0) I e III 1-1) II 2-2) III e IV 3-3) IV 4-4) V

UFPE
F A S E

22. Aps a fertilizao, o vulo desenvolve-se originando semente e o ovrio tambm cresce e transformase no fruto. Em alguns casos, porm, a parte comestvel no provm do ovrio ou apenas dele. Analise o quadro abaixo, referente a tipos de fruto (angiospermas) e as alternativas propostas. 0-0) Fruto carnoso, indeiscente, com pericarpo suculento, do tipo baga.

1-1)

Fruto seco, tipo drupa, com semente encerrada em um caroo.

2-2)

Fruto carnoso, do tipo cariopse, com uma semente concrescida com o endocarpo, isto , intimamente aderida a ele.

3-3)

Fruto seco, indeiscente, tipo aqunio.

4-4)

Pseudofruto no qual a parte comestvel o pednculo da flor.

71

BIOLOGIA

UFRGS

1. Em determinada formao vegetal brasileira, o estudo das relaes solo/vegetao evidenciou que o aspecto caracterstico das plantas que a formam se deve NO falta de gua (apesar de uma estao seca pronunciada, h razovel umidade no solo a partir de dois metros de profundidade) e sim, a um solo pobre em nutrientes, cido e com um alto teor de alumnio, o que provoca o denominado escleromorfismo oligotrfico aluminotxico. O texto acima se refere a que tipo de vegetao? a) Mata de restinga b) Caatinga c) Cerrado d) Mata com araucria e) Campo rupestre

2. Relacione as atividades humanas (coluna da direita) com os problemas ambientais diretamente envolvidos (coluna da esquerda). 1. 2. 3. 4. Urbanizao crescente Extrativismo desenfreado Queima de combustveis fsseis Agricultura intensiva ( ( ( ( ) ) ) ) eroso e infertilidade do solo acelerao da eutrofizao dos recursos hdricos chuva cida extino de recursos naturais de origem vegetal e/ou animal

A seqncia numrica correta, de cima para baixo, na coluna da direita, : a) 4 - 1 - 3 - 2 b) 3 - 4 - 2 - 1 c) 2 - 1 - 3 - 4 d) 1 - 2 - 3 - 4 e) 2 - 4 - 1 - 3

3. A frase abaixo apresenta cinco segmentos sublinhados. Assinale a letra correspondente ao segmento que contm um erro.
A sobrevivncia de uma floresta pluvial tropical em um solo de baixa fertilidade pode ser (a) explicada pelo acmulo da maior parte das reservas dos nutrientes ocorrer na biomassa. (b) vegetal, e a ciclagem dos elementos essenciais se dar de forma rpida e eficiente, o que permite, no (c) (d)

caso de retirada da floresta, o imediato restabelecimento da comunidade anterior, atravs de uma sucesso secundria. (e)

72

BIOLOGIA

UFRGS

4. O esquema abaixo representa parte dos organismos que constituem uma comunidade e as inter-relaes que os mantm.

Considerando este esquema, qual das seguintes afirmativas est correta? a) b) c) d) O fitoplncton e o zooplncton constituem o nvel trfico dos produtores. Gaivotas e bigus so predadores e fazem parte do 4 nvel trfico. Peixes e moluscos, por serem onvoros, so considerados consumidores primrios. As bactrias e fungos so os auttrofos decompositores com maior disponibilidade energtica. e) Os moluscos so consumidores primrios, tendo a sua disposio uma menor quantidade energtica. 5. As afirmaes abaixo esto relacionadas ao conceito de nicho ecolgico. I - A grande variedade de formas de vida que freqentemente os ectonos apresentam pode ser explicada por seu elevado nmero de nichos ecolgicos. II - Em determinado ecossistema, espcies diferentes que tm o mesmo habitat podem, durante muito tempo, ocupar um nicho ecolgico idntico. III - Espcies taxonomicamente distantes, como o biso norte-americano e os grandes cangurus australianos, podem ocupar nichos ecolgicos semelhantes em regies geogrficas diferentes. Quais afirmativas esto corretas? a) Apenas I b) Apenas II c) Apenas III d) Apenas I e III e) Apenas II e III

73

BIOLOGIA

UFRGS

6. Relacione os processos biolgicos listados na coluna da esquerda com um ou mais dos ciclos biogeoqumicos na coluna da direita. 1. Fotossntese 2. Respirao vegetal 3. Decomposio aerbica de restos orgnicos por microorganismos (a) Ciclo do carbono (b) Ciclo do oxignio (c) Ciclo do nitrognio

Assinale a alternativa que melhor representa estas relaes: a) b) c) d) e) 1 (a) (b) 1 (a) (b) 1 (b) 1 (b) 1 (b) (c) - 2 (a) (b) - 2 (a) (b) - 2 (a) - 2 (a) - 2 (c) - 3 (a) (b) (c) - 3 (c) - 3 (a) (b) - 3 (b) (c) - 3 (b) (c)

7. Essencial para a vida terrestre, o surgimento de um ovo, com uma casca resistente e flexvel, com uma membrana interna e muito vitelo para nutrir o embrio, foi capaz de proteger a prole de um determinado animal contra a dessecao e o choque fsico durante o desenvolvimento embrionrio. O texto acima se refere, na escala evolutiva, a que animal? a) peixe b) anfbio c) rptil d) ave e) mamfero

8. O esquema ao lado representa a distribuio de cinco grupos de organismos em trs eras geolgicas. A largura das reas representa o nmero relativo de espcies de cada grupo. Dos grupos apresentados, o que melhor caracteriza a era III a) A, porque deixa nesse tempo geolgico o maior nmero de espcies. b) B, por ter-se originado nesse tempo geolgico e se extinguido em outros. c) C, por deixar nesse tempo geolgico o menor nmero de espcies. d) D, por ter-se distribudo eminentemente nesse tempo geolgico. e) E, por ter sido o ltimo a aparecer nesse tempo geolgico.

74

BIOLOGIA

UFRGS

9. Escolha a alternativa que melhor completa o quadro abaixo, com relao aos grupos de invertebrados inferiores:

10. Existem duas grandes teorias que tentam explicar os mecanismos pelos quais os organismos evoluram e continuam a evoluir. Tanto Lamarck como Darwin apresentam um fator como primordial para a evoluo. A diferena que, para Lamarck, este fator a causa direta das variaes e, para Darwin, este mesmo fator seria o que seleciona dentre as variaes possveis a mais adaptada. Este fator a) b) c) d) e) o ambiente. a grande capacidade de reproduo. a competio. a variao hereditria transmissvel. a migrao.

75

BIOLOGIA

UFRGS

11. Em uma comparao sob o ponto de vista de favorecimento evolutivo e adaptao, a reproduo sexuada mais importante que a assexuada. Qual das alternativas abaixo, com relao reproduo sexuada, melhor justifica esta afirmativa? a) b) c) d) e) Sempre se processa aps meiose que produz gametas. exclusiva de formas de vida mais evoluda. D origem a um maior nmero de descendentes. Permite uma maior constncia no genoma dos descendentes. Promove uma maior variabilidade gentica na populao.

12. Existem conceitos abrangentes em biologia que so considerados generalizaes cientficas de alto nvel, por serem passveis de aplicao a todos os organismos conhecidos. No caso, a seleo natural um bom exemplo deste tipo de conceito. Podemos considerar, como condio primordial para que ocorra a seleo natural, a) b) c) d) e) a existncia de elevada taxa de mutao. a existncia de diversidade fenotpica. a existncia de diversidade genotpica. uma baixa taxa de mortalidade na populao. um ambiente em modificao.

13. Os animais, atravs de seu metabolismo, produzem substncias que devem ser excretadas. Dentre estas, encontramos as nitrogenadas. A substncia a menos txica dos catablitos nitrogenados e tambm a menos solvel em gua, sendo a excreta de e . Assinale a alternativa que contm os termos que preenchem corretamente as lacunas da afirmativa acima.
SUBSTNCIAS ORGANISMOS

1. 2. 3. 4. 5.

uria amnia cido rico guanina alanina

(a) (b) (c) (d) (e)

mamferos rpteis aves peixes anfbios d) 4 - (d) - (e) e) 5 - (d) - (e)

a) 1 - (a) - (b) b) 2 - (b) - (c) c) 3 - (b) - (c)

76

BIOLOGIA

UFRGS

14. A relao superfcie/volume implica vrias conseqncias fisiolgicas. Assim, a taxa de troca com o ambiente diretamente proporcional superfcie de contato, e a massa proporcional ao volume; ou seja: maior volume, mais consumo de alimento e de oxignio. Com relao ao pargrafo anterior, so feitas as seguintes afirmativas: I - A estrutura respiratria de grandes animais apresenta dobramentos que aumentam a rea de absoro. II - Os animais de grande porte tm uma taxa metablica maior do que a dos pequenos. III - Em vrios animais, o aumento da superfcie se d em relao a dimenses lineares, e o volume aumenta em relao a dimenses cbicas. Quais esto corretas? (a) Apenas I (b) Apenas II (c) Apenas I e II (d) Apenas I e III (e) Apenas II e III

15. Relacione a coluna da direita com a coluna da esquerda, identificando corretamente os tipos de estruturas excretoras dos animais: 1) 2) 3) 4) 5) ameba rato minhoca planria gafanhoto ( ( ( ( ( ) ) ) ) ) tubos de Malpighi nefrdios rins vacolo pulstil clulas-flama

A ordem correta, na coluna da direita, de cima para baixo, a) 2 - 1 - 5 - 3 - 4 b) 5 - 3 - 2 - 1 - 4 c) 4 - 1 - 2 - 5 - 3 d) 1 - 4 - 2 - 3 - 5 e) 5 - 4 - 3 - 2 - 1

16. O esquema ao lado representa um corte transversal de um animal, mostrando os trs folhetos embrionrios bsicos. Considere as seguintes afirmativas, a partir do esquema: I - O animal um celomado. II - Os rins originam-se do mesoderma; folheto embrionrio indicado pelo n 3. III - O tubo neural, que formar o sistema nervoso, origina-se do ectoderma; folheto embrionrio indicado pelo n 1. Quais esto corretas? a) Apenas I b) Apenas II c) Apenas III d) Apenas II e III e) I, II e III

77

BIOLOGIA

UFRGS

17. Qual a razo de os vasos nos quais cultivamos plantas terrestres ornamentais possurem um orifcio no fundo para drenar o excesso de gua? a) b) c) d) e) Elevar o pH da soluo do solo e assim permitir uma melhor nutrio das plantas. Diminuir a velocidade da decomposio da matria orgnica do solo. Evitar o crescimento demasiado das razes e a conseqente necessidade de transplante do vegetal. Evitar uma quantidade excessiva de nutrientes disponveis planta. Permitir a presena de espaos arejados no solo, com oxignio suficiente para a respirao das razes.

18. Monocotiledneas e dicotiledneas podem geralmente ser diferenciadas I - pelo tipo de clima no qual se desenvolvem. II - pelas caractersticas das folhas, dos caules e das razes. III - pelas estruturas das sementes. Quais esto corretas? a) Apenas I b) Apenas II c) Apenas III d) Apenas II e III e) I, II e III

19. Entre os tecidos vegetais, a madeira desempenhou um papel decisivo na histria da humanidade. Sob o ponto de vista anatmico, a madeira corresponde ao a) xilema secundrio. b) floema primrio. c) sber. d) crtex. e) cmbio vascular.

20. Os espermatfitos compreendem as plantas vasculares que se reproduzem sexualmente atravs de sementes. Fazem parte deste grupo a) b) c) d) e) samambaias, palmeiras e pinheiros. samambaias, gramneas e leguminosas. samambaias, algas macroscpicas e leguminosas. palmeiras, musgos e leguminosas. palmeiras, gramneas e pinheiros.

21. Durante o processo reprodutivo das angiospermas, o transporte do gro de plen da antera ao estigma da flor e a funo do ncleo vegetativo denominam-se, respectivamente, a) b) c) d) e) formao do tubo polnico e polinizao. formao do tubo polnico e fecundao. polinizao e formao do tubo polnico. fecundao e polinizao. polinizao e fecundao.

78

BIOLOGIA

UFRGS

22. Relacione a coluna da esquerda, que apresenta as funes, com a coluna da direita, onde esto as organelas. 1) 2) 3) 4) digesto celular respirao presena de material gentico presena de substncias para exportao ( ( ( ( ) ) ) ) ncleo mitocndria lisossoma complexo de golgi

A seqncia numrica correta, de cima para baixo, na coluna da direita, a) 3 - 4 - 2 - 1 b) 3 - 2 - 1 - 4 c) 4 - 3 - 2 - 1 d) 3 - 1 - 2 - 4 e) 4 - 3 - 1 - 2

23. O deslocamento de um atleta para locais de grande altitude pode acarretar-lhe algumas alteraes no organismo. A curto prazo, provoca modificaes da atividade respiratria e, a longo prazo, produz alteraes sangneas. Assim, este indivduo apresenta a) b) c) d) e) hiperventilao e diminuio do nmero de hemcias. hiperventilao e aumento do nmero de hemcias. hipoventilao e diminuio do nmero de hemcias. hipoventilao e aumento do nmero de hemcias. isoventilao e manuteno do nmero de hemcias.

24. A uma condio padro de temperatura e de presso, 1 mol de um gs ocupa 22,3 litros de volume. Considerando que a solubilidade do dixido de carbono no plasma de 0,03 mmol/mm Hg/litro de plasma, calcule esta solubilidade em mililitros/mm Hg/litro de plasma. a) 0,10 b) 1,37 c) 0,67 d) 2,34 e) 11,89

25. A secreo do hormnio de crescimento STH produz quais dos seguintes efeitos? ( = estimula) ( = inibe)

79

BIOLOGIA

UFRGS
lana sangue, atravs da(s) para os pulmes,

26. A sstole do(a) .


1. 2. 3. 4.

Veias Pulmonares Direita Veia Cava Superior trio Direito Ventrculo Direito

6. Ventrculo Esquerdo 7. trio Esquerdo 8. Veias Pulmonares Esquerda 9. Artria Pulmonar 10. Asorta

5. Veia Cava Inferior

Assinale a alternativa cujos termos preenchem corretamente as lacunas da afirmativa acima. a) b) c) d) e) 4 6 3 7 6 9 10 2e5 1e8 10 onde ocorrer a hematose de onde ir para todo o corpo onde ocorrer a hematose onde ser oxigenado onde ser oxigenado

27. A hipotermia pode ser utilizada em algumas intervenes cirrgicas. Entre estas podemos citar a cirurgia cardaca. Quais as vantagens de submetermos um organismo hipotermia? I - A circulao fica muito reduzida. II - O sangramento mnimo. III - H uma elevao da taxa metablica. Quais esto corretas? a) Apenas I b) Apenas II c) Apenas III d) Apenas I e II e) Apenas II e III

28. Para que um impulso nervoso possa ser transmitido de um neurnio para outro, necessria a liberao, na fenda sinptica, de mediadores qumicos. Um desses mediadores a a) insulina. b) tirosina. c) vasopressina. d) acetilcolina. e) histamina.

80

BIOLOGIA

UFRGS

29. A comunicao da vescula biliar com o intestino delgado feita pelo ducto biliar, que libera a bile. Se por algum motivo houver uma obstruo do ducto biliar, o que poder ocorrer? a) b) c) d) e) A pepsina perder sua atividade. A digesto dos carboidratos ocorrer mais rapidamente. Produzir uma alcalinizao do intestino. A digesto dos lipdios ser mais lenta. Haver mais tripsina atuando sobre os lipdios.

30. Quando uma pessoa picada por um animal peonhento, deve procurar socorro atravs de a) b) c) d) e) soro, que induzir a formao de anticorpos. soro, porque composto de antgenos especficos. soro, porque contm anticorpos prontos. vacina, porque fornecer ao organismo elementos de defesa. vacina, para eliminar quimicamente o veneno.

31. Muitas vezes, durante a realizao de eventos esportivos, realizada a determinao do sexo gentico. Este exame feito pela observao dos cromossomos de clulas epiteliais. Pode-se afirmar que neste exame a) mulheres normais deveriam apresentar uma estrutura chamada corpsculo de Barr, que corresponde a um dos cromossomos X. b) homens normais deveriam apresentar uma estrutura chamada corpsculo de Barr, que corresponde ao cromossomo Y. c) mulheres normais deveriam apresentar duas estruturas chamadas corpsculos de Barr, que correspondem aos dois cromossomos X. d) homens normais deveriam apresentar uma estrutura chamada corpsculo de Barr, correspondente ao cromossomo X. e) mulheres normais na fase adulta no deveriam apresentar corpsculo de Barr. 32. Existe uma doena condicionada por um gene recessivo, localizado na regio no homloga do cromossomo X, que leva a uma debilidade muscular progressiva (distrofia muscular progressiva). - Uma mulher com fentipo normal tem um irmo afetado e um tio materno tambm afetado pela doena; - seus pais no so afetados; - ela est casada com um homem normal. Qual a probabilidade de que um(a) filho(a) deste casal venha a ser afetado pela doena? a) b) c) d) e) Se a mulher for homozigota, a probabilidade 100%. Se a mulher for homozigota, a probabilidade zero. Se a mulher for homozigota, a probabilidade 25%. Se a mulher for heterozigota, a probabilidade 75%. Se a mulher for heterozigota, a probabilidade 100%.

81

BIOLOGIA

UFRGS

33. Se um carter tem trs alelos possveis, podendo haver seis gentipos, e um segundo carter apresenta oito gentipos possveis, quando ambos forem estudados simultaneamente, podem ocorrer a) 7 gentipos. b) 12 gentipos. c) 24 gentipos. 34. Considere as seguintes afirmativas: I - As protenas so molculas de grande importncia para os organismos - atuam tanto estruturalmente como tambm metabolicamente. II - As enzimas so protenas que atuam como catalisadores biolgicos. III - Existem protenas que atuam como linhas de defesa do organismo e algumas delas so conhecidas como anticorpos. Quais esto corretas? a) Apenas I b) Apenas II c) Apenas III d) Apenas II e III e) I, II e III d) 48 gentipos. e) 96 gentipos.

35. O esquema abaixo mostra a via metablica de um aminocido, levando formao de alguns hormnios:

A respeito deste esquema so feitas as seguintes afirmaes: I - Um indivduo com a enzima 1 inibida acumular grandes quantidades de aminocido 3. II - Um indivduo com a enzima 1 inibida deixa de produzir as enzimas 2 e 3. III - Um indivduo com a enzima 1 inibida precisa receber os hormnios 1 e 2 de fontes externas. Quais esto corretas? a) Apenas I b) Apenas II c) Apenas III d) Apenas I e II e) Apenas II e III

82

BIOLOGIA
Nas questes de 1 a 14, marque:

UnB

itens CERTOS, na coluna I; itens ERRADOS, na coluna II. Use, para as devidas marcaes, a Folha de Rascunho e, posteriormente, a Folha de Respostas. Questo 1 A aventura humana na Terra tem-se caracterizado, com o passar dos tempos, por um esforo contnuo rumo a novas descobertas. Considerando algumas caractersticas desse esforo, julgue os itens abaixo. (0) A Biologia, tal qual as outras cincias, segue o procedimento geral do mtodo cientfico: no possui tcnicas particulares nem evoluo prpria. (1) Leis e teorias podem ser corrigidas, aperfeioadas e at substitudas, medida que se descobrem novos fatos ou se realizam novas experincias. (2) Embora no empregue testes controlados, o conhecimento comum descreve a aparncia dos fenmenos, examinando seus efeitos. Questo 2 Em seus estudos sobre fotossntese, um cientista colocou uma planta em um sistema fechado, sujeito a diferentes intensidades luminosas. Medindo a variao de O2, obteve valores que permitiram a construo do seguinte grfico, no qual a indicao positiva da variao de O2 significa a produo desse gs.

Em relao anlise desse grfico, julgue os itens seguintes. (0) No segmento AB, a taxa de respirao menor que a taxa de fotossntese. (1) No ponto B, a taxa de fotossntese igual da respirao. (2) No ponto C, a planta atingiu o ponto de saturao: o aumento da intensidade luminosa no faz aumentar a taxa da fotossntese. (3) O mesmo experimento poderia ser efetuado, substituindo-se a planta por um fungo.

83

BIOLOGIA
Questo 3

UnB

Os primeiros citologistas, observando alguns seres vivos, detectaram unidades morfolgicas comuns a todos eles as clulas , cada uma delas contendo as informaes hereditrias de todo o organismo. Com relao clula, julgue os itens a seguir. (0) A destruio do nuclolo de uma clula no afeta o metabolismo dessa clula. (1) Se a membrana lisossmica se romper dentro de uma clula, causar a destruio desta. (2) Diferentemente de uma ameba marinha, a ameba de gua doce pode no apresentar vacolos pulsteis. (3) As clulas epiteliais do intestino grosso apresentam microvilosidades que aumentam a superfcie de absoro do alimento. Questo 4 O fermento do po, assim como o levedo da cerveja, so constitudos por uma levedura geralmente do gnero Saccharomyces. Uma suspenso dessa levedura em meio nutritivo foi submetida a diferentes condies, registrando-se o seguinte resultado.

PRODUO DE LCOOL CONDIO I CONDIO II + no ocorreu

PRODUO DE CO2 no ocorreu +

MASSA CELULAR APS 24H + +++

Com auxlio das informaes acima, julgue os itens seguintes. (0) Na condio I, h ausncia de O2 no meio nutritivo. (1) H maior massa celular na condio II, porque o processo de obteno de energia que nela ocorre tem maior rendimento que o processo da condio I. (2) Na condio I, ocorre menos produo de ATP do que na condio II. (3) As clulas, na condio II, esto realizando fermentao. (4) Para a produo de cerveja, colocam-se as clulas em condies semelhantes condio I.

84

BIOLOGIA
Questo 5

UnB

Em um experimento realizado com duas espcies de protozorios do mesmo nvel trfico, o crescimento das populaes de cada uma das espcies, em um meio de cultura, foi registrado em duas situaes diferentes: com as duas espcies em separado e com as duas espcies juntas. Os resultados obtidos so apresentados nos grficos abaixo.

Em relao a esse experimento, julgue os seguintes itens. (0) A capacidade de suporte de espcie I maior que a da espcie II. (1) As duas espcies no coexistem, no experimento, porque possuem tamanhos de populao diferentes quando crescem no meio em separado. (2) A populao da espcie II diminuiu, quando junto com a espcie I, porque esgotaram-se os recursos do meio de cultura. Questo 6 A evoluo dos vertebrados do meio aqutico para o ambiente terrestre foi acompanhada de modificaes morfofisiolgicas no aparelho respiratrio. Com relao a esse tema, julgue os itens que se seguem. (0) Nos anfbios, a pele constitui um importante auxiliar na respirao, devido ao fato de a taxa de oxignio ser maior no ar do que na gua. (1) Os peixes utilizam, para respirar, o oxignio dissolvido na gua. (2) A fecundao, nos rpteis, externa e dependente de gua; nos anfbios, no. (3) Nos mamferos, os pulmes alveolares atingem a mxima complexidade entre os vertebrados.

85

BIOLOGIA
Questo 7

UnB

O ciclo de vida dos vegetais terrestres pode ser esquematizado conforme o diagrama abaixo.

Com relao a esse tema, julgue os itens a seguir. (0) A durao da fase esporoftica, nos vegetais, aumenta progressivamente dos vegetais inferiores (Brifitos) para os superiores (Angiospermas). (1) A meiose ocorre no esporfito. (2) Para que a fertilizao ocorra nos Pteridfitos, necessria a presena de gua. (3) A fase diplide (2n), nos vegetais, vai da fertilizao aos esporos (ou plen). Questo 8 Em 1859, Charles Darwin props, no livro intitulado A origem das espcies, a sua teoria da evoluo dos organismos. Julgue os itens abaixo, relativos a esse tema. (0) Entre os agentes de evoluo dos organismos esto o acasalamento no-aleatrio e a seleo natural. (1) A seqncia de fsseis de espcies diferentes das atuais, em camadas de rochas antigas, uma evidncia a favor da evoluo dos organismos. (2) A resistncia dos insetos ao DDT e das bactrias aos antibiticos serve de evidncia para a existncia da seleo natural. (3) A descoberta recente de vestgios de vida microbiana fssil em um meteorito proveniente do planeta Marte, anunciada pela NASA, se for confirmada, ser uma evidncia contra a evoluo da vida na Terra.

86

BIOLOGIA
Questo 9

UnB

O Lago Parano, em Braslia, recebe guas de vrias fontes, incluindo crregos e afluentes, contendo diferentes tipos de pesticida. Esses pesticidas so absorvidos inicialmente pelas algas e por outras plantas, ou so absorvidos em detritos orgnicos, podendo, depois de certo tempo, chegar at o homem. O esquema abaixo representa a seqncia citada.

A partir das informaes acima, julgue os itens seguintes. (0) Uma cadeia alimentar uma seqncia de organismos no mesmo nvel trfico, como, por exemplo: gara martim pescador peixes maiores. (1) As algas podem ser consideradas consumidores primrios. (2) A maior concentrao de pesticidas ser encontrada no homem. (3) O esquema apresenta simplificadamente uma rede alimentar. Questo 10 O povo brasileiro viveria melhor e os gastos com tratamentos de sade seriam em grande parte evitados se houvesse um maior esclarecimento da populao com relao preveno de doenas. Acerca de algumas das informaes que poderiam ser veiculadas para se obter esse maior esclarecimento, julgue os itens que se seguem. (0) Alguns fatores que predispem ao enfarte do miocrdio so o estresse, o fumo, a alta taxa de colesterol e o uso de pilulas anticoncepcionais. (1) A hepatite, a verminose e a febre amarela so algumas das doenas parasitrias que podem ser evitadas consumindo-se sempre gua filtrada. (2) Doenas causadas por protozorios como o Trypanosoma cruzi e o Plasmodium sp podem, atualmente, ser evitadas pela vacinao. (3) Os riscos de se contrarem doenas como a gonorria, a AIDS e a sfilis podem ser diminudos, entre outras formas, por um comportamento sexual monogmico.

87

BIOLOGIA
Questo 11

UnB

O tema da fecundao in vitro tem aparecido com freqncia nos meios de comunicao, seja pelo uso por pessoas famosas, seja pelo debate tico suscitado, por exemplo, pela destruio massiva de embries humanos. Basicamente, a tcnica de fecundao in vitro consiste nos seguintes passos: III III IV Vinduo de superovulao na mulher com o uso de hormnios; cirurgia para a extrao dos vulos maduros (cerca de oito); fecundao in vitro dos vulos por espermatozides; introduo de cerca de quatro dos embries resultantes no tero da mulher; e congelamento dos embries restantes em N2 lquido, na maioria das vezes.

Mesmo com esse procedimento, utilizando-se vrios embries, a gravidez ocorre em cerca de apenas 30% dos casos, nas clnicas mais avanadas. Considerando o exposto acima, julgue os itens que se seguem. (0) A probabilidade de nidao ser maior ou menor, dependendo da fase do ciclo menstrual em que se encontra a mulher que recebe os embries. (1) O passo I necessrio porque, normalmente, s amadurecem dois vulos por ms, um em cada ovrio. (2) Se, no passo IV, houver nidao de dois embries, haver o nascimento de gmeos idnticos. (3) O passo V preserva os embries pela drstica diminuio do metabolismo desses. Questo 12 O esquema abaixo representa a sntese protica.

Analise o esquema e julgue os itens a seguir. (0) (1) (2) (3) (4) Ao final da sntese, todas as protenas representadas no esquema sero iguais. Os ribossomos esto se movendo da parte de baixo para a de cima do esquema. No esquema, no est representado o RNA transportador, que tambm participa do processo. O conjunto representado pode estar localizado no retculo endoplasmtico rugoso. O nmero de aminocidos da protena depender do tamanho do RNA mensageiro.

88

BIOLOGIA
Questo 13

UnB

Est em andamento o projeto Genoma, pelo qual se pretende seqenciar totalmente o DNA humano, ou seja, obter a seqncia de bases nitrogenadas do DNA de todos os cromossomos de uma pessoa. A esse respeito, julgue os itens abaixo. (0) Uma vez obtida a seqncia de um gene, ser possvel conhecer a seqncia de aminocidos da protena correspodente. (1) As seqncias obtidas correspondero exatamente ao DNA de qualquer pessoa. (2) A seqncia de todos os cromossomos ser do mesmo tamanho. (3) O projeto Genoma tem levantado questes ticas, pelo eventual uso inadequado que se possa fazer do conhecimento obtido, como, por exemplo, a discriminao de pessoas. (4) Todos os genes tero a mesma proporo de adenina e guanina. Questo 14 Com relao colorao da pelagem de gatos, observam-se os seguintes fentipos:
FMEAS AMARELAS MACHOS AMARELOS FMEAS MANCHADAS (AMARELO E PRETO) FMEAS PRETAS MACHOS PRETOS

Utilizando as informaes acima, julgue os itens a seguir. (0) O carter cor da pelagem, nos gatos, condicionado por genes ligados ao cromossomo X, no havendo dominncia. (1) As fmeas manchadas so heterozigotas para o carter cor da pelagem. (2) Os descendentes de uma fmea amarela, cruzada com um macho preto, sero manchados. (3) Um gato macho no transmite para seus filhos machos os genes para cor da pelagem.

89

BIOLOGIA
Na questo 15 a seguir, marque: o algarismo das DEZENAS, na coluna I; o algarismo das UNIDADES, na coluna II.

UnB

O algarismo das DEZENAS deve ser obrigatoriamente marcado, mesmo que seja igual a zero. Use, para as devidas marcaes, a Folha de Rascunho e, posteriormente, a Folha de Respostas. Questo 15 Dois casais suspeitavam da troca de seus bebs no berrio da maternidade. Os casais e os bebs foram submetidos tipagem do sangue quanto ao sistema ABO, cujos resultados obtidos so mostrados na tabela abaixo. Analisando-os, pode-se identificar os pais de cada beb.

PESSOA beb n 1 beb n 2 me X pai X me Y pai Y

TIPO SANGNEO O A AB B B B

Aps identificar os pais do beb n 2, calcule a probabilidade, em porcentagem, de que um futuro irmo deste beb seja do sexo masculino e venha a ter tipo sangneo diferente do do irmo. Despreze a parte fracionria do seu resultado, caso exista.

90

BIOLOGIA

UNESP
F A S E

1. Qual das afirmativas melhor explica o aumento da competio entre os membros de uma populao de ratos selvagens numa determinada rea?

a) Aumento na taxa de reproduo de ratos selvagens. b) c) d) e) Aumento de falces predadores. Epidemia de raiva. Aumento da temperatura. Aumento da alimentao.

2. Observe a figura. O bioma representado conhecido como a) mata atlntica. b) tundra. c) taiga. d) cerrado. e) floresta decdua temperada.

3. Uma diferena bsica entre plantas e animais a capacidade que as plantas apresentam para a) digerir carboidratos. b) concentrar o CO2 c) realizar a respirao. d) adaptar-se a ambientes. e) resistir s doenas.

4. Em moscas de frutas, a cor branca dos olhos devida herana ligada ao sexo. Uma mosca fmea com olhos coloridos, cuja me tinha olhos brancos, cruza-se com macho de olhos brancos. Qual a probabilidade de se obter uma fmea de olhos brancos deste cruzamento? a) 1,00. b) 0,67. c) 0,50. d) 0,25. e) 0,00.

91

BIOLOGIA
5. A transfuso de sangue do tipo B para uma pessoa do grupo A resultaria em a) b) c) d) e) reao de anticorpos anti-B do receptor com os glbulos vermelhos do doador. reao dos antgenos B do receptor com os anticorpos anti-B do doador. formao de anticorpos anti-A e anti-B pelo receptor. nenhuma reao, porque A receptor universal. reao de anticorpos anti-B do doador com antgenos A do receptor.

UNESP
F A S E

6. O diagrama ilustra uma fase da sntese de protenas. Os algarismos I, II, III e IV correspondem, respectivamente, a: a) b) c) d) e) ribossomo, cdon, RNAm e RNAt. RNAt, RNAm, ribossomo e cdon. RNAt, RNAm, ribossomo e anticdon. RNAm, RNAt, ribossomo e cdon. RNAm, RNAt, ribossomo e anticdon.

7. A especiao do Homo sapiens tem pouca chance de ocorrer, considerando-se a atual condio da espcie humana. Assinale a afirmao que melhor sustenta esta hiptese. a) b) c) d) e) A cincia moderna tem eliminado as mutaes humanas. Os medicamentos atuais diminuem a incidncia de doenas. Os postulados de Darwin no se aplicam espcie humana. As alteraes ambientais que favorecem a especiao so cada vez menores. Os meios modernos de locomoo e comunicao tm diminudo ou eliminado os isolamentos geogrficos.

8. Os itens de I a VI apresentam, no necessariamente na seqncia, os passos pelos quais um vrus replicado. III III IV VVI sntese das protenas do vrus. adeso da capa do vrus com a membrana celular. produo de protenas. abandono da cpsula. liberao do vrus da clula. replicao do RNA viral.

Assinale a alternativa que apresenta todos esses passos na seqncia correta. a) II - IV - I - VI - III - V. b) VI - IV - I - III - V - II. c) II - VI - IV - III - I - V. d) V - II - I - IV - VI - III. e) II - IV - VI - I - III - V.

92

BIOLOGIA
9. So exemplos de tecido de sustentao, conduo e proteo, respectivamente: a) b) c) d) e) sber - traquedeos - esclernquima. epiderme - esclernquima - sber. sber - colnquima - fibras. esclernquima - traquedeos - sber. colnquima - xilema - traquedeos.

UNESP
F A S E

10. Considere os esquemas, numerados de 1 a 6, que mostram os diferentes estgios que ocorrem durante o processo de clivagem. Observe que eles no esto na seqncia correta de acontecimentos. Em qual alternativa o desenvolvimento embrionrio est em ordem seqencial totalmente correta? a) 3 - 6 - 1 - 4 - 5 - 2. b) 5 - 3 - 1 - 4 - 6 - 2. c) 3 - 5 - 2 - 1 - 6 - 4. d) 1 - 3 - 5 - 6 - 4 - 2. e) 3 - 1 - 5 - 2 - 6 - 4.

11. Considere o esquema do sistema digestivo humano ao lado. Os rgos que produzem enzimas digestivas que digerem protenas so: a) 1, 4 e 5. b) 1, 4 e 6. c) 4, 5 e 6. d) 1, 3 e 7. e) 2, 3 e 8.

12. A reabsoro de gua pelos rins regula a osmorregularidade do sangue, graas ao de um hormnio produzido pela hipfise. Esse hormnio : a) somatotrofina. b) epinefrina. c) secretina. d) hormnio antidiurtico. e) hormnio luteinizante.

93

BIOLOGIA

UNESP
F A S E

1. Um determinado candidato a prefeito prometeu que, se fosse eleito, faria uma grande ampliao da rede de esgotos e do tratamento de gua de sua cidade, para erradicar ou diminuir a doena de Chagas e a malria. Ele realizou a sua promessa, mas falhou parcialmente no seu intento; entretanto, conseguiu erradicar o clera e a amebase. Qual a explicao biolgica para a) a falha apontada? b) o sucesso conseguido? 2. Quatro exemplares de uma mesma espcie de planta, de tamanhos iguais, foram colocados separadamente em quatro tubos com gua, numerados de 1 a 4. Cada tubo foi exposto a uma diferente intensidade luminosa. O gs produzido por cada planta foi coletado, medido e identificado como sendo oxignio. Os resultados obtidos esto representados no grfico.
Volume de O2 em ml

a) Identifique e explique o fenmeno que produziu esses resultados. b) Qual a previso para produo de O2, se um quinto tubo (n 5), recebendo maior intensidade de luz, fosse colocado no experimento? 3. Em gatos, as cores marrom e branca dos pelos tm sido descritas como devidas a, pelo menos, um par de genes. Considere o cruzamento de gatos homozigotos brancos e marrons. Qual a proporo fenotpica esperada em F1 se a) o modo de herana for do tipo dominante? b) o modo de herana for do tipo co-dominante? 4. Uma revista publicou uma reportagem com o ttulo Atleta com anomalia gentica faz operao para definir seu sexo e poder competir na classe feminina de jud. A matria dizia, ainda, que a jovem era um caso de pseudo-hermafroditismo, pois apresentava rgos sexuais internos masculinos e rgos sexuais externos femininos. Os testculos da atleta foram extirpados para que ela pudesse competir na equipe feminina de jud. a) Que vantagem a atleta levaria sobre as demais competidoras, se tivesse os testculos durante a competio? b) Sabendo-se que ela foi considerada do sexo feminino, que teste citogentico voc faria para comprovar esta afirmao?

94

BIOLOGIA
5. A capacidade de coagular o sangue reduzida nos indivduos hemoflicos.

UNESP
F A S E

a) Qual a protena do sangue que atua no final do processo da coagulao? b) Que tipo de clula sangnea responsvel pela liberao de substncias que auxiliam na coagulao? 6. A anlise qumica em amostras de cinco lminas com cidos nucleicos apresentou os seguintes resultados: 1 lmina: ribose 2 lmina: uracila 3 lmina: dupla-hlice 4 lmina: timina 5 lmina: 15% de guanina e 25% de citosina. a) Entre estas lminas, quais se referem a DNA? b) Justifique o resultado obtido com a 5 lmina. 7. Um casal tem cinco filhos: Alex, Pedro, Mrio, rica e Ana. Dois dos irmos so gmeos univitelinos. rica, um dos gmeos, sofreu um acidente e precisa urgentemente de uma transfuso de sangue, e os nicos doadores disponveis so seus irmos. Na impossibilidade de se fazer um exame dos tipos sangneos, responda: a) Entre seus irmos, qual seria a pessoa mais indicada para ser o doador? b) Justifique sua resposta. 8. Um animal tem quatro cavidades no corao, com a artria aorta voltada para a direita, e apresenta hemcias nucleadas. a) A que classe de vertebrados pertence este animal? b) Cite outra caracterstica exclusiva desta classe. 9. Quando um neurnio estimulado, vrias alteraes eltricas ocorrem em sua membrana (axnio), as quais so basicamente comandadas pelos movimentos de ons. Quando o nvel do estmulo suficiente, forma-se o impulso nervoso. a) Quais so os ons que comandam estas alteraes eltricas que formam o impulso nervoso? b) Que nome se d regio entre os neurnios, onde ocorre a transmisso do impulso? 10. Os vegetais apresentam, entre outros compostos orgnicos, a celulose, de alto valor energtico (calrico). Ruminantes, que so essencialmente herbvoros, no apresentam enzimas para digerir este carboidrato. a) De que forma os ruminantes conseguem o aproveitamento deste nutriente? b) Cite um invertebrado que se utiliza do mesmo processo para se alimentar da celulose.

95

FSICA

CESGRANRIO

1. Durante as Olimpadas de 96, estimou-se que, ao ser batida uma falta por um jogador brasileiro, a bola atingia a velocidade de 187 km/h. Considere o campo com 110 m de comprimento. Uma falta batida do crculo central contra o gol adversrio. Supondo que a bola se desloque praticamente em linha reta e com velocidade constante, o tempo que ela levar para atingir a meta vale, em segundos, aproximadamente: a) 1,0 b) 1,2 c) 1,5 d) 1,8 e) 2,0

2. Trs blocos, A, B e C, de mesmo peso P, esto empilhados sobre um plano horizontal. O coeficiente de atrito entre esses blocos e entre o bloco C e o plano vale 0,5. Uma fora horizontal F aplicada ao bloco B, conforme indica a figura. O maior valor que F pode adquirir, sem que o sistema ou parte dele se mova, : a) P/2 b) P c) 3P/2 d) 2P e) 3P

3. Um bloco de ferro mantido em repouso sob o tampo de uma mesa, sustentado exclusivamente pela fora magntica de um m, apoiado sobre o tampo dessa mesa. As foras relevantes que atuam sobre o m e sobre o bloco de ferro correspondem, em mdulo, a: P1: peso do m. F1: fora magntica sobre o m. N1: compresso normal sobre o m. P2: peso do bloco de ferro. F2: fora magntica sobre o bloco de ferro. N2: compresso normal sobre o bloco de ferro. Sendo P1 a) b) c) d) e) N1 P1 P1 P1 F1 P2, correto escrever: N2 2F1 F2 P2 F1 P2 N1 F2 P1 P2

4. Um carrinho de brinquedo movido a pilha tem 0,5 kg de massa total e desloca-se em linha reta com movimento uniformemente acelerado sobre uma superfcie horizontal. Uma fotografia estroboscpica registra a posio do carrinho a cada 1,0 s, conforme mostra a figura. Em t 0, a velocidade do carrinho nula.

O mdulo, em newtons, da resultante das foras que agem sobre o carrinho durante o movimento vale: a) 0,1 b) 0,2 c) 0,3 d) 0,4 e) 0,5

96

FSICA

CESGRANRIO

5. Dois copinhos de massa desprezvel so pendurados nas extremidades de uma haste de alumnio, sendo o conjunto suspenso por um fio, conforme indica a figura abaixo.

O copinho da esquerda (A) contm 60 gros de feijo, e a massa da haste de alumnio equivale a 48 gros de feijo (suponha gros de massas idnticas). Logo, o nmero de gros de feijo que deve ser colocado no copinho da direita (B) para que o sistema permanea em equilbrio, com a haste na posio horizontal, : a) 61 b) 63 c) 65 d) 67 e) 69

6. Uma chapa de metal, homognea e fina (de espessura constante), cortada para formar as faces de dois cubos ocos C1 e C2 , sendo que a aresta de C2 o dobro da aresta de C1. A densidade do cubo menor d. Logo, a densidade do cubo maior : a) 2d b) d c) d/2 d) d/4 e) d/8

7. De acordo com um locutor esportivo, em uma cortada do Negro (titular da Seleo Brasileira de Voleibol), a bola atinge a velocidade de 108 km/h. Supondo que a velocidade da bola imediatamente antes de ser golpeada seja desprezvel e que a sua massa valha aproximadamente 270g, ento o valor do impulso aplicado pelo Negro bola vale, em unidades do S.I., aproximadamente: a) 8,0 b) 29 c) 80 d) 120 e) 290

8. Um palhacinho de papelo est suspenso a uma bola de aniversrio. O conjunto paira no ar, sem subir nem descer. Assim, correto afirmar que a(o): a) b) c) d) e) densidade do palhacinho menor que a densidade da bola. densidade do conjunto igual densidade do ar. empuxo que o ar exerce sobre a bola igual ao peso do palhacinho. peso do palhacinho igual ao peso da bola. o peso da bola menor que o peso do palhacinho.

97

FSICA

CESGRANRIO

9. A montanha russa Steel Phantom do parque de diverses de Kennywood, nos EUA, a mais alta do mundo, com 68,6 m de altura acima do ponto mais baixo. Caindo dessa altura, o trenzinho desta montanha chega a alcanar a velocidade de 128 km/h no ponto mais baixo. A percentagem de perda da energia mecnica do trenzinho nesta queda mais prxima de: a) 10% b) 15% c) 20% d) 25% e) 30%

10. Na superfcie horizontal do patamar superior de uma escada, uma esfera de massa 10 g rola de um ponto A para um ponto B, projetando-se no ar a partir desse ponto para os degraus inferiores. Cada degrau tem altura de 20 cm e largura de 30 cm.

Considerando-se desprezvel a resistncia do ar e g 10 m/s , a velocidade mnima que a esfera deve ter ao passar pelo ponto B, para no tocar no primeiro degrau logo abaixo, , em m/s, igual a: a) 0,6 b) 0,8 c) 1,0 d) 1,2 e) 1,5

11. Uma caixa de filme fotogrfico traz a tabela apresentada abaixo, para o tempo de revelao do filme, em funo da temperatura dessa revelao. TEMPERATURA TEMPO (em minutos) 65F (18C) 10,5 68F (20C) 9 70F (21C) 8 72F (22C) 7 75F (24C) 6

A temperatura em F corresponde exatamente ao seu valor na escala Celsius, apenas para o tempo de revelao, em min, de: a) 10,5 b) 9 c) 8 d) 7 e) 6

12. Em um calormetro ideal misturam-se uma certa massa de vapor dgua a 100C com uma outra massa de gelo a 0C, sob presso que mantida normal. Aps alguns minutos, estabelecese o equilbrio trmico. Dentre as opes abaixo, assinale a nica que apresenta uma situao impossvel para o equilbrio trmico. a) b) c) d) e) Gelo e gua a 0C. Apenas gua a 0C. Apenas gua a 50C. Apenas gua a 100C Apenas vapor dgua a 100C.

98

FSICA
13. So quatro as etapas do ciclo de funcionamento de uma mquina trmica. 1 etapa (A B): expanso isobrica 2 etapa (B C): expanso isotrmica 3 etapa (C D): contrao isobrica 4 etapa (D A): compresso isomtrica Assinale o diagrama P

CESGRANRIO

V (presso versus volume) correspondente a este ciclo.

14. Uma fibra ptica consiste em um tubo estreito e macio, tendo basicamente um ncleo e uma casca constitudos de dois tipos de vidro (ou plstico) de ndices de refrao diferentes. A luz que penetra por uma extremidade da fibra, atravs do ncleo, sofre mltiplas reflexes totais na superfcie lateral que separa o ncleo da casca at sair pela outra extremidade. A figura abaixo representa um raio de luz que penetra na fibra, proveniente do ar, emergindo na extremidade oposta.

Sendo nA, nN e nC os ndices de refrao do ar, do ncleo e da casca, respectivamente, a relao correta : a) b) c) d) e) nA nA nA nA nA nC nC nC nC nC nN nN nN nN nN

99

FSICA

CESGRANRIO

15. Considere um espelho esfrico cncavo, de foco F e centro de curvatura C, como representado abaixo.

Objetos colocados nas regies 2, 3 e 4 tero imagens formadas, respectivamente, nas regies: a) 8, 6 e 7 b) 7, 6 e 5 c) 5, 8 e 7 d) 5, 7 e 6 e) 1, 8 e 7

16. Um raio luminoso, propagando-se no ar, atravessa uma lente de vidro plano-cncava, como est representado nas figuras abaixo.

Dentre as configuraes apresentadas, est(o) correta(s): a) apenas a I. b) apenas a II. c) apenas I e a III. d) apenas a I e a IV. e) apenas a II e a III.

17. No circuito ao lado, sabe-se que a resistncia equivalente entre os pontos A e B vale 3 Ohms. Ento, o valor da resistncia R, em Ohms, deve ser igual a: a) 3 b) 4 c) 5 d) 6 e) 7

18. Trs lmpadas, cujas resistncias internas valem R, 2R e 3R, so ligadas a 110 Volts, conforme indica o circuito ao lado. A razo entre as ddp na lmpada de resistncia R e na lmpada de resistncia 3R vale: a) 1/5 b) 1/3 c) 2/5 d) 3/5 e) 5/6

100

FSICA
19. A figura ao lado representa as linhas de um campo eltrico uniforme. A ddp entre os pontos A e B vale 24 Volts. Assim, a intensidade desse campo eltrico, em Volt/metro, vale: a) 60 b) 80 c) 120 d) 150 e) 200

CESGRANRIO

3 cm

20. Um condutor XY percorrido por uma corrente eltrica de intensidade i, gerando, ao seu redor, um campo magntico de intensidade B.

Uma partcula de carga eltrica positiva q lanada com velocidade inicial V0, paralelamente ao condutor e logo abaixo dele, ficando submetida a uma fora magntica Fm. Assinale a opo que representa corretamente o vetor fora Fm, no instante em que a carga q lanada.

101

FSICA
1. Os corpos A, B e C tm massas iguais. Um fio inextensvel e de massa desprezvel une o corpo C ao B, passando por uma roldana de massa desprezvel. O corpo A est apoiado sobre o B. Despreze qualquer efeito das foras de atrito. O fio f mantm o sistem em repouso. Logo que o fio f cortado, as aceleraes aA, aB e aC dos corpos A, B e C sero, a) b) c) d) e) aA = 0; aA = g/3; aA = 0; aA = 0; aA = G/2; aB = g/2; aB = g/3; aB = g/3; aB = g; aB = g/2; aC = g/2 aC = g/3 aC = g/3 aC = g aC = g/2

FUVEST
F A S E

2. Um holofote constitudo por dois espelhos esfricos cncavos E1 e E2, de modo que a quase totalidade da luz proveniente da lmpada L seja projetada pelo espelho maior E1, formando um feixe de raios quase paralelos. Neste arranjo, os espelhos devem ser posicionados de forma que a lmpada esteja aproximadamente a) b) c) d) e) nos focos dos espelhos E1 e E2. no centro de curvatura de E2 e no vrtice de E1. no foco de E2 e no centro de curvatura de E1. nos centros de curvatura de E1 e E2. no foco de E1 e no centro de curvatura de E2.

3. Uma bolinha pendurada na extremidade de uma mola vertical executa um movimento oscilatrio. Na situao da figura, a mola encontra-se comprimida e a bolinha est subindo com velocidade V. Indicando por F a fora da mola e por P a fora peso aplicadas na bolinha, o nico esquema que pode representar tais foras na situao descrita acima

4. Dois recipientes de material termicamente isolante contm cada um 10 g de gua a 0C. Deseja-se aquecer at uma mesma temperatura os contedos dos dois recipientes, mas sem mistur-los. Para isso usado um bloco de 100 g de uma liga metlica inicialmente temperatura de 90C. O bloco imerso durante um certo tempo num dos recipientes e depois transferido para o outro, nele permanecendo at ser atingido o equilbrio trmico. O calor especfico da gua dez vezes maior que o da liga. A temperatura do bloco, por ocasio da transferncia, deve ento ser igual a a) 10C b) 20C c) 40C d) 60C e) 80C

102

FSICA
5. Um carro se desloca numa trajetria retilnea e sua velocidade em funo do tempo, a partir do instante t = 10 s, est representada no grfico. Se o carro partiu do repouso e manteve uma acelerao constante at t = 15 s, a distncia percorrida, desde sua partida at atingir a velocidade de 6 m/s, vale: a) 12,5 m b) 18,0 m c) 24,5 m d) 38,0 m e) 84,5 m

FUVEST
F A S E

6. O circuito da figura formado por 4 pilhas ideais de tenso V e dois resistores idnticos de resistncia R. Podemos afirmar que as correntes i1 e i2, indicadas na figura, valem a) i1 = 2 V/R e i2 = 4 V/R b) i1 = zero e i2 = 2 V/R c) i1 = 2 V/R e i2 = 2 V/R d) i1 = zero e i2 = 4 V/R e) i1 = 2 V/R e i2 = zero

7. Um raio de luz , no plano da folha, incide no ponto C do eixo de um semi-cilindro de plstico transparente, segundo um ngulo de 45 com a normal OC face plana. O raio emerge pela superfcie cilndrica segundo um ngulo de 30 com a direo de OC. Um raio II incide perpendicularmente superfcie cilndrica formando um ngulo 0 com a direo OC e emerge com direo praticamente paralela face plana. Podemos concluir que a) b) c) d) e) 0 = 0 0 = 30 0 = 45 0 = 60 a situao proposta no enunciado no pode ocorrer.

8. Quatro cargas pontuais esto colocadas nos vrtices de um + quadrado. As duas cargas Q e Q tm mesmo valor absoluto e as outras duas, q1 e q2, so desconhecidas. Afim de determinar a natureza destas cargas, coloca-se uma carga de prova positiva no centro do quadrado e verifica-se que a fora sobre ela F, mostrada na figura. Podemos afirmar que a) q1 > q2 > 0 b) q2 > q1 > 0 c) q1 + q2 > 0 d) q1 + q2 < 0 e) q1 = q2 > 0

103

FSICA

FUVEST
F A S E

9. Na figura da esquerda esquematizada uma mquina de solda eltrica. So feitas medidas da voltagem V em funo da corrente I que circula atravs do arco, obtendo-se a curva mostrada na figura da direita

Nos grficos ao lado, as curvas que qualitativamente melhor representem a potncia dissipada P e a resistncia R (R = V/I) do arco, em funo da corrente I so, respectivamente, a) A e Z b) C e Z c) B e Y d) A e X e) B e X

10. Duas esferas de ao, ocas e rgidas, com 1 kg de massa e 3 litros de volume esto cheias de ar e so mantidas submersas e em equilbrio, muito prximas superfcie de um lago, por foras de valor F dirigidas para baixo, como mostra a figura. A esfera A totalmente fechada e a esfera B tem um pequeno furo em sua parte inferior o qual permite a entrada da gua. Puxa-se as duas esferas at uma profundidade de 10 metros abaixo da superfcie do lago. Para mantlas em equilbrio nesta profundidade, os novos valores das foras FA e FB, aplicadas respectivamente nas esferas A e B so tais que a) FA > F e FB < F b) FA = F e FB = F c) FA > F e FB > F d) FA > F e FB = F e) FA = F e FB < F

11. No Sistema Internacional de Unidade (SI), as sete unidades de base so o metro (m), o quilograma (kg), o segundo (s), o kelvin (K), o ampre (A), a candela (cd) e o mol (mol). A lei de Coulomb da eletrosttica pode ser representada pela expresso 1 Q1Q2 F = 2 r 4 0 onde 0 uma constante fundamental da fsica e sua unidade, em funo das unidades de base do SI, a) m s A 3 1 2 b) m kg A 3 1 4 2 c) m kg s A
2 2 2

d) m kg s e) adimensional

104

FSICA
12. Uma certa massa de gs ideal sofre uma compresso isotrmica muito lenta passando de um estado A para um estado B. As figuras representam diagramas TP e TV, sendo T a temperatura absoluta, V o volume e P a presso do gs. Nesses diagramas, a transformao descrita acima s pode corresponder s curvas a) I e IV b) II e V c) III e IV d) I e VI e) III e VI

FUVEST
F A S E

13. Uma pirmide reta, de altura H e base quadrada de lado L, com massa m uniformemente distribuda, est apoiada sobre um plano horizontal. Uma fora F com direo paralela ao lado AB aplicada no vrtice V. Dois pequenos obstculos O, fixos no plano, impedem que a pirmide se desloque horizontalmente. A fora F capaz de fazer tombar a pirmide deve ser tal que L mg 2 d) | F | > H L mg 2 e) | F | >

mgH a) | F | >

L + H2 2
b) | F | > mg mgH c) | F | > L 2

L + H2 2

14. Um avio que voa a grande altura pressurizado para conforto dos passageiros. Para evitar sua exploso estabelecido o limite mximo de 0,5 atmosfera para a diferena entre a presso interna no avio e a externa. O grfico representa a presso atmosfrica P em funo da altura H acima do nvel mar. Se o avio voa a uma altura de 7.000 metros e pressurizado at o limite, os passageiros ficam sujeitos a uma presso igual que reina na atmosfera a uma altura de aproximadamente a) 0 m b) 1.000 m c) 2.000 m d) 5.500 m e) 7.000 m

105

FSICA

FUVEST
F A S E

15. Quando se aproxima um basto B, eletrizado positivamente, de uma esfera metlica isolada e inicialmente descarregada, observa-se a distribuio de cargas representada na Figura 1.

Mantendo o basto na mesma posio, a esfera conectada terra por um fio condutor que pode ser ligado a um dos pontos P, R ou S da superfcie da esfera. Indicando por ( ) o sentido do fluxo transitrio (Q) de eltrons (se houver) e por (+), () ou (0) o sinal da carga final (Q) da esfera, o esquema que representa e Q

16. Uma pequena esfera de massa de modelar est presa na extremidade de um fio formando um pndulo de comprimento L. A esfera abandonada na posio I e, ao atingir o ponto inferior II de sua trajetria, se choca com outra esfera igual ficando grudadas uma na outra e depois prosseguindo juntas at atingirem uma altura mxima h = L/4. Considere a hiptese de que as trs grandezas fsicas dadas na tabela abaixo se conservam. Com relao a essa hiptese, a nica alternativa de acordo com o que aconteceu durante a coliso
ENERGIA CINTICA QUANTIDADE DE MOVIMENTO ENERGIA MECNICA TOTAL

a) b) c) d) e)

Falsa Verdadeira Verdadeira Falsa Falsa

Verdadeira Falsa Verdadeira Falsa Verdadeira

Verdadeira Verdadeira Verdadeira Falsa Falsa

106

FSICA

FUVEST
F A S E

17. Uma jovem, repousando margem de um canal, observa uma garrafa levada pela correnteza com velocidade Vg e um barquinho B preso s margens por fios fixados nos pontos M e N. No canal se propaga uma onda com velocidade Vo > Vg no mesmo sentido que a correnteza. Todas as velocidades so medidas em relao jovem. A distncia entre cristas sucessivas da onda, representadas no desenho por C1, C2 e C3, . A jovem v ento a garrafa e o barquinho oscilando para cima e para baixo com freqncias fg e fB que valem

VO + Vg a) fg = VO Vg b) fg = VO c) fg = VO Vg d) fg = VO e) fg =

V0 fB = VO + Vg fB = VO Vg fB = VO fB = VO fB =

18. Trs ims iguais em forma de barra, de pequena espessura, esto sobre um plano. Trs pequenas agulhas magnticas podem girar nesse plano e seus eixos de rotao esto localizados nos pontos A, B e C. Despreze o campo magntico da Terra. A direo assumida pelas agulhas, representadas por (), melhor descrita pelo esquema:

107

FSICA
19. Dois termmetros de vidro idnticos, um contendo mercrio (M) e outro gua (A), foram calibrados entre 0C e 37C, obtendo-se as curvas M e A, da altura da coluna do lquido em funo da temperatura. A dilatao do vidro pode ser desprezada. Considere as seguintes afirmaes:

FUVEST
F A S E

I - O coeficiente de dilatao do mercrio aproximadamente constante entre 0C e 37C. II - Se as alturas das duas colunas forem iguais a 10 mm, o valor da temperatura indicada pelo termmetro de gua vale o dobro da indicada pelo de mercrio. III - No entorno de 18C o coeficiente de dilatao do mercrio e o da gua so praticamente iguais. Podemos afirmar que s so corretas as afirmaes a) I, II e III b) I e II c) I e III d) II e III e) I

20. Um atleta est dentro de um elevador que se move para cima com velocidade constante V. Ele comea a levantar uma massa de 100 kg, inicialmente apoiada no piso do elevador, quando este passa pela altura z = 0,0 m, e termina quando o piso do elevador passa por z = 27,0 m. A massa levantada pelo atleta at uma altura de 2,0 m acima do piso do elevador. O trabalho realizado pelo atleta sobre a massa W. A variao da energia potencial da massa durante o levantamento, em relao ao referencial da Terra, U. Podemos 2 afirmar, usando g = 10 m/s , que a) b) c) d) e) W = 2.000 J W = 2.000 J W = 27.000 J W = 2.000 J W = 29.000 J e e e e e U = 2.000 J U = 29.000 J U = 27.000 J U = 27.000 J U = 29.000 J

1. Um automvel com massa de 1000 Kg percorre, com velocidade constante V = 20m/s (ou 72 Km/h), uma estrada (ver figura) com dois trechos horizontais (I e III), um em subida (II) e um em descida (IV). Nos trechos horizontais o motor do automvel desenvolve uma potncia de 30lW para vencer a resistncia do ar, que pode ser considerada constante ao longo de todo o 2 trajeto percorrido. Suponha que no h outras perdas por atrito. Use g = 10m/s . So dados: sen = 0,10 e sen = 0,15. Determine: a) o valor, em newtons, da componente paralela a cada trecho da estrada das foras FI, FII e FIV, aplicadas pela estrada ao automvel nos trechos I, II e IV, respectivamente. b) o valor, em kW, da potncia PII que o motor desenvolve no trecho II.

F A S E

2
108

FSICA

FUVEST
F A S E

2. A figura representa uma lente convergente L, com focos F e F, e um quadrado ABCD, situado num plano que contm o eixo da lente. Construa, na prpria figura, a imagem A B C D do quadrado, formada pela lente. Use linha tracejadas para indicar todas as linhas auxiliares utilizadas para construir as imagens. Represente com traos contnuos somente as imagens dos lados do quadrado, no que couber na folha. Identifique claramente as imagens A, B, C, D dos vrtices.

3. Dois satlites artificiais A e B descrevem rbitas circulares no plano equatorial da Terra. O satlite A est a uma distncia R do centro da Terra e estacionrio com relao a um observador A fixo em um ponto do equador da Terra. a) Esse mesmo observador v o satlite B passar por uma mesma posio, numa vertical sobre ele, a cada dois dias, sempre mesma hora. Quais os dois possveis valores da velocidade angular de B, no referencial inercial em relao ao qual a Terra gira em torno de seu eixo com um perodo de 24 h ? Expresse o resultado em rd/h. b) Calcule, em funo de RA, os valores dos raios das rbitas correspondentes s velocidades angulares encontradas no item superior.

109

FSICA
4. Um cilindro macio, de massa m = 45 kg, altura H = 0,30m e 2 base de rea S= 0,050 m , est imerso em gua, como mostra a figura, sendo mantido em equilbrio esttico por um fio fino ao 2 qual se aplica uma tenso T0. Use g = 10m/s e considere a 3 massa especfica da gua rm = 1000 kg/m . Comea-se ento a puxar o cilindro na direo , para cima, com velocidade constante e muito pequena.

FUVEST
F A S E

a) trace no papel do grfico abaixo o valor, em newtons, da tenso T no fio em funo da posio y da base inferior do cilindro, desde y = -0,70m at y = +0,50m. Marque os valores da escala utilizada no eixo da tenso T. b) Determine o trabalho total W, em joules, realizado pela tenso T, para o deslocamento descrito no item a.

5. Um conjunto de dois carrinhos com um rapaz sentado no carrinho dianteiro, e nele preso pelo cinto de segurana, encontra-se inicialmente na altura h (posio A da figura) de uma montanha russa. A massa m do rapaz igual massa de cada um dos carrinhos. O conjunto comea a descida com velocidade inicial nula. Ao chegar ao ponto B da parte plana da trajetria, o rapaz solta o carrinho traseiro e o empurra para trs com impulso suficiente para faz-lo retornar ao ponto A de partida, onde o carrinho chega com velocidade nula. Despreze os atritos.

a) Determine a altura mxima H a que chega o carrinho dianteiro. b) Houve variao de energia mecnica do conjunto quando o rapaz empurrou o carrinho traseiro? Se houve, calcule essa variao. Se no houve, escreva a energia mecnica se convervou.

110

FSICA

FUVEST
F A S E

6. O som de freqncia mais baixa, dita fundamental, emitido por um tubo sonoro fechado numa extremidade, corresponde a um comprimento de onda igual a quatro vezes o comprimento L do tubo. Sabe-se que o valor V da velocidade do som no ar pode ser obtido pela expresso V = 20 T , onde V em m/s e T a temperatura absoluta do ar em kelvin (K). Quando o tubo contm ar e estando ambos a 300K (temperatura ambiente), a freqncia fundamental emitida F0=500 hertz. a) Determine o comprimento L do tubo. b) Desprezando a dilatao do tubo, determine a temperatura T, comum ao tubo e ao ar nele contido, na qual a freqncia fundamental emitida 2f0. c) Considerando agora a dilatao do tubo, o valor da freqncia fundamental emitida temperatura T, calculada no item anterior, ser maior, igual ou menor do que 2f0 ? Justifique. 8. Um pesquisador estuda a troca de calor entre um bloco de ferro e certa quantidade de uma substncia desconhecida, dentro de um calormetro de capacidade trmica desprezvel (Ver Figura 1). Em sucessivas experincias, ele coloca no calormetro a substncia desconhecida, sempre no estado slido temperatura T0 - 20C, e o bloco de ferro, a vrias temperaturas iniciais T, medindo em cada caso a temperatura final de equilbrio trmico Te. O grfico da Figura 2 representa o resultado das experincias. A razo das massas do bloco de ferro e da substncia desconhecida mf/ms = 0,8. Considere o valor do calor especfico do ferro igual a 0,1 cal/(gC). A partir destas informaes, determine para a substncia desconhecida:

a) a temperatura de fuso, Tfuso b) o calor especfico, c , na fase slida s c) o calor latente de fuso L

8. Duas cargas pontuais positivas, q1 e q2 = 4q1, so fixadas a uma distncia d uma da outra. Uma terceira carga negativa q3 colocada no ponto p entre q e q2, a uma distncia X da carga q1, 1 conforme mostra a figura. a) Calcule o valor de X para que a fora sobre a carga q3 seja nula. b) Verifique se existe um valor de q3 para o qual tanto a carga q1 como a q2 permanecem em equilbrio, nas posies do item a), sem necessidade de nenhuma outra fora alm das eletrostticas entre as cargas. Caso exista, calcule este valor de q3; caso no exista, escreva no existe e justifique.

111

FSICA

FUVEST
F A S E

9. O circuito abaixo formado por quatro resistores e um gerador ideal que fornece uma tenso V = 10 volts. O valor da resistncia do resistor R desconhecido. Na figura esto indicados os valores das resistncias dos outros resistores.

a) Determine o valor, em ohms, da resistncia R para que as potncias dissipadas em R1 e R2 sejam iguais. b) Determine o valor, em watts, da potncia P dissipada no resistor R1, nas condies do item anterior. 10. Um mol de gs ideal levado lentamente do estado inicial A ao estado final C, passando pelo estado intermedirio B. A figura 1 representa a variao do volume, V do gs, em litros (l), em funo da temperatura absoluta T, para a transformao em questo. A constante universal dos gases vale R = 0,082 atm.l/(mol.K). a) Dentre as grandezas presso, volume e temperatura, quais permanecem constantes no trecho AB ? E no trecho BC ? b) Construa na Figura 2 o grfico da presso P em funo da temperatura absoluta T. Indique claramente os pontos correspondentes aos estados A, B e C. Marque os valores da escala utilizada no eixo da presso P. c) Escreva a funo P(T) que representa a presso P do gs em funo da temperatura absoluta T, no intervalo de 300K a 600K, com seus coeficientes dados numericamente.

112

FSICA
QUESTES DE 1 A 7

UFBA
F A S E

INSTRUO: Assinale as proposies verdadeiras, some os nmeros a elas associados e marque o resultado na Folha de Respostas. Questo 1 Entre duas placas planas e paralelas, eletrizadas, dispostas na direo horizontal, onde se estabelece um campo eltrico uniforme, lanado horizontalmente um feixe de eltrons. Desprezando-se a ao do campo gravitacional, cada eltron, ao atravessar a regio entre as placas: (1) (2) (4) (8) (16) tem a componente vertical da velocidade perpendicular ao vetor campo eltrico. tem a componente horizontal da velocidade modificada. descreve trajetria circular, qualquer que seja a intensidade do campo eltrico. fica submetido a uma acelerao constante. tem a energia cintica modificada.

Questo 2 Um corpo de massa m, inicialmente em repouso sobre um plano horizontal rugoso, adquire movimento retilneo uniforme sob a ao de uma fora constante F , paralela ao plano e de mdulo igual metade do peso do corpo. Sendo g o mdulo da acelerao da gravidade local, correto afirmar: (1) Sobre o corpo em movimento, atua uma fora resultante de direo horizontal. (2) O coeficiente de atrito cintico, para o par de superfcies em contato, 0,5.

5 mg. 2 (8) A fora de atrito esttico mxima, para o par de superfcies em contato, tem o mdulo menor do que o de F . (16) Duplicando-se o mdulo de F , o mdulo da fora de atrito cintico fica reduzido metade.
(4) A resultante das foras que o corpo aplica sobre o plano tem mdulo igual a Questo 3 Um cubo macio e homogneo, totalmente imerso num fluido ideal contido num recipiente aberto, encontra-se em equilbrio, sob a ao da gravidade, conforme figura ao lado. Sendo p0 a presso atmosfrica local, correto afirmar: (1) (2) (4) (8) (16) (32) A densidade do cubo igual densidade do fluido. A diferena de presso, entre os pontos 1 e 2, nula. A presso do fluido, em qualquer profundidade, no depende da forma do recipiente. A presso atmosfrica atua na base inferior do cubo. A intensidade do empuxo que atua no cubo igual do peso do fluido deslocado pelo cubo. A intensidade do empuxo sobre o cubo tanto maior, quanto maior for a profundidade em que o cubo se encontre.

113

FSICA
Questo 4

UFBA
F A S E

Um disco de raio R e um cubo macio de aresta a e densidade d, constitudos do mesmo material, encontram-se em equilbrio trmico, temperatura 0, dilatando-se quando so aquecidos simultaneamente, at a temperatura . Considerem-se R = a e o coeficiente de dilatao linear mdio do material, no intervalo de 0 a . Nessas condies, correto afirmar: (1) Ao passarem da temperatura 0 para , disco e cubo recebem energia sob forma de calor sensvel. (2) O coeficiente de dilatao superficial do disco igual ao coeficiente de dilatao volumtrica do cubo. -1 (4) A densidade do cubo, na temperatura , expressa por d[1 + 3 ( - )] . (8) A conveco trmica responsvel pela propagao do calor, tanto no disco quanto no cubo. (16) Para R =

2 , a dilatao superficial do disco se iguala, numericamente, dilatao 3 volumtrica do cubo.

Questo 5 Os raios luminosos R1, R2, R3, R4 e R5, propagando-se no ar, incidem nos sistemas pticos representados a seguir. A lente biconvexa e o prisma so feitos de vidro transparente; o ngulo limite, para o par de meios ar-vidro, de aproximadamente 42.

Nessas condies, correto afirmar: (1) A direo de propagao de R1 modificada, devido reflexo no espelho plano. (2) R2 ser totalmente absorvido pela superfcie espelhada, se o ponto de incidncia for o vrtice do espelho cncavo. (4) R3 produz, na face AC do prisma, um ngulo de incidncia menor do que 45. (8) R3 totalmente refletido pela face AC do prisma. (16) Ao emergirem da lente biconvexa, R4 e R5 so desviados de modo a se tornarem convergentes.

114

FSICA
Questo 6 Na figura ao lado, esto representadas as linhas de fora e as superfcies eqipotenciais de um campo eltrico uniforme E , de 2 intensidade igual a 10 V/m. Uma partcula de -9 -8 massa igual a 2 x 10 kg e carga eltrica de 10 C abandonada em repouso, no ponto A. Desprezando-se as aes gravitacionais, correto afirmar:

UFBA
F A S E

A distncia d entre as superfcies eqipotenciais 1 m. -7 O trabalho realizado pela fora eltrica, para deslocar a partcula de A at B, 10 J. A velocidade da partcula, no ponto B, 10 m/s. A energia mecnica da partcula mantm-se constante durante seu deslocamento do ponto A ao ponto B. (16) Colocada a partcula no ponto C, a sua energia potencial eltrica maior do que no ponto B. Questo 7 INSTRUO: Efetue os clculos necessrios e marque o resultado na Folha de Respostas. No circuito representado ao lado, os fios de ligao so ideais, a diferena de potencial fornecida pelo gerador G igual a 20V, e as resistncias eltricas dos resistores hmicos R1, R2 e R3 so, respectivamente, 2, 1 e 14. Determine o nmero de resistores de 2 que devem ser associados em srie, entre os prontos A e B, para que o resistor R1 dissipe uma potncia igual a 18W.

(1) (2) (4) (8)

115

FSICA
QUESTES DE 1 A 10

UFBA
F A S E

INSTRUO: Assinale as proposies verdadeiras, some os nmeros a elas associados e marque o resultado na Folha de Respostas. Questo 1
Um satlite artificial, de massa m, descreve movimento circular com velocidade v , em torno da Terra, considerada como esfrica, a uma altura h em relao superfcie terrestre. Admite-se que R e M so, respectivamente, o raio e a massa da Terra e que G a constante de gravitao universal. Considerando-se exclusivamente a interao do satlite com a Terra, correto afirmar: (1) A fora de atrao da Terra sobre o satlite provoca variao no mdulo de v .

2 B

P R O V A

(2) A fora gravitacional mantm o satlite em rbita.

2 (4) O perodo de rotao do satlite igual a | | (R + h). v


(8) A acelerao centrpeta do satlite igual a GM(h + R) . (16) Se existisse um fio ideal interligando o satlite ao centro da Terra, a trao, no fio, seria mM igual a G 2 . h Questo 2 Dois corpos, A e B, de capacidades trmicas CA e CB, massas mA e mB e temperaturas A > B, encontram-se num recipiente termicamente isolado, de capacidade trmica desprezvel. Nessas condies, correto afirmar: (1) Ao atingir o equilbrio trmico, a quantidade de calor cedida pelo corpo A necessariamente igual quantidade de calor absorvida pelo corpo B. (2) A capacidade trmica do sistema igual a CA + CB. (4) Se CA = CB, os corpos A e B so constitudos da mesma substncia. (8) Se a temperatura do equilbrio trmico for BX ocorrer mudana de estado fsico no , corpo B.
-2

CA CB (16) Se m = m , os corpos A e B atingiro a mesma temperatura, ao receberem iguais A B quantidades de calor.

116

FSICA
Questo 3 Com base nos conhecimentos sobre Termodinmica, correto afirmar:

UFBA
F A S E

(1) Quando um gs ideal comprimido rapidamente, a energia interna do gs aumenta. (2) O ciclo de Carnot composto por transformaes isomtricas e isobricas. (4) O rendimento de uma mquina trmica depende exclusivamente da temperatura da fonte quente. (8) No refrigerador, o gs refrigerante remove calor da fonte fria, evaporando-se, e transfere calor fonte quente, condensando-se. (16) Admitindo-se o universo como sistema fsico isolado, a entropia do universo sempre aumenta. Questo 4 Com base nos conhecimentos sobre a interao da luz com a matria, correto afirmar: No eclipse solar, a Lua se interpe entre o Sol e a Terra. A cor que se v num objeto corresponde parcela de luz que absorvida por ele. A polarizao da luz o fenmeno responsvel pela formao do arco-ris. Em qualquer meio material, a luz se propaga com velocidade de aproximadamente 8 3 x 10 m/s. (16) A reflexo luminosa pode ser explicada de acordo com o modelo corpuscular de Newton. (32) Ao sofrer difrao, a luz apresenta comportamento ondulatrio. Questo 5 O grfico ao lado representa as posies ocupadas, em funo do tempo, por um mvel de massa igual a 1kg, que oscila em movimento harmnico simples. (1) (2) (4) (8)

2 B

P R O V A

Nessas condies, correto afirmar: 3 . (1) A funo horria da elongao x = 5 cos t + 4 2 (2) A funo horria da velocidade escalar instantnea v = (4) No instante 2s, a velocidade escalar do mvel nula.
5 2 m/s . (8) No instante 6s, a acelerao escalar do mvel igual a 16 (16) No instante 8s, a energia cintica do mvel nula.
2

5 sen t . 4 4

117

FSICA
Questo 6

UFBA
F A S E

Uma corda de comprimento L e densidade linear encontra-se distendida, com as extremidades fixas, e ressoa, no seu segundo harmnico, com freqncia f. Nessa situao, a corda apresenta: (1) configurao de onda estacionria. (2) oscilao com freqncia fundamental. (4) dois pontos onde a amplitude de vibrao mxima. (8) ondas de comprimento igual a L.

2 B

1 (16) ondas de velocidade igual a L f. 2 2 2 (32) tenso igual a f L .


Questo 7 No circuito eltrico representado ao lado, o capacitor plano de capacitncia C est carregado com carga Q e interligado a dois resistores hmicos, R1 e R2, atravs da chave K, que se encontra inicialmente aberta. Desprezando-se a resistncia eltrica dos fios de ligao e da chave K, correto afirmar: (1) A resistncia equivalente do circuito igual a

P R O V A

R1 R 2 . R1 + R 2

(2) A diferena de potencial, na chave, igual a Q . C (4) No resistor R2, a queda de potencial nula. (8) A energia armazenada no capacitor igual a Q C. (16) No instante em que a chave fechada, a intensidade de corrente que percorre o circuito Q igual a . R1 + R 2 C
2

(32) Com a chave fechada, a intensidade de corrente do circuito permanece constante. (64) Fechando-se a chave, haver converso de energia eltrica em energia trmica, no circuito.

118

FSICA
Questo 8 A figura ao lado representa dois trilhos condutores, de resistncia eltrica desprezvel, separados por uma distncia d, os quais formam um ngulo com a superfcie horizontal e se encontram numa regio onde existe um campo de induo magntica uniforme B . Uma barra condutora, de peso P , desliza sobre esses trilhos, sem atrito, com velocidade V aproximadamente constante. Uma lmpada de resistncia eltrica R ligada aos trilhos.

UFBA
F A S E

2 B

P R O V A

Considerando-se constante a resistncia eltrica da lmpada, correto afirmar: (1) O mdulo da fora magntica que o campo exerce sobre a barra igual ao mdulo do peso da barra. (2) A corrente induzida percorre a barra no sentido MN. (4) A fora eletromotriz induzida no circuito igual a Bvsen. (8) A potncia dissipada na lmpada igual a (Bvdcos ) . R
2

(16) O mdulo da velocidade V igual a

PRtgq B d cos
2 2

QUESTES 9 E 10 INSTRUO: Efetue os clculos necessrios e marque o resultado na Folha de Respostas. Questo 9 Dois blocos, A e B, idnticos, macios e homogneos esto interligados atravs de uma polia e um fio ideais. O bloco A encontra-se submerso em gua, e o bloco B, apoiado numa rampa perfeitamente polida, de inclinao , conforme a figura ao lado. Considerem-se: g o mdulo da acelerao da gravidade local; 3 sen = 0,5; e 1g/cm a massa especfica da gua. Determine, em g/cm , a densidade dos blocos, para que o sistema permanea em equilbrio. Questo 10 Um gs ideal est contido em um recipiente com pisto mvel, presso de 4,0 N/cm2. Quando aquecido, o gs sofre expanso isobrica, variando o volume em 0,5 L. Determine, em joules, a parcela de energia trmica convertida em energia mecnica.
3

119

FSICA

UFMG
F A S E

1. Um barco tenta atravessar um rio com 1,0 km de largura. A correnteza do rio paralela s margens e tem velocidade de 4,0 km/h. A velocidade do barco, em relao gua, de 3,0 km/h perpendicularmente s margens. Nessas condies, pode-se afirmar que o barco a) atravessar o rio em 12 minutos. b) atravessar o rio em 15 minutos. c) atravessar o rio em 20 minutos. d) nunca atravessar o rio. 2. A figura mostra uma bola descendo uma rampa. Ao longo da rampa, esto dispostos cinco cronmetros, C1, C2,... C5, igualmente espaados. Todos os cronmetros so acionados, simultaneamente (t = 0), quando a bola comea a descer a rampa partindo do topo. Cada um dos cronmetros pra quando a bola passa em frente a ele. Desse modo, obtm-se os tempos que a bola gastou para chegar em frente de cada cronmetro.

1 A

P R O V A

A figura que melhor representa as marcaes dos cronmetros em um eixo de tempo

Scan (faltando)

120

FSICA

UFMG
F A S E

3. Uma bola desliza inicialmente sobre um plano inclinado (trecho 1), depois, sobre um plano horizontal (trecho 2) e, finalmente, cai livremente (trecho 3) como mostra a figura.

1 A

P R O V A

Desconsidere as foras de atrito durante todo o movimento. Considere os mdulos das aceleraes da bola nos trechos 1, 2 e 3 como sendo a , a , e a 1 3 3 respectivamente. Sobre os mdulos dessas aceleraes nos trechos do movimento da bola, pode-se afirmar que a) a < a e a c) a = a e a = 0
1 1 2 3 3 2 1 1 2 3 3 2

b) a < a e a = 0

d) a = a e a = 0

4. Uma pessoa entra num elevador carregando uma caixa pendurada por um barbante frgil, como mostra a figura. O elevador sai do 6 andar e s pra no trreo.

correto afirmar que o barbante poder arrebentar a) no momento em que o elevador entrar em movimento, no 6 andar. b) no momento em que o elevador parar no trreo. c) quando o elevador estiver em movimento, entre o 5 e o 2 andares. d) somente numa situao em que o elevador estiver subindo.

121

FSICA

UFMG
F A S E

5. A figura mostra trs vasos V1 V2 e V3 cujas bases tm a mesma rea. Os vasos esto cheios de lquidos l1, l2 e l3 at uma mesma altura. As presses no fundo dos vasos so P1, P2 e P3, respectivamente.

1 A

P R O V A Com relao a essa situao, correto afirmar que a) b) c) d) P1 = P2 = P3 somente se os lquidos ll1, l2 e l3 forem idnticos. P1 = P2 = P3 quaisquer que sejam os lquidos ll1, l2 e l3 P1 > P2 > P3 somente se os lquidos ll1, l2 e l3 forem idnticos. P1 > P2 > P3 quaisquer que sejam os lquidos l1, l2 e l3

26. A figura mostra um brinquedo, comum em parques de diverso, que consiste de uma barra que pode balanar em torno de seu centro. Uma criana de peso P0 senta-se na extremidade da barra a uma distncia X do centro de apoio. Uma segunda criana de peso PN senta-se do lado oposto a uma distncia X/2 do centro.

Para que a barra fique em equilbrio na horizontal, a relao entre os pesos das crianas deve ser a) PN = P0 /2 b) PN = P0 c) PN = 2P0 d) PN = 4P0

122

FSICA

UFMG
F A S E

7. A figura representa um escorregador, onde uma criana escorrega sem impulso inicial. Se ela sair da posio P1, ultrapassa a posio X; se sair de P2, pra em X e, se sair de P3, no chega a X.

1 A

Com relao a esta situao, pode-se afirmar que a energia potencial da criana. a) b) c) d) em P2, igual sua energia potencial em X. em P3, igual sua energia potencial em X. em P3, maior do que em X. em P0, igual soma de suas energias potencial e cintica em X.

P R O V A

8. Um bloco de gelo, dentro de um recipiente de isopor, est temperatura de -10C. Coloca-se um caneco com chope a uma temperatura de 30C sobre o bloco. Aps atingir o equilbrio trmico, a temperatura do chope de 5C. O grfico que melhor representa a temperatura do gelo e posteriormente da gua e a temperatura do chope, em funo do tempo,

123

FSICA

UFMG
F A S E

9. Um mergulhador, na superfcie de um lago onde a presso de 1,0 atm, enche o balo com ar e ento desce a 10 m de profundidade. Ao chegar nessa profundidade, ele mede o volume do balo e v que este foi reduzido menos da metade. Considere que, dentro dgua, uma variao de 10 m na profundidade produz uma variao de 1 atm de presso. Se TS a temperatura na superfcie e Tp a temperatura a 10 m de profundidade, pode-se afirmar que a) TS < Tp b) TS = Tp c) TS > Tp d) no possvel fazer comparao entre as duas temperaturas com os dados fornecidos. 10. Um cozinheiro quer comprar uma panela que esquente rpida e uniformemente. Ele deve procurar uma panela feita de um material que tenha a) b) c) d) alto calor especfico e alta condutividade trmica. alto calor especfico e baixa condutividade trmica. baixo calor especfico e alta condutividade trmica. baixo calor especfico e baixa condutividade trmica.

1 A

P R O V A

11. Um menino caminha pela praia arrastando uma vareta. Uma das pontas da vareta encosta na areia e oscila, no sentido transversal direo do movimento do menino, traando no cho uma curva na forma de uma onda, como mostra a figura.

Uma pessoa observa o menino e percebe que a freqncia de oscilao da ponta da vareta encostada na areia de 1,2 Hz e que a distncia entre dois mximos consecutivos da onda formada na areia de 0,80 m. A pessoa conclui ento que a velocidade do menino a) 0,67 m/s. b) 0,96 m/s. c) 1,5 m/s. d) 0,80 m/s.

124

FSICA

UFMG
F A S E

12. Uma onda sonora de uma determinada freqncia est se propagando dentro de um tubo com gs. A figura representa, em um dado instante, a densidade de molculas do gs dentro do tubo: regio mais escura corresponde a maior densidade.

1 A

P R O V A Se a fonte sonora que emitiu esse som aumentar sua intensidade. a) a densidade do gs na regio M aumenta e a densidade em N diminui. b) a densidade do gs na regio M diminui e a densidade em N aumenta. c) a distncia entre as regies M e N diminui. 13. O princpio bsico de funcionamento de uma fibra ptica consiste em colocar um material X, com ndice de refrao nx, no interior de outro material Y, com ndice de refrao ny. Um feixe de luz que incide em uma extremidade de X atravessa para a outra extremidade, sem penetrar no material Y, devido a mltiplas reflexes totais. Essa situao est ilustrada na figura.

Para que isso acontea, necessrio que a) nx < ny b) nx = 0 c) nx = ny d) nx > ny

125

FSICA

UFMG
F A S E

14. A figura I mostra um objeto situado no ponto M, prximo a uma lente de distncia focal F. A imagem corresponde a esse objeto se forma do ponto N.

1 A

P R O V A O objeto ento retirado do ponto M e colocado no ponto N, conforme mostra a figura II.

As dimenses nas figuras so proporcionais s dimenses reais. A imagem formada nessa nova situao melhor representada por

15. Atrita-se um basto com l de modo que ele adquire carga positiva. Aproxima-se ento o basto de uma esfera metlica com o objeto de induzir nela uma separao de cargas. Essa situao mostrada na figura. Pode-se ento afirmar que o campo eltrico no interior da esfera a) b) c) d) diferente de zero, horizontal, com sentido da direita para a esquerda. diferente de zero, horizontal, com sentido da esquerda para a direita. nulo apenas no centro. nulo em todos os lugares.

126

FSICA

UFMG
F A S E

16. A figura mostra, esquematicamente, as partes principais de uma impressora a jato de tinta.

1 A

Durante o processo de impresso, um campo eltrico aplicado nas placas defletoras de modo a desviar as gotas eletrizadas. Dessa maneira as gotas incidem exatamente no lugar programado da folha de papel onde se formar, por exemplo, parte de uma letra. Considere que as gotas so eletrizadas negativamente. Para que elas atinjam o ponto P da figura, o vetor campo eltrico entre as placas defletoras melhor representado por

P R O V A

17. Uma lmpada fluorescente contm em seu interior um gs que se ioniza aps a aplicao de alta tenso entre seus terminais. Aps a ionizao, uma corrente eltrica estabelecida e os 18 ons negativos deslocam-se vom uma taxa de 1,0 x 10 ons/segundo para o plo A. Os ons positivos se deslocam, com a mesma taxa, para o plo B.

Sabendo-se que a carga de cada on positivo de 1,6 x 10 eltrica na lmpada ser a) 0,16 A b) 0,32 A. c) 1,0 x 10 A.
18

-19

C, pode-se dizer que a corrente

d) nula.

127

FSICA

UFMG
F A S E

18. A figura representa um longo fio conduzido corrente eltrica i . Em um dado instante, duas cargas, uma positiva e outra negativa, esto com velocidade uma de cada lado do fio.

1 A

A configurao que melhor representa as foras do fio sobre cada uma das cargas

P R O V A

19. Um aro metlico com uma certa resistncia eltrica desce um plano inclinado. Em determinado trecho, ele passa por uma regio onde existe um campo magntico, como mostra a figura. Com relao a essa situao, correto afirmar que a) nada se pode dizer sobre a influncia do campo magntico no tempo de queda, sem conhecer a resistncia eltrica do aro. b) o campo magntico no influenciar no tempo de descida do aro. c) o tempo gasto pelo aro, para atingir a base do plano, maior do que o tempo que ele gastaria se o campo magntico no existisse. d) o tempo gasto pelo aro, para atingir a base do plano, menor do que o tempo que ele gastaria se o campo magntico no existisse.

128

FSICA

UFMG
F A S E

20. O diagrama apresenta o espectro eletromagntico com as identificaes de diferentes regies em funo dos respectivos intervalos de comprimentode onda no vcuo.

1 A

Comprimento de onda (m) 107 105 103 ondas de rdio 101 10-1 10-3 10-5 infravermelho 10-7 10-9 10-1 10-11 10-13 10-15

ondas longas

ultra- raios x violeta

raios

P R O V A

luz visvel

correto afirmar que, no vcuo, a) b) c) d) os raios se propagam com maiores velocidades que as ondas de rdio. os raios X tm menor freqncia que as ondas longas. todas as radiaes tm a mesma freqncia. todas as radiaes tm a mesma velocidade de propagao.

1. Na ltima Olmpiada, o vencedor da prova dos 100 m rasos foi o canadense Donovan Bailey e o da maratona (42,2 km) foi o sul-africano Josia Thugwane. Os valores mais prximos para as velocidades mdias desses atletas so, respectivamente, a) 1,0 m/s e 0,5 m/s. b) 10 m/s e 0,5 m/s. c) 10 m/s e 5,0 m/s. d) 50 m/s e 5,0 m/s.

F A S E

1 B

2. A figura mostra trs engrenagens, E1, E2 e E3, fixas pelos seus centros, e de raios, R1, R2 e R3, respectivamente. A relao entre os raios R1 = R3 < R2. A engrenagem da esquerda (E1) gira no sentido horrio com perodo T1. Sendo T2 T3 os perodos de E2 e E3, e respectivamente, pode-se afirmar que as engrenagens vo girar de tal maneira que a) b) c) d) T1 = T2 = T3, com E3 girando em sentido contrrio a E1 T1 = T3 T1, com E3 girando em sentido contrrio a E1 T1 = T1 = T3, com E3 girando no mesmo sentido que E1 T1 = T3 T2, com E3 girando no mesmo sentido que E1

P R O V A

129

FSICA

UFMG
F A S E

3. Uma bola rola sobre a superfcie de uma mesa at cair de sua extremidade com uma certa velocidade. A alternativa que melhor representa a trajetria da bola

1 B

P R O V A

4. Um paraquedista, alguns minutos aps saltar do avio, abre seu paraquedas. As foras que atuam sobre o conjunto paraquedista/equipamentos so, ento, o seu peso e a fora de resistncia do ar. Essa fora proporcional velocidade. Desprezando-se qualquer interferncia de ventos, pode-se afirmar que, a) b) c) d) a partir de um certo momento, o paraquedista descer com velocidade constante. antes de chegar ao cho, o paraquedista poder atingir velocidade nula. durante toda a queda, a fora resultante sobre o conjunto ser vertical para baixo. durante toda a queda, o peso do conjunto menor do que a fora de resistncia do ar.

5. Um sistema hidrulico tem trs mbolos mveis, L, M e N com reas A, 2A e 3A, como mostra a figura. Quantidade diferentes de blocos so colocadas sobre cada mbolo. Todos os blocos tm o mesmo peso. Para que, em equilbrio, os mbolos continuem na mesma altura, o nmero de blocos colocados sobre os mbolos L, M, e N podem ser, respectivamente,

a) 1, 2 e 3

b) 1, 4 e 9

c) 3, 2 e 1.

d) 9, 4 e 1.

130

FSICA

UFMG
F A S E

6. Aps receber um impulso inicial, um foguete se afasta da superfcie da Terra com seus motores desligados. Desconsidere a resistncia do ar. O grfico que melhor representa o mdulo da acelerao do foguete em funo da distncia Terra, aps o desligamento dos motores,

1 B

P R O V A

7. Atira-se uma bola, verticalmente, para cima. A bola sobe e desce, caindo no mesmo ponto de onde foi lanada. Desprezando-se o atrito com o ar, pode-se dizer que a) a energia cintica da bola 1/4 da energia cintica inicial quando ela, na subida, atinge a metade da altura mxima. b) a energia cintica da bola a mesma, tanto na subida quanto na descida, quando ela estiver na metade da altura mxima. c) a energia cintica da bola mxima quando ela atinge o ponto mais alto de sua trajetria. d) a energia potencial da bola mxima no ponto de partida. 8. O coeficiente de dilatao trmica do alumnio (Al) , aproximadamente, duas vezes o coeficiente de dilatao trmica do ferro (Fe). A figura mostra duas peas onde um anel feito de um desses metais envolve um disco feito do outro. A temperatura ambiente, os discos esto presos aos anis. Se as duas peas forem aquecidas uniformemente, correto afirmar que a) apenas o disco de Al se soltar do anel de Fe. b) apenas o disco de Fe se soltar do anel de Al. c) os dois discos se soltaro dos respectivos anis. d) os discos no se soltaro dos anis.

131

FSICA

UFMG
F A S E

9. A figura mostra dois botijes A e B, de volumes VB = 2VA , isolados termicamente. Os dois recipientes contm um mesmo gs ideal e esto em comunicao atravs de um tubo onde existe uma vlvula.

1 B

P R O V A

Na situao inicial, a vlvula est fechada e as temperaturas, as presses e os nmeros de molculas, nos dois recipientes, esto assim relacionados: TA = TB; PA = 2PB; nA = nB. Num certo momento, a vlvula aberta. Depois de atingida a nova situao de equilbrio, tem-se a) b) c) d) TA = TB; TA = TB; TA = TB; TA = TB/2; PA = PB; PA = PB; PA = 2PB; PA = PB; nA = nB nA = nB / 2 nA = nB nA = nB

10. Uma batata recm-cozida, ao ser retirada da gua quente, demora para se esfriar. Uma justificativa possvel para esse fato pode ser dada afirmando-se que a batata tem a) b) c) d) alta condutividade trmica. alto calor especfico. baixa capacidade trmica. baixa quantidade de energia interna.

11. Um menino, balanando em uma corda dependurada em uma rvore, faz 20 oscilaes em um minuto. Pode-se afirmar que seu movimento tem a) b) c) d) um perodo de 3,0 segundos. um perodo de 60 segundos. uma freqncia de 3,0 Hz. uma freqncia de 20 Hz.

132

FSICA

UFMG
F A S E

12. Duas pessoas esticam uma corda, puxando por suas extremidades, e cada uma envia um pulso na direo da outra. Os pulsos tm o mesmo formato, mas esto invertidos como mostra a figura.

1 B

P R O V A Pode-se afirmar que os pulsos a) passaro um pelo outro, cada qual chegando outra extremidade. b) se destruiro, de modo que nenhum deles chegar s extremidades. c) sero refletidos, ao se encontrarem, cada um mantendo-se no mesmo lado em que estava com relao horizontal. d) sero refletidos, ao se encontrarem, porm invertendo seus lados com relao horizontal. 13. Trs peixes, M, N e O, esto em um aqurio com tampa no transparente com um pequeno furo como mostra a figura.

Uma pessoa com o olho na posio mostrada na figura provavelmente ver a) apenas o peixe M. b) apenas o peixe N. O. c) apenas o peixe d) os peixes N e O.

133

FSICA

UFMG
F A S E

14. Um feixe de luz do Sol decomposto ao passar por um prisma de vidro. O feixe de luz visvel resultante composto de ondas com a) apenas sete freqncias que correspondem s cores vermelha, alaranjada, amarela, verde, azul, anil e violeta. b) apenas trs freqncia que correspondem s cores vermelha, amarela e azul. c) apenas trs freqncias que correspondem s cores vermelhas, verde e azul. d) uma infinidade de freqncias que correspondem a cores desde a vermelha at a violeta. 15. Um professor apresenta a figura abaixo aos seus alunos e pede que eles digam o que ela representa. Andra diz que a figura pode representar as linhas de campo eltrico de duas cargas eltricas idnticas; Beatriz diz que a figura pode representar as linhas de campo eltrico de duas cargas eltricas de sinais contrrios; Carlos diz que a figura pode representar as linhas de induo magntica de dois plos magnticos idnticos; Daniel diz que a figura pode representar as linhas de induo magntica de dois plos magnticos contrrios. Os alunos que responderam corretamente so a) Andra e Carlos. b) Andra e Daniel. c) Beatriz e Carlos. d) Beatriz e Daniel.

1 B

P R O V A

16. A figura mostra duas cargas positivas, Q e Qx , de massas desprezveis, colocadas sobre os braos de mesmo comprimento de uma balana nas distncias indicadas. A balana est em uma regio onde existe um campo eltrico uniforme E na direo mostrada.

Para que a balana fique em equilbrio na horizontal, pode-se afirmar que o valor de Qx ser igual a a) Q/3. b) Q. c) 3Q. d) 9Q.

134

FSICA
17. A figura mostra uma parte de um circuito eltrico de um automvel contendo trs lmpadas sendo alimentado pela bateria. As resistncias das lmpadas L1, L2 e L3 so, respectivamente, R1 = 21 R2 = 3 e R3 = 5. Chamando de i1, i2 e i3 as correntes eltricas nas lmpadas L1, L2, e L , respectivamente, 3 correto afirmar que a) i1 = i2 = i3 b) i1 = i2 i3 c) i1 > i2 > i3 d) i2 > i1 > i2

UFMG
F A S E

1 B

P R O V A

18. Um eltron entra com uma velocidade v em uma regio onde existem um campo eltrico E e um campo magntico B uniformes e perpendiculares entre si, como mostra a figura. A velocidade v perpendicular aos dois campos.

O eltron no sofre nenhum desvio ao cruzar a regio dos campos. As foras eltricas Fe, e a magntica, Fm, que atuam sobre o eltron, nessa situao, so melhor representadas por

135

FSICA
19. A figura mostra um im e um aro circular. O eixo do im (eixo x) perpendicular ao plano do aro, (plano yz) e passa pelo seu centro. NO aparecer corrente na espira se ela a) b) c) d) deslocar-se ao longo do eixo x. deslocar-se ao longo do eixo y. girar em torno do eixo x. girar em torno do eixo y.

UFMG
F A S E

1 B 2 C

P R O V A

20. As ondas eletromagnticas, ao contrrio das ondas mecnicas, no precisam de um meio material para se propagar. Considere as seguintes ondas: som, ultra-som, ondas de rdio, microondas e luz. Sobre essas ondas correto afirmar que a) b) c) d) luz e microondas so ondas eletromagnticas e as outras so ondas mecnicas. luz onda eletromagntica e as outras so ondas mecnicas. som onda mecnica e as outras so ondas eletromagnticas. som e ultra-som so ondas mecnicas e as outras so ondas eletromagnticas.

QUESTO 01 Uma balsa de madeira macia flutua com 3/4 de seu volume fora dgua. O volume da balsa 3 3 2,0 m e a densidade da gua 1,0 g/cm .

F A S E

1. a) DESENHE na figura as foras que atuam sobre a balsa e IDENTIFIQUE o agente causador correspondente a cada uma dessas foras. b) DETERMINE a densidade da balsa de madeira.

P R O V A

136

FSICA

UFMG
F A S E

2. Estime a temperatura de ebulio da substncia, quando ela se encontra presso constante de 0,6 atm. EXPLIQUE o raciocnio utilizado.

3. RESPONDA se essa substncia pode ser sublimada presso atmosfrica normal. JUSTIFIQUE sua resposta com base nos dados apresentados no grfico.

4. CONCEITUE ponto triplo e ESTIME-O para essa substncia.

137

FSICA
QUESTO 02 A figura mostra o diagrama de fase de uma substncia hipottica. Observando o grfico, responda s questes que se seguem. 1. ASSOCIE as regies I, II e II com as fases slida, lquida e gasosa dessa substncia. JUSTIFIQUE sua resposta.

UFMG
F A S E

QUESTO 03 Um automvel de 1,0 tonelada colidiu frontalmente com um caminho de 9,0 toneladas. A velocidade do automvel era de 80 km/h para a direita e a do caminho, de 40 km/h para a esquerda. Aps a coliso, os dois veculos permaneceram juntos. 1. DETERMINE a velocidade do conjunto caminho e automvel logo aps a coliso.

2. RESPONDA se, em mdulo, a fora devido coliso que atuou sobre o automvel maior, menor ou igual quela que atuou sobre o caminho. JUSTIFIQUE sua resposta.

138

FSICA

UFMG
F A S E

3. Areia colocada sobre a balsa at que essa flutue com sua parte superior coincidindo com o nvel da gua, como mostra a figura.

CALCULE a massa da areia colocada sobre a balsa.

QUESTO 04 O pndulo de um relgio constitudo de uma massa suspensa por uma haste fina de ao, de comprimento l = 1,6 m. O pndulo ligado aos ponteiros do relgio por um sistema de engrenagens. Esse sistema funciona de tal modo que, toda vez que o pndulo completa um determinado nmero de oscilaes, o relgio marca um minuto a mais. O perodo do pndulo dado pela expresso 2 T = 2pl / g. Considere p = 3 e g = 10 m/s para os clculos que se fizerem necessrios. 1. CALCULE o nmero de oscilaes do pndulo no intervalo de tempo de um minuto.

2. RESPONDA se o relgio vai adiantar ou atrasar caso sua temperatura diminua. JUSTIFIQUE sua resposta.

139

FSICA
QUESTO 05

UFMG
F A S E

Durante o dia, uma pessoa encontra-se numa sala em frente a uma janela de vidro plano transparente e, atravs dela, v a paisagem do lado de fora. noite, a pessoa estando no mesmo local com a luz da sala acesa, olha para a janela, v sua imagem refletida, mas no v mais a paisagem. Usando conceitos de tica, EXPLIQUE esse fato.

QUESTO 06 Duas esferas metlicas de dimetros diferentes, apoiadas em bases isolantes, esto inicialmente em contato. Aproxima-se delas, sem toc-las, um basto carregado positivamente, como mostra a figura. Com o basto ainda prximo das esferas, a esfera B afastada da esfera A. Considerando a situao final, responda s questes abaixo. 1. CITE os sinais das cargas que as esferas A e B iro adquirir. JUSTIFIQUE sua resposta.

2. COMPARE o mdulo das cargas das esferas. JUSTIFIQUE sua resposta.

140

FSICA
QUESTO 07

UFMG
F A S E

Uma pessoa gira uma espira metlica, com velocidade angular constante, na presena de um campo magntico, como mostra a figura. A espira tem resistncia eltrica R e seu movimento sem atrito.

1. EXPLIQUE por que, nessa situao, aparece uma corrente eltrica na espira.

2. Em um determinado momento, a pessoa pra de atuar sobre a espira. RESPONDA se, aps esse momento, a velocidade angular da espira aumenta, diminui ou permanece constante. JUSTIFIQUE sua resposta.

141

FSICA
QUESTO 08

UFMG
F A S E

A figura mostra, esquematicamente, os nveis de energia permitidos para eltrons de um certo elemento qumico. Quando esse elemento emite radiao, so observados trs comprimentos de onda diferentes, la, lb e lc. >

ENERGIA

E3 E2

E1

1. Com base na figura, EXPLIQUE a origem da radiao correspondente aos comprimentos de onda la, lb e lc.

2. Considere que la < lb < lc. Sendo h a constante de Planck e c a velocidade da luz, DETERMINE uma expresso para o comprimento de onda la.

142

FSICA

UFPA
F A S E

1. As ordens de grandezas do peso em dina e da altura em centmetro de um jogador da seleo 2 brasileira de voleibol (supondo a acelerao da gravidade igual a 10 m/s ) so respectivamente: a) 10 e 10 3 2 b) 10 e 10 4 1 c) 10 e 10
2 2

d) 10 e 10 8 2 e) 10 e 10

2. Um rapaz e uma moa saem de suas casas um ao encontro do outro, caminhando sempre com velocidade respectivamente de 3,5 km/h e 2,5 km/h. Estando a 100 m da moa, em linha reta, o rapaz, ao avist-la, aciona o seu cronmetro, travando-o apenas no instante em que os dois se encontram. O intervalo de tempo, em minuto, registrado pelo cronmetro vale: a) 1 b) 6 c) 9 d) 10 e) 12

3. A figura abaixo representa um homem empurrando um automvel em uma estrada horizontal e asfaltada com uma fora tambm horizontal, cujo mdulo vale F. Suponha que: F - mdulo da fora com que o automvel empurra o homem. F1 - mdulo da fora de atrito entre os pneus e a estrada. F2 - mdulo da fora de atrito entre os ps do homem e a estrada.

ento o automvel est(r) a) parado porque F = F b) em movimento quando F > F c) em movimento quando F > F1 d) parado porque F1 = F2 e) em movimento quando F2 < F

4. Ao mergulhar-se, em gua quente, um termmetro comum de haste de vidro com mercrio, pode acontecer de a coluna de mercrio sofrer uma pequena descida antes de principiar a subir. Isto pode ser explicado porque o(a) a) coeficiente de dilatao do vidro maior que o coeficiente de dilatao do mercrio. b) vidro que envolve o mercrio se aquece um pouco antes de a temperatura do mercrio comear a elevar-se. c) volume do vidro maior do que o volume do mercrio. d) mercrio, ao dilatar-se, o faz de maneira anmala, semelhante a da gua. e) capacidade trmica do mercrio maior que a capacidade trmica do vidro.

143

FSICA

UFPA
F A S E

5. As unidades que nos permitem avaliar, respectivamente, a altura, a intensidade e o nvel de intensidade de uma propagao sonora so o a) hertz, o watt/m e o decibel 2 b) metro, o hertz e o watt/m c) metro, o newton e o decibel
2

d) hertz, o newton e o decibel 2 e) hertz, o decibel e o watt/m

6. A figura abaixo representa o esquema de um periscpio rudimentar que utiliza dois espelhos planos E1 e E2 de tal forma que o observador consegue ver, por reflexo, a imagem do objeto real O. Esta imagem

a) real e invertida b) real e ampliada c) real e reduzida

d) virtual e direita e) virtual e invertida

7. No trecho de circuito da figura abaixo as duas lmpadas possuem resistncias iguais a 50 W cada uma. Se a leitura indicada pelo ampermetro 100 mA, ento, em volt, a leitura indicada pelo voltmetro

a) 10,0

b) 7,5

c) 5,0

d) 2,5

e) 1,0

144

FSICA
8. Na figura abaixo, um im natural, cujos plos magnticos norte N e sul S esto representados, equilibra dois pregos 1 e 2. Os pontos A e B pertencem a 1 e os pontos C e D pertencem a 2. Nesta situao. a) b) c) d) e) B e C so plos norte A um plo norte e D um plo sul A e D so plos sul A um plo sul e B um plo norte B um plo sul e D um plo norte

UFPA
F A S E

1 2 A

1. Uma esfera metlica macia de massa 0,5 kg est em repouso sobre um plano horizontal liso quando ento golpeada horizontalmente por um martelo de massa 2kg e velocidade 20 m/s. Aps o choque o martelo fica em repouso e a esfera passa a se deslocar com velocidade constante de 10 m/s. Admitindo que a esfera absorva 60% do calor desenvolvido no choque, que o calor especfico da substncia da esfera seja 0,03 cal/g . C e que 1 cal = 4,18J, pede-se: a) b) c) d) A variao da energia cintica, em joule, do martelo A variao da energia cintica, em joule, da esfera A quantidade total de calor, em joule, produzida no choque A variao de temperatura, em C, sofrida pela esfera

F A S E

2. A figura abaixo representa um grande reservatrio de gua de uma represa, com uma canalizao 2 nele acoplada, cujas reas das seces so 900 cm em 1 e 600 cm2 em 2. Admita que a gua possa ser considerada um fluido ideal e que escoe em regime permanente. Sabendo-se que a 5 2 acelerao da gravidade vale 10 m/s2 e que a presso atmosfrica igual a 10 N/m , pede-se:

P R O V A

a) A velocidade, em m/s, com que a gua flui no ponto 2 3 b) A vazo, em m /s, da gua 2 c) A presso, em N/m , no ponto 1

145

FSICA

UFPA
F A S E

3. Suponha que uma cerca mquina trmica possa operar em ciclo de Carnot. Ela possui uma caldeira, que a fonte quente, e um condensador, que a fonte fria. A temperatura da caldeira mantida constantemente em 177C enquanto o condensador constitudo de gelo fundente 3 a 0C. Verifica-se que 10 Kg de gelo so fundidos aps a mquina ter operado durante 10 min. O calor latente de fuso do gelo vale 80 cal/g. Pede-se: a) O esboo de um ciclo de Carnot num diagrama temperatura (ordenadas) versus volume (abscissas) nominando as transformaes ocorridas, para um gs ideal. b) A quantidade de calor, em caloria, rejeitada pela mquina durante os 10 min. c) A potncia em Kcal/min da mquina. 4. O canal auditivo de um ouvido humano funciona como um tubo sonoro aberto numa extremidade e fechado na outra, com um comprimento da ordem de 2,5 cm conforme figura abaixo, considere a intensidade sonora de referncia -12 2 igual a 10 W/m . a) Admitindo que o som se propague com velocidade de 340 m/s, qual , em Hz, a freqncia natural fundamental do canal auditivo? b) Sabendo que sons acima de 160 dB podem romper o tmpano, qual a mxima intensidade sonora suportada pelo tmpano? 2 c) Se uma onda sonora de 70 dB atinge um tmpano cuja rea da ordem de 0,5 cm , durante 1 minuto, qual a energia, em joule, que ele absorve? 5. A figura abaixo representa uma lente convergente de distncia focal igual a 16 cm e um objeto distante 80 cm da mesma. Pede-se:

2 A

P R O V A

a) b) c) d)

Construir graficamente, sem escala, a imagem do objeto. Determinar a distncia, em cm, da imagem ao objeto. A ampliao linear da imagem. A nova posio da imagem se uma outra lente de distncia focal igual a 40cm fosse justaposta primeira lente.

146

FSICA

UFPA
F A S E

6. Uma lmpada de resistncia igual a 117 ligada em srie a um motor de fora contraeletromotriz igual a 60 V e resistncia interna igual a 1, sendo ambos ligados tambm em srie a um gerador de fora eletromotriz igual a 120 V e resistncia interna igual a 2. Com o circuito em funcionamento, pergunta-se: a) Qual o valor, em ampre, da corrente circulante? b) Se bloquearmos mecanicamente o eixo do motor, impedindo o seu giro, o brilho da lmpada aumenta, diminui, ou no se altera? c) Na situao ainda do tem b), qual o valor, em ampre, da corrente circulante? 7. Na figura abaixo, um trilho metlico, em forma de U, de resistncia eltrica desprezvel, inclinado de 30 em relao horizontal, est imerso em uma regio onde existe um campo -2 magntico de induo B uniforme, normal ao plano do trilho, cujo mdulo vale 10 T. Uma barra condutora de comprimento igual a 1 m e resistncia 0,5 movimentada para baixo com velocidade constante de 2 m/s como mostrado. Ento, desprezando os atritos:

2 A

P R O V A

a) b) c) d) e)

Qual o sentido da corrente eltrica induzida na barra? Qual a fora eletromotriz, em volt, induzida na barra? Qual o mdulo da fora magntica, em newton, que atua na barra? Aps a barra descer 1 m, na vertical, qual a energia dissipada em joule? Se o trilho fosse de material isolante, apareceria fora eletromotriz na barra? em caso afirmativo, qual o seu valor?

8. Suponha que se pudssemos construir uma nave espacial capaz de deslocar-se sempre com velocidade de 0,6c (c = velocidade da luz no vcuo) em uma viagem de ida e volta a uma regio do universo distante 15 anos-luz da Terra. Tendo em vista a Teoria da Relatividade Restrita e os dados acima responda: a) b) c) d) Quais os dois postulados nos quais se baseia esta teoria? Qual a durao, em ano, desta viagem, para um observador na Terra? E para um observador situado na nave, qual seria, em ano, a durao da viagem? Ainda para um observador na nave, qual a distncia, em ano-luz, que ele mediria, do ponto de retorno da nave at a Terra?

147

FSICA

UFPA
F A S E

1. A figura abaixo mostra um bloco de 1 kg sendo arrastado ao longo de um plano horizontal spero, por uma fora constante de 10 N, inclinada de 60 em relao a horizontal. Considerando um deslocamento de 10 m, a partir do repouso e que a sua velocidade seja 5 m/s ao fim desse 2 percurso, e supondo que a acelerao da gravidade seja 10 m/s , determinar:

2 B

P R O V A

a) b) c) d)

O trabalho mecnico, em joule, realizado pela fora F. A variao da energia cintica, em joule, do bloco. O trabalho mecnico, em joule, da fora de atrito. A intensidade, em newton, da fora de atrito.

2. A figura abaixo mostra uma mquina hidrulica, tipo prensa, acionada por uma alavanca rgida interfixa, usada para elevao de cargas, representada pelo corpo C. Admitindo desprezveis quaisquer atritos, bem como as massas dos mbolos, cujas seces tm reas 2 S1 = 100 cm2 e S2 = 600 cm e sendo OA = 100 cm e OB = 50 cm, pede-se calcular:

a) O peso, em newton, mximo do corpo C, que poder ser elevado, se usarmos uma fora F de 10 N aplicada em A. 2 b) A presso, em N /m , sobre qualquer dos mbolos, quando a fora aplicada em A for igual a 20 N. c) O deslocamento do mbolo menor, em cm, sabendo que o mbolo maior sofreu um deslocamento de 10 cm na vertical.

148

FSICA

UFPA
F A S E

11. Um estudante, usando seus conhecimentos sobre conservao de energia, deseja aquecer gua em um recipiente sacudindo-o vrias vezes. Para isto, ele utiliza uma garrafa trmica, na qual introduz 500 g de gua a uma temperatura de 20C, fechando-a logo em seguida. Admita que em cada sacudida da garrafa, o centro de massa da poro de gua sofra uma queda de 20 cm e que no h perda de calor para o exterior. 2 O calor especfico da gua vale 1 cal/g . C e a acelerao da gravidade 10 m/s . Para que a gua se aquea at 30C, pergunta-se: a) Qual a quantidade de calor, em joule, necessria para isto? b) Qual o nmero de sacudidas que o estudante deve realizar? c) Supondo que ele realize 40 sacudidas por minuto, a uma taxa constante, qual o tempo, em minuto, gasto neste processo? 12. Na figura abaixo um automvel cuja sirene emite um som de freqncia 500 Hz desloca-se afastando-se de um observador em repouso O e aproximando-se de uma parede refletora de som P. Supondo que a velocidade do automvel seja 30 m/s e a do som no ar 340 m/s, ento:

2 B

P R O V A

a) Quanto vale a freqncia, em Hz, do som percebido pelo observador? b) Quanto vale a freqncia, em Hz, do som refletido pela parede? c) Admitindo agora que o intervalo entre o som refletido pela parede e o som da sirene seja igual a uma oitava, qual a nova velocidade, em m/s, que deve ter o automvel? 13. Nas figuras abaixo representa-se de forma simplificada um olho humano normal.

a) Em (1), raios paralelos provenientes de um objeto no infinito, produzem uma imagem na retina, que se situa no fundo do olho a uma distncia de 2 cm. Qual a distncia focal, em cm, do olho nesta situao? b) Em (2), se o objeto trazido at 40 cm do olho, para que a imagem se forme sobre a retina, qual o novo valor, em cm, da distncia focal? c) Supondo agora um olho hipermtrope, a imagem de um objeto se formar antes ou depois da retina? Qual o tipo de lente usada para corrigir este defeito? Qual a vergncia desta lente, em dioptrias para que o hipermtrope, cujo ponto prximo est a 1 m, possa ler de forma ntida a uma distncia de 25 cm, considerando desprezvel a distncia da lente ao olho?

149

FSICA
14. No circuito mostrado na figura ao lado, tem-se a fora eletromotriz E igual a 120 V, os resistores R1 = 40, R2 = 5, R3 = 20 e o capacitor C = 10F. Considerando o capacitor carregado, calcule a: a) b) c) d) Corrente, em ampre, atravs de R1 Corrente, em ampre, atravs de R2 Potncia, em watt, dissipada em R3 Carga, em coulomb, em uma das placas do capacitor

UFPA
F A S E

2 B

15. A figura ao lado mostra a vista frontal de uma caixa, no interior da qual h um campo magntico uniforme, de induo B, orientado perpendicularmente parede do fundo da caixa penetrando na mesma. Atravs de um orifcio lateral, um eltron projetado para o interior 4 da caixa com uma velocidade de 3,14 x 10 m/s, perpendicularmente ao campo B, de modo a sair por outro orifcio lateral, perpendicularmente parede da caixa, numa posio definida pela distncia x. A posio h, que define a entrada do eltron na caixa -19 vale 3,14 cm. Considere a carga do eltron 1,6 10 -31 C e a massa 9,1 10 kg. Com base nesses dados pede-se: a) O valor de x em centmetro. b) O mdulo da induo magntica B em tesla. c) O tempo, em segundo, do eltron no interior da caixa, decorrido desde sua entrada at a sada.

P R O V A

150

FSICA
Dados: acelerao da gravidade: 10 m/s2 sen 45 cos 45
2 2

UFPE
F A S E

1. Qual a ordem de grandeza da velocidade angular, em rotaes / min, do movimento de rotao da Terra em torno do seu eixo? a) 10
1

b) 10

c) 10

d) 10

e) 10

2. No instante t 0, dois automveis, A e B, partem do repouso seguindo no mesmo sentido ao longo de uma estrada retilnea. O diagrama abaixo representa a variao com o tempo da posio de cada um desses automveis. Sabendo-se que o automvel B manteve uma acelerao constante durante o movimento, determine a razo VA / VB entre as velocidades dos dois veculos no instante t 5 s.

a) 3

b) 2

c) 1

d) 1/2

e) 1/3

3. Dois blocos A e B de massas respectivamente iguais a 5kg e 10kg esto inicialmente em repouso, encostados um no outro, sobre uma mesa horizontal sem atrito. Aplicamos uma fora horizontal F 90N, como mostra a figura. Os valores, em N, das foras resultantes que atuam sobre os blocos A e B so, respectivamente:

a) 40 e 50 b) 45 e 45 c) 90 e 90

d) 20 e 70 e) 30 e 60

151

FSICA

UFPE
F A S E

4. Num certo trecho de uma montanha russa os pontos A e B tm o mesmo raio de curvatura. Podemos afirmar que os ocupantes de um carrinho, ao passarem por estes pontos, tm a sensao de:

a) b) c) d) e)

diminuio de peso em A e aumento de peso em B. aumento de peso em A e diminuio de peso em B. diminuio de peso em A e B. aumento de peso em A e em B. nada se pode afirmar, pois no se conhece as velocidades do carrinho em A e em B.

5. Numa coliso unidimensional entre duas esferas de mesma massa, inicialmente uma das esferas est em repouso e a outra se move com velocidade V. Podemos afirmar que o centro de massa do sistema se move, antes e depois do choque: a) b) c) d) e) com velocidade V e V/2, respectivamente. com velocidade 2V e V, respectivamente. com a mesma velocidade 2V. com a mesma velocidade V. com a mesma velocidade V/2.

6. Um atleta salta por cima do obstculo indicado na figura e seu centro de gravidade atinge a altura de 2,2m. Atrs do obstculo existe um colcho de ar, com 40cm de altura, para atenuar a queda do atleta, que cai deitado. Qual a velocidade, em m/s, com que o atleta atingir a superfcie do colcho? (Despreze a resistncia do ar).

a) 1,0

b) 3,0

c) 6,0

d) 8,5

e) 9,0

152

FSICA

UFPE
F A S E

7. Uma pedra, de massa igual a 0,2 kg, lanada verticalmente para cima e atinge uma altura mxima de 10m. Desprezando a resistncia do ar, qual dos grficos abaixo representa a energia cintica da pedra em funo de sua altura h, durante a subida?

8. O preo do quilowatt-hora (kWh) cobrado na conta mensal da CELPE R$ 0,15. Quanto custa um banho de 12min com um chuveiro de 3kW? a) R$ 0,25 b) R$ 0,15 c) R$ 0,12 d) R$ 0,09 e) R$ 0,06

9. So misturadas massas iguais de dois lquidos homogneos, de densidades dA dB 1,0 g/cm3. Qual a densidade da mistura, em g/cm3? a) 1,2 b) 1,5 c) 1,8 d) 2,0 e) 2,2

3,0 g/cm3 e

10. Qual a maior altura em centmetros que pode ter uma coluna cilndrica feita de um metal de densidade igual a 3,0 103 kg/m3 para que possa ser colocada em p com segurana sobre um piso que resiste a uma presso mxima de 9,0 104 N/m2? a) 300 b) 270 c) 60 d) 30 e) 27

153

FSICA

UFPE
F A S E

11. Ao flutuar na superfcie de um lago, um pequeno barril desloca 10 l de gua. Para flutuar sobre um lquido duas vezes mais denso que a gua, um barril idntico dever deslocar quantos litros desse lquido? a) 25 b) 20 c) 15 d) 10 e) 5

12. Uma rgua de alumnio, com coeficiente de dilatao linear 25 10 6 K 1, tem o comprimento de 200,0 cm a 20C. Qual o valor, em cm, do seu comprimento a 60C? a) 200,1 b) 200,2 c) 200,3 d) 200,4 e) 200,5

13. Uma bebida refrescante pode ser obtida pela mistura de ch quente com gua gelada. Qual a temperatura final (em C) de uma mistura preparada a partir de 100g de ch a 80C com 400g de gua a 5C? Considere o calor especfico do ch igual ao da gua (1,0 cal/g. C). a) 20 b) 18 c) 16 d) 14 e) 12

14. A figura mostra o caminho de um raio de luz atravessando trs lquidos no miscveis, transparentes e superpostos. Examinando a trajetria da luz nos trs lquidos, podemos afirmar que sua velocidade: a) b) c) d) e) a mesma nos trs lquidos. maior no lquido I do que no lquido II. menor no lquido I do que no lquido II. a mesma nos lquidos I e III. maior no lquido II do que no lquido III.

15. Um prisma imerso no ar deve ser usado para mudar a direo do feixe de luz incidente por 90, de modo que a luz no transmitida atravs da superfcie BC. Dentre os valores abaixo, qual o menor valor admissvel para o ndice de refrao do prisma?

a) 1,27

b) 1,30

c) 1,45

d) 1,73

e) 1,82

154

FSICA

UFPE
F A S E

16. No circuito abaixo nula a corrente no fio de resistncia R. Qual o valor, em , da resistncia X?

a) 3

b) 4

c) 5

d) 6

e) 7

NOS PROBLEMAS ABAIXO CONSIDERE: Acelerao de Gravidade: 10 m/s2 Massa Especfica da gua; 103 kg/m3 Presso de 760 mm Hg 1,0 105 N/m2 Carga de Eltron: 1,6 10 19 C

F A S E

2 A

cos sen

0 1

30
3

45

60 1/2

90 0 P R O V A

2 2 2 2

1/2

3 2

1. O valor tpico do consumo de oxignio de um atleta jogando basquetebol 2,4 litros/min. Ao ser usado pelo metabolismo o oxignio libera cerca de 20000 J/litro. Determine, em unidades de 10 W. o valor da potncia exigida do atleta ao praticar esse esporte. 2. Um jogador de futebol est conduzindo a bola correndo com uma velocidade de 6 m/s. Sua trajetria faz um ngulo de 60 com as linhas laterais do campo. Qual o valor em m/s da velocidade com que ele est se aproximando da linha de fundo?

155

FSICA

UFPE
F A S E

3. No grfico abaixo est representada a variao com o tempo da posio horizontal x (t) e vertical y (t) de uma asa delta em pleno vo.

2 A

P R O V A

Se a velocidade da asa delta, em unidades de m/s, vale v, qual o valor de seu quadrado, v2?

4. Dois bocais de mangueiras de jardim, A e B, esto fixos ao solo. O bocal A perpendicular ao solo e o outro est inclinado de 60 em relao direo de A. Correntes de gua jorram dos dois bocais com velocidades idnticas. Qual a razo entre as alturas mximas de elevao da gua?

5. Na cobrana de um pnalti o jogador consegue imprimir bola de 400 g a velocidade de 72 km/h. Qual foi a fora mdia, em Newtons, aplicada bola se o tempo de contato entre ela e o p do jogador foi de 0,1 s?

6. Um bloco de 6,0 kg sobe o plano inclinado da figura, sob a ao de uma fora externa paralela ao plano. O coeficiente de atrito entre o plano e o bloco
3 . Qual ser o valor da 2 fora externa F, em Newtons, para que o bloco esteja em movimento uniforme?

156

FSICA

UFPE
F A S E

7. O desempenho de um pequeno motor durante 6,0 horas de operao pode ser representado pelo grfico abaixo. Calcule o trabalho total, em joules, efetuado pelo motor nas trs ltimas horas de operao.
P(10
3

Watts)

2 A

8. Um bloco de massa m 100 g desliza sem atrito ao longo do trecho AOB de um hemisfrio circular. De que altura HA, em centmetros, ele deve ser solto para que sua velocidade no ponto O seja igual a 4,0 m/s?

P R O V A

9. Qual a potncia mdia, em Watts, que dever ter um conjunto motor-bomba para elevar 1000 litros de gua at a altura de 6,0 m em 20 min?

10. Se o fluxo sangneo no fosse ajustado pela expanso de artrias, para uma pessoa em p a diferena de presso arterial entre o corao e a cabea seria de natureza puramente hidrosttica. Nesse caso, para uma pessoa em que a distncia entre a cabea e o corao vale 50 cm, qual o valor em mmHg dessa diferena de presso? (Considere a densidade do sangue igual a 103 kg/m3).

11. A figura abaixo ilustra o diagrama p V de um gs ideal em uma transformao cclica. Qual a quantidade de calor, em Joules, absorvida pelo gs durante o ciclo?

157

FSICA

UFPE
F A S E

12. Uma corda esticada, presa por suas extremidades, entra em ressonncia na sua freqncia mais baixa, igual a 200 Hz. Qual o comprimento da corda, em centmetros, se a velocidade das ondas transversais na corda de 300 m/s?

2 A

13. Um aqurio feito de um vidro com 24 mm de espessura e ndice de refrao igual a 3/2 est cheio de gua (ndice de refrao 4/3) at uma profundidade de 50 cm. Qual a espessura aparente (em milmetros) da parede do fundo do aqurio quando vista verticalmente atravs do ar por um observador em O?

P R O V A

14. Atravs da seo reta de um fio de cobre passam 2,0 valor da corrente, em Ampres, no fio?

1020 eltrons por segundo. Qual o

15. Qual a diferena de potencial, em Volts, entre os pontos A e B do circuito abaixo?

16. Partculas carregadas, com velocidade igual a 6,0 105 m/s e com uma razo de carga sobre massa q/m 108 C/kg, entram em um campo magntico de 0,3 T, perpendicular direo de seus movimentos. Qual o raio de suas trajetrias, em milmetros?

158

FSICA
NOS PROBLEMAS ABAIXO CONSIDERE: Acelerao de Gravidade: 10 m/s2 Massa Especfica da gua; 103 kg/m3 Presso de 760 mm Hg 1,0 105 N/m2 Carga de Eltron: 1,6 10 19 C

UFPE
F A S E

2 B

cos sen

0 1 0

30 0,87 0,50

45 0,71 0,71

60 0,50 0,87

90 0 1

P R O V A

1. Um corao humano bate em mdia 120.000 vezes por dia. Determine, em unidades de 108, o nmero de vezes que, desde o nascimento, j bateu o corao dessa pessoa ao completar 50 anos. (Despreze a diferena no nmero de dias nos anos bissextos).

2. Para corrigir o desalinhamento do dente incisivo A de um paciente, um dentista fez passar um elstico por esse dente e o amarrou a dois dentes posteriores, conforme a figura ao lado. Sabendo-se que a tenso no elstico de 10 N e que cos 0,85, determine o valor em Newtons da fora total aplicada pelo elstico sobre o dente A.

3. Um gafanhoto adulto pode saltar at 0,80 m com um ngulo de lanamento de 45. Desprezando a resistncia do ar e a fora de sustentao aerodinmica sobre o gafanhoto, calcule quantos dcimos de segundo ele permanecer em vo.

4. A figura abaixo representa duas pessoas, A e B, situadas ao longo de uma linha frrea retilnea. Quando A golpeia o trilho, B percebe o barulho atravs do ar 10 s aps o impacto. Quantos centsimos de segundo aps a batida de A, poderia o observador B ter sentido a vibrao atravs do trilho, se a velocidade do som atravs do trilho de 6800 m/s? (Sabe-se que a velocidade do som atravs do ar vale 340 m/s).

Scan

159

FSICA

UFPE
F A S E

5. Um automvel deve contornar uma praa circular seguindo uma trajetria com raio de 100 m. Supondo que a rodovia horizontal e que o coeficiente de atrito cintico entre os pneus e a estrada 0,4, qual a velocidade mxima, em km/h, que o carro pode atingir para contornar a praa sem derrapar?

2 B

6. Um balano de comprimento L 1,6 m solto da horizontal (ponto A da figura), e na ausncia de resistncia do ar adquire um movimento ao longo do semi-crculo de raio L. Qual a velocidade, em m/s, do balano ao passar pelo ponto B de sua trajetria?

P R O V A

7. A figura abaixo representa a variao da fora aplicada a um corpo de 6 kg que se move sem atrito sobre um plano horizontal. Qual a velocidade, em m/s, do corpo no ponto x 3 m se em x 0 a sua velocidade era 2 m/s?

8. A rea da seo transversal de uma seringa 1,52 cm2 e a da agulha 0,5 mm2. Se a presso sangnea do paciente 10 mmHg, determine o valor mnimo, em unidades de 10 2 N, da fora F que deve ser aplicada sobre o mbolo da seringa para que seu contedo possa ser injetado no paciente.

160

FSICA
9. Um balo de vidro, de volume V 1 litro e contendo hlio a uma presso de 1 atm, ligado por um tubo fino a um outro balo idntico que contm o mesmo gs a 5 atm e mesma temperatura. Determine o valor em atmosferas da presso em cada um dos bales alguns minutos aps a vlvula de conexo S ter sido aberta, se a temperatura for mantida constante durante todo o processo.

UFPE
F A S E

2 B

10. Um objeto O colocado 30 cm esquerda de uma lente convergente, cuja distncia focal de 10 cm. A que distncia, em cm, da lente ser formada a imagem?

P R O V A

11. Uma camada fina de lquido com ndice de refrao igual a 1,39 reveste a superfcie horizontal de um semicilindro de vidro, cujo ndice de refrao de 1,60. Qual o ngulo crtico de reflexo total interna, em graus, para incidncia de luz, conforme indicado na figura?

12. O circuito abaixo alimentado por uma bateria de resistncia interna desprezvel e fora eletromotriz igual a 12 V. Qual a diferena de potencial, em Volts, entre os pontos A e B?

161

FSICA
13. Uma lmpada especial tem uma curva de corrente versus diferena de potencial conforme indicado na figura. Qual a potncia que ser dissipada, em Watts, na lmpada quando ela estiver submetida diferena de potencial de 10 V?
i(A)

UFPE
F A S E

2 B 2 C

14. Partculas de massa m 1,6 10 26 kg e carga q 1,6 10 19 C, aps serem aceleradas desde o repouso por uma diferena de potencial de 2000 V, entram em um campo magntico igual a 0,5 T, perpendicular direo de seus movimentos. Qual o raio de suas trajetrias em milmetros? 15. Um mol de um gs ideal monoatmico, com valor especfico a volume constante igual a 3,0 cal/(molC), realiza uma transformao a volume constante, enquanto sua temperatura se eleva de 27C a 50C. Qual a variao da energia interna do gs, em calorias? 16. Cerca de 107 ons de Na penetram em uma clula a cada milissegundo, atravessando uma membrana de seo reta igual a 8 10 10 m2. Determine o valor, em A/m2, da densidade da corrente eltrica atravs da membrana.

P R O V A

NOS PROBLEMAS ABAIXO CONSIDERE: Acelerao de Gravidade: 10 m/s2 Massa Especfica da gua; 103 kg/m3 Calor latente de fuso do gelo 80 cal/g Calor especfico da gua 1 cal/gC

F A S E

cos sen

0 1 0

30 0,87 0,50

45 0,71 0,71

60 0,50 0,87

90 0 1

P R O V A

1. O corpo de massa M 6,85 kg est suspenso por uma corda inextensvel ABC que se apia na barra inclinada BD. Calcule a fora, em Newtons, que atua ao longo da barra BD.

162

FSICA

UFPE
F A S E

2. Numa competio de salto em distncia, um atleta de 60 kg consegue atingir a distncia de 9,0 m. Desprezando a resistncia do ar e supondo que o salto foi feito com um ngulo de inclinao de 45, calcule a energia cintica do atleta, em unidades de 100 Joules, ao iniciar o salto.

2 C

3. A parte mais externa de um disco, com 0,25 m de raio, gira com uma velocidade linear de 15 m/s. O disco comea ento a desacelerar uniformemente at parar, em um tempo de 0,5 min. Qual o mdulo de acelerao angular do disco em rad/s2?

4. Uma locomotiva puxa 3 vages de carga com uma acelerao de 2,0 m/s2. Cada vago tem 10 toneladas de massa. Qual a tenso na barra de engate entre o primeiro e o segundo vages, em unidades de 103 N? (Despreze o atrito com os trilhos).

P R O V A

5. Um automvel de 1000 kg tem um motor de 70 kWatts. Se o motor desenvolve esta potncia velocidade de 36 km/h, qual a acelerao mxima, em m/s2, que o carro poderia ter nesta velocidade se toda a potncia fornecida pelo motor fosse aproveitada para o movimento?

6. Um balano consiste de uma haste de peso desprezvel, de comprimento L 1,6 m e uma massa m 200 kg. Na ausncia de resistncia do ar, quando ele solto da horizontal (ponto A da figura) adquire um movimento ao longo do semicrculo de raio L. Qual a tenso na haste, em unidades de 103 N, quando o balano passa pelo ponto B de sua trajetria?

7. medida que se aproxima da superfcie de um planeta, uma sonda espacial envia dados para a Terra. A tabela abaixo indica os valores medidos para a acelerao da gravidade desse planeta como funo da distncia h da sonda sua superfcie. g(m/s2) 0,6 2,4 h(km) 4,8 103 0,7 103

Com base nesses dados, determine o valor do raio desse planeta, medido em unidades de 105 m.

163

FSICA

UFPE
F A S E

8. Um longo tubo vertical, de seo reta S1 5,0 cm2 foi conectado a um tanque metlico. A tampa do tanque, de rea total S2 1,0 m2, pode suportar uma presso monomtrica mxima igual a 1,7 105 N/m2. O tanque progressivamente cheio de gua atravs do tubo. Quando o tanque estiver completamente cheio, qual a altura mxima h, em metros, que a coluna de gua poder atingir no tubo antes da ruptura da tampa do tanque?

2 C

P R O V A

9. Um gs ideal absorve 64 J de calor ao se expandir isotermicamente, de um volume inicial de 105 N/m2 at um volume final de 70cm3, a 2.0 105 N/m2 (trecho AB do 20 cm3, a 6,0 diagrama abaixo). Qual o trabalho total, em Joules, produzido pelo gs durante o ciclo ABCA?

10. Considere um raio de luz contido em um plano perpendicular aos dois espelhos planos, conforme a figura abaixo. O raio refletido formar o ngulo , em graus, considerando que 37.

164

FSICA
11. Um bloco de massa m 100g, inicialmente em repouso sobre um plano inclinado de 30, est a uma distncia L de uma mola ideal de constante elstica k 200N/m. O bloco ento solto e quando atinge a mola fica preso nela, comprimindo-a at um valor mximo D. Supondo que L D 0,5 m, qual o valor, em centmetros, da compresso mxima da mola? (Despreze o atrito entre o plano e o bloco).

UFPE
F A S E

2 C

12. Quantos gramas de gelo, a 0C, devem ser misturados a 280 g de gua a 45C, de modo que a temperatura final da mistura seja igual a 20C? 13. Uma camada esfrica isolante, cujos raios interno e externo valem, respectivamente, R1 10 cm e R2 20 cm carregada com cargas Q1 24 10 3 C e Q2 0,2 10 3 C, cada uma homogeneamente distribuda em uma de suas superfcies (ver figura).

P R O V A

Uma carga de prova q 1,0 10 6 C colocada a uma distncia r 1,0 m do centro da camada esfrica. Determine a razo F1/F2 entre as foras exercidas, respectivamente, pelas cargas Q1 e Q2 sobre a carga de prova. 14. No circuito ao lado, a resistncia RAB uniforme e tem comprimento igual a 10 cm e um valor total igual a 15 . O contato F deslizante. O grfico abaixo mostra o valor da tenso entre os pontos C e D, como funo da distncia x entre o contato F e o ponto B. Determine o valor em Volts da fora eletromotriz da bateria .

scan 329

165

FSICA

UFPE
F A S E

15. O grfico abaixo representa a variao da diferena de potencial entre as placas de um capacitor plano de placas paralelas e capacitncia igual a 5,0 10 6 F, quando carregado de uma carga inicial qi 0 at uma carga final qF 5,0 10 5 C Determine o valor, em unidades de 10
5

J da energia armazenada no capacitor.

2 C

P R O V A

16. O grfico abaixo representa o comportamento da induo magntica em pontos situados a uma distncia r de um fio retilneo e muito longo. Se B foi medido em Teslas, qual o valor em Ampres da corrente transportada pelo fio?

166

FSICA

UFRGS

1. O mecanismo de um relgio move-se um pouco mais rpido do que deveria, porm regularmente, de modo que o relgio adianta 10 minutos por dia. Que erro resulta ao se medir com esse relgio um intervalo de tempo que num relgio certo de 6 h? O erro acidental ou sistemtico? a) b) c) d) e) 1,66 min. Acidental. 1,66 min. Sistemtico. 2,50 min. Acidental. 2,50 min. Sistemtico. 5,00 min. Sistemtico.

2. A Lua dista da Terra 3,8 10 m. Admitindo-se que a luz se propaga com uma velocidade constante de 300.000 km/s, quanto tempo, aproximadamente, leva a luz para percorrer a distncia Terra-Lua? a) 0,78 s b) 1,27 s c) 12,7 s d) 127 s e) 1270 s

3. Os grficos abaixo representam acelerao contra tempo para cinco objetos diferentes. Todos os eixos possuem a mesma escala. No intervalo de tempo entre t0 e t1, qual dos objetos sofre a maior variao de velocidade?

4. O grfico velocidade contra tempo representa o movimento retilneo de um objeto. No instante t 65 s, o mdulo da acelerao instantnea desse objeto , aproximadamente, igual a a) 0,46 m/s 2 b) 1 m/s 2 c) 2 m/s
2

d) 10 m/s 2 e) 30 m/s

167

FSICA

UFRGS

5. medida que cresce a velocidade de um objeto que cai em linha reta em direo ao solo, cresce tambm a fora de atrito com o ar, at que, em determinado instante, torna-se nula a fora resultante sobre esse objeto. A partir desse instante, o objeto a) b) c) d) e) interrompe sua queda em direo ao solo. inverte o sentido da sua velocidade. continua caindo com velocidade crescente. continua caindo, mas a velocidade decrescente. continua caindo, mas a velocidade constante.

6. Na figura, o segmento AB representa uma barra homognea, de 1 m de comprimento, que mantida em equilbrio mecnico na posio horizontal. A barra est apoiada num ponto a r 25 cm da extremidade A, e o mdulo da fora F, aplicada na extremidade B, 2 N. Qual o peso da barra?

a) 0,66 N

b) 1 N

c) 4 N

d) 6 N

e) 8 N

7. Um sistema formado por duas massas, m1 e m2, com uma mola de massa desprezvel comprimida entre elas, encontra-se inicialmente em repouso. Quando as massas so liberadas, elas se afastam uma da outra, impulsionadas pela mola, e atingem velocidades mximas cujos mdulos so v1 e v2, respectivamente. A figura representa essa situao.

Supondo que no sejam exercidas foras externas sobre o sistema, o quociente m1/m2 dado por a) v2/v1 b) v1/v2 c) v1 / v2 d) v2 / v1 e) (v2/v1)
2

8. Uma pedra de 4 kg de massa colocada em um ponto A, 10 m acima do solo. A pedra deixada cair livremente at um ponto B, a 4 m de altura. Quais so, respectivamente, a energia potencial no ponto A, a energia potencial no ponto B e o trabalho realizado sobre a pedra pela fora peso? (Use g 10 m/s2 e considere o solo como nvel zero para a energia potencial.) a) 40 J, 16 J e 24 J. b) 40 J, 16 J e 56 J. c) 400 J, 160 J e 240 J. d) 400 J, 160 J e 560 J. e) 400 J, 240 J e 560 J.

168

FSICA
Instruo: O enunciado abaixo refere-se s questes de nmeros 9 e 10:

UFRGS

Um objeto em forma de bloco, partindo do repouso, desliza ao longo de um plano inclinado de comprimento L, livre de qualquer atrito.

9. Que distncia percorre o bloco sobre o plano inclinado at adquirir a metade da energia cintica que ter no final do plano? a) L/4 b) L ( 2 c) L/2

1)

d) L/ 2 e) (3 L) / 4

10. Que distncia percorre o bloco sobre o plano inclinado at adquirir a metade da quantidade de movimento que ter no final do plano? a) L/4 b) L ( 2 c) L/2
1)

d) L/ 2 e) (3 L) / 4

11. Um recipiente cbico, de aresta h, est repleto de um lquido de massa especfica . O cubo transportado por um elevador que se move com acelerao constante a, dirigida para cima, numa regio onde a acelerao da gravidade g. Nesta situao, a presso exercida pelo lquido em qualquer ponto da base do cubo dada por a) ah b) gh c) 2gh d) (g e) (g a)h a)h

12. Uma esfera macia e homognea, de massa especfica igual a 2,4 g/cm3, flutua mantendo 20% do seu volume acima da superfcie livre de um lquido. A massa especfica desse lquido, em g/cm3, igual a a) 1,9 b) 2,0 c) 2,5 d) 3,0 e) 12,0

13. Um recipiente contm um gs ideal temperatura T. As molculas deste gs tm massa m e velocidade quadrtica mdia v. Um outro recipiente contm tambm um gs ideal, cujas molculas tm massa 3m e a mesma velocidade quadrtica mdia v. De acordo com a teoria cintica dos gases, qual a temperatura deste segundo gs? a) T/9 b) T/3 c) T d) 3T e) 9T

169

FSICA
14. Uma amostra de certa substncia slida est contida em um recipiente e recebe calor de uma fonte trmica, a uma taxa constante em relao ao tempo. O grfico representa, de forma qualitativa, a variao da temperatura (T) da amostra em funo do tempo (t), entre os instantes ta e tf. Em qual dos intervalos assinalados no grfico a amostra passa gradativamente do estado slido para o estado lquido? a) ta tb b) tb tc c) tc td d) td te e) te tf

UFRGS

15. Uma amostra de gs ideal realiza o ciclo termodinmico representado no diagrama pV da figura ao lado. No ponto I, a temperatura do gs TI. Em que ponto a temperatura do gs TI? a) J b) K c) L d) M e) N

16. Um gs ideal sofre um processo em duas etapas, conforme ilustra o diagrama pV ao lado. Na etapa A ele aumenta seu volume em uma expanso isotrmica, ao passo que na etapa B ele aquecido a volume constante. Selecione a alternativa que preenche corretamente as lacunas na afirmao seguinte:

a) b) c) d) e)

aumenta - permanece constante permanece constante - aumenta permanece constante - diminui diminui - aumenta aumenta - diminui

Na etapa A a energia interna do gs que na etapa B a energia interna do gs


, ao passo .

170

FSICA

UFRGS

17. Durante um ciclo termodinmico, uma mquina trmica realiza o trabalho W, que igual a Q1 Q2 , onde Q1 o calor extrado de uma fonte quente, e Q2 o calor descarregado no ambiente. O rendimento dessa mquina trmica dado por a) b) c) d) e) (Q1 Q2) / Q1 (Q1 Q2) / Q2 Q1 / (Q1 Q2) Q2 / (Q1 Q2) (Q1 Q2) / Q2

18. Uma partcula, com carga eltrica q, encontra-se a uma distncia d de outra partcula, com carga 3q. Chamando de F1 o mdulo da fora eltrica que a segunda carga exerce sobre a primeira e de F2 o mdulo da fora eltrica que a primeira carga exerce sobre a segunda, podemos afirmar que a) b) c) d) e) F1 F1 F1 F1 F1 3F2 e as foras so atrativas. 3F2 e as foras so repulsivas. F2 e as foras so atrativas. F2 e as foras so repulsivas. F2/3 e as foras so atrativas.

19. Trs esferas metlicas idnticas, mantidas sobre suportes isolantes, encontram-se inicialmente afastadas umas das outras, conforme indica a figura (a). Duas das esferas esto eletricamente carregadas, uma com 9 10 6 C e a outra com 15 10 6 C, enquanto a terceira est descarregada. As trs esferas so ento colocadas em contato, de modo que se toquem mutuamente, conforme indica a figura (b). Assinale a alternativa que fornece os valores corretos das cargas eltricas que as esferas apresentam aps terem sido postas em contato: a) b) c) d) e) 0 C, 0 C, 0 C 9 10 6 C, 15 10 6 C, 0 C 12 10 6 C, 12 10 6 C, 0 C 8 10 6 C, 8 10 6 C, 8 10 6 C 2 10 6 C, 2 10 6 C, 2 10 6 C

171

FSICA

UFRGS

20. Quando uma diferena de potencial aplicada aos extremos de um fio metlico, de forma cilndrica, uma corrente eltrica i percorre esse fio. A mesma diferena de potencial aplicada aos extremos de outro fio, do mesmo material, com o mesmo comprimento mas com o dobro do dimetro. Supondo os dois fios mesma temperatura, qual ser a corrente eltrica no segundo fio? a) i b) 2 i c) i/2 d) 4 i e) i/4

21. Considere o circuito eltrico representado na figura abaixo. Selecione a alternativa que preenche corretamente as lacunas na afirmativa seguinte:

a) b) c) d) e)

300 mA; 300 mA 200 mA; 200 mA 200 mA; 240 mA 900 mA; 780 mA 200 mA; 150 mA

22. O rtulo de um chuveiro eltrico indica 4500 W e 127 V. Isso significa que, ligado a uma rede eltrica de 127 V, o chuveiro consome a) b) c) d) e) 4500 joules por segundo. 4500 joules por hora. 571500 joules por segundo. 4500 calorias por segundo. 4500 calorias por hora.

Com a chave C aberta, a corrente eltrica que passa pela resistncia de 20 de com a chave C fechada, a corrente eltrica que passa pela resistncia de 20 de

; .

172

FSICA

UFRGS

23. A figura (a) representa uma metade magnetizada de uma lmina de barbear, com os plos norte e sul indicados respectivamente pelas letras N e S. Primeiramente, esta metade de lmina dividida em trs pedaos, como indica a figura (b). A seguir, os pedaos 1 e 3 so colocados lado a lado, como indica a figura (c).

A alternativa que preenche corretamente as lacunas na afirmativa acima : a) b) c) d) e) atrairo - norte - sul atrairo - sul - norte repeliro - norte - sul repeliro - sul - norte atrairo - sul - sul

Instruo: O enunciado abaixo refere-se s questes de nmeros 24 e 25: Um segmento retilneo de fio conduz uma corrente eltrica i, em uma regio onde existe um r campo magntico uniforme B. Devido a este campo magntico, o fio fica sob o efeito de uma r fora de mdulo F, cuja direo perpendicular ao fio e direo de B. 24. Se duplicarmos as intensidades do campo magntico e da corrente eltrica, mantendo inalterados todos os demais fatores, a fora exercida sobre o fio passar a ter mdulo a) 8F b) 4F c) F d) F /4 e) F /8

25. O efeito ao qual se refere o enunciado constitui o princpio de funcionamento de a) motores eltricos. b) aquecedores eltricos. c) capacitores. d) reostatos. e) eletroscpios.

Nestas condies, podemos afirmar que os pedaos 1 e 3 se plo e Q um plo .


, pois P assinala um

173

FSICA

UFRGS

26. Num transformador, a razo entre o nmero de espiras no primrio (N1) e o nmero de espiras no secundrio (N2) N1/N2 10. Aplicando-se uma diferena de potencial alternada V1 no primrio, a diferena de potencial induzida no secundrio V2. Supondo tratar-se de um transformador ideal, qual a relao entre V2 e V1? a) b) c) d) e) V2 V2 V2 V2 V2 V1/100 V1/10 V1 10V1 100 V1

27. Dois corpos de massas diferentes, cada um preso a uma mola distinta, executam movimentos harmnicos simples de mesma freqncia e tm a mesma energia mecnica. Neste caso, a) b) c) d) e) o corpo de menor massa oscila com menor perodo. o corpo de menor massa oscila com maior perodo. os corpos oscilam com amplitudes iguais. o corpo de menor massa oscila com menor amplitude. o corpo de menor massa oscila com maior amplitude.

28. Considere as afirmaes abaixo: I - O som se propaga no ar com uma velocidade de aproximadamente 340 m/s. II - As velocidades de propagao do som no ar e no vcuo so aproximadamente iguais. III - O eco devido reflexo do som. Quais delas so corretas? a) Apenas I b) Apenas I e II c) Apenas I e III 29. Considere as afirmaes abaixo: I - As ondas luminosas so constitudas pelas oscilaes de um campo eltrico e de um campo magntico. II - As ondas sonoras precisam de um meio material para se propagar. III - As ondas eletromagnticas no precisam de um meio material para se propagar. Quais delas so corretas? a) Apenas I b) Apenas I e II c) Apenas I e III d) Apenas II e III e) I, II e III d) Apenas II e III e) I, II e III

174

FSICA
30. Um raio de luz, proveniente da esquerda, incide sobre uma lmina de vidro de faces paralelas, imersa no ar, com ngulo i de incidncia 1 na interface ar-vidro. Depois de atravessar i a lmina, ele emerge do vidro com ngulo 1 . O trajeto do r raio luminoso est representado na figura, onde 1 designa o i ngulo de refrao no vidro, e 2 , o ngulo de incidncia na interface vidro-ar. Nessa situao, pode-se afirmar que i r a) 1 2 b) 1 2 i r c) i1 2 r d) e) i1 i
1

UFRGS

i2 i
2

31. A figura representa uma lente esfrica delgada de distncia focal f. Um objeto real colocado esquerda da lente, numa posio tal que sua imagem real se forma direita da mesma. Para que o tamanho dessa imagem seja igual ao tamanho do objeto, esse deve ser colocado a) esquerda de G. b) em G. c) entre G e F. d) em F. e) entre F e a lente.

32. Selecione a alternativa que preenche corretamente as lacunas do seguinte texto. Uma pessoa v nitidamente um objeto quando a imagem desse objeto se forma sobre a retina. Em pessoas mopes, a imagem se forma frente da retina. Em pessoas hipermtropes, os raios luminosos so interceptados pela retina antes de formarem a imagem (diz-se, ento, que a imagem se forma atrs da retina).

Pessoas mopes devem usar culos com lentes usar culos com lentes . a) b) c) d) e) convergentes - biconvexas convergentes - divergentes plano-convexas - divergentes divergentes - bicncavas divergentes - convergentes

e pessoas hipermtropes devem

175

FSICA
33. Considere as afirmaes abaixo:

UFRGS

I - A distncia focal de uma lente depende do meio que a envolve. II - A luz contorna obstculos com dimenses semelhantes ao seu comprimento de onda, invadindo a regio de sombra geomtrica. III - Luz emitida por uma fonte luminosa percorre o interior de fibras ticas, propagando-se de uma extremidade outra. Os fenmenos ticos melhor exemplificados pelas afirmaes I, II e III so, respectivamente, os seguintes: a) b) c) d) e) refrao, difrao e reflexo total. refrao, interferncia e polarizao. espalhamento, difrao e reflexo total. espalhamento, interferncia e reflexo total. disperso, difrao e polarizao.

34. O diagrama ao lado representa alguns nveis de energia do tomo de hidrognio. Qual a energia do fton emitido quando o tomo sofre uma transio do primeiro estado excitado para o estado fundamental? a) 1,8 eV b) 5,0 eV c) 10,2 eV d) 12,0 eV e) 17,0 eV

35. O alcance de partculas de 4 MeV no ar 2,4 cm (massa especfica do ar: 1,25 10 3 g/cm3). Admitindo-se que o alcance seja inversamente proporcional massa especfica do meio, o alcance das partculas de 4 MeV na gua (massa especfica da gua: 1,00 g/cm3) a) 1,92 103 cm b) 3 cm c) 1,92 cm d) 3 e) 3 10 10
1

cm 3 cm

176

FSICA

UnB

Nas questes de 1 a 9, marque: itens CERTOS, na coluna I; itens ERRADOS, na coluna II. Use, para as devidas marcaes, a Folha de Rascunho e, posteriormente, a Folha de Respostas.

Questo 1 O estabelecimento das idias a respeito da gravitao universal considerado uma das conquistas mais importantes no desenvolvimento das cincias em geral e, particularmente, da Fsica. A sua compreenso fundamental para o entendimento dos movimentos da Lua, dos planetas, dos satlites e mesmo dos corpos prximos superfcie da Terra. Em relao a esse assunto, julgue os itens abaixo. (0) Para que a Lua descreva o seu movimento orbital ao redor da Terra, necessrio que a resultante das foras que atuam sobre ela no seja nula. (1) Um satlite em rbita circular ao redor da Terra move-se perpendicularmente ao campo gravitacional terrestre. (2) A fora gravitacional sobre um satlite sempre realiza trabalho, independentemente de sua rbita ser circular ou elptica. (3) Um corpo, quando solto prximo superfcie terrestre, cai em direo a ela pelo mesmo motivo que a Lua descreve sua rbita em torno da Terra.

Questo 2 Um pescador inexperiente, desanimado com os resultados obtidos em um dia de pescaria, resolveu relembrar os seus conhecimentos de Fsica. Sendo assim, ele abandonou a vara de pesca, passando a observar cuidadosamente o que acontecia ao seu redor; fixou a ateno nas guas calmas e cristalinas do lago e observou a movimentao de um peixe grande, de 5 kg, que estava por ali, em busca de alimento. Julgue os itens seguintes, a respeito das reflexes do pescador. (0) No momento em que o peixe est parado, totalmente imerso na gua, distante do fundo do lago, seu peso igual ao empuxo que ele recebe. (1) O peixe observado pesa menos de 5 N. (2) Se o peixe observado pelo pescador, nadando a 1 m/s, engole um outro peixe de 0,25 kg, que vinha nadando distrado em sua direo, em sentido oposto, de modo que, aps esse belo almoo, o peixe grande fica parado em funo do choque, ento a velocidade do peixe menor, antes de ser engolido, era de 4 m/s. (3) Se uma pequena pedra for atirada, a partir da margem do lago, com o objetivo de acertar o peixe, ao atingir a superfcie da gua, a pedra no-necessariamente comear a afundar.

177

FSICA
Questo 3 Uma esfera de ferro fixada por uma mola em uma plataforma giratria, como mostra a figura ao lado. Dois observadores, um na plataforma e o outro fixo ao solo, em repouso, observam o movimento da esfera, que, quando est a meio caminho entre o eixo de rotao e a borda da plataforma circular, causa uma distenso de 5 cm na mola. Julgue os itens que se seguem, relativos situao apresentada.

UnB

(0) Para o observador situado sobre a plataforma girante, a fora centrfuga que distende a mola. (1) Para o observador em repouso, no solo, a fora centrpeta aplicada pela mola distendida que mantm a esfera em movimento circular, junto com a plataforma girante. (2) A esfera ter a sua velocidade linear reduzida pela metade, quando a distenso da mola for de 10 cm. (3) Se for liberada da mola, a esfera escapar da plataforma e o observador em repouso, no solo, ver que ela descreve um movimento circular, at atingir o repouso. Questo 4 Dois colegas de trabalho, discutindo alguns fenmenos que envolvem conceitos de Fsica, propuseram trs diferentes situaes, representadas na figura abaixo, nas quais alguns desses conceitos aparecem. Nas trs situaes, um pequeno bloco de ferro, de peso igual a 10 N, elevado at uma altura de 3 m e, depois, desliza, sem atrito, em trs rampas diferentes.

Julgue os itens a seguir, relativos a algumas concluses surgidas no decorrer da discusso entre os colegas. (0) Nas trs situaes, o trabalho total realizado para vencer a fora de atrao gravitacional de 30 J. (1) Na situao III, o bloco apresenta acelerao crescente, embora sua velocidade diminua. (2) A situao II a nica na qual o bloco desliza aumentando sua velocidade, com acelerao decrescente. (3) Na situao I, o bloco desliza com uma acelerao constante, porm menor do que a acelerao da gravidade.

178

FSICA
Questo 5

UnB

No sistema de cargas abaixo representado, as cargas Q esto fixas, eqidistantes da origem 0, mas a carga q pode mover-se livremente sobre o eixo y.

Supondo que a carga q seja abandonada no ponto de coordenada (0, a), a partir do repouso, julgue os itens abaixo. (0) (1) (2) (3) A velocidade de q ser mxima na origem e, nesse ponto, a acelerao ser nula. Depois de passar pela origem, a carga ser freada pela fora resultante que atuar sobre ela. Sendo o sistema conservativo, a velocidade da carga ser nula, no ponto de coordenadas (0, a). Se as duas cargas fixas fossem substitudas por cargas negativas, o comportamento da carga q no seria alterado.

Questo 6 Um vestibulando, depois de tomar banho frio por alguns dias, resolveu construir um resistor para substituir a resistncia queimada do chuveiro de sua casa. Depois de fazer uma pesquisa, optou pela utilizao de um fio de constant (liga metlica), com seo transversal (bitola) de 2 2 8,0 10 mm . Sabendo que o chuveiro tem potncia de 4.400 W, quando alimentado com uma tenso eltrica de 220 V, julgue os itens seguintes. (0) A corrente eltrica que passar pelo resistor ser superior corrente que circula pelo motor de uma geladeira que tem as especificaes de 500 W e 220 V. 2 2 (1) Se o fio tivesse uma bitola igual a 16,0 10 mm , o vestibulando poderia ter usado um fio com metade do comprimento. (2) Se o material utilizado na confeco do resistor tivesse sido o cobre, o valor da resistncia do chuveiro seria diferente. (3) A necessidade de se construir a resistncia do chuveiro com um fio enrolado pode ser explicada pelo fato de o fio ter comprimento bem superior s dimenses do prprio chuveiro.

179

FSICA
Questo 7

UnB

O autodidata Michael Faraday (1791-1867), notvel cientista ingls, dedicou seus estudos a diversos ramos da Fsica, entre eles o Eletromagnetismo. Nesse ramo, a sua grande contribuio foi, sem dvida, a descoberta do fenmeno da induo eletromagntica, que possibilitou o surgimento e o desenvolvimento dos grandes geradores eltricos e dos transformadores, equipamentos imprescindveis aos atuais sistemas eltricos de energia, utilizados em todo o mundo. Antes de Faraday anunciar a sua descoberta, o que se sabia era que uma corrente eltrica, ao percorrer um condutor, produzia um campo magntico. Tal fenmeno foi estudado por vrios cientistas, entre eles Andr-Marie Ampre (1775-1836), fsico e matemtico francs, que construiu o primeiro eletrom. Com relao ao eletromagnetismo, julgue os itens que se seguem. (0) Faraday descobriu que, em determinadas condies, um campo magntico pode originar uma corrente eltrica. (1) Nos enrolamentos de um transformador, h induo de fora eletromotriz devido ao movimento relativo entre eles e o campo magntico existente no ncleo de ferro do transformador. (2) O funcionamento do gerador eltrico (alternador) de um automvel moderno pode ser explicado com a utilizao da Lei de Faraday. (3) A Lei de Faraday, apesar de sua inquestionvel importncia para o eletromagnetismo, tem uma grande limitao: s aplicvel a equipamentos em corrrente contnua, isto , no-alternada.

Questo 8 As ondas, perturbaes em um meio, que implicam a transmisso de energia e de momento linear, sem que haja transporte de matria, so um dos assuntos fascinantes da Fsica. Instrumentos de corda, transmisso de TV e radares so algumas das muitas aplicaes desse tipo de conhecimento. Com relao a esse assunto, julgue os seguintes itens. (0) A velocidade de propagao de uma onda em uma corda depende da trao a que est sujeita essa corda. (1) Se a velocidade de uma onda, em uma corda esticada, de 170 m/s, quando a trao de 120 N, aumentando-se a trao para 180 N, a velocidade da onda passar a ser de 255 m/s. (2) Supondo-se que Tristo sopre um apito e que Isolda escute o som, ela o ouvir em uma freqncia maior, se estiver correndo ao encontro de Tristo.

180

FSICA
Questo 9 Pela associao de roldanas fixas e mveis, uma pessoa pode levantar pesos muito grandes, acima de sua capacidade muscular. Por isso, v-se, com freqncia, sistemas de roldanas sendo utilizados em canteiros de obras de construo civil. Suponha que a figura ao lado represente o sistema utilizado pelos operrios de uma obra, para erguer, do solo at o segundo pavimento, um elevador de material de construo, com peso de 100 kgf. Com base na associao mostrada na figura, julgue os itens que se seguem.

UnB

(0) Se o peso das polias for desprezvel, um operrio dever aplicar uma fora F igual a 25 kgf para equilibrar o sistema. (1) Se cada polia pesar 0,5 kgf, a fora F que equilibrar o sistema ser de 26,5 kgf. (2) Supondo que cada polia tenha um peso de 0,5 kgf, a reao do suporte, no ponto C, ser igual a 51,25 kgf. (3) Um operrio, ao suspender o elevador, utilizando uma associao de polias como esta, realiza um trabalho bem menor do que aquele que realizaria sem tal dispositivo. Questo 10 Um material denominado supercondutor quando, abaixo de uma certa temperatura, chamada de temperatura crtica (Tc), passa a ter resistncia nula, caracterstica que justifica o nome do material. Considere que, no circuito abaixo esquematizado, o resistor R seja feito de um material supercondutor, cuja temperatura crtica seja Tc 2,0C. O valor da resistncia R, para temperaturas acima de Tc igual a 20 . A lmpada L, colocada no circuito para indicar a circulao de corrente, possui resistncia interna de 2 .

Calcule, em ampres, a corrente eltrica do circuito, a uma temperatura ambiente de 25C. Desconsidere a parte funcionria do seu resultado, caso exista.

181

FSICA
Questo 11

UnB

O funcionamento de alguns instrumentos de medidas eltricas, como, por exemplo, o galvanmetro, baseia-se no efeito mecnico que os campos magnticos provocam em espiras que conduzem correntes eltricas, produzindo o movimento de um ponteiro que se desloca sobre uma escala. O modelo abaixo mostra, de maneira simples, como campos e correntes provocam efeitos mecnicos. Ele constitudo por um fio condutor, de comprimento igual a 50 cm, suspenso por uma mola de constante elstica igual a 80 N/m e imerso em um campo magntico uniforme, de intensidade B igual a 0,25 T, com direo perpendicular ao plano desta folha e sentido de baixo para cima, saindo do plano da folha. Calcule, em ampres, a corrente eltrica I que dever percorrer o condutor, da esquerda para a direita, para que a mola seja alongada em 2,0 cm, a partir da posio de equilbrio estabelecida com corrente nula. Desconsidere a parte fracionria do seu resultado, caso exista.

Questo 12 Indeciso com relao convocao dos jogadores que deveriam compor a seleo universitria de futebol da UnB, para disputar os Jogos Universitrios do DF (JUDF), o tcnico, dispondo de vrios jogadores de mesmo nvel tcnico, resolveu lanar um desafio, garantindo participao no time para aqueles que respondessem corretamente ao seguinte problema: na cobrana de um pnalti, em uma partida de futebol, uma bola de massa igual a 0,40 kg chutada com velocidade inicial de 25 m/s. O tempo de contato entre o p do jogador e a bola de 0,05 s. Calcule, em newtons, a fora mdia aplicada bola pelo p do jogador. Para marcar a sua resposta, divida a fora calculada por 10, desprezando, depois, a parte fracionria do seu resultado, caso exista.

Questo 13 Um prisma reto de vidro cuja base um tringulo retngulo issceles foi totalmente mergulhado em gua. Calcule o menor ndice de refrao que tal prisma dever ter, para que reflita por completo um raio que incida normalmente em uma das faces menores. Considere que o ndice de refrao da gua seja igual a 0,95 2 , multiplique por 10 o valor calculado, desconsiderando, depois, a parte fracionria de seu resultado, caso exista.

182

FSICA
Questo 14

UnB

Uma senhora diz a sua filha de 10 anos para encher a banheira de modo que possa tomar um banho. A criana abre apenas a torneira de gua quente, deixando que 94,625 L de gua a 80C sejam colocados na banheira. Calcule, em litros, a quantidade de gua a 20C necessria para baixar a temperatura da gua da banheira para 50C. Desconsidere a capacidade trmica da banheira e a parte fracionria de seu resultado, caso exista.

Questo 15 Na figura abaixo, que representa uma placa homognea, admita que cada quadrado tenha lado igual a 10 cm. Determine, em centmetros, a soma das coordenadas do ponto correspondente ao centro de gravidade da placa. Desconsidere a parte fracionria de seu resultado, caso exista.

183

FSICA

UNESP
F A S E

1. O grfico na figura representa a posio x de um mvel, que se deslocou ao longo de uma linha reta, em funo do tempo t.

A velocidade do mvel foi constante e diferente de zero durante o intervalo de tempo que vai dos instantes a) 0 a t1 b) t1 a t1 c) t2 a t3 d) t3 a t4 e) t4 a t5

2. Dois corpos, de pesos 10 N e 20 N, esto suspensos por dois fios, P e Q, de massas desprezveis, da maneira mostrada na figura. As intensidades (mdulos) das foras que tensionam os fios P e Q so, respectivamente, de a) 10 N e 20 N. b) 10 N e 30 N. c) 30 N e 10 N. d) 30 N e 20 N. e) 30 N e 30 N.

3. Duas pequenas esferas idnticas, 1 e 2, so lanadas do parapeito de uma janela, perpendicularmente parede, com velocidades horizontais , 1 e 2, v v com v2 > v1, como mostra a figura, e caem sob a ao da gravidade. A esfera 1 atinge o solo num ponto situado distncia x1 da parede, t1 segundos depois de abandonar o parapeito, e a esfera 2 num ponto situado distncia x2 da parede, t2 segundos depois de abandonar o parapeito. Desprezando a resistncia oferecida pelo ar e considerando o solo plano e horizontal, podemos afirmar que a) x1 = x2 e t1 = t2 b) x1 < x2 e t1 < t2 c) x1 = x2 e t1 > t2 d) x1 > x2 e t1 < t2 e) x1 < x2 e t1 = t2

184

FSICA
4. A intensidade (mdulo) da resultante das foras que atuam num corpo, inicialmente em repouso, varia como mostra o grfico. Durante todo o intervalo de tempo considerado, o sentido e a direo dessa resultante permanecem inalterados. Nestas condies, a quantidade de movimento, em kg.m/s (ou N.s), adquirida pelo corpo a) 8 b) 15 c) 16 d) 20 e) 24

UNESP
F A S E

5. Um corpo slido e insolvel foi pendurado num dinammetro por meio de um fio fino e flexvel. Com o corpo imerso no ar, o dinammetro indicou uma fora de valor P. Quando totalmente imerso na gua, como mostra a figura, P decresceu de uma quantidade P, devido ao do empuxo.

A experincia foi repetida com vrios outros corpos, todos slidos e insolveis. Para um deles, em particular, o decrscimo na indicao do dinammetro, quando o corpo passou do ar para a gua, foi tambm P, embora o dinammetro tivesse indicado, com o corpo imerso no ar, um valor P diferente de P. Analisando o fato das indicaes P e P terem decrescido de uma mesma quantidade P quando esses corpos passaram do ar para a gua, estudantes apresentaram trs concluses diferentes: I - Os dois corpos tm a mesma massa. II - Os dois corpos tm o mesmo volume. III - Os dois corpos tm a mesma massa especfica. Dessas concluses, a) Apenas I est correta. b) Apenas II est correta. c) Apenas III est correta. d) Apenas I e III esto corretas. e) I, II e III esto corretas.

185

FSICA

UNESP
F A S E

6. Um bloco de certa liga metlica, de massa 250 g, transferido de uma vasilha, que contm gua fervendo em condies normais de presso, para um calormetro contendo 400 g de gua temperatura de 10 C. Aps certo tempo, a temperatura no calormetro se estabiliza em 20 C. Supondo que toda a quantidade de calor cedida pela liga tenha sido absorvida pela gua do calormetro, pode-se dizer que a razo entre o calor especfico da gua e o calor especfico da liga metlica igual a a) 1 b) 2 c) 3 d) 4 e) 5

7. Uma pequena bola, abandonada de certa posio, cai sob a ao da gravidade, sem encontrar qualquer resistncia. 0,5s aps ter sido abandonada, atinge o solo, onde sofre uma coliso perfeitamente elstica, volta para a posio original e torna a cair. Desprezando o tempo de interao da bola com o solo e imaginando que o fenmeno se repita indefinidamente, podese afirmar que a freqncia, em hertz, com que a bola colide com o solo de a) 0,5 b) 1,0 c) 2,0 d) 3,0 e) 4,0

8. Analise a tabela e responda.


SUBSTNCIA NDICE DE REFRAO EM RELAO AO AR

gua lcool etlico glicerina quartzo cristalino vidro comum

1,33 1,63 1,47 1,54 1,50

Para um mesmo ngulo de incidncia diferente de zero, o maior desvio na direo de um raio de luz que se propaga no ar ocorrer quando penetrar a) na gua. b) no lcool etlico. c) na glicerina. 9. Assinale a alternativa correta. a) Quando algum se v diante de um espelho plano, a imagem que observa real e direita. b) A imagem formada sobre o filme, nas mquinas fotogrficas, virtual e invertida. c) A imagem que se v quando se usa uma lente convergente como lente de aumento (lupa) virtual e direita. d) A imagem projetada sobre uma tela por um projetor de slides virtual e direita. e) A imagem de uma vela formada na retina de um olho humano virtual e invertida. d) no quartzo cristalino. e) no vidro comum.

186

FSICA

UNESP
F A S E

10. Duas esferas condutoras idnticas, carregadas com cargas +Q e -3Q, inicialmente separadas por uma distncia d, atraem-se com uma fora eltrica de intensidade (mdulo) F. Se as esferas so postas em contato e, em seguida, levadas de volta para suas posies originais, a nova fora entre elas ser a) maior que F e de atrao. b) menor que F e de atrao. c) igual a F e de repulso. d) menor que F e de repulso. e) maior que F e de repulso.

11. Dois resistores, um de 20 e outro de resistncia R desconhecida, esto ligados em srie com uma bateria de 6,0 V e resistncia interna desprezvel, como mostra a figura.

Se a corrente no circuito de 0,1 A, o valor da resistncia R, em , a) 20 b) 30 c) 40 d) 50 e) 60

12. Sabe-se que no ponto P da figura existe um campo magntico na direo da reta RS e apontando de R para S. Quando um prton (partcula de carga positiva) passa por esse ponto com a velocidade mostrada na figura, atua sobre ele uma fora, devida a esse campo magntico, v

a) b) c) d) e)

perpendicular ao plano da figura e penetrando nele. na mesma direo e sentido do campo magntico. na direo do campo magntico, mas em sentido contrrio a ele. na mesma direo e sentido da velocidade. na direo da velocidade, mas em sentido contrrio a ela.

187

FSICA

UNESP
F A S E

1. O tempo de reao (intervalo de tempo entre o instante em que uma pessoa recebe a informao e o instante em que reage) de certo motorista 0,7 s, e os freios podem reduzir a velocidade de seu veculo razo mxima de 5 m/s em cada segundo. Supondo que esteja dirigindo velocidade constante de 10 m/s, determine: a) o tempo mnimo decorrido entre o instante em que avista algo inesperado, que o leva a acionar os freios, at o instante em que o veculo pra; b) a distncia percorrida nesse tempo. 2. Uma carreta de 50 toneladas levada de uma margem a outra de um lago, por meio de uma balsa. Para sair da balsa, depois que esta atraca no ancoradouro, a carreta inicia seu movimento com acelerao constante, percorrendo 8,0 metros em 10 segundos. Nestas condies, determine: a) a acelerao da carreta; b) a fora adicional a que fica submetido o cabo que mantm a balsa atracada, enquanto a carreta est se deslocando sobre ela com movimento uniformemente acelerado. 3. Um corpo de massa 1,0 kg lanado obliquamente, a partir do solo, sem girar. O valor da componente vertical da velocidade, no instante do lanamento, 2,0 m/s e o valor da componente horizontal 3,0 m/s. Supondo que o corpo esteja sujeito exclusivamente ao da gravidade, determine sua energia cintica: a) no instante do lanamento; b) no ponto mais alto da trajetria. 4. Um carrinho, A, de massa m, e outro, B, de massa 2m, mantidos em repouso sobre uma superfcie plana e horizontal, esto comprimindo uma mola, de massa desprezvel, como mostra a figura.

2 A

P R O V A

Quando os carrinhos so liberados simultaneamente, a mola se distende, impulsionando-os, e B adquire, depois que a mola estiver totalmente distendida, uma velocidade de 1,0 m/s. a) Nessas condies, determine a velocidade adquirida por A. b) Denominando hA e hB as alturas mximas alcanadas, respectivamente, pelos carrinhos h A e B, ao subirem as rampas mostradas na figura, determine a razo A . hB

188

FSICA
5. Para realizar a experincia que leva seu nome, Torricelli tomou um tubo de vidro, com cerca de 1 metro de comprimento, fechou uma de suas extremidades e encheu-o completamente com mercrio (Figura I). Tampando a extremidade livre e invertendo o tubo, mergulhou essa extremidade em um recipiente que tambm continha mercrio. Ao destampar o tubo, Torricelli verificou que o nvel da coluna lquida descia, at estacionar a uma altura de cerca de 76 cm acima do nvel do mercrio no recipiente (Figura II).

UNESP
F A S E

2 A

P R O V A

Concluiu, ento, que a presso atmosfrica, pa, atuando na superfcie do lquido no recipiente, equilibrava a coluna de mercrio e, portanto, que a presso atmosfrica equivalia presso exercida pelo peso de uma coluna de mercrio de 76 cm. a) Se essa experincia fosse realizada na Lua, em condies tais que o mercrio no se solidificasse, toda a coluna lquida desceria para o recipiente. Explique por qu. b) Determine a altura da coluna de mercrio, imaginando essa experincia realizada em um planeta onde a presso atmosfrica fosse 10 vezes menor que a presso atmosfrica na Terra e a acelerao da gravidade na superfcie 2,5 vezes menor que a acelerao da gravidade na Terra. (Suponha desprezvel a variao da massa especfica do mercrio com a gravidade e com a temperatura.)

6. As figuras representam feixes paralelos de luz monocromtica, incidindo pela esquerda, nas caixas A e B, que dispem de aberturas adequadas para a entrada e a sada dos feixes.

Para reproduzir esses efeitos, dispunha-se dos trs elementos seguintes:

a) Copie a Figura A no seu caderno de respostas. Em seguida, desenhe no interior da caixa, na posio correta, um dos trs elementos disponveis, que produza o efeito mostrado, e complete a trajetria dos raios. b) Copie a Figura B no seu caderno de respostas. Em seguida, desenhe no interior da caixa, na posio correta, um dos trs elementos disponveis, que produza o efeito mostrado, e complete a trajetria dos raios.

189

FSICA

UNESP
F A S E

7. Transfere-se calor a um sistema, num total de 200 calorias. Verifica-se que o sistema se expande, realizando um trabalho de 150 joules, e que sua energia interna aumenta. a) Considerando 1 cal = 4 J, calcule a quantidade de energia transferida ao sistema, em joules. b) Utilizando a primeira lei de termodinmica, calcule a variao da energia interna desse sistema. 8. A figura mostra uma esfera condutora ligada Terra por meio de um galvanmetro G. Com esta montagem, observou-se que o galvanmetro indica: um pulso de corrente, enquanto se aproxima da esfera condutora um basto carregado com cargas positivas, e outro pulso de corrente, mas de sentido contrrio ao primeiro, quando se leva para longe da esfera o mesmo basto. Usando a seguinte representao carga positiva: + carga negativa: carga nula: n a) copie e complete a figura em seu caderno de respostas, mostrando tanto a distribuio de cargas na esfera condutora como o sentido da corrente de eltrons que flui pelo galvanmetro G, enquanto se aproxima da esfera o basto carregado; b) copie novamente e complete a figura em seu caderno de respostas, mostrando tanto a distribuio de cargas na esfera condutora como o sentido da corrente de eltrons que flui pelo galvanmetro G, quando se leva para longe da esfera o basto carregado. 9. A figura representa uma associao de trs resistores, todos com a mesma resistncia R.

2 A

P R O V A

a) Denominando V1 e V2, respectivamente, as tenses entre A e B e entre B e C, quando a V2 associao est ligada a uma bateria, determine a razo V . 1 b) Sabendo que a potncia dissipada no resistor colocado entre B e C igual a 1,2 watts, determine a potncia dissipada em cada um dos outros dois resistores.

190

FSICA
1. Um carrinho de 2,0 kg, que dispe de um gancho, movimenta-se sobre um plano horizontal, com velocidade constante de 1,0 m/s, em direo argola presa na extremidade do fio mostrado na figura. A outra extremidade do fio est presa a um bloco, de peso 5,0 N, que se encontra em repouso sobre uma prateleira.

UNESP
F A S E

2 B

Enganchando-se na argola, o carrinho puxa o fio e eleva o bloco, parando momentaneamente quando o bloco atinge a altura mxima h acima da prateleira. Nestas condies, determine: a) a energia cintica inicial do carrinho; b) a altura h, supondo que ocorra perda de 20% da energia cintica inicial do carrinho quando o gancho se prende na argola. (Despreze quaisquer atritos e as massas das polias.) 2. Dois blocos, A e B, ambos de massa m, esto ligados por um fio leve e flexvel, que passa por uma polia de massa desprezvel, que gira sem atrito. O bloco A est apoiado sobre um carrinho de massa 4m, que pode se deslocar sobre a superfcie horizontal sem encontrar qualquer resistncia. A figura mostra a situao descrita.

P R O V A

Quando o conjunto liberado, B desce e A se desloca com atrito constante sobre o carrinho, acelerando-o. Sabendo que a fora de atrito entre A e o carrinho, durante o deslocamento, equivale a 0,2 do peso de A (ou seja, Fat = 0,2 mg) e fazendo g = 10 m/s2, determine: a) a acelerao do carrinho; b) a acelerao do sistema constitudo por A e B.

191

FSICA

UNESP
F A S E

3. Ao subir do fundo de um lago para a superfcie, o volume de uma bolha de gs triplica. Sabese, ainda, que a presso exercida pelo peso de uma coluna de gua de 10,0 metros igual presso atmosfrica na regio em que o lago se localiza. a) Qual seria a profundidade desse lago, supondo que a temperatura no fundo fosse igual temperatura na superfcie? b) Qual seria a profundidade desse lago, supondo que a temperatura absoluta no fundo fosse 4% menor que a temperatura na superfcie? 4. As figuras representam feixes paralelos de luz monocromtica incidindo, pela esquerda, nas caixas A e B, que dispem de aberturas adequadas para a entrada e a sada dos feixes.

2 B

P R O V A

Para produzir esses efeitos, dispunha-se de um conjunto de lentes convergentes e divergentes de diversas distncias focais. a) Copie a Figura A no seu caderno de respostas. Em seguida, desenhe no interior da caixa uma lente que produza o efeito mostrado, complete a trajetria dos raios e indique a posio do foco da lente. b) Copie a Figura B no seu caderno de respostas. Em seguida, desenhe no interior da caixa um par de lentes que produza o efeito mostrado, complete a trajetria dos raios e indique as posies dos focos das lentes. 5. Os grficos na figura mostram o comportamento da corrente em dois resistores, R1 e R2, em funo da tenso aplicada.

a) Considere uma associao em srie desses dois resistores, ligada a uma bateria. Se a tenso no resistor R1 for igual a 4 V, qual ser o valor da tenso em R2? b) Considere, agora, uma associao em paralelo desses dois resistores, ligada a uma bateria. Se a corrente que passa pelo resistor R1 for igual a 0,30 A, qual ser o valor da corrente por R2?

192

GEOGRAFIA

CESGRANRIO

1. O potencial de crescimento do mercado de automveis, associado a outros fatores como os incentivos fiscais, vem atraindo, para o Brasil, investimentos por parte das grandes montadoras, algumas delas j aqui instaladas. Repercusses e impactos desses novos investimentos j vm sendo observados na organizao da produo e do mercado de trabalho. Isto pode ser constatado pela tendncia : a) substituio da mo-de-obra brasileira de baixa qualificao por trabalhadores do Mercosul. b) atuao mais combativa dos sindicatos localizados prximo s grandes montadoras de So Paulo e de Minas Gerais. c) implantao das novas fbricas nas Zonas Francas existentes nas Regies Norte e Nordeste do pas. d) diminuio do nmero de empregos pela robotizao de linhas de montagem. e) eliminao de prticas de terceirizao entre os fornecedores de autopeas e as grandes montadoras. 2.

Fonte: ANTUNES, Celso. Geografia e participao. So Paulo, Scipione. p. 31.

A populao sertaneja que se desloca nas estaes secas encontra na Zona da Mata uma demanda por mo-de-obra sazonalmente aquecida pela: a) b) c) d) e) formao de pastagens novas em reas de matas. realizao da colheita de cana-de-acar. necessidade de replantio de velhos cafezais. implantao de grandes projetos de reflorestamento. ocorrncia do plantio de lavouras de algodo.

193

GEOGRAFIA

CESGRANRIO

3. No final do sculo XIX, houve a fundao de um grande nmero de indstrias no Rio de Janeiro. Inicialmente, estes estabelecimentos se fixaram no centro da cidade. Depois, comearam a se transferir para um bairro de localizao privilegiada (pela proximidade do porto e de ramais ferrovirios) e que possua infraestrutura (decorrente do fato de ter sido rea residencial da nobreza do Imprio). Esse bairro : a) Mier. b) Estcio. c) Cidade Nova. d) So Cristvo. e) Del Castilho.

4. Em 1993, a participao dos bancos no Produto Interno Bruto (PIB) era da ordem de 13%. Em 1996, esta porcentagem caiu para 7,7%. Vrias instituies bancrias, algumas de grande porte, entraram em crise, levando o Governo Federal a criar um polmico programa de reestruturao do sistema bancrio, o PROER. Um dos objetivos do PROER o de: a) elevar a rentabilidade do setor pelo estmulo a ganhos inflacionrios. b) financiar fuses entre instituies financeiras para reduzir o tamanho do sistema. c) recapitalizar os bancos com a criao da contribuio provisria sobre movimentao financeira (CPMF). d) aumentar o poder poltico-eleitoral dos bancos estaduais, em fase de estatizao. e) implementar rgido controle sobre a movimentao de capital estrangeiro nas bolsas de valores. 5. A retomada do crescimento econmico no Estado do Rio de Janeiro passa pela revalorizao de suas vantagens comparativas, de sua populao e de suas vocaes naturais. Neste contexto, a recuperao da Baa da Guanabara est mais do que justificada. Do ponto de vista tcnico, o projeto de recuperao envolve a: a) reverso do avanado estgio de eutrofizao das guas atravs da implantao de sistemas e redes de saneamento no Grande Rio. b) retirada da vegetao dos manguezais responsveis pelo acmulo de lama, detritos e sedimentos grosseiros. c) anulao da capacidade de eroso e de sedimentao das correntes existentes na Baa pela construo de diques e de quebra-mares. d) reduo das taxas atuais de assoreamento pela transferncia do movimento porturio para a Baa de Sepetiba. e) diminuio dos impactos ambientais dos aterros sanitrios, em virtude de sua implantao em reas menos poludas no fundo da Baa.

194

GEOGRAFIA

CESGRANRIO

6. o Estado foi incapaz de controlar a resistncia da populao excluda e de atender s demandas localizadas, que eclodiram em uma frente de conflitos expressa em movimentos sociais de base territorial.
FONTE: BECKER, B. e EGLER, C. Brasil: uma nova potncia regional na economia-mundo. Rio de Janeiro, Bertrand-Brasil. p. 217.

Dentre esses movimentos sociais de base territorial, o Movimento dos Sem-Terra um dos mais importantes nos dias de hoje, sendo a sua atuao baseada na defesa de ampla reforma agrria, o que envolve a adoo das medidas abaixo, COM EXCEO DE UMA. Assinale-a. a) Acesso a mercados consumidores sem a excessiva intermediao dos atravessadores. b) Implantao de fazendas de administrao estatal de acordo com o modelo dos sovkhozes. c) Difuso de cultivos ecolgica e economicamente sustentveis entre os pequenos produtores. d) Estabelecimento de polticas de crdito rural compatveis com os riscos das atividades agropecurias. e) Estruturao de redes de armazns e silos como proteo para as flutuaes sazonais dos preos agrcolas. 7. Com relao s estruturas industriais das Regies Sul e Nordeste do Brasil, representadas no grfico ao lado, so feitas as afirmativas abaixo. I - A concentrao de capitais se mostra maior no Sul do pas, em razo da permanncia de grande nmero de pequenas empresas fundadas por imigrantes, no incio deste sculo. II - A absoro de mo-de-obra no Nordeste relativamente mais baixa, em virtude do alto nvel de mecanizao das indstrias de bens de consumo intermedirios e durveis. III - O processo de industrializao no Sul do pas decorreu diretamente da implantao de polticas de desconcentrao da indstria paulista, atravs da concesso de incentivos fiscais. IV - A localizao das indstrias modernas privilegiou as grandes cidades do Nordeste, especialmente Salvador, Recife e Fortaleza, pelo fato de elas serem pontos de maior desenvolvimento que os espaos interioranos. Esto corretas as afirmativas: a) I e II somente. b) I e III somente. c) II e III somente. d) II e IV somente. e) III e IV somente.

195

GEOGRAFIA

CESGRANRIO

8. Procurando um melhor entendimento do processo de urbanizao, o IBGE estabeleceu critrios para classificar os mais de 5000 municpios, hierarquiz-los, e, com isso, desenhar a rede urbana brasileira. A evoluo dessa rede vem recentemente registrando significativas alteraes, conforme o(a): a) b) c) d) e) 9. crescimento da importncia relativa das chamadas capitais regionais. xodo da populao das cidades mdias para as megalpoles. extino da classe de pequenas cidades consideradas como centros locais. estagnao econmica e cultural das metrpoles regionais. incorporao de mais municpios classe das metrpoles nacionais.

O mapa acima identifica as regies econmicas que compem o Sudeste Brasileiro. Assinale a nica associao correta entre uma dessas regies e sua respectiva caracterstica econmica. REGIO a) b)
Extremo Norte

CARACTERSTICA ECONMICA
Economia regional especializada na produo de soja para exportao. Setor de laticnios bem desenvolvido e articulado com os mercados consumidores de So Paulo e Rio de Janeiro. Agricultura tipicamente voltada subsistncia e baseada em trabalho familiar. Complexos agroindustriais de grande porte recentemente implantados por empresas do MERCOSUL. Pecuria de corte baseada em prticas intensivas e de alta produtividade

Sul de Minas

c) d)

Centro-Oeste Paulista

Vale do Ribeira

e)

Serras e Planaltos do Sudeste

196

GEOGRAFIA

CESGRANRIO

10. Nos anos 70, o Governo Federal considerava urgente o rpido desenvolvimento da capacidade de gerao de energia atmica. Hoje, passadas duas dcadas de sua implantao, especialistas acreditam que o Programa Nuclear Brasileiro deve, no entanto, ser repensado, em virtude dos fatores apresentados a seguir, COM EXCEO DE UM. Assinale-o. a) Custo elevado de desenvolvimento numa conjuntura de crise econmica. b) Dependncia de tecnologias estrangeiras derivadas do uso de urnio enriquecido como combustvel. c) Escolha de modelo ultrapassado de reator para equipar a Usina de Angra I. d) Carter pacifista do programa pela ausncia de interesses das Foras Armadas no seu desenvolvimento. e) Impasse quanto deciso definitiva sobre o local para deposio do lixo atmico. 11. O terrorismo continua atuando, no devido a seu poder ideolgico ou fsico, mas porque as naes que dele so vtimas no conseguem det-lo.
A.M. Rosenthal. O Globo, 02/08/96, p. 07

No caso dos EUA, o terrorismo est relacionado ao de grupos radicais de origens externas e internas. Com relao a estes ltimos, notrio o fato de que se trata de organizaes de extrema direita, que, como objetivo, procuram: a) b) c) d) e) implantar uma repblica sindicalista no pas. suprimir da Constituio as emendas de inspirao nazifascista. reafirmar o poder da maioria anglo-saxnica frente s minorias negra e hispnica. reagir contra o aumento da influncia das Foras Armadas na poltica nacional. protestar contra o aumento da pobreza nas reas rurais norte-americanas.

12. Leia e analise os dados abaixo. III III IV Origem na base das nuvens do tipo cumulus nimbus. Ocorrncia maior nas estaes da primavera e do vero. Tpico de pases de dimenses continentais como os E.U.A. e a Austrlia. Velocidades de deslocamento de at 800 km/h.

O fenmeno meteorolgico que rene as caractersticas apresentadas acima conhecido como: a) frente ocluda b) tromba dgua c) vento anabtico d) furaco e) tornado

197

GEOGRAFIA

CESGRANRIO

13. H dois sculos discute-se o que o Governo deve fazer. Que tal comear a discutir s o que o Governo pode fazer?
Adaptado da Coluna de Joelmir Beting. O Globo. 28/07/96. p. 50

Entre os economistas, a definio do tamanho adequado e das funes a serem realizadas pelo Estado uma preocupao constante. Para os economistas que seguem o neoliberalismo, o Estado deve procurar este tamanho ideal, implementando medidas que: a) b) c) d) e) elevem a concesso de subsdios governamentais a empresas em dificuldades financeiras. sustentem a obteno de recursos para o patrocnio de amplos programas previdencirios. permitam conciliar crescimento econmico com elevadas taxas de inflao anuais. fortaleam as finanas das empresas estatais frente a tendncias privatistas. garantam a abertura do mercado interno concorrncia e a fluxos de comrcio internacionais.

14. Uma comparao entre as redes de transportes do Brasil e dos pases europeus mostra prontamente a importncia que as hidrovias tm para estes ltimos. As hidrovias transportam um maior volume de cargas a baixo custo. Por isso, muitos centros urbanos e plos industriais se desenvolveram ao longo dos rios. Considera-se como a mais importante hidrovia europia ocidental a que aproveita o vale do: a) Tejo b) Reno c) Tmisa d) Loire e) Elba

15. A posio da Austrlia, em relao s rotas de navegao, foi um dos motivos que retardou a incorporao de seu territrio ao horizonte geogrfico europeu. Sua ocupao s viria a ocorrer de modo efetivo a partir do sculo XVIII, e seria marcada pela influncia de fatores naturais, o que pode ser constatado pela: a) descoberta de jazidas de ouro que atraram milhares de imigrantes a partir de 1850. b) existncia de forte atividade madeireira, nas reas de vastas florestas de conferas, na poro oeste do pas. c) implantao de lavouras de exportao de cana-de-acar e cacau devido abundncia de gua, em todo o territrio. d) ocorrncia, no centro do pas, de solos tropicais de alto potencial de aproveitamento para o cultivo de trigo. e) caracterstica montanhosa do relevo, o que impediu a integrao da rede de transportes no sentido norte-sul.

198

GEOGRAFIA

CESGRANRIO

16. Entre os pases do Terceiro Mundo, a ndia vem sendo considerada um dos mercados emergentes neste final de sculo. Todavia, este enorme potencial poder no se efetivar. que h inmeros fatores polticos e socioeconmicos que podem frear ou mesmo limitar seriamente o processo indiano de modernizao. Um desses fatores o(a): a) fortalecimento de organizaes guerrilheiras que pretendem implantar o socialismo no pas. b) controle severo do governo em relao s taxas de crescimento demogrfico nos ltimos anos, o que j vem repercutindo na reduo da oferta de mo-de-obra. c) inexistncia de setores industriais avanados, em razo do grande atraso tecnolgico. d) participao pouco expressiva de capitais nacionais em atividades urbanas e industriais como conseqncia do carter rural da sociedade. e) permanncia de rivalidades tnico-religiosas que representam uma real ameaa de ocorrncia de conflitos separatistas. 17.Em primeiro lugar, preciso reconhecer que algo, de fato, deu errado na Unio Sovitica. No havia nenhuma semelhana entre o processo de emancipao socialista imaginado por Marx e aquilo que existiu naquele pas.
Fonte: ARBEX JR., Jos. Revoluo em trs tempos; URSS, Alemanha, China. Ed. Moderna.

Assinale, dentre as opes abaixo, aquela que expressa um fator que tenha contribudo decisivamente para a ex-URSS ter dado errado. a) Descentralizao das atividades de planejamento econmico. b) Controle do poder poltico por apenas dois partidos: o Comunista e o Social-Democrata. c) Adeso tardia das autoridades soviticas ao mercado comum dos pases socialistas, o COMECOM. d) Predomnio de uma estrutura de economia paralela de mercado sobre os setores estatais. e) nfase na produo industrial militar e aeroespacial em prejuzo de setores civis. 18. A destruio dos grandes domnios florestais do planeta no significa apenas o risco de se extinguirem diversas espcies animais e vegetais, a maior parte delas ainda sequer conhecidas e catalogadas pela cincia. O desmatamento em grande escala pode trazer uma srie de conseqncias ambientais, como a(o): a) b) c) d) e) perenizao do regime dos cursos e bacias hidrogrficas. paralisao dos processos erosivos atuantes na formao de voorocas. entrada de um maior volume de gua das chuvas nos lenis freticos. aumento do escoamento das guas pluviais sobre a superfcie dos solos. incremento das taxas de umidade do ar nas reas desflorestadas.

199

GEOGRAFIA

CESGRANRIO

19. Nos desertos localizados em latitudes subtropicais, como o caso do Saara, na frica, registrase uma tendncia ocorrncia de uma notvel amplitude trmica diria que pode, inclusive, superar os 40C. A explicao que se pode dar a este fenmeno est na(o): a) baixa concentrao dos gases responsveis pela conservao do calor na atmosfera, como o metano e o gs carbnico. b) circulao de ventos ciclnicos caractersticos de grandes extenses desrticas. c) ocorrncia de rpido processo de inverso trmica pela passagem de frentes frias. d) acmulo, nos solos excessivamente argilosos, de toda a energia trmica irradiada pela luz solar. e) registro de altas presses atmosfricas, associadas a massas de ar ascendentes. 20. Leia e analise os dados abaixo: I - Sua administrao realizada, desde 1842, por uma das principais potncias europias. II - Tem sua economia fortemente baseada em atividades tercirias, constituindo grande centro econmico e financeiro. III - Apresenta legislao e regime de trabalho que permitem elevada explorao da mode-obra. IV - porta de entrada para o grande mercado representado pela China. As caractersticas apresentadas acima se referem a um dos chamados Tigres Asiticos. Assinale-o. a) Macau. b) Coria do Sul. c) Hong Kong d) Formosa e) Cingapura

200

GEOGRAFIA

FUVEST
F A S E

1. Consumo brasileiro de energia segundo as fontes primrias, por setor (IBGE, 1992)

1 B

P R O V A

Na legenda do grfico acima, os algarismos correspondentes s fontes de energia no renovveis so: a) I e II b) I e V c) III e I 2. d) III e IV e) V e II

A observao dos mapas do Estado de So Paulo permite afirmar que, de modo geral, as temperaturas decrescem a) de sudeste para noroeste sem grandes oscilaes, exceto no Vale do Paraba, sempre mais frio que o restante do Estado. b) de oeste para leste com acentuada queda das temperaturas ao sul do trpico de Capricrnio. c) de oeste para leste, excetuando-se o centro, onde h permanentemente uma ilha de temperaturas mais elevadas. d) de leste para oeste, excetuando-se o noroeste onde as temperaturas so sempre superiores s das demais reas. e) de noroeste para sudeste, interrompida pela ntida queda de temperatura nas terras elevadas do planalto Atlntico.

201

GEOGRAFIA
Mapa para as questes 3 e 4

FUVEST
F A S E

1 B

P R O V A

3. Considere, no mapa acima, o conjunto I, formado pelos pases dos trs Plos de Deciso, e os conjuntos II e III, formados, respectivamente, pelos pases da Amrica Latina e da frica. A tabela abaixo indica, em bilhes de dlares (1992), o total das exportaes de mercadorias oriundas dos pases constituintes de cada um dos trs conjuntos e tendo como destino pases dos mesmos trs conjuntos.
Destino Origem I II I Plos de Deciso 1736 93 II Amrica Latina 126 27 III frica 72 3

III

71

A partir dessas informaes e dos seus conhecimentos sobre o comrcio mundial, indique a afirmativa correta a respeito da natureza das mercadorias que dominam, em valor, os fluxos comerciais indicados por sua origem e destino. a) b) c) d) e) No interior de I, manufaturados; de I para II, produtos agrcolas. No interior de I, manufaturados; de I para III, produtos agrcolas. No interior de I bem como no interior de II, manufaturados. No interior de I, produtos agrcolas; de III para I, produtos minerais. No interior de I bem como de II e III para I, produtos minerais.

4. A partir de seus conhecimentos sobre projees cartogrficas e analisando a que foi utilizada no mapa ao lado, voc pode inferir que se trata da projeo a) b) c) d) e) de Mercator, adequada para estabelecer a direo das rotas comerciais martimas. polar, adequada para representaes geoestratgicas e geopolticas. de Peters, adequada para representar a rea dos continentes, sem deformaes. cilndrica, adequada para a representao centrada nas regies polares. cnica, adequada para representar as regies de latitudes mdias.

202

GEOGRAFIA

FUVEST
F A S E

5. O Indicador de Desenvolvimento Humano (IDH) vem substituindo o Produto Interno Bruto por habitante (PIB/hab) como parmetro mais adequado para avaliar o nvel mdio de prosperidade e bem-estar da populao de um pas. O IDH leva em conta o PIB/hab, corrigido pela diferena do poder de compra em cada pas, e os respectivos nveis mdios de instruo e sade. A tabela mostra, para alguns pases da OPEP, sua posio na classificao, feita em ordem decrescente, de cada um desses indicadores.
PIB/hab Emirados rabes Unidos Qatar Kuwait Arbia Saudita Om 16 22 26 31 35 IDH 62 56 51 67 92

1 B

P R O V A

Analisando-a e usando conhecimentos gerais sobre a geografia desses pases, possvel afirmar que as diferenas sistemticas entre as duas classificaes tm como origem comum a) o fracasso da OPEP em conseguir um preo justo para o petrleo bruto no mercado internacional. b) uma elevada densidade demogrfica que dificulta um atendimento satisfatrio aos problemas de sade e instruo. c) a submisso ao poder dos grupos transnacionais que controlam a comercializao do petrleo. d) uma estrutura socioeconmica arcaica que, privilegiando a concentrao de renda, agrava as desigualdades sociais. e) a ocorrncia de grandes extenses desrticas, imprprias para a agricultura e pecuria. 6.

No esquema acima, X e Y so, respectivamente, a) mangans e bauxita. b) mangans e cobre. c) cobre e cassiterita. d) ferro e cassiterita. e) ferro e bauxita.

203

GEOGRAFIA
7.
Joo saiu com a famlia num desespero sem nome. Ele, os filhos e Maria estavam mortos de fome. Que destino tomaria? Onde iria trabalhar? E sua volta ele via terra e mais terra vazia, milho e cana a verdejar. [Ferreira Gullar, 1962]

FUVEST
F A S E

1 B

Analisando as questes abordadas no poema acima, pode-se afirmar que no Brasil, nas trs ltimas dcadas, a) vem aumentando, gradativamente, a ocupao pelo Governo Federal de latifndios improdutivos e terras devolutas para a produo de lcool e alimentos para o consumo interno. b) diminuram, em freqncia e intensidade, as oposies entre terras de negcio e terras de trabalho na Amaznia e no Centro-Sul, graas aos assentamentos realizados pelo INCRA. c) reduziram-se as migraes sazonais, permanecendo apenas os corumbas que, na poca das colheitas, se deslocam da Zona da Mata para o Agreste. d) diminuram a fome e o desemprego no campo, devido expanso da produo de alimentos para a populao e de matrias-primas para as indstrias. e) intensificou-se o xodo rural, em decorrncia da maior concentrao da propriedade fundiria e das transformaes nas relaes de trabalho no campo. 8.

P R O V A

Refletindo sobre o desenho acima, em uma rea Tropical, podemos inferir que a) em matas e bosques, a grande oscilao diuturna da temperatura mantm as nuvens baixas, fazendo com que chova mais. b) em reas com escassa cobertura vegetal, o ar frio e seco empurra as nuvens para cima, fazendo com que chova menos. c) o ar mais mido e quente sobre as matas e bosques ocasiona maior regularidade pluviomtrica. d) as pequenas amplitudes trmicas sobre as plantaes produzem uma camada estacionria de ar mido, impedindo a presena de nuvens baixas. e) em reas com abundante cobertura vegetal, o ar mais frio e rarefeito facilita a descida das nuvens mais pesadas.

204

GEOGRAFIA

FUVEST
F A S E

9. Indique a alternativa que no descreve corretamente uma caracterstica das florestas tropicais midas. a) b) c) d) e) Apresentam a maior biodiversidade dos biomas terrestres. Ocupam reas de solos com alto teor de nutrientes minerais. Representam cerca de 40% de toda a fitomassa terrestre. A polinizao predominantemente feita por animais e no pelo vento. So perenes, mas com uma grande e contnua reciclagem de sua biomassa.

1 B

10. Os desmatamentos, as queimadas, o estabelecimento da agropecuria extensiva ou da agricultura itinerante, seguidos pela lixiviao dos solos, podem acarretar, nas zonas tropicais, a) b) c) d) e) a exposio de lateritas ou crostas ferruginosas. a alterao da fertilidade dos solos podzis. a concentrao excessiva de fosfatos nos tchernozions. o empobrecimento dos solos de pradarias. o aumento do latossolo nas regies semi-ridas.

P R O V A

11. O II Plano Nacional de Desenvolvimento criou, na dcada de 70, os chamados plos regionais. Um deles, o Polamaznia, implicava carrear recursos e viabilizar projetos destinados a reas especficas, privilegiando a) b) c) d) e) o setor de indstrias de base e a infra-estrutura urbana. a reduo das disparidades regionais, atendendo as reas de maior pobreza. os setores agropecurio e mineral. a construo de hidrovias e a ampliao do sistema rodovirio. a demarcao das terras indgenas e das reservas extrativistas.

12. A existncia de extensas reas secas localizadas nas costas ocidentais dos continentes em latitudes vizinhas a ambos os trpicos determinada, essencialmente, pela a) b) c) d) e) dinmica atmosfrica controlada pela zona de convergncia intertropical. presena de reas de baixa presso atmosfrica. alternncia entre massas polares e equatoriais em tais latitudes. presena de correntes martimas quentes ao longo dos litorais. presena de correntes martimas frias ao longo dos litorais.

205

GEOGRAFIA

FUVEST
F A S E

13. A rgida diviso do trabalho do taylorismo e a procura de mo-de-obra barata e de mercados nacionais esto cedendo lugar s novas necessidades industriais do capitalismo detectadas, a partir da dcada de 70, nos pases do Norte.

1 B

P R O V A

A figura acima ilustra essa nova maneira de produzir, que privilegia a) as alianas empresariais, restritas ao interior de blocos regionais, mas disputando a hegemonia sobre mercados supranacionais. b) a economia de redes transnacionais, multiplicando as trocas entre o mundo da tecnologia, das indstrias e dos servios. c) a ao de multinacionais que, competindo pelo domnio da tecnologia, colocam os mercados nacionais acima dos imperativos da globalizao. d) o papel da mdia, padronizando as informaes e criando necessidades de consumo de novos produtos industriais. e) a modernizao da linha de montagem do Fordismo, possibilitando a complexa ampliao da produo e do consumo em massa. 14. Observe as figuras abaixo.

De modo geral, seguindo o percurso do rio de jusante para montante, a seqncia correta das figuras a) X, Z e Y b) X, Y e Z c) Y, X e Z d) Z, Y e X e) Z, X e Y

206

GEOGRAFIA
15.

FUVEST
F A S E

1 B

Observe a escala do tempo geolgico para identificar os processos naturais que ocorreram, respectivamente, nas eras Paleozica e Cenozica. a) Formao de jazidas carbonferas e dobramentos do tipo alpino-himalaio. b) Oscilaes do nvel do mar nos ltimos perodos glaciais e formao das bacias petrolferas do Oriente Mdio. c) Configurao atual dos continentes e oceanos e dobramentos do tipo alpino-himalaio. d) Formao das bacias petrolferas do Oriente Mdio e soterramento das florestas que originaram o carvo mineral. e) Oscilaes do nvel do mar nos ltimos perodos glaciais e configurao atual dos continentes e oceanos. 16. Para alguns produtos agrcolas brasileiros, a produo se encontra fortemente concentrada em um nico estado. Assinale a alternativa que faz corretamente a associao entre cada uma das lavouras indicadas e o estado onde sua produo alcana o maior percentual, na atualidade. arroz a) b) c) d) e) BA RS BA RS SP cana-deacar SP RJ PE SP PE caf MG SP PR MG PR algodo SP PR PB PR PB P R O V A

17. Indique a alternativa que associa corretamente um impacto ambiental sua rea de ocorrncia na atualidade. a) Contaminao de solos e guas pelo uso intensivo de agrotxicos, nas monoculturas do noroeste do Rio Grande do Sul. b) Eroso de terraos e plancies fluviais por jatos de gua usados em garimpos, no sudeste de Minas Gerais. c) Cicatrizes no solo e grande volume de rejeitos resultantes da minerao do carvo a cu aberto, no vale do Itaja, no norte de Santa Catarina. d) Mar negra causada pela extrao de petrleo, na plataforma continental da Regio Norte do Brasil. e) Empobrecimento dos solos arenosos pelos desmatamentos e prticas inadequadas da pecuria, na regio de Ribeiro Preto - SP.

207

GEOGRAFIA

FUVEST
F A S E

18. A tabela abaixo fornece alguns indicadores sobre demografia, sade e educao no Brasil, Cuba, Paraguai e Uruguai, em 1992.

Pas

Populao urbana (%) 89 74 47 75

Populao adulta alfabetizada (%) 96.5 94.5 90.8 82.1

Escolaridade mdia da populao adulta (anos) 8.1 8.0 4.9 4.0

Mortalidade infantil () 20 14 47 63

1 B

I II III IV

P R O V A

Identifique o conjunto de indicadores que caracteriza, na ordem, o Brasil e Cuba. a) II e I b) III e I c) III e II d) IV e II e) IV e III

19. Neste final de sculo, especialmente aps o encerramento da guerra fria, recrudesceram, em vrios pases da Europa e da sia, conflitos de natureza tnica. Considere os conjuntos abaixo, um incluindo grupos em conflito entre si e o outro regies da Europa e da sia.

Grupos em conflito I curdos e seus vizinhos II hutus e tutsis III bsnios e srvios IV tchetchenos e russos

R S Y Z

Regies de conflito Leste europeu Sul da Europa Oriente Mdio Sudeste da sia

Assinale a alternativa que contm somente associaes corretas. a) I e Y, II e Z, III e S. b) I e S, III e Y, IV e R. c) I e Y, II e Z, III e S. d) I e Y, III e S, IV e R. e) I e Z, III e R, IV e Y.

208

GEOGRAFIA
20. Com base no grfico ao lado e nos conhecimentos gerais sobre o processo de urbanizao no Brasil, correto afirmar que

FUVEST
BRASIL - Populao Urbana, em %, por Regio

a) um verdadeiro processo de urbanizao ocorreu apenas no SE, apesar de ter havido um crescimento urbano generalizado nesse perodo. b) a mudana da capital para Braslia e a expanso da moderna agricultura de gros ajudam a entender porque no CO se registrou o maior salto na taxa de urbanizao. c) a estrutura de propriedades familiares, favorecendo a permanncia do homem no campo, evitou, aps 1970, que a urbanizao no S acompanhasse o ritmo do SE. d) as Superintendncias Regionais, criadas na dcada de 60, conseguiram frear o rpido processo de urbanizao em andamento no NE. e) a internacionalizao da Amaznia, com a implantao de modernas agroindstrias, explica o modesto crescimento urbano na regio N. 1. Alguns dos graves problemas que afligem a frica na atualidade tm suas origens na dominao colonial desse continente pelos europeus. Discuta a afirmao acima, explicando as relaes que podem ser feitas entre: a) o imperialismo europeu e as guerras tribais. D um exemplo para justificar sua resposta. b) o sistema agrcola introduzido pelos colonizadores e a atual situao socio-econmica da frica Negra. Exemplifique com um caso especfico. 2. Considere os conjuntos de climogramas obtidos em trs estaes meteorolgicas brasileiras (I, II, III) e de fotos de formaes vegetais (A, B e C). As precipitaes so dadas em mm e as temperaturas em C.

F A S E

1 B 2

P R O V A

F A S E

a) Identifique pelo nome o tipo climtico a que se refere cada um dos climogramas. b) Identifique pelo nome as formaes vegetais de cada foto. c) Faa a associao entre a vegetao e o clima no qual ela se desenvolve naturalmente.

209

GEOGRAFIA

FUVEST
F A S E

3. Com base na figura: a) Explique por que muitos dos grandes rios brasileiros tm origem na regio de Braslia. b) Indique os nomes de duas bacias hidrogrficas cujas nascentes de alguns de seus formadores esto localizadas nesta regio. c) Comente a intensa utilizao econmica de uma dessas bacias.

4.
Percentual da populao economicamente ativa (PEA) por setores da economia em pases americanos. Primrio Pases I II Brasil Colmbia Canad Estados Unidos 1970 45 39 8 5 1980 31 34 6 4 1992 29 26 5 3 Secundrio 1970 22 23 31 34 1980 26 24 28 30 1992 25 26 22 24 1970 33 38 61 61 Tercirio 1980 43 42 66 66 1992 46 48 73 73

Considerando, separadamente, os pases dos blocos I e II, a) examine a tabela e compare a evoluo da PEA no setor primrio, analisando suas razes e conseqncias. b) examine a tabela e compare a evoluo da PEA no setor secundrio, analisando suas razes e conseqncias. c) conceitue o que setor tercirio e analise suas diferenas de estrutura nos dois blocos. 5. No incio do sculo, o Brasil recebeu importante contingente de imigrantes japoneses. Hoje o fluxo se inverteu e so os brasileiros que emigram para o Japo. Explique essa inverso a partir das principais mudanas socio-econmicas que ocorreram nos dois pases, durante o sculo XX.

210

GEOGRAFIA

FUVEST
F A S E

6. A forma de uma pirmide etria reflete a variao das taxas de natalidade e de mortalidade de um pas ao longo de quase um sculo. A figura representa a pirmide etria da Frana, no ano de 1975 (populao em milhares de habitantes).

Explique, objetivamente, em termos de taxas de natalidade e de mortalidade, correlacionandoas com os fatos histricos que lhes deram origem: a) a reentrncia na faixa etria de 30 a 35 anos. b) a reentrncia na faixa etria de 55 a 60 anos. c) a assimetria M-H que ocorre nas faixas etrias superiores a 75 anos, a qual muito maior que a normalmente observada na populao idosa dos pases desenvolvidos. 7. Sobre a extrao e industrializao do petrleo no Brasil, analise e compare a situao das regies Sudeste e Nordeste.

8. A tabela abaixo fornece valores anuais mdios (em US$ bilhes) do crescimento anual da dvida externa brasileira (fonte: BACEN) e do saldo do comrcio exterior (exportaes menos importaes, fonte: IBGE). Analise-a e use seus conhecimento sobre a economia brasileira, no perodo de 1960 aos dias de hoje, para responder ao que se pede. a) Explique as variaes na taxa de crescimento da Valores Mdios em US$ bilhes dvida externa, destaCrescimento anual Saldo comercial Perodo cando os perodos 1969da dvida anual 84 e de 1985 em diante. 60-64 0,1 - 0,1 b) Quais as principais me65-68 0,4 + 0,4 didas tomadas pelo go69-74 2,0 - 1,5 verno com respeito ao 75-78 6,5 - 2,7 pagamento das parcelas 79-80 8,6 - 4,5 da dvida que iam ven81-84 9,5 + 0,4 cendo ao longo do perodo considerado. 85-87 5,7 + 9,2 c) Com relao poltica 88-93 2,5 + 13,5 econmica cujo slogan foi exportar o que importa, comente algumas de suas repercusses sobre a sociedade brasileira.

211

GEOGRAFIA

FUVEST
F A S E

9. Em 1967, na Guerra dos Seis Dias, Israel conquistou, de seus vizinhos rabes, as reas hachuradas assinaladas no mapa por 1, 2, 3 e 4. a) Identifique-as e explique o interesse de Israel pelas mesmas. b) Considerando os acordos de paz realizados at o final de 1995, comente a atual situao poltica de cada uma delas.

10. A urbe distingue-se do ambiente rural pela sua densidade e sua capacidade. O termo megalpole comeou a ser usado h cerca de 30 anos para descrever uma constelao de grandes cidades que bordejam a costa nordeste dos Estados Unidos numa faixa quase ininterrupta de 1.000 km de extenso. Aceitas essas afirmaes, considere o mapa onde indicado o contorno do que comea a se definir como a megalpole brasileira. a) Explique o que conurbao e descreve dois exemplos do fenmeno, presentes na regio representada no mapa. Localize-os, identificando-os. b) Localize no mapa, identificando-as, mais trs cidades que se destacam pela sua populao e/ou atividade econmica. c) A regio destacada no mapa ocupa apenas 0,5% do territrio brasileiro. Justifique porque cham-la de esboo da megalpole brasileira em termos de sua participao na populao e na produo industrial nacionais (dar aproximadamente a porcentagem).

212

GEOGRAFIA
QUESTES DE 1 A 14

UFBA
F A S E

INSTRUO: Assinale as proposies verdadeiras, some os nmeros a elas associados e marque o resultado na Folha de Respostas.

Questo 1 GEOGRAFIA, ESPAO E AS TRANSFORMAES DA SOCIEDADE

Com base nos conhecimentos sobre o espao geogrfico, pode-se afirmar: (1) A paisagem representa, na anlise geogrfica, um sistema em permanente mudana, tornada, cada vez mais, complexa e heterognea, em funo do trabalho humano, continuamente inovador. (2) O processo de transformao de uma paisagem natural resulta em um fato cultural, destacando-se, particularmente, nesse processo, a construo de cidades. (4) O processo de modernizao que vem ocorrendo no espao geogrfico mundial relacionase, basicamente, ao desenvolvimento das foras produtivas, formao de capital, mobilizao de recursos materiais e construo de identidades nacionais. (8) Alm de condies polticas, sociais e econmicas favorveis, o conhecimento o principal meio de que o homem dispe, para transformar em riqueza os recursos naturais do seu espao geogrfico. (16) A Terra, ao final do milnio, ficou um pouco maior, devido ao atual domnio, pelo homem, do tempo e da velocidade nas comunicaes, pois, quanto mais velozes, mais se intensificam as distncias geogrficas. (32) As tecnologias modernas implicam uma mudana essencial nas relaes das sociedades com seus territrios, mantendo, assim, a concepo defendida pelos deterministas geogrficos. (64) A internacionalizao da produo e do comrcio acarretou a mundializao do espao geogrfico e da prpria natureza, que parte integrante do processo produtivo.

213

GEOGRAFIA
Questo 2 OCUPAO DO ESPAO URBANO DA CIDADE DO SALVADOR

UFBA
F A S E

Com base nos mapas acima e nos conhecimentos sobre a cidade do Salvador, pode-se afirmar: (1) No mapa I, utilizou-se uma escala cartogrfica menor que a utilizada no mapa II, o que garante ao mapa I maior grau de generalizao da rea representada. (2) No mapa II, A e B representam, em relao trajetria do sol na linha do horizonte, sobre Salvador, as posies correspondentes, respectivamente, ao nascente e ao poente. (4) O bairro do Rio Vermelho situa-se ao sul de Itapagipe e a sudoeste do Aeroporto. (8) Traando-se um tringulo com vrtices sobre os bairros de Itapu, Paripe e Barra, o vrtice localizado na Barra apontaria no sentido do sudeste. (16) A ocupao da cidade do Salvador ocorreu, primeiramente, na margem oriental da Baa de Todos os Santos. (32) A expanso urbana de Salvador ocorreu, principalmente, atravs da ocupao das reas centrais e da orla martima, projetando-se tambm no sentido norte. (64) O ncleo antigo da cidade do Salvador situa-se sobre um graben tectnico, produzido por um movimento orogentico que resultou na formao dos stios onde se instalaram a Cidade Alta e a Cidade Baixa.

214

GEOGRAFIA
QUESTES 3 e 4 O HOMEM E AS ALTERAES DO MEIO AMBIENTE

UFBA
F A S E

Questo 3 Com base na anlise do quadro geoambiental, pode-se afirmar: (1) O clima, o solo e a distribuio espacial da vegetao so determinados, principalmente, pelo efeito da zonalidade, ou seja, pela influncia da longitude. (2) O ambiente urbano propicia temperaturas mais amenas no seu ncleo espacial do que as existentes no seu entorno. (4) medida que o espao urbano se expande, h uma significativa reduo da fauna e da cobertura vegetal, proporcionando o aumento gradativo da contaminao das reas de mananciais, com reflexos negativos sobre a qualidade de vida. (8) O solo e a vegetao so considerados elementos biticos da paisagem e, como recursos naturais renovveis, devem ser conservados e utilizados racionalmente pelo homem. (16) A verticalizao do espao urbano funciona como labirinto de reflexo do calor, sendo que, noite, a poluio do ar bloqueia a disperso da temperatura, produzindo ilhas de calor. (32) A apropriao inadequada do relevo, nas reas urbanas, para fins de moradia, acelera a eroso das vertentes, contribuindo para o assoreamento dos leitos fluviais e, conseqentemente, aumentando a velocidade e o poder erosivo de suas guas. (64) Os mananciais aqferos so alimentados diretamente pelas chuvas, segundo o ritmo e a intensidade com que caem sobre eles.

215

GEOGRAFIA
Questo 4

UFBA
F A S E

Considerando-se a interao dos elementos constitutivos do meio ambiente, possvel afirmar: (1) Em funo da latitude e das condicionantes climticas, os latossolos so pouco lixiviados e, conseqentemente, ricos em nutrientes minerais necessrios vida vegetal e recomendados para fins agrcolas. (2) O processo de eutrofizao consiste em uma tcnica moderna de despoluio dos rios que atravessam as reas urbanas, onde o assoreamento dos canais de drenagem cada vez mais intenso. (4) A impermeabilizao do solo urbano, a retirada da vegetao e as diversas modalidades de edificaes reduzem a infiltrao das guas pluviais e aumentam, gradativamente, o volume do escoamento superficial, nas grandes cidades. (8) As chuvas cidas trazem conseqncias ecolgicas danosas para os ambientes fluviais e lacustres, dentre outros, provocando a concentrao de diversas substncias txicas nas cadeias alimentares dos locais afetados e ameaando a sade do homem. (16) As inverses trmicas, as chuvas cidas, a concentrao de poluentes em suspenso sobre as reas urbanas, registradas num dado momento, podem ser consideradas como elementos do clima. (32) A minerao e a explorao de pedreiras contribuem para a ocorrncia de impactos ambientais, cujos reflexos mais diretos ocorrem no fitoambiente, na degradao do modelado e no assoreamento gradativo dos rios. (64) A vulnerabilidade do meio ambiente aos processos erosivos, acelerados pela ao, antrpica, varia na razo inversa da presso demogrfica e da ocupao desordenada do espao.

Questo 5 Com relao dinmica do ambiente atmosfrico, pode-se afirmar: (1) O clima de uma regio corresponde a uma condio momentnea de diferentes estados da atmosfera sobre um lugar, sendo, portanto, altamente dinmico e mutvel. (2) O clima de uma regio depende de fatores geogrficos condicionantes, a exemplo da latitude e da proximidade do mar. (4) A dinmica das massas de ar responsvel pela maior parte das alteraes do clima, a exemplo da penetrao das frentes frias, que proporcionam a ocorrncia de chuvas e geadas, dentre outros fenmenos atmosfricos. (8) As mais elevadas temperaturas absolutas do globo so registradas nas latitudes equatoriais, devido perpendicularidade dos raios solares, durante todo o ano, sobre essas latitudes, o que provoca chuvas extremamente abundantes. (16) Os danos causados ao longo da trajetria dos furaces so provocados pela sua velocidade, a qual, por sua vez, resulta da existncia de presses atmosfricas mais altas nos locais para onde se dirigem. (32) As guas quentes do Golfo do Mxico assim como o deslocamento constante de grandes correntes ocenicas quentes favorecem a ocorrncia de furaces nesse setor do Atlntico Norte.

216

GEOGRAFIA
Questo 6 A CIDADE: OPO OU NECESSIDADE?

UFBA
F A S E

O processo de globalizao determinante dos modos de insero das grandes metrpoles no quadro das economias em franca transformao, com seus traos marcadamente contraditrios, ao promover tanto a integrao como a fragmentao do espao urbano. o que explica a formao de cidades prsperas, com oferta de um leque variado de servios e incorporao dos ltimos avanos tecnolgicos, em convvio com a pobreza e a misria escancaradas. (PANIZZI, p. 9)

A ilustrao e o texto acima, associados aos conhecimentos sobre o processo de urbanizao, permitem afirmar: (1) Na maioria das grandes metrpoles dos pases perifricos, constata-se atualmente uma tendncia diminuio populacional, em conseqncia do deslocamento do plo de atrao para outros centros urbanos e do caos e insegurana que caracterizam a vida nessas metrpoles. (2) As favelas decorrem de um complexo de problemas sociais, econmicos e polticos, entre os quais a distribuio da renda e da terra assume posio central. (4) Nas reas perifricas da maioria das grandes cidades do mundo subdesenvolvido, os investimentos em saneamento bsico so prioritrios e tm melhorado, consideravelmente, a qualidade de vida das populaes residentes. (8) As grandes cidades dos pases perifricos apresentam um trao comum: paisagens onde convivem, lado a lado, o luxuoso e o pauprrimo, como resultado da estrutura econmica existente e do desnvel entre o ritmo de urbanizao e a oferta de novos empregos urbanos. (16) O processo de modernizao das atividades econmicas extremamente seletivo, uma vez que apenas uma parcela da populao participa ativamente das inovaes, resultando na substituio gradativa das atividades tradicionais, antes dominantes. (32) No Brasil, as desigualdades no ritmo do processo de urbanizao refletem as disparidades econmicas regionais e a prpria insero diferenciada de cada regio no contexto da economia nacional. (64) A industrializao e a mecanizao do campo so as principais causas da urbanizao dos pases perifricos; j nos pases desenvolvidos, a criao de novos empregos no meio rural, num ritmo menor que o do crescimento demogrfico, foi a base do processo de urbanizao.

217

GEOGRAFIA
Questo 7

UFBA
F A S E

Em relao s atividades agrrias e estrutura fundiria do espao brasileiro, pode-se afirmar: (1) A recente mecanizao da agricultura reduziu a oferta de trabalho no campo, contribuindo, assim, para a formao do contingente de trabalhadores sem terra. (2) O processo de modernizao da agricultura subordinou a agropecuria s necessidades do capital urbano-industrial, fazendo com que a economia rural se transformasse, antes de tudo, em fornecedora de matria-prima para as indstrias. (4) Os focos de tenso e conflito pela posse da terra localizam-se, sobretudo, nas regies mais densamente povoadas. (8) O uso abusivo do binmio terra abundante trabalho barato trouxe, como resultado, a terra devastada e a pobreza do homem do campo. (16) A expanso acelerada das fronteiras agrcolas e a estrutura fundiria concentradora geram permanentes e crescentes conflitos pela posse da terra, resultando, quase sempre, na expropriao dos grileiros pelos posseiros. (32) A dificuldade de implantao da reforma agrria se deve, entre outros fatores, valorizao de produtos de fcil comercializao no exterior (carne, soja, pasta de papel, madeira etc.) e demanda interna pelo lcool, o que fortalece, cada vez mais, o papel poltico e econmico dos grandes proprietrios rurais. QUESTES DE 1 A 10 INSTRUO: Assinale as proposies verdadeiras, some os nmeros a elas associados e marque o resultado na Folha de Respostas. QUESTES DE 1 A 3 O TEMPO E AS TRANSFORMAES DO ESPAO E DAS IDIAS

F A S E

2 C

P R O V A

218

GEOGRAFIA
Questo 1

UFBA
F A S E

Com base em conhecimentos sobre a cincia geogrfica, a regionalizao do espao mundial e as tcnicas de conhecimento do territrio, possvel afirmar: (1) Os diversos modelos de regionalizao do espao mundial, na sua temporalidade e diversas tendncias, permitiram cincia geogrfica introduzir novos mtodos e tcnicas de investigao do meio ambiente. (2) Os diferentes ambientes terrestres absorvem e emitem, diferenciadamente, energia, que captada por satlites que realizam o trabalho de monitoramento atravs de imagens. (4) As possibilidades de utilizao das tcnicas de sensoriamento remoto tm proporcionado uma aplicao cada vez maior da Geografia em outras reas do conhecimento humano. (8) A expresso a Terra e o Homem, utilizada pela Geografia Tradicional, baseava-se na descrio do papel do homem em seu quadro natural, enfocando a realidade de uma forma descritiva. (16) A Geografia Crtica contrape-se Geografia Tradicional, pois esta considera o espao geogrfico como produto do trabalho do homem, que transforma em segunda natureza a natureza original. (32) A regionalizao do espao mundial, baseada em critrios naturais, apia-se nos postulados da Nova Geografia e apresenta grande complexidade, devido dinmica das transformaes ambientais. (64) A regionalizao do espao mundial, com base em critrios socioeconmicos, est relacionada ordem internacional prevalecente num determinado momento histrico, ao equilbrio instvel entre os pases e disputa entre as grandes potncias. Questo 2 Os conhecimentos sobre a Geopoltica e sobre as transformaes do espao geogrfico mundial permitem concluir: (1) A espiral do tempo apresenta uma evoluo crescente, onde cada acontecimento tem um referencial geogrfico e um registro histrico, evidenciando as relaes entre as variveis tempo e espao. (2) As transformaes do espao geogrfico, ao longo do tempo, exigem constantes redefinies cartogrficas, uma vez que elas representam a realidade de cada espao em momentos histricos especficos. (4) Do ponto de vista geogrfico, os dois grandes conflitos mundiais representaram momentos de desequilbrio na ordem internacional, originando novas fronteiras e novos centros hegemnicos do poder. (8) A multipolaridade contempornea contrape-se quela do incio do sculo, uma vez que hoje existe um fortalecimento do Estado-Nao e um enfraquecimento dos organismos internacionais de deciso poltica. (16) A rivalidade geopoltica entre as duas maiores potncias, no perodo do ps-guerra, provocou a bipartio do espao geogrfico mundial em dois blocos antagnicos, prevalecendo, durante o perodo da Guerra Fria, de um lado, um setor de economia estatizada e planificada e, do outro, uma economia que catalisou a construo do capitalismo mundial. (32) A nova Diviso Internacional do Trabalho (DIT) foi resultado da imploso do socialismo, que teve origem na Unio Sovitica e se expandiu por diversas reas do globo, beneficiando, sobretudo, os pases de economia perifrica.

2 C

P R O V A

219

GEOGRAFIA

UFBA
F A S E

Questo 3 Com base nos conhecimentos sobre a economia mundial, o processo de globalizao e o progresso tcnico-cientfico, pode-se afirmar: (1) As atividades econmicas de maior crescimento, nos dias atuais, so aquelas que transformam as matrias-primas em produtos manufaturados, em detrimento daquelas que produzem servios, idias e tcnicas. (2) O taylorismo e o fordismo aspectos importantes da Segunda Revoluo Industrial do sculo XX se baseiam, respectivamente, na organizao do trabalho voltada para a maior produtividade, em menos tempo e na produo e consumo em massa. (4) Os Estados Unidos, apesar de serem atualmente a nica superpotncia militar do globo, no detm a liderana econmica mundial, devido aos grandes dficits comerciais e oramentrios e ao agravamento das desigualdades sociais internas. (8) O processo de globalizao que se verifica nos dias contemporneos, associado difuso internacional da notcia atravs de redes tipo Internet, exige um mundo cada vez mais unitrio, fragmentando as fronteiras geopolticas na conquista de uma unicidade tecnolgica. (16) A expresso aldeia global sugere que, ao final do milnio, formou-se a comunidade mundial, concretizada com as realizaes e as possibilidades de informao e comunicao abertas pela informtica, ao tempo em que os entretenimentos e as idias so produzidos, comercializados e consumidos como mercadorias, simultaneamente em todas as partes do mundo. (32) As questes ambientais, durante a organizao bipolar do espao mundial, ocuparam uma posio de vanguarda nas discusses polticas das grandes potncias, despertando, assim, a conscincia ecolgica para a preservao dos recursos naturais. (64) A diversidade biolgica, no contexto da questo ambiental, uma condio fundamental para o desenvolvimento da biotecnologia, que corresponde a uma das indstrias mais promissoras da Terceira Revoluo Industrial, que ocorre atualmente. Questo 4 Os conhecimentos sobre populao, sua dinmica e suas relaes com o crescimento econmico permitem afirmar: (1) A taxa de crescimento real de uma populao o balano resultante entre o nmero de nascimentos, a entrada de imigrantes e as perdas ocorridas em razo de morte e emigrao. (2) Os movimentos migratrios, tanto nacionais como internacionais, resultam de circunstncias sociais, econmicas e polticas, estando relacionados a grandes disparidades entre o local de origem e o de destino. (4) Do ponto de vista ocupacional, a populao economicamente ativa (PEA) de um pas corresponde quele grupo que est efetivamente trabalhando, isto , exercendo atividade remunerada. (8) Nos pases subdesenvolvidos, a populao economicamente ativa (PEA) tende a ser proporcionalmente maior, devido maior percentagem de jovens em relao populao total. (16) As migraes pendulares correspondem ao deslocamento dirio de trabalhadores de suas residncias para os locais de trabalho e so tpicas das grandes cidades industriais e metrpoles mundiais. (32) Embora o salrio mnimo seja um importante indicador de renda para a populao, seu valor efetivo, no Brasil, tem decrescido desde que se acelerou o processo urbano-industrial. (64) O crescimento da economia brasileira bem como a acumulao e a produo de riquezas tm contribudo para uma melhor distribuio de renda e menor concentrao do capital nas mos de uma minoria da populao.

2 C

P R O V A

220

GEOGRAFIA
Questo 5 Com base em conhecimentos sobre a geografia da circulao, pode-se afirmar:

UFBA
F A S E

(1) As desigualdades regionais e intra-regionais geram fluxos constantes de mercadorias, capitais e populao, responsveis pela concretizao da atividade produtiva, tornando os espaos interdependentes e integrados. (2) As vias de circulao e os sistemas de transporte so manifestaes fsicas necessrias que possibilitam a visualizao do fluxo de bens, na paisagem. (4) Nos pases subdesenvolvidos, as redes de transporte so sempre perifricas e convergentes para os portos exportadores, refletindo maior interdependncia regional desses pases e promovendo maior integrao territorial. (8) O transporte rodovirio permite grande capacidade de carga, menor consumo proporcional de combustvel e menos dependncia do petrleo, tendo, portanto, custo operacional inferior ao do transporte ferrovirio e hidrovirio. (16) No Brasil, a substituio do modelo agroexportador por um modelo urbano-industrial, entre outros fatores, provocou a decadncia da rede ferroviria e deu prioridade ao transporte rodovirio, impulsionado pela criao da Petrobrs e pelo desenvolvimento da indstria automobilstica. (32) A maior densidade da rede de transportes, na faixa litornea brasileira, especialmente nos estados do Sudeste e do Sul, reflete a intensidade dos fluxos que ocorrem nessa regio. Questo 6 Com base em conhecimentos sobre o espao industrial, no Brasil e no mundo, pode-se afirmar: (1) A indstria de bens de consumo precedeu a indstria pesada nos pases industriais antigos, o que explica a distribuio preferencial daquele primeiro tipo de indstria junto s fontes de matrias-primas. (2) A industrializao tardia, tpica dos pases perifricos, tem como caractersticas principais: maior desenvolvimento das indstrias de base, predominncia de capitais estrangeiros e tecnologia prpria. (4) Um dos fatores marcantes relacionados industrializao brasileira, ao final do sculo XIX, foi a presena de imigrantes estrangeiros, que foram os primeiros trabalhadores assalariados do pas e os responsveis pelo aumento inicial do mercado consumidor brasileiro. (8) A concentrao industrial no Brasil Sudeste gerou uma ntida polarizao do desenvolvimento nessa regio, ao tempo em que acarretou desequilbrios nas demais regies do pas, numa reproduo das mesmas relaes internacionais existentes entre centro (pases desenvolvidos) e periferia (pases subdesenvolvidos). (16) A economia brasileira, especialmente no setor industrial, vem sofrendo, nesta ltima dcada, algumas transformaes tpicas do capitalismo monopolista: maior concentrao e centralizao do capital, construo de conglomerados e crescente participao do Estado nas atividades econmicas. (32) As indstrias de ponta eletrnica, informtica, biotecnologia, entre outras so geralmente localizadas longe dos ncleos metropolitanos, em subrbios afastados ou pequenas cidades, contrapondo-se aos fatores tradicionais de localizao industrial. (64) O Pas do Sol Nascente, devido vastido do seu espao geogrfico, forte concentrao e densidade populacional, ampla disponibilidade de tecnologia avanada e de recursos naturais abundantes, apresenta um crescimento econmico elevado, despontando como uma grande potncia neste final de sculo.

2 C

P R O V A

221

GEOGRAFIA
Questo 7

UFBA
F A S E

Com base em conhecimentos sobre os movimentos da Terra e suas conseqncias geogrficas, pode-se afirmar: (1) O movimento cclico e sazonal do mecanismo das mars advm da influncia exercida pela Lua sobre as massas ocenicas, principalmente durante o perodo das mars de quadratura, quando a posio do aflio da Lua, em relao Terra, provoca as mais fortes ressacas. (2) Duas localidades antpodas possuem latitudes e longitudes contrrias, doze fusos horrios de diferena e estaes do ano opostas, mas situam-se em um mesmo tipo de zona climtica. (4) Um turista, ao cruzar a Linha Internacional da Data precisamente zero hora, no sentido oeste-leste, deve encontrar, no seu destino, uma data anterior quela de sua partida. (8) Nas regies de alta latitude, o Sol forma, ao meio-dia, um ngulo reto em relao superfcie terrestre, durante o equincio de vero, perodo em que o dia tem a durao de seis meses consecutivos. (16) Se a inclinao da eclptica terrestre fosse ampliada de 2327 para 4027, haveria uma reduo das reas intertropicais, que continuariam com baixas presses atmosfricas anuais, enquanto, nas latitudes extratropicais, as reas de clima temperado se expandiriam. (32) Uma aeronave, deslocando-se no interior das latitudes intertropicais, em sentido contrrio ao movimento de rotao da Terra, deve encontrar sua frente uma forte resistncia dos ventos alsios e, conseqentemente, sua velocidade tende a ser reduzida. (64) As estaes do ano alternam-se, nos hemisfrios, em conseqncia da maior ou menor proximidade da Terra em relao ao Sol, como resultado do movimento de translao efetuado ao longo do ano. QUESTES 8 e 9 DOMNIOS MORFOCLIMTICOS

2 C

P R O V A

222

GEOGRAFIA
Questo 8

UFBA
F A S E

Associando-se a ilustrao aos conhecimentos sobre os domnios morfoclimticos, pode-se afirmar: (1) A denominao domnio morfoclimtico identifica uma paisagem natural que quase sempre transcende o limite espacial de uma regio geogrfica. (2) O regime fluvial do tipo tropical austral est relacionado ao clima tropical (Aw); j o domnio das formaes xerfilas, que recobre as superfcies de aplainamento, caracterstico dos climas marcados pela inexistncia de um perodo seco pronunciado. (4) A vegetao de transio entre os dois domnios fitogeogrficos evidencia a influncia de fatores climticos que proporcionam a ocorrncia de solos mais espessos e mais lixiviados por causa do aumento da umidade. (8) As veredas so vales midos que ocorrem no ambiente dos cerrados, representando um tipo de fitoambiente comumente denominado oueds, uma vez que seus rios so de fluxo perene. (16) A mata galeria corresponde a um tipo de formao arbrea que margeia os canais de drenagem submetidos aos climas afetados pela semi-aridez, recobrindo os solos ricos em aluvies. (32) O domnio dos cerrados pode ser individualizado em vrios subconjuntos naturais, vinculados diretamente s caractersticas do solo. (64) O relevo pouco influencia a natureza e espessura dos solos, e a estrutura geolgica possui pequena atuao no processo de elaborao dos modelados estruturais. Questo 9 Em relao aos domnios morfoclimticos apresentados, pode-se afirmar: (1) O domnio I representa superfcies planlticas, de solos laterticos recobertos por cerrados, submetidas a clima supermido com elevadas temperaturas mdias anuais. (2) O domnio II representa um setor susceptvel a um processo de degradao ambiental, causada pelas aes antrpicas sobre o meio natural e por longas estiagens. (4) O domnio II apresenta maior amplitude trmica diria e maior ndice de evapotranspirao que o domnio I, em virtude da cobertura vegetal e do clima. (8) Os rios que fluem do domnio I para o II apresentam, nas suas cabeceiras, maior volume de gua e maior dinmica fluvial. (16) Os regimes fluviais que ocorrem em I e II, apesar de estarem submetidos a regimes pluviais heterogneos, podem ocorrer em zonas climticas semelhantes. (32) Os domnios I e II caracterizam-se por chuvas irregulares, ao longo do ano, sendo que, em II, a durao e a intensidade do perodo seco so menores, o que favorece a ocorrncia de solos mais rasos. (64) O desnvel topogrfico de quase 300 metros marca a transio do domnio I para o II, apresentando, na borda do planalto, significativas modificaes ambientais, como reduo da intensidade das chuvas locais, cobertura vegetal mais compacta e temperaturas mais elevadas.

2 C

P R O V A

223

GEOGRAFIA
Questo 10

UFBA
F A S E

Com base nos conhecimentos sobre o ambiente costeiro e seus problemas ecolgicos, podese afirmar: (1) As dunas so formaes arenosas resultantes da acumulao elica, que, em virtude da remoo da vegetao e da utilizao inadequada por parte do homem, passam a se locomover gradativamente, chegando a ameaar edificaes e agrupamentos urbanos. (2) Os manguezais so formaes vegetais que se desenvolvem nos ambientes fluviomarinhos e geralmente recobrem as costas baixas, a exemplo dos esturios. (4) Os recifes de arenito existentes no litoral da Bahia elevam-se do fundo do mar, so recobertos por colnias de corais e ficam totalmente emersos durante a preamar. (8) As restingas so formaes arenosas construdas pela ao das ondas nas zonas de arrebentao, de solos pobres em argila e matria orgnica, que apresentam baixa capacidade para reter gua e nutrientes e que, a longo prazo, retificam a linha do litoral de baas e enseadas. (16) As praias so ambientes costeiros formados por areias e, em alguns casos, por cascalhos, o que evidencia intenso processo de acumulao marinha. (32) A especulao imobiliria e a expanso do turismo esto ameaando de destruio os ecossistemas costeiros do litoral ocidental do Nordeste brasileiro. (64) Entre os impactos ambientais que afetam o litoral brasileiro, destaca-se a mar negra, resultante da poluio derivada do lixo e de efluentes urbanos.

2 C

P R O V A

224

GEOGRAFIA
1. Analise a fotografia area.

UFMG
F A S E

1 A

P R O V A

A partir da anlise dessa fotografia, assinale a alternativa INCORRETA. a) A faixa central de tonalidade escura orientada em direo nordeste/sudoeste ocupada por vegetao de porte arbreo. b) A mudana fisionmica da vegetao, que ocorre de sul para norte, indica que a rea localiza-se em uma zona de transio climtica. c) A poro oeste da rea cortada por uma estrada de orientao norte/ sul, s margens da qual adensam-se as atividades humanas. d) A poro sudeste da rea caracterizada pela presena de cultivos agrcolas homogneos, os quais ocupam grande extenso.

225

GEOGRAFIA

UFMG
F A S E

2. Encarte um recurso cartogrfico que permite complementar a informao representada na imagem principal, qual se encontra associado. Analise as figuras onde so apresentadas as imagens A, B e C.

1 A

RELEVO, VEGETAO E DISTRIBUIO DOS POVOADOS NA REGIO DE GOURMA

P R O V A

Todas as alternativas contm afirmaes corretas sobre as imagens apresentadas em A, B e C, EXCETO a) A imagem apresentada em A mostra a posio geogrfica continental e nacional da rea cartografada em C. b) A imagem apresentada em B mostra a posio geogrfica regional da rea cartografada em C. c) A imagem apresentada em C mostra os aspectos geogrficos da rea cartografada, enunciados no ttulo da figura. d) As imagens A, B e C, dadas suas funes, foram construdas em escalas decrescentes.

226

GEOGRAFIA

UFMG
F A S E

3. Suponha que sejam 9 horas em Belo Horizonte e que voc, estando aqui, precisa planejar uma ligao interurbana para uma pessoa em Rio Branco, Acre, que poder ser encontrada, nessa cidade, s 11 horas, hora local desse Estado. Nesse caso, voc deve a) aguardar duas horas para fazer sua ligao porque no Brasil, embora sejam reconhecidos os limites tericos dos fusos horrios de 15 de longitude, consideram-se apenas os limites prticos definidos pelas fronteiras estaduais. b) aguardar que sejam 11 horas em Braslia, hora oficial do pas, que se aplica a toda a poro ocidental do territrio brasileiro. c) fazer sua ligao, imediatamente, porque o horrio do fuso em que se encontra o Estado de Minas Gerais de duas horas a menos em relao ao fuso da poro mais ocidental do Brasil. d) ligar s 13 horas, no horrio de Belo Horizonte, porque o horrio do fuso em que se encontra o Estado de Minas Gerais adiantado em duas horas em relao ao da poro mais ocidental do Brasil. 4. A instituio do horrio de vero no Brasil possvel tendo em vista a existncia de fatores que, conjugados, podem explicar por que grande parte do territrio brasileiro recebe mais luz e calor em determinado perodo do ano. Todas as alternativas apresentam esses fatores, EXCETO a) b) c) d) A inclinao do eixo terrestre, sempre voltado para a mesma direo. A grande extenso territorial do Brasil no sentido leste/oeste. A posio astronmica da Terra, no seu movimento de translao. A posio austral do pas, em relao ao Equador.

1 A

P R O V A

227

GEOGRAFIA

UFMG
F A S E

5. O grfico representa, para uma regio coberta por vegetao natural, o comportamento do escoamento superficial da gua a partir de um perodo de precipitao pluvial.

1 A

P R O V A

O grfico que caracteriza, corretamente, o comportamento do escoamento superficial da gua pluvial aps o desmatamento da vegetao natural e aps a urbanizao da regio,

228

GEOGRAFIA
6. Analise o mapa.

UFMG
F A S E

1 A

P R O V A

Os blocos diagramas I, II, III e IV representam, esquematicamente, a configurao das placas tectnicas nas reas assinaladas no mapa. Todas as representaes esto corretas, EXCETO

229

GEOGRAFIA
7. Observe o perfil da distribuio da vegetao nos Andes ao longo da latitude 16 S.

UFMG
F A S E

1 A

P R O V A

A partir da anlise desse perfil, assinale a alternativa INCORRETA. a) A altitude constitui um parmetro ambiental importante na distribuio vertical das diferentes formaes vegetais. b) O flanco ocidental andino, comparado ao flanco oriental, caracteriza-se pela ocorrncia de condies climticas mais ridas. c) O flanco oriental andino, comparado ao flanco ocidental, caracteriza-se por vales mais profundos e relevo mais recortado. d) O limite das nuvens, em ambos os flancos, caracterizado por vegetao de gramneas adaptada a severas condies de aridez. 8. Todas as caractersticas apresentam caractersticas das reas de cerrado no Brasil, EXCETO a) Ocorrncia de extensos campos midos que ocupam o topo dos chapades no Brasil Central e que encerram importantes mananciais de gua. b) Ocorrncia de solos profundos e cidos que, adequadamente corrigidos, podem apresentar elevada produtividade agrcola. c) Ocorrncia de uma biodiversidade reduzida expressa pela sua significativa homogeneidade fisionmica. d) Ocorrncia de uma biomassa subterrnea onde razes, rizomas e tubrculos so mais numerosos que a biomassa area de troncos e galhos.

230

GEOGRAFIA
9. Observe o mapa

UFMG
F A S E

1 A

P R O V A

Todas as alternativas apresentam caractersticas da rea em destaque nesse mapa, EXCETO a) Presena de temperaturas mdias anuais elevadas, pequena amplitude trmica, forte umidade relativa do ar e intensa nebulosidade. b) Presena de terras firmes ou baixos planaltos, os quais apresentam uma topografia montona caracterizada por amplos interflvios e colinas tabulares. c) Presena de tipos diferenciados de formaes florestais pereniflias em funo de variaes climticas, topogrficas e variaes de solo e de hidrografia. d) Presena de uma paisagem fluvial modelada em funo de canais caracterizados por trechos encachoeirados e corredeiras, onde a gua escoa em alta velocidade.

10. No Brasil, a criao de novas fronteiras agrcolas e a expanso de antigas reas agricultveis tm provocado graves problemas ambientais. Sobre esses problemas, assinale a afirmativa INCORRETA. a) A expanso da agricultura e da pecuria extensiva no norte do pas, ao longo das principais rodovias e em torno das reas urbanas, tem reduzido as reservas extrativistas. b) A expanso das reas de monocultura modernizada na regio Centro-Oeste tem levado exausto do solo e contaminao dos recursos hdricos. c) A expanso da cultura cafeeira no vale do Rio Paraba do Sul, atravs da queima de remanescentes da Mata Atlntica, tem aumentado a concentrao de gs carbnico na atmosfera. d) A expanso dos reflorestamentos de eucalipto na poro centro/norte de Minas Gerais tem levado ao assoreamento e alterao da vazo dos cursos de gua.

231

GEOGRAFIA
11. Observe no mapa as reas assinaladas.

UFMG
F A S E

1 A

P R O V A

Em relao a essas reas, correto afirmar que nelas predominam a) b) c) d) culturas comerciais, grandes e mdias propriedades. culturas de subsistncia e minifndios. culturas irrigadas e microempresas agrcolas. policultura e pequenas propriedades familiares.

12. Todas as alternativas apresentam concluses corretas sobre os resultados do Censo/91, EXCETO a) Aumentou o percentual de indivduos com idade superior a 65 anos na populao. b) Houve um aumento da participao relativa das populaes das regies Sudeste e Nordeste. c) Houve uma reduo percentual dos trabalhadores rurais na formao da PEA (Populao Economicamente Ativa). d) Persistiram, ainda que enfraquecidos, os fluxos migratrios em direo s regies Norte e Centro-Oeste.

232

GEOGRAFIA

UFMG
F A S E

13. Todas as alternativas apresentam caractersticas corretas do Vale do Jequitinhonha em Minas Gerais, EXCETO a) Apresenta nveis de misria, analfabetismo e mortalidade infantil que superam, significativamente, as mdias nacionais. b) Enfrenta o esgotamento das reservas minerais, o que expulsa os grandes capitais pblicos e privados tradicionalmente investidos na regio. c) Serve-se da gua do rio que lhe d nome, secularmente agredido pela presena dos garimpos s suas margens. d) Vem perdendo parte da sua populao ativa rural que se emprega como bia-fria nas reas de agricultura comercial importante.

1 A

P R O V A

14. Assistiu-se no pas, nas ltimas dcadas, a um movimento migratrio sem precedentes, que foi alvo de investigao por parte do Itamaraty. Sobre esse movimento, assinale a alternativa INCORRETA. a) A entrada de brasileiros em alguns pases facilitada por ancestrais que alimentaram, no passado, correntes no sentido inverso. b) A entrada de brasileiros em outros pases garantida por diversos fatores entre os quais se inclui a ausncia de restries imigrao de naturais desses pases no territrios nacional. c) A transformao do Brasil em pas de emigrao se deu recentemente e teve, como alvo preferencial de destino, o prprio continente em que o pas est localizado. d) O levantamento foi comprometido pelo carter de clandestinidade que marca a permanncia de muitos brasileiros em diversos pases.

15. De acordo com dados da ONU, at o final do sculo, mais da metade da populao do mundo viver em cidades. Sobre o processo que levar expanso da populao mundial urbana, assinale a alternativa INCORRETA. a) Na frica, a urbanizao tem atingido ndices sem precendentes, em parte como resultado da ampliao da rea de seca que limita os espaos cultivveis. b) Na Amrica Latina, experimenta-se uma urbanizao intensa, em parte como resultado de se concentrarem, espacialmente, os escassos recursos econmicos. c) Na sia, vem se registrando um processo de urbanizao lento, em parte como resultado das campanhas governamentais de incentivo ocupao das terras agrcolas. d) Na Europa, o aumento da populao das cidades tem sido insignificante porque o crescimento demogrfico baixo e a urbanizao, um processo muito antigo.

233

GEOGRAFIA
16. Leia o texto
A Era de Ouro, perodo que compreendeu, grosso modo, os anos 50, 60 e 70, caracterizava-se por precisar, cada vez mais, de macios investimentos e, cada vez menos, de gente, a no ser como consumidores. Contudo, o mpeto e a rapidez do surto econmico foram tais que, durante uma gerao, isso no foi bvio. Pelo contrrio, a economia cresceu to depressa que, mesmo nos pases industrializados, a classe operria industrial se manteve ou aumentou. Em todos os pases avanados, novos contingentes de mo-de-obra foram atrados da zona rural e da imigrao estrangeira. Mulheres casadas, at ento mantidas fora do mercado de trabalho, entraram nele em nmero crescente. Apesar disso, o ideal a que a Era de Ouro aspirava s se realizaria aos poucos. Esse ideal era a produo, ou mesmo o servio, sem seres humanos, que s eram essenciais para tal economia num aspecto: como compradores de bens e servios. A estava seu problema central.

UFMG
F A S E

1 A

P R O V A

Adaptado de: HOBSBAWN, E. Era dos Extremos. O Breve Sculo XX: 1914-1991.So Paulo: Companhia das Letras, 1995. p. 262.

Todas as alternativas contm afirmaes que podem ser comprovadas pelo texto, EXCETO a) A utilizao de tecnologias que necessitam de elevado emprego de capital e pouca mode-obra uma caracterstica atribuda economia da Era de Ouro. b) O extraordinrio crescimento econmico da Era de Ouro ampliou a composio da mo-de-obra das economias mais avanadas, que passou a incluir mulheres casadas e estrangeiras. c) O problema central da economia da Era de Ouro consistiu na incapacidade de as tecnologias avanadas atenderem o aumento de consumo de novos contingentes de mo-de-obra. d) O reconhecimento de que os seres humanos perdiam sua importncia como produtores, na Era de Ouro, foi encoberto pelo notvel crescimento da economia da poca.

17. O processo de globalizao est associado a mudanas de natureza distinta no mundo como um todo. Todas as alternativas apresentam modificaes decorrentes desse processo, EXCETO a) b) c) d) Formao de grandes blocos econmicos regionais. Maior segurana em relao melhoria dos padres sociais. Rpida movimentao de capitais internacionais por todo o mundo. Reduo do poder do Estado na proposio de polticas econmicas internas.

234

GEOGRAFIA

UFMG
F A S E

18. De acordo com os dados recentemente publicados pela ONU, de todas as regies do mundo, a menos afetada pela crise de desemprego a sia, onde, apesar de essa crise ter crescido nos ltimos 15 anos, o ndice de desemprego continua baixo. Essa situao vantajosa da sia, em relao s demais regies, explica-se por diversos fatores. Todas as alternativas apresentam exemplos desses fatores, EXCETO a) A queda do crescimento populacional da sia, nos anos 70, reflete-se numa insuficiente oferta de fora de trabalho. b) O crescimento econmico dos Tigres Asiticos atraiu capital tambm para as reas vizinhas, ampliando as oportunidades de emprego. c) O prestgio da mo-de-obra oriental, fundamentada em aspectos de educao e disciplina, atraiu investimentos multinacionais. d) Os sindicatos existentes na regio atuam muito mais no sentido de aumentar a produtividade do que propor reivindicaes.

1 A

P R O V A

19. Em relao Comunidade de Estados Independentes (CEI), assinale a afirmativa INCORRETA. a) A CEI, criada com a finalidade de resolver as questes de fronteiras do antigo bloco sovitico, transformou-se ao longo do tempo e hoje um organismo de integrao econmica do tipo mercado comum. b) Apenas trs das antigas repblicas soviticas, as localizadas na regio do Mar Bltico, recusaram-se a participar da Comunidade de Estados Independentes. c) As desigualdades existentes entre os participantes da CEI so grandes, tanto no que diz respeito renda per capita quanto ao nvel de industrializao e valor do Produto Nacional Bruto (PNB). d) Muitos dos pases integrantes da CEI vm enfrentando problemas econmicos graves, como o elevado dficit da balana comercial e o crescimento negativo ou quase nulo da produo.

20. O final da dcada de 90 tem sido caracterizado pelo surgimento e pela expanso de novas epidemias infecto-contagiosas em todo o globo, motivadas, em grande parte, por fatores de ordem geogrfica. Todas as alternativas apresentam fatores que contribuem para a proliferao dessas epidemias, EXCETO a) A expanso da faixa tropical em direo s baixas latitudes, em funo do aquecimento global, o que tem favorecido o surgimento de epidemias tipicamente tropicais em regies temperadas. b) A expanso de reas urbanas e agroindustriais, o que tem reduzido a biodiversidade global e eliminado os predadores naturais de muitos vetores de doenas infectocontagiosas. c) A exploso demogrfica em alguns pases que, aliada ao crescimento da pobreza em escala global, tem auxiliado o colapso dos sistemas de sade pblica. d) A intensificao do processo de globalizao da economia, o que tem ocasionado uma grande mobilidade da populao e de mercadorias.

235

GEOGRAFIA
1. Analise a fotografia area.

UFMG
F A S E

1 B

P R O V A

A partir dessa anlise, assinale a alternativa INCORRETA. a) A expanso das reas urbanas tem sido efetivada sobretudo ao longo das margens das principais vias de acesso regio. b) A rea urbana localiza-se em zona de risco, estando sujeita a inundaes que podem invadir ruas e residncias. c) As atividades agrcolas desenvolvidas na regio foram implantadas sobre uma ampla plancie fluvial. d) As margens do rio que drena a regio so revestidas por mata galeria na qual predominam formaes de porte arbreo.

236

GEOGRAFIA
2. Observe as representaes grficas apresentadas em A e B.

UFMG
F A S E

1 B

P R O V A

237

GEOGRAFIA

UFMG
F A S E

1 B

P R O V A

O mapa topogrfico, como o mostrado em A, e a planta baixa, como a mostrada em B, tm como exigncia comum para sua construo, a) a considerao da curvatura da superfcie terrestre, a partir de um sistema de projeo. b) a observao da dimenso real dos elementos representados, utilizando uma escala. c) a considerao da posio dos elementos cartografados, atravs de um sistema universal de referncia, as coordenadas geogrficas. d) a representao de caractersticas do relevo, como a altitude e a declividade, atravs das curvas de nvel e dos pontos cotados.

3. Sobre os fusos horrios, INCORRETO afirmar que, a) no Brasil, como em qualquer outro pas do Ocidente, a hora legal est adiantada em relao hora legal dos pases do Oriente. b) no Brasil, como no mundo inteiro, os fusos horrios so definidos em relao ao fuso horrio inicial que, por conveno, o delimitado pelo meridiano de Greenwich. c) no Brasil e em muitas outras reas do globo, os limites tericos so substitudos por limites prticos que levam em conta fronteiras polticas. d) no Brasil e em muitos outros pases do globo, as diferenas de fusos horrios devem-se grande extenso longitudinal de seus territrios.

238

GEOGRAFIA

UFMG
F A S E

4. O Brasil tem institudo o horrio de vero que, no entanto, no abrange todo o territrio nacional. Os Estados brasileiros excludos do horrio de vero so aqueles a) situados em reas de altos ndices pluviomtricos e de nebulosidade, que ocasionam fraca luminosidade em todas as estaes do ano. b) situados em reas de grande altitude, que registram temperaturas absolutas inferiores mdia nacional em todas as estaes do ano. c) situados na poro equatorial, onde a variao da durao diria do perodo claro imperceptvel nas diferentes estaes do ano. d) situados na poro temperada do pas, onde a durao diria do perodo escuro maior do que a do perodo claro em todas as estaes do ano.

1 B

P R O V A

5. Este desenho representa, esquematicamente, o mecanismo envolvido durante um fenmeno de precipitao orogrfica. A, B, C e D: posies da massa de ar Os grficos I, II, III e IV representam as variaes de altitude e temperatura sofridas pela massa de ar em sua trajetria sobre o continente. O grfico que representa corretamente essas alteraes a) Grfico I b) Grfico II

c) Grfico III

d) Grfico IV

A, B, C e D: posies da massa de ar

239

GEOGRAFIA
6. Leia o texto.
Embora a evidncia de deslocamentos laterais dos continentes fosse mais ou menos forte, a maioria dos gelogos resistiu, durante muito tempo, idia desses deslocamentos. Essa resistncia era, em grande parte, ideolgica, a julgar pela extraordinria ira da controvrsia contra o principal proponente da deriva continental, Alfred Wegener. De qualquer modo, o argumento de que esses deslocamentos no eram verdadeiros porque no se conhecia nenhum mecanismo geofsico para causar tais movimentos no era mais convincente a priori, em vista da evidncia acima referida. Contudo, desde a dcada de 1960, o antes impensvel tornou-se a ortodoxia da geologia do dia-a-dia: um globo de placas gigantescas mudando de lugar, s vezes, rapidamente (placas tectnicas).

UFMG
F A S E

1 B

P R O V A

Adaptado de: HOBSBAWN, E. Era dos Extremos. O Breve Sculo XX: 1914-1991. So Paulo: Companhia das Letras, 1995. p. 530.

Todas as afirmativas contm afirmaes que podem ser comprovadas pelo texto, EXCETO a) A teoria da deriva continental foi, por muito tempo, considerada inaceitvel por se desconhecer o mecanismo geofsico que pudesse explic-la. b) A teoria das placas tectnicas considerada, atualmente, a explicao mais aceitvel e defensvel sobre a posio das massas continentais e a configurao da litosfera. c) As evidncias de que as terras emersas se deslocavam lateralmente sugeriram a teoria segundo a qual a litosfera era formada por vrias placas, em vez de uma nica, imvel sobre o manto. d) O relato sobre a aceitao de uma nova teoria sugere que observaes, embora inexplicveis pelo conhecimento cientfico de uma poca, so prontamente aceitas pelos cientistas. 7. Observe, no mapa ao lado, a diviso do relevo brasileiro, sistematizada a partir de levantamentos realizados pelo Projeto RADAMBRASIL. Todas as alternativas apresentam alteraes introduzidas por essa diviso EXCETO a) Ampliao do nmero de unidades do relevo. b) Aumento na extenso das reas de plancie. c) Diminuio da extenso de algumas reas de planalto. d) Introduo das depresses como unidades de relevo.

240

GEOGRAFIA
8. Analise o perfil.

UFMG
F A S E

1 B

P R O V A

O mapa apresenta a localizao de quatro sees (I, II, III e IV).

As caractersticas da vegetao natural reproduzidas no perfil esto presentes ao longo da seo a) I b) II c) III d) IV

241

GEOGRAFIA
9. Observe o mapa

UFMG
F A S E

1 B

P R O V A

Todas as alternativas caracterizam a rea em destaque nesse mapa, EXCETO a) Atuao expressiva do Estado e da iniciativa privada atravs da implantao de grandes projetos agropecurios baseados, em sua grande maioria, em modernos sistemas de irrigao. b) Desenvolvimento de um sistema de transporte hidroferro-rodovirio orientado segundo um eixo norte-sul, fundamental integrao do espao nordestino ao centro/sul do pas. c) Ocorrncia de uma rede de drenagem com fraco encaixamento fluvial que, de modo geral, constituda por rios intermitentes subordinados bacia do Rio So Francisco. d) Presena de um relevo plano, topograficamente deprimido, limitado a leste e a oeste por planaltos que se configuram regionalmente como importantes divisores hidrogrficos. 10. Todas as alternativas sobre a agricultura brasileira esto corretas, EXCETO a) A agricultura moderna e a rudimentar coexistem e utilizam tcnicas que agridem o meio ambiente. b) A capitalizao do campo ocorreu com fortes desigualdades nas diversas regies do pas. c) A maior parte dos solos frtil, mas a obteno de insumos qumicos, facilitada pelo baixo custo, induz ao uso desnecessrio desses produtos. d) O aumento da produo agrcola se deu mais pela ampliao da fronteira que por ganhos de produtividade.

242

GEOGRAFIA

UFMG
F A S E

11. Sobre a evoluo da economia brasileira nos ltimos anos, assinale a alternativa INCORRETA. a) A terceirizao de atividades, adotada pelas empresas, reduz percentualmente a participao do setor secundrio na gerao de empregos e de renda. b) O crescimento do setor informal da economia registrado com profundidade e importncia menores do que a forma em que vem ocorrendo. c) O crescimento negativo do PIB pode ser corrigido se, a exemplo de pases mais desenvolvidos, a indstria voltar a comandar o crescimento. d) O setor de servios passou a apresentar melhor desempenho garantindo a maior parte do crescimento da economia.

1 B

P R O V A

12. Leia a afirmativa Na dcada atual, as grandes cidades brasileiras apresentaram as mais elevadas taxas de crescimento e se constituram, portanto, nas principais responsveis pelo aumento da populao urbana. Essa afirmativa est a) CORRETA, pois as grandes metrpoles continuam a atrair a maior parte dos habitantes que abandonam o meio rural e mostram, ainda, tendncia ao aumento da taxa de crescimento nos prximos anos. b) CORRETA, pois, enquanto as demais categorias de cidades apresentaram um aumento relativo de habitantes pouco significativo, as grandes cidades registraram taxas de crescimento superiores a todas as expectativas levantadas. c) INCORRETA, pois, ao contrrio dos anos 80, quando os grandes centros foram os que mais cresceram e contriburam para o aumento da populao urbana, na atual dcada, as cidades mdias assumiram esses papis. d) INCORRETA, uma vez que, nos ltimos anos, houve um decrscimo da populao das megacidades brasileiras, resultante do grande nmero de pessoas que deixou essas cidades em razo da violncia.

13. Projees da ONU e do Instituto de Pesquisa Econmica Aplicada (IPEA) sobre o perfil demogrfico do Brasil, no prximo sculo, indicam a necessidade de se rever a poltica de empregos e os investimentos em educao, sade e previdncia social. Todas as afirmativas contm dados corretos do perfil demogrfico do Brasil no prximo sculo, EXCETO a) A populao ativa continuar a crescer em ritmo acelerado, o que implicar na queda drstica do valor dos salrios. b) O crescimento vegetativo dever manter tendncia de queda, o que poder representar diminuio dos encargos com a populao mais jovem. c) O nmero de pessoas idosas prosseguir aumentando, o que exigir maiores investimentos na rea de sade e na previdncia social. d) O ritmo de crescimento da populao escolar continuar mais lento, o que dever significar diminuio da necessidade de expanso da rede escolar.

243

GEOGRAFIA

UFMG
F A S E

14. Todas as alternativas apresentam aspectos que evidenciam a progressiva consolidao do Mercosul, EXCETO a) A ampliao da rea geogrfica de gerao do PIB (Produto Interno Bruto). b) A expanso do raio de consumo da produo dos centros mais dinmicos dessa organizao. c) O aumento do fluxo de veculos e a diversificao da rede de transportes. d) O crescente controle da economia pelo setor agropecurio registrado nos ltimos anos.

1 B

15. A situao atual do continente africano, objeto de preocupao de diversos organismos internacionais, est corretamente caracterizada em todas as alternativas, EXCETO em a) Incapacidade de atrair investimentos estrangeiros que se dirigem a outras regies menos desenvolvidas do globo onde a mo-de-obra mais qualificada. b) Intensificao do bloqueio econmico imposto pelos grandes importadores de produtos primrios, que visam forar mudanas nas polticas sociais internas. c) Intensificao dos conflitos tribais explicada, entre outros fatores, pelo acirramento da disputa pela posse dos meios de sobrevivncia. d) Intensificao dos desmatamentos, o que tem provocado uma expanso da rea de incidncia da seca.

P R O V A

16. Sobre os movimentos nacionalistas/separatistas, verificados a partir da dcada de 70, INCORRETO afirmar que, a) em pases da Europa, como Reino Unido, Espanha, Itlia, e no Canad, esses movimentos tm ocorrido sem, todavia, resultarem na formao de novos Estados. b) na Europa e no Canad, eles tm sido estimulados pelos organismos supranacionais de integrao econmica e poltica como a Unio Europia e o Nafta, desejosos de limitar o nmero de participantes. c) na Europa Oriental e na sia Central, esses movimentos foram fortalecidos e eclodiram aps a crise do comunismo e a dissoluo da URSS, originando novos Estados. d) na Europa Oriental e na sia Central, eles resultam, sobretudo, do enfraquecimento do modelo de estado multitnico e do fortalecimento dos movimentos tnico-nacionalistas.

17. possvel afirmar que o processo de globalizao das atividades econmicas tem acompanhado a histria da humanidade e que ele se acelerou aps a dcada de 70. Todas as alternativas apresentam caractersticas ou indicadores do processo de globalizao das atividades econmicas neste final de sculo, EXCETO a) A acelerao das transformaes demogrficas. b) A revoluo ocasionada pelos avanos da informtica. c) O aperfeioamento dos meios de transporte e das comunicaes. d) O desenvolvimento tecnolgico e cientfico.

244

GEOGRAFIA

UFMG
F A S E

18. Considerado como um dos males do final do sculo, o desemprego atinge hoje sociedades com distintos nveis de desenvolvimento econmico. Sobre a questo do desemprego, todas as afirmativas esto corretas, EXCETO a) A adoo pelas grandes empresas da prtica da terceirizao de atividades tem mudado a distribuio espacial do desemprego. b) A disperso geogrfica do desemprego interfere na propenso dos grupos humanos a migrar. c) A falta de emprego, embora em intensidade diferente, tem sido alvo de preocupao tanto dos que dele precisam quanto dos que o promovem. d) O ndice de desemprego tem sido alimentado pela falta de investimento em tecnologia e modernizao das empresas.

1 B

P R O V A

19. O ndice de Desenvolvimento Humano (IDH), criado pela ONU, avalia com mais preciso as condies humanas e sociais de um pas. Assinale a alternativa que apresenta os indicadores utilizados na determinao do IDH. a) b) c) d) Consumo de calorias, taxa de analfabetismo, nmero de mdicos por 1.000 habitantes. Renda per capita, escolaridade, taxa de alfabetizao entre adultos e expectativa de vida ao nascer. Taxa de fecundidade, PIB por habitante, populao empregada no setor primrio. Taxa de mortalidade infantil, valor do salrio mnimo, taxa de natalidade.

20. Observe no mapa a distribuio geogrfica, por regies, do nmero de pessoas infectadas com o vrus HIV. Considerando as caractersticas geogrficas das regies mostradas no mapa, assinale a alternativa INCORRETA. a) Nas regies que se destacam, no mundo, pela grande concentrao demogrfica e urbana, a AIDS se manifesta de forma mais intensa. b) Nos pases socialistas ou naqueles que at recentemente adotavam esse regime, a AIDS se manifesta de forma menos intensa. c) Nas regies altamente industrializadas e desenvolvidas, assim como naquelas cujas economias ainda dependem do setor primrio, a AIDS est presente. d) Nas reas de climas frios e temperados assim como nas zonas climticas tropicais e equatoriais, h incidncia da doena.

245

GEOGRAFIA
QUESTO 01 Analise o mapa e execute as tarefas indicadas.

UFMG
F A S E

1. TRACE o perfil topogrfico X-Y no local indicado, utilizando a escala vertical apresentada.

2. Sabendo que a rea representada caracterizada pela presena de duas bacias hidrogrficas (Bacia A e Bacia B), TRACE no mapa, utilizando a simbologia indicada na legenda, os seguintes elementos: rede hidrogrfica divisor hidrogrfico entre as bacias A e B

246

GEOGRAFIA
QUESTO 02 Analise o grfico

UFMG
F A S E

Os intervalos A, B, C e D, delimitados no grfico, caracterizam quatro diferentes regies do globo segundo os parmetros de temperatura mdia, precipitao e profundidade da rocha alterada. IDENTIFIQUE, para cada uma das regies, o tipo climtico e a cobertura vegetal. 1. Regio A Tipo climtico: Cobertura vegetal: 2. Regio B Tipo climtico: Cobertura vegetal: 3. Regio C Tipo climtico: Cobertura vegetal: 4. Regio D Tipo climtico: Cobertura vegetal:

247

GEOGRAFIA
QUESTO 03 Observe, no mapa, duas importantes regies agrcolas do Brasil, identificadas pelas letras A e B.

UFMG
F A S E

1. CITE o nome dessas regies. Regio A: Regio B: 2. CARACTERIZE o tipo de uso do solo agrcola da regio A.

3. APRESENTE duas mudanas que ocorreram recentemente no uso do solo agrcola da regio B. Mudana 1: Mudana 2:

QUESTO 04 Observe as duas divises regionais do Brasil mostradas nos mapas abaixo.

1. CITE um critrio comum s duas divises regionais. 2. IDENTIFIQUE um estado brasileiro cuja evoluo, nas ltimas dcadas, tenha provocado a sua incluso em uma regio de menor nvel de desenvolvimento. 3. CITE uma razo dessa incluso.

248

GEOGRAFIA
QUESTO 05

UFMG
F A S E

Pelo menos durante uma dcada, o Brasil conseguiu, por anos consecutivos, um supervit comercial relativamente elevado. Entretanto, nos ltimos anos, o saldo comercial brasileiro no tem conseguido se manter positivo nem mesmo por alguns meses consecutivos. Parece que os longos perodos de altos supervits comerciais dificilmente voltaro a fazer parte do cenrio econmico do pas. Razes diversas, tanto de carter interno como externo, explicam as mudanas recentes na balana comercial brasileira. APRESENTE duas razes, uma de carter interno, e outra de carter externo, das mudanas recentes da balana comercial brasileira. EXPLIQUE-AS. 1. Razo interna: Explicao: 2. Razo externa: Explicao:

QUESTO 06 O Chile est sendo visto como um dos pases latino-americanos que primeiro se livrar dos problemas do subdesenvolvimento. CITE trs caractersticas do atual quadro econmico do Chile que permitem fazer esta previso.

QUESTO 07 Atualmente muitas unidades polticas esto estabelecidas em pequenos territrios e possuem uma populao pouco numerosa. Entre essas unidades, consideradas sem expresso econmica ou poltica at os anos 50, algumas tm florescido tanto ou mais que as grandes economias nacionais, oferecendo recursos e servios diretamente economia global e apresentando bons indicadores demogrficos e sociais. Essas unidades podem ser encontradas em todos os continentes, mas algumas das situadas na Amrica Central, nas Pequenas e Grandes Antilhas, so particularmente exemplares.

1. CITE uma atividade terciria desenvolvida nas pequenas unidades polticas da Amrica Central e JUSTIFIQUE o desenvolvimento dessa atividade. Atividade Terciria: Justificativa:

249

GEOGRAFIA
QUESTO 08

UFMG
F A S E

A partir dos anos 80, um nmero cada vez maior de confrontos armados tem eclodido ou recrudescido em muitas partes da Europa, sia e frica. Essas lutas no tm sido relacionadas ou no puderam ser relacionadas com o embate das duas superpotncias, EUA e URSS, tal como acontecia durante a vigncia da Guerra Fria.

1. APRESENTE quatro fatores que, genericamente, podem ser apontados como motivadores dos conflitos atuais. Fator 1: Fator 2: Fator 3: Fator 4: 2. APRESENTE duas razes que justifiquem o fato de muitos pases com poderio econmico, poltico e militar no intervirem definitiva e militarmente, para regrar muitos dos contlitos armados do mundo atual, limitando-se, na maior parte das vezes, ao assistencialismo e tentativa de elaborao e de manuteno de acordos que visam paz.

250

GEOGRAFIA

UFPA
F A S E

1. Atravs das polticas de populao, os pases tm controlado seus crescimentos populacionais, da entender-se por que a) pases receptores como os da Europa Ocidental (Portugal, Inglaterra e Frana) tm facilitado atualmente a entrada de imigrantes. b) alguns continentes, como o americano e o australiano, tiveram seus povoamentos garantidos pelas migraes internacionais na poca contempornea. c) em pases populosos como na ndia e no Paquisto, os efeitos de punes migratrias so facilmente perceptveis. d) pases de emigrao, como os EUA, tm estimulado a sada de contingentes populacionais a fim de aliviar altas densidades de regies mais nobres. e) a queda do muro de Berlim e a desestruturao da ex-URSS no tm interferido na migrao internacional na Europa Ocidental. 2. A produo energtica est estreitamente associada s inovaes tecnolgicas, podendo-se afirmar que a) os pases que apresentam maior nmero de usinas hidreltricas e rios de alto potencial hidreltrico so: EUA, Brasil, Rssia e Canad. b) os investimentos dos pases produtores em energia nuclear, a partir da II Guerra, tm sido mais reduzidos do que os de outras fontes energticas. c) a partir da dcada de 80, com a grande crise do petrleo e com a elevao de seu preo, novas alternativas tm sido pesquisadas e testadas. d) o carvo ainda continua sendo a principal fonte de energia consumida mundialmente, apesar da produo de petrleo ter sido crescente. e) a utilizao de biodigestores tem constitudo uma das principais alternativas para a complementao energtica de pases como EUA, China e ndia. 3. Com relao produo e apropriao dos recursos minerais, verifica-se que a) a produo mineral mundial encontra-se atualmente pouco vinculada chamada indstria de bens intermedirios, como as siderrgicas e metalrgicas. b) os recursos minerais so extrados principalmente da hidrosfera, embora haja grande concentrao de minerais na litosfera. c) no existe relao direta entre a presena de bauxita e a produo de alumnio, pois pases, como a Guin e a Jamaica, possuem bauxita e no entanto, no produzem alumnio. d) o Banco Mundial tem incentivado e financiado a indstria siderrgica em pases como o Canad, Japo e EUA, favorecendo fornecedores de equipamentos e mquinas industriais. e) processos industriais complexos e de alta tecnologia para o enriquecimento do urnio tm sido praticados comercialmente pelos EUA, Brasil e Japo.

251

GEOGRAFIA

UFPA
F A S E

4. A expanso do capitalismo no campo tem se manifestado atravs de processos industriais de produo. Podemos, ento, destacar a(s) seguinte(s) caracterstica(s): a) reduo da produtividade do trabalho e crescente tendncia policultura. b) processo produtivo cada vez mais natural, pois o interesse do capital potencializar o seu ritmo. c) tendncia das propriedades serem menores e da produo ser menos especializada. d) produo agrcola voltada para exportao pressupe grande compromisso com o mercado interno. e) expanso do uso da racionalidade capitalista como fundamento da sobrevivncia da explorao agrcola. 5. A organizao espacial, assim como a sociedade brasileira, refletem marcas profundas decorrentes do processo de colonizao, da observar-se que: a) b) c) d) as densidades demogrficas, no litoral, so to grandes quanto as do interior. a maioria das Regies Metropolitanas se localizaram na fachada atlntica. a economia brasileira sempre se manteve independente dos centros mundiais do capitalismo. os solos mais frteis so utilizados para a produo de alimentos para a populao e no de gneros para exportao. e) o surgimento de ilhas de povoamento foi verificado no litoral, em funo da ocorrncia dos recursos minerais que l estavam alocados. 6. Quanto aos Domnios Morfoclimticos Brasileiros, grandes unidades de paisagens naturais brasileiras, podemos dizer que: a) o Domnio Amaznio, ou a Amaznia, tem apresentado uma ocupao racional, com baixos ndices de desmatamento florestal. b) o Domnio do Cerrado possui solos de grande fertilidade natural, nos quais tem se mantido predominantemente a agricultura de subsistncia. c) no Domnio da Caatinga so freqentes os inselbergs e sua ocupao tem se realizado pela prtica da pecuria extensiva. d) o Domnio das Araucrias apresenta solos pobres e constitui um dos poucos domnios que mantm sua floresta isenta do desmatamento antrpico. e) no Domnio das Pradarias destacam-se as coxilhas, nas quais a ocupao econmica efetivou-se atravs da minerao e cultura algodoeira.

252

GEOGRAFIA

UFPA
F A S E

7. As migraes internas no Brasil vm ocorrendo desde a poca colonial, entretanto, elas tm se intensificado principalmente aps a I Guerra Mundial. Da dizer-se que: a) as migraes inter-regionais mais numerosas foram as de nordestinos e mineiros para as grandes cidades do Centro-Sul. b) as secas continuam sendo o principal elemento de explicao para o refluxo populacional nordestino que se origina principalmente no Serto. c) a modernizao e a mecanizao da agricultura so fatores que tm sido considerados desprezveis para o entendimento do xodo rural. d) o latifndio por sua vez, muito presente nas regies Nordeste e Sul, tem sido incuo no direcionamento de fluxos migratrios para a Amaznia e o Centro-Oeste. e) as migraes pendulares tm tido um crescimento inverso ao das cidades, sendo consideradas inexpressivas nos grandes centros. 8. O capitalismo selvagem e dependente predominante no Brasil e tem se manifestado no meio ambiente atravs das seguintes caractersticas: a) as devastaes florestais so praticamente inexistentes na Amaznia, onde se observa que a primeira natureza mantm-se intacta. b) as metrpoles ainda apresentam baixos ndices de poluio, pois possuem sistemas de controle de emisso de gases das chamins e escapamentos de veculos. c) as reas verdes nas grandes e mdias cidades, correspondem s mdias internacionais de 16m2 por habitante, como as de muitas cidades europias. d) as reas rurais tm sido poupadas dos problemas ambientais, especialmente aquelas que passam por um processo de modernizao agrria. e) os intensos desmatamentos, as queimadas, a monocultura e a pecuria tm levado desertificao de reas agrcolas principalmente no Nordeste e no Rio Grande do Sul. 1. QUEDA DE BRAO ENTRE GIGANTES REDEFINE ECONOMIA GLOBAL
Durante cem anos, o eixo da economia mundial repousou sobre o Atlntico Norte, o mar interior que conecta os Estados Unidos Europa Ocidental. H duas dcadas se deslocou para a Bacia do Pacfico... (Jornal O Mundo Setembro de 1995)

F A S E

Considerando as informaes acima e seu conhecimento sobre o processo de transformao da economia global, relacione e explique dois fatos que justificam esse deslocamento do eixo econmico.

253

GEOGRAFIA

UFPA
F A S E

2. O nvel de desenvolvimento scio-econmico dos pases latino-americanos no igual, porm alguns desses pases possuem caractersticas comuns, como o caso do Brasil e Mxico, destacados no planisfrio abaixo.

Explique trs dessas caractersticas.

3. Pedimos um mundo sem impiedosos bloqueios, que causam a morte de homens, mulheres e crianas,
jovens e idosos, como bombas atmicas silenciosas. (Trecho do discurso de Fidel Castro na Assemblia Geral da ONU em outubro de 1995)

Faa uma anlise sucinta da temtica acima. Explique em sua anlise: a) O fato ao qual se refere o lder cubano. b) A relao existente entre a atual crise econmica de Cuba e a desintegrao da URSS. 4. As indstrias brasileiras possuem irregular distribuio no espao social. O Centro-Sul do pas, em especial o Estado de So Paulo, apresenta uma grande concentrao industrial. A razo desta forte concentrao tem suas origens no processo de industrializao do Brasil e pode ser explicada atravs de vrios fatores que, conjugados, contriburam para o surgimento do maior parque industrial da Amrica Latina. A partir do texto e de seus conhecimentos sobre o assunto explique dois fatores importantes no processo de industrializao do Centro-Sul e particularmente do Estado de So Paulo.

254

GEOGRAFIA
5. Quem quer democracia tem que querer reforma agrria

UFPA
F A S E

(VEIGA, Jos Eli. O que reforma agrria.)

O texto, o mapa e a charge se referem a uma das questes mais importantes da atualidade brasileira, a Reforma Agrria, da qual fazem parte os seguintes personagens: GovernoLatifundirios e Trabalhadores rurais sem terra. a) No mapa esto assinaladas e numeradas as principais reas dos atuais conflitos enfrentados pelo Movimento dos Sem-Terra (MST). Identifique essas reas. b) Usando as informaes do texto, a charge e seus conhecimentos sobre o assunto responda: - O que Reforma Agrria? - Qual rgo do Governo responsvel por sua implantao? 6. A populao brasileira sempre apresentou elevado grau de mobilidade. O primeiro censo demogrfico do Brasil (1872), indicava que quase metade da populao brasileira vivia na Regio Nordeste. Os dados atuais, registram que mais de 30% dos brasileiros vivem fora de seus Estados de origem, ocasionando um declneo da populao nordestina. Os intensos movimentos das migraes internas foram responsveis por este declnio. O mapa abaixo representa os grandes fluxos emigratrios da regio nordeste.

Usando os dados indicados no mapa: a) Identifique as reas de atrao indicadas pelas setas. b) Explique que fatores de atrao levaram ocorrncia de cada um destes movimentos.

255

GEOGRAFIA

UFPA
F A S E

7. Foi num trecho de corredeira no baixo curso do rio, a 300 km de Belm, que a Eletronorte comeou
a construir, em 1975, uma parede de concreto com mais de 70 metros de altura a partir do fundo do leito. A barragem da Hidreltrica de Tucuru, a segunda maior do pas e a quinta do mundo, foi inaugurada em 1984. Passados 10 anos, a enorme estrutura continua a ser um divisor artificial de guas do Tocantins, comprometendo de vez a navegabilidade em uma das mais importantes bacias fluviais do planeta e afetando todo sistema de vida da regio. (PINTO, Lcio Flvio. Jornal Pessoal. Maro de 1994)

No texto acima, o autor expressa preocupao com os danos ambientais e sociais causados regio com a construo da Hidreltrica de Tucuru no Rio Tocantins. Sobre o assunto responda: a) Com que objetivo foi construda a Hidreltrica de Tucuru? b) Explique dois graves problemas causados populao daquela rea, em conseqncia da construo da Hidreltrica?

8. Das regies brasileiras, a Amaznia a que concentra maior ndice de malria, inclusive em algumas reas considerada doena endmica. A foto ao lado mostra uma dessas reas. Considerando essas informaes relacione e explique dois fatores naturais e dois scio-econmicos que favorecem a propagao e permanncia da doena nessas reas.

256

GEOGRAFIA

UFPE
F A S E

1. Observe a figura e as proposies abaixo em relao ao movimento de rotao da terra.

Instante 1

Instante 2

Instante 3

1. A terra gira em torno de seu eixo imaginrio no sentido OESTE - LESTE. 2. A rotao da terra tem influncia na circulao atmosfrica e no movimento das correntes martimas. 3. Devido a este movimento um satlite que percorre a rota S pode fotografar todos os pontos do planeta. 4. Em conseqncia deste movimento, a terra ocupa diferentes posies na ecltica, determinadas pelos pontos denominados equincios e solstcios. 5. Por causa deste movimento, o nvel do oceano ligeiramente mais baixo na costa leste do que na costa oeste dos continentes. Esto corretas: a) 1, 2 e 3 b) 2, 3 e 4 c) 3, 4 e 5 d) 2, 3 e 5 e) 1, 3 e 4

2. Em relao ao relevo terrestre e seus agentes de formao, incorreto afirmar: a) Os movimentos tectnicos, tambm chamados diastrofismos, so foras oriundas do interior da terra que atuam de forma lenta e prolongada na crosta terrestre. b) Intemperismo ou meteorizao um conjunto de processos fsicos, qumicos e biolgicos que operam na superfcie terrestre ocasionando a desintegrao e decomposio das rochas. c) Nos climas secos e quentes (ridos e frios-nevosos) predomina a desagregao mecnica das rochas. d) O produto final da meteorizao a formao do chamado manto ou regolito, material solto, resultante da decomposio das rochas, e que se encontra na superfcie, formando um horizonte sobre a rocha matriz. e) Os movimentos tectnicos verticais so chamados de epirogenticos. As grandes cordilheiras atuais dos Alpes, Andes e Himalaia foram formadas por esses movimentos no perodo tercirio.

257

GEOGRAFIA
O RELEVO SUBMARINO

UFPE
F A S E
( ( ( ( ) ) ) ) Fossas abissais Talude continental Dorsal submarino Plataforma continental

3. Observe a figura e estabelea a correspondncia entre os pontos numerados no mapa e seu significado: A seqncia correta : a) 4, 1, 3 e 2 b) 3, 2, 1 e 4 c) 4, 2, 3 e 1 d) 1, 2, 3 e 4 e) 2, 1, 4 e 3

4. Em relao vegetao no Brasil, incorreto afirmar: a) A grande extenso territorial e latitudinal e a diversidade morfoclimtica do Brasil explicam a sua grande riqueza vegetal. b) Os cocais ou babuuais so extensas reas de palmeiras encontradas principalmente no Maranho e Piau, constituindo uma vegetao de transio entre a floresta amaznica mida e a caatinga seca. c) A formao vegetal que mais caracteriza a regio sudeste brasileira o cerrado tpico, representado por rvores e arbustos associados a uma vegetao baixa e inferior formada de gramneas. d) Os campos da Campanha Gacha ocupam grandes extenses de terrenos quase planos e so famosos pela sua utilizao na prtica da pecuria. e) A caatinga a vegetao que caracteriza o Nordeste semi-rido apresentando solos ricos em sais minerais, mas pobres em matria orgnica. 5. Analise as proposies abaixo: 1. As curvas de nvel so linhas isomtricas, isto , linhas que unem pontos de mesma altitude representadas numa carta ou mapa. 2. A escala numrica fornece a relao entre os comprimentos de uma linha no mapa e o correspondente comprimento no terreno em forma de frao, tendo a unidade para numerador. 3. As projees azimutais baseiam-se na projeo de paralelos e meridianos em um cilindro envolvente, posteriormente planificado. 4. Em um mapa de escala 1 : 2.500.000, 8cm equivalem a 120 km. 5. O princpio fundamental da Cartografia consiste no estabelecimento sobre a superfcie da Terra de um sistema de coordenadas, ao qual pode ser referido qualquer ponto da mesma. Esto corretas: a) 1, 2 e 5 b) 2, 3 e 4 c) 3, 4 e 5 d) 1 e 3 e) 2, 4 e 5

258

GEOGRAFIA
6. No que se refere s classificaes climticas, incorreto afirmar:

UFPE
F A S E

a) Em se tratando de clima, podemos afirmar que no existem duas reas que possuam rigorosamente o mesmo clima, isso porque o mesmo dinmico e envolve uma infinidade de fatores e elementos. b) A classificao climtica de Arthur Straller considera como base do seu sistema de anlise os conhecimentos sobre a circulao geral da atmosfera, isto , o controle do clima pelas massas de ar. c) A classificao climtica de Wilhem Kppen baseia-se fundamentalmente na relao dos elementos climticos, temperatura e pluviosidade, com a vegetao. d) Segundo Arthur Straller os climas das latitudes baixas so controlados pelas massas de ar equatoriais e tropicais. e) Pela classificao de Kppen, o clima Cfa corresponde ao tropical mido em todas as estaes e com veres quentes. 7. No que se refere aos solos, incorreto afirmar: a) Na agricultura o termo solo refere-se camada superficial de terra arvel, dotada de vida microbiana. b) O elemento principal para explicar a formao e os tipos de solos o clima, j que o solo resultante da decomposio das rochas pelo intemperismo. c) Um dos solos de maior fertilidade no Brasil a terra roxa, resultante da decomposio do granito e do gnaisse, encontrados em grande quantidade nos Estados de So Paulo e Paran. d) A maioria dos solos no Brasil tropical frgil e requer cuidados especiais para se manter agricultvel. e) No serto semi-rido nordestino, onde as chuvas so escassas, predomina o intemperismo fsico na formao dos solos. 8. Analise a representao do mecanismo dos ventos, na figura ao lado, e as proposies apresentados. 1. A rea I corresponde a uma regio de baixa presso ou ciclonal. 2. Ao chegar na rea IV o ar ainda quente e mido retorna, em altitude, para os trpicos. 3. Em funo das correntes de ar descendentes encontramos nas reas II e III regies instveis, com predominncia de chuva. 4. As reas II e III corerspondem respectivamente s regies de altas presses ou anticlonais. 5. A rea I apresenta forte instabilidade devido s correntes de ar ascendentes. Esto corretas apenas: a) 1, 2 e 3 b) 1, 4 e 5 c) 2, 3 e 4 d) 3 e 4 e) 2, 4 e 5

259

GEOGRAFIA
9. Observe o grfico e as proposies.

UFPE
F A S E

1. O ritmo de crescimento da populao brasileira se intensificou aps a Segunda Guerra Mundial. 2. O aumento populacional verificado no perodo 1940 - 1980, quando a populao nacional passou de 41 para 120 milhes de habitantes, foi muito mais funo do crescimento vegetativo do que da imigrao. 3. O crescimento populacional brasileiro observado no perodo 1890 - 1900 deve-se contribuio da imigrao que ocorreu graas ao surto da borracha. 4. A realizao do Recenseamento no pas apresentou, ao longo dos tempos, uma alterao em sua periodicidade. 5. A populao brasileira apesar de ter apresentado, a princpio, crescimento declinante, logo passou a ter um comportamento constante. Esto corretas: a) 1, 3 e 5 b) 2, 3 e 4 c) 3, 4 e 5 d) 1, 2 e 4 e) 1, 4 e 5

260

GEOGRAFIA
10. As cidades se diferenciam muito umas das outras por suas funes urbanas. 1. 2. 3. 4. 5. ( ( ( ( ( ) ) ) ) ) Washington e Braslia Pittsburg e Volta Redonda Coimbra e Cambridge Veneza e Mnaco Roma e Meca Industrial Universitria Administrativa Religiosa Turstica

UFPE
F A S E

A seqncia correta a) 2, 3, 1, 5 e 4 b) 1, 2, 3, 5 e 4 c) 3, 5, 1, 4 e 2 11. Analise as proposies: 1. A diferena existente entre os preos dos produtos do setor primrio e os preos dos produtos industrializados uma determinante para o aumento da dvida externa dos pases do Terceiro Mundo. 2. A globalizao uma organizao de integrao das economias nacionais, a nvel mundial. 3. A produo brasileira de sal originada, principalmente, do Rio Grande do Norte e do Maranho. 4. O Cartel um acordo entre empresas de um mesmo ramo de produo que no ultrapassa as fronteiras de um pas. 5. O NAFTA um acordo de livre comrcio, assinado pelo Canad, Estados Unidos e Mxico. Esto incorretas: a) 1, 3 e 4 b) 2, 3 e 4 c) 1, 2 e 3 d) 3, 4 e 5 e) 2, 4 e 5 d) 4, 2, 1, 3 e 5 e) 5, 4, 1, 2 e 3

12. No que se refere aos pases ditos do Terceiro Mundo, correto afirmar: a) b) c) d) Possuem uma elevada expectativa de vida e uma alta taxa de fecundidade. O poder poltico se encontra concentrado nas mos das oligarquias. Apresentam uma hipertrofia do setor secundrio. A populao absoluta numerosa, ao contrrio do que acontece com a densidade demogrfica. e) Contribuem para uma maior degradao do meio ambiente do que os pases desenvolvidos.

261

GEOGRAFIA
13. Observe a tabela, relacionando-a com a proposies. Brasil Capacidade geradora de energia eltrica segundo as fontes e regies produtoras, em MW. 1987
REGIES Norte Nordeste Sudeste Sul Centro-Oeste Brasil CAPACIDADE GERADORA / FONTES TOTAL 3.696 6.731 25.753 7.006 800 43.986 HIDRULICA 2.752 5.668 22.331 5.663 557 36.971 TRMICA 944 1.063 3.422 1.343 243 7.015

UFPE
F A S E

Fonte: IBGE. Anurio Estatstico do Brasil, 1988

1. As regies Sudeste e Sul participam com 84% da capacidade geradora de energia do pas. 2. Em termos percentuais, a fonte trmica tem uma participao sempre inferior a 32%, em todas as regies do pas. 3. A regio Norte representa 8,4% da capacidade geradora de energia do pas, superior apenas representatividade de 3% do Centro-Oeste. 4. A produo de energia hidrulica do Nordeste corresponde a aproximadamente um quinto da produo do Sudeste. 5. Ocorre no Brasil elevado predomnio da hidreletricidade: 84%, sobre a Termeletricidade. Esto incorretas: a) 1, 2 e 3 b) 2, 3 e 5 c) 3, 4 e 5 d) 1, 4 e 5 e) 1, 3 e 4

14. No que se refere caa e pesca, existem pases que regulamentam o perodo permitido prtica dessas atividades. Nas alternativas abaixo, identifique aquela que justifica esta forma de atuao. a) Promover um gerenciamento dos recursos no sentido de evitar a extino. b) Atrair pessoas apenas em um determinado perodo do ano. c) Coincidir o desenvolvimento dessas atividades com o perodo em que os animais aparecem em uma maior quantidade. d) Procurar associar a atividade extrativa atividade comercial. e) A menor demanda pelos produtos em uma determinada poca do ano.

262

GEOGRAFIA
15. Observe a tabela e as proposies: Brasil Produo de Petrleo segundo Unidades da Federao 1987.
3

UFPE
F A S E

ESTADO TOTAL Rio de Janeiro Bahia Rio G. do Norte Sergipe Cear Esprito Santo Alagoas Maranho Brasil 19.938 4.636 2.972 2.850 1.011 830 589 2 32.828

PRODUO (1.000 m ) EM TERRA 4.350 1.887 2.063 129 770 589 2 9.790 NA PLATAFORMA 19.938 286 1.085 787 882 60 23.038
Fonte: IBGE

1. A regio Sudeste do pas contribui com cerca de 65% da produo nacional. 2. A produo de petrleo oriunda da explorao no continente, em vrias bacias sedimentares brasileiras, pouco superior a 30% do total. 3. A plataforma continental responde atualmente por 70% da produo nacional. 4. A produo de petrleo brasileira a grande responsvel pelo desenvolvimento da sua indstria siderrgica, sendo o Rio de Janeiro responsvel por 61% de toda a produo nacional. 5. A regio nordestina representada na produo de petrleo brasileiro por cerca de 66,6% dos seus Estados. Esto incorretas: a) 2, 3 e 4 b) 1, 4 e 5 c) 3, 4 e 5 d) 1, 2 e 4 e) 1, 3 e 5

263

GEOGRAFIA
16. Analise as proposies:

UFPE
F A S E

1. A poluio sonora um srio problema ambiental do sculo XX, tendo-se intensificado a partir da dcada de 1970 com o desenvolvimento da indstria eletrnica. 2. A explorao da madeira feita em larga escala e por processos racionais, nos pases temperados e frios. 3. A maior ou menor rentabilidade da agricultura est condicionada s condies naturais, sendo pouco influenciada pelo fator humano. 4. O sistema agrcola do campons europeu formou-se a partir da Revoluo Industrial quando a atividade agrcola passou a ser respaldada pela indstria. 5. A agricultura moderna se caracteriza pela especializao das vrias regies em certos produtos, pela facilidade de escoamento da produo, pela cultura feita mecanicamente e em larga escala bem como pelo fato de estar a atividade cada vez mais voltada para o mercado. Esto corretas: a) 2, 3 e 4 b) 1, 2 e 5 c) 1, 2 e 3 d) 3, 4 e 5 e) 2, 4 e 5

PARA CADA UMA DAS QUESTES A SEGUIR ASSINALE NA COLUNA I A(S) PROPOSIO(ES) VERDADEIRA(S) E NA COLUNA II, A(S) FALSA(S): 1. Assinale as pro posies abaixo: 0-0) A forma da Terra sem dvida esfrica, porm, por no ser uma esfera perfeita, pois h um pequeno achatamento nos plos e um abaulamento no Equador, atribui-se-lhe a forma geide. 1-1) O ano-luz a distncia percorrida por um raio luminoso, em um ano, razo de 300 km por hora. 2-2) O sistema geocntrico, que teve em Cludio Ptolomeu seu principal defensor, considerava a Terra em estado imvel, no centro do Universo, tendo a girar em torno de si os astros ento conhecidos. 3-3) Pela Lei da Gravitao Universal, Isaac Newton afirmou que tudo se passa no Universo, como se os corpos se atrassem na razo direta das suas massas e na razo inversa do quadrado das distncias que os separam. 4-4) A durao do movimento de rotao da Terra depende de um ponto referencial. Se este ponto for o sol a sua durao ser de 23 horas, 56 minutos e 4 segundos. 2. Em relao poluio ambiental, podemos afirmar: 0-0) A chuva cida resultante da combinao qumica de substncias poluentes, principalmente o dixido de enxofre e os xidos de nitrognio, provocando a alterao qumica do solo e das guas, interferindo nas lavouras, florestas, estruturas metlicas e edificaes. 1-1) Os aerossis provocam uma diminuio da camada de oznio, fato que impede a filtragem adequada da radiao infravermelha e ocasiona problemas srios para a vida no globo terrestre. 2-2) Entende-se como poluio atmosfrica a presena de poluentes na atmosfera, tornando o ar impuro e prejudicial sade. 3-3) A emisso desordenada de poluentes para a atmosfera pode refrear a ao do efeito estufa, e, em conseqncia, a diminuio da temperatura mdia do ar. 4-4) O excesso de material orgnico no mar leva proliferao descontrolada de microorganismos, que acabam por formar as chamadas mars vermelhas, que matam peixes e deixam os frutos do mar imprprios para o consumo.

F A S E

264

GEOGRAFIA
3. Em se tratando de precipitao atmosfrica, observe a figura e analise as proposies:

UFPE
F A S E

0-0) 1-1) 2-2) 3-3) 4-4)

A ilustrao mostra a chuva do tipo orogrfica. A vertente da montanha que fica voltada para a direo dos ventos denomina-se sotavento. Esse tipo de chuva ocorre somente nas regies equatoriais. A vertente de barlavento sempre mais mida e vegetada do que a vertente de sotavento. A chuva mostrada na ilustrao resulta da subida da massa de ar mida em contato com a vertente da montanha.

4. Observe a figura e analise as proposies:

0-0) As correntes martimas so movimentos advectivos das guas quentes ou frias atravs dos oceanos, produzindo efeitos climticos nos litorais por elas banhados. 1-1) As correntes quentes originam-se nas baixas latitudes e as frias em elevadas latitudes. 2-2) Tanto no hemisfrio norte como no sul, movimentam-se as correntes equatoriais, conduzindo guas quentes no mesmo sentido do movimento de rotao terrestre. 3-3) A Gulf Stream uma corrente quente que ameniza os rigores do inverno na Europa Ocidental. 4-4) No oceano ndico, de configurao e situao geogrfica muito diferentes, predominam as chamadas correntes de mones, caracterizados pela direo e temperatura dos ventos que lhes do origem.

265

GEOGRAFIA
5. Em relao s formas de relevo litorneas, podemos afirmar:

UFPE
F A S E

0-0) Podem resultar tanto da ao erosiva como da deposio, que caracterizam as costas escarpadas e as costas baixas ou planas. 1-1) A praia o conjunto de sedimentos, depositados ao longo do litoral, que se encontram em constante movimento. 2-2) As restingas so formadas por faixas arenosas, depositadas paralelamente praia, que se alongam tendo ponto de apoio nos cabos e nas salincias do litoral. 3-3) No litoral do Rio Grande do Sul, extensa plancie de restinga separa a Lagoa dos Patos das guas atlnticas. 4-4) Em geral, o sedimento dominante nas praias so areias, mas tambm existem praias formadas por cascalhos e seixos. 6. H vrias maneiras de se fazer a representao grfica do relevo terrestre. A ilustrao representa uma delas, da qual podemos afirmar:

0-0) A representao grfica da ilustrao a que apresenta o relevo em forma de hachuras. 1-1) Entre as duas montanhas representadas existe um vale profundamente encaixado e situado a 490m de altitude. 2-2) A eqidistncia entre as curvas de 310m. 3-3) As vertentes das montanhas assinaladas com a letra A so mais ngremes que as vertentes assinaladas com a letra B. 4-4) As partes mais altas das duas elevaes esto situadas acima de 800 m de altitude.

266

GEOGRAFIA
7. Em relao hidrografia brasileira, podemos afirmar:

UFPE
F A S E

0-0) A vasta extenso territorial do pas e o fato desta imensa rea apresentar, na sua maior parte, condies excepcionais de umidade, propiciaram a compartimentao de vrias e grandes bacias fluviais. 1-1) Os principais divisores de guas da bacia de So Francisco e a dos rios da vertente litornea do leste so as serras Geral, do Mar e da Mantiqueira. 2-2) Os rios So Francisco e Paran tm alguns trechos navegveis, separados por zonas de quedas dgua, e considervel potencial hidrulico. 3-3) O clima fator de grande importncia na hidrografia, sobretudo no que concerne aos regimes fluviais. No caso brasileiro, so as chuvas que mais interferem na variao dos mesmos. 4-4) No regime fluvial semi-rido o principal aspecto o carter temporrio dos rios. Permanecem secos total ou parcialmente durante o longo perodo de estiagem que ocorre no vero outono, no serto nordestino.

8. Analise as propores abaixo: 0-0) D-se o nome de vegetao ao conjunto de plantas nativas que cobre quase toda a superfcie das terras firmes. 1-1) As florestas subtropicais manifestam-se principalmente na bacia do Mediterrneo ao sul dos Estados Unidos e ao sul do trpico de Cncer, na Amrica do Sul. 2-2) A vegetao dos desertos quentes desaparece completamente ou constituda por raras espcies xerfilas, enquanto a fauna tambm se restringe a animais carnvoros, alguns rpteis e roedores. 3-3) A vegetao nos desertos polares no se desenvolve onde os gelos so eternos. Ela aparece sob a forma de musgos e lquens, naquelas reas em que os solos descongelam no curto vero polar. 4-4) As florestas conferas, ricas em pinheiros, sequias e btulas so tpicas dos climas desrticos frios. 9. Observe o mapa, comparando-o s proposies abaixo, que dizem respeito s 27 unidades poltico-administrativas que integram o Brasil. 0-0) 1-1) 2-2) 3-3) 4-4) 23 estados, 3 territrios e 1 distrito federal. 25 estados, 1 territrio e 1 distrito federal. 24 estados, 2 territrios e 1 distrito federal 22 estados, 4 territrios e 1 distrito federal 26 estados e 1 distrito federal.

267

GEOGRAFIA

UFPE
F A S E

10. Observe a tabela abaixo e, analisando as proposies, indique a(s) que esto de acordo com os dados apresentados. Frota Automobilstica de Alguns Pases
TIPO DE VECULO PASES PARTICULAR Alemanha ustrlia Brasil China Ex-URSS EUA Japo Total Mundial 37.410 7.583 10.450 1.250 12.750 145.043 37.076 449.525 UTILITRIO 3.095 2.183 2.450 4.172 9.500 45.698 22.838 139.611 NMERO DE HABITANTES POR VECULO PARTICULAR 2,1 2,2 14,7 92,1 22,9 1,7 3,3 12,0

Fonte: GARNIER, B et all. Images conomiques du Monde - 1994.

0-0) As menores relaes habitantes por veculo particular so verificadas em pases da Amrica do Norte, Europa e sia, onde os valores se mostram inferiores a 3,0. 1-1) A frota de veculo particular brasileira representa um percentual de 2,3% da frota mundial. 2-2) A Amrica Anglo-Saxnica e a sia de Sudeste representadas, respectivamente, pelos Estados Unidos e pelo Japo, detm as maiores frotas de veculos utilitrios do mundo. 3-3) Os pases listados na tabela tm uma participao na frota particular e utilitria mundial de cerca de 30% e 35%, respectivamente. 4-4) A frota de veculo utilitrio da China equivale a aproximadamente trs vezes a frota australiana. 11. Observe as proposies concernentes atividade industrial no mundo: 0-0) O processo de industrializao da Repblica Sul-africana teve o seu maior impulso durante a Segunda Guerra Mundial, apoiando-se em uma srie de fatores, entre eles a riqueza minerolgica do seu subsolo. 1-1) Na indstria argentina, como em vrios pases da Amrica Latina, as filiais de multinacionais alcanam um percentual significativo na produo industrial. 2-2) Conhecidos como Tigres Asiticos, a Coria do Sul, Hong Kong, Japo, Cingapura e Taiwan so pases que adotaram um modelo de industrializao voltado para as exportaes. 3-3) A regio do rio Elba, que se localiza no territrio que correspondia ex-Alemanha Ocidental, uma das principais reas industriais da Europa, contando com centros industriais, como Dresden e Leipzig. 4-4) A presena de vrios recursos minerais e energticos foi um dos pilares do processo de industrializao da Mandchria, situada no nordeste da China.

268

GEOGRAFIA
12. Analise as proposies abaixo:

UFPE
F A S E

0-0) O petrleo tem se destacado como um dos grandes poluentes dos oceanos e mares. 1-1) Chama-se poluente ao conjunto de equipamentos antipoluio. 2-2) O transporte ferrovirio brasileiro no foi capaz de integrar as diversas regies do pas, uma vez que, orientadas no sentido interior - litoral, as redes ferrovirias eram sempre isoladas umas das outras. 3-3) A regio norte-americana que se estende de Boston a Washington e que caracteriza o maior conjunto urbanizado dos Estados Unidos pode ser considerada como uma megalpole. 4-4) O lixo, que constitui um grande problema para a maioria dos centros urbanos, pode em grande parte, ser recuperado e transformado em adubo para o solo agrrio. 13. Analise a tabela e as proposies: Brasil 1975/85
EXPORTAES Produtos Bsicos Semimanufaturados Manufaturados Total BILHES DE DLARES/ANO 1975 5,027 0,850 2,793 8,670 1985 8,553 2,760 14,062 25,375

Fonte: Fundao Centro dos Estados do Comrcio Exterior

0-0) Os produtos bsicos representavam, em 1975, cerca de 60% do valor das exportaes, decrescendo para uma representatividade de 35% em 1985. 1-1) Observa-se uma grande ampliao nas exportaes de produtos industrializados, superando as exportaes de produtos primrios. 2-2) Houve no perodo um considervel aumento da produo industrial no Brasil em detrimento da produo agrcola. 3-3) Houve aumento superior a 50% no valor da exportao considerando-se o perodo de 10 anos. 4-4) O comrcio exterior do Brasil sofreu alteraes tanto do ponto de vista qualitativo, como quantitativo, no perodo 1975/85. 14. Analise as proposies abaixo: 0-0) A partir da dcada de 1950 a populao brasileira vem crescendo em ritmo cada vez mais lento, em funo da reduo das taxas de mortalidade. 1-1) Normalmente, as reas que recebem muitos migrantes costumam apresentar uma proporo de populao masculina ligeiramente superior de mulheres. 2-2) A taxa de mortalidade um indicador social importante para que se possa avaliar a qualidade de vida de um pas ou de uma populao. 3-3) A teoria malthusiana apia-se em dois princpios: a populao mundial cresceria segundo uma progresso aritmtica, enquanto a produo de alimentos se ampliaria obedecendo a uma progresso geomtrica. 4-4) Os neomalthusianos aceitam a idia de Malthus de que a populao mundial cresceria segundo uma progresso geomtrica, mas admitem que o problema da produo de alimentos supervel.

269

GEOGRAFIA
15. Analise as proposies que versam sobre estrutura agrria e ocupao dos solos.

UFPE
F A S E

0-0) O milho, alimento bsico dos astecas, maias e quichuas, muito difundido fora da Amrica, na frica, sia e na Europa Mediterrnea, sendo usado tanto na alimentao humana como na animal. 1-1) Nas regies frias, ficando o solo coberto por uma camada de gelo durante grande parte do ano, natural que apenas poucas plantas que se adaptam s baixas temperaturas e que tenham ciclo vegetativo longo possam sobreviver. 2-2) A parceria e o arrendamento so exemplos de relaes de trabalho em que a propriedade da terra e a sua explorao no se encontram em mos de uma mesma pessoa. 3-3) No Brasil, o indivduo pode obter a propriedade da terra pela compra, por herana ou pela concesso de terras devolutas. 4-4) A estrutura fundiria visa a uma modificao da estrutura agrria existente, e no ocupao de reas novas, no apropriadas. 16. Observe as proposies abaixo: 0-0) A caa, a pesca e a silvicultura so consideradas como atividades econmicas predatrias. 1-1) Toda a costa europia rea de grande atividade pesqueira, onde so pescados o arenque, a sardinha e o atum. Na costa canadense do Pacfico realizada a pesca de salmo, peixe marinho que costuma subir as correntes fluviais na poca da desova. 2-2) So graves as conseqncias no meio ambiente provocadas pelo desmatamento, entre elas as alteraes no ciclo do carbono. 3-3) O homem, hoje em dia, no utiliza apenas a energia trmica, a hidreltrica e a energia solar, mas tambm as formas de energia usada no passado. 4-4) O petrleo, conhecido desde a mais remota antigidade, passou a ser utilizado em propores comerciais a partir do sculo XX, substituindo gradativamente o carvo como fonte de energia.

270

GEOGRAFIA
1. A posio indicada na figura ao lado marca, em nosso planeta, o incio de uma estao em cada hemisfrio. Esta estao a) b) c) d) e) a primavera no Hemisfrio Norte, estando a Terra no equincio. o outono no Hemisfrio Sul, estando a Terra nas proximidades do solstcio. o vero meridional, estando a Terra no solstcio. o outono setentrional, estando a Terra no equincio. o vero setentrional, estando a Terra no solstcio.

UFRGS

2. Para cada tipo de representao existe uma escala numrica apropriada. Assim, os mapas podem ser divididos em trs categorias bsicas: escalas grande, mdia e pequena. Associe as escalas numricas mais apropriadas para as finalidades dos mapas. 1. 2. 3. 4. mapas topogrficos plantas urbanas planisfrios plantas arquitetnicas ( ) 1:50 a 1:100 ( ) 1:25.000 a 1:250.000 ( ) 1:500 a 1:20.000

A seqncia numrica correta, de cima para baixo, na coluna da direita, : a) 4 - 3 - 1 b) 4 - 1 - 2 c) 2 - 3 - 4 d) 4 - 2 - 1 e) 3 - 1 - 4

3. A tabela abaixo apresenta dados de temperaturas mnimas e mximas verificadas no dia 6 de agosto de 1996, em quatro diferentes cidades do Mundo.
Diferena de horas em relao ao horrio de Braslia 0 +11 -2 +5 Temperatura mnima/mxima (C) 6/14 23/28 13/17 21/35

Cidade 1 2 3 4

As cidades de nmeros 1, 2, 3 e 4 podem ser, respectivamente, a) b) c) d) e) Santiago, Tquio, Los Angeles e Moscou. Buenos Aires, Pequim, Lima e Madri. Montevidu, Moscou, Nova Iorque e Paris. Rio de Janeiro, Moscou, Toronto e Sidnei. So Paulo, Pequim, Buenos Aires e Lisboa.

271

GEOGRAFIA
4. Observe a figura abaixo, que representa as curvas de nvel de uma regio hipottica.

UFRGS

Qual o perfil topogrfico que melhor representa o segmento A - A na figura acima?

5. O grfico abaixo mostra a distribuio mdia anual da precipitao e da evaporao, segundo a latitude, no globo terrestre.

A anlise do grfico permite afirmar que a) b) c) d) e) na zona equatorial e nas latitudes superiores a 40, a precipitao supera a evaporao. na zona equatorial e nas latitudes superiores a 40, a evaporao supera a precipitao. na zona polar e nas latitudes inferiores a 30, a evaporao supera a precipitao. na zona polar e nas latitudes inferiores a 30, a precipitao supera a evaporao. nas zonas tropicais, a precipitao supera a evaporao.

272

GEOGRAFIA
6. Considere as seguintes afirmaes:

UFRGS

I - As regies litorneas apresentam amplitudes trmicas to elevadas como aquelas de regies situadas no interior dos continentes. II - Lugares situados prximos ao mar apresentam veres mais quentes e invernos mais frios que lugares de grande continentalidade. III - As guas do mar, dos oceanos e dos rios demoram mais para se aquecer e para se resfriar do que as terras continentais. Quais esto corretas? a) Apenas I b) Apenas II c) Apenas III d) Apenas I e II e) Apenas II e III

7. Pesquisa feita por gegrafos da Universalidade de So Paulo, em 1995, ampliou para 28 o nmero de existentes no Brasil. O trabalho se baseou em levantamento realizado entre 1970 e 1985 pelo projeto Radambrasil. Segundo esta pesquisa, desapareceram classificaes tradicionais como as de . a) b) c) d) e) formaes vegetais - Mata de Araucria e Floresta Amaznica climas - Clima Tropical e Clima Equatorial unidades de relevo - Planalto Central e Planalto das Guianas bacias hidrogrficas - Rios de Planalto e Rios de Plancie domnios morfoclimticos - Domnio da Caatinga e Domnio do Cerrado

8. Com relao ao ciclo hidrolgico e seus efeitos sobre a poluio das guas, so feitas as seguintes afirmaes: I - A mais importante entrada para o ciclo hidrolgico so as precipitaes, e as mais importantes sadas so a evaporao e a transpirao. II - O desmatamento favorece a eroso do solo e o assoreamento dos cursos fluviais, embora no influencie na ocorrncia de enchentes, nem de estiagens mais prolongadas. III - Os agrotxicos utilizados nas lavouras escoam por ao da chuva e penetram nos lenis freticos, poluindo os rios. Quais esto corretas? a) Apenas I b) Apenas II c) Apenas I e III d) Apenas II e III e) I, II e III

273

GEOGRAFIA

UFRGS

9. O Programa Nacional de Microbacias Hidrogrficas, institudo em 1987, provou nestes ltimos anos ser uma iniciativa de grande importncia no combate degradao dos solos e das guas. Qual das medidas abaixo NO corresponde a prticas de conservao dos solos e das guas? a) b) c) d) e) Compactar e fertilizar o solo. Preservar a qualidade das guas. Controlar o regime hdrico. Reflorestar as encostas e margens dos mananciais hdricos. Cultivar seguindo as curvas de nvel.

10. A causa principal para a existncia do clima semi-rido no Nordeste brasileiro a) b) c) d) e) so as barreiras orogrficas que impedem a passagem do ar mido vindo do oceano. a circulao geral da atmosfera, fenmeno externo regio. a alta refletividade da superfcie nordestina. a inexistncia de vegetao de grande porte na regio. a presena de redes hidrogrficas hierarquizadas, com drenagem aberta para o mar.

11. O Brasil um dos pases de maior biodiversidade do planeta. Somente a Amaznia abriga mais de 20 em cada 100 espcies de animais e plantas existentes. Assinale a alternativa que contempla somente pases com grande diversidade de espcies. a) b) c) d) e) Mxico, Colmbia e Indonsia. Colmbia, Equador e Alemanha. Zaire, Costa Rica e Arglia. Argentina, Uruguai e Paraguai. Mxico, Peru e Uruguai.

12. Em vista das dificuldades de conciliao entre o desenvolvimento econmico e as necessidades scio-ambientais na Amaznia, o governo brasileiro criou o Projeto SIVAM - Sistema de Vigilncia da Amaznia. Com ele podero ser definidas estratgias e implementadas polticas orientadas para o desenvolvimento sustentvel da regio. Sobre o projeto SIVAM so feitas as seguintes afirmativas: I - Estabelece condies de controle e defesa do meio ambiente, permitindo a identificao de reas sujeitas ao predatria das queimadas, poluio dos rios e desmatamentos. II - Permite adquirir dados meteorolgicos, hidrolgicos e ambientais da regio amaznica, atravs da tcnica de Sensoriamento Remoto. III - Utiliza a tecnologia de Sistemas de Informaes Geogrficas, que possibilita, por exemplo, a gerao de mapas atualizados sobre desmatamentos, cobertura vegetal, uso do solo, entre outros. Quais esto corretas? a) Apenas I b) Apenas II c) Apenas I e III d) Apenas II e III e) I, II e III

274

GEOGRAFIA
13. As afirmativas abaixo referem-se s regies polares.

UFRGS

I - A Antrtica um continente de aproximadamente 14 milhes de km , quase totalmente coberto por um manto de gelo que pode ultrapassar 4.500 metros de espessura e que se constitui na maior reserva de gua doce do planeta. II - A regio rtica constituda por um oceano (Oceano rtico) coberto por uma fina camada de mar congelado, vrios arquiplagos e ilhas isoladas. Abrange, tambm, geleiras situadas na parte setentrional das principais massas continentais do planeta (sia, Europa e Amrica do Norte). III - Ambas as regies polares, habitadas por esquims, tm seus territrios disputados por vrios pases. Ao Brasil cabe parte da Pennsula Antrtica e das Ilhas Shetland do Sul, onde se situa a Estao Brasileira Comandante Ferraz. Quais esto corretas? a) Apenas I b) Apenas I e II c) Apenas I e III d) Apenas II e III e) I, II e III

14. Dados sobre o aquecimento global vm indicando mudanas ambientais que podem provocar problemas ao homem j no prximo sculo. Os cientistas j verificaram, nas ltimas dcadas, uma tendncia de elevao do nvel do mar, estimada at o ano 2100, entre 0,60 m at 1,00 m acima do nvel atual. Sobre este assunto, assinale a alternativa INCORRETA. a) Atualmente observa-se um derretimento progressivo tanto nas geleiras perifricas aos plos quanto nas de altas montanhas. b) Observa-se uma tendncia geral de aumento na taxa de precipitao, em face da maior quantidade de vapor dgua na atmosfera, causada pelo Efeito Estufa. c) A Antrtica tem papel fundamental no condicionamento climtico do planeta, devido a seu grande volume de gelo e influncia nas circulaes atmosfrica e ocenica. d) Observa-se um aumento do Buraco de Oznio na atmosfera, provocado pelo Efeito Estufa. e) O Efeito Estufa um processo natural que vem sendo acentuado pela ao humana.

275

GEOGRAFIA
15. Assinale a alternativa que preenche corretamente as lacunas abaixo.

UFRGS

Anomalias intermitentes da temperatura superficial da gua do mar, ao longo da costa, podem ocorrer a cada dois anos. No entanto, a cada quatro ou cinco anos esta anomalia pode ser mais acentuada, aumentando a temperatura da gua em alguns graus. A zona de gua quente expandese nestes anos, impedindo a ressurgncia de guas frias ricas em nutrientes, causando a morte macia de peixes e, por conseqncia, a populao de pssaros dizimada. Este fenmeno acompanhado por chuvas intensas na regio continental vizinha, resultando em enchentes e perda de colheitas. Finalmente, anomalias climticas so registradas em regies a milhares de quilmetros de distncia.

O pargrafo acima se refere ao fenmeno conhecido como . a) b) c) d) e) Mones - o sudeste asitico Efeito Estufa - a costa leste dos E.U.A. El Nio - a costa do Peru e Equador Mones - a frica Ocidental destruio da camada de oznio - a Antrtica

, e a regio mais afetada

16. O litoral do Rio Grande do Sul sofre com graves problemas ambientais de saneamento, inadequao de obras costeiras e disposio de resduos slidos. Tais fatos se devem, fundamentalmente, carncia de planejamento regional. Como fatores agravantes do desequilbrio neste ambiente, podemos considerar: I - ocorrncia de ressacas com alto poder de eroso, fenmeno agravado pela presena de obras e retirada de material das praias. II - elevada porosidade do terreno, aliada s baixas altitudes, que facilitam a contaminao do lenol fretico e do manancial de guas das lagoas e lagunas do litoral. III - grande variao sazonal de populao no litoral, aliada falta de infra-estrutura, que contribuem para a contaminao das praias e setores lagunares. Quais esto corretos? a) Apenas I b) Apenas I e II c) Apenas I e III d) Apenas II e III e) I, II e III

17. Considere as associaes abaixo, entre o tipo de problema ambiental e a respectiva rea de ocorrncia no litoral brasileiro: I - efluentes carbonferos - litoral do Paran II - poluio dos rios pelo vinhoto - litoral de Pernambuco III - dejetos industriais nos mangues - litoral do Rio Grande do Sul Quais esto corretas? a) Apenas I b) Apenas II c) Apenas III d) Apenas I e II e) I, II e III

276

GEOGRAFIA

UFRGS

18. Os nmeros 1, 2, 3 e 4, representados no mapa abaixo, referem-se a reas com diferentes caractersticas fsicas e econmicas, no Rio Grande do Sul.

Associe o nmero da rea, constante no mapa, com a atividade econmica e a caracterstica fsica da regio. ( ( ( ( ) plantio da soja e triticultura - rochas vulcnicas ) plo industrial metal-mecnico - mata de araucria ) criao extensiva - processos de arenizao ) extrao mineral - rochas granticas

A seqncia numrica correta, de cima para baixo, : a) 4 - 1 - 3 - 2 b) 2 - 4 - 3 - 1 c) 1 - 4 - 3 - 2 d) 4 - 1 - 2 - 3 e) 3 - 1 - 4 - 2

19. A taxa de fecundidade no Brasil vem caindo drasticamente, conforme os resultados dos ltimos censos divulgados pelo IBGE. Entre as causas diretas desta diminuio, NO se pode incluir a) b) c) d) e) a utilizao de mtodos contraceptivos. a idade mais avanada da mulher ao casar. o ingresso da mulher no mercado de trabalho. a vasectomia. a laqueadura de trompas.

277

GEOGRAFIA

UFRGS

20. Segundo dados dos ltimos censos do IBGE (1980 e 1991), a expectativa de vida da populao brasileira vem aumentando. A partir deste indicador de qualidade de vida, torna-se necessria a implementao, nas prximas dcadas, de polticas pblicas III III IV de incremento na criao de empregos. de sade pblica nas reas de puericultura e obstetrcia. educacionais na ampliao de vagas no ensino de primeiro grau. de qualificao do cotidiano da Terceira Idade.

Quais esto corretas? a) Apenas I, II e III b) Apenas I, II e IV c) Apenas II e III 21. Observe o quadro abaixo.
INDICADORES DE DESENVOLVIMENTO HUMANO DE ALGUMAS UNIDADES DA FEDERAO DO BRASIL Unidades da Federao RS DF SP RJ ES Fonte: Ipea, 1996 Expectativa de vida (anos) 74,6 70,1 68,9 68,8 71,4 Escolaridade (%) 84 83 86 83 82 Renda per capita (em US$ mil) 5,6 10,2 8,8 6,7 2,7

d) Apenas I e IV e) I, II, III e IV

A partir dos dados apresentados no quadro acima, pode-se afirmar que a) o Rio Grande do Sul se apresenta como o estado com melhores ndices em relao aos demais. b) os capixabas apresentam todos os indicadores inferiores aos do Rio Grande do Sul. c) o Distrito Federal e So Paulo apresentam uma eqitativa distribuio de renda em relao aos demais estados. d) o Rio Grande do Sul e o Distrito Federal vm apresentando uma diminuio no ndice de escolaridade. e) o Rio de Janeiro apresenta os piores ndices em relao aos demais estados citados no quadro. 22. A charge ao lado representa um tipo de mobilidade espacial da populao, caracterstico do Brasil entre 1960 e 1980. Este tipo de mobilidade conceituado como a) migrao pendular. b) transumncia. c) migrao rural-urbana. d) nomadismo. e) deslocamento intra-urbano.

Fonte: A estruturao urbana e a participao da comunidade Charge: Moa

278

GEOGRAFIA
Instruo: A tabela a seguir refere-se s questes de nmeros 23 e 24.
Esperana de vida ao nascer (anos) * 60 64 78 71 56 58 Analfabetismo entre adultos (em %) ** 23 52 7 22 48 31

UFRGS

Pases Bolvia Egito Espanha Malasia PapuaNova Guin Qunia

PNB per capita (em dlares)* 760 660 13.590 3.140 1.130 270

* dados relativos a 1993, ** dados relativos a 1990. Fonte: Banco Mundial, Relatrio para o Desenvolvimento Mundial, 1995.

23. Considere as seguintes afirmaes relativas aos dados da tabela. I - Os pases que apresentam as mais elevadas taxas de analfabetismo so os que possuem os mais baixos ndices de PNB per capita. II - Malasia e Espanha apresentam os mais baixos ndices de analfabetismo e possuem os mais elevados ndices de PNB per capita. III - Os pases que apresentam PNB per capita acima de US$ 2.000 so os que tambm possuem os melhores indicadores de esperana de vida. Quais esto corretas? a) Apenas I b) Apenas II c) Apenas III d) Apenas I e II e) Apenas II e III

24. A respeito das informaes constantes na tabela correto afirmar que a) o crescimento da renda per capita o fator determinante para conter as taxas de analfabetismo. b) melhorias na distribuio de renda no influem nos ndices de esperana de vida. c) o PNB per capita uma mdia e, portanto, uma medida falsa que no permite revelar as caractersticas econmicas ou sociais de um pas, nem mesmo quando associadas a outras informaes estatsticas. d) todos os pases localizados em zona intertropical apresentam indicadores de esperana de vida abaixo de 70 anos. e) entre os pases com PNB per capita inferior a US$ 1.000, os localizados na frica apresentam as maiores taxas de analfabetismo.

279

GEOGRAFIA
25. Selecione a alternativa que completa corretamente as lacunas da frase abaixo:

UFRGS

No ano de 1996 ocorreu, na cidade de Istambul, a Conferncia de Cpula da ONU, denominada , tendo como tema principal . a) b) c) d) e) Habitat II - os assentamentos humanos sustentveis Habitat I - a urbanizao mundial Cpula da Terra - o desenvolvimento auto-sustentvel Cpula Mundial Social II - os problemas sociais mundiais Cpula Mundial Social I - a mortalidade infantil

26. O Corredor Biocenico por rodovias, em construo atravs de deciso dos governos do Chile, Argentina e Brasil, est praticamente concludo e unir os portos de Rio Grande, no sul do Brasil, a Antofagasta, no Chile, ampliando os mercados do Atlntico e do Pacfico para o MERCOSUL. A importante ponte que est sendo construda para a concluso desta obra a a) b) c) d) e) Ponte Barraco - Santiago del Estero sobre o Rio Uruguai. Ponte So Borja - Santo Tom sobre o Rio Paran. Ponte Barraco - Corrientes sobre o Rio Ibicu. Ponte So Borja - Santo Tom sobre o Rio Uruguai. Ponte Corrientes - Santiago del Estero sobre o Rio Paran.

27. Entre os equipamentos urbanos que condicionam a descentralizao das atividades econmicas e contribuem para o processo de decadncia das reas centrais das grandes cidades esto os I - postos de sade. II - shopping centers. Quais esto corretos? a) Apenas I, II e III b) Apenas I, II e IV c) Apenas I e II d) Apenas II e IV e) I, II, III e IV III - cemitrios. IV - hipermercados.

28. A regionalizao do mundo utilizada pelo Banco Mundial agrupa os pases de acordo com o nvel de renda. Segundo esse critrio, so economias de baixa renda e de alta renda, respectivamente: I - Moambique e Nova Zelndia. II - Chade e Finlndia. III - China e Kuwait. Quais esto corretas? a) Apenas I b) Apenas I e II c) Apenas I e III d) Apenas II e III e) I, II e III

280

GEOGRAFIA
Instruo: O mapa abaixo refere-se questo de nmero 29.

UFRGS

29. Associe os nmeros que identificam as regies apresentadas no mapa s diferentes reas da organizao do espao econmico brasileiro. ( ( ( ( ( ) rea pouco povoada, com economia estagnada ) reas agropecurias modernas ) reas agropecurias tradicionais ) rea pouco povoada, com paisagens naturais pouco alteradas ) rea de industrializao dinmica e intensa urbanizao

A seqncia numrica correta, de cima para baixo, : a) 1 - 2 - 3 - 4 - 5 b) 2 - 4 - 5 - 1 - 3 c) 3 - 4 - 5 - 2 - 1 d) 3 - 1 - 4 - 2 - 5 e) 2 - 1 - 5 - 3 - 4

30. Assinale a alternativa que preenche corretamente as lacunas do trecho abaixo. , comum o sem-terra ser filho de agricultor que no possui terra suficiente para partilhar. , entre os sem-terra, predominam os bias-frias das grandes fazendas de cana-de-acar. , entre os sem-terra predominam, em funo da seca, agricultores e emigrantes do meio rural, que, por no encontrarem meio de sobrevivncia urbana, retornam ao campo. a) b) c) d) e) Em Santa Catarina - Em Rondnia - No Mato Grosso Em Minas Gerais - Na Bahia - Em Tocantins No Rio Grande do Sul - Em So Paulo - Em Pernambuco Em Rondnia - Em Tocantins - Em Sergipe No Rio Grande do Sul - No Rio de Janeiro - No Par

281

GEOGRAFIA

UFRGS

31. A urbanizao acelerada ocorrida no Brasil durante os anos 50 e 60 provocou, entre outros fatores, o surgimento de favelas nas grandes cidades. Entre as caractersticas fsico-sociais das favelas, relacionadas abaixo, assinale a INCORRETA. a) b) c) d) e) Alto ndice de mortalidade infantil. Significativo adensamento populacional. Carncia de saneamento bsico. Populao empregada na economia informal. Ocupaes regulares de terrenos pblicos e privados.

32. A geografia da populao tem por objetivo estudar os aspectos demogrficos de distribuio e mobilidade da populao em nvel mundial e regional, bem como o crescimento demogrfico ao longo da histria da humanidade. Neste particular, entre as causas principais das grandes mortalidades do passado esto as doenas a) b) c) d) e) epidmicas, endmicas e guerras. infecciosas, o tabagismo e guerras. degenerativas, endmicas e guerras. degenerativas, infecciosas e endmicas. degenerativas, infecciosas e o tabagismo.

33. Assinale a afirmativa correta a respeito das associaes, blocos ou organizaes internacionais. a) Entre as vrias organizaes que congregam os diversos pases, nenhuma tem um carter to regional quanto a ONU. b) A Unio Europia caracteriza-se por ser uma associao que busca a formao de uma Federao de Estados, cada um com sua autonomia, mas sem barreiras circulao de mercadorias e pessoas entre os pases. c) O Grupo dos sete pases mais ricos, o G7, funciona como um comit que encaminha decises prvias OTAN ou seus organismos subjacentes, como o Banco Mundial ou a Organizao Mundial do Comrcio. d) Os dois novos pases membros do Mercosul so Chile e Equador, ambos considerados oficialmente parceiros comerciais, fato que viabilizou o rpido aceite de suas participaes pelos demais. e) O NAFTA, associao de livre comrcio formada por Estados Unidos, Canad, Mxico e Groenlndia, no tem apresentado viabilidade de unio poltica, devido s grandes diferenas lingsticas entre as diversas nacionalidades.

282

GEOGRAFIA
Instruo: O mapa a seguir refere-se questo de nmero 34.

UFRGS

34. A parte sombreada do mapa acima mostra a rea ocupada por uma etnia que luta para conquistar sua autonomia e constituir um pas numa regio rica em petrleo. Recentemente (setembro de 1996), o governo norte-americano atacou tropas iraquianas que invadiram a rea ocupada por este povo. Que povo este? a) b) c) d) chechenos palestinos chipriotas curdos d) curdos e) bsnios

Instruo: O mapa abaixo refere-se questo de nmero 35. 35. A respeito das regies A e B, indicadas no mapa, so feitas as seguintes afirmaes. I - Na regio B, entre as economias de alta renda encontram-se os pases exportadores de petrleo. II - Na regio A, encontram-se as economias de mais elevado PNB e de maior consumo de bens industriais. III - Na regio B, encontram-se os pases em que mais da metade da renda nacional est concentrada no segmento constitudo pelos 20% mais ricos da populao. IV - Na regio A, encontram-se os pases em que mais da metade da renda nacional est concentrada no segmento constitudo pelos 20% mais ricos da populao. Quais esto corretas? a) Apenas II e III b) Apenas II e IV c) Apenas I e IV d) Apenas I, II e III e) Apenas I, II e IV

283

GEOGRAFIA
Nas questes de 1 a 15, marque:

UnB

Itens Certos, na coluna I; Itens Errados, na coluna II. Use, para as devidas marcaes, a Folha de Rascunho e, posteriormente, a Folha de Respostas. Questo 1 A estrutura espacial do territrio norte-americano originou-se da modernizao industrial, ao trmino da Guerra de Secesso, que destruiu o poder poltico do Sul e promoveu a unidade social e econmica da nao, sob a hegemonia das elites do Norte. Com o auxlio desta afirmao, julgue os itens abaixo. (0) A ocupao histrica do territrio norte-americano teve como foco inicial a costa do Pacfico, realizando-se, portanto, no sentido oeste-leste. (1) No seu incio, o desenvolvimento industrial norte-americano foi protegido por elevadas taxas alfandegrias e financiado pelas exportaes agrcolas. (2) A economia ps-industrial norte-americana assenta-se no domnio do capital financeiro, na expanso dos servios e do comrcio e na transferncia da fora de trabalho do setor secundrio para o tercirio. Questo 2 A qualidade das guas dos lagos naturais ou artificiais suscetvel deteriorao, como conseqncia das atividades humanas. Partindo desta afirmao, julgue os itens seguintes. (0) Uma forma de degradao fsica o assoreamento produzido por um aumento de sedimentos nos rios que alimentam os lagos, em virtude da urbanizao e da agricultura. (1) A eutrofizao dos lagos um processo natural e lento, produzido por processos geolgicos e biolgicos. (2) Nos ecossistemas lacustres, a fotossntese ocorre na zona profunda. Questo 3 Sabe-se que o desmatamento produz conseqncias sobre o clima da regio amaznica, causando impactos de diversas naturezas. Com relao a esses impactos, julgue os itens que se seguem. (0) O desmatamento, de maneira geral, modificar o tempo de permanncia da gua na bacia hidrogrfica, alterando os componentes do balano hdrico. (1) So previsveis inundaes mais intensas nas pocas chuvosas. (2) Espera-se um maior ndice de precipitao pluviomtrica em virtude do aumento dos processos de evaporao e de transpirao.

284

GEOGRAFIA
Questo 4

UnB

O consumo de energia determinado pelo modelo de desenvolvimento econmico e social adotado em cada pas. Com o auxlio desta afirmao, julgue os itens a seguir. (0) Nas sociedades primitivas, a fonte de energia bsica a fora muscular de seus integrantes. (1) No incio do sculo, o petrleo supria a totalidade das necessidades mundiais de energia. (2) A baixa qualidade do carvo mineral e uma estrutura de transporte deficiente foram fatores que dificultaram a produo e o consumo desse recurso energtico brasileiro. (3) Do ponto de vista ambiental, o programa governamental Prolcool favoreceu o desequilbrio ecolgico ao fomentar a prtica da monocultura. Questo 5 A criao do Mercado Comum do Sul, Mercosul, representa uma mudana nos limites fsicos, econmicos e polticos. na verdade a superao da escala nacional. Com relao a esse processo de integrao, julgue os itens abaixo. (0) (1) (2) (3) No Brasil, o comrcio exterior representa a maior parcela da riqueza nacional. O ncleo geoeconmico do Mercosul a regio platina. As prioridades geopolticas do Chile explicam a sua adeso ao Mercosul. A posio geogrfica do Uruguai determina a sua condio de elo entre as principais potncias do Cone Sul.

Questo 6 Em Geografia, a escala expressa a relao existente entre as dimenses apresentadas nos mapas e os seus valores reais correspondentes, no terreno. Partindo desta afirmao, julgue os itens a seguir. (0) A escala de um mapa pode ser determinada a partir da comparao da distncia real entre dois pontos do terreno com a respectiva distncia grfica no mapa. (1) Uma escala ser tanto maior quanto menor for o valor do denominador da frao que representa. (2) A rea que representa um terreno, no mapa, aumenta medida que aumenta a escala deste. (3) Um centmetro em mapas de escala 1:12.500 corresponde a 125 km, no terreno.

285

GEOGRAFIA
Questo 7

UnB

Com relao diviso internacional do trabalho e aos fluxos internacionais de capital e de mercadorias, julgue os itens seguintes. (0) No Brasil, a indstria de autopeas cresceu apoiada nas demandas criadas pelas empresas multinacionais do setor automobilstico. (1) O peso das importaes brasileiras recai sobre produtos bsicos, matrias-primas e combustveis e sobre bens de capital. (2) As regies do globo que concentram maior atividade produtiva tendem a canalizar para si o fluxo mais intenso de capitais e de mercadorias. (3) Atualmente, os pases so classificados, na diviso internacional do trabalho, pelos produtos que exportam ou importam, como, por exemplo, matrias-primas ou manufaturados. (4) O Brasil um pas de industrializao recente devido ao rpido crescimento industrial ocorrido somente aps a Segunda Guerra Mundial.

Questo 8 Apesar de o Brasil possuir recursos naturais e humanos indispensveis implantao de uma importante e prspera agricultura, no faltam distores, conflitos e degradao ambiental em seu espao rural. Com referncia atividade agrcola no pas, julgue os itens que se seguem. (0) A dimenso de um mdulo rural fixada de acordo com a regio e o tipo de explorao. (1) A expanso da fronteira agrcola, a partir dos anos 70, propiciou uma reforma na estrutura fundiria e uma distribuio mais justa de terras. (2) A organizao do trabalho nos minifndios baseia-se fundamentalmente na famlia, incluindo o proprietrio e seus dependentes, que trabalham sem remunerao. (3) As grandes propriedades, em geral, orientam as suas atividades para a produo de gneros alimentcios como forma de garantir o abastecimento da maioria da populao. (4) A queimada uma prtica agrcola rudimentar, j erradicada do pas.

286

GEOGRAFIA
Questo 9

UnB

Em 1970, a grande de So Paulo tinha 43,5% das indstrias do pas, quantidade que caiu para 26,2% em 1990. O interior [de So Paulo], h 26 anos, era dono de 14,7% das fbricas brasileiras. Duas dcadas depois, a regio [o interior paulista] tem 23% das plantas de transformao operando no pas.
DAmorin, Sheila Marcha para o Interior In: Correio Braziliense, 14/7/96, p. 26.

Com o auxlio das informaes contidas na citao acima, julgue os itens a seguir, relativos industrializao brasileira. (0) Essa desconcentrao industrial deve-se basicamente ao que se chama de deseconomia de escala. (1) Na regio Sul do pas, verifica-se um arrefecimento nas atividades industriais, em funo da transferncia de empresas para o interior paulista. (2) No contexto industrial brasileiro, So Paulo perdeu lugar para Minas Gerais, que desponta, hoje, como o maior plo industrial do pas. (3) Iseno de impostos, infra-estrutura urbana e oferta de energia esto entre os atrativos oferecidos pelos estados para a instalao de indstrias em seus municpios. Questo 10 A partir de 1950, as taxas de crescimento populacional mundial e regionais exibem considerveis mudanas de grande impacto no ambiente social e fsico dos pases. Com efeito, essas mudanas refletem transformaes no que diz respeito fertilidade e mortalidade que, em conjunto, determinam o crescimento da populao mundial. Com relao variao das taxas de mortalidade e de fertilidade mundiais, julgue os seguintes itens. (0) As causas verificadas para o declnio da fertilidade no mundo esto fortemente correlacionadas com a urbanizao, com as mudanas estruturais na fora de trabalho, com o aumento da escolaridade e com o acesso informao e aos meios contraceptivos. (1) A queda na taxa de mortalidade tem sido um fenmeno observado exclusivamente em pases desenvolvidos. (2) Pases da frica, como a Nigria e o Congo, esto entre os que apresentam as menores taxas de crescimento populacional no mundo, em funo de alta taxa de mortalidade existente. Questo 11 A inter-relao funcional entre o Sol e a Terra de vital importncia, uma vez que a vida terrestre depende da energia solar. A respeito dos movimentos de rotao e de revoluo da Terra, que alteram constantemente a sua posio em relao ao Sol, julgue os itens abaixo. (0) O movimento aparente do Sol, da Lua e das estrelas oposto ao verdadeiro sentido da rotao terrestre. (1) A variao da exposio da Terra luz solar influencia o movimento dos ventos. (2) Os raios solares que incidem na superfcie da Terra o fazem em diferentes ngulos de incidncia, de acordo com a latitude, o que se traduz em diferentes quantidades de absoro de calor por parte da Terra.

287

GEOGRAFIA
Questo 12

UnB

Globalizao e regionalizao so processos em curso no mundo. Se, por um lado, a globalizao de mercados avana sobre fronteiras entre pases, por outro, a regionalizao cria barreiras com o objetivo de proteger, de fortalecer as grandes zonas internacionais estabelecidas. Com referncias s atuais tendncias, julgue os itens seguintes. (0) Na chamada nova ordem mundial, elimina-se a hegemonia de naes ou centros de poderio econmico, tecnolgico e poltico-administrativo, em face do processo de globalizao. (1) Um fato resultante da regionalizao a ocorrncia da produo e do consumo em um mesmo lugar. (2) Um efeito positivo da globalizao tem sido, com certeza, a diminuio da discrepncia, na qualidade de vida, entre a populao dos pases pobres e a dos pases ricos. Questo 13 O artigo 225 da Constituio Federal do Brasil declara que Todos tm direito ao meio ambiente ecologicamente equilibrado, [sendo este] bem de uso comum do povo e essencial sadia qualidade de vida.... Com base nas polticas ambientais do pas, julgue os itens que se seguem. (0) A Floresta Amaznica brasileira, a Mata Atlntica, a Serra do Mar, o Pantanal MatoGrossense e a zona costeira so considerados patrimnio nacional, o que significa que deveriam permanecer intactos. (1) As florestas e as demais formas de vegetao natural localizadas ao redor de lagoas, lagos ou reservatrios de gua naturais ou artificiais , so consideradas de preservao permanente, no podendo, em hiptese alguma, ser desmatadas, como forma de proteo contra a eroso. (2) Aquele que explorar recursos minerais est obrigado a recuperar o meio ambiente degradado. Questo 14 Observe o mapa ao lado. Julgue os itens a seguir, relativos regio destacada. (0) Do ponto de vista natural, duas caractersticas marcam a regio: climas ridos e semi-ridos e a existncia de relevo acidentado na sua poro setentrional. Tais caractersticas tiveram importncia nas estratgias utilizadas em vrios conflitos nela ocorridos. (1) A regio conhecida como o mundo rabe islmico. Nos ltimos anos, o islamismo tem sido apontado, entre as grandes religies, como a que apresenta maior crescimento em todo o mundo. (2) A importncia estratgica da regio est no fato de haver reas grandes produtoras e exportadoras de petrleo, bem como de possuir rotas de navegao importantes para o mundo. (3) Os conflitos religiosos existentes tm desdobramentos fora dos limites geogrficos da regio.

288

GEOGRAFIA
Questo 15

UnB

O conhecimento das tendncias demogrficas de um pas merece ser objeto de estudo, em razo das suas implicaes nas demandas por polticas pblicas. De acordo com a anlise dos resultados do Censo Demogrfico de 1991 e da Pesquisa Nacional por Amostra de Domiclios (PNDA) de 1992 e 1993, julgue os seguintes itens. (0) A regio Nordeste, ainda hoje, pode ser considerada como uma regio na qual se verifica perda de populao. (1) Nas regies Norte e Centro-Oeste so aquelas que apresentam os menores saldos positivos migratrios. (2) A reduo do peso relativo da populao jovem, em funo do declnio da fecundidade, combinada com aumentos graduais da expectativa de vida, culminou na diminuio da participao de adultos e idosos no total da populao brasileira. (3) O nmero mdio de filhos das mulheres brasileiras vem se reduzindo, entre outras causas, pelo uso de anticoncepcionais.

289

GEOGRAFIA
1. Observe o mapa e as gravuras. No mapa do Brasil, os algarismos romanos indicam os tipos climticos e as gravuras numeradas de 1 a 3 representam alguns tipos de vegetao. Assinale a alternativa que apresenta a correspondncia correta entre os tipos climticos e as paisagens vegetais. a) b) c) d) e) I-1; II-2; III-3. II-1; III-3; IV-2. V-1; III-2; IV-3. II-3; III-1; V-2. I-3; II-2; V-3.

UNESP
F A S E

2. Observe a tabela. As maiores economias do mundo - 1992.


PASES PNB TOTAL RENDA PER CAPITA

(bilhes de dlares)

(em dlares)

Estados Unidos Japo Alemanha Frana Itlia Reino Unido Canad Espanha Rssia

5.686 3.338 1.517 1.161 1.072 964 569 487 480

22.560 26.920 23.650 20.600 18.580 16.750 21.260 12.460 3.220

Fonte: Vesentini, J.W. - A Nova Ordem Mundial. Editora tica, 1996.

Com a crise verificada no mundo socialista, um continente se destacava, em 1992, pelo nmero de pases colocados entre os nove de melhor desempenho econmico. Trata-se do Continente a) Europeu. b) Asitico. c) Americano. d) Africano. e) Euro-Asitico.

3. No Brasil, o Planalto Atlntico obriga a elevao dos ventos vindos do oceano carregados de umidade. Ao encontrar camadas mais frias de ar, o vapor da atmosfera se condensa e se precipita em forma de chuva. Estas caractersticas individualizam as chuvas a) frontais. b) polares. c) mediterrneas. d) orogrficas. e) trmicas.

290

GEOGRAFIA
4. Leia o texto.

UNESP
F A S E

... E se somos Severinos iguais em tudo na vida, morremos de morte igual, mesma morte severina: que a morte de que se morre de velhice antes dos trinta, de emboscada antes dos vinte (de fraqueza e de doena que a morte severina ataca em qualquer idade, e at gente no nascida). Somos muitos Severinos iguais em tudo e na sina: a de abrandar estas pedras suando-se muito em cima, a de tentar despertar terra sempre mais extinta, a de querer arrancar algum roado da cinza. Mas, para que me conheam melhor Vossas Senhorias e melhor possam seguir a histria de minha vida, passo a ser o Severino que em Vossa presena emigra.

Esta pequena parte do Auto de Natal pernambucano - Morte e Vida Severina, de Joo Cabral de Melo Neto - retrata a realidade do nordeste do Brasil. Assinale a alternativa que melhor expressa tal realidade. a) b) c) d) e) Audes, desnutrio e imigrao. Solo pedregoso, imigrao e doenas. Desnutrio, emigrao e escassez de gua. Solo pedregoso, emigrao e alta esperana de vida. Escassez de gua, roado e imigrao.

5. No Brasil, o desemprego um dos maiores problemas enfrentados pela populao. Pesquisas da Fundao Seade e do Dieese mostram que na regio metropolitana de So Paulo a taxa de desemprego atingiu, em maio de 1996, nvel recorde de 16,2%, totalizando 1.363.000 pessoas, em idade ativa, sem trabalho.
GRANDE SO PAULO Taxa de desemprego 1995/96

Pelas informaes do grfico, conclui-se que: a) b) c) d) e) a elevao da taxa de desemprego foi sempre contnua no perodo considerado. os meses correspondentes ao ano de 1995 apresentaram as maiores taxas de desemprego. os meses de agosto/95 e janeiro/96 foram os que apresentaram os maiores ndices de desemprego. o ltimo trimestre considerado revela as menores taxas de desemprego do perodo. a taxa de desemprego acelerou-se abruptamente no ltimo quadrimestre do perodo considerado.

291

GEOGRAFIA

UNESP
F A S E

6. O grfico representa a distribuio da temperatura e da pluviosidade em Bombaim, ndia. Assinale a alternativa que indica o tipo climtico caracterstico da rea onde se localiza a referida cidade. a) Monnico. b) Semi-rido. c) Temperado. d) Mediterrneo. e) Equatorial.

7. Examine o grfico e assinale a alternativa correta.


BRASIL. PERCENTUAL DE PESSOAS QUE TRABALHAVAM 40 HORAS SEMANAIS OU MAIS COM RENDIMENTO DE TRABALHO INFERIOR A 1 SALRIO MNIMO EM RELAO POPULAO OCUPADA, EM 1990.

Fonte: IBGE/PNAD 1990.

a) A sazonalidade do trabalho responsvel pelo menor valor percentual de pessoas com rendimento inferior a um salrio mnimo apresentado pela regio sul. b) O percentual de pessoas com rendimento inferior a um salrio mnimo registrado na regio sudeste indica que ela era o plo de atrao das pessoas que almejavam melhores salrios. c) Por ser uma regio de desenvolvimento recente, o centro-oeste apresenta o mais elevado percentual de pessoas com rendimento de trabalho inferior a um salrio mnimo. d) O maior valor percentual registrado na regio nordeste significa que ela era um plo de atrao para os trabalhadores que buscavam melhor remunerao. e) Depois do nordeste, era a regio norte que apresentava o maior percentual de pessoas recebendo menos de um salrio mnimo por 40 horas de trabalho.

292

GEOGRAFIA

UNESP
F A S E

8. Debates realizados no Habitat 2, em Istambul, Turquia (1996), destacaram que os veculos particulares so os principais causadores de poluio, sendo responsveis por 54% do monxido de carbono lanado na atmosfera, o que signifca cerca de 1,6 trilho de toneladas de CO por ano. Calculou-se que na virada do sculo existam aproximadamente 600 milhes de carros circulando no mundo e a estimativa que totalizem quase 800 milhes em 2010. Observe o grfico.

Assinale a alternativa que indica corretamente as reas que estaro em situao mais crtica pela poluio por CO em 2010. a) b) c) d) e) sia, Amrica Latina, Oceania e frica. Amrica do Norte, ex-URSS e Amrica Latina. sia, Europa, Oceania e frica. sia, ex-URSS e Amrica Latina. Europa, Amrica do Norte e sia.

9. Assinale a alternativa que indica corretamente as principais caractersticas da populao australiana. a) b) c) d) e) Predominantemente branca, numerosa e bem distribuda no espao. De origem amerndia, rural, com altas taxas de natalidade. Predominantemente branca, urbana, com elevada esperana de vida. De origem britnica, urbana, com altas taxas de mortalidade infantil. De origem americana, numerosa e concentrada na poro centro-norte do pas.

293

GEOGRAFIA

UNESP
F A S E

10. O mundo moderno conta com o transporte atravs de dutos. Este sistema foi desenvolvido, principalmente, para transportar a longas distncias dois produtos muito importantes para a economia neste final de sculo. Esses produtos so a) carvo e gs natural. b) petrleo e carvo. c) gs natural e urnio. 11. Examine o grfico. No perodo considerado, a) houve um contnuo dficit na balana comercial brasileira. b) houve um contnuo crescimento no valor das exportaes. c) a maior movimentao financeira ocorreu no ano de 1991. d) os maiores saldos na balana comercial ocorreram em 1990 e 1993. e) o menor valor de exportao brasileira verificou-se em 1992. d) nquel e petrleo. e) petrleo e gs natural.

12. O grfico representa a evoluo das reas de pastagens naturais e cultivadas no Estado de So Paulo, no perodo de 1983/1994. correto afirmar que a) o aumento contnuo das reas de pastagens cultivadas est relacionado criao intensiva de sunos. b) as reas de pastagens naturais foram sempre o dobro daqueles ocupadas por pastagens cultivadas. c) o aumento contnuo das reas de pastagens cultivadas est relacionado criao intensiva de bufalinos. d) o aumento da rea de pastagens cultivadas est relacionado criao intensiva de gado bovino. e) as reas de pastagens cultivadas aumentaram continuamente no perodo considerado.

294

GEOGRAFIA

UNESP
F A S E

1. formada por terrenos sedimentares recentes, entre 100 e 200 metros acima do nvel do mar, 2 com rea superior a 100.000 km e cortada por muitos rios. Na estao chuvosa, de setembro a abril, os rios transbordam, inundando grandes reas, chegando, s vezes, a trs metros de altura, formando inmeros lagos de muitos quilmetros. Na vazante, o capim brota formando extensas pastagens e, em alguns locais, formam-se salinas naturais. a) Qual a rea brasileira descrita e quais as caractersticas de sua vegetao? b) Como o homem aproveita economicamente essa regio? 2. Observe a tabela. Maiores Cidades do Mundo - 1995 e 2015, em milhes de habitantes.
1995 Tquio, Japo So Paulo, Brasil Nova York, EUA Cid. do Mxico, Mxico Bombaim, ndia Xangai, China Los Angeles, EUA Pequim, China Calcut, ndia Seul, Coria do Sul 26,8 16,4 16,3 15,6 15,1 15,1 12,4 12,4 11,7 11,6 Tquio, Japo Bombaim, ndia Lagos, Nigria Xangai, China Jacarta, Indonsia So Paulo, Brasil Karachi, Paquisto Pequim, China Dacca, Bangladesh Cid. do Mxico, Mxico 2015 28,7 27,4 24,4 23,4 21,2 20,8 20,6 19,4 19,0 18,8
Fonte: ONU, 1995.

Estimativas feitas pela ONU indicam que no ano 2025 dois teros da populao mundial estaro vivendo nas cidades. a) Identifique as cidades includas entre as 10 maiores do mundo que, em 1995, no se localizavam nas reas mais ricas do planeta. Destaque trs aspectos relacionados qualidade de vida, que so caractersticos das referidas cidades. b) Compare os dados de 1995 com a projeo feita para o ano 2015 e destaque as mudanas que, possivelmente, ocorrero quanto localizao geogrfica das maiores concentraes urbanas.

295

GEOGRAFIA
3. Observe o mapa e responda.
EROSO URBANA EM MUNICPIOS DO ESTADO DE SO PAULO - 1996

UNESP
F A S E

POPULAO DOS MUNICPIOS ONDE A SITUAO MAIS CRTICA Menos de 20.000 hab. 20.000 a 50.000 hab. 51.000 a 100.000 hab. Mais de 100.000 hab. 0 100 km

Fonte: IPT, 1996.

A eroso, tanto rural como urbana, um problema mundial que resulta das interaes sociedade-natureza. a) Descreva trs causas e trs conseqncias do processo erosivo. b) Com base no mapa, relacione a ocorrncia de eroso urbana com a populao dos municpios onde a situao mais crtica. 4. O Oriente Mdio uma rea estratgica do mundo atual, no s por ser uma regio de passagem entre a Europa, a sia e a frica, mas tambm por possuir mais da metade das reservas de petrleo do mundo. Esta riqueza no est igualmente distribuda pelos pases que o constituem. Considere estas informaes e responda. a) Quais so os pases do Oriente Mdio que possuem as maiores reservas de petrleo? Por que a maioria de suas populaes no se beneficia igualmente das divisas geradas por este importante recurso? b) Por que Arbia Saudita e Ir disputam a soberania sobre o Estreito de Ormuz? 5. O maior problema do Serto Nordestino relaciona-se gua, pois no clima semi-rido as longas estiagens secam o leito dos rios. Contudo, s vezes, no meio de uma rea seca encontramse terrenos mais midos, que apresentam fontes dgua permanentes. a) Como se denominam e como se formam essas reas mais midas no Serto Nordestino? b) Como elas so aproveitadas economicamente?

296

GEOGRAFIA
6. Analise a tabela. Temperatura e Umidade Relativa em algumas capitais brasileiras em 1992.
Capital Temperatura do Ar em C Mxima Mnima Absoluta Absoluta 36,2 31,6 35,3 32,8 32,2 8,9 7,0 4,1 20,6 21,0 Umidade Relativa em %

UNESP
F A S E

Goinia Braslia Campo Grande So Lus Fortaleza

63 66 75 87 77
Fonte: IBGE, Brasil em Nmeros, 1994.

a) Identifique os conjuntos de cidades considerando as amplitudes trmicas e os valores de umidade relativa do ar. b) Explique o fator responsvel pela distribuio e caractersticas dos dados apresentados. 7. A anlise do crescimento infantil uma das formas de avaliar o ndice de desnutrio em um pas. De acordo com o padro internacional de crescimento recomendado pela Organizao Mundial de Sade, percentuais de crianas de baixa estatura superiores a 3% indicam desnutrio da populao infantil (OMS, 1986). O grfico representa este indicador nas reas urbanas e rurais de regies brasileiras e em alguns pases em 1989.
Porcentagem de crianas com baixa estatura no Brasil e alguns pases-1989.
%

a) Que fatores explicam a situao das regies sudeste, sul e centro-oeste, em comparao com os nveis do Chile e da Costa Rica? b) Compare os percentuais encontrados na populao rural das regies brasileiras com os de Mali e Gana e explique seu significado.

297

GEOGRAFIA
8. Leia o texto e responda.
Vaca Louca ou Tecnologia Maluca? Francisco Graziano Neto

UNESP
F A S E

Existe na Inglaterra um pequeno grupo de agricultores que est sendo beneficiado com a doena da vaca louca. So 200 produtores de carne e leite que conduzem seu rebanho segundo as regras da agricultura orgnica, onde se exige o pastoreio natural e, quando acrescentadas, silagem e protenas vegetais advindas de campos isentos de produtos qumicos. Nada de inseticidas, herbicidas e adubos solveis. Entre 140 mil pecuaristas ingleses, os orgnicos respondem por apenas 12 mil dos 3,3 milhes de reses abatidas anualmente. Poucos, mas felizes. A BSE (Bovine Spongiform Encephalophthy) est causando a maior crise da pecuria inglesa. ...O grande erro est na origem, na tecnologia de produo do gado europeu. Mtodos antinaturais, visando unicamente e rentabilidade imediata, foram sendo introduzidos na criao animal, transformando a pecuria numa espcie de fbrica. Numa triste indstria. Nesse processo tecnolgico, alcunhado de moderno, leis da natureza foram desprezadas. Os animais foram confinados, no pastam mais, poucos vem o sol. Enjaulados, passaram a comer restos de aves, vsceras de carneiro, crebros de boi. De herbvoros foram transformados em saprfitos, ou canibais. O estresse aumentou e as doenas atacaram. Os antibiticos e uma parafernlia de drogas entraram em ao. Tudo foi se artificializando. Deu na vaca louca... ... A transio da tecnologia maluca para a criao equilibrada ser difcil. Depender da ao do governo, incentivando os agricultores necessria reverso tecnolgica na criao de gado. Depender do comportamento dos consumidores e da luta dos ambientalistas. Mas que ningum duvide: os novos rumos da pecuria europia esto traados. E so irreversveis. Bom para os animais, que sero melhor tratados. Excelente para os humanos, que sero melhor alimentados. timo para os pecuaristas, que se livraro dessas doenas terrveis. Junto com os produtores orgnicas, milhares de pequenos agricultores familiares, que foram considerados sentimentais porque mantiveram seus rebanhos pastoreando, se fortalecero no campo. Fonte: Folha de S. Paulo, 14/04/96.

a) A que etapa do processo de modernizao da agricultura o autor est se referindo? Que setor da agricultura est sendo alvo desse processo? b) Que alternativa apontada pelo autor para enfrentar as conseqncias danosas decorrentes de tal processo? 9. A partir do sul de Santa Catarina at o extremo sul do pas, a plancie litornea alarga-se, mudando a paisagem. Aparecem faixas de areia depositadas paralelamente ao litoral e, entre elas e o continente, formam-se lagoas. a) Como se denominam estas faixas de areia e qual o estado brasileiro onde se localizam as maiores lagoas por elas formadas? b) Explique o processo de formao das lagoas.

298

HISTRIA

CESGRANRIO

1. Entre os sculos XV e XVIII, a transio do feudalismo para o capitalismo, no mundo ocidental, engloba um conjunto de transformaes econmicas e sociais, entre as quais identificamos corretamente a(o): a) b) c) d) e) fragmentao da propriedade fundiria senhorial e monrquica. substituio da produo das manufaturas pelo sistema de corporaes de ofcios. supremacia das rotas terrestres e mediterrneas no comrcio com o oriente. fortalecimento dos laos de servido e vassalagem. desenvolvimento da vida urbana atravs das atividades comerciais.

2. A administrao colonial portuguesa no Brasil se organizava em vrios nveis, cuja autonomia e jurisdio variaram em diversos momentos da colonizao. Assinale a opo que expressa corretamente uma caracterstica dessa administrao. a) H crescente centralizao e reforo da autoridade dos governadores e vice-reis no sculo XVIII, principalmente aps o perodo pombalino. b) As Cmaras Municipais sempre foram o principal ncleo de poder da colnia, por exercerem as funes policiais e judicirias. c) Os Governadores Gerais tiveram funo essencialmente militar, mantendo-se com os Donatrios o controle da administrao fiscal e tributria. d) As autoridades judicirias como os Juzes, Ouvidores e Ministros dos Tribunais eram independentes, reportando-se diretamente ao Rei. e) As necessidades de conquista do territrio generalizaram a prtica de conceder total autonomia administrativa aos que dirigiam expedies de desbravamento. 3. A Revoluo Cientfica, ocorrida na Europa Moderna entre os sculos XVI e XVII, caracterizou-se por: a) acentuar o esprito crtico do homem atravs do desenvolvimento da cincia experimental. b) reforar as concepes antinaturalistas surgidas nos primrdios do Renascimento. c) comprovar a tese de um universo geocntrico contrria explicao tradicional aceita pela Igreja Medieval. d) negar os valores humanistas, fortalecendo assim as idias racionalistas. e) confirmar os fundamentos lgicos e empiristas da filosofia escolstica em sua crtica aos dogmas catlicos medievais. 4. No sculo XVII, as invases do nordeste brasileiro pelos holandeses estavam relacionadas s mudanas do equilbrio comercial entre os pases europeus porque: a) a Holanda apoiava a unio das monarquias ibricas. b) a aproximao entre Portugal e Holanda era uma forma de os lusos se liberarem da dependncia inglesa. c) as Companhias das ndias Orientais e Ocidentais monopolizavam o escambo do pau-brasil. d) os holandeses tinham grandes interesses no comrcio do acar. e) Portugal era tradicionalmente rival dos holandeses nas guerras europias.

299

HISTRIA

CESGRANRIO

5. Durante o sculo XVII, ocorreram diversas transformaes polticas e econmicas na Inglaterra, que a consolidaram como uma potncia de projeo mundial nos sculos seguintes. Marque a opo que apresenta corretamente um dos fatores que se encontram na origem dessa posio preponderante. a) Instituio do Ato de Supremacia, que criou a Comunidade Britnica (Commonwealth), por Henrique VIII. b) Restaurao do Absolutismo ingls e sua poltica expansionista com a dissoluo do Parlamento controlado pela nobreza conservadora, por Carlos II. c) Declarao do Ato de Tolerncia, que instituiu o catolicismo como religio oficial, encerrando as guerras religiosas, por Guilherme III. d) Extino da monarquia parlamentar constitucional com a vitria dos segmentos liberais e burgueses na Revoluo Gloriosa. e) Promulgao dos Atos de Navegao durante a Repblica Puritana liberada por Oliver Cromwell. 6. A poltica econmica do Estado Absolutista, o Mercantilismo, reuniu prticas e doutrinas que, em suas diversas modalidades entre os sculos XVI e XVIII, caracterizou-se por um(a): a) liberalismo econmico como forma de manuteno da aliana poltica do Rei com os segmentos burgueses. b) protecionismo alfandegrio atravs de proibies das exportaes que visava ao equilbrio da balana comercial do Estado. c) intervencionismo estatal nas atividades comerciais lucrativas que proibiu a concesso de monoplios a grupos privados. d) expanso do poderio naval como garantia das comunicaes martimas entre as metrpoles e seus imprios coloniais. e) restrio dos privilgios senhoriais relacionados participao da nobreza no comrcio ultramarino e nas companhias comerciais do Estado, tais como a Companhia das ndias Orientais e das ndias Ocidentais. 7. A formao do territrio brasileiro no perodo colonial resultou de vrios movimentos expansionistas e foi consolidada por tratados no sculo XVIII. Assinale a opo que relaciona corretamente os movimentos de expanso com um dos Tratados de Limites. a) A expanso da fronteira norte, impulsionada pela descoberta das minas de ouro, foi consolidada nos Tratados de Utrecht. b) A regio missioneira no sul constituiu um caso parte, s resolvido a favor de Portugal com a extino da Companhia de Jesus. c) O Tratado de Madri revogou o de Tordesilhas e deu ao territrio brasileiro conformao semelhante atual. d) O Tratado do Pardo garantiu a Portugal o controle da regio das Misses e do rio da Prata. e) Os Tratados de Santo Ildefonso e Badajs consolidaram o domnio portugus no sul, passando a incluir a regio platina.

300

HISTRIA

CESGRANRIO

8. A transferncia da corte portuguesa para o Brasil, em 1808, acelerou transformaes que favoreceram o processo de independncia. Entre essas transformaes, podemos citar corretamente a(s): a) b) c) d) e) ampliao do territrio com a incorporao definitiva de Caiena e da Cisplatina. implantao, na colnia, de vrios rgos estatais e de melhoramentos como estradas. reduo da carga tributria sobre a colnia, favorecendo-lhe a expanso econmica. poltica das Cortes portuguesas de apoio autonomia colonial. restries comerciais implantadas por interesse dos comerciantes portugueses.

9. Na primeira metade do sculo XIX, diversos movimentos pela independncia eclodiram nas colnias espanholas da Amrica, marcando a luta de seus povos contra o domnio da metrpole Ibrica. Marque a opo que se refere, corretamente, a um desses movimentos. a) Na Argentina, os comerciantes portenhos aliados ao lder militar Manuel Belgrano extinguiram os Cabildos e as Juntas Governativas, controladas por representantes da Coroa Espanhola. b) No Chile, a forte presena militar inglesa aquartelada no norte do pas impediu o avano do movimento de independncia formado por segmentos populares liderados por Bernardo O. Higgins que, derrotado, exilou-se na Venezuela. c) No Mxico, a elite criolla, que ocupava os altos cargos da administrao colonial, aliada aos espanhis da metrpole, proclamou Fernando VII da Espanha como Imperador do Mxico, sobrevivendo monarquia mexicana at o advento da Revoluo Zapatista. d) No Peru, o principal centro de resistncia espanhola tornou-se independente aps a tomada conjunta de Lima pelos exrcitos de Bolvar e San Martin, tornando-se este ltimo o primeiro presidente perptuo da Repblica Peruana. e) No Uruguai, a conquista da independncia no encerrou o poder personalista dos caudilhos, mas fortaleceu os segmentos burgueses atuantes em Montevidu. 10. A Constituio imperial brasileira, promulgada em 1824, estabeleceu linhas bsicas da estrutura e do funcionamento do sistema poltico imperial tais como o(a): a) equilbrio dos poderes com o controle constitucional do Imperador e as ordens sociais privilegiadas. b) ampla participao poltica de todos os cidados, com exceo dos escravos. c) laicizao do Estado por influncia das idias liberais. d) predominncia do poder do Imperador sobre todo o sistema atravs do Poder Moderador. e) autonomia das Provncias e, principalmente, dos Municpios, reconhecendo-se a formao regionalizada do pas.

301

HISTRIA

CESGRANRIO

11. Ao longo do sculo XIX, a difuso da Revoluo Industrial alterou as condies de vida nas diversas reas atingidas pelo processo de industrializao, o que fez surgirem novas concepes e doutrinas comprometidas com o desenvolvimento ou com a reforma da sociedade capitalista. Dentre as propostas dessas doutrinas sociais, identificamos corretamente a: a) crtica da propriedade privada, formulada pelo marxismo cientfico. b) submisso integral do trabalhador ao capital, expressa pela doutrina social da Igreja, na Rerum novarum. c) defesa da livre associao dos trabalhadores em corporaes e sindicatos profissionais, proposta pelo liberalismo doutrinrio. d) subordinao do cidado a um Estado totalitrio, pregada pelo movimento anarquista. e) opo pela democracia partidria, defendida pelos socialistas utpicos, sendo o sufrgio universal censitrio o nico meio de o proletariado alcanar o poder. 12. Na segunda metade do sculo XIX, a introduo, de forma crescente, de trabalhadores livres na economia brasileira est ligada : a) crise da escravido, principalmente aps o fim do trfico negreiro. b) restrio de diversos pases europeus imigrao de seus excedentes nacionais. c) forma pacfica como foi encaminhada a Abolio, permitindo a utilizao do antigo escravo como trabalhador livre. d) acelerada criao de indstrias de base que no utilizavam trabalho escravo. e) poltica contrria escravido, adotada pelo governo imperial ao longo de toda a sua histria. 13. Assinale a opo que apresenta uma afirmativa correta sobre o processo de unificao da Alemanha (1871) e da Itlia (1870). a) Na Itlia, a proclamao da Repblica por Giuseppe Garibaldi, lder do movimento carbonrio e republicano, estabilizou economicamente o pas, permitindo a fixao das fronteiras internacionais italianas e sua unificao interna. b) Na Itlia, com o apoio do Papa Pio IX, o movimento unificador difundiu-se a partir da cidade de Roma, sendo contrrio aos interesses econmicos da burguesia do Piemonte e do norte do pas. c) Na Alemanha, Bismarck implementou a unificao com a ajuda econmica e militar do Imprio Austraco, opondo-se poltica separatista da Prssia de Guilherme I. d) A criao da Unio Alfandegria (Zollverein) entre os estados alemes desenvolveu a industrializao e a economia da Confederao Germnica, culminando na unificao poltica com a criao do Segundo Reich (imprio) Alemo. e) Ambos os processos unificadores resultaram da derrota dos movimentos nacionalistas locais frente reao das foras monrquicas reunidas pelo Congresso de Viena.

302

HISTRIA

CESGRANRIO

14. No perodo da chamada crise do Imprio, a partir de 1870, vrios fatores contriburam para provocar a queda da monarquia, em 1889, dentre os quais se destaca o(a): a) envolvimento continuado do Imprio em conflitos externos, principalmente na regio platina. b) conflito entre o Imprio e a Igreja, que era simptica s novas idias filosficas como o positivismo. c) incompatibilidade de amplos setores do Exrcito com a monarquia. d) expanso da lavoura cafeeira e da indstria, ampliando o uso da mo-de-obra escrava. e) posio contrria ao federalismo adotada pelos republicanos, o que lhes garantiu o apoio das oligarquias agrrias. 15. A Revoluo de 1930 pode ser relacionada a vrias transformaes da sociedade brasileira, entre as quais NO podemos incluir: a) o abandono dos setores agrcolas pelo governo, que privilegiou a industrializao. b) a insatisfao dos setores mdios urbanos com o domnio do processo poltico pelas oligarquias agrrias. c) a crescente organizao e mobilizao da classe operria, surgida com o processo de industrializao. d) a mobilizao de setores militares, principalmente dos oficiais mais jovens, contra o regime. e) as dissidncias oligrquias, materializadas na formao da Aliana Liberal. 16. A poltica do New Deal (1933-39), implementada nos Estados Unidos pelo presidente Franklin Roosevelt, significou um(a): a) b) c) d) e) combate ao liberalismo atravs da conteno dos nveis de consumo interno. estmulo poltica de criao de empregos com grandes investimentos em infra-estrutura. proibio da emisso monetria, o que impediu o crescimento da inflao. retrao da produo industrial, o que provocou o recrudescimento da economia. reduo acentuada dos gastos governamentais, o que estabilizou as finanas pblicas.

17. O envolvimento do Brasil na Segunda Guerra Mundial, ao lado dos pases aliados, guarda relao com questes internas como a(o): a) importncia crescente dos mercados alemes e japoneses para os produtos brasileiros. b) mobilizao dos grupos de inspirao fascista, como os integralistas, que apoiavam o Estado Novo. c) posio dos partidos majoritrios no Congresso Nacional, favorvel aos aliados. d) interesse do Brasil em se colocar como lder hegemnico dos pases americanos. e) apoio dos Estados Unidos ao projeto de industrializao, simbolizado na construo da usina de Volta Redonda.

303

HISTRIA

CESGRANRIO

18. Ao final da Segunda Guerra Mundial, a ruptura do acordo que unira os aliados vitoriosos gerou um ordenamento poltico internacional baseado na bipolaridade. Nesse contexto, crises polticas e tenses sociais desencadearam um processo de construo do socialismo em diversos pases. Assinale a opo que apresenta uma afirmativa correta sobre a construo do socialismo no mundo do ps-guerra. a) Na Iugoslvia (1944-45), o regime comunista implantado pelo Marechal Tito submeteu-se hegemonia poltica e econmica sovitica, o que acarretou sua expulso do movimento dos pases no alinhados. b) Na Tchecoslovquia (1946), o socialismo reformista, baseado na descentralizao e liberalizao do sistema frente ao modelo stalinista, retomado na poltica de Brejnev, foi interrompido pela represso russa, encerrando a Primavera de Praga. c) Na China (1949), a revoluo comunista derrubou o regime imperial e expulsou os invasores japoneses da Manchria, reunindo os nacionalistas, os senhores da guerra e os comunistas maostas em um governo de coalizo que instituiu uma repblica popular no pas. d) Na Coria (1950-53), a interveno militar norte-americana impediu o avano das foras revolucionrias comunistas que ocupavam o norte do pas, reunificando as duas Corias sob a tutela do Conselho de Segurana da ONU. e) Em Cuba (1959), a vitria dos revolucionrios castristas foi favorecida pela promulgao da Emenda Platt no Senado americano, que regularizou o envio de armamentos aos guerrilheiros contrrios ditadura de Fulgncio Batista. 19. O fim da Guerra Fria, expresso na extino da Unio Sovitica, em 1991, acarretou um novo equilbrio e o ordenamento das relaes internacionais, que se caracteriza por um(a): a) enfraquecimento dos movimentos nacionalistas regionais e das tendncias de globalizao na Europa ocidental. b) declnio da liderana poltica internacional das superpotncias em virtude da transferncia do controle de seus arsenais nucleares para a Assemblia Geral da ONU. c) revitalizao das alianas militares estratgico-defensivas, conforme os pactos polticos da Europa central e do leste. d) formao de megablocos poltico-econmicos que favoreceram a internacionalizao dos fluxos de capitais, tais como a da Comunidade Europia e a do Nafta. e) decadncia econmica dos pases da bacia do Pacfico que haviam mantido uma posio de neutralidade durante a Guerra Fria, tais como Cingapura e Malsia.

304

HISTRIA

CESGRANRIO

20. Recentemente o Presidente Fernando Henrique Cardoso anunciou um plano de metas a serem cumpridas pelo seu governo, mantendo-se a prtica de os governos brasileiros elaborarem programas de planificao econmica. A esse respeito, assinale a nica afirmativa correta. a) O II PND (Plano de Desenvolvimento) do Governo Geisel foi o instrumento de acelerao do crescimento econmico no chamado milagre brasileiro. b) O Plano Lafer (Plano Nacional de Reaparelhamento Econmico) do Segundo Governo Vargas visava incrementar a indstria de base e a ele se ligam as criaes do BNDES (Banco Nacional de Desenvolvimento Econmico) e da Petrobrs. c) O Plano Trienal do Governo Joo Goulart efetivou a implantao das reformas de base, as quais tiveram como ponto de partida a reforma agrria. d) O PAEG (Plano de Ao Econmica do Governo), adotado no primeiro governo militar aps 1964, favoreceu a indstria nacional, restringindo a entrada de capitais estrangeiros no pas. e) O Programa de Metas do Presidente Juscelino Kubitschek privilegiou o apoio produo agrcola e integrao do pas atravs de amplo programa ferrovirio.

305

HISTRIA

FUVEST
F A S E

1. Do ponto de vista cultural, na passagem da Antigidade para a Idade Mdia, correto afirmar que o patrimnio greco-romano: a) b) c) d) e) s no sofreu perda maior devido ao esclarecida de muitos chefes brbaros. perdeu-se quase completamente porque, dado o seu carter pago, foi rejeitado pela Igreja. foi rejeitado pelos brbaros em razo do carter cristo com que foi revestido pela Igreja. no desapareceu com a Antigidade porque a Igreja serviu de conduto para sua sobrevivncia. escapou do desaparecimento graas preservao fortuita de textos antigos.

1 B

2. Ajudaram os espartanos a vencer os atenienses na Guerra do Peloponeso, mas no foram eles que acabaram por conquistar toda a Grcia. Pelo contrrio, posteriormente, eles foram tambm conquistados e integrados a um novo imprio. Trata-se dos: a) egpcios e do Imprio Romano. b) fencios e do Imprio Cartagins. c) persas e do Imprio Helenstico. d) siracusanos e do Imprio Siciliota. e) macednios e do Imprio Babilnico.

P R O V A

3. No Brasil Colonial, a escravido caracterizou-se essencialmente: a) por sua vinculao exclusiva ao sistema agrrio exportador. b) pelo incentivo da Igreja e da Coroa escravido de ndios e negros. c) por estar amplamente distribuda entre a populao livre, constituindo a base econmica da sociedade. d) por destinar os trabalhos mais penosos aos negros e os mais leves aos ndios. e) por impedir a emigrao em massa de trabalhadores livres para o Brasil. 4. Sobre a Reforma religiosa, do sculo XVI, correto afirmar que: a) nas reas em que ela penetrou, obteve ampla adeso em todas as camadas da sociedade. b) foi um fenmeno to elitista quanto o Renascimento, permanecendo afastada das massas rurais e urbanas. c) nada teve a ver com o desenvolvimento das modernas economias capitalistas. d) fundamentou-se nas doutrinas da salvao pelas obras e na falibilidade da Igreja e da Bblia. e) acabou por ficar restrita Alemanha luterana, Holanda calvinista e Inglaterra anglicana. 5. A chamada Guerra dos Mascates, ocorrida em Pernambuco em 1710, deveu-se: a) ao surgimento de um sentimento nativista brasileiro, em oposio aos colonizadores portugueses. b) ao orgulho ferido dos habitantes da vila de Olinda, menosprezados pelos portugueses. c) ao choque entre comerciantes portugueses do Recife e a aristocracia rural de Olinda pelo controle da mo-de-obra escrava. d) ao choque entre comerciantes portugueses do Recife e a aristocracia rural de Olinda cujas relaes comerciais eram, respectivamente, de credores e devedores. e) a uma disputa interna entre grupos de comerciantes, que eram chamados depreciativamente de mascates.

306

HISTRIA

FUVEST
F A S E

6. Os movimentos fundamentalistas, que tudo querem subordinar lei islmica (Sharia), so hoje muito ativos em vrios pases da frica, do Oriente Mdio e da sia. Eles tiveram sua origem histrica: a) b) c) d) e) no desenvolvimento do Islamismo, durante a Antigidade, na Pennsula Arbica. na expanso da civilizao rabe, durante a Idade Mdia, tanto a Ocidente quanto a Oriente. na derrocada do socialismo, depois do fim da Unio Sovitica, no incio dos anos noventa. no estabelecimento do Imprio turco-otomano, com base em Istambul, durante a Idade Moderna. na ocupao do mundo rabe pelos europeus, entre a segunda metade do sculo XIX e primeira do XX.

1 B

P R O V A

7. Podemos afirmar que o perodo da minerao no Brasil que: a) atrados pelo ouro, vieram para o Brasil aventureiros de toda espcie, que inviabilizaram a minerao. b) a explorao das minas de ouro s trouxe benefcios para Portugal. c) a minerao deu origem a uma classe mdia urbana que teve papel decisivo na independncia do Brasil. d) o ouro beneficiou apenas a Inglaterra, que financiou sua explorao. e) a minerao contribuiu para interligar as vrias regies do Brasil, e foi fator de diferenciao da sociedade. 8. Qual dos pases abaixo, no passou por nenhuma das vrias revolues polticas que marcaram a Europa no sculo XIX ? a) Itlia b) Espanha c) Inglaterra d) Alemanha e) Frana

9. (Em) Massachussetts o esprito do capitalismo estava presente antes do desenvolvimento capitalista


... neste caso, a relao causal , certamente, a inversa daquela sugerida pelo ponto de vista materialista. (Max Weber, A tica e o Esprito do Capitalismo)

A afirmao: a) valoriza a viso do materialismo sobre o desenvolvimento do capitalismo na Nova Inglaterra. b) sustenta, ao contrrio do marxismo, que o esprito capitalista foi o criador do capitalismo moderno. c) coincide com a crtica marxista ao materialismo sobre a existncia do capitalismo na Nova Inglaterra. d) diverge do marxismo ao defender a existncia de uma fase de acumulao primitiva de capital. e) defende uma concepo consensual entre os historiadores sobre a origem do capitalismo.

307

HISTRIA
10. Sobre o chamado despotismo esclarecido correto afirmar que:

FUVEST
F A S E

a) foi um fenmeno comum a todas as monarquias europias, tendo por caracterstica a utilizao dos princpios do Iluminismo. b) foram os dspotas esclarecidos os responsveis pela sustentao e difuso das idias iluministas elaboradas pelos filsofos da poca. c) foi uma tentativa bem intencionada, embora fracassada, das monarquias europias reformarem estruturalmente seus Estados. d) foram os burgueses europeus que convenceram os reis a adotarem o programa de modernizao proposto pelos filsofos iluministas. e) foi uma tentativa, mais ou menos bem sucedida, de algumas monarquias reformarem, sem alter-las, as estruturas vigentes. 11. A poltica do caf, durante a Primeira Repblica, a) chegou ao auge do protecionismo com o Convnio de Taubat, passando depois a reger-se pelas leis do mercado. b) procurou atender aos interesses dos cafeicultores atravs de constantes medidas de proteo ao produto. c) pode ser equiparada de outras produes agrcolas, todas elas amparadas por Planos de Defesa. d) atendeu exclusivamente aos interesses dos grandes grupos internacionais, atravs dos Planos de Defesa. e) foi dirigida pelo governo do Estado de So Paulo, enquanto o poder federal mantinha uma atitude distante e neutra. 12. Sobre o processo de independncia poltica da Amrica Espanhola possvel afirmar que: a) diferentemente do Brasil, a longa guerra, que teve importante participao popular, fez emergir interesses sociais conflitantes. b) a Espanha, sob domnio francs, ficou de mos atadas, sem poder intervir no combate aos rebeldes. c) a participao macia de escravos ao lado dos rebeldes contrastou com a apatia das massas indgenas. d) a Igreja Catlica e os comerciantes abastados assumiram posies idnticas, a favor da Coroa espanhola. e) os acordos polticos, levados frente pelas elites, garantiram aos menos privilegiados as reformas sociais pelas quais tinham lutado. 13. A Sabinada, que agitou a Bahia entre novembro de 1837 e maro de 1838, a) tinha objetivos separatistas, no que diferia frontalmente das outras rebelies do perodo. b) foi uma rebelio contra o poder institudo no Rio de Janeiro que contou com a participao popular. c) assemelhou-se Guerra dos Farrapos, tanto pela postura anti-escravista quanto pela violncia e durao da luta. d) aproximou-se, em suas proposies polticas, das demais rebelies do perodo pela defesa do regime monrquico. e) pode ser vista como uma continuidade da Rebelio dos Alfaiates, pois os dois movimentos tinham os mesmos objetivos.

1 B

P R O V A

308

HISTRIA

FUVEST
F A S E

14. O Brasil recuperou-se de forma relativamente rpida dos efeitos da crise de 1929 porque: a) o governo de Getlio Vargas promoveu medidas de incentivo econmico, com emprstimos obtidos no exterior. b) o pas, no tendo uma economia capitalista desenvolvida, ficou menos sujeito aos efeitos da crise. c) houve reduo do consumo de bens e, com isso, foi possvel equilibrar as finanas pblicas. d) acordos internacionais, fixando um preo mnimo do caf, facilitaram a retomada da economia. e) um efeito combinado positivo resultou da diversificao das exportaes e do crescimento industrial. 15. A Revoluo Mexicana de 1910, do ponto de vista social, caracterizou-se: a) b) c) d) e) pela intensa participao camponesa. pela aliana entre operrios e camponeses. pela liderana de grupos socialistas. pelo apoio da Igreja aos sublevados. pela forte presena de combatentes estrangeiros.

1 B

P R O V A

16. Em 1947, o Partido Comunista foi colocado na ilegalidade no Brasil. Esta deciso se explica basicamente: a) b) c) d) e) pela bipartio do mundo em blocos antagnicos, conseqncia da guerra fria. pela linha insurrecional dos comunistas que pretendiam iniciar uma revoluo a curto prazo. por ser o Partido Comunista frgil e destitudo de expresso social. por um acordo firmado pela UDN, o PSD e o PTB. pelo desejo de acalmar as Foras Armadas que ameaavam interromper o jogo democrtico.

17. A ascenso de Hitler ao poder, no incio dos anos trinta, ocorreu: a) pelas mos do exrcito alemo que quis desforrar-se das humilhaes impostas pelo Tratado de Versalhes. b) atravs de uma ao golpista cuja ponta de lana foram as foras paramilitares do partido nazista. c) em conseqncia de uma aliana entre os nazistas e os comunistas. d) a partir de sua convocao pelo presidente Hindenburg, para chegar uma coalizo governamental. e) atravs de uma mobilizao semelhante que ocorreu na Itlia, com a marcha de Mussolini sobre Roma.

309

HISTRIA
18. A vitria do Brasil na Copa do Mundo de 70:

FUVEST
F A S E

a) no teve qualquer repercusso no campo poltico, por se tratar de um acontecimento estritamente esportivo. b) alentou o trabalho das oposies que deram destaque capacidade do povo brasileiro de realizar grandes proezas. c) propiciou uma operao de propaganda do governo Mdici, tentando associar a conquista ao regime autoritrio. d) favoreceu o projeto de abertura do general Geisel, ao criar uma linha de otimismo pelas realizaes do governo. e) alcanou repercusso muito limitada, pois os meios de comunicao no tinham a eficcia que tm hoje. 19. Qual das seguintes explicaes explica, sinteticamente, o fim da Unio Sovitica ? a) o regime entrou em colapso porque os dirigentes estavam desmoralizados, desde as denncias de Kruschev no XX Congresso do Partido. b) o regime deixou de ser sustentado pelo exrcito, adversrio tradicional do partido comunista. c) a vitria militar dos Estados Unidos na guerra fria tornou invivel a manuteno do regime. d) o colapso do regime deveu-se crise generalizada da economia estatal, combinada com o fracasso da abertura controlada de Gorbachev. e) os lderes soviticos abandonaram a crena no socialismo e decidiram transformar a Unio Sovitica em um pas capitalista. 20. A poltica cultural do Estado Novo com relao aos intelectuais caracterizou-se: a) pela represso indiscriminada, por serem os intelectuais considerados adversrios de regimes ditatoriais. b) por um clima de ampla liberdade pois o governo cortejava os intelectuais para obter apoio ao seu projeto nacional. c) pela indiferena, pois os intelectuais no tinham expresso e o governo se baseava nas foras militares. d) pelo desinteresse com relao aos intelectuais, pois o governo se apoiava nos trabalhadores sindicalizados. e) por uma poltica seletiva atravs da qual s os adversrios frontais do regime foram reprimidos.

1 B

P R O V A

310

HISTRIA
1. Quem foram os cartagineses e qual sua importncia na trajetria histrica romana ?

FUVEST
F A S E

2. Pela palavras das Escrituras somos instrudos de que h duas espadas: a espiritual e a temporal...
preciso que uma espada esteja sob o domnio da outra por conseguinte que o poder temporal se submeta ao espiritual (Bonifcio VIII, Bula Unam Sanctum, 1302). Quando... o Papa... se atribui a plenitude de poder sobre qualquer governante, comunidade ou pessoa individual, uma tal pretenso imprpria e errada, e se afasta das divinas Escrituras e das demonstraes humanas, ou melhor, at as contradiz (Marsilio Ficino, O Defensor da Paz, 1324).

Explicite e comente o conflito histrico presente nestes dois textos do incio do sc. XIV.

3. Nos dois sculos iniciais da era moderna (XV e XVI), a Itlia e a Espanha ocupavam posio de liderana na Europa, e a Holanda e a Inglaterra tinham um papel secundrio; nos dois sculos seguintes, essas posies se inverteram. Indique as razes dessa inverso.

4. Em outubro de 1949, Mao Ts Tung, derrotando os nacionalistas, proclamou a Repblica Popular da China. Mostre a importncia desse fato no interior do chamado campo socialista.

5. No Brasil e no Caribe, a escravido africana constituiu-se na principal modalidade de trabalho. Na Amrica de colonizao espanhola Mxico, Peru predominou o trabalho indgena compulsrio. Explique as origens dessas diferenas.

6. Indique as principais razes da insurreio pernambucana contra os holandeses, ocorrida entre 1645 e 1654.

7. A Constituio Imperial de 1824 estabelece que o governo monrquico hereditrio, constitucional e representativo (artigo 3) e que a pessoa do Imperador inviolvel e sagrada, no estando sujeita a responsabilidade alguma (artigo 99). Comente estes textos constitucionais, definidores da monarquia brasileira.

311

HISTRIA

FUVEST
F A S E

8. ... o que de corao desejo ver concluda esta maldita guerra, que j tanto tem arruinado nosso pas.
Ofcio confidencial de Caxias dirigido ao Ministro da Guerra brasileiro, em Tuiuti, 10 de Junho de 1867.

a) A que guerra Caxias se refere ? Que pases estavam envolvidos ? b) Quais repercusses dessa guerra para o Brasil ? 9. Nos dias de hoje, a imprensa tem se referido ao desmantelamento da Era Vargas. Comente o significado dessa expresso no que diz respeito legislao trabalhista.

10. Em A Era dos Extremos, o historiador Eric Hobsbawm afirma que a mudana social mais impressionante e de mais longo alcance da segunda metade deste sculo, e que nos isola para sempre do mundo do passado, a morte do campesinato. Comente esta afirmao.

312

HISTRIA
QUESTES DE 1 A 7

UFBA +F A S E

INSTRUO: Assinale as proposies verdadeiras, some os nmeros a elas associados e marque o resultado na Folha de Respostas.

Questo 1
Chamo, pois, de Governo ou administrao suprema o exerccio legtimo do poder executivo, e de prncipe ou magistrado o homem ou o corpo encarregado dessa administrao. (ROUSSEAU, p. 75)

Com base no texto e nos conhecimentos sobre as diferentes formas de governo e administrao existentes ao longo da histria, pode-se afirmar: (1) Os califados representavam uma forma de organizao poltico-administrativa dos muulmanos ps-Maom, fundamentada na identidade religiosa e garantida pela presena de poderosa fora militar. (2) As capitulares, normas administrativas estabelecidas por Carlos Magno para o Imprio Carolngio, contriburam para a unidade poltica dos estados monrquicos europeus na Alta Idade Mdia. (4) O modelo poltico-administrativo que serviu como base de sustentao do Estado Absolutista fundamentava-se na teoria da separao dos poderes, elaborada pelos idelogos Jean Bodin e Jacques Bossuet. (8) As cmaras municipais, no contexto da administrao do Brasil colonial, assumiram papel preponderantemente legislativo, encarregadas de estabelecer as bases polticas da Colnia. (16) A administrao estabelecida na Frana por Napoleo Bonaparte, a partir do Consulado, fortaleceu a descentralizao poltica originria do Antigo Regime. (32) Do ponto de vista da relao entre governo central e governos locais, no Brasil da Primeira Repblica, o federalismo representou, simultaneamente, a defesa da unidade nacional e da autonomia regional. (64) O modelo poltico-administrativo instalado no Brasil, aps o movimento militar de 1964, caracterizou-se por ser essencialmente tecnoburocrtico, associado ao capitalismo nacional e internacional.

313

HISTRIA
Questo 2

UFBA
F A S E

O mundo moderno formado por cerca de 200 Estados, cada um com territrio definido, povo e governo soberano, fazendo com que a humanidade seja representada politicamente por diversos governos individuais. Desde 1945, registra-se um aumento marcante no nmero de atribuies de organizaes internacionais, a tal ponto que, s vezes, elas so vistas como substitutas do Estado e realmente modificam o carter das relaes internacionais. (ENCICLOPDIA, p. 286)

Com base na anlise do texto e nos conhecimentos sobre relaes internacionais ao longo da histria, pode-se afirmar: (1) Na Grcia antiga, a Liga de Delos representou uma das primeiras experincias de relaes internacionais, ao aglutinar igualmente, no seu interior, cidades partidrias de Atenas e de Esparta. (2) No incio da Baixa Idade Mdia, as cruzadas permitiram a ampliao das relaes internacionais, visto que possibilitaram a convergncia de interesses religiosos, comerciais e polticos europeus e desses, com setores comerciais e polticos do Oriente Prximo. (4) A bula Inter Ctera do Papa Alexandre VI, em 1493, ao assegurar o monoplio ibrico sobre as terras descobertas no alm-mar, exemplifica uma tentativa da Igreja de funcionar como elemento conciliador das tenses internacionais, no incio dos tempos modernos. (8) O Tratado de Methuen (1703) representou o poder da dominao inglesa sobre o comrcio luso-colonial, consolidada em 1810, com a assinatura de tratados que colocariam definitivamente Portugal e Brasil sob a dependncia econmica britnica. (16) Durante a Primeira Repblica, o Brasil enfrentou questes territoriais nas atuais regies do Amap e do Acre, solucionadas em acordos diplomticos sob a mediao da Organizao dos Estados Americanos (OEA). (32) Os anos 70 do sculo XX registraram o estabelecimento dos acordos internacionais conhecidos como SALT I e SALT II, atravs dos quais os pases signatrios no interfeririam na produo de armas nucleares estratgicas nem em experincias atmicas subterrneas.

314

HISTRIA
Questo 3

UFBA
F A S E

Deus quis que, entre os homens, uns fossem senhores e outros servos, de tal maneira que os senhores estejam obrigados a venerar e amar a Deus, e que os servos estejam obrigados a amar e venerar o seu senhor... (Angers, apud VICENTINO, p. 55)

A anlise do texto e os conhecimentos sobre as igualdades e desigualdades sociais ao longo da histria permitem afirmar: (1) As relaes sociais indicadas no texto fundamentavam-se na concepo humanstica do mundo, defendida pela Igreja Catlica. (2) As leis consuetudinrias vigentes entre os povos germnicos possibilitavam a existncia de sociedade complexas e organizadas, nas quais o poder era exercido em nome das divindades. (4) A concepo de igualdade entre os homens e sua relao com a garantia de liberdade tornou-se o eixo do pensamento iluminista, o que revelava, por diferentes motivos, as aspiraes da burguesia e das classes populares europias. (8) A campanha coordenada pela Confederao Abolicionista, no Brasil dos anos 80 do sculo XIX, preconizava a extino das desigualdades civis e jurdicas entre livres, libertos e escravos. (16) O estabelecimento de leis especficas em defesa dos direitos de crianas, adolescentes, mulheres e negros tem demonstrado a eficcia dos princpios preconizados na Declarao Universal dos Direitos Humanos, publicada pela ONU aps a Segunda Guerra Mundial. (32) O princpio de autodeterminao dos povos, reconhecido internacionalmente, garantiu s colnias africanas o direito independncia, alcanado aps a Segunda Guerra Mundial e efetivado fora do controle dos interesses neocolonialistas das naes capitalistas. (64) De acordo com a legislao brasileira, os povos indgenas recebem tratamento jurdico diferenciado, por no serem considerados responsveis perante a lei, impedidos, portanto, do livre exerccio da cidadania e da participao poltica.

315

HISTRIA
Questo 4

UFBA
F A S E

Com base nos conhecimentos sobre o papel poltico do poder blico e sobre o desenvolvimento de tcnicas de guerra na histria dos diferentes povos, possvel afirmar:

(1) O exrcito romano, constitudo como fora regular no perodo da Repblica, foi fator preponderante para a conquista e preservao das fronteiras do Imprio, bem como fator de desagregao do prprio Estado j no sculo III d.C., em funo da interferncia das legies na sucesso imperial. (2) A Guerra dos Cem Anos exigiu o estabelecimento de um exrcito profissional, nacional e permanente na Frana, bem como contribuiu para o nascimento do sentimento nacional, de vital importncia para a instalao do absolutismo francs. (4) A Primeira Grande Guerra constituiu-se o primeiro conflito armado de carter mundial, tendo sua curta durao relacionada ao limitado avano tecnolgico da indstria blica dos pases envolvidos e s dificuldades de recrutamento de exrcitos regulares. (8) O prejuzo para as populaes civis norte-americanas e a desorganizao da economia nacional, frente aos ataques da aviao alem, levaram os Estados Unidos a participarem da Segunda Guerra Mundial ao lado dos pases democrticos aliados. (16) Na Guerra do Golfo, travada por pases do Ocidente contra o Iraque, apesar de os aliados disporem de exrcito numeroso e de sofisticados armamentos e tcnicas de ataque, evitouse o uso de armas nucleares, verificando-se a sobrevivncia de antigas estratgias de guerra, entre elas ataques s reas urbanas, destruio e bloqueio dos centros de abastecimento de alimentos e matrias-primas do inimigo. (32) Os conflitos localizados que ocorrem nos dias atuais, na Europa Central, no Norte da Rssia, no Oriente Mdio, entre outros, resultam de questes particulares e especficas, no resolvidas ao longo do processo histrico, desvinculadas dos interesses polticos, econmicos e estratgicos das grandes potncias.

316

HISTRIA
Questo 5

UFBA
F A S E

Sobre o uso da educao como instrumento de dominao poltica e ideolgica, pode-se afirmar:

(1) Durante a Idade Mdia, os senhores feudais detinham o controle das instituies educacionais, sobretudo das universidades, enquanto a burguesia nascente buscava controlar os centros produtores de conhecimento cultural e cientfico. (2) At o fim do perodo colonial, no Brasil, o ensino esteve sob o monoplio dos jesutas, que direcionavam os estudos no sentido de atender s exigncias da evangelizao e s necessidades de formao de uma elite intelectual politicamente consciente. (4) A instalao de escolas tcnicas e de nvel superior, no Brasil, aps a transferncia da Corte portuguesa, resultou da necessidade de suprir a deficincia de quadros burocrticos, tcnicos e especializados, necessrios ao funcionamento do poder luso que se estabelecera na Colnia. (8) Na resistncia dominao poltico-ideolgica de governos autoritrios, no Brasil, destaca-se a ao dos estudantes atravs da Unio Nacional dos Estudantes (UNE) que, criticando os fundamentos fascistas e antidemocrticos desses governos, alertavam a populao contra o perigo da perda dos seus direitos polticos e civis. (16) A Revoluo Cultural, desenvolvida na China comunista dos anos 60 deste sculo, visava elevao intelectual das massas e divulgao de prticas polticas voltadas para a livre expresso da vontade popular. (32) A reforma administrativa proposta pelo atual governo federal e o projeto da Lei de Diretrizes e Bases da Educao, aliados a poltica de investimentos do Banco Mundial, podero resultar no fortalecimento do ensino superior pago e na liquidao das universidades pblicas brasileiras. (64) O Movimento Brasileiro de Alfabetizao (Mobral), estabelecido no Plano de Metas do governo de Juscelino Kubistchek, foi absorvido pelos governos militares como programa destinado especificamente alfabetizao de populaes rurais.

317

HISTRIA
Questo 6

UFBA
F A S E

Texto I: Quando a mquina a vapor entrou em cena, na virada do sculo XVIII para o XIX, ela tambm provocou um choque de acelerao produtiva. (NASCIMENTO NETO, p. 82)

Texto II: No sculo XVIII foi a mquina a vapor. Neste fim do sculo XX, o motor da nova revoluo a tecnologia, o aperfeioamento dos transportes e das comunicaes. (Ibid.)

Com base na anlise dos textos e nos conhecimentos sobre a evoluo da indstria no processo histrico, possvel afirmar:

(1) A acelerao produtiva referida no texto I relaciona-se com a alterao das relaes de produo, a instalao da produo em srie e a ampliao do consumo. (2) O choque ocasionado pela mquina a vapor, mencionado no texto I, redimensionou o mercado de trabalho, provocando o desemprego em certos setores e o desespero de trabalhadores, fazendo-os atacar fbricas e destruir mquinas. (4) Como resultado do processo referido no texto I, as relaes comerciais entre pases fornecedores e consumidores conservaram as mesmas prticas existentes no comrcio internacional, poca do mercantilismo. (8) A situao descrita no texto II relaciona-se com a influncia da informtica e da racionalizao da produo nas modificaes do mercado de trabalho, mais acentuadas em pases desenvolvidos e, em menor escala, em pases emergentes. (16) O texto II permite compreender as modificaes que ocorrem nas relaes econmicas internacionais, nas quais as fronteiras tornam-se frgeis, permitindo o crescimento de uma economia caracterizada como transnacional. (32) Nos pases em desenvolvimento, verificam-se profundas modificaes nas relaes sociais, em decorrncia das prticas da globalizao econmica em seus territrios.

318

HISTRIA
Questo 7

UFBA
F A S E

Com base na anlise dos mapas e nos conhecimentos sobre conflitos regionais do mundo atual, possvel afirmar: (1) O mapa I indica a regio onde os conflitos opem catlicos republicanos a protestantes monarquistas. (2) O mapa II registra a tentativa norte-americana de invaso a Cuba, nos anos 90 com o objetivo de derrubar o governo Fidel Castro. (4) Os conflitos registrados no mapa III originam-se da luta pela terra, de contradies etnoculturais e da ao de minorias em defesa de sentimentos nacionalistas. (8) O conflito registrado no mapa IV tem suas razes na expanso do comunismo na sia, aps a Segunda Guerra Mundial, e na adeso da China Nacionalista ao bloco capitalista. (16) O acirramento dos conflitos registrados no mapa I resulta da influncia da crise do Leste Europeu e da desagregao da Unio Sovitica. (32) Atualmente, os conflitos registrados no mapa III tm se ampliado em razo do desinteresse norte-americano pelo estabelecimento de acordos de paz. (64) A complexidade das relaes internacionais, a profundidade dos problemas regionais, a necessidade de respeito autonomia das naes e o jogo de interesses de grupos econmicos transnacionais tm dificultado a soluo dos conflitos localizados, no mundo atual.

319

HISTRIA
QUESTES DE 1 A 10

UFBA
F A S E

INSTRUO: Assinale as proposies verdadeiras, some os nmeros a elas associados e marque o resultado na Folha de Respostas.

2 D

Questo 1 Com referncia ao desenvolvimento dos conhecimentos cientficos na histria da humanidade, possvel afirmar:

P R O V A

(1) Na Grcia antiga, no havia clara distino entre cincia e filosofia, dedicando-se os pensadores, ao mesmo tempo, s indagaes filosficas e s observaes cientficas. (2) Na Roma antiga, os estudos de medicina foram apoiados pelos imperadores, os quais incentivaram a construo de hospitais pblicos e a criao de uma escola de medicina. (4) A capacidade de assimilao e sistematizao de conhecimentos demonstrada pelos rabes levou-os a absorver a produo intelectual de outros povos, atrofiando seus prprios estudos e experincias cientficas. (8) Na Idade Mdia, a ruralizao da economia e a concentrao da vida intelectual nos mosteiros resultou na subordinao da cincia teologia, privilegiando o princpio da autoridade em detrimento da experimentao. (16) O crescimento demogrfico registrado na Europa, na segunda metade do sculo XIX, resultou, entre outros fatores, dos avanos no campo da medicina, responsveis pela identificao dos agentes transmissores de doenas como a tuberculose, a clera e a lepra. (32) A divulgao de conhecimentos relativos ao controle de doenas transmissveis, no Brasil do incio do sculo XX, resultou na campanha da vacinao antivarilica obrigatria, que contou com o amplo apoio dos diversos segmentos da sociedade.

320

HISTRIA
Questo 2

UFBA
F A S E

No a primeira vez que se decreta o fim do capitalismo. Quando as bolsas de valores quebraram nos Estados Unidos, no crack de 1929, as economias da Europa se afundaram em crises por vrios anos. A palavra globalizao foi utilizada amplamente na poca. Nos anos 70, em plena crise do petrleo, mais uma vez se vislumbrou o apocalipse. Nas crises da bolsa americana de 1987 e na recesso de 1990, enxergaram-se sinais de trincamento do sistema capitalista mesmo sem ter mais o comunismo para se contrapor a ele. (ALCNTARA, p. 101)

2 D

A anlise do texto e os conhecimentos sobre a evoluo do capitalismo permitem afirmar:

P R O V A

(1) A primeira crise do capitalismo, registrada na dcada de 70 do sculo XIX, teve sua superao possibilitada pela expanso da dominao imperialista europia sobre regies afro-asiticas. (2) ...as economias da Europa se afundaram... em conseqncia da insignificante produo industrial nos setores qumicos, mecnicos e eltricos, existentes na Inglaterra, na Frana e na Alemanha. (4) A crise capitalista dos anos 70 deste sculo relacionou-se com a derrota norte-americana na Guerra do Vietn e com a conseqente elevao dos preos de armamentos, alimentos e fretes no mercado internacional. (8) A crise da bolsa norte-americana, em 1987, ameaou o equilbrio do sistema capitalista mundial, visto que a liderana dos Estados Unidos permanecia sem concorrentes, nas relaes poltico-econmicas internacionais. (16) A crise que provocou a recesso dos anos 90 deste sculo influiu diretamente na economia brasileira, levando o Estado a adotar sucessivos planos econmicos, destinados a equilibrar o sistema financeiro nacional. (32) Os blocos econmicos transnacionais, que caracterizam as relaes internacionais neste fim do sculo XX, constituem-se alternativas viveis para a soluo de problemas e crises macrorregionais, dificultando a permanncia de projetos isolados.

321

HISTRIA
Questo 3

UFBA
F A S E

(...) a Igreja-hierarquia tem que entrar no campo poltico-ideolgico, mas no se deixar de forma alguma envolver pela problemtica partidria. A Igreja, para falar a todos, tem de estar acima dos partidos. Ela no deve ser uma agenciadora de poltica, mas, sim, assumir uma misso crtica, um papel de denncia violncia poltica. dever dos pastores denunciar os que contrariam os projetos divinos atinentes comunidade humana. Sua tarefa, sem questionar a prpria identidade religiosa, condenar a tortura fsica e psicolgica, os seqestros, a perseguio de dissidentes polticos ou de suspeitos e a excluso da vida pblica por causa das idias... a violncia terrorista e guerrilha... (ANDRADE FILHO, p. 234-5)

2 D

P R O V A

A anlise do texto e os conhecimentos sobre o papel da Igreja Catlica na histria permitem afirmar:

(1) O movimento da Contra-Reforma, que caracterizou a ao da Igreja nos sculos XVI e XVII, opunha-se aos objetivos estabelecidos no texto, por ter adotado mecanismos de represso material, ideolgica, espiritual e poltica contra setores ligados ao protestantismo, ao judasmo e ao islamismo. (2) A Constituio Civil do Clero, aprovada no incio da Revoluo Francesa, constituiu-se como modelo de separao entre a Igreja e o Estado, permitindo o desenvolvimento da misso crtica referida no texto. (4) O movimento ecumnico tem dificultado o dilogo entre a Igreja Catlica e outros grupos religiosos, a exemplo de protestantes, judeus e ortodoxos, pois preconiza a manuteno do seu monoplio religioso sobre a sociedade. (8) O texto defende a existncia de uma Igreja dissociada da questo poltica, concordando, nesse caso, com a prtica do padroado, que, no Brasil-Imprio, separou a religio do poder do Estado. (16) Durante a vigncia dos governos militares no Brasil, a omisso da Igreja-hierarquia frente s questes sociais e polticas contribuiu para a eficcia da represso e do autoritarismo do Estado sobre a sociedade civil. (32) As idias contidas no texto vo ao encontro de princpios bsicos da Teologia da Libertao, movimento de renovao da Igreja Catlica, voltado para a opo pelos pobres, para a anlise crtica da sociedade e para a defesa dos oprimidos.

322

HISTRIA
Questo 4

UFBA
F A S E

Sobre a formao do papel dos partidos e grupos polticos, em diferentes momentos da histria, possvel afirmar:

2 D

(1) Os tories e os wighs, faces polticas que se constituram na Inglaterra do sculo XVII e desempenharam papel relevante na formao e na crise do absolutismo ingls, deram origem aos atuais partidos Conservador e Liberal, respectivamente. (2) Na Revoluo Francesa, os girondinos, representantes da burguesia, assumiram posies moderadas, temerosos da projeo poltica das classes populares, enquanto os jacobinos, representantes da pequena burguesia, operrios e desvalidos, assumiram posies radicais, exigindo mudanas a favor dos oprimidos. (4) As expresses direita e esquerda, hoje utilizadas para caracterizar respectivamente posies reacionrias e revolucionrias, originam-se de conflitos entre faces polticas rivais, que disputavam o poder durante a Revoluo Russa de 1917. (8) No Imprio brasileiro, os liberais, originrios dos exaltados e restauradores, defendiam os interesses dos trabalhadores urbanos, camponeses e estrangeiros, enquanto os conservadores, representantes dos grandes proprietrios, defendiam a manuteno da grande propriedade e do trabalho assalariado no campo. (16) A proclamao da Repblica, no Brasil de 1889, resultou da ao efetiva e da propaganda do Partido Republicano, organizado no Rio de Janeiro e em So Paulo e difundido em todas as provncias do Imprio. (32) Durante a Repblica Velha, os partidos polticos apresentavam um carter nacional e democrtico, expressando as reivindicaes dos diferentes segmentos da sociedade. (64) Em regimes polticos totalitrios, a presena do unipartidarismo garante a permanncia no poder do grupo dominante, a imposio de sua ideologia, o controle da vida pblica e a identificao e aniquilao das oposies.

P R O V A

323

HISTRIA
Questo 5

UFBA
F A S E

Este foi um pas de escravos. O maior pas de escravos dos tempos modernos, talvez. Ou, pelo menos, o pas moderno mais dependente de escravos. Ou, pelo menos, o maior e mais dependente de escravos do continente americano. (TOLEDO, p. 52)

2 D

A anlise do texto e os conhecimentos sobre a escravido, no Brasil, permitem afirmar:

P R O V A

(1) Ao afirmar que Este foi um pas de escravos, o texto sugere que a escravido, j amplamente conhecida e utilizada em pases europeus, na Idade Moderna, foi estendida naturalmente ao Brasil, em razo da inexistncia de outras formas de relao de trabalho voltadas para a produo agrcola de exportao. (2) A dependncia do trabalho escravo tem suas origens na escravido do indgena, desenvolvida atravs de prticas diversas, entre elas a captura, o apresamento e a guerra justa. (4) A partir do sculo XVIII, registra-se a ampliao do trabalho escravo em atividades urbanas, sobretudo nas reas do artesanato, comrcio ambulante e servios, em razo do preconceito dos brancos contra o trabalho manual. (8) A solidariedade que aproximava escravos africanos e crioulos dificultava a utilizao do trabalho escravo por negros libertos e facilitava a unio da massa escrava em movimentos de rebeldia contra a dominao dos senhores. (16) A longa permanncia da escravido, no Brasil, decorreu, entre outros fatores, do carter lucrativo do trfico de escravos africanos, que beneficiava o Estado, os traficantes, os revendedores e os grandes proprietrios. (32) Aps a abolio da escravido, as atividades econmicas, no Brasil, fundamentaram-se no trabalho livre e assalariado, o qual ainda carecia de regulamentao relativa aos benefcios e direitos reivindicados pelos trabalhadores. (64) No Brasil atual, aps um sculo da extino do trabalho escravo, verifica-se o desaparecimento de prticas semi-escravistas e semi-servis, no campo e na cidade, tendo em vista a rigorosa vigilncia pelo cumprimento da legislao trabalhista ora em vigor.

324

HISTRIA
Questo 6

UFBA
F A S E

O que est provocando todo esse misto de ansiedade e ressentimento se chama globalizao. um processo de acelerao capitalista, num ritmo jamais visto, em que o produtor vai comprar matria-prima em qualquer lugar do mundo onde ela seja melhor e mais barata. Instala a fbrica nos pases onde a mo-de-obra fique mais em conta, no importa se no Vietn ou na Guatemala. Vende a mercadoria para o mundo inteiro. Em resumo: o entrelaamento econmico das parquias um processo que comeou na Pr-Histria, mas sempre progrediu em marcha lenta. Neste momento, est na velocidade da luz... (NASCIMENTO NETO, p. 81-2)

2 D

P R O V A

Anlise do texto e os conhecimentos sobre relaes internacionais no mundo atual permitem afirmar:

(1) O texto caracteriza a globalizao como um fenmeno internacional, prpria do capitalismo atual, que interliga, em maior ou menor grau, diferentes interesses econmicos mundiais. (2) O fenmeno da globalizao, nos dias atuais, beneficia-se da acelerao propiciada pela difuso dos recursos da informtica, permitindo a rapidez nos investimentos, nas transaes financeiras e comerciais e nas informaes culturais. (4) A crena, hoje generalizada, de que o fenmeno da globalizao ser responsvel pela futura derrubada das fronteiras nacionais entra em contradio com o fortalecimento de sentimentos nacionalistas e particularistas, verificados atualmente em conflitos polticos, a exemplo dos da Bsnia e da Chechnia. (8) O ritmo jamais visto nas comunicaes, que entrelaam as diversas regies do globo, responsvel pela diminuio acelerada das desigualdades entre pases ricos e pobres, j perceptvel na avaliao do nvel de vida, realizada por organismos internacionais. (16) O nvel de desemprego verificado no Brasil atual uma conseqncia direta dos efeitos da globalizao no mercado de trabalho de todas as regies do pas.

325

HISTRIA
Questo 7

UFBA
F A S E

I II III IV V VI

- Acredita! Obedece! Luta! - Quem tem ao tem po! - Mais canho, menos manteiga! - Nada jamais foi ganho na histria sem derramamento de sangue! - melhor um dia de leo do que cem anos de carneiro! - A guerra para o homem o que a maternidade para a mulher!

2 D

VII - Um minuto no campo de batalha vale por uma vida inteira de paz! VIII - A liberdade um cadver em putrefao! (VICENTINO, p. 270)

P R O V A

A anlise do texto e os conhecimentos sobre o militarismo em diferentes momentos da histria permitem afirmar:

(1) I exemplifica, na Antiguidade, os fundamentos da sociedade macednica, voltada para a militarizao de todos os homens, independentemente de sua classe social. (2) III, IV e VI aplicam-se poltica de expanso industrial dos Estados Unidos no sculo XIX, que resultou na conquista de mercados consumidores de seus manufaturados, na Amrica Latina. (4) II e IV podem ser relacionados poltica desenvolvida por Bismarck, no processo de unificao da Alemanh, que exigiu a conquista de territrios pertencentes ustria, Frana e Dinamarca. (8) V, VI e VII representam as idias militaristas dos regimes nazifascistas, nos quais o culto fora do Estado assume o valor mais elevado, minimizando a importncia dos interesses individuais. (16) VIII aplica-se prtica corporativista que, organizando a sociedade em sindicatos, constitua-se em peculiaridade do nazismo alemo. (32) III e VII constituram-se como ideologia fundamental da poltica externa dos governos militares que dominaram o Brasil de 1964 a 1985. (64) Todas as afirmativas contriburam para a criao do comportamento poltico responsvel, entre outros fatores, pelo clima de tenses que resultaram na ecloso da Segunda Guerra Mundial.

326

HISTRIA
Questo 8

UFBA
F A S E

A manuteno do carter primrio-exportador fortaleceu a subordinao da economia brasileira aos centros capitalistas hegemnicos, isto , aqueles onde o processo de industrializao estava em desenvolvimento, como a Europa Ocidental e os Estados Unidos. (ALBUQUERQUE, p. 268)

2 D

A anlise do texto e os conhecimentos sobre a relao da economia brasileira com o capitalismo internacional permitem afirmar:

P R O V A

(1) A subordinao referida no texto resulta tambm da especializao monocultora, da fragilidade da produo industrial e da dependncia s oscilaes do merdado externo que, entre a segunda metade do sculo XIX e incio do sculo XX, caracterizaram a economia brasileira. (2) Os capitais estrangeiros, no Brasil da Primeira Repblica, concentraram-se em investimentos na rea da indstria de base e da siderurgia, permitindo a quebra do carter dependente da economia nacional. (4) Os acordos financeiros firmados pelo Brasil com o Banco Rotschild, da Inglaterra, e com o Fundo Monetrio Internacional contriburam, tanto na Primeira Repblica quanto nos governos militares, para o alcance da autonomia brasileira no setor da produo industrial. (8) Em meados do sculo XX, a subordinao referida no texto evidenciou-se na participao do Brasil no programa da Aliana para o Progresso, estabelecida pelos Estados Unidos com o objetivo de garantir suas exportaes e impedir a expanso do comunismo no continente. (16) No Brasil dos anos 90 deste sculo, verifica-se a manuteno da poltica protecionista de reserva do mercado interno produo nacional, dificultando-se as importaes atravs da imposio de altas taxas alfandegrias.

327

HISTRIA
Questo 9

UFBA
F A S E

2 D

P R O V A

A anlise da gravura e os conhecimentos sobre a posse da terra no processo histrico permitem afirmar:

(1) A gravura expressa uma modalidade de propriedade da terra denominada beneficium, cujas origens localizam-se na Baixa Idade Mdia. (2) No perodo histrico em que preponderou o tipo de propriedade expresso na gravura, a posse da terra era alcanada atravs de doaes, herana, contratos de casamento e conquista militar. (4) O fortalecimento da pequena propriedade e a redistribuio dos feudos improdutivos aos pequenos proprietrios enfraqueceram a aristocracia agrria, retardando a instalao da Revoluo Industrial na Inglaterra. (8) O livre acesso do campons propriedade da terra constituiu-se um projeto presente na Revoluo Francesa; j na Revoluo Russa, esse acesso foi garantido atravs da organizao de propriedades agrcolas coletivas. (16) As concepes e as prticas de reforma agrria, revoluo agrria e cooperativas rurais, adotadas nas revolues socialistas da China e de Cuba, contriburam para a modificao, nesses pases, dos princpios responsveis pela concentrao da propriedade da terra. (32) A estrutura de propriedade expressa na gravura transferiu-se integralmente para o Brasil colonial, consolidando-se pela distribuio de terras atravs dos sistemas de capitanias hereditrias, sesmarias, parquias e aldeamentos. (64) A concentrao que caracteriza a propriedade da terra, no Brasil atual, tem sido responsvel pelos sucessivos conflitos com os trabalhadores sem terra e pelos prejuzos contra a preservao das reservas indgenas.

328

HISTRIA
Questo 10

UFBA
F A S E

No que se refere histria da Bahia, possvel afirmar:

2 D

(1) O territrio do atual Estado da Bahia originou-se da incorporao das terras das antigas capitanias da Bahia de Todos os Santos, So Jorge dos Ilhus, Porto Seguro, Itaparica e Paraguau. (2) O federalismo que se instalou na Bahia, com a Constituio Estadual de 2 de julho de 1891, representou a concretizao de idias federalistas j defendidas no Estado, desde os movimentos dos Mals e dos Alfaiates. (4) A represso que culminou com a destruio do arraial de Canudos em 1897 atendeu, entre outros, aos interesses dos latifundirios do Nordeste baiano, temerosos com a possibilidade de perda de suas propriedades. (8) O rio So Francisco, importante via de comunicao do territrio baiano desde a poca colonial, acha-se atualmente integrado ao projeto de preservao e renovao ambiental, responsvel pela recuperao da navegao ao longo do seu curso. (16) A diversificao da economia agroindustrial da regio cacaueira baiana tem sido responsvel pela reduo dos efeitos negativos causados pela atual crise do cacau na vida de proprietrios e trabalhadores. (32) O retardo no processo de industrializao da Bahia relaciona-se, entre outros fatores, com a longa permanncia da economia agroexportadora e com a predominncia do capital comercial na estrutura de produo do Estado. (64) As primeiras reformas urbanas responsveis pela modificao do aspecto tradicional da cidade do Salvador ocorreram durante a dcada de 60 deste sculo, com a destruio de nmero expressivo de exemplares da arquitetura colonial.

P R O V A

329

HISTRIA
1. A Reforma Religiosa do sculo XVI teve como desdobramento a) a consolidao do poder dos prncipes do Imprio Germnico. b) a constituio de mais de uma igreja crist no ocidente. c) a diviso da Igreja em ramos: Ortodoxo e Romano. d) a subordinao da Igreja Catlica ao Estado.

UFMG
F A S E

1 A

2. A cidade que os portugueses construram na Amrica no produto mental, no chega a contradizer


o quadro da natureza, e sua silhueta se enlaa na linha da paisagem. Nenhum rigor, nenhum mtodo, nenhuma providncia, sempre esse significativo abandono que exprime a palavra desleixo. (HOLANDA, Srgio Buarque de. O Semeador e o Ladrilhador. In: Razes do Brasil. Rio de Janeiro: Jos Olympio, 1956.)

P R O V A

A urbanizao no Brasil colonial at o sculo XVII vista como sendo provisria e acanhada. Um dos motivos pelos quais Portugal deixou em segundo plano a questo da urbanizao foi a) a inutilidade dos centros urbanos j que na colnia a administrao ficava a cargo dos Donatrios. b) as dificuldades para contratar tcnicos especializados que pudessem organizar as cidades. c) as lutas com os espanhis para a manuteno das terras coloniais que impediram o desenvolvimento da colnia do Brasil. d) o predomnio da vida rural, nos engenhos e nas fazendas de criao, o que diminuiu a importncia das cidades.

3. Todas as alternativas apresentam afirmaes corretas sobre a escravido no Brasil, EXCETO a) O contingente de escravos era diversificado e abrigava conflitos em seu interior. b) O escravo foi sempre uma mercadoria cara, s acessvel aos grandes senhores de terra. c) Os escravos de ganho realizavam servios variados a mando do senhor visando obteno de benefcios. d) Um escravo, como qualquer outra mercadoria, podia ser objeto de compra, venda, emprstimo, doao ou penhora.

4. Assinale a alternativa que caracteriza o sistema de trabalho conhecido como mita. a) Trabalho escravo de negros nas plantaes de acar do Caribe. b) Trabalho forado de ndios e mestios nas plantaes de caf da Colmbia. c) Trabalho forado de ndios nas minas de ouro e prata do Peru e Alto Peru. d) Trabalho escravo de ndios nas minas de salitre e cobre do Chile.

330

HISTRIA
5. Assinale a alternativa que apresenta a concepo de trabalho de Adam Smith.

UFMG
F A S E

a) A diviso do trabalho deve ser controlada pelo Estado, de forma a garantir a estabilidade na oferta de empregos. b) A maior produtividade pressupe a especializao do trabalho, a diviso entre vrios homens daquilo que anteriormente era produzido por um s. c) Os parasitas, aqueles que no trabalham, no podem participar e nem se beneficiar da riqueza produzida pela coletividade. d) Uma maior colaborao entre produtores diretos garante uma maior socializao das riquezas e o Estado do Bem-Estar Social. 6. Observe a gravura

1 A

P R O V A

Vista panormica de um Falanstrio tal qual foi proposto por Fourier. Os falanstrios, idealizados por Charles Fourier no sculo XIX, pretendiam a) b) c) d) facilitar o acesso do trabalhador s unidades de trabalho, ao criar as vilas operrias. organizar e racionalizar o espao fabril, alm de impor a diviso das tarefas. reforar o ideal de vida corporativa e socializada e unir as unidades produtivas rurais e urbanas. revolucionar o espao produtivo e criar mecanismos de higienizao e controle da produo.

7. ... A histria de todas as sociedades existentes at hoje a histria da luta de classes...


(MARX, K., ENGELS, F. Manifesto Comunista. 1848)

A partir dessa idia central do pensamento marxista, pode-se afirmar que, a) na sociedade capitalista, as classes fundamentais se diferenciam mais claramente e a conscincia de classe se desenvolve de maneira mais completa. b) na sociedade feudal, as classes ficam mais definidas devido extrema explorao dos senhores sobre os seus servos e sua impossibilidade de deter a posse das terras. c) nas sociedades antigas, as classes so definidas de acordo com os costumes comunais ficando a liderana dos grupos para aqueles que detiverem poderes sagrados. d) no mundo moderno, as classes alcanam um maior grau de diferenciao e conseguem definir seus papis sociais buscando uma maior integrao entre si.

331

HISTRIA
8. Todas as alternativas apresentam caractersticas do caudilhismo, EXCETO

UFMG
F A S E

a) Foi um fenmeno poltico prprio da Amrica Hispnica no sculo XIX, aps as independncias. b) Relacionava-se com as dificuldades de consolidao dos Estados Nacionais na Amrica Hispnica. c) Teve como principal objetivo a defesa de um Estado unificado e com forte poder central. d) Teve expressiva presena na regio do Prata, especialmente nos atuais Estados da Argentina e do Uruguai.

1 A

P R O V A

9. Em relao expanso imperialista na sia, na segunda metade do sculo XIX, pode-se afirmar que o Imprio Chins foi a) anexado ao Japo anulando a ameaa imperialista. b) desmembrado em colnias pelas potncias europias. c) dividido em zonas de influncia pelos pases ocidentais. d) incorporado ao Imprio Britnico compondo a Commonwealth.

10. Todas as alternativas abaixo apresentam afirmaes corretas sobre a economia cafeeira no Brasil, EXCETO a) Ao longo do sculo XVIII, a produo de caf servia apenas para consumo local e no tinha expresso comercial. b) No final do sculo XX, os grandes cafezais encontram-se nos estados do Paran, So Paulo, Mato Grosso e Minas Gerais. c) No incio do sculo XX, a diminuio dos cafezais gerou uma poltica de valorizao artificial do produto. d) Por volta da metade do sculo XIX, o caf passou a ocupar o primeiro lugar na pauta de exportaes brasileiras.

332

HISTRIA
11. Observe o mapa.

UFMG
F A S E

1 A

P R O V A

Assinale a alternativa que ilustra a conjuntura representada no mapa. a) A derrota da Trplice Entente, articulada com o o desembarque na Normandia. b) A expanso francesa, detida pelo Imprio Russo aps a Batalha de Stalingrado. c) A expanso alem e austraca, que gerou uma poltica de alianas e a guerra. d) A unio entre Estados Unidos e Frana, determinante para a vitria dos aliados.

12. Todas as alternativas apresentam caractersticas do cenrio sociopoltico da Primeira Repblica no Brasil, EXCETO a) Aglutinao de jovens oficiais que questionavam as regras e os rumos da poltica oligrquica. b) Constituio de partidos e coligaes polticas visando ao retorno normalidade democrtica. c) Movimento militar revolucionrio pelo interior do pas com o objetivo de reconstruir a nao. d) Movimentos populares de cunho religioso provocados pelas ms condies de vida e de trabalho no campo.

333

HISTRIA
13. Leia o texto abaixo.
O Estado Novo recolheu os triunfos da dcada de 30, ao expor com todo o esplendor esta extraordinria transformao do direito de organizao e ao coletivos em meio de identidade entre poder, lei e sociedade, de modo que a fora prometida por uma sociedade crescentemente mobilizada em seu desejo de mudana foi substituda pela onipotncia e eficcia das instituies governamentais e da represso, ambas sustentadas pela mais brilhante inveno do perodo: o espetculo da identidade entre Estado e povo, entre o Chefe do Estado e seu povo.

UFMG
F A S E

1 A

P R O V A

(PAOLI, M. C. A Era do Estado. Folha de So Paulo. 31 dez. 1988. Folhetim. p. G-6)

Todas as alternativas abaixo apresentam estratgias adotadas no Estado Novo que confirmam as idias contidas nesse texto, EXCETO a) A difuso da imagem de Getlio Vargas como Pai dos Pobres e Protetor dos Trabalhadores. b) A garantia de associao profissional e sindical condicionada ao seu reconhecimento pelo governo. c) A proliferao de partidos empenhados na construo de uma nova ordem poltica na sociedade brasileira. d) O controle e a censura dos meios de comunicao pelo Departamento de Imprensa e Propaganda.

14. Examine este grfico. Assinale a alternativa que apresenta a interpretao correta do grfico. a) A Alemanha, sob a Repblica de Weimar, assistiu ao incio de uma queda significativa no seu PIB. b) O Brasil, durante o 1 governo Vargas, experimentou um crescimento sem precedentes de seu PIB. c) O Reino Unido, sob o governo Tatcher, viu despencar as taxas de seu PIB. d) Os Estados Unidos, nos anos do New Deal, so o nico pas que mantm o crescimento do PIB.

334

HISTRIA

UFMG
F A S E

15. As diferentes formas de imperialismo e de colonizao se sobrepem e se interpenetram. O mesmo


ocorre com os chamados fenmenos de descolonizao e de independncia dos povos que se libertaram. Em sua maioria, eles se emanciparam entre 1945 e 1965 (FERRO, Marc Histria das colonizaes: das conquistas s independncias sculos XIII a XX. So Paulo, Companhia das Letras, 1996. p. 38.)

1 A

Todas as alternativas apresentam exemplos do processo de descolonizao ocorridos no perodo a que se refere o autor, EXCETO a) Emancipao do Panam. b) Guerra da Arglia. c) Independncia da ndia. d) Libertao da Indochina.

P R O V A

16. Todas as alternativas apresentam acontecimentos que abalaram os anos 60 do sculo XX, EXCETO a) A Guerra do Vietn e a afirmao do regime socialista em Cuba. b) A vitria do governo socialista de Salvador Allende no Chile e o golpe militar do General Pinochet. c) As manifestaes estudantis na Frana, Alemanha, Estados Unidos e o movimento hippie. d) O assassinato do lder negro Martin Luther King e o acirramento dos conflitos raciais nos Estados Unidos.

17. Brasil vivia guerra aberta no final de 1969. Essa afirmativa, que d ttulo a uma matria do jornal Folha de S. Paulo de 12 de maio de 1996, refere-se a) disposio do governo Costa e Silva de lutar contra a inflao e a alta constante das taxas de juros. b) disposio do governo de combater, a todo custo e por todos os meios, a rede do narcotrfico. c) ao enfrentamento armado, naquele perodo, entre foras de esquerda e o governo militar no Brasil. d) s aes tticas empreendidas pelos sem terra, que desejavam instaurar uma nova ordem no pas.

335

HISTRIA
18. Observe a charge. Todas as alternativas apresentam interpretaes corretas dessa charge, EXCETO a) A charge contm uma crtica explcita ao comprometimento do governo FHC com os interesses das elites brasileiras. b) A referncia ao banqueiro predileto de FHC denuncia o tratamento especial que o governo vem dando ao setor financeiro. c) O ttulo da charge sugere que o presidente tem sido extremamente atencioso com alguns polticos, partidos e rgos de classe. d) Os presentes oferecidos ao presidente traduzem o sentimento de satisfao do conjunto da sociedade com o seu governo. 19. Leia o seguinte trecho de Haiti, msica de Caetano Veloso e Gilberto Gil.
...Quando voc for convidado pra subir no adro Da Fundao Casa de Jorge Amado Pr ver do alto a fila de soldados, quase todos pretos Dando porrada na nuca de malandros pretos De ladres mulatos e outros quase brancos tratados como pretos S pr mostrar aos outros quase pretos (e so quase todos pretos) E aos quase brancos pobres como pretos Como que pretos, pobres e mulatos E quase brancos quase pretos de to pobres so tratados E no importa se olhos do mundo inteiro Possam estar um momento voltados para o largo Onde escravos eram castigados...

UFMG
F A S E

1 A

P R O V A

A partir desse trecho, pode-se afirmar que a) a represso policial dos quase pretos trata violentamente apenas os malandros quase brancos. b) em nosso pas os mulatos so tratados com mais violncia, quando identificados com os quase brancos. c) os olhos do mundo, em relao ao Brasil, preocupam-se apenas com a situao poltica dos negros e dos quase mulatos. d) os quase brancos, os quase pretos e os pretos, quando pobres, so tratados socialmente da mesma maneira no Brasil.

336

HISTRIA
20. Todas as alternativas apresentam afirmaes corretas sobre o Mercosul EXCETO

UFMG
F A S E

a) As empresas instaladas nos pases do Mercosul devero associar-se para a troca de conhecimentos tecnolgicos, a fim de aumentar a produtividade, melhorar a qualidade e diminuir o custo dos produtos. b) As empresas transnacionais tendem a concentrar a produo de determinados bens em um nico pas, entre os integrantes do Mercosul, ou seja, naquele onde os custos forem menores. c) As taxas alfandegrias sobre os produtos comercializados entre os pases integrantes esto sendo gradualmente eliminadas ou reduzidas, provocando o crescimento do comrcio intraMercosul. d) Os indicadores econmicos e sociais rea, populao, Produto Interno Bruto, renda per capita, analfabetismo etc. dos pases signatrios do acordo demonstram um equilbrio intraMercosul. 1. Todas as alternativas apresentam fatores que caracterizaram os Estados Nacionais formados a partir do sculo XV, EXCETO a) b) c) d) Criao de um exrcito permanente. Manuteno dos privilgios das corporaes. Organizao de um sistema nacional de impostos. Ordenao de uma administrao centralizada.

1 A 1 B

P R O V A

F A S E

2. Sobre a economia do perodo colonial, correto afirmar que a) a economia aurfera se caracterizou pela imobilidade social, bipolarizada entre o senhor e o escravo. b) a pecuria se baseou na criao intensiva, assentada no latifndio exportador e no trabalho escravo. c) a produo colonial foi orientada para a exportao de gneros para o mercado externo. d) a produo aucareira fixou a populao no litoral e criou uma expressiva camada mdia. 3. Leia o texto abaixo
... No castigar os excessos que eles [os escravos] cometem seria culpa no leve, porm estes [senhores] ho de averiguar antes, para no castigar inocentes, e se ho de ouvir os delatados e, convencidos, castigar-se-o com aoites moderados ou com os meterem em uma corrente de ferro por algum tempo ou tronco. Castigar com mpeto, com nimo vingativo, por mo prpria e com instrumentos terrveis e chegar talvez aos pobres com fogo ou lacre ardente, ou marc-los na cara, no seria para se sofrer entre os brbaros, muito menos entre os cristos catlicos. (ANTONIL, Andr Joo. Cultura e Opulncia do Brasil. 1711)

P R O V A

Esse texto, escrito por um padre jesuta em 1711, pode ser relacionado a) b) c) d) associao entre a escravido e a moral crist. condenao dos castigos aplicados aos escravos. oposio do clero catlico escravido. regulamentao das relaes entre senhores e escravos.

337

HISTRIA
4. Assinale a alternativa que apresenta um princpio filosfico do Sculo das Luzes. a) Crena na razo como fonte para a crtica social e poltica. b) Defesa do ideal monrquico para a garantia da unidade poltica. c) Idia do direito divino dos reis para legitimar o absolutismo. d) Idia de indivisibilidade do Estado em poderes independentes.

UFMG
F A S E

1 B

5. Os governos regenciais no Brasil (1831-1840) se caracterizaram por a) buscar a afirmao do poder poltico central para satisfazer os exaltados. b) fortalecer o poder poltico do Imperador, ao promover o Golpe da Maioridade. c) promover a descentralizao, o que gerou diversas revoltas regionais. d) satisfazer o desejo dos moderados, que buscavam a restaurao da monarquia.

P R O V A

6. Entende-se por anarquismo o movimento que a) afirma que o trabalho regulado, racionalmente, e a produo no visa ao lucro e ao enriquecimento dos indivduos, mas, s necessidades da comunidade que ter bens em abundncia. b) atribui ao indivduo e coletividade o direito liberdade de agir sem sofrer opresso por qualquer tipo de autoridade, admitindo, exclusivamente, os obstculos da natureza, da opinio, do senso comum e da vontade da comunidade geral. c) busca a garantia dos direitos individuais do homem, salientando o momento da participao democrtica na direo poltica de um pas e objetivando manter inalterados os equilbrios sociais existentes. d) prega o ideal de vida em comum na pobreza e na caridade e o desapego aos bens terrenos, propagando que a natureza colocou tudo para uso de todos, criando o direito comum em oposio ao direito privado.

7. Todas as alternativas apresentam aspectos da expanso da fronteira norte-americana, na segunda metade do sculo XIX, EXCETO a) Desenvolvimento da indstria txtil e metalrgica na costa do Pacfico. b) Dizimao dos indgenas e confinamento das comunidades remanescentes. c) Expanso das ferrovias ligando o Vale do Mississpi ao Oeste. d) Explorao de ouro, prata e outros minerais em vrias regies do Oeste.

338

HISTRIA
8. Observe a figura

UFMG
F A S E

1 B

P R O V A

Indique a alternativa que explica o que essa figura representa. a) A adoo pelos negros dos hbitos da classe dominante como forma de se integrarem sociedade. b) A busca da juventude pelos escravos para serem beneficiados pela Lei do Ventre Livre. c) A recusa dos proprietrios em perderem seus escravos devido Lei dos Sexagenrios. d) A tentativa dos negros em se manterem jovens para garantir a entrada no mercado de trabalho livre. 9. Leia o texto.
O sculo burgus foi uma era de melhoramentos, mais para os burgueses, talvez, do que para qualquer outro grupo de pessoas. Sua ideologia carregada de esperanas no era apenas uma mscara para encobrir o desespero, mas uma crena sincera no progresso. (GAY, Peter. A experincia burguesa da rainha Vitria a Freud: a educao dos sentidos. So Paulo: Companhia das Letras, 1988. p. 55.)

Todas as alternativas apresentam indicadores do progresso na Europa na segunda metade do sculo XIX, EXCETO a) A crena dos indivduos em relao s possibilidades de ascenso social. b) A expanso das ferrovias como meio de intensificar a troca de mercadorias. c) A garantia de uma renda mnima s famlias de trabalhadores. d) O aumento da capacidade produtiva atravs da ampliao do nmero de fbricas.

339

HISTRIA

UFMG
F A S E

10. Em 1891, Premph I, rei dos Ashanti, na Costa do Ouro (atual Gana, frica), respondeu da seguinte forma a uma consulta:
A proposta para o pas Ashanti, na presente situao, colocar-se sob proteo de Sua Majestade a Rainha e Imperatriz da ndia foi objeto de exame aprofundado, mas me permitam dizer que chegamos seguinte concluso: meu reino, o Ashanti, jamais aderir a tal poltica.

1 B

A partir do texto acima, pode-se afirmar que a conquista da frica pelos pases europeus a) baseou-se exclusivamente em operaes militares. b) contou com apoio das populaes locais. c) encontrou resistncia de chefes e reis africanos. d) enfrentou a concorrncia de imprios asiticos.

P R O V A

11. Todas as alternativas apresentam conflitos ligados expanso imperialista das potncias europias na frica e na sia, no sculo XIX, EXCETO a) A disputa entre ingleses e bores na frica do Sul. b) A disputa entre os interesses americanos e russos no Oriente. c) Os conflitos para abrir o mercado chins aos interesses ingleses. d) Os movimentos de resistncia africana contra a dominao estrangeira.

12. Durante a Revoluo de 1917, quase todas as nacionalidades da Rssia enxergaram na queda do
czarismo e, depois, na do governo provisrio a oportunidade para recuperarem sua liberdade. (FERRO, Marc. Histria das Colonizaes: das conquistas s independncias sculo XIII a XX. So Paulo: Companhia das Letras, 1996.)

Todas as alternativas apresentam afirmaes corretas sobre a questo das nacionalidades na URSS, EXCETO a) A tese da revoluo mundial promoveu uma reviso pelos bolcheviques do princpio da autodeterminao dos povos. b) Lenin, enquanto lder expressivo da Revoluo Russa, sempre se manifestou contra o princpio da autodeterminao dos povos. c) O direito autodeterminao dos povos, embora proclamado pelos revolucionrios de 17, nunca foi efetivamente praticado. d) O fracasso na resoluo do problema das nacionalidades pelos governos comunistas ficou evidente no momento da fragmentao da antiga URSS.

340

HISTRIA

UFMG
F A S E

13. Foi um perodo de intensa atividade sindical e poltica nos meios operrios, fundando-se inmeras
organizaes de classe e crculos polticos e desempenhando tambm uma indita atividade cultural. (RODRIGUES, Jos A. Sindicato e Desenvolvimento no Brasil. So Paulo: Difel, 1968. p. 12.)

1 B

A afirmao acima refere-se cena republicana nas duas primeiras dcadas deste sculo, no Brasil. Assinale a alternativa que apresenta caractersticas desse momento histrico. a) A luta pelas reformas de base que abrangia tanto as questes sindicais como a expectativa pela mudana na estrutura educacional do pas. b) A mobilizao de amplas camadas da populao exigindo a expanso do mercado de trabalho e da oferta de produtos industrializados. c) A promulgao das leis trabalhistas e de um novo cdigo civil que regulava questes importantes da cidadania no pas. d) As mobilizaes dos martimos, ferrovirios e porturios com a ecloso de algumas greves gerais e a intensificao do debate sobre a questo operria. P R O V A

14. Sobre a geopoltica na conjuntura imediatamente ps Segunda Guerra, pode-se afirmar que a) as reas que no se envolveram, diretamente, no conflito conseguiram alcanar um amplo desenvolvimento econmico baseado em uma poltica de exportao. b) as diversas formas de dominao colonial e de explorao que caracterizavam, historicamente, as relaes entre o centro e a periferia foram mantidas. c) os pases aliados estabeleceram uma poltica de arrasamento dos pases vencidos inviabilizando o crescimento mundial durante dcadas. d) os pases vencidos se agruparam formando o bloco dos no-alinhados viabilizando, assim, sua recuperao uma vez que no foram levados em considerao pelos vencedores.

15. Todas as alternativas apresentam repercusses do perodo de distenso da Guerra Fria, EXCETO a) b) c) d) Criao, na Polnia, do Solidariedade, primeiro sindicato livre do mundo socialista. Primavera de Praga, que foi sufocada pela invaso das tropas do Pacto de Varsvia. Queda do muro de Berlim, acontecimento emblemtico da reunificao alem. Revoluo de Veludo Tcheca, que significou o fim do monoplio do partido comunista.

16. Todas as alternativas apresentam aspectos importantes assumidos pela Teologia da Libertao, na Amrica Latina, a partir dos anos 60 do sculo XX, EXCETO a) b) c) d) Defesa da luta armada para alcanar o socialismo. Opo preferencial pelos pobres, oprimidos e injustiados. Participao ativa nos movimentos populares urbanos e rurais. Tentativa de combinao terica de concepes catlicas e marxistas.

341

HISTRIA
17. Observe o quadro TAXAS REGIONAIS DE URBANIZAO
1940 Centro-Oeste Nordeste Norte Sudeste Sul 21,52 23,42 27,75 39,42 27,73 1960 35,02 34,24 37,80 57,36 37,58 1980 67,75 50,44 51,69 82,79 62,41

UFMG
F A S E

1 B

P R O V A

Todas as alternativas apresentam interpretaes corretas a partir desse quadro, EXCETO a) A maior taxa de urbanizao nos trs momentos pertence regio de maior desenvolvimento econmico. b) A variao da taxa de urbanizao do Nordeste reflete o movimento migratrio para o eixo Rio-So Paulo. c) O crescimento da taxa de urbanizao do Centro-Oeste est relacionado expanso da fronteira agrcola. d) Os dois momentos mais recentes marcam uma acelerao no crescimento das taxas de urbanizao em todas as regies. 18. Leia o texto. RIO 40 GRAUS
(Fernanda Abreu, Fausto Fawcett e Laufer)

Rio 40 graus
Cidade maravilha Purgatrio da beleza e do caos

Capital do sangue quente do Brasil Capital do sangue quente Do melhor e do pior do Brasil Cidade sangue quente Maravilha mutante
O Rio uma cidade de cidades misturadas O Rio uma cidade de cidades camufladas Com governos misturados, camuflados, paralelos Sorrateiros ocultando comandos...

A letra dessa msica retrata a situao atual da cidade do Rio de Janeiro. Todas as alternativas apresentam indicadores corretos dessa situao, EXCETO a) b) c) d) Atuao do crime organizado junto s comunidades carentes, exercendo funes de governo. Coexistncia da beleza natural com as mazelas do processo de urbanizao. Deteriorao social e violncia caractersticas do Brasil de hoje. Incapacidade de convivncia entre a cidade oficial e a marginal.

342

HISTRIA
19. Sobre a situao econmica de Cuba, na conjuntura atual, pode-se afirmar que

UFMG
F A S E

a) a Rssia e os pases do Leste Europeu continuam a ser os principais parceiros comerciais de Cuba. b) Cuba atravessa uma fase de graves problemas econmicos, relacionados, principalmente, com a falta de produtos bsicos. c) Cuba mantm a sua economia fechada para os investimentos privados de pases capitalistas. d) os Estados Unidos suspenderam o bloqueio econmico sobre Cuba, em decorrncia do fim da Guerra Fria. 20. At o ano 2000 metade da populao do mundo morar e trabalhar nas cidades. O Habitat 2, conferncia convocada pela ONU em junho de 1996, discutiu a urbanizao do mundo. Todas as alternativas apresentam objetivos do Habitat 2, EXCETO a) Encontrar formas de satisfazer as necessidades de moradia e infra-estrutura das cidades. b) Melhorar a economia urbana, reduzir a pobreza e criar empregos. c) Melhorar a participao da populao na gesto dos assuntos pblicos. d) Possibilitar uma maior integrao entre pases pobres e ricos. QUESTO 01 IDENTIFIQUE as atividades produtivas e as relaes de trabalho predominantes em cada uma das seguintes regies, nos sculos XVI e XVII. a) Nordeste brasileiro: b) Antilhas: c) Mxico e Peru: QUESTO 02 Leia o texto.
O interesse dos mercadores dos Pases-Baixos pelo Brasil foi um fato que antecedeu de muito os ataques empreendidos pela Companhia das ndias Ocidentais, em 1624 contra a Bahia e, em 1630, contra Pernambuco. Estes ataques explicam-se por aquele interesse (). Faz-se, pois, necessrio recuar um pouco no tempo, para uma perspectiva melhor dos acontecimentos que na segunda e terceira dcadas de 1600 se desenrolam em nosso pas. (MELLO, J. A. Gonsalves de. O domnio holands na Bahia e no Nordeste. In: HOLANDA, S. B. de (dir.). Histria Geral da Civilizao Brasileira. So Paulo: Difel, 1981. t. I, v. 1, p. 235.)

1 B 2

P R O V A

d) Regio Platina: e) Colnias Inglesas do Norte: f) Colnias Inglesas do Sul:

F A S E

1. CITE a forma de participao dos mercadores dos Pases-Baixos no comrcio do acar anterior ao domnio holands no nordeste aucareiro. 2. RELACIONE os acontecimentos polticos da histria europia com a conquista do nordeste aucareiro pelos holandeses.

343

HISTRIA

UFMG
F A S E

3. CITE as medidas adotadas pelos holandeses que lhes permitiram contar com o apoio da populao local. 4. INDIQUE os motivos que geraram o descontentamento da populao local e que culminaram na expulso dos holandeses.

QUESTO 03 Leia o texto.


Creio que nos ltimos anos foi cercado mais da quarta parte de todo o condado de Derby As vantagens que emanam dos cercamentos so muito grandes, o gado progride porque cada proprietrio, emancipado dos costumes do lugar, desfruta de liberdade para melhorar o rebanho de acordo com seu prprio critrio e a experincia demonstra que os animais de todos os cercados so, em geral, superiores aos dos campos comuns. (BROWN, Thomas. Viso geral da agricultura no Condado de Derby. 1794.)

1. CITE os dois tipos de propriedade aos quais o texto se refere: 2. IDENTIFIQUE o processo scio-econmico, alm dos cercamentos, vivido pela Inglaterra no Sculo XVIII. 3. CITE as duas classes sociais que se consolidaram nesse processo e ANALISE o impacto social dessas mudanas para cada uma delas.

QUESTO 04 Leia o texto.


A forma peculiar como se processou a independncia da Amrica portuguesa teve conseqncias fundamentais no seu subseqente desenvolvimento. (FURTADO, Celso. Formao Econmica do Brasil. So Paulo: Nacional, 1971. p. 35.)

Com base na citao acima, 1. EXPLIQUE as razes por que o processo de independncia da Amrica portuguesa considerado peculiar. 2. CITE duas insurreies ocorridas no Nordeste brasileiro durante o processo de independncia: 3. A Inglaterra foi um dos pases mais beneficiados com a emancipao poltica da Amrica portuguesa. JUSTIFIQUE essa afirmativa:

344

HISTRIA

UFMG
F A S E

4. A peculiaridade do processo de independncia do Brasil imprimiu ao processo scio-poltico da jovem nao, ao longo do sculo XIX, caractersticas distintas da maioria das naes latino-americanas. CITE duas dessas caractersticas. QUESTO 05 Leia os textos.
Exigimos para o povo alemo a igualdade de direitos para com as outras naes, a ab-rogao dos Tratados de Versalhes e de Saint Germain. (Programa do Partido Nacional Socialista dos Trabalhadores Alemes, 1920.) Ns romperemos o tratado de paz de bandidagem que nos foi imposto em Versalhes, assim como o Plano Young que submeteu a Alemanha (). (Programa do Partido Comunista Alemo, 1930.) Na sua precipitao em suster a clera dos alemes como uma cauo da vitria, os aliados [em 1919] no se apercebiam de que perdiam a paz no preciso momento em que ganhavam a guerra. (FERRO, Marc. Histria da Primeira Guerra Mundial. Lisboa: Edies 70, 1992. p. 319.)

Sobre o Tratado de Versalhes, assinado entre os aliados e a Alemanha, aps o trmino da Primeira Guerra Mundial, 1. CITE duas disposies desse Tratado que explicam o descontentamento dos alemes. 2. RELACIONE o disposto no Tratado de Versalhes com a conjuntura alem nos anos 1920. 3. EXPLIQUE em que medida, ao trmino da Primeira Guerra Mundial, os aliados perdiam a paz no preciso momento em que ganhavam a guerra. QUESTO 06 Leia o telegrama.
Dada precipitao acontecimentos fundamos Ao Integralista Brasileira. Assemblia criao movimento formada estudantes, operrios, elementos classe mdia. Tudo dentro princpios j acertados nossa correspondncia anterior: sindicalizao, representao exclusivamente profissional. Base Estado na famlia, municpio, sindicato. Avise nossos amigos Bahia e Belo Horizonte restabelecer articulao, fomentar propaganda. Espero sua vinda aqui urgente. Viva Brasil dirigido mocidade. (Telegrama Plnio Salgado para Olbiano de Melo. Apud TRINDADE, H. Integralismo: a ascenso das idias autoritrias no Brasil. Histria do Sculo XX. So Paulo: Abril Cultural, 1968.)

1. CARACTERIZE a Ao Integralista Brasileira. 2. RELACIONE o surgimento dessa organizao com o contexto poltico internacional. 3. CITE 3 outros pases onde ocorreram movimentos com as mesmas caractersticas e DENOMINE-OS. 4. CITE duas caractersticas comuns a estes movimentos.

345

HISTRIA
QUESTO 07

UFMG
F A S E

No decorrer do ltimo ano, os sem terra ocuparam por diversas vezes as pginas da grande imprensa no Brasil. 1. CITE a principal reivindicao dos movimentos dos sem terra. 2. DESCREVA a situao da propriedade fundiria no Brasil atual. 3. RELACIONE essa situao com as condies histricas da ocupao do territrio brasileiro, a) no perodo colonial: b) no sculo XIX: 4. CITE dois movimentos sociais no perodo republicano, que tiveram como bandeira a distribuio da terra no Brasil.

346

HISTRIA

UFPA
F A S E

1. A Declarao de Independncia dos EUA, redigida por Thomas Jefferson e proclamada em 4 de julho de 1776, apresenta como caracterstica fundamental a a) defesa da liberdade e propriedade individual, bem como dos direitos fundamentais do Homem. b) crena nos ideais humanistas, despertados a partir da Revoluo Francesa no sculo XVII. c) busca de uma sociedade republicana e federalista, onde o Estado Nacional fosse soberano e forte. d) tentativa de separao poltica da metrpole, muito embora a dependncia econmica com a Inglaterra fosse mantida. e) abolio da escravido em todas as colnias da Amrica, de acordo com o que prescrevia a Declarao dos Direitos do Homem e do Cidado. 2. O socialismo cientfico, nascido na Europa a partir da segunda metade do sculo XIX, propunha a seguinte teoria: a) criao de uma sociedade comunista baseada no princpio newtoniano da ao e reao humana e defensora do livre arbtrio dos homens. b) fundao de uma organizao socialista onde no haveria nenhuma forma de autoridade imposta, nem mesmo a do Estado. c) triunfo da classe trabalhadora sobre a burguesia, com a transio do capitalismo para o socialismo. d) derrubada do governo russo, o fim das eleies burguesas e a instaurao da ditadura do proletariado. e) crena na natural e cientfica evoluo do homem, o qual, vindo do macaco, teria se desenvolvido e caminharia progressivamente rumo ao socialismo iminente. 3. Getlio Vargas, no Brasil, e Juan Domingo Pern, na Argentina, encarnam a melhor expresso do fenmeno fascista na Amrica Latina. Esse perodo caracterizado pela a) planificao da economia por parte do Estado, visando o desenvolvimento do setor agrrio e o fortalecimento do Poder Executivo apoiado nas decises do Legislativo. b) marginalizao dos setores dominantes, considerados um entrave ao programa de privatizao de empresas estatais e ao desenvolvimento da industrializao do pas. c) radicalizao poltica que culminar com a proibio de funcionamento de todos os sindicatos e o estabelecimento da censura imprensa escrita. d) conservao e fortalecimento do capitalismo atravs de investimentos no setor privado e pela participao de setores urbanos no processo de legitimao do Estado autoritrio. e) interveno do Estado em assuntos econmicos, supresso de liberdades individuais, corporativismo, dissoluo dos partidos polticos e pela mstica nacionalista.

347

HISTRIA

UFPA
F A S E

4. A Amaznia o lugar onde Gaia (a Terra) mostra a luxuriante riqueza de seu corpo. E tambm o
lugar onde ela mais sofre violncia (...). A se perpetraram todos os pecados capitais (pecados mortais e pecado do capital). (BOFF, Leonardo. Ecologia: Grito da Terra, Grito dos Pobres. So Paulo: tica, 1995, p. 135)

Do trecho anterior, depreende-se que a Amaznia a) o pulmo do mundo, onde h perfeito equilbrio entre oxignio e a cadeia alimentar das plantas, homem e natureza. b) o local perfeito para a convivncia pacfica entre ndios e brancos, visto que os mesmos so detentores de saberes ancestrais sobre os segredos da natureza. c) abriga milhares de camponeses vindos do Nordeste e de vrias regies do pas em busca de ouro e pedras preciosas utilizando somente a tcnica da cata e de esteiras manuais. d) sofre agresses brutais dos grandes projetos minerais financiados pelo capital internacional e pelos projetos agropecurios envolvendo grandes empresas nacionais. e) um palco de sacrifcios de garimpeiros e bias-frias que se rebelam contra a ao da FUNAI, dos missionrios americanos e contra os tratores e moto-serras. 5. O final do sculo XIX e incio do XX caracteriza-se pelas transformaes histricas que se refletem na paisagem urbana das principais cidades brasileiras. Na prtica, essas transformaes traduziam-se em a) estratgias de controle da populao de formas variadas, de modo que se impusesse uma ordenao do espao urbano, necessria ao clima de modernizao. b) disciplinarizao dos trabalhadores de feiras-livres e de condutores de bondes eltricos, de maneira que os ofcios fossem exercidos, exclusivamente, por nacionais. c) controle dos trabalhadores, da moradia e do jogo da capoeira, com a finalidade de tornar livres as cidades brasileiras, da presena de portugueses e negros libertos. d) construo de edifcios pblicos que obedecessem ao estilo barroco, dando s cidades brasileiras uma imagem modernizada, de acordo com os padres europeus. e) aumento da exportao da borracha, baseada nas relaes escravistas de produo e no desenvolvimento da massa do proletariado urbano. 6. Leia atentamente a afirmativa abaixo e identifique o que ocorreu no Brasil depois de 1928:
No perodo de 1928 existiam (...) pelo menos 3 propostas de revoluo vindas de agrupamentos polticos diferentes: o partido Democrtico, os tenentes e o Bloco Operrio e Campons. (DE DECCA, Edgar S. O Silncio dos Vencidos. So Paulo, Brasiliense: 1981, p. 81)

a) o Bloco Operrio e Campons subiu ao poder atravs de Getlio Vargas, promotor da Revoluo de 1930 e idealizador das leis trabalhistas brasileiras. b) o Partido Democrtico ganha as eleies de 1930 atravs do candidato Getlio Vargas, sendo que este revoluciona o sistema eleitoral com o fim da poltica dos governadores. c) o tenentismo ganhou fora aps 1930, com uma Revoluo de carter militar, cuja deflagrao inaugurou o chamado Estado Novo. d) em 1930, Getlio Vargas, candidato presidncia da Repblica derrotado por Jlio Prestes, toma o poder atravs de uma revoluo, colocando-se acima de todas as antigas tendncias polticas. e) os tenentes, liderados por Vargas, tomam o poder auxiliados pelo Partido Democrtico e pelo Bloco Operrio e Campons, derrotando as oligarquias rurais

348

HISTRIA

UFPA
F A S E

7. No final da dcada de 60, do sculo XX, o Brasil passa por um momento de intensa agitao poltica. Todos os segmentos da sociedade, operrios, estudantes e polticos, clamavam pela redemocratizao do pas. A respeito das manifestaes culturais, podemos afirmar que a(o) a) expresso da cultura brasileira passa por um momento de identificao com as razes afrobrasileiras, expressa nas composies de msicas carnavalescas. b) confronto entre o i-i-i e os shows montados pelo movimento da Jovem Guarda refletir no engajamento dos jovens intelectuais ao movimento musical norte-americano. c) arte, representada pelos festivais de msica popular, pelo cinema, teatro e literatura, passa a ser utilizada tambm, como um instrumento de conscientizao poltica. d) massificao de programas levou ao grande pblico msicas sertanejas e popularizou cantores de cabeleira encaracolada que utilizavam a guitarra eltrica e) choro, a seresta e o samba passam a ser cantados nas salas de visitas, nos bares e quintais, respectivamente, como forma de reconhecimento da cidadania. 8. Do governo Figueiredo at as eleies de 1989, instalou-se um processo constante de confrontao com o regime vigente, em busca de uma proposta real de democratizao do pas. Sobre as caractersticas das eleies presidenciais de 1989 no Brasil, podemos afirmar que a) disputada por Lula e Collor, destacou-se pela luta entre dois grandes partidos (PT e PSDB) que desde 1982 consolidaram-se na poltica brasileira. b) foi marcada por ser a pioneira, j que constituiu-se na primeira eleio direta para presidente desde 1960, quando subiu ao poder Jnio Quadros. c) foi caracterizada pela morte do presidente eleito (Tancredo Neves) e pela posse de Jos Sarney, o qual implantou um plano econmico denominado de Cruzado. d) caracterizada pela poltica do palanque eletrnico, marcou-se por campanhas eleitorais corruptas e milionrias onde os principais candidatos foram comprovadamente indiciados no esquema PC Farias. e) foi marcada pela implementao da consulta eleitoral em duas fases, sendo a primeira obrigatria apenas aos eleitores menores de 18 anos. 1. A respeito da hierarquia familiar, afirmava o filsofo Aristteles que na antiga cidade-estado de Atenas:
O Homem livre manda no escravo, o macho na fmea e o pai na criana (...).

Em relao ao perodo acima referido, responda: a) Qual o regime poltico predominante nas duas principais cidades-estados gregas (Atenas e Esparta)? b) Como se justifica a rgida hierarquia social familiar da sociedade ateniense? 2. O tempo medieval chamado pelo historiador Georges Duby de o tempo das heresias vencidas, ou antes, das heresias sufocadas. Considerando ento a heresia num perodo histrico bem delimitado, responda: a) O que era ser hertico no tempo medieval? b) Qual o instrumento de represso mais freqentemente utilizado pela Igreja contra os herticos?

F A S E

349

HISTRIA

UFPA
F A S E

3. A histria colonial brasileira est intrisicamente ligada poltica mercantilista dos Estados Modernos.
O mercantilismo considerado antes de tudo, um servio da poltica, uma administrao do tesouro real, um instrumento de grandeza poltica e militar. (DEYON, Pierre. O Mercantilismo. So Paulo: Perspectiva, 1985. p. 43)

Com base no texto: a) Explicite dois princpios bsicos do mercantilismo. b) De que maneira o mercantilismo se constituiu num fator de unificao nacional? c) Qual o papel desempenhado pelo Brasil dentro da poltica econmica mercantilista portuguesa? 4. Em 14 de julho de 1789, uma multido invadiu a Bastilha na capital da Frana, e libertou os presos que l se encontravam. Este ato no foi isolado. Ele fazia parte de uma srie de revoltas populares devido a carestia dos preos em geral, e especialmente, revoltas pelo aumento no preo do po. Enquanto isso, em Versalhes, o rei Luis XVI e sua corte comiam e bebiam fartamente como se nada estivesse acontecendo. Sobre estes acontecimentos podemos perguntar: a) Como foram denominados os acontecimentos que se sucederam na Frana aps o 14 de julho de 1789? b) Como estava dividida a sociedade francesa at 1789, ou seja, quantos eram os Estados e quem participava de cada um deles? c) Aps 1789 as desigualdades sociais e econmicas foram eliminadas na Frana? Justifique sua resposta. 5. Em 17 de setembro de 1759, o Marqus de Pombal expulsou os frades da Companhia de Jesus do Brasil e com isto desestabilizou toda uma organizao social que estes clrigos haviam implementado na colnia como um todo, e na Amaznia, em particular. a) Por que estes clrigos foram expulsos? Cite pelo menos duas causas. b) Por que a expulso dos clrigos desestabilizou a antiga organizao social da Amaznia? 6. Leia atentamente o trecho do poema transcrito abaixo que exalta a figura de um lder cabano e responda as questes propostas:
Nascido no Cear frente de mil cabanos, desta Belm, do Par, tu expulsaste os tiranos. (HURLEY, Jorge. Traos Cabanos. Belm: Oficinas Grficas do Instituto Lauro Sodr, 1936, p. 44)

a) Quem foi e qual a atuao do lder cabano referido no texto? b) Quem eram os tiranos que ele ajudou a expulsar? c) Quando e como foi o fim desse lder dos mil cabanos paraenses?

350

HISTRIA

UFPA
F A S E

7. 20 de novembro de 1995. O Brasil celebra os 300 anos da morte de Zumbi, lder do Quilombo dos Palmares, smbolo da luta contra a desumanizao do negro escravizado. O processo de luta pela abolio da escravatura foi lento e cruel, desenvolvendo-se de diferentes maneiras, tanto no Parlamento como nas aes cotidianas dos escravos e abolicionistas. Responda: a) De que modo a escravido foi extinta no Brasil? b) De que maneira atuavam os abolicionistas? c) Qual o significado poltico da figura de Zumbi para a luta dos negros no Brasil? 8. A poltica e a ideologia populistas so, at o presente, uma realidade atuante no formao social
brasileira. O que ocorre que o perodo de 1930-1964 o nico perodo da histria do Brasil no qual a poltica populista afirma-se como elemento especfico definidor da poltica de desenvolvimento do Estado brasileiro. (BOITO Jr., Armando. O Golpe de 1964: A burocracia contra o populismo. Coleo Tudo histria. So Paulo: Brasiliense, 1982. p. 21)

A partir da leitura do texto, respoda: a) O que foi o populismo no Brasil? b) Por que a Era Vargas considerada o perodo ureo do populismo no Brasil? c) Por que o governo de Joo Goulart caracteriza a crise do populismo brasileiro?

351

HISTRIA

UFPE
F A S E

1. A arqueologia tcnica cientfica usada pelos historiadores para resgatar o passado mais remoto da humanidade atravs de seus vestgios materiais; graas arqueologia que existe o conhecimento da Pr-histria. Assinale a alternativa que no corresponde Pr-histria: a) Estudos comprovam que os tipos Neanderthalenses e Arcantropinos habitavam parte da frica, sia e Europa h cerca de 200 mil anos atrs. b) As raas Grimalde e a Cro-Magnon so consideradas Homo Sapiens. c) No mesoltico as tcnicas de produo dos artefatos variavam entre as formas paleolticas anteriores e as formas neolticas posteriores. d) O Homem de Neanderthal desenvolveu sua cultura material no paleoltico mdio. e) Os historiadores tm compreendido mais claramente a vida cotidiana da cidade de Pompia, na Itlia, a partir das escavaes arqueolgicas. 2. A arquitetura, no sentido de arte, tcnica, canne, ..., foi regulamentada e conhecida primeiramente no Egito. Assinale a alternativa correta sobre esta parte no Egito: a) As construes de pirmides no Egito tm incio com as pirmides de Kops, Kfren e Miquerinos. b) A arquitetura egpcia antiga se caracteriza por seu sentido monumental e de imobilidade com vistas ao ritual religioso-funerrio. c) Os faras egpcios do Antigo Imprio determinaram as construes de mastabas e menhires no Vale dos Reis. d) A idia religiosa egpcia de vida aps a morte foi comum quela sociedade apenas depois das invases dos hicsos. e) A construo das pirmides e de outros edifcios no Egito antigo teve uma funo econmica a guarda dos gros para os anos de escassez. 3. A chegada dos brbaros Europa ocidental fez desaparecer a unidade do Imprio Romano. Entre os anos 500 e 1000 de nossa era os povos da Europa ocidental evoluram em sentidos diferentes e procuraram seus prprios meios de expresso. Com relao ao perodo medieval assinale a alternativa incorreta: a) Como conseqncia da dominao do Imprio Romano a lngua utilizada oficialmente em toda a Europa ocidental durante a Idade Mdia foi o latim. b) Com relao ao estilo romnico pode-se afirmar que se manifestou de forma diferente quando comparada sua apario entre os diversos grupos tnicos europeus. c) A igreja Crist durante a Idade Mdia deteve o poder poltico e aliou-se a uma elite de cavaleiros que, atravs de guerras de conquista, ampliou suas terras. d) Atravs dos rabes os europeus conheceram o pensamento dos filsofos gregos e dos bizantinos. Assim, toda a Europa medieval adotou sua cultura. Exemplo: a religio ortodoxa, arquitetura, calendrio e alfabeto. e) As cidades no possuam um papel econmico forte mas os condados, ducados, e principados dirigidos pela nobreza adquiriram crescente autonomia econmica e poltica.

352

HISTRIA

UFPE
F A S E

4. A crise do sistema feudal acelerou-se no sculo XIV. Esta crise geral manifestou-se de vrias maneiras. Assinale a alternativa incorreta: a) Devido forma de explorao utilizada durante toda a Idade Mdia houve esgotamento do solo e conseqentemente a produo agrcola diminuiu. b) A queda da produo agrcola teve como conseqncia imediata a subida dos preos. c) Com a falta de produtos os mercados tendiam a fechar nas cidades e a fome atingiu tambm a populao do campo. d) Neste perodo a peste negra assolava em toda a Europa causando a morte da populao. e) Com a diminuio da taxa de crescimento populacional os preos tenderam a baixar e os senhores feudais e nobres mantiveram seu padro econmico. 5. O nacionalismo pode ser considerado como a ideologia burguesa do sculo XIX, contudo a esse sentimento nacional se vincula a idia imperialista de expanso das potncias europias. Assinale a alternativa abaixo que confirma esta afirmao: a) A unificao italiana foi realizada sob a ideologia nacionalista e logo depois deu-se a expanso neocolonialista na direo da frica: a Itlia anexou a Lbia, a Etipia e parte da Somlia. b) O movimento nacionalista possibilitou o desenvolvimento da Frana que se aventurou na Guerra Franco-Prussiana anexando a Alscia e a Lorena. c) A Blgica movida pelas idias nacionalistas expande seu territrio na direo da Dinamarca anexando o Schleswig Holstein. d) Os Estados Unidos conquistando sua unidade na Guerra da Seco realizaram uma poltica expansionista em Angola e Moambique. e) A frica no sculo XIX sofreu invases muulmanas como decorrncia do processo nacionalista e expansionista do mundo rabe. 6. Aps a 1 Guerra Mundial o socialismo surgia como uma alternativa poltico-ideolgica ao capitalismo. Com relao ao surgimento do socialismo assinale a alternativa incorreta: a) No incio do sculo XX, a Rssia rural, atrelada ainda a algumas instituies feudais, conviveu com cidades em pleno desenvolvimento industrial como Petrogrado e Moscou. b) Os camponeses participaram ativamente da Revoluo Socialista mesmo a despeito da doutrina marxista ter se dirigido mais para a organizao da classe operria. c) A primeira experincia com o socialismo como alternativa ao sistema capitalista foi realizada durante a repblica de Weimar, na Alemanha. d) O socialismo utpico, que pregava a igualdade social e reformas econmicas, surgiu aps a revoluo de 1830 na Frana, mas no teve peso suficiente para se transformar como alternativa ao capitalistmo. e) O mais clebre terico do socialismo foi o judeu alemo Karl Marx.

353

HISTRIA

UFPE
F A S E

7. Aps a segunda Guerra Mundial a Europa exibia as marcas da guerra. Entre os doze pases mais sacrificados a Unio Sovitica era o primeiro com a perda de 20 milhes de seres humanos e a Gr Bretanha o dcimo segundo com 388 mil. Nas alternativas abaixo assinale a que indica os fatores responsveis por esse conflito: a) b) c) d) e) o revanchismo dos vencedores da Primeira Guerra a falta de autoridade da Sociedade das Naes a competio entre as grandes potncias o cordo sanitrio que foi imposto Unio Sovitica os planos do nazismo em construir uma nova ordem mundial a diviso e partilha da frica, sia e Amrica pelas grandes potncias os planos expansionistas de Hitler a expanso socialista na frica a disputa entre os italianos pelo territrio do Vaticano a aliana militar americano-japonesa no Pacfico o desenvolvimento do socialismo na Itlia e Alemanha a neutralidade da Inglaterra o sistema de alianas e dentro dele a aliana do Japo e Unio Sovitica a aliana de Mussolini e Franco a anexao do corredor polons Rssia o Tratado de Versailhes que no garantiu a paz entre as naes as novas tecnologias blicas associadas s polticas de extermnio desenvolvidas nos Estados Unidos e Alemanha a expanso turca na pennsula itlica

8. Leia o trecho da Carta de Padre Antnio Vieira ao Padre Provencial dos Jesutas em 1635 e depois assinale a alternativa que interpreta historicamente os interesses do Estado portugus, dos colonos, e dos padres jesutas na colonizao.
a ...Na primeira carta disse a V.Rev. a grande perseguio que padecem os ndios, pela cobia dos portugueses em os cativarem... O modo como esses ndios recebiam os portugueses era ordinariamente de paz... E perguntando eu a um dos cabos desta entrada como se haviam com eles me respondeu com grande desabafo e paz de alma: a esses davmos-lhe uma carga cerrada, caam uns, fugiam outros, entrvamos nas aldeias e tomvamos aquilo que havamos mister... Todos estes latrocnios e homicdios se toleram em um reino to catlico como Portugal... Estas so, Padre Provincial a as notcias que posso dar a V.Rev. desta conquista do Maranho de onde fao esta. Antnio Vieira.

a) O Estado portugus queria fazer do ndio um sdito, os colonos reduzi-los ao trabalho escravo e os jesutas convert-los f crist e utiliz-los como mo de obra. b) O Estado portugus desejava o extermnio da raa para que os colonos pudessem transferir populaes africanas para a Amrica e os jesutas estimulavam a escravido indgena. c) A coroa portuguesa aliada aos franceses iniciou o processo de aculturao e desmantelamento das naes indgenas para facilitar o trabalho dos colonos. d) Os jesutas, cumprindo ordens da metrpole, isolaram os ndios em misses, no litoral, facilitando o aprisionamento deles pelos colonos para quem o ndio bom era o ndio morto. e) Os colonos utilizaram os jesutas na escravido dos gentios e o Estado portugus, em compensao, distribuiu sesmarias somente s ordens religiosas que haviam participado da conquista.

354

HISTRIA

UFPE
F A S E

9. A partir de 1840 medidas antiliberais do governo monrquico brasileiro, como a retirada da autonomia do Ato Adicional e do Cdigo de Processo Criminal, provocaram uma srie de revoltas no pas. Sobre este tema faa a correlao entre as colunas: (1) (2) (3) (4) (5) ( ( ( ( ) ) ) ) a Revoluo Liberal de So Paulo a Revoluo Praieira em Pernambuco a Revista Progresso a Revoluo Liberal em Minas Gerais o Jornal Dirio Novo

Antnio Pedro foi o seu lder com ntida influncia dos socialistas franceses a voz do Partido da Praia movimento chefiado pelo Brigadeiro Tobias Aguiar um dos seus lderes, Antnio Borges da Fonseca, defendeu no parlamento reformas sociais e a nacionalizao do comrcio a retalho ( ) movimento rebelde chefiado por Tefilo Otoni e facilmente neutralizado por Lus Alves de Lima e Silva A seqncia correta : a) 3, 5, 1, 2, 4 b) 4, 2, 1, 5, 3 c) 3, 1, 2, 4, 5 d) 1, 3, 2, 4, 5 e) 4, 5, 3, 1, 2

10. O relatrio do parlamentar do Imprio Nabuco de Arajo informa que o trfico (de escravos) interprovincial esvaziou o sistema (escravocrata) nas provncias do Norte, portanto, a batalha final vai se dar no corao do Imprio Rio, Minas, So Paulo no reinado do caf. Entre as alternativas abaixo indique a que complementa o enunciado: a) A estagnao e a decadncia do vale do Paraba esgotaram o sistema escravista, tambm na regio sudeste. b) A abolio da escravido no Cear desestabilizou completamente o sistema, inclusive no maior centro cafeicultor que era So Paulo. c) Enquanto no sudeste o caf prosperava, no nordeste o acar declinava. A escravido no era mais uma questo nacional. O movimento abolicionista e o capital ingls pressionavam na direo do trabalho livre assalariado, no maior centro de poder do pas: a Corte. d) A transferncia de escravos de uma regio para outra reforou o sistema: substitua o trabalho livre pelo trabalho compulsrio. e) A Lei do Ventre Livre possibilitou a decadncia do sistema escravocrata criando as condies necessrias para a abolio da escravido.

355

HISTRIA

UFPE
F A S E

11. A Guerra de Canudos e o banditismo social, o Cangao, so fenmenos histricos atrelados s mesmas razes scio-econmicas. Assinale nas alternativas abaixo a que indica esta identidade: a) A origem desses descontentamentos situou-se na religiosidade popular traduzida nas profecias de msticos e fanticos como Antnio Conselheiro: o serto vai virar praia e a praia virar serto. b) Surgiram do movimento tenentista outras manifestaes de inconformismo. Soldados bem treinados sem perspectiva de trabalho se adentraram aos sertes e se rebelaram contra o governo. c) A valorizao da terra em fins do sculo XIX, as injustias sociais, o fenmeno da seca e o isolamento dessas populaes produziram os cangaceiros e os fanticos. d) O no cumprimento do Pacto dos Coronis pela oligarquia Acioli no Cear, levou o Padre Ccero Romo Batista a organizar uma reao em Canudos. e) As populaes sertanejas deserdadas da terra encontraram na marcha da Coluna Prestes um caminho para o fanatismo e para o cangaceirismo. 12. Durante a Repblica Velha o presidente Campos Sales governou de 1898 a 1902 e inaugurou uma prtica poltica conhecida como a poltica dos governadores. Indique nas alternativas abaixo a que no corresponde a esta prtica poltica: a) O presidente apoiaria a oligarquia dominante vencedora nos Estados, independente de partidos polticos. b) Uma das chaves da poltica dos governadores era a Comisso de Verificao, tirada do poder legislativo; tinha como funo legitimar as eleies estaduais, mesmo que os resultados fossem contestados. c) A oligarquia estadual deveria garantir a eleio de candidatos oficiais para o Congresso Brasileiro. d) A poltica dos governadores inclua a derrubada dos governadores que haviam apoiado Deodoro da Fonseca. e) A poltica dos governadores tambm se apoiava no coronelismo, fenmeno poltico que se desenvolveu nas oligarquias regionais. 13. Enquanto o presidente Washington Lus classificou a questo social como caso de polcia, a Constituio de 1934, influenciada pela Revoluo de 1930, legislou sobre o mundo do trabalho. Assinale a alternativa abaixo que no corresponde ao enunciado: a) No texto constitucional proibiam-se as diferenas salariais com base em distines de sexo, idade, nacionalidade e estado civil; b) O salrio mnimo foi regionalizado e a jornada de trabalho passou a ser limitada a 8 horas dirias; c) O texto do artigo 121 da referida Constituio estabeleceu descanso semanal e frias anuais remuneradas; d) O Fundo de Garantia do trabalhador deveria ser liberado, caso este fosse demitido com ou sem justa causa; e) A nova Legislao previa indenizao ao trabalhador demitido sem justa causa.

356

HISTRIA

UFPE
F A S E

14. Assinale a alternativa que corresponde ao fenmeno poltico denominado populismo. a) uma forma tpica dos governantes democrticos dos Estados Unidos conciliarem seus problemas tnicos; b) um modo de governar sem nenhuma sensibilidade para as presses populares ou polticopartidrias; c) A ambigidade uma das suas caractersticas: relaciona-se com a crise das oligarquias, da democracia e do desenvolvimento econmico; d) Expresso poltica para o deslocamento do eixo da economia industrial para a economia comercial internacional; e) Movimento que impediu a posse do presidente Joo Goulart, quando da renncia de Jnio Quadros. 15. Assinale a alternativa abaixo que responde por fatos que ocorreram durante a Nova Repblica: a) A criao do MDB (movimento democrtico brasileiro) e da ARENA (aliana renovadora nacional); b) A fundao do partido dos trabalhadores (PT), a eleio de Tancredo Neves para presidente da Repblica e o Plano Cruzado; c) O milagre econmico brasileiro traduzido pelas medidas adotadas no governo ps 64 e a deposio de Joo Goulart; d) A dissoluo de organizaes classificadas como subversivas , exemplo: o comando geral dos trabalhadores (CGT) e a unio nacional dos estudantes (UNE); e) A aprovao pelo Congresso Nacional em 1965 da emenda das inelegibilidades e a edio do AI n 2 que dissolveu os partidos polticos. 16. As artes e a literatura brasileiras na atualidade no esto articuladas a movimentos coletivos. Entretanto, at os anos sessenta algumas manifestaes culturais foram de ordem mais coletiva. Sobre este tema faa a relao entre as colunas: (1) (2) (3) (4) (5) ( ( ( ( ( ) ) ) ) ) Movimento armonial A tropiclia Movimento modernista Bossa nova Cinema Novo Caetano Veloso Ariano Suassuna Glauber Rocha Mrio de Andrade Antnio Jobin

Assinale a alternativa correta: a) 1, 2, 3, 4, 5 b) 5, 3, 2, 1, 4 c) 2, 1, 5, 3, 4 d) 5, 3, 1, 4, 2 e) 2, 5, 1, 4, 3

357

HISTRIA

UFPE
F A S E

PARA CADA UMA DAS QUESTES A SEGUIR, ASSINALE NA COLUNA I A(S) PROPOSIO(ES) VERDADEIRA(S) E NA COLUNA II, A(S) FALSA(S): 1. A Historiografia o modo como se escreve a Histria. As proposies abaixo correspondem s correntes historiogrficas que se desenvolveram ao longo da histria da humanidade? 0-0) Os gregos contaram o incio da sua histria usando a mitologia. 1-1) Durante a Idade Mdia os cronistas contaram a histria dos reis, das batalhas, da Igreja, considerando que todos os fatos histricos ou no histricos eram movidos pela vontade de Deus. 2-2) No sculo XIX, a Histria relacionou-se com outras cincias, como a Sociologia, a Geografia, a Antropologia e utilizou a anlise dos documentos, ampliando suas respostas com base no empirismo. 3-3) Tambm no sculo XIX a Histria ultrapassou a narrativa e introduziu a explicao dos fatos, fundamentando-se no sistema produtivo das sociedades, nas estruturas e na luta de classes. 4-4) No sculo XX, a nova histria se ope histria interpretativa e valoriza a cultura, o pensamento, as representaes, o cotidiano e a narrativa. 2. A arte uma criao humana, fundamentada na cultura e no processo histrico. Sobre as vrias modalidades de arte pode-se afirmar: 0-0) Durante a Pr-histria o homo-sapiens desenvolveu a tcnica da pintura sobre a rocha, conhecida como arte rupestre, e atravs dela nos deixou imagens de outros aspectos de sua cultura. 1-1) Os romanos desenvolveram sua arte com base na arte etrusca e tambm receberam influncia dos gregos, no que diz respeito arquitetura e aos motivos governamentais. 2-2) O movimento literrio, artstico e cientfico que nasceu na Itlia e se propagou pela Europa Ocidental dos sculos XV e XVI denomina-se Renascimento. 3-3) A arquitetura moderna teve nos irmos Perret e em Le Corbusier seus precursores, que a partir de 1922 utilizaram em lugar da construo metlica a construo em cimento armado. 4-4) Das experincias dos irmos franceses Lumire e do americano Thomas Edison surgiu uma nova arte o cinema que rapidamente se transformou em uma promissora indstria de arte e entretenimento. 3. Sobre a sociedade oligrquica e militar espartana podemos afirmar que: 0-0) Em Esparta a propriedade da terra era coletiva e o regime era feudal. 1-1) A constituio espartana estabelecia que o governo da cidade deveria ser regido por dois reis, um conselho e uma assemblia. 2-2) Somente os espartanos e os periecos eram considerados cidados em Esparta. 3-3) A sociedade espartana estava constituda por trs camadas: os espartanos, os hilotas e os periecos. 4-4) Entre as reformas econmicas realizadas em Esparta, destaca-se a que aboliu a propriedade familiar e em seu lugar instituiu a propriedade estatal a terra cvica dividida em lotes.

358

HISTRIA
4. Sobre a cidadania e os direitos da Repblica romana pode-se afirmar:

UFPE
F A S E

0-0) Ser cidado romano exigia coragem, lealdade, respeito aos deuses e culto glria. 1-1) A vida do cidado romano era regulada por duas leis: a lei pblica e a lei privada. Do direito pblico faziam parte o Direito Civil (jus civile) e o Direito Estrangeiro (jus gentium) em oposio ao Direito Privado, que regulamentava as relaes entre as famlias. 2-2) O Estado, todo poderoso, exercia um grande poder sobre a famlia, destruindo o ptriopoder, herana dos gregos. 3-3) Os cidados romanos dividiam-se em cinco classes, conforme sua riqueza; as classes eram subdivididas em centrias, as quais se constituam de ptrios e plebeus, separadamente. 4-4) A partir das lutas empreendidas pelos plebeus para o fortalecimento da cidadania, as leis votadas na Assemblia da Plebe passaram a ter validade em todo o estado. Era a deciso da plebe ou Plebiscito. 5. O sistema feudal sofreu diferentes transformaes nos reinos europeus. Se compararmos, por exemplo, a Rssia do sculo XII, com o que hoje chamamos Itlia, tambm no sculo XII, verificaremos grandes desigualdades. Que elementos condicionaram as diversas transformaes nesse sistema? 0-0) As cruzadas desorganizaram o sistema produtivo na chamada Europa Ocidental. 1-1) Em regies de grande densidade populacional e pequena disponibilidade de terra o feudalismo se esgotou com maior rapidez. 2-2) Os senhores feudais, originrios dos antigos proprietrios romanos, cuja produo havia regredido subsistncia, reagiram fortemente s mudanas adivindas com o comrcio. 3-3) A perda de parte da mo-de-obra, motivada pelas guerras, pestes, estimulou o esprito cientfico objetivando melhorar a produtividade. A ferradura, o arado, a trao peitoral so o resultado desse esforo. 4-4) Em regies de baixa densidade demogrfica, alta disponibilidade de terras e com proprietrios de origem germnica, houve uma grande abertura s inovaes do mundo e do capital comercial. 6. As reformas religiosas do sculo XVI no apenas romperam a unidade do cristianismo no ocidente mas modificaram as estruturas eclesisticas e a doutrina da salvao. Sobre essas reformas podemos afirmar: 0-0) Os luteranos entendiam a reforma como restaurao do verdadeiro cristianismo contra a laicizao de Roma. 1-1) A Reforma Anglicana em 1534, pelo Ato de Supremacia de Henrique VIII, criou uma Igreja Nacional na Inglaterra, modificando o culto religioso. 2-2) Calvino tornou o culto mais complexo, condenou a doutrina da predestinao e recusou sacramentos como o batismo e a comunho. 3-3) A Contra-Reforma catlica definiu medidas de combate aos protestantes: criou colgios destinados ao ensino dos jovens, difundiu a catequese entre os povos no cristos e conteve o protestantismo utilizando os tribunais da Inquisio. 4-4) Do Conclio de Trento saiu uma Igreja reformada e modernizada. A autoridade papal foi diminuda e a doutrina tradicional da salvao competente negada com a criao do Catecismo e do Missal.

359

HISTRIA

UFPE
F A S E

7. Quais as semelhanas ou diferenas observadas entre a escravido brasileira, na idade moderna e a escravido grega na antigidade? 0-0) Numa relao de valor, o escravo brasileiro e o grego valiam menos que a terra. 1-1) A escravido grega, ao contrrio da brasileira, no foi a base do sistema produtivo. 2-2) A escravido grega muitas vezes se dava por questes de no pagamento de dvidas, enquanto que a escravido no Brasil realizou-se dentro de um sistema que produzia para o mercado externo. 3-3) Tanto o escravo brasileiro como o escravo grego aparecem nos testamentos, na diviso dos bens mveis: gado, escravos, metais, vasos, armas, vesturio. 4-4) A escravido brasileira e a grega tinham componente racial: negros e ndios no Brasil; negros e chineses na Grcia. 8. Sobre a restaurao pernambucana e o fim do domnio holands podemos afirmar que: 0-0) A substituio de Nassau na administrao da Nova Holanda, em 1644, pelo Supremo Conselho, acirrou conflitos entre devedores (luso-brasileiros) e credores (holandeses). 1-1) Andr Vidal de Negreiros e Joo Fernandes Vieira, senhores de engenho e aliados dos holandeses, apoiaram a nova poltica financeira do Supremo Conselho. 2-2) Vrios combates foram travados pelas tropas luso-brasileiros e holandesas. Os mais importantes foram: o do Monte das Tabocas, o da Conveno de Beriberi e o do Monte Guararapes. 3-3) A partir de 1551, a guerra naval entre Inglaterra e Holanda possibilitou a diminuio do auxlio dos Estados Gerais aos holandeses em Pernambuco. 4-4) Os ingleses que haviam excludo a Holanda do seu comrcio, auxiliaram os lusobrasileiros na restaurao. 9. Leia o texto com ateno:
Sendo a terra que d ouro esterilssima de tudo o que se h mister para a vida humana, e no menos estril a maior parte dos caminhos das minas, no se pode crer o que padeceram a princpio os mineiros por falta de mantimentos, achando seno poucos mortos com uma espiga de milho na mo, sem terem outro sustento. Antonil, in Cultura e Opulncia do Brasil.

Sobre o abastecimento na regio das minas e sobre a circulao do ouro pode-se afirmar: 0-0) Apesar das grandes jazidas aurferas a regio foi abandonada devido esterilidade do solo. 1-1) A regio foi abastecida por produtos de outras regies, o que intensificou o comrcio pelo interior do Brasil. 2-2) Os mineiros abandonaram a atividade mineradora para se dedicarem ao cultivo do cacau e do fumo. 3-3) Com a produo de ouro, os mineiros puderam pagar, com esta moeda, produtos importados; conseqentemente, houve circulao de parte da produo aurfera. 4-4) O Rio So Francisco foi a grande via por onde passavam as boiadas, o fumo, o acar e outros produtos em direo s Minas Gerais.

360

HISTRIA

UFPE
F A S E

10. O panorama econmico da Colnia, no final do sculo XVIII, indica uma diversificao da produo agrcola, como conseqncia da decadncia da minerao e da nova poltica econmica da coroa portuguesa. Sobre esta diversificao pode-se afirmar: 0-0) A explorao do pau-brasil fazia parte dessa nova poltica econmica e do ponto de vista da extenso territorial ocupava o primeiro lugar. 1-1) Os cultivos de fumo na Bahia, de algodo e arroz no Maranho e Par faziam parte da poltica de diversificao da produo agrcola, desse perodo. 2-2) As plantaes de caf no Par e a caa baleia na regio litornea criaram outra opo econmica, que, traduzida em lucro, substituiu as exportaes de minrio. 3-3) A produo de acar, ocupando a regio mida litornea desde o nordeste at o sudeste, foi reativada e, embora, outros produtos tropicais fossem lucrativos nenhum o substituiu, na pauta das exportaes. 4-4) As drogas do serto, cravo, pimenta, cacau, localizadas na regio amaznica, conseguiram ocupar o primeiro lugar na economia de exportao. 11. Sobre os movimentos revolucionrios ocorridos na Frana, durante o sculo XIX, a historiografia d nfase s condies da pequena burguesia e do proletariado. Sobre estes movimentos podemos afirmar: 0-0) As medidas que Carlos X adotou, em 1830, atingiam a pequena burguesia e o proletariado: dissolveu a cmara, restringiu o direito ao voto e promulgou lei contra a imprensa. 1-1) A insatisfao da pequena burguesia e do proletariado, que trabalhava quatorze horas por dia, o apoio da Guarda Nacional e de republicanos radicais possibilitaram, aps lutas cruentas em Paris, a instalao do governo republicano provisrio, em 1848. 2-2) Na disputa para presidente da repblica francesa entre o General Cavagnac e Lus Bonaparte (sobrinho de Napoleo), venceu este ltimo. Apoiado pelo exrcito e pelos pequenos proprietrios de terras faz uma poltica favorvel pequena burguesia e ao operariado francs, culminando com o golpe de estado conhecido como 18 Brumrio. 3-3) O governo de Carlos X desgostava a nobreza por no indeniz-la das perdas territoriais e apoiava as reivindicaes do proletariado quanto diminuio das horas dirias de trabalho. 4-4) A reao ao governo de Lus Felipe resultou na Revoluo de 1848 na Frana, que fracassou por causa da aliana do proletariado com os grupos mais conservadores.

361

HISTRIA

UFPE
F A S E

12. A poltica externa brasileira em direo regio do Prata, no sculo XIX, caracterizou-se por intervenes e guerras: 0-0) O Uruguai esteve sob o controle brasileiro de 1816 a 1827, sob o nome de Provncia Cisplatina. 1-1) O Vice-Reino do Prata se constitua das provncias que hoje correspondem ao Uruguai, Paraguai, Argentina e Rio Grande do Sul. Por interveno de D. Joo VI o Rio Grande do Sul se desmembrou e foi anexado ao Imprio Brasileiro. 2-2) As lutas na regio Cisplatina levariam o Brasil a mais uma interveno nessa regio a guerra contra Oribe e Rosas, entre 1850 e 1852. 3-3) A trplice aliana entre Uruguai, Brasil e Argentina, em 1865, que redundou na Guerra do Paraguai, pode ser explicada pelo fato do Brasil e da Argentina estarem integrados ordem mundial, dominada pela Inglaterra, e o Paraguai ter se transformado num pas de economia slida e fora militar considervel, independente dessa ordem. 4-4) A Argentina enfrentou a Guerra do Paraguai praticamente sozinha, pois Brasil e Uruguai estavam empenhados em resolver seus problemas de fronteira, ainda decorrentes da Colnia do Sacramento e dos sete Povos do Uruguai. 13. O Convnio de Taubat, firmado pelos governadores de So Paulo, Rio de Janeiro e Minas Gerais, em 27 de fevereiro de 1906, resultou para a economia cafeeira nas seguintes medidas: 0-0) garantia de preos mnimos para os produtores. 1-1) avaliao e critrios para a exportao do produto; o caf inferior ao tipo 7 deveria ser consumido no pas. 2-2) diminuio dos impostos para facilitar a ampliao das reas de cultivo. 3-3) compra e reteno, pelo governo, de toda a produo cafeeira e responsabilidade total do governo na exportao do produto. 4-4) organizao de servio permanente de propaganda da produo e comercializao do caf. 14. A industrializao brasileira o resultado de um longo processo em que se conjugaram vrios fatores. Quais as vrias etapas deste processo? 0-0) A substituio do trabalho escravo pelo trabalho assalariado. 1-1) O processo de substituio de importaes que se estende da Primeira Guerra Mundial at 1930. 2-2) A partir de 1950, a industrializao se caracterizar pela produo de bens perecveis e semi-durveis e pela hegemonia das empresas nacionais. 3-3) A partir do segundo governo Vargas a industrializao se vincula, totalmente, ao capital internacional americano. 4-4) A industrializao, durante o governo de Juscelino Kubitschek, fez-se dentro de um processo ambguo em relao burguesia brasileira, com a penetrao do capital estrangeiro.

362

HISTRIA

UFPE
F A S E

15. A frica conheceu vrias modalidades de conflito em seu processo de descolonizao, durante os sculos XIX e XX. Sobre este processo assinale, falso ou verdadeiro nas proposies abaixo: 0-0) O imprio colonista mais extenso na frica esteve sob o controle da Gr-Bretanha e dos britnicos. 1-1) As colnias francesas ocupavam uma extenso de quase 1/3 do territrio africano e a maior parte se localizava ao noroeste. 2-2) A independncia da Libria e a independncia da Nambia so os limites do incio e do fim desse processo. 3-3) Aps a descolonizao poucas regies conheceram conflitos ps-coloniais devido Organizao dos Estados Africanos ter controle sobre as novas naes. 4-4) A Rodsia, aps a independncia, retomou o nome pelo qual seus habitantes naturais a chamavam Zimbbue. 16. Entre os anos 50 e 60, o Brasil assistiu a uma verdadeira exploso cultural, revelando tanto um Brasil urbano quanto um Brasil rural, ambos conflitados pela luta de classes. Identifique nas proposies abaixo as que se relacionam com a cultura desse perodo: 0-0) Os filmes O Cangaceiro , de Lima Barreto e O Pagador de Promessas, de Anselmo Duarte, foram premiados em Cannes. 1-1) Nessa dcada Carmem Miranda tornou-se a embaixatriz do samba nos Estados Unidos e a msica de Ari Barroso, a Aquarela do Brasil foi escolhida como trilha sonora de um filme de Walt Disney. 2-2) Guimares Rosa publicou Grande Serto: Veredas, renovando a linguagem literria. 3-3) Nasce o rdio no Brasil, e com ele se desenvolve uma cultura de massas. O samba, a msica francesa e a norte-americana foram popularizadas. 4-4) Inaugura-se a televiso brasileira e surge o movimento Bossa Nova, com as composies de Joo Gilberto, Tom Jobin e cantoras como Nara Leo e Silvinha Telles.

363

HISTRIA

UFRGS

1. O soberano dividiu o seu imprio em provncias, chamadas satrapias, sendo a terra considerada como propriedade real e trabalhada pelas comunidades. Estas caractersticas identificam o a) imprio dos persas durante o reinado de Dario. b) imprio babilnico durante o governo de Hamurabi. c) antigo imprio egpcio durante a dinastia de Quops. d) reino de Israel sob o comando de Davi. e) estado espartano durante a vigncia das leis de Dracon. 2. Em relao sociedade espartana, assinale a opo que no corresponde camada social dos hilotas. a) Constituam a massa de populao vencida, subjugada e pertencente ao Estado. b) Enquanto fora-de-trabalho, eram expropriados pelos espartanos. c) Cultivavam a terra com os seus instrumentos de trabalho, pagando uma renda fixa em espcie. d) Como preveno de revoltas e frente ao perigoso aumento demogrfico que apresentavam, sofriam regularmente os kriptios, formas de represso e extermnio realizados por jovens espartanos. e) Desenvolviam atividades mercantis que lhes possibilitavam acumular pequenas fortunas com as quais compravam ttulos de cidadania. 3. Entre os fatores que explicam o renascimento do comrcio, a partir do sculo XI na Europa ocidental, podemos apontar: I - A invaso da Europa por diversos povos brbaros que estimularam as trocas comerciais. II - Uma renovao das prticas agrcolas com a difuso de instrumentos de trabalho como o arado de ferro, a foice e a enxada. III - O movimento das cruzadas que, ao reabrir o Mediterrneo, intensificou os contatos com o Oriente. Quais esto corretos? a) Apenas I b) Apenas I e II c) Apenas I e III d) Apenas II e III e) I, II e III

4. O processo de formao do Estado Nacional na Pennsula Ibrica est diretamente ligado Reconquista, que significou a) b) c) d) e) cobrana de impostos efetivada pelas casas reinantes aos invasores turcos. formao de exrcitos nacionais para enfrentar o particularismo feudal. luta dos cristos para recuperar os territrios ocupados pelos muulmanos. confisco dos bens da Igreja para aumentar o poder feudal. ocupao de territrios invadidos por proprietrios rurais e pela burguesia comercial urbana.

364

HISTRIA

UFRGS

5. Na explorao da mo-de-obra indgena na Amrica, os espanhis criaram duas instituies: a) os adelantados e a reserva. b) as capitanias e os forais. c) a encomienda e a mita. 6. Considere o mapa abaixo: d) os conselhos das ndias e as haciendas. e) os quinhes e os monoplios.

Fonte: CHLA, vol. I, p. 451

O mapa refere-se I - s principais rotas comerciais portuguesas dos sculos XVI e XVII. II - s viagens de Cristovo Colombo no sculo XVI. III - s viagens de circunavegao de Ferno de Magalhes. Quais esto corretas? a) Apenas I b) Apenas II c) Apenas III e) I, II e III d) Apenas I e III

7. Motivos por que a Companhia das ndias Ocidentais deve tentar tirar ao rei da Espanha a terra do Brasil e lista de tudo o que o Brasil pode produzir anualmente. Este ttulo de livro da poca nos d uma viso do esprito que norteou o movimento das invases holandesas. Sobre estas podemos afirmar que a) a poltica de invases dos holandeses visava restabelecer o protecionismo ao comrcio colonial, porque os produtos brasileiros s podiam ser comprados pelos comerciantes espanhis. b) a criao da Companhia das ndias Ocidentais visava romper o bloqueio econmico portugus que impedia o livre comrcio do acar. c) os planos dos holandeses visavam reapropriao dos lucros da distribuio e venda do acar brasileiro, prejudicados pela dominao filipina sobre Portugal. d) a hegemonia holandesa j estava estabelecida na Europa e era necessrio agora ocupar a rea aucareira com trabalhadores livres. e) os espanhis, ao dominarem o Brasil, pretendiam desenvolver uma colonizao fora do sistema mercantilista, e isto era lesivo aos interesses da Companhia das ndias Ocidentais.

365

activities with no

HISTRIA

UFRGS

8. Considere abaixo, a nota de 1776 do Marqus de Pombal ao Embaixador da Frana em Lisboa.


1) 2) 3) as colnias devem estar debaixo da imediata dependncia de proteo dos fundadores; o comrcio e a agricultura delas devem ser exclusivos dos mesmos fundadores; aos fundadores pertencem tambm privativamente os teis provenientes da agricultura, comrcio e navegao das colnias; para que prestem a utilidade desejada, as colnias no podem ter o necessrio para subsistir por si sem dependncia da metrpole; quando entretm algum comrcio com estrangeiros, tudo o que importa esse comrcio clandestino e essas mercadorias introduzidas um verdadeiro furto que se faz respectiva metrpole e um furto punvel pelas leis dos respectivos soberanos [...]; portanto, no atentam contra a liberdade do comrcio as potncias que o restringem nas colnias a favor dos seus vassalos, e todo o governo que por indiferena tolere nos seus portos a contraveno dos cinco princpios anteriores pratica uma poltica destrutiva do comrcio e da riqueza da sua nao.

4)

5)

6)

Com relao a essa nota, so feitas as seguintes afirmativas: I - A liberdade de comrcio a base de todo o antigo sistema colonial. II - A subordinao das colnias s metrpoles abrange a poltica, a agricultura, o comrcio e a navegao. III - O comrcio entre as colnias e metrpoles no estabelece dependncia, tornando a colnia livre em termos de poltica econmica. Quais esto corretas? a) Apenas I b) Apenas II c) Apenas III d) Apenas II e III e) I, II e III

9. A charge abaixo uma homenagem-denncia do cartunista gacho Santiago aos desaparecidos latino-americanos e suas famlias, atingidos pela violncia terrorista das ditaduras militares durante as dcadas de 60 a 80. Como referncia, o cartunista utilizou uma conhecida obra de um gnio do Renascimento. O artista renascentista e o ttulo da obra que inspirou a charge so, respectivamente: a) Masaccio - A expulso de Ado e Eva do Paraso. b) Leonardo da Vinci - A Virgem e o Menino. c) Rafael - Madona Sixtina. d) El Grecco - A Crucificao. e) Michelngelo - Piet.

366

HISTRIA

UFRGS

10. A fim de reafirmar a unidade da Igreja Catlica, o Papa Paulo III convocou o Conclio de Trento para organizar a Contra-Reforma. Tal fato visava: I - reafirmar que o nico texto autntico para leitura e interpretao da Bblia era a Vulgata traduo latina feita por So Jernimo no sculo IV. II - confirmar os dogmas e prticas rituais catlicos tais como a presena de Cristo na Eucaristia, o culto Virgem e aos Santos e os Sete Sacramentos. III - manter a proibio do casamento para os padres, criando seminrios para formao dos mesmos. Quais esto corretas? a) Apenas I b) Apenas II c) Apenas III d) Apenas I e II e) I, II e III

11. Nas origens das revolues democrtico-burguesas dos sculos XVIII e XIX, encontram-se condies que envolvem o conflito entre o que se poderia denominar de foras de transformao e as chamadas foras de conservao, isto : a) de um lado, as relaes sociais estabelecidas essencialmente em torno da terra e, de outro, um modelo de explorao baseado na posse do homem pelo homem. b) os direitos de socializao da terra pelo proletariado agrcola e uma economia assentada predominantemente na agricultura. c) de um lado, o capitalismo agrrio e industrial e as reivindicaes da burguesia e, de outro, os remanescentes da economia feudal e os setores privilegiados representados pelo clero e pela aristocracia. d) o progresso do poder real que tende a organizar o Estado Moderno e a permanncia do exerccio da justia pelo soberano sobre seus vassalos. e) a resistncia crescente dos camponeses s exigncias dos senhores e o aparecimento de uma forma de transio entre a economia agrcola feudal e a economia agrcola capitalista: o trabalho escravo. 12. A imigrao europia, do sculo XIX, oportunizou mudanas para o Rio Grande do Sul, tendo como resultado a) o predomnio da grande explorao agrcola, utilizando uma mo-de-obra extensiva. b) o desenvolvimento de culturas tropicais, aliado formao de comunidades de pequenos proprietrios. c) a ampliao da mo-de-obra escrava devido ineficincia do imigrante europeu. d) o aparecimento de um campesinato voltado para a produo de alimentos bsicos e a utilizao da mo-de-obra familiar. e) a utilizao da mo-de-obra indgena j completamente catequizada pelas misses crists.

367

HISTRIA

UFRGS

13. Um projeto alternativo ao Estado Nacional Brasileiro estabelecido pela Carta Constitucional de 1824 e defendido na Guerra dos Farrapos apresentou a a) b) c) d) e) concentrao do poder no Imperador e no Conselho de Estado. instalao de uma Repblica. instaurao de uma Monarquia Constitucional. criao de uma Assemblia Nacional Popular. organizao de Comits Revolucionrios para sustentar o governo.

12. Entre as medidas liberais determinadas pelo Ato Adicional de 1834, encontra-se a a) b) c) d) e) instituio do poder Moderador. convocao de Assemblia Constituinte para elaborao de novo projeto constitucional. eleio de uma Regncia Trina Provisria em substituio ao Imperador Pedro I. criao de Assemblias Legislativas Provinciais. extenso do voto para todos os brasileiros.

15. Das rebelies internas ocorridas no Brasil durante o II Reinado destaca-se o sentido social da Revoluo Praieira de 1848, porque a) o governo rebelde aprovou uma Constituio que tornava cidados brasileiros os portugueses residentes no Brasil. b) pelo Manifesto ao Mundo os revoltosos pregavam o voto livre e universal para os brasileiros. c) o Imperador Pedro II estabeleceu uma poltica de conciliao, anistiando os lderes revoltosos e integrando-os ao Senado Vitalcio. d) entre as intenes dos revoltosos estava o desejo de livrar-se dos impostos excessivos sobre a extrao do ouro. e) o movimento visava isentar de servir no Exrcito chefes de famlia e proprietrios rurais.

368

HISTRIA
Instruo: Os dois documentos abaixo referem-se questo de nmero 16: Documento 1

UFRGS

...a converso da propriedade privada em propriedade coletiva, to preconizada pelo socialismo, no teria outro efeito que no fosse o de tornar a situao dos operrios mais precria, retirando-lhes a livre disposio do seu salrio... e toda a possibilidade de aumentar o seu patrimnio e melhorar a sua situao... A propriedade privada e pessoal para o homem de direito natural... O homem deve tomar com pacincia a sua condio; impossvel que, na sociedade civil, toda a gente seja elevada ao mesmo nvel... Contra a natureza todos os esforos so vos... O erro capital... o de crer que as duas classes so inimigas natas uma da outra, como se a natureza tivesse armado os ricos e os pobres para se combaterem mutuamente... No pode haver capital sem trabalho, nem trabalho sem capital. A concordncia engendra a ordem e a beleza... Toda a economia das verdades religiosas... de natureza a aproximar... os ricos e os pobres, lembrando s duas classes os seus mtuos deveres...

Documento 2
O Congresso reunido em Saint Imier declara: 1. Que a destruio de todo poder poltico o primeiro dever do proletariado. 2. Que toda organizao de um poder que pretende ser provisrio e revolucionrio para efetivar aquela destruio um engano, e seria to perigoso para o proletariado como todos os governos que existem hoje. 3. Que, rejeitando todo compromisso para chegar realizao da Revoluo Social, os proletrios de todos os pases devem estabelecer, fora de toda poltica burguesa, a solidariedade da ao revolucionria.

16. Os documentos acima correspondem a duas interpretaes sobre a realidade social do sculo XIX. Respectivamente, a) ao Socialismo Cientfico e Doutrina Social da Igreja. b) Doutrina Social da Igreja e ao Socialismo Libertrio. c) ao Socialismo Libertrio e ao Socialismo Utpico. d) ao Socialismo Utpico e ao Socialismo Cientfico. e) Doutrina Social da Igreja e ao Socialismo Cientfico.

369

HISTRIA
17. Considere as seguintes afirmaes sobre a Comuna de Paris.

UFRGS

I - Ocorreu como desdobramento imediato da crise provocada pela queda de Napoleo III e a consumao da derrota francesa ante a Prssia em 1871. II - Apresentou importantes medidas de cunho popular-progressista, visando dissolver o exrcito permanente, separar a Igreja do Estado, instituir o ensino gratuito e entregar as fbricas direo dos trabalhadores. III - Constituiu o exemplo histrico europeu mais bem sucedido, no sculo XIX, de conquista do poder pela burguesia liberal. Quais esto corretas? a) Apenas I b) Apenas II c) Apenas III d) Apenas I e II e) Apenas I e III

18. Sobre o movimento do Contestado (1912-1916), so feitas as seguintes afirmativas: I - ocorreu numa regio de disputa territorial entre os Estados de Santa Catarina e Paran. II - apresentou contedo religioso aliado a reivindicaes sociais. III - um de seus principais lderes foi Antnio Conselheiro. Quais esto corretas? a) Apenas I b) Apenas II c) Apenas III d) Apenas I e II e) I, II e III

19. Sobre a Aliana Liberal, formada no decorrer do processo de sucesso ao Governo Washington Lus, correto afirmar que a) visava alterar a estrutura da economia brasileira, atravs de um novo processo de redistribuio de terras. b) pretendia combater o sistema capitalista, estabelecendo uma poltica de compromisso com os trabalhadores. c) defendia o fim da propriedade privada e de qualquer forma de governo, sendo influenciada pelas idias de liberdade e igualdade. d) incentivava a sada do Estado como agente interventor nas relaes econmicas, abrindo espao para a livre competio e modernizao. e) anunciava o rompimento da poltica do caf com leite, marcando o fim do antigo pacto das oligarquias dominantes.

370

HISTRIA
Instruo: O mapa abaixo refere-se questo de nmero 20

UFRGS

O mapa acima refere-se frica colonial em 1914 e registra a diviso colonial do territrio pelas metrpoles europias.

20. Os territrios do imprio colonial portugus naquele perodo so os identificados no mapa atravs da legenda

371

HISTRIA
Instruo: As seguintes declaraes referem-se questo de nmero 21: Os verdadeiros chefes no tm nenhuma necessidade de cultura e cincia.

UFRGS

(H. Goering)

Quando ouo a palavra cultura, ponho a mo no revlver.


(J. Goebbels)

Os intelectuais so como as rainhas que vivem das abelhas trabalhadoras.


(A. Hitler)

Sem esprito militar a escola alem no poder existir. Um professor pacifista um palhao ou um criminoso. Deve ser exterminado.
(Ministro Schewemm - Bavria)

Professores alemes... nenhum menino e nenhuma menina da escola devem sair de vossas aulas sem o sagrado e firme propsito de ser um inimigo mortal do bolchevismo judeu, na vida e na morte.
(F. Weachter)

21. Contextualizando historicamente as declaraes acima, de lideranas nazistas na Alemanha, pode-se afirmar que a) o nazismo no tinha nenhum projeto para as reas de educao e cultura, pois dentro da perspectiva do culto ao corpo e da obedincia sem questionamentos, aquelas lhes eram completamente indiferentes. b) ao contrrio da produo cultural, qual eram refratrios, os nazistas permitiram a permanncia das diretrizes educacionais da Repblica de Weimar. c) tanto a educao como a cultura foram reas enquadradas dentro dos pressupostos bsicos do regime transformando-se em instrumentos ideolgicos de controle e propaganda. d) o Estado nazista interveio fortemente somente nas escolas freqentadas por alunos noarianos e filhos de pais bolcheviques. e) educao e militarizao da sociedade eram projetos excludentes dentro do projeto nazista de dominao.

372

HISTRIA
Instruo: Os textos abaixo referem-se questo de nmero 22:

UFRGS

Foi o senhor quem fez isso?, perguntou a Picasso o embaixador alemo em Paris, durante a Segunda Guerra Mundial, diante de uma foto do quadro Guernica. No, foram vocs, respondeu o artista. Um momento, senhores: vocs falam de Guernica? Ah, sim, lembro. Foi uma espcie de banco de provas da Luftwaffe. [...] Sim, foi lamentvel. Mas no podamos fazer outra coisa. Naquela poca, experincias assim no podiam ser feitas em outra parte.
Hermann W. Goering, interrogado em Nremberg.

22. Os relatos acima referem-se questo da internacionalizao da Guerra Civil Espanhola. Paradoxalmente, um dos elementos mais importantes para se entender o desfecho do conflito foi a) o apoio sovitico aos comunistas do POUM, o que provocou profundas divises entre os setores republicanos. b) a total ausncia de solidariedade internacional com a Repblica espanhola, contrastando com o apoio explcito de Hitler e Mussolini sublevao da direita anti-democrtica. c) a poltica de no-interveno anglo-francesa, justificada pelo que aquelas potncias consideravam ser um conflito interno dos espanhis. d) a incorporao militar da Espanha ao Eixo, no incio da Segunda Guerra Mundial. e) a falta de esprito de luta dos setores republicanos que, mesmo tendo armamento sovitico, claudicaram rapidamente. 23. Associe as obras literrias de fundo histrico, listadas na coluna da esquerda, aos temas indicados na coluna da direita. ( ( ( ( ) Memrias do crcere, Graciliano Ramos ) Guerra e paz, Leon Tolstoi ) Videiras de cristal, Luis Antnio de Assis Brasil ) Os Sertes, Euclides da Cunha (1) Guerra Napolenica na Rssia (2) Estado Novo no Brasil (3) Guerra Civil Espanhola (4) Episdio dos Muckers no Rio Grande do Sul (5) Guerra de Canudos

( ) Por quem os sinos dobram , Ernest Hemingway

A relao numrica correta, de cima para baixo, na coluna da esquerda, a) 2 - 1 - 4 - 5 - 3 b) 1 - 3 - 5 - 4 - 3 c) 2 - 3 - 4 - 5 - 1 d) 1 - 5 - 4 - 2 - 3 e) 2 - 1 - 3 - 5 - 4

373

HISTRIA

UFRGS

24. Em relao ao golpe de 1964, que expressa o colapso do populismo no Brasil, so feitas as seguintes afirmaes: I - As classes dominantes civis e militares e setores das classes mdias apoiaram a derrubada do Presidente Joo Goulart, pois ficaram temerosos com a crescente politizao das massas populares. II - Entre as foras da chamada direita destacavam-se organizaes como o Instituto de Pesquisas e Estudos Sociais (IPES) e o Instituto Brasileiro de Ao Democrtica (IBAD) financiadas pelos E.U.A. III - Os governadores de So Paulo, de Minas Gerais e da Guanabara conspiravam com a ala militar golpista. Quais esto corretas? a) Apenas I b) Apenas II c) Apenas III d) Apenas I e II e) I, II e III

25. Considere o texto abaixo da obra Morte e vida Severina de Joo Cabral de Melo Neto.
E se somos severinos Iguais em tudo na vida Morremos de morte igual Mesma morte Severina Que a morte de que se morre De velhice antes dos trinta De emboscada antes dos vinte, De fome um pouco por dia.

O texto acima est correlacionado com a a) b) c) d) e) dificuldade de sobrevivncia dos seringueiros no Amazonas no final do sculo XIX. guerrilha do Araguaia nos anos 70. misria crnica da populao no serto nordestino. luta dos camponeses no Movimento de Canudos. onda de greves da classe operria de So Paulo de 1917.

26. Considere as seguintes caractersticas do Regime Militar instaurado no Brasil em 31 de maro de 1964. I - Interveno do Estado na economia. II - Regime de cunho autoritrio, baseado na Doutrina de Segurana Nacional. III - Poder controlado pela cpula militar, rgos de informao e represso e burocracia tcnica do Estado. Quais esto corretas? a) Apenas I b) Apenas II c) Apenas III d) Apenas II e III e) I, II e III

374

HISTRIA

UFRGS

27. O II PND - Plano Nacional de Desenvolvimento - estabelecido durante o Governo Geisel visava substituio de importaes e busca de auto-suficincia do pas no setor de insumos bsicos, atravs de projetos tais como a a) assinatura de contratos de risco com empresas estrangeiras para prospeco do petrleo e o Programa Nacional do lcool. b) construo da rodovia Transamaznica e a destinao de uma srie de incentivos fiscais Zona Franca de Manaus para desenvolver o consumo na regio. c) manuteno do nvel das importaes e renegociao da dvida externa com a captao de recursos para o setor pblico no exterior. d) recuperao do crescimento econmico com a introduo das ORTN - Obrigaes Reajustveis do Tesouro Nacional - como fonte de recursos para a indstria automobilstica. e) criao da Superintendncia do Desenvolvimento do Nordeste (SUDENE) e o rompimento com o FMI para diminuir a dvida externa brasileira. 28. Em 1976, o Decreto-Lei n 6.639 da Presidncia da Repblica, mais conhecido como Lei-Falco, a) b) c) d) criou o voto distrital misto para municpios com mais de 200 mil habitantes. proibiu os comcios e reunies partidrias. limitou a propaganda eleitoral no rdio e na televiso. reestruturou as eleies diretas para prefeitos nos municpios enquadrados como de Segurana Nacional. e) estabeleceu o bipartidarismo no Brasil. 29. Considere a charge ao lado que retrata o General Ernesto Geisel: A charge est relacionada com a) a msica de Chico Buarque de Holanda da pea teatral Roda Viva. b) a composio de Geraldo Vandr vetada pela censura em 1968 e liberada durante a abertura. c) a produo musical de Caetano Veloso e Gilberto Gil feita no exterior durante o exlio. d) o filme Pr frente Brasil de Roberto Farias que trata da represso e tortura nos anos 70. e) a pea Boa Noite General, de Joo Vianny, que agitou o debate em torno da censura.

375

HISTRIA
30. Considere o texto abaixo sobre a Nova Guerra Fria.

UFRGS

A nova Guerra Fria consiste esquematicamente no seguinte: os Estados Unidos desencadeiam uma corrida armamentista convencional e estratgica - cujo ponto mximo a militarizao do espao pela IDS, ou projeto guerra nas estrelas - que os colocam em superioridade estratgica sobre a U.R.S.S. e abala a economia sovitica; a U.R.S.S., debilitada pelo aumento dos gastos militares e pelo embargo comercial dos E.U.A. e aliados, v-se obrigada a limitar seu apoio s revolues do Terceiro Mundo como contrapartida para uma reduo da presso militar americana contra si; assim, Washington e seus aliados mais militarizados (como Israel, Paquisto e frica do Sul) poderiam sufocar os movimentos e regimes revolucionrios surgidos na dcada anterior; paralelamente, buscariam controlar seus aliados-rivais economicamente bem-sucedidos (Europa e Japo), dividindo com eles o fardo dos gastos armamentistas e afastando-os da vantajosa cooperao econmica com a U.R.S.S. e a Europa oriental (da a luta contra a construo do gasoduto Sibria-Europa e a venda de tecnologia avanada aos pases socialistas); finalmente, os E.U.A. tentariam abrir os pases socialistas penetrao econmica ocidental, a qual aumentaria o controle sobre a poltica do bloco sovitico e forneceria alternativas financeiras e comerciais para a superao da estagnao do sistema capitalista. (Vizentini, P. Da Guerra Fria Crise. Porto Alegre: Ed. Universidade/UFRGS, 1996, p. 85)

Segundo o autor, pode-se afirmar que a) sem condies materiais de competir com a U.R.S.S., os E.U.A. estimularam a penetrao econmica e militar do Japo e da Alemanha Ocidental nos pases perifricos politicamente prximos do bloco sovitico. b) o projeto guerra nas estrelas levou a U.R.S.S. a abandonar todos seus interesses no Terceiro Mundo optando por uma estratgia geopoltica de ampliao do seu apoio Alemanha e ao Japo. c) a nova estratgia militar norte-americana foi muito bem sucedida, obrigando a U.R.S.S. a retirar-se gradativamente do Terceiro Mundo, e evitando que Japo e Alemanha Ocidental fossem satelitizados pelos soviticos. d) sofrendo o impacto das exigncias da nova competio militar, a U.R.S.S. recua do seu papel de potncia de atuao mundial, diminuindo sua presena no Terceiro Mundo e tendo dificuldades para controlar e evitar a penetrao capitalista dentro das fronteiras do leste europeu. e) perdendo contato com as economias do Japo e da Alemanha Ocidental, a U.R.S.S. intensifica relaes comerciais com os pases socialistas do Terceiro Mundo para que estes financiem a manuteno da cada vez mais sofisticada parafernlia militar da lgica da Guerra Fria.

376

HISTRIA

UFRGS

31. A charge abaixo expressa um dos problemas mais prementes da sociedade europia na atualidade.

E ele? Onde esto seus documentos?, diz o policial francs. A charge publicada pelo Journal du Jeudi, de Uagadugu, capital de Burkina Faso, ironiza a invaso da polcia francesa a uma igreja de Paris, no ms passado, para expulsar imigrantes africanos que vivem ilegalmente no pas. Folha de S. Paulo, 08/09/96.

Em relao presena de imigrantes de pases pobres nos pases desenvolvidos da Europa Ocidental fazem-se as seguintes afirmaes: I - A onda xenfoba que tem varrido a Europa nos ltimos anos est relacionada forte presena de africanos e latino-americanos, entre outros, nos pases mais desenvolvidos da regio. II - A presena de imigrantes clandestinos na Europa Ocidental resulta da tendncia de grande parte dos capitalistas europeus, que os exploram como mo-de-obra, mesmo que ilegal, extraindo deles uma mais-valia bem superior que possvel retirar da fora-de-trabalho nacional. III - O problema da imigrao e a procura de trabalho no mercado europeu tem sido utilizado para esconder os efeitos do custo do processo de integrao europeu, da livre circulao de capital procura de maiores taxas de lucro, e do desemprego estrutural. Quais esto corretas? a) Apenas I b) Apenas II c) Apenas III d) Apenas II e III e) I, II e III

377

HISTRIA

UFRGS

32. Considerando a histria como processo em constante construo, transformao e acumulao, pode-se afirmar que o fenmeno conhecido como Globalizao ou mundializao do capitalismo a) corresponde exclusivamente ao contexto da desagregao do socialismo na U.R.S.S. e no Leste europeu. b) inicia somente aps a desestruturao do estado de bem-estar social capitalista. c) um fator que se manifesta pela primeira vez com a Nova Ordem Mundial da dcada de 90 e com o desmantelamento dos blocos econmicos. d) faz parte de um processo que se inicia com a reconstruo do mundo europeu, logo aps a 1 Guerra Mundial. e) tem antecendentes nos desdobramentos da Revoluo Industrial, bem como na exportao de capitais, caracterstica do Imperialismo a partir do final do sculo XIX. 33. A ocupao e colonizao da Faixa de Gaza, Cisjordnia e das Colinas de Golan por Israel sobre seus vizinhos rabes, foi iniciada a partir da a) Guerra dos Seis Dias (1967). b) Guerra do Yom Kippur (1973). c) Revoluo Islmica (1979). d) Intifada (1987). e) Guerra do Golfo (1991).

34. Em 1996 o Prmio Nobel da Paz foi concedido a dois lderes da luta pela independncia de Timor Leste. Esta regio situada no Pacfico Sul foi invadida e anexada pela Indonsia e ex-colnia a) da Inglaterra. b) de Portugal. c) da Frana. 35. Considere o texto abaixo:
Que pas este
(Renato Russo) Nas favelas, no Senado Sujeira pra todo lado Ningum respeita a Constituio Mas todos acreditam no futuro da nao Que pas este (....)

d) dos Estados Unidos. e) da Itlia.

Em relao sociedade brasileira o texto acima I - denuncia a existncia de impunidade no conjunto da sociedade e a falta de compostura de parte da elite poltica brasileira. II - explicita a inexistncia de desigualdade social e concentrao de renda no pas. III - esconde as mazelas da sociedade atravs de um discurso alienado e ufanista. Quais esto corretas? a) Apenas I b) Apenas II c) Apenas I e II d) Apenas I e III e) I, II e III

378

HISTRIA

UnB

Nas questes de 1 a 15, marque: itens CERTOS, na coluna I; itens ERRADOS, na coluna II. Use, para as devidas marcaes, a Folha de Rascunho e, posteriormente, a Folha de Respostas. Questo 1 Leia o trecho abaixo, extrado do poema de Tirteu (sc. VII a.C. Esparta) chamado aret (excelncia).
1

10

um bem comum para a cidade e todo o povo / que um homem aguarde, de ps fincados, na primeira fila, / encarniado e de todo esquecido da fuga vergonhosa, / expondo sua vida e nimo sofredor, / e, aproximando-se, inspire confiana com suas palavras ao que lhe fica ao lado. / Um homem assim distinguese no combate. / Em breve derrota as falanges furiosas dos inimigos, / com seu ardor detm as vagas da batalha. / Se ele cair na primeira fila, perdendo a cara vida, / deu glria cidade, ao povo e ao pai, / (...) O seu tmulo, os seus filhos sero notveis entre os homens, / bem como os filhos dos filhos, e toda a posteridade. / Jamais perecer a sua nobre glria e o seu renome, / (...).

Com o auxlio do texto, julgue os itens seguintes, relativos histria da Grcia arcaica. (0) No momento de constituio da polis, valores e poderes aristocrticos ainda se encontravam presentes na formao do homem grego. (1) No sc. VII a.C. espartano, a antiga aristia combate singular entre dois guerreiros j cede lugar s batalhas hoplticas, em que o sucesso militar depende do desempenho coletivo da falange, dos ps fincados, na primeira fila (li. 2), do compromisso com o companheiro que lhe fica ao lado (li. 6). (2) O atributo maior do heri homrico, a valentia, fundamental para a conquista da fama, mantm-se e transforma-se no renome do soldado da polis, que d glria cidade, ao povo e ao pai (li. 10-11). (3) A definio do estatuto dos cidados como semelhantes e iguais, base para a consolidao da polis, contradiz as transformaes militares que substituem o combate individual pelo soldado hoplita.

379

HISTRIA
Questo 2

UnB

A respeito de alguns aspectos polticos da civilizao do ocidente medieval, julgue os itens abaixo. (0) Apesar de fascinados pela idia imperial romana, estranha a seus costumes, os brbaros somente reuniram condies para concretiz-la no sculo VIII, por meio do reino franco e da figura de Carlos Magno. (1) O fato de os francos terem sido os primeiros germnicos a se converterem ao cristianismo, ponto de partida para os laos estreitos que estabeleceram com a Igreja nos anos setecentos, favoreceu o projeto unificador carolngio. (2) A mxima do cristianismo, que insistia em dar a Deus o que de Deus e a Csar o que de Csar, manteve-se na Idade Mdia, pois nem monarcas nem papas confundiam as esferas do temporal e do espiritual. (3) Uma das causas da emergncia poltica dos poderes particularistas (feudos) e nacionalistas (monarquias) deveu-se aos choques constantes, que passaram a ocorrer a partir do sculo VIII, entre os poderes universalistas da Igreja e os do Imprio. Questo 3 Para ganhar o favor popular, o candidato deve conhecer os eleitores por seu nome, elogilos e bajul-los, ser generoso, fazer propaganda e levantar-lhes a esperana de um emprego no governo. (...) A generosidade um tema amplo. Talvez sua renda privada no possa atingir todo o eleitorado, mas seus amigos podem ajud-lo a agradar a plebe. Oferea banquetes e providencie que seus amigos faam o mesmo, procurando atingir os eleitores ao acaso e o eleitorado especfico de cada tribo. (...) Faa com que os eleitores falem e pensem que voc os conhece bem, que se dirige a eles pelo nome, que sem parar e conscienciosamente procura seu voto, que voc generoso e aberto, que, mesmo antes do amanhecer, sua casa est cheia de amigos, que todas as classes so suas aliadas, que voc fez promessas para todo mundo e que as cumpriu, realmente, para a maior parte das pessoas.
Ccero. Notas sobre as eleies, versculos 41, 50, 52, 54, apud: P. Mackendrick, The Roman mind at work, p. 178-9.

Com o auxlio das palavras de Ccero (106-43 a.C.), julgue os itens a seguir, relativos histria da Roma antiga. (0) As prticas clientelistas eram inexistentes no mundo poltico republicano, sendo a amizade e o compadrio relaes que no ultrapassavam a esfera do privado. (1) O Tribunal Eleitoral romano geralmente punia os abusos do poder econmico com a cassao dos candidatos infratores. (2) Na poca de Ccero e mesmo depois, com a poltica do po e circo, o povo, a plebe ou a massa, constitua um elemento a ser cativado e no coagido. (3) A propaganda eleitoral da Repblica visava principalmente mulheres e escravos, que formavam percentagem considervel do colgio de votantes.

380

HISTRIA
Questo 4

UnB

Leia o texto que se segue, trecho da resposta do Rei Lus XV ao Parlamento de Paris, em 1766. exclusivamente na minha pessoa que reside o poder soberano (...) s de mim que os meus tribunais recebem a sua existncia e a sua autoridade; a plenitude dessa autoridade, que eles no exercem seno em meu nome, permanece sempre em mim, e o seu uso no pode nunca ser voltado contra mim; a mim unicamente que pertence o poder legislativo sem dependncia e sem partilha (...) a ordem pblica inteira emana de mim, e os direitos e interesses da Nao, de que se ousa fazer um corpo separado do Monarca, esto necessariamente unidos com os meus e repousam unicamente nas minhas mos.
Gustavo de Freitas. 900 textos e documentos de Histria.

Com o auxlio das informaes contidas no texto, julgue os itens abaixo, relativos ao Estado nacional moderno. (0) Formado na crise do sistema feudal, o Estado moderno ops-se tanto aos particularismos urbanos, feudais e regionais quanto ao universalismo da Igreja e ao antigo ideal romanogermnico de imprio. (1) Em O prncipe, Maquiavel defende a existncia de um Estado unificado, com um poder poltico forte, centralizado e laico. (2) A expresso maquiavelismo pode ser entendida a partir da concepo, presente em O prncipe, de que no h limite tico ou moral s aes do soberano que, visando manuteno da vida e do Estado, est livre para o emprego de quaisquer meios. (3) A doutrina do direito divino dos reis, elaborada por Thomas Hobbes, em seu livro Leviat, constituiu o nico caminho de justificao terica e de legitimao ideolgica do absolutismo. Questo 5 O monoplio do comrcio da colnia, portanto, com todos os outros expedientes mesquinhos e malignos do sistema mercantilista, deprime a indstria de todos os outros pases, mas principalmente a das colnias, sem que aumente em nada pelo contrrio, diminui a indstria do pas em cujo benefcio adotado.
Adam Smith. A riqueza das naes.

Com o auxlio das informaes contidas no texto, julgue os itens seguintes. (0) O liberalismo econmico, teoria surgida no final do sculo XVIII, opunha-se s prticas mercantilistas que alimentavam o poder do Estado com a concesso de monoplios, protecionismo e privilgios a determinados grupos, defendendo a livre concorrncia e o cmbio livre. (1) Contestando a interferncia controladora do Estado na economia, liberais como Adam Smith viam a atividade econmica regida por leis naturais, centrando sua filosofia na lei da oferta e da procura. (2) Identificado com o capitalismo que as revolues burguesas e a Revoluo Industrial consolidavam, o liberalismo defendia a diviso do trabalho, tanto no plano interno quanto no internacional. (3) Embora integrantes de um mesmo contexto histrico, Iluminismo e liberalismo econmico divergiam em um ponto central: a limitao do poder estatal, tese no-encampada pelos iluministas.

381

HISTRIA
Questo 6

UnB

A Revoluo Americana (1775-1783), a primeira Revoluo Francesa (1789-1799) e a Revoluo Industrial na Inglaterra (1760-1830) definiram num espao de menos de 30 anos os contornos de um novo processo social, econmico, poltico e cultural. Essas trs revolues, mais os movimentos de independncia nas colnias europias na Amrica Latina, abrem uma nova poca no Ocidente: a Histria Contempornea. A Histria Contempornea se inicia marcada por um novo conceito de civilizao. Formaram-se, ento, em oposio aos valores e s caractersticas do perodo anterior (Histria Moderna), novos conceitos que merecem destaque e que passaram a fazer parte do universo mental do homem contemporneo, como: civilizao industrial; democracia representativa e cidadania; soberania nacional e independncia; liberalismo e socialismo.
Carlos Guilherme Mota. Histria moderna e contempornea.

Com o auxlio das informaes contidas no texto, julgue os itens a seguir. (0) A Revoluo Industrial unifica capital e trabalho, produtor e meios de produo, alm de consolidar a dinmica sociedade estamental. (1) O liberalismo combate, no campo econmico, as estruturas do mercantilismo e do colonialismo e, no poltico, as formas absolutistas de Estado. (2) O socialismo, que ganha corpo no decorrer do sculo XIX, corresponde a uma viso de mundo que se ope ordem burguesa. (3) O fracasso dos movimentos de independncia na Amrica Latina, ao longo da primeira metade do sculo XIX, pode ser explicado pela atuao da Inglaterra em defesa do pacto colonial. Questo 7 Leia o seguinte texto, retirado de uma declarao do prefeito de Guernica, em 1937. Guernica foi ferida, mas no morrer. Da rvore brotaro novas folhas verdes em toda primavera, seus filhos a ela retornaro, suas casas sero reconstrudas, suas igrejas escutaro novamente seus hinos e preces (...) Guernica, smbolo de nossas liberdades nacionais e smbolo da ferocidade do fascismo internacional, no pode morrer.
Editora Abril. Histria do sculo XX.

Julgue os itens abaixo, referentes ao perodo entre guerras (1918-1939). (0) A dcada de 1930, marcada por uma das maiores crises econmicas mundiais a Grande Depresso assistiu a uma vigorosa e tensa polarizao ideolgica entre esquerda (frentes populares) e direita (partidos fascistas). (1) O carter perifrico da Espanha, especialmente em funo de seu atraso econmico, impediu que a Guerra Civil Espanhola (1936-39) tivesse uma possvel vinculao com o quadro poltico europeu da poca. (2) O impacto da implantao de regimes fascistas em pases como Alemanha, Itlia, Espanha, Portugal e Japo acabou por fazer refluir a trajetria socialista da Unio Sovitica, minando seu esforo de consolidao. (3) Liderada pelos Estados Unidos, a Liga das Naes foi uma experincia vitoriosa de ordenamento das relaes internacionais, tendo inclusive impedido que os regimes totalitrios praticassem a poltica expansionista que preconizavam.

382

HISTRIA
Questo 8 Julgue os itens seguintes, relativos ao mundo contemporneo.

UnB

(0) Alteraes no Conselho de Segurana, reduo do corpo burocrtico, superao do crnico dficit oramentrio e reestruturao para atuar em uma nova realidade internacional so alguns dos desafios que a ONU, com mais de cinqenta anos de existncia, est enfrentando. (1) A criao do Estado da Palestina, resultante da vitria iraquiana na Guerra do Golfo, contribuiu para a reduo das tenses no Oriente Mdio. (2) De origem desconhecida, com mais de cinco mil anos de histria, os bascos ocupam territrios espanhis e franceses e lutam pela criao de um Estado independente. (3) Entre todos os pases rabes, o Ir o que possui o maior contingente de cristos em seu territrio, os quais vivem em luta incessante contra o governo da maioria islmica. Questo 9 A respeito da histria contempornea do Chile, julgue os itens que se seguem. (0) Quando presidente, Salvador Allende estatizou vrias indstrias e acelerou o processo de reforma agrria, o que deu a seu governo caractersticas claramente socialistas. (1) No incio da dcada de 70, o Chile acolheu vrios exilados brasileiros, ideologicamente identificados com a esquerda e perseguidos pela ditadura militar. (2) O presidente Augusto Pinochet, sucessor de Allende, deu continuidade poltica socialista, constituindo-se, alm disso, em um dos principais defensores dos direitos humanos na Amrica Latina. (3) Os Estados Unidos mantiveram-se afastados da poltica chilena, exceo feita contribuio da CIA na conteno das foras radicais de direita, no final do governo Allende. Questo 10 A respeito da histria contempornea de Cuba, julgue os itens seguintes. (0) A chamada crise dos msseis cubanos desencandeou-se quando a URSS descobriu que os EUA estavam construindo uma base para o lanamento de msseis de mdio alcance naquela ilha. (1) Sob a liderana de Fidel Castro, a Cuba ps-revolucionria manteve viva a pluralidade poltica, consubstanciada pelo rico e multifacetado sistema partidrio. (2) Precocemente industrializada no contexto latino-americano, Cuba no teve dificuldades em se auto-sustentar economicamente aps a revoluo socialista. (3) A revoluo cubana liderada por Castro e Guevara funcionou como uma espcie de modelo para outros movimentos revolucionrios de esquerda que se desenvolveram na Amrica Latina nos anos 60.

383

HISTRIA
Questo 11

UnB

Quantos aos aspectos polticos e administrativos da vida colonial do Brasil no sculo XVI, julgue os seguintes itens. (0) O absolutismo portugus estendia-se ao Brasil, centralizando o monarca as decises mais relevantes da vida poltica e administrativa da colnia. (1) O imperador governava diretamente a colnia, sem delegar poderes a uma burocracia e sem rgos executores da poltica administrativa no Brasil. (2) A criao do governo-geral foi uma forma de garantir efetivamente a unidade e a centralizao administrativa da colnia. (3) As eleies municipais, em princpio indiretas, eram conduzidas pelos homens bons, reunidos nas cmaras. Questo 12 Julgue os itens abaixo, relativos vida econmica das duas primeiras dcadas do Brasil Imperial. (0) Os Tratados de 1810, entre Portugal e Inglaterra, herdados pelo Brasil independente, implicaram a transplantao da dependncia do Brasil Imperial em relao aos interesses britnicos. (1) A invaso de produtos ingleses, nos primeiros tempos do Imprio, foi acompanhada pelo aumento expressivo das exportaes brasileiras. (2) A expanso da agricultura tradicional brasileira, ampliada pela situao internacional favorvel e pela renovao de mtodos na produo domstica, levou retomada do desenvolvimento das regies Norte e Nordeste do pas. (3) O caf, apesar de ter vivido sua fase urea na segunda metade do sculo XIX, j era produto expressivo nas exportaes brasileiras.

384

HISTRIA
Leia o texto a seguir e responda s questes 13 e 14.
1

UnB

10

A partir do golpe de 1930 fora possvel apresentar inovaes em todas as esferas de poder, de tal modo que, impulsionados pela corroda imagem da R e p b l i c a Ve l h a , o s a s s i m c h a m a d o s revolucionrios de 30 puderam, com o apoio popular, acabar com a existncia dos partidos polticos, o produto-tipo da cultura poltica liberal. Com o golpe de 1930, sequer houve um partido para a arregimentao das massas que, no s comearam a identificar-se prontamente com a p ro p o s t a d e u m e s t a d o t o t a l i t r i o , c o m o demonstravam cada vez mais seu clamor por Getlio Vargas.
Elizabeth Cancelli. O mundo da violncia.

Questo 13 Com referncia ao mundo poltico anterior Revoluo de 1930, julgue os seguintes itens. (0) A chamada Repblica Velha viveu o paradoxo de ter um sistema poltico formal avanado e uma sociedade cujas prticas polticas eram anacrnicas. (1) A existncia de partidos conferia verniz liberal estrutura poltica da Repblica Velha. (2) O voto universal, que inclua analfabetos, mulheres e indgenas, foi uma das marcas do liberalismo das primeiras dcadas da Repblica. (3) A poltica dos governadores, o caf-com-leite, o curral eleitoral e o voto de cabresto foram prticas marcantes da poltica brasileira na Repblica Velha. Questo 14 Com referncia s inovaes em todas as esferas de poder (li. 2) a que o texto se refere, julgue os itens a seguir. (0) Os revolucionrios de 1930 mantiveram instituies polticas liberais, como o sistema partidrio da Repblica Velha. (1) Vargas, depois de liderar a Revoluo de 1930, e especialmente depois de promover o golpe do Estado Novo, criou um regime com pouca ou nenhuma intermediao de instituies polticas autnomas entre a sociedade e o Estado. (2) O golpe de 1937 veio a ser a comprovao definitiva das caractersticas centralistas e autoritrias que vinham se desenhando desde a Revoluo de 1930. (3) A mentalidade de massa, o Estado de massa, o personalismo carismtico do lder e as formas corporativas foram caractersticas tpicas da Era Vargas.

385

HISTRIA
Questo 15

UnB

A respeito das mudanas ocorridas no Brasil contemporneo, especialmente dos anos cinqenta aos dias atuais, julgue os itens que se seguem. (0) O Brasil viu transformado seu carter essencialmente agrcola no de um pas tambm urbano, industrial e com crescente participao do setor de servios na economia. (1) Aps a implantao do regime militar, a industrializao aumentou consideravelmente, na mesma proporo em que a reforma agrria, foi implementada. (2) Depois de elevadssimas taxas inflacionrias no final da dcada de oitenta e no incio da de noventa, a economia brasileira vem, nos ltimos dois anos e meio, estabilizando seus nveis de preos. (3) Os indicadores sociais do Brasil sugerem que o processo de modernizao industrial e a urbanizao acelerada geraram uma sociedade mais igualitria do ponto de vista da distribuio da riqueza.

386

HISTRIA

UNESP
F A S E

1. Em troca do reconhecimento de sua independncia por parte da Inglaterra, o Brasil assinou um tratado, em 1826, incluindo clusulas para pr termo a) ao trfico negreiro. b) ao tratado comercial de 1810. c) escravido africana. d) autonomia municipal. e) ao pacto colonial.

2. Foi assim possvel dispor um segundo ataque ao Brasil, desta vez contra uma capitania mal aparelhada
na sua defesa, mas a principal e a mais rica regio produtora de acar do mundo de ento. Existiam a e nas capitanias vizinhas mais de 130 engenhos que, nas melhores safras, davam mais de mil toneladas do produto. (J.A. Gonsalves de Mello.)

O texto refere-se a) Guerra dos Mascates. b) invaso francesa. c) invaso holandesa. d) Revolta de Beckman. e) invaso inglesa.

3. Diferente dos movimentos de libertao da segunda metade do sculo XX, a primeira descolonizao
foi feita por iniciativa dos prprios europeus, ou seja, por colonos que viviam alm-mar e pouco devem aos povos nativos dominados por esses colonos. (Marc Ferro, Histria das Colonizaes.)

Enquadram-se na primeira descolonizao acima referida as independncias a) b) c) d) e) dos EUA e das colnias espanholas. do Brasil e das colnias africanas. do Brasil e do Haiti. do Haiti e de Cuba. das colnias africanas e espanholas.

4. O descontentamento com a desigualdade social crescia em todos os setores populares (...) Uma
situao francamente revolucionria s se criou quando a este descontentamento generalizado se somaram dois fatos novos. Primeiro, uma grave dissenso no patriciado poltico motivada pelo continusmo de Porfrio Dias (...) Segundo, e principalmente, o surgimento de duas lideranas camponesas autnticas: a de Emiliano Zapata (...) e a de Francisco Villa (...) (Darcy Ribeiro, As Amricas e a Civilizao.)

O texto refere-se a) Revoluo Sandinista. b) Revoluo Cubana. c) Guerra do Pacfico.

d) Guerra do Chaco. e) Revoluo Mexicana.

387

HISTRIA
5. Foram caractersticas do Governo Juscelino Kubitschek (1956-1961):

UNESP
F A S E

a) Plano de Metas, apoio da UDN, oposio frontal dos comunistas e abertura ao capital estrangeiro. b) Plano de Metas, desenvolvimento industrial, apoio da aliana PSD-PTB e oposio da UDN. c) Plano de Metas, apoio da aliana PSD-PTB, restrio presena do capital estrangeiro e apoio dos comunistas. d) Plano de Metas, instabilidade poltica, marcante presena do Estado na economia e oposio da aliana PSD-PTB. e) Plano de Metas, apoio dos comunistas, instabilidade poltica e restrio presena do Estado na economia. 6. Leia os seguintes versos.
Sobre a cabea os avies sob os meus ps os caminhes aponta contra os chapadres meu nariz Eu organizo o movimento eu oriento o carnaval eu imagino o monumento no planalto central do pas

Esses so versos iniciais da cano de Caetano Veloso que constituiu a matriz esttica de um movimento musical do final da dcada de 60, de curta durao, em que, segundo estudiosos, a preocupao poltica foi deslocada da rea da revoluo social para o eixo da rebeldia, da interveno localizada, da poltica concebida enquanto problemtica cotidiana, ligada vida, ao corpo, ao desejo, cultura em sentido amplo.
(Helosa Buarque de Holanda e Marcos A. Gonalves, Cultura e participao nos anos 60.)

Esse movimento musical conhecido como a) Bossa Nova. b) Concretismo. c) Primitivismo. d) Antropologia. e) Tropicalismo.

7. A conseqncia mais aparente das invases foi a destruio quase integral da civilizao micnica. No
espao de um sculo, as criaes orgulhosas dos arquitetos aqueus, palcios e cidadelas, no so mais do que runas. Ao mesmo tempo vemos desaparecer a realeza burocrtica, a escrita, que no passava de uma tcnica de administrao, e todas as criaes artsticas... (Pierre Lvque, A aventura grega.)

O texto refere-se s invases a) persas. b) germnicas. c) macednicas. d) drias. e) cretenses.

388

HISTRIA

UNESP
F A S E

8. Quando Maom fixou residncia em Yatrib, teve incio uma fase decisiva na vida do Profeta, em
seu empenho de fazer triunfar a nova religio. A cidade de Yatrib, que doravante seria chamada de Madina al-nabi (Medina, a cidade do Profeta), tornou-se a sede ativa de uma comunidade da qual Maom era o chefe espiritual e temporal. (Robert Mantran, Expanso muulmana.)

Essa mudana para Medina, que assinala o incio da era muulmana, ficou conhecida como a) Xiismo. b) Sunismo. c) Islamismo. d) Hgira. e) Copta.

9. Hoje no vemos em Petrarca seno o grande poeta italiano. Entre os seus contemporneos, pelo
contrrio, o seu principal ttulo de glria estava em que de algum modo ele representava pessoalmente a Antigidade (...) Acontece o mesmo com Boccio (...) Antes do seu Decameron ser conhecido (...) admiravam-no pelas suas compilaes mitogrficas, geogrficas e biogrficas em lngua latina. (Jacob Buckardt, A civilizao da Renascena Italiana.)

Petrarca e Boccio esto intimamente relacionados ao a) nascimento do humanismo. b) declnio da literatura barroca. c) triunfo do protestantismo. d) apogeu da escolstica. e) racionalismo clssico.

10. A monarquia absoluta foi uma forma de monarquia feudal diferente da monarquia dos Estados medievais
que a precedeu; mas a classe dominante permaneceu a mesma, tal como uma repblica, uma monarquia constitucional e uma ditadura fascista podem ser todas [elas] formas de dominao burguesa. (Christopher Hill, Um comentrio, citado por Perry Anderson em Linhagens do Estado absolutista.)

O texto apia a seguinte afirmao: a) b) c) d) e) os Estados medievais precederam a monarquia. a expresso monarquia feudal no aplicvel aos Estados medievais. os Estados medievais podem ser considerados Estados de transio. o absolutismo foi uma forma de dominao feudal. o absolutismo foi politicamente neutro do ponto de vista social.

389

HISTRIA

UNESP
F A S E

11. A crena liberal no equilbrio espontneo do mercado foi reforada em 1803 pela lei de Say. Formulada pelo francs Jean-Baptiste Say, essa lei afirmava que toda oferta cria a sua demanda e inversamente, de tal modo que exclua a possibilidade de crise de superproduo no capitalismo. Qual, dentre os seguintes acontecimentos, constitui a refutao mais importante e direta da lei de Say? a) b) c) d) e) Revoluo Russa de 1917. Crise de 1929. Movimento de independncia da Amrica Latina. Unificao da Alemanha. Ascenso dos Estados Unidos depois da Segunda Grande Guerra.

12. Como terror entende-se (...) um tipo de regime particular, ou melhor, o instrumento de emergncia a
que um Governo recorre para manter-se no poder. (N. Bobbio, Dicionrio de poltica.)

O mencionado instrumento de emergncia o terror foi aplicado em sua forma tpica, na Revoluo Francesa, a) b) c) d) e) durante a reao aristocrtica de 1787 - 1788. por Napoleo Bonaparte, na fase do Diretrio. no perodo da ditadura do Comit de Salvao Pblica. pelos girondinos contra os bonapartistas. por Lus XVI contra os camponeses da Vendia.

1. Constantino, cada vez mais cristo, comeou a favorecer e a enriquecer a Igreja, e a transcrever
em sua legislao os princpios da moral crist. Constncio, mais ariano que ele, perseguiu no somente os pagos (intermitentemente), mas tambm os ortodoxos (...), fazendo jus resposta famosa de ssio de Crdova que, pela primeira vez, recusava ao prncipe o direito de imiscuir-se nos assuntos espirituais (No interfiras nos assuntos da Igreja). O problema cristolgico, suscitado pela questo ariana, continuava a apaixonar e a dividir a opinio, e os conclios multiplicaram-se, sem chegar a qualquer soluo. (Paul Petit, Histria Antiga.)

F A S E

O texto refere-se a dois problemas enfrentados pela Igreja no Baixo Imprio: o cesaropapismo e a heresia. a) Com base no texto, d o significado de cesaropapismo. b) No texto, qual termo refere-se crena hertica que negava a divindade de Cristo?

390

HISTRIA

UNESP
F A S E

2. O yeoman por excelncia era o campons livre, que possua o campo no qual vivia e que explorava
pessoalmente (...) O que caracterizava o yeoman era sua independncia. Devia sobretudo a ela suas robustas qualidades e o papel que desempenhou na histria da antiga Inglaterra. Entre os yeomen foi recrutada, na Idade Mdia, a terrvel infantaria, os espadachins e arqueiros que decidiram as vitrias em Crcy, Poitiers e Azincourt. Mais tarde, convertidos em protestantes e puritanos, foram os mais firmes baluartes da reforma inglesa e combateram nos exrcitos de Fairfax e Cromwell. (Paul Mantoux, A revoluo industrial no sculo XVIII).

De acordo com o texto, os yeomen tiveram atuao significativa em vrios momentos da histria inglesa. As perguntas seguintes esto relacionadas a dois desses momentos. a) A que conflito medieval pertencem as batalhas de Crcy (1346), Poitiers (1356) e Azincourt (1415), nas quais destacaram-se os yeomen? b) O autor menciona os exrcitos de Fairfax e Cromwell. Qual o nome e as principais caractersticas do processo histrico ao qual o autor se refere? 3. ...do sculo XII ao sculo XIV, um certo nmero de cidades da Itlia ou das margens do mar do
Norte conseguiram tornar-se quase independentes: esta situao, rara, foi tambm muito transitria e deve analisar-se mais como fase de crescimento que como situao estvel (...) De fato, no poderia haver integrao econmica completa do sistema feudal: essa integrao supunha um domnio dos negociantes, que era contraditrio com as bases do sistema. (Alain Guerreau, O feudalismo - um horizonte terico.)

Por que razo o domnio dos negociantes estava em contradio com as bases do sistema feudal?

4. O texto refere-se ao lanamento do Plano Marshall.


Trs meses aps o discurso que definia a doutrina Truman, a orao capital pronunciada na Universidade de Harvard pelo Secretrio de Estado norte-americano, o General Marshall, no comeo de julho de 1947, marca uma data importante na evoluo das relaes internacionais, no s das relaes entre os Estados Unidos e a Europa, mas tambm das relaes entre as duas partes do continente europeu. (Ren Rmond, O sculo XX.)

a) D o nome e as caractersticas da nova poltica internacional que ganhou forma com a doutrina Truman. b) No que consistiu o Plano Marshall? 5. Imediatamente aps a sua ascenso ao poder, Lnin disse:
Este governo declara solenemente sua disposio de concluir imediatamente a paz (...) igualmente justa para todas as naes e nacionalidades, sem exceo.

Em maro de 1918 foi assinado o armistcio de Brest Litovsk e os soviticos saram da Primeira Guerra. a) Justifique por que a assinatura do armistcio de Brest Litovsk foi um ato coerente com a poltica externa do governo bolchevista. b) Por que a Alemanha tinha interesse em concluir a paz com os soviticos?

391

HISTRIA

UNESP
F A S E

6. Em uma esquematizao levada ao extrema, pode-se dizer que os primeiros cento e cinqenta
anos da presena espanhola nas Amricas foram marcados por grandes xitos econmicos para a Coroa e para a minoria espanhola que participou diretamente da conquista, e pela destruio de grande parte da populao indgena preexistente... (Celso Furtado, Formao econmica da Amrica Latina.)

a) A que principal atividade ligam-se os grandes xitos econmicos? b) A que se deve a destruio de grande parte da populao indgena preexistente? 7. Para Caio Prado Jr., o sculo XIX no Brasil marca uma etapa decisiva em nossa evoluo e inicia, em todos os terrenos, social, poltico e econmico, uma nova fase. O mesmo autor considera a segunda metade do referido sculo o perodo de maior transformao na histria brasileira. Indique e caracterize, em cada um dos terrenos mencionados (social, poltico e econmico), uma transformao ocorrida, a partir de 1850.

8. Neste mesmo ms, trs mil integralistas a luz da nova era , segundo seu chefe, Plnio Salgado ,
promovem seu Segundo Congresso Nacional... Sob o ridculo das saudaes, da indumentria, dos rituais, havia planos concretos de influir no processo de deciso poltica. Alm dos gestos e dos textos, eles saam s ruas. Aparentemente, a Lei de Segurana Nacional se dirigia aos camisas verdes. Na realidade, o que visava eram as ameaas de mobilizao... como a da Aliana Nacional Libertadora. (Paulo Srgio Pinheiro, Estratgias da Iluso.)

a) A que perodo de nossa histria correspondem os fatos mencionados no texto? b) Caracterize de maneira sumria o movimento chefiado por Plnio Salgado. 9. O regime democrtico em formao no Brasil no avana sem recuos e distores. Amplos segmentos
da realidade econmico-social, poltica, jurdica, educacional, etc. ou no foram atingidos pelos requisitos democrticos de vida ou o foram de modo incompleto, s vezes distorcido. Os prprios fundamentos da democracia os poderes legislativo, judicirio e executivo - no se acham ainda suficientemente estruturados e em funcionamento consentneo com os processos do sistema. Em 1954, 1961 e 1964 todo o sistema poltico sofreu abalos e retrocessos. (Octvio Ianni, Poltica e revoluo social no Brasil.)

Caracterize, de maneira sumria, os abalos e retrocessos a que se refere o texto.

10. No obstante, o processo de industrializao foi imensamente influenciado pela onda de imigrao
europia. Em contraste com a fora de trabalho nativa, os imigrantes haviam sido, amide, habitantes de cidades ou tinham, pelo menos, experincia de trabalho assalariado e eram sensveis aos seus incentivos. (Warren Dean, A industrializao durante a Repblica Velha, in Histria Geral da Civilizao Brasileira. O Brasil Republicano.)

Caracterize, de maneira sumria, a importncia da imigrao europia e do trabalho assalariado para o desenvolvimento industrial do Brasil.

392

INGLS
The idea that laughter promotes good health first received widespread attention in 1979. But centuries earlier astute observers had ascribed physical benefits to humor. Today, scientific research carried out in California show that laughter sharpens most of the instruments in our immune systems tool kit. Perhaps the most obvios effect of laughter is on our mood. After all, laughter is ultimately an expression of emotion joy, surprise, nervousness, amusement. More than a decade of research has begun unraveling the details of the laughtermood connection: stressed-out folks with a strong sense of humor become less depressed and anxious than those whose sense of humor is less well-developed; students who use humor as a coping mechanism were more likely to be in a positive mood; in a study of depressed and suicidal citizens, the patients who recovered were the ones who demonstrated a sense of humor. All of this makes sense in light of laughters numerous physiological effects. After you laugh, you go into a relaxed state, and both your blood pressure and heart rate drop below normal. Laughter also indirectly stimulates endorphins, the brains natural painkillers. In addition to its biological effects, laughter may also improve our mood through social means. Telling a joke increases our sense of belonging and social cohesion. By psychologically connecting us to others, laughter counteracts feelings of alienation, a major fact in depression and suicide. Laughter also helps us think more creatively. Humor loosens up the mental gears. It encourages out-of-the-ordinary ways of looking at things, says John Morreall, Ph.D., president of Humorworks Seminars in Tampa, Florida. Humor guru William Fry, MD, professor emeritus of psychiatry at Stanford University, takes this idea one step further. Creativity and humor are identical, he claims. They both involve bringing together two items which do not have an obvious connection, and creating a relationship. Laughter may not be a panacea, but there is much to be gained from it. And, truth to be told, there is room for plenty of additional happy laughs in our lives. By the time the average kid reaches kindergarten, he or she is laughing some 300 times a day, whereas the typical adult laughs only 17 times a day.

CESGRANRIO

10

15

20

25

30

35

40

393

INGLS
Fortunately, if you are attracted by the idea of using laughter to improve your spirit and health, chances are you have already got a sense of humor. And if you also want to benefit from laughter as a preventive medicine, the humor guru William Fry recommends this two-step process: first, figure out your humor profile. Listen to yourself for a few days and see what makes you laugh out loud. Be honest with yourself. Next, use your comic profile to start building your own humor library: books, magazines, videos. If possible, set aside a portion of your bedroom or office as a humor corner to house your collection. Then, when life gets you down, dont hesitate to visit. Even a few minutes of laughter will be valuable.
Psychology Today. July/August 1996, pp. 33-35 (slightly adapted).

CESGRANRIO

45

50

1. The first paragraph of the text aims to show: a) b) c) d) e) the concluding results of a ten-year research carried out in California. the scientific proof of the details of the humor-health connection. some scientific evidence of health benefits from humor. how our immune systems tool kit can be sharpened. how we can profit from laughter to live longer.

2. ASCRIBED (line 3) means: a) described b) credited c) aspired for d) spread out e) experimented upon

3. Mark the sentence below that must be completed with the same verb tense as HAD ASCRIBED. a) b) c) d) e) (visit) you. If I had the time, I If I go the Post Office, I (mail) this letter for you. If she (see) the movie, she would have told you. He would tell you about it if he (be) there. I could understand him if he (not speak) so quickly.

4. UNRAVELING (line 9) means: a) making clear. b) unrolling c) unsettling d) relating e) turning up

394

INGLS

CESGRANRIO

5. Mark the sentence that can only be completed with WHOSE, the relative pronoun in line 11. a) b) c) d) e) This is Patricia, sister you met last week. One should be loyal to one is married. Shes married to a doctor of you have heard. AIDS, kills thousands of people, hasnt been wiped out. lose their tempers easily. I dont like people

6. LAUGHTER is an uncountable noun. Mark the sentence below that shows the correct usage of countable and uncontable nouns. a) b) c) d) e) Ill have just a bread for dinner. John has fewer money than Paul. Can you give me an advice? I need some informations. My mother is in very good health.

7. According to paragraph 2, mark the physiological effect you DONT experience after you laugh. a) b) c) d) e) You feel at ease Your blood pressure drops. Your heart rate declines. Endorphins are activated. Painkillers stimulate the brain.

8. The main idea of the third paragraph is that: a) b) c) d) e) many biological effects are caused by laughter. laughter can help make you socially adjusted. people who do not laugh at jokes are socially alienated. people who tell jokes are psychologically connected to others. if you laugh enough you will never feel depressed or suicidal.

9. Mark the only item that DOES NOT contain a correct statement about laughter, according to lines 27-35. a) b) c) d) e) Laughter makes us all have more creative thoughts. John Morreall believes that creativity is a synonym for humor. Good-humored people tend to see life from a new perspective. Humor and creativity establish a connection between apparently unrelated items. As far as the mind is concerned, humor makes people feel more relaxed.

395

INGLS
10. It can be inferred the ideas found in Paragraph 6 that: a) b) c) d) e) not all laughs are happy laughs. grown-up people should laugh more. kids that laugh much become unhappy adults. kindergarten children shouldnt laugh so much. the happiest adults are those who behave like kids.

CESGRANRIO

11. The statements below are correct, EXCEPT one. Mark it. a) The phrases truth to be told (line 37) and you have already got (lines 42-43) are in the passive voice. b) The opposite of happy (line 38) is sad. c) Lives (live 38) is the plural of life. d) In the text, average (line 38) and typical (line 40) have the same meaning. e) In line 39, some could be replaced by approximately. 12. The word BOTH appears in lines 19 and 34. Mark the sentence in which it is used in the same sense as in line 19. a) b) c) d) e) Both solutions are good. Both their parents are alive. Some problems affect both men and women. The doctor got angry with both of his patients. Most of them speak either English or German or both.

13. The purpose of following the advice of humor guru William Fry in paragraph 7 is to: a) b) c) d) e) get to know ones humor profile. collect humorous books, magazines and videos. be able to practice laughing out loud and to be honest. avoid future health problems through humor and laughter. have a corner in ones bedroom to house ones humor library.

396

INGLS

CESGRANRIO

14. Check the items that DO NOT present a reason for visiting your humor corner, as mentioned in lines 49-51. III III IV VWhen you feel discouraged. When you are in a dilemma. Everytime your spirits are low. If you feel inclined to be dishonest. Whenever life makes you unhappy.

The correct items are: a) II and IV only. b) III and V only. c) I, III and V only. d) I, II, III and IV only. e) I, II, IV and V only.

15. The item that presents, respectively, a synonym for BUT and the opposite of MUCH in but there is much to be gained (lines 36 - 37) is: a) nevertheless - anything. b) even though - a few. c) besides - a little. d) however - few. e) yet - little.

16. Find the title that summarizes the ideas presented in the text. a) b) c) d) e) How to succeed in life being funny Laughter: the cure for all illnesses The secret of hapiness is a healthy humor Laughter may help you become happier and healthier Researches advertise the biological effects of laughter

397

INGLS
TEXTO II

CESGRANRIO

17. Find the correct prepositions and adverbs that complete the following sentences with LOOK. III III IV The duty of the government is to look the citizens interests. I look to seeing you in Brasil. He consulted his dictionary to look the meaning of a word. Many people are looking work in the food industry. II up about over forward up to III for into after up over IV for at to for after

a) b) c) d) e)

I to at forward after about

18. The older man told his friend that: a) b) c) d) e) since he met Cora he rarely dates other women. he enjoys looking at the attractive woman in the picture. he used to admire beautiful women, but not very often. he never glanced at any attractive women since he got married. he grew increasingly interested in pretty women as the years went by.

19. Mark the sentence in wich AS is used with the same meaning as in the cartoon. a) b) c) d) e) Do as she tells you! You like chocolate as much as me. As he couldnt wait for you, he left. She cried sadly as she told her story. He works as a consultant for a foreign firm.

20. You probably laughed at the cartoon because of KNOCKED ME OUT, which appears twice, meaning, respectively: a) attracted me deeply; hit me. b) impressed me; surprised me. c) treated me roughly; struck me. d) drove me crazy; shocked me. e) blew me; criticized me.

398

INGLS
Miss Emlyn read us some of it. I asked Mummy to read some more. I liked it. It has a wonderful sound. A brave new world. There isnt anything really like that, is there? You dont believe in it? Do you? There is always a brave new world, said Poirot, but only, you know, for very special people. The lucky ones. The ones who carry the making of that world within themselves.

FUVEST
F A S E

(Agatha Christie, Halloween Party, pp. 85-86)

1. The text above is a dialogue between a) Mummy and Poirot. b) a mother and her child. c) Miss Emlyn and Poirot. 2. According to Poirot, a) b) c) d) e) brave new worlds exist within everyone. lucky people can create a brave new world of their own. it is extremely easy to carry a world within oneself. a brave new world is a social concept. it brings lucky to make and carry a world within yourself. d) a child and Poirot e) Poirot and his mother

3. We can conclude from the text above that a) b) c) d) e) Poirot is talking to a person who enjoyed reading A brave new world. someone who absolutely disbelieves in a brave new world is talking to Poirot. Mummy and Miss Emlyn liked A brave new world because of its wonderful sound. anyone who has read A brave new world can be considered lucky. Poirots interlocutor listened to passages from A brave new world read by two different people.

4. Choose another way of saying There isnt anything really like that. a) b) c) d) e) There is nothing really like that. There arent many things really like that. There arent no things really like that. There is anything hardly really like that. There are a few things really like that.

399

INGLS

FUVEST
F A S E

And if I ever catch you downloading dirty pictures from the Internet again, young man, Ill wash your mouse out with soap ! (Newsweek, April 10, 1995)

Considere a imagem e o texto acima. 5. The boys mother is expressing a) a promise. b) a threat. c) a challenge. d) an order. e) a request.

6. A me est aborrecida porque a) o menino est viciado em Internet. b) as imagens na tela do computador esto sujas, borradas. c) o menino est vendo figuras indecentes no computador. d) o menino jovem demais para lidar com o computador. e) o menino no quer desligar o computador para ir tomar banho.

7. Qual seria o correspondente, no passado, de if I ever catch Ill wash ? a) If I ever were to catch Ill wash. b) If I ever caught Id wash. c) If I ever would catch I washed. d) If I ever caught Id have washed. e) If I had ever caught I would wash.

400

INGLS
A pretty scary read, War of the Worlds by Mark Slouka (Little, Brown, 9.99, ISBN 0 349 10785 8) can make your hair stand on end. It is nothing less than an alarm call that there is a danger of whole societies, computer-drunk, swapping living in the uncomfortable real world for that of life in virtual reality or cyberspace. The latter, of course, is controllable, full of pleasure and free from moral constraints. Already there is a term for those who find such an idea horrifying. It is PONA, a Person Of No Account. The thesis is plausible. The price, for a slim paperback, is not.

FUVEST
F A S E

(New Scientists, 2 March 1996)

8. Segundo o texto, War of the Worlds pode a) inquietar o leitor. b) deleitar o leitor. c) intrigar o leitor. d) aterrorizar o leitor. e) empolgar o leitor.

9. Em War of the Worlds, o autor a) alerta para o perigo de uma guerra que utilize recursos da ciberntica. b) prova que no h nada mais alarmante do que sociedades inteiras inebriadas pela realidade virtual. c) adverte sobre o risco de sociedades inteiras adotarem a realidade virtual em substituio vida real. d) faz a apologia de uma sociedade totalmente embriagada pela ciberntica. e) sugere que no h nada mais desconfortvel do que a vida real.

10. Segundo o texto, a vida na realidade virtual a) ideal para as sociedades livres. b) no est isenta de restries de cunho moral. c) livre para qualquer prazer. d) proporciona muitos prazeres. e) est sujeita a alguns regulamentos.

11. O texto afirma que o livro de Mark Slouka a) prope uma tese admissvel, em poucas pginas. b) bonito, mas assusta. c) vendido em edio de capa dura. d) apresenta uma tese plausvel, a um preo justo. e) agrada aos leitores em busca de idias horripilantes.

401

INGLS
Why do bees fuss about so much when they fly, instead of forming a tidy flock like birds? Birds flying in a flock keep to a highly ordered pattern, whereas a swarm of bees is a cloud of chaos. This difference has long puzzled scientists, but now a team of Japanese researchers has come up with a simple mathematical model to explain it. [The researchers] began with a simple analogy. Stars in a galaxy move under the influence of each others gravity in a way that can be described by Newtons laws. Identify the influences felt by an insect or bird, the researchers reasoned, and its flying patterns should be just as easy to predict.

FUVEST
F A S E

(Adapted from New Scientist, 15 June 1996).

De acordo com o texto acima 12. a)ao voarem em grupo, as abelhas no se posicionam de forma ordenada. b) os cientista acreditam que as abelhas no gostam de voar em grupo. c) as aves agitam-se tanto quanto as abelhas durante o vo. d) ao aproximar-se de um bando de aves, um enxame de abelhas transforma-se em uma nuvem de caos. e) quanto maior o bando de aves, mais ordenado seu vo.

13. a)as leis de Newton descrevem algumas influncias sentidas por um inseto ou ave durante o vo. b) impossvel prever se o vo de um grupo de insetos ou de aves ser ordenado ou catico. c) mais fcil prever a posio dos insetos ou aves durante o vo do que a das estrelas em uma galxia. d) algumas influncias sentidas por um inseto ou ave, durante o vo, so anlogas s experimentadas pelas estrelas em uma galxia. e) conhecidas as influncias que afetam cada ave ou inseto, a organizao de vo do grupo torna-se previsvel.

14. A forma correta do singular de Why do bees fuss about so much when they fly? : a) Why does bee fuss about so much when it fly? b) Why do an bee fusses about so much when it flies? c) Why does a bee fuss about so much when it flies? d) Why does the bee fuss about so much when it fly? e) Why does a bee fusses about so much when it flies?

402

INGLS
QUESTES DE 1 A 7

UFBA
F A S E

INSTRUO: Assinale as proposies verdadeiras, some os nmeros a elas associados e marque o resultado na Folha de Respostas. QUESTES DE 1 A 4 TEXTO:
THE PURSUIT OF HAPPINESS David G. Myers and Ed Diener Compared with misery, happiness was given little attention by social scientists until 1984. However, this trend has undergone timely changes since then. During the past two decades, dozens of investigators throughout the world have asked thousands of people to reflect on their happiness and satisfaction with life and some surprising 5 findings have been uncovered. People are happier than we generally believe them to be, and happiness does not depend significantly on external circumstances. Although viewing life as a tragedy has a long and honorable history, the responses of random samples paint a much rosier picture. How can social scientists measure something as hard to pin down as happiness? 10 Most researchers simply ask people to report their feelings of happiness or unhappiness and to assess how satisfying their lives are. The daily mood ratings of those who say they are happy and satisfied seem to reveal more positive emotions, and they smile more than those who call themselves unhappy. Unlike the depressed, happy people are less self-focused, less hostile and abusive, and less susceptible to 15 disease. Interviews with people of all ages reveal that no time of life is notably happier or unhappier. Similarly, men and women are equally likely to declare themselves very happy and satisfied with life. Even knowing someones ethnicity gives little clue to subjective well-being. Scientists assert that people in disadvantaged groups maintain 20 their self-esteem by valuing things at which they excel, by making comparisons within their own groups and by blaming problems on external sources such as prejudice. Wealth is also a poor predictor of happiness. People have not become happier over time as their cultures have become more affluent. Even very rich people are only slightly happier than the average American. Indeed, in most nations the correlation 25 between income and happiness is negligible except in the poorest countries, for instance, Bangladesh and India, where income is a good measure of emotional well-being. Furthermore, factors as diverse as civil rights, literacy and duration of democratic government, all of which also promote reported life satisfaction, tend to go hand in hand with national wealth. As a result, it is impossible to tell whether the 30 happiness of people in wealthier nations is based on money or is a by-product of other felicities.

403

INGLS
Although happiness is not easy to predict from material circumstances, it seems consistent for those who have it. In study after study, four traits characterize happy people. First, especially in individualistic Western cultures, they like themselves. They 35 have high self-esteem and usually believe themselves to be more ethical, more intelligent, less prejudiced, better able to get along with others, and healthier than the average person. Second, happy people typically feel personal control. Those with little or no control over their lives for inatance, prisoners, nursing home patients, severely impoverished groups or individuals, and citizens of totalitarian regimes suffer lower 40 morale and worse health. Third, happy people are usually optimistic. Fourth, most happy people are extroverted. Although one might expect that introverts would live more happily in the serenity of their lives, extroverts are happier whether alone or with others. Students of happiness are now beginning to examine happy peoples exercise 45 patterns, worldviews and goals. It is possible that some of the patterns discovered in the research may offer clues for transforming circumstances and behaviors that work against well-being into ones that promote it. Ultimately, then, the scientific study of happiness could help us understand how to build a world that enhances human well-being and to aid people in getting the most satisfaction from their circumstances. SCIENTIFIC AMERICAN, New York. n.5. May 1996, p. 70-2. (adapted text)

UFBA
F A S E

Questo 1 uma idia presente no texto: (1) A partir de 1984, os cientistas sociais comearam a dar maior ateno a pesquisas que pretendem avaliar o grau de satisfao das pessoas. (2) Segundo estudos j realizados, as pessoas, no mundo moderno, aparentam ser mais infelizes do que realmente so. (4) O grau de felicidade das pessoas est diretamente relacionado s circunstncias externas que determinam o seu status e o seu modo de vida. (8) Idade, sexo e poder aquisitivo so fatores que determinam o grau de felicidade da maioria das pessoas. (16) A privao da liberdade e a falta de controle sobre sua prpria vida so fatores que influenciam o moral, a sade e a felicidade das pessoas. (32) Pessoas de grupos tnicos discriminados revelam maior grau de infelicidade, por serem socialmente compelidas a baixar a sua auto-estima. (64) As caractersticas mais comuns e consistentes das pessoas felizes so: auto-estima, independncia, otimismo e extroverso.

404

INGLS
Questo 2 A respeito das pesquisas realizadas sobre a felicidade, pode-se afirmar:

UFBA
F A S E

(1) Apesar de todo o esforo dos cientistas sociais, tem sido difcil depreender das pesquisas uma definio para algo to complexo como o estado de felicidade. (2) Na cultura ocidental, as pesquisas sobre felicidade chegam a resultados insatisfatrios, porque as pessoas esto condicionadas a simular bem-estar e felicidade. (4) Pesquisas realizadas em naes desenvolvidas so conclusivas quanto a ser o grau de felicidade, nessas naes, resultante da riqueza e de outros fatores a ela associados. (8) Descobertas sobre o que faz as pessoas felizes ou infelizes podem ajudar na transformao das circunstncias e comportamentos responsveis por sua infelicidade. (16) Nos ltimos tempos, os cientistas sociais vm tentando descobrir se a prtica de atividade fsica, as vises de mundo e os objetivos individuais tm influncia na felicidade das pessoas. (32) A despeito dos avanos alcanados pela cincia, pouco provvel que se consiga transformar as circunstncias adversas do mundo atual, responsveis pela infelicidade das pessoas.

Questo 3 H oposio de sentido entre: (1) misery (li. 1) (2) then (li. 2) (4) happier (li. 5) (8) hard (li. 9) (16) disadvantaged (li. 19) (32) democratic (li. 28) (64) First (li. 34) Wealth (li. 22) now (li. 44) healthier (li. 36) easy (li. 32) prejudiced (li. 36) totalitarian (li. 39) Ultimately (li. 47)

Questo 4 H expresses com idntica funo gramatical, em: (1) timely (li. 2) (2) viewing (li. 7) (4) ratings (li. 11) (8) Interviews (li. l6) (16) income (li. 25) (32) life (li. 28) simply (li. 10) average (li. 24) ethnicity (li. 18) prejudiced (li. 36) rights (li. 27) impoverished (li. 39)

405

INGLS
QUESTES DE 5 A 7 TEXTO:

UFBA
F A S E

ARE YOU READY FOR VIRTUAL LOVE? Avodah Offit, M. D.

10

15

20

25

30

35

40

45

50

Jane sits at her computer in Chicago and communicates with Peter in New York. They make contact two or three times a day and tell each other everything. They share a love that travels in the virtual reality between their computers, a virtual love. Their degree of intimacy would not be unusual if they were old college friends or longtime colleagues, but the two of them have never looked into each others eyes. Not only have they never met, but they see no reason to. They give one another all the love and support they need strictly on E-mail. From friendship to dating, from courtship to marriage, some peoples lives channel almost entirely through computers. Much may be said in praise of E-mail as Cupid. But how do psychiatrists view this phenomenon? Is it psychologically healthy or some disease of technology? How is it that people can fall in love via machine, often without ever seeing each other? What are the benefits, and what are the hazards to reckon with? Should we take precautions before jumping into cyberspace? Certainly, romance seems to flower for thousands of strangers who are finding their mental mates on E-mail. Psychiatrists think these attempts at sensible affiliation are brave and adventurous. E-mail companionship can be a new kind of spectral friendship, even a spiritual companionship, unlike any in the accustomed world. However, even virtual love affairs can run into real trouble when the communicators turn out to have little or no physical compatibility. Although peoples minds are perfectly in tune, their corporal selves can be disastrously mismatched. The investment of extensive emotional commitment in a figure of ones imagination also becomes a threat in a variety of other ways peculiar to E-mail. Perhaps it most obviously invites people to flirt, allowing a wide selection of lovers, people with whom one is more intimate than simply friendly. There are no rules or contracts about fidelity: you can have as many virtual lovers as you want, with no particular ethical consequences. On the other hand, the E-mail devotee may become addicted to the thrill of many affairs or suffer unrequited love. Invasion of privacy, jealousy, harassment and victimization, obscenity and sexual pathology are all out there in cybesrpace as they are elsewhere. Of course, beyond the main emotional problem inherent in cyberspace (its tendency to enhance feelings of love) lie the quicksands created by a new breed of jokers, charlatans, and even deviants. These people may assume multiple identities and present themselves as different from what they are, pretending to be anything from a tender person to a wild, experienced man or woman. The schemers may involve innocents, even children, in their charades, which can lead to unfortunate meetings in real time and place. And its the more vulnerable among us, those more likely to operate from the safety of their home than to go out into the world, who are affected most. A person presenting an emotional problem or desire for communication may be harassed by individuals who sit at their computers all day looking for victims to torment. While there are many wonderful people inspired by love and world community in cyberspace, the numbers of the perverse are growing, and you should therefore enter the public groups with caution. For instance, a story appeared recently in the press about a man who lured a child on-line to an off-line assignation. Given that children are often more computer-literate than their parents, families would do well to monitor their kids cyberspace habits with as much interest as they would give their childrens whereabouts in other situations. As a psychiatrist, Im also aware of the capacity of E-mail to feed into the desire to escape reality and live in the world of fantasy. But to view that only as potential danger is to fall in with the moralists who have a long tradition of warning the world against the perils of reading, thinking, acting in new ways. Just as the world has come to terms with other pastimes once looked upon as subversive, we will all come to terms, eventually, with the problems and pleasures of cyberspace. COSMOPOLITAN. New York, v. 218, n.1, Jan. 1995. (adapted text)

406

INGLS
Questo 5 uma idia presente no texto:

UFBA
F A S E

(1) Em alguns espaos virtuais, possvel uma pessoa assumir tantas identidades ou personalidades quantas desejar. (2) Est comprovado que as pessoas aventureiras, sem escrpulos e de conduta sexual patolgica so as que se dispem a estabelecer relacionamentos amorosos via computador. (4) H usurios do computador cuja inteno principal atormentar e causar malefcios s suas vtimas. (8) Mesmo as pessoas bem intencionadas podem passar a manifestar um comportamento patolgico, devido falta de compromisso, peculiar ao espao ciberntico. (16) A comunicao via computador favorece um alto grau de intimidade e liberdade entre pessoas estranhas. (32) Atualmente, j existem mecanismos, no mundo virtual, capazes de proteger as crianas de adultos mal-intencionados e inescrupulosos. (64) H pessoas que consideram desnecessrio o encontro pessoal com o seu parceiro amoroso do mundo virtual.

Questo 6 Ao posicionar-se sobre o amor virtual, o autor: (1) inteiramente favorvel ao uso do computador no relacionamento amoroso das pessoas e no v perigo nisso. (2) tem conscincia de que a comunicao via computador pode alimentar fantasias danosas e fuga do mundo real. (4) pronuncia-se contra o uso do computador como veculo do amor virtual, pelos grandes perigos que isso representa. (8) considera um direito das pessoas usar o computador livremente, para qualquer finalidade que lhes proporcione satisfao. (16) teme o perigo potencial de um moralismo que passe a censurar arbitrariamente tudo que novo e diferente. (32) acredita que saberemos chegar a um ponto de equilbrio entre os prazeres e os perigos do ciberespao.

Questo 7 H correspondncia entre a expresso citada e a idia por ela sugerida, em: (1) ...would not be unusual if... (li. 4) (2) Should we take precautions...?(li. 12) (4) ...can be a new kind... (li. 16) (8) ...may become addicted to... (li. 26) (16) ...will all come... (li. 51) improbabilidade obrigao possibilidade permisso promessa

407

INGLS
QUESTES DE 1 A 10

UFBA
F A S E

INSTRUES: Assinale as proposies verdadeiras, some os nmeros a elas associados e marque o resultado na Folha de Respostas. QUESTES DE 1 A 4 TEXTO:
What explains the current fascination with the novels of Jane Austen? Her 19thcentury dramas of British gentility offer no violence or explicit sex, the action is limited to raised eyebrows or the glimpse of a heaving breast. There is romance, as well as cleverness and some nice scenery. Still, a deeper curiosity must have caused 3.7 5 million American households to tune in to the six-hour television adaptation of Pride and Prejudice recently, and many millions more to have flocked to screen versions of Persuasion and Sense and Sensibility. Maybe it is because we are drawn to a long-ago world in which people did not pressure intimacy, a culture in which emotions were expressed with a degree of 10 reticence, a society in which people minded their manners. A society, in short, quite unlike our own. Most people today are accustomed to a culture that is coarser, more flagrant, more intrusive, more rude more in your face. People have become accustomed to vulgarity, as well as other common indignities their neighbors threatening to sue them, their colleagues whining, their 15 former lovers spilling their secrets, and perfect strangers insulting them, or worse, confiding in them. Possibly, they are growing sick of it. The cult of Jane Austen, with its nostalgia for a more decorous and polite age, is one small sign. Manners can, of course, be a thinly disguised tool to keep the lower orders in place. In England, modes of dress and speech long bespoke ones class. Jane 20 Austens England was stuffy and hierarchal; she used irony to poke fun at its snobbishness. Now we seek to show respect for every culture. Political correctness does not seem to have conferred dignity so much as an air of permanent grievance and a got mine sense of entitlement. Manners have been replaced by mau-mauing. By getting 25 rid of manners we have not abolished class distinction. The human impulse to sort and sift and feel superior is too strong, and it is hardly restricted to what used to be called the upper crust. Actually, members of the upper class were often slobs: they could afford to be indifferent to appearance. It was the middle class that self-consciously strove to demonstrate their status to show that they were not working class. 30 Is there a way to have the manners celebrated by Jane Austen without the snobbery she satirized? The real lesson is Austen is not how to look but how to act with deference and respect. It is no doubt unrealistic to think that people will have a true change of heart and become suddenly more self-restrained and considerate. But with a conscious effort, appearances can change. We may be selfish animals at heart, but we 35 can at least pretend. A little hypocrisy is not always a bad thing. NEWSWEEK. New York, Feb. 12, 1996, p. 50. (adapted text)

2 D

P R O V A

408

INGLS
Questo 1 Information which can be found in the text is in:

UFBA
F A S E

(1) Jane Austen was an English writer whose novels have attracted millions of peoples attention lately. (2) To match the taste of modern audiences, adaptations of Austens works portray too much violence and sex. (4) Jane Austens novels are about ancient peoples that lived with little emotion and without privacy. (8) Jane Austen wrote about a society which was much alike the contemporary one. (16) In a style both mordacious and nostalgic, Jane Austen despised affectation and exalted the past. (32) Austens characters belonged to the nobility and behaved in a politically correct manner, respecting the cultures of other classes. (64) Jane Austen seemed to abhor the bourgeoisies ostentatious and uncultivated manners in society. Questo 2 Synonymy occurs in: (1) (2) (4) (8) gentility (li. 2) pressure (li. 9) bespoke (li. 19) sort (li. 25) kindness push revealed mix (16) Actually (li. 27) (32) celebrated (li. 30) (64) pretend (li. 35) Indeed praised mean

2 D

P R O V A

Questo 3 A correct reference to the verbal tense or aspect found in the except is in: (1) ...people did not pressure intimacy,... (li. 8-9) An action which began and finished in the past. (2) ...they are growing sick of it. (li. 16) An action supposed to take place in the future. (4) ...we have not abolished class distinction. (li. 25) A past action whose results can still be noticed in the present. (8) ...people will have a true change of heart... (li. 32-3) A promise not yet fulfilled. (16) ...hypocrisy is not always a bad thing... (li. 35) A hypothetical present event. Questo 4 A suitable title for the passage could be: (1) (2) (4) (8) (16) (32) (64) Ode to British Nobility British Etiquette: Dos and Donts Social Stratification in Contemporary Britain Jane Austens Criticism of Hypocrisy Being Polite without Snobbery Praise of Hypocrisy A Plea for Good Manners

409

INGLS
QUESTES DE 5 A 8 TEXTO:
Mr. [Henry] Dashwoods disappointment was, at first, severe; but his temper was cheerful and sanguine, and he might reasonably hope to live many years, (...). But the fortune, which had been so tardy in coming, was his only one twelvemonth. He survived his uncle no longer; and ten thousand pounds, including the late legacies, was all that remained for his widow and daughters. His son was sent for, as soon as his danger was known, and to him Mr. Dashwood recommended, with all the strenght and urgency which illness could command, the interest of his mother-in-law and sisters. Mr. John Dashwood had not the strong feelings of the rest of the family; but he was affected by a recommendation of such a nature at such a time, and he promised to do every thing in his power to make them comfortable.

UFBA
F A S E

2 D

10

P R O V A

15

20

25

30

35

40

No sooner was his fathers funeral over, than Mrs. John Dashwood, without sending any notice of her intention to her mother-in-law, arrived with her child and their attendants. No one could dispute her right to come; the house was her husbands from the moment of his fathers decease; but the indelicacy of her conduct was so much the greater, and to a woman in Mrs. Dashwoods situation, with only common feelings, must have been highly unpleasing; but in her mind there was a sense of honour so keen, a generosity so romantic, that any offence of the kind, by whomsoever given or received, was to her a source of immoveable disgust. (...) So acutely did Mrs. Dashwood feel this ungracious behaviour, and so earnestly did she despise her daughter-in-law for it, that, on the arrival of the later, she would have quitted the house for ever, had not the entreaty of her eldest girl induced her first to reflect on the propriety of going, and her own tender love for all her three children determined her afterwards to stay, and for their sakes avoid a breach with their brother. Elinor, this eldest daughter whose advice was so effectual, possessed a strength of understanding, and coolness of judgement which qualified her, though only nineteen, to be the counsellor of her mother, and enabled her frequently to counteract, to the advantage of them all, the eagerness of mind in Mrs. Dashwood which must generally have led to imprudence. She had an excellent heart; her disposition was affectionate, and her feelings were strong; but she knew how to govern them: it was a knowledge which her mother had yet to learn, and which one of her sisters had resolved never to be taught. Mariannes abilities were, in many respects, quite equal to Elinors. She was sensible and clever; but eager in everything; her sorrows, her joys, could have no moderation. She was generous, amiable, interesting: she was everything but prudent. The resemblance between her and her mother was strikingly great. Elinor saw, with concern, the excess of her sisters sensibility; but by Mrs. Dashwood it was valued and cherished. They encouraged each other now in the violence of their affliction. The agony of grief which overpowered them at first, was voluntarily renewed, was sought for, was created again and again. They gave themselves up wholly to their sorrow, seeking increase of wretchedness in every reflection that could afford it, and resolved against ever admitting consolation in future.

AUSTEN, Jane. Sense and Sensibility. London: Pan Books, 1972. p. 18-20.

410

INGLS
Questo 5 The reading of the text allows one to infer that: (1) (2) (4) (8) (16) (32) (64)

UFBA
F A S E

Mr. Henry Dashwood lamented his uncles death but soon recovered. Mr. Henry Dashwood must have died without any concern about his familys future. Mr. John Dashwood inherited all his fathers properties and money. Mrs. John Dashwood seemed to be rather sympathetic with her husbands relatives misfortune. Mrs. Henry Dashwoods relationship with her stepsons wife was smooth. Marianne Dashwoods personality was very similar to her mothers. Elinor Dashwood worried about Mariannes ever-growing anxiety.

2 D

P R O V A

Questo 6 The idea expressed by the cited verb phrase is correctly indicated in: (1) (2) (4) (8) (16) ... he might reasonably hope to live... (li. 2) ...which illness could command... (li. 7) ...No one could dispute... (li. 14) ...must have been highly unpleasing;... (li. 17) ...which must generally have led to imprudence.( li. 28-9) (32) ...could have no moderation. (li. 34-5) Questo 7 There is correspondence between the quoted pronoun and who it refers to, in: (1) (2) (4) (8) (16) (32) (64) his (li. 8) his (li. 12) their (li. 13) her (li. 17) her (li. 24) them (li. 28) They (li. 38) Mr. John Dashwood Mr. Henry Dashwood Mrs. John Dashwood and her child Mrs. John Dashwood Mrs. Henry Dashwood Mrs. Henry Dashwood and her daughters Mrs. Henry Dashwood and Elinor possibility probability prohibition obligation

deduction incapability

Questo 8 According to the context, the class of the quoted word is correctly indicated in: (1) (2) (4) (8) (16) (32) (64) affected (li. 10) dispute (li. 14) kind (li. 18) whomsoever (li. 18) first (li. 22) understanding (li. 26) but (li. 35) verb noun adjective pronoun adverb verb conjunction

411

INGLS
QUESTES 9 E 10 TEXTO:
Austen draws on a respectable tradition in the female-authored novel of fictional contrasts between sisters, often quite explicitly in her choice of names for her heroines. Austens particular treatment of this counterpointing of two sisters places her firmly in the context of a conservative moralism which asserts that objective evidence should be preferred to private intuition. This moral conclusion is, however, reached through a discussion of the exposure of a specific group gentry women to the ideals and aesthetics of a particular and problematic quality, sensibility. Sensibility is best understood less as an antonym of sense than as a variant upon it. Samuel Johnsons Dictionary of the English Language definition of 1755 is quickness of sensation or perception. Sense is, by contrast, the faculty or power by which external objects are perceived. If sensibility first denotes a quality found in individual behaviour, later in the century it acquires the further connotation of a form of aesthetic response to external objects. The idea of sensibility refines an earlier idea of sentiment, the engendering of a sympathetic response to the suffering of others that is (at least theoretically) inclined to lead to ameliorative social action. Whereas the midcentury novel of sentiment attempts to stimulate the readers sympathy for a virtuous and persecuted protagonist, the novel of sensibility offers a detailed study of the sympathetic motions of feeling on the part of a central character in response to the narrative of suffering he or she observes. In other words, the traditional position produced for the reader to occupy outside the text in the novel of sentiment is occupied, or rather co-opted, by a character within the text in the novel of sensibility. This, in turn, enables the reader to take a critical perspective on the hero or heroine of sensibility, seen here as self-gratifying corruption of the valued social response and collective responsability that sentiment engenders. Perception of external objects becomes a wholly aesthetic indulgence. Heroes and heroines of sensibility prefer their cottages ruined, their fields suffocated by dead leaves, their landscapes free of human life, so that they can focus on the complexities and rhythms of their own experience of perception. BALLASTER, Ros. [Introduction]. In: AUSTEN, Jane. Sense and sensibility. London: Penguin Books, 1995. p. xiii - xiv. (adapted text)

UFBA
F A S E

2 D

P R O V A

10

15

20

25

Questo 9 According to the text, authors of novels of sensibility: (1) (2) (4) (8) (16) (32) (64) were predominantly romantic and conservative women. usually wrote about moralistic female figures. created personages who were over-sensitive and pathetic. led main characters to scrutinize their own perceptions. invited readers to deal with protagonists more closely. described heroes and heroines who enjoyed living in isolation. drove their characters closer to nature to make them aware of their misery.

412

INGLS
Questo 10 The affix in the word on the left bears the meaning on the right in: (1) (2) (4) (8) (16) (32) (64) respectable (li. 1) explicitly (li. 2) counterpointing (li. 3) moralism (li. 4) quickness (li. 10) virtuous (li. 16) self-gratifying (li. 23) action manner opposition capability comparison quantity reflexivity

UFBA
F A S E

2 D

P R O V A

QUESTO DISCURSIVA Based on the analysis of the terms sense ans sensibility, found in the text on page 9, write a short composition, in Portuguese, associating these attributes to characters described in the text on page 6.

413

INGLS

UFMG
F A S E

INSTRUCTIONS: Read the texts carefully and then choose the alternative which best completes the following statements. TEXT 01 QUESTIONS 1 TO 7 HO HO HO!
According to Personnel Management magazine, laughter can help you stay healthy. Medical research has revealed that it helps strengthen the bodys immune system, raising resistance to germs. (It appears that lighthearted laughter is best. defiant or self-deprecating humour doesnt have the same beneficial effect.) Employers are advised that, by encouraging more mirth in the workplace, they can save themselves a fortune in sick pay. So next time the boss catches you sharing a joke by the photocopier, you can tell her its all for the good of the company. Honest. Cosmopolitan, December 1990, p. 159 (compiled by Tania Unsworth)

1 A

P R O V A

Glossary: mirth = laughter, amusement 1. The item says that medical research has shown that laughter a) b) c) d) decreases peoples work rate. increases peoples immunity. kills germs in peoples bodies. makes people more defiant.

2. The research suggests that laughter helps you a) avoid sickness. b) become rich. c) keep your job. d) work harder.

3. Making fun of someone or of yourself, apparently, a) b) c) d) has helped medical research a gret deal. is less effective than other kinds of humour. strengthens peoples immune system best. tends to be beneficial at killing germs.

4. According to the item, employers should a) b) c) d) advise their workers about germs. encourage workers to laugh more. pay workers more if they are sick. tell workers jokes about the company.

414

INGLS
5. According to the text, in workplaces where there is more fun a) employers are rich people. b) people save more time. c) the boss shares good jokes. d) there are fewer sick people.

UFMG
F A S E

1 A

6. Another good title for this piece of news could be a) Laugh less and stay healthier. b) Laughter is the best medicine. c) Never laugh near your boss. d) Make money and laugh. P R O V A

7. The word her, in ...you can tell her its all for the good of the company, refers to the a) boss b) company c) joke d) photocopier

TEXT 02 QUESTIONS 2 TO 16
FTER AN EXHAUSTING 12-hour drive to our honeymoon destination in Daytona Beach, Fla., my husband and I decided to refresh ourselves with a dip in the motel pool. I must have dropped a few pounds to pre-wedding jitters, because each time I dived into the pool, I lost either the top or the bottom of my skimpy new bikini. We had the pool for ourselves, so we just laughed and retrieved the pieces. Later we dressed for dinner and went down to the motel restaurant. Waiting for a table, we sat in the lounge and ordered drinks. Above the bar was a huge, empty, glistening fish tank. Curious, my husband asked, Why is such a beautiful fish tank empty? The bartender grinned from ear to ear as he replied, Thats not a fish tank. Its the swimming pool. Tara Kelly Waworth (Geneva)

Glossary: jitters = feeling of nervousness skimpy = too small to grin = to smile broadly

8. The narrator and her husband went to Daytona Beach soon after a) their pre-wedding contract. b) their wedding anniversary. 9. The couple went to the motel to a) b) c) d) celebrate their wedding anniversary. enjoy their first days of married life. lose weight before getting married. pass the day in the swimming pool. c) their wedding ceremony. d) their wedding proposal.

415

INGLS
10. The couple went swimming because a) the tank had no fish in. b) there was nobody there. c) they were hot and tired. d) they were without clothes.

UFMG
F A S E

1 A

11. While the couple were using the swimming pool, a) b) c) d) the man asked the woman to marry him. the waiter warned them about the fish. they kept in touch with the other people. they thought they were all by themselves. P R O V A

12. The woman felt that her bikini was loose because she had a) bought a skimpy one. b) dropped her clothes. c) probably lost weight. d) spent a few pounds.

13. The only statement which is probably false is a) Nobody saw them in the pool. b) The couple had fun in the pool. c) They went to the motel by car. d) They were in the pool alone.

14. The bartender was probably smiling because a) b) c) d) he had found the womans lost bikini. hed seen them swimming in the pool. he was trying to be friendly to them. there were not any fish in the tank.

15. When the narrator and her husband were in the bar, they realized that a) b) c) d) nobody usually swam in that swimming pool. the bartender was also in the swimming pool. the swimming pool cool be seen from the bar. the swimming pool was in fact a tank for fish.

16. The couple were surprised later because they a) b) c) d) had wanted fish for dinner at the motel. realized people had seen them swimming. spoke to the waiter about the fish tank. thought the fish tank was a swimming pool.

416

INGLS

UFMG
F A S E

INSTRUCTIONS: Read the texts carefully and then choose the alternative which best completes the following statements. TEXT 1 QUESTIONS 1 TO 9

1 B

Reality Check
Big wheels

Spielbergs List Price

P R O V A

TS PROBABLY not too wise to lob an anti-Semitic crack at Steven Spielberg. Shortly after Spielberg bought a car for a friend at Santa Monica car dealership, a salesman bragged,

I sold a car to a Jew for full price. Another customer overheard the comment and called Spielbergs Amblin Entertainment, and he immediately canceled the order. The car dealer profusely apologized to Spielberg and asked if there was anything he could do to make it right. Spielberg, however would have nothing to do with it, says a source, and bought another car. But did he get it wholesale? Esquire, June, 1994. p. 30.

Glossary: to lob = to throw crack = a humorous comment 1. The car Spielberg bought was for a) a Jewish man. b) another person. c) his company. d) his customer.

2. The salesman in the story was proud of himself because he a) b) c) d) had sold a full-priced car to a Jew. heard a funny joke about Jewish people. knew that the new car had a problem. thought Spielberg would love his cars.

3. The car dealers comment implies that Jewish people a) always ask for a discount. b) dont usually buy cars. c) never buy new cars. d) seldom spend money.

417

INGLS
4. The salesman did not notice that a) another car dealer overheard him. b) another customer canceled the order. c) somebody in the hall dealt with cars. d) someone had heard his remark.

UFMG
F A S E

1 B

5. The customer telephoned Spielbergs office to let him know about a) the anti-semitic statement. b) the cancellation of the order. c) the defects of the car. d) the right price for the car.

P R O V A

6. After the incident, the car dealer a) sold many more cars. b) had no more regrets. c) said he was sorry. d) went to Spielbergs.

7. Spielberg refused to buy the car a) although the car dealer had apologized. b) because the car dealer overheard him. c) in spite of the full price he had got. d) when he knew he could make it right.

8. Spielberg no longer wanted the car because a) he had found a cheaper model elsewhere. b) his friend liked the car dealers manners. c) the car dealer had been rude about Jews. d) the car was too expensive for his friend.

9. The word it in Its probably not too wise. refers to a) to lob an anti-semitic crack at Spielberg. b) to buy a car for a friend in Santa Monica. c) to overhear comments and call Spielberg. d) to sell a car to a Jew for full price.

418

INGLS
TEXT 02 QUESTIONS 10 TO 16

UFMG
F A S E

NE SATURDAY NIGHT, during a visit to a fast-food restaurant with my wife and children, I noticed a womans purse beneath a table in the corner. I gave

1 B

the purse to the manager, and moments later an elderly couple returned in a panic to claim it. The gentleman insisted on giving me a ten-dollar reward for my honesty. In the car on our way home, my four kids began singing, For Hes a Jolly Good Fellow. I explained that I had done nothing extraordinary, but I appreciated the song anyway. Were not singing about you, Daddy, they responded. Were singing about the man who gave us ten dollars. Jack Lyles (Forest, Va.)

P R O V A

10. The man who took his family to the restaurant a) found a purse worth ten dollars. b) handed the lost purse to the manager. 11. The narrator found the purse a) after the couple came in. b) after he entered the place. 12. The couple were in a panic because a) b) c) d) it was time for them to leave the place. the manager claimed the purse was lost. the wife had left the purse in the restaurant. they did not know what had happened. c) just before the couple left. d) when the couple returned. c) offered ten dollars to the waiter. d) saw an old woman lose her purse.

13. In fact, the children were singing because a) b) c) d) the family received a reward for finding the purse. the lady at the restaurant had praised them a lot. the restaurant manager had given them a reward. the waiter at the restaurant was very kind fellow.

14. The father thought the children were singing because a) he had returned the purse. b) the restaurant had been fun. c) they enjoyed going out. d) they usually sang for him.

419

INGLS
15. The best title for this text would be a) Honesty always pays. b) Respect your elders. c) Sing for your supper. d) Time is money.

UFMG
F A S E

1 B 2

16. For the children, He in the sentence For Hes a Jolly Good Fellow refers to a) the father. b) the gentleman. c) the manager. d) the waiter. P R O V A

QUESTION 01 VOCABULARY Circle the word which does not belong to the same family. EXAMPLE Avenue

F A S E

WORD FAMILY

River Road Street

1. Book Television Magazine Newspaper 3. Desk Ruler Eraser Pencil 5. Fork Cake Pie Popcorn

2. Arm Head Hand Hat 4. Doctor Nurse Sailor Dentist

420

INGLS
QUESTION 02 Fill in the blanks with the missing words. The last item is done for you as an example.

UFMG
F A S E

Travel My worst travel problem


THIS SUMMER MY FIANC AND 1 flew to Pittsburgh to visit relatives. During the 2 we experienced major turbulence. The 3 reassured us several times that things were under control, but when I looked out the 4 window, all I see were black 5 and rain. I was scared we werent going to make it. We seemed to be dropping hundreds of feet at a time, the plane kept making these awful whoomp sounds and the wings looked as if they were going to break off. Even though we 6 safely (in a different state), we now have a hard time feeling comfortable flying. Any suggestions?
Sarah Stier, Missoula, Montana Glamour, february 1996. p. 204

Answers 1. 2. 3. 4. 5. 6. landed

421

INGLS
QUESTION 03 SUPERLATIVES Read the text and then complete the sentences below:

UFMG
F A S E

urs is a good restaurant, said the manager. If you order an egg, you get the freshest egg in the world. If you order hot coffee, you get the hottest coffee in the world, and I believe you, said the customer. I ordered a small steak.
Jim Reed, Treasury of Ozark Country Humor

Model: If you order a fresh apple, you get the freshest apple in the world. 1. If you order a small steak, you 2. If you order a spicy omelette, you 3. If you order a thick sandwich, you 4. If you order a good meal, you 5. If you order an expensive dish, you QUESTION 04 CORRECTING MISTAKES Each sentence below has ONE mistake. Re-write the sentences in their correct form. 1. Why the woman was in jail for a night? 2. Why the woman refused to pay the taxi? 3. What did happen there? 4. Who did come to investigate the robbery? 5. I didnt go out last night because was raining. 6. I had thirsty, so I drank some water. 7. I dont have got long black hair. 8. She was going to the post office for to buy some stamps. 9. I started work when I was with nineteen. 10. When Ill arrive home, Ill write to you.

422

INGLS
QUESTION 05 - COMPLETE THE STORY Write the other sentences describing Marks daily activities.

UFMG
F A S E

1. Mark wakes up early every morning.

2.

3.

4.

5.

6.

423

INGLS
QUESTION 06 NOTICES Here are some notices. What do they mean? Complete the sentences.

UFMG
F A S E

1. Please,

2. If you

3. You

4. You

5. You

QUESTION 07 DESCRIPTION Suppose you are applying for a job as a teacher. The interviewer asks you to talk about yourself. What would you say? Write a paragraph of five sentences.

424

INGLS
WHEN DRUG ABUSE BEGINS Eight out of ten adult drug users started as teens; half started at age 16 or younger.

UFPA
F A S E

Drug abuse is a plague that continues to tear American communities apart. It is linked to crime, AIDS, family violence. How can we stop its spread? Common sense says block it before it begins: during the teen-age years when eight out of ten current adult drug users started using drugs. (READERS DIGEST, September, 1995:115) MY FRIEND DEALS DRUGS

Im really close to my friend Thomas we can talk about almost anything. But just recently, I found out that he deals acid and pot. I was shocked when I found out, and asked him why he does it. He said he needs the money. But I dont understand why he has to get money this way, and I dont know what Im supposed to do about it. Mostly, Im afraid hell get hurt. Should I try to persuade him to stop?

You could try to persuade him to stop and convince him that hes hurting himself as well as other people. You could also let him know youre upset about this because you care about him, and you think he could get another (legal) job a job that he could someday put on a rsum. Believe it or not, he might not have thought about some of this. On the other hand, he might think about it all the time and still decide the easiest way to make money is to deal so your talk might not achieve anything. In that case, do yourself a favor: stay away from him when hes dealing (or using), and when hes not, keep letting him know this whole thing is very uncool with you. And cross your fingers for the guy hopefully, hell get his act together before he ends up in jail. Or worse. Cathi Hanauer (Seventeen, September, 1995:103)

BASED ON THE TEXTS CHECK THE ALTERNATIVES THAT COMPLETE THE SENTENCES CORRECTLY:

1. Drug abuse would not increase if it were ... a) b) c) d) e) stimulated in adulthood. obstructed during adolescence. detected after teen-age years. prohibited in American communities. related to crime and AIDS.

2. The majority of adult drug addicters began using drugs as... a) babies. b) children. c) adolescents. d) adults. e) oldsters.

425

INGLS
3. Thomass close friend... a) uses drugs. b) deals drugs. c) lends him money. 4. Thomas... a) makes money by selling drugs. b) enjoys helping his friend. c) has a legal and profitable job. 5. The adviser... a) b) c) d) e) says Thomas should not quit smoking pot. thinks Thomas is being cool with his friend. tries to persuade the writer to send her a rsum. tells Thomass friend to start dealing drugs with him. doubts Thomas can be persuaded to stop dealing drugs. d) wants to be taken to jail. e) makes his friend pround of him. d) is worried about him. e) buys acid and pot from him.

UFPA
F A S E

6. The pronouns which refer to Thomass friend in the texts are... a) I he b) I you c) you he 7. A job that can be put on a rsum... a) b) c) d) e) is an illegal activity. can cause you trouble. is an easy way to get money. is something you are ashamed of. can be included in a curriculum vitae. d) him himself e) yourself himself

8. People who deal drugs... a) b) c) d) e) get money in accordance with law. are aware of the benefits of acid and pot. are a threat to the welfare of society. help stopping the spread of family violence. are easily convincend to quit doing it.

426

INGLS

UFPA
F A S E

APPRECIATING YOUR CHILDS ART


Do you like my picture? Many parents have faced this question while being presented with what looks like a mishmash of squiggles and lines jostling together on a rectangle of paper. Then, struggling to sound knowledgeable and encouraging, theyve said something like, Its lovely. What is it? or, Its lovely, but if you made the eyes bigger it would look just like Mommy. Often, the picture then is hung with pride on the refrigerator door or is squirreled away with others as part of the record of the childs life.
Young children create and combine visual images as a way of constructing, expressing, and communicating their ideas and feelings about themselves and their world. Visual images help them go beyond the confines of verbal language to express their thinking. For example, a scaterring and pileup of clay may depict falling from a high place, or a painting of red blobs may depict when I had a pain in my tummy the other day. Its important, however, to know how to respond appropriately to childrens early paitings, clay sculptures, collages, and drawings. Children are naturally proud of their efforts and want their parents acknowledgement and encouragement. Just as parents supported a childs first efforts to talk and even grew to learn what the non-words meant, parents can learn how to support and understand their childs early endeavors at art. So, to the question Do you like my painting? parents can learn much if they simply say, Itt a wonderful painting. Tell me more about your idea, or I can see there are lots of important things happening in your picture. Can you tell me all about them? Young children love to talk about their art work and, in the telling, give clues to their thinking and insights about their world, insights that are often moving and always profound. A conversation about a childs art work offers the opportunity for parents to learn what pictures and sculptures mean to their child.

(Adapted from BURTON, Judith M. Appreciating your childs art. Crayola Kids, February/March, 1995:42/47)

Responda em portugus s questes abaixo, de acordo com o texto. 1. O que fazem muitos pais aps elogiarem e comentarem a pintura de seu filho pequeno? 2. Qual o significado que a criao e a combinao de imagens visuais em trabalhos como desenhos, colagens e esculturas tm para as criancinhas? 3. Que atitude dos pais comparvel s tentativas de compreenso e encorajamento dos primeiros esforos artsticos de seu filho? 4. Como os pais podem vir a compreender o significado dos trabalhos de arte infantil?

427

INGLS
Read the text carefully to answer items 1 to 5.
Cash if y Dont Cull ou

UFPE
F A S E

(57) (58)

1976, Officials at South Africas Kruger National Park thousands of surplus elephants.

Now the killing may stop. The U.S. Humane Society and U.K. based International Fund for Animal Welfare have offered Pretoria a fiveyear, $5 million grant to buy new (59) lands the sur plus pachyderms can roam free. Theres a cath. To get the cash, South Africa must stop culling elephants in other parks and forswear attempts to repeal the worldwide ban on ivory trade. The plan has split the 18 - member National Parks Board; some members say it tramples on state sovereignty. A decision is expected by April. (Cash if You Dont Cull, SOUTH AFRICA, PERISCOPE, NEWSWEEK, February 26, 1996, page 3.)

1. a) At b) For c) As 2. a) has culled b) will cull c) have culled 3. a) where b) which c) that

d) On e) Since

d) culls e) would cull

d) who e) how

4. Which is the main idea of the text? a) b) c) d) e) The killing of elephants in South Africa has split the members of the National Parks Board. A binational grant may stop the killing of elephants in South Africa. The trapped elephants could not roam free on the new lands. The worldwide ban on ivory trade is expected by April. The officials at South Africas Kruger National Park will catch surplus elephants.

428

INGLS
5. The plan has been.............members of the National Parks Board. a) approved by some b) rejected by all c) signed by all d) accepted by most e) canceled by most

UFPE
F A S E

Wh do I get chapped lips? y The skin covering your lips is usually moist and soft. But too much sun, wind, cold, or dryness will suck the moisture out. Then lips get cracked and dry. To propect your lips, coat them with lip balm.

Question submitted by Mark Bublitz, II Brainerd, Minnesota. (National Geographic/WORLD, August 1996, page 15)

6. The expression suck... out in But too much sun, wind, cold, or dryness will suck the moisture out means: a) absorb b) freeze c) increase d) release e) deliver

7. Identify in the text the two (2) expressions which are equivalent: a) fuel and machine b) calories and stores c) cracked and chapped d) needs and keeps e) lip and coat

On TV and radio, in print and over the internet, news is everywhere. But are we better informed or just overwhelmed? (The News Wars, by Richard Zoglin, Business. TIME, October 21, 1996, vol. 148, n 18, page 44.) (64)

Richard Zoglin in the subtitle of his article on The News Wars seems to be explosion due to the development of technology.

the information

8. a) enthusiastic about b) critical of c) amazed at

d) tolerant of e) receptive to

429

INGLS

UFPE
F A S E

PARA CADA UMA DAS QUESTES A SEGUIR, ASSINALE NA COLUNA I A(S) PROPOSIO(ES)VERDADEIRA(S) E N COLUNA II A(S) FALSA(S). A Read the dialog carefully to answer items 1 to 7.
Johnny: More and more American adults are enrolling in continuing education programs on US campuses, making them the new majority and the largest growth in education. Nancy: What do they use continuing education for? Johnny: Both for pleasure or to satisfy a yearning for learning. Nick: Dont you think economic forces are at the heart of this boom? Johnny: Absolutely. And more specifically, a changing work force and the instability of the work market are necessitating this return to the classroom. Helen: Besides that, adults are willing to keep up with technological shifts, to change careers when their field becomes redundant, or simply to become versatile.

Nancy: Whatever the reason, the notion of education one does early in life is disappearing. (Adapted into dialog form from Education. Back to School, MODERN LIFE NEWS, SPEAK UP May 1996, Ano 9, n 110, page 33.)

1. According to the characters point of view, the reasons for this return to the classroom at adulthood are: 0-0) onefold 1-1) twofold 2-2) manifold 3-3) varied 4-4) unique

2. Some of the reasons mentioned in the dialog are: 0-0) 1-1) 2-2) 3-3) 4-4) to keep up with technological shifts. to satisfy a yearning for learning. to become marketing students. to adjust oneself to changing economic forces. to learn more about life.

3. These kinds of students stand for: 0-0) 1-1) 2-2) 3-3) 4-4) the smallest growth group in American education. the new minority on US campuses. the latest boom in American education. the largest growth group in continuing education in the US. The new majority in continuing education programs in America.

4. Through this continuing education programs the ancient assumption that education one does early in life has been: 0-0) largely questioned. 1-1) totally maintained. 2-2) entirely acceptable. 3-3) enthusiastically approved. 4-4) widely rejected.

430

INGLS
5. Absolutely is equivalent to: 0-0) Not at all. 1-1) Perfectly. 2-2) Exactly. 3-3) Undoubtedly. 4-4) Unquestionably.

UFPE
F A S E

6. The expression to keep up with in adults are willing to keep up with technological shifts means: 0-0) to be at the same pace as 1-1) to be a little bit ahead of 2-2) to go forward at the same speed as 3-3) to be much better than 4-4) to be a little behind of

7. Yearning in to satisfy a yearning for learning signifies: 0-0) a determined willingness 1-1) a strong desire 2-2) a serious longing 3-3) a tender longing 4-4) an earnest desire

Read this text to answer question 8 to 12


SMOKING AND HEALTH Cigarettes May Promete Blindness Not that you needed another reason to avoid smoking. But in case cancer, heart disease and bad breath (8) two new studies offer another one: blindness. Writing in last weeks Journal of the American (9) Medical Association, , strongly link cigarette smoking to age-related macular degeneration (10) (AMD), a degenerative eye condition is the leading cause of blindness in older Americans. No one knows how smoking promotes AMD; it may impede circulation or simply accelerate the aging of cells.

(NEWSWEEK, October 21, 1996, page 54)

In items 8, 9 and 10 choose the most appropriate expression(s) to fill in the blanks in the text. 8. 0-0) are no reason sufficient 1-1) arent sufficient reason 2-2) are no sufficient reason 9. 0-0) 1-1) 2-2) 3-3) 4-4) a pair of Boston-based research teams Boston-based a pair of research teams teams of Boston-based of a research pair research a pair of Boston-based teams teams a pair of Boston-based research 3-3) arent enough reason 4-4) are no reason enough

431

INGLS
10. 0-0) what 1-1) that 2-2) who 3-3) how 4-4) which

UFPE
F A S E

In items 11 and 12 identify the expression(s) which is (are) equivalent to that (those) from the text. 11. The expression link in a pair of Boston-based research teams strongly link cigarette smoking to means: 0-0) connect 1-1) blame 2-2) associate 3-3) oppose 4-4) relate

12. The word aging in it may impede circulation or simply accelarate the aging of cells means: 0-0) renewing 1-1) getting old 2-2) renovating 3-3) growing old 4-4) getting stronger

FORMAL, INFORMAL OR SLANG? One of the trickier aspects of speaking a foreign language is knowing how to choose the most appropriate word when there are several which seem to have the same meaning. For instance, it is quite appropriate to use informal words in informal conversation, but sland words should be avoided by foreigners, since they are generally used only between people who know each other very well. And it is also important to remember that at the other end of the spectrum there are some words which are extremely formal. These might not be used very often at all in conversation: they are more appropriate in writing. (Adapted from LADIES AND GENTS..., SPEAK UP, Language and Literature, February 1992, pages 44-45.)

13. Which choices best summarize the main idea of the paragraph? 0-0) the memorization of words that are extremely formal. 1-1) the correct use of words both in speaking and in writing 2-2) the frequent use of sland words by foreigners. 3-3) the appropriateness of vocabulary in language usage. 4-4) the appropriate use of informal words in writing.

432

INGLS

UFPE
F A S E

14. Since in since they are generally used only between people who know each other very well is equivalent to: 0-0) otherwise 1-1) for 2-2) because 15. They in the above sentence refers to: 0-0) formal words. 1-1) informal words. 2-2) foreigners. 3-3) people. 4-4) slang words. 3-3) as 4-4) but

16. The equivalent active sentences to These might not be used very often at all in conversation are: 0-0) 1-1) 2-2) 3-3) 4-4) One might not use these very often at all in conversation. We may not use these very often at all in conversation. No one might use these very often at all in conversation. People may not use these very often at all in conversation. You might not use these very often at all in conversation.

433

INGLS
Instruo: As questes de nmero 11 a 18 referem-se ao texto abaixo:
1.
2. 3. 4. 5. 6. 7. 8. 9. 10. 11. 12. 13. 14. 15. 16. 17. 18. 19. 20. 21. 22. 23. 24. 25. 26. At least half of all emergency room visits are for problems that dont require immediate attention or that would be better handled by a patients own doctor. Here, guidelines for judging which is which. A high fever is not necessarily urgent. Even a temperature of 40C wont harm those without chronic medical conditions such as diabetes, emphysema, or hear t disease. If you are achy all over and have a fever, call your doctor. Exception: any fever in a child younger than 2 months or an adult older than 65. In those cases, get medical attention right away. Heavy bleeding from a small wound on the face, scalp, or hands is less serious than commonly believed. These areas enjoy a rich blood supply, so a small cut may bleed profusely. Your only option is to apply pressure with a clean gauze pad. Sutures are necessary if a wound is deep or jagged or if it hasnt stopped bleeding after applying pressure. Doctors wont suture a wound thats been open for 12 hours or more. Coughing up blood is not always a medical emergency and it is rarely a sign of lung cancer or tuberculosis. Anyone with a bad cold or bronchitis can cough up a little bloody sputum, so if you are otherwise healthy, just call your doctor.

UFRGS

1. O texto nos diz que a) b) c) d) e) um corte que sangra abundantemente no pode ser suturado. bebs e pessoas idosas tm prioridade nos pronto-socorros. febre alta no deve preocupar portadores de diabetes ou enfizema. a maioria das pessoas no bem atendida nos pronto-socorros. metade dos casos de pronto-socorro no so de urgncia.

2. O ttulo mais apropriado para esse texto seria a) b) c) d) e) Guidelines for the inexperienced doctor. When not to go to the emergency room. You and your doctor. What to do at the emergency room. How to get alternative therapy.

434

INGLS
3. A expresso que melhor substitui right away (li.12) a) in a while. b) later on. c) sooner or later. d) at once. e) afterwards.

UFRGS

4. A melhor traduo da palavra enjoy (li. 15) nesse contexto a) emprestam. b) rejeitam. c) gostam. d) enjoam. e) recebem.

5. A frase if you are otherwise healthy (li. 25) seria melhor traduzida como a) b) c) d) e) se voc no saudvel de outras maneiras. se voc tiver pouca sade. se voc for saudvel e esperto. se voc no tem outras doenas. se voc tem um plano de sade.

6. Assinale a alternativa em que a forma -ING no usada como em heavy bleeding (li. 12-13). a) b) c) d) e) Vomiting is sometimes cause for immediate attention. Diminished hearing is less serious than diminished vision. The doctor is now suturing a deep cut in her hand. Animal bite requires immediate cleaning and oral antibiotics. Once bleeding stops, wash the wound with soap and water.

7. Na frase ...and it is rarely a sign of lung cancer (li. 22-23) o pronome it se refere a a) coughing up blood (li. 21). b) a medical emergency (li. 22). c) a sign (li. 22-23). d) lung cancer (li. 23). e) tuberculosis (li. 23).

8. Complete a frase abaixo com a alternativa correta.


If you have had a tetanus shot in the past five ... eight years, ... injection isnt required, ... the wound is very dirty.

a) or - many - but b) to - another - unless c) to - other - yet

d) for - some - except e) or - none - if

435

INGLS
Instruo: As questes de nmeros 9 a 17 referem-se ao texto abaixo:
Squirrel vs. Squirrel in Britain 1. 2. 3. 4. 5. 6. 7. 8. 9. 10. 11. 12. 13. 14. 15. Up a perilous tree, Britains native red squirrels are being overrun by their larger cousins, Nor th American gray squirrels. Brought to Britain as a novelty in 1876, grays outcompete reds for food. Only about 160,000 reds remain, against an onslaught of some 2.5 million grays. Grays may have a secret ally. Some scientists suspect that grays carry a virus called parapox, which is killing reds in nor thern and eastern England. But the cause and the origin of the disease are still unknown. To beef up reds chances, landowners are urged to plant the proper tree mix. Red squirrels need conifer seeds in winter - they have difficulty digesting acorns, which grays readily wolf down.

UFRGS

9. O ttulo que no caberia a este texto a) Yankees Go Home. b) Cousins but Enemies. c) An Uneven Match. 10. O texto apresenta a) b) c) d) e) uma discusso do valor dos esquilos no ecossistema da Gr-Bretanha. uma estatstica dos danos causados aos esquilos cinzentos. uma possvel soluo para o problema dos esquilos vermelhos. uma alternativa para reconduzir os esquilos cinzentos ao seu habitat. uma sugesto para combater eficazmente o vrus parapox. d) A Parallel Story. e) A Family Business.

11. A relao correta de acordo com o texto a) gray squirrels - weaker b) red squirrels - perilous c) gray squirrels - sick d) red squirrels - novelty e) red squirrels - smaller

12. O ano 1876 mencionado no texto como o ano em que a) b) c) d) e) grandes novidades surgiram vindas da Amrica. os cientistas diagnosticaram o vrus parapox. havia um grande nmero de esquilos cinzentos na Inglaterra. os norte-americanos importaram um grande nmero de esquilos. os primeiros esquilos cinzentos foram levados para a Gr-Bretanha.

436

INGLS
13. A palavra some (li. 6) poderia ser substituda sem alterao do sentido por a) fully. b) partly. c) approximately. d) more than. e) average.

UFRGS

14. O verbo carry (li. 8) est usado incorretamente na alternativa a) b) c) d) e) We should carry dearly for our families. Police officers in Britain do not carry guns. Malaria is a disease carried by mosquitoes. Some cars carry a 12-month guarantee. Todays newspapers all carry the same story.

15. A expresso to beef up reds chances (li. 11-12) sugere o mesmo que a) to cut off reds chances. b) to increase reds chances. c) to shut up reds chances. d) to postpone reds chances. e) to defeat reds chances.

16. A palavra proper (li. 13) poderia ser traduzida sem alterao do sentido por a) preparada. b) proposta. c) pronta. d) adequada. e) importante.

17. A palavra readily (li. 15) poderia ser substituda sem alterao do sentido por a) hardly. b) promptly. c) mostly. d) practically. e) wildly.

437

INGLS
Instruo: As questes de nmeros 53 a 61 referem-se ao texto abaixo:
1. 2. 3. 4. 5. 6. 7. 8. 9. 10. 11. 12. 13. 14. 15. 16. 17. 18. 19. 20. 21. 22. 23. 24. 25. 26. 27. 28. 29. The stor y of Mexico echoes the land itself, an arduous landscape of peaks and valleys rising and falling like the countrys tumultuous histor y. Growth and decline, hope and disappointment all play out across the centuries here. The Mexican land, which nourished Americas first great civilizations, has endured the shock of European conquest, frequent wars, and hard pover ty to be born again as something new. Rich in natural resources, blessed with strong family ties and a hardworking people, Mexico is ready to move from the ranks of developing nations into a new role, this time as a modern player on the world stage. But those hopes have been tarnished, at least for the moment, by political corruption, civil unrest, environmental pollution, and the Mexican governments devaluation of the peso. The resulting economic woes have exacerbated tensions along the US Mexican border, where drug trafficking and illegal immigration rise each time the peso falls. Such ripples touch neighbors in all directions, for our lives are ever more closely linked by the Nor th American Free Trade Agreement, by the recent guarantee of 20 billion dollars in US loans to Mexico, by the growing influence of Hispanic culture spreading nor th of the border, and by modern communications that shrink the world with each passing day.

UFRGS

18. The text contrasts a) b) c) d) e) European influence in Mexico and Hispanic culture in the US. ancient Mexican civilizations and present day hardworking people. Mexican and US respective participations in the NAFTA. the countrys geography and its political development. Mexicos hard poverty and its rich cultural tradition.

19. Which of the following statements is not correct according to the text? a) b) c) d) e) US investments in Mexico have brought both countries closer than ever before. Mexico is undergoing a moment of painful rebirth. Frequent wars and environmental pollution have ruined Mexicos natural resources. Mexican economic difficulties cause US drug problems to increase. Mexico is about to take a more important position in the modern world.

438

INGLS
20. The word tarnished (li. 15) may be replaced, without altering the meaning by a) distinguished b) furnished c) augmented 21. The best translation for endured (li. 7) is a) suportou. b) endureceu. c) transportou. d) criticou. e) transmitiu. d) painted e) diminished

UFRGS

22. The expression with each passing day (li. 29) means, in Portuguese, a) a cada dia passando. b) a cada dia que passa. c) como passar os dias. d) como passam os dias. e) com os que passam diariamente.

23. Choose the best alternative to complete the sentence below correctly:
Mexico ... many difficult crises in history, but now it ... its own future.

a) has faced - is shaping b) faced - was shaped c) have faced - shapes

d) have been facing - shaped e) faces - has been shaped

24. The expression developing nations (li. 12) means in Portuguese a) naes que se destacam. b) desenvolvimento de naes. c) naes em desenvolvimento. d) destaque das naes. e) naes desenvolvidas.

25. Choose the best word to replace which in which nourished Americans first great civilizations (li. 6-7). a) whether b) who c) whose d) those e) that

26. Fill in the blank below with the best alternative.


Political corruption and civil unrest are Mexicos modern problems.

a) because b) between c) throughout

d) among e) although

439

INGLS
Instruo: As questes de nmeros 27 a 35 referem-se ao texto abaixo:
1. 2. 3. 4. 5. 6. 7. 8. 9. 10. 11. 12. 13. 14. 15. 16. 17. 18. 19. 20. 21. 22. 23. 24. 25. 26. 27. 28. 29. 30. 31. 32. Hammett came to the foot of the stairs and in a whisper said, Come down. Be ver y quiet. When you get to the last few steps, crouch very low so that you cant be seen through the window. His voice was excited and happy. I crawled the steps after the landing, crawled across the room, and he raised me slowly to my feet. Before me was the finest sight of my life, so stunning, so unbelievable, that I began making choking sounds until Hammett put his hand in my mouth. On the wide road from the lake at least twenty deer, moving slowly, were joining a larger group who were wondering up the shorter path through the fruit trees. All of them, small and large, pale and darker, moved without fear, stopping along the way to nibble at the May buds. Eight of them had moved close to the terrace, were looking up at the house, but without curiosity, as if it were another kind of tree. Then a group of them went past the terrace and up into the rock garden where they found such lovely things to eat that they were joined by six or seven others. In all they certainly numbered forty or fifty does, bucks, fawns, moving as I think few people have ever been allowed to see them, untroubled, not even on their usual alert for smell or rustle. It was after six oclock when the deer began to disappear. There were four last stragglers who stopped to examine a small dogwood immediately off the terrace, but one of our dogs barked in the distant kennels and the deer were off into the woods.

UFRGS

27. According to the text, the house a) has more than one floor. b) is located in a busy urban area. c) is crowded with people. d) is too small and dark. e) has several windows.

28. The question that cannot be answered using the information given in the text is a) b) c) d) e) What season of the year is it? Are all the deer of the same kind? Are the deer frightened? How long did it take the deer to go up the rock garden? Why has the author crawled when coming down the stairs?

440

INGLS
29. In this text the author describes a) b) c) d) e) the behaviour of domestic animals. a moment of rare beauty. a dream she had. the parts of a garden. her feelings for Hammett.

UFRGS

30. The author is told to be very quiet a) b) c) d) e) in order not to frighten the birds. because others were sleeping in the house. in order not to disturb her friend Hammett. so that the surprise would be total. because any sound might cause the deer to run away.

31. The phrase the foot of the stairs (li. 1) is the opposite of a) the front of the stairs. b) the bottom of the stairs. c) the top of the stairs. d) the back of the stairs. e) the land of the stairs.

32. A correct active version of the expression so that you cant be seen (li. 4) is a) so that anything cant see you. b) so that nothing cant see you. c) so that anybody can see you. d) so that no one can see you. e) so that none cant see you.

33. The word that cannot receive the prefix un- as in unbelievable (li. 9) is a) fortunate. b) important. c) ability. d) predictable. e) skillful.

34. The word that does not form the comparative in the same way as large (li. 13) or short (li. 14) is a) bad. b) sad. c) glad. d) great. e) late.

35. The expressions immediately off (li. 30) and were off (li. 32) mean respectively a) close to - went away b) approximately - far off c) in - gathered d) nearby - far away e) together - jumped

441

INGLS
Nas questes de 1 a 14, marque: itens CERTOS, na coluna I; itens ERRADOS, na coluna II.

UnB

Use, para as devidas marcaes, a Folha de Rascunho e, posteriormente, a Folha de Respostas.

Read the following text and answer questions 1 to 3 below.


1 After years of struggling with obesity, a lady finally managed to go down to her ideal weight in 6 months following a revolutionary method adopted by a very famous doctor. She is grateful and goes to the doctors office to thank him. Doctor, says she, there are some things you cant pay for. I thought that you would like to accept this wallet which I made for you with my own hands. 10 Madam, answered the doctor rudely, I dont practice medicine for sentimental reasons. You must pay me cash. Your bill is two thousand dollars. The lady opened the wallet, took out the five thousand dollars she had put in it and gave the doctor two thousand.

Questo 1 According to the text: (0) The doctor lost five thousand dollars. (1) The lady wanted to return the doctors wallet. (2) The money in the wallet was less than the doctors fee. Questo 2 According to the text: (0) If the doctor had accepted the ladys gift, he would have received U$ 5,000. (1) The lady had been trying to lose weight for six months. (2) The lady had a wallet made for the doctor. Questo 3 In the text: (0) managed to go down (li. 2) could be correctly replaced by succeeded in going down. (1) cash (li. 11) could be correctly by in money. (2) bill (li. 11) is the same as account.

442

INGLS
Read the following text and answer questions 4 to 6 below.
1 The original Olympic Games can be traced back to 776 BC, as a festival to honour the gods of ancient Greece. But the modern Olympic Games are only a century old. They began in 1896, in Athens, Greece, and were largely due to the inspiration of a Frenchman, Baron Pierre de Coubertin. He believed that by developing healthier minds and bodies in the young people of all nations, the Olympic Games would promote world peace. De Coubertins ideals of good sportsmanship and fair play now seem to many people to have been perverted by the spirit of politics and commerce which surrounds the Olympiad. The first, and perhaps worst, example of the use of the Olympics for political ends came in 1936 in Berlin. Luckily, Hitlers aim of exploiting the Games to demonstrate the superiority of pure blond Aryan youth was frustrated by the black American athlete, Jesse Owens, who dominated the track events and won four gold medals. The intense nationalism and the fierce competition for medals aroused by the Games also conflict with Baron de Coubertins most famous saying: The important thing in the Olympic Games is not winning, but taking part. The essential thing in life is not conquering, but fighting well.

UnB

10

15

20

Questo 4 According to the text: (0) In ancient Greece, the Olympic Games had a religious purpose. (1) Baron Pierre de Coubertin came from Greece. (2) The first modern Olympic Games were held in Athens. Questo 5 According to the text: (0) In 1936, Hitler wanted to prove that whites were superior to blacks. (1) Hitler achieved the result he wanted from the Olympic Games. (2) Baron de Coubertin believed that intense nationalism was good.

443

INGLS
Questo 6 In the text: (0) (1) (2) (3) perhaps (li. 14) could be correctly replaced by maybe. ends (li. 15) could be correctly replaced by finishes. came (li. 15) is the past simple tense of the verb to come. youth (li. 17) means young people.

UnB

Read the following text and answer questions 7 and 8 below.


1 Its time to rethink nature. Every day, science seems to reject our autonomy. When researchers arent uncovering physical differences in the way men and women use their brains, theyre discovering genetic influences on intelligence, sexual orientation, obesity or alcoholism. Or theyre suggesting that the level of some brain chemical substance affects ones chance of committing violent crimes. Each new finding leaves the impression that biology is destiny and free will an illusion.

Questo 7 According to the text: (0) (1) (2) (3) (4) Science seems to prove our autonomy. A physical difference may be determined by the way we use our brains. There may be a relation between human behaviour and level of substances found in our brains. Alcoholism may be the result of genetic influence. Biology tends to deny the existence of free will.

Questo 8 In the text: (0) (1) (2) (3) (4) (5) seems (li. 2) is the same as appears. their (li. 2) refers to researchers (li. 3). some (li. 7) may be correctly replaced by any. leaves (li. 9) is a plural form. finding (li. 9) is a verb. will (li. 10) is a noun.

444

INGLS
Read the following text and answer questions 9 to 11 below.
1 What is being discussed, of course, is the role of women. To defend the two-parent family is to defend an institution in which the woman is subordinated to her husband, confined to domestic activities with no opportunity to pursue a career, and taught to indoctrinate her children with a belief in the rightness of this arrangement. To some critics, the woman here is not simply constrained, she is abused. The traditional family, in this view, is an arena in which men are free to hit, rape and exploit women. To defend the traditional family is to defend sexism.

UnB

10

Questo 9 According to the text: (0) To some critics the womans role is open to abuse. (1) To defend the two parent-family is to defend the husbands superior role. (2) In a traditional view of a family, men can subordinate their wives. Questo 10 According to the text: (0) Some critics believe that women in the context described are both constrained and abused. (1) Women are taught to pursue a career. (2) The traditional view of a family has the same principles as the two-parent family. Questo 11 In the text: (0) (1) (2) (3) (4) (5) the woman (li. 3) is the subject of taught (li. 5). to pursue (li. 5) is the same as to follow. rightness (li. 6) means moral justification. is abused (li. 8) is an active form. which (li. 9) refers to an arena (li. 10). exploit (li. 10) and explore are synonyms.

445

INGLS
Read the following text and answer questions 12 to 14 below.
1 Brazil actually is far more conservative than the beach scenes at Ipanema would suggest. Young women still value virginity. Gays and lesbians suffer serious discrimination. Abortion is illegal and the governments family planning efforts have been timid, at best. Few women dare suggest their lovers wear condoms for fear of insulting them, or being considered prostitutes. Parents have been surprised by recent research which found that children are increasingly confused over sexual values.

UnB

Questo 12 According to the text: (0) The behaviour on the beach of Ipanema gives a false impression of sexual liberalism in Brazil. (1) Young women no longer value virginity. (2) Few women are afraid to suggest their lovers wear condoms. (3) Brazil is a rather sexually liberal country. (4) The Governments family planning hasnt been as effective as it should be. (5) Sexual values are becoming clearer to children. Questo 13 In the text: (0) actually (li. 1) is synonymous with really. (1) far more (li. 1) can be correctly replaced by much more. (2) at best (li. 6) is the opposite of at worse. Questo 14 In the text: (0) dare (li. 6) can be correctly replaced by need. (1) their lovers (li. 6) is a direct object. (2) Parents (li. 8) can be correctly replaced by Relatives.

446

INGLS
Na questo 15 a seguir, marque:

UnB

o algarismo das DEZENAS, na coluna I; o algarismo das UNIDADES, na coluna II; o algarismo das DEZENAS deve ser obrigatoriamente marcado, mesmo que seja igual a zero. Use, para as devidas marcaes, a Folha de Racunho e, posteriormente, a Folha de Respostas.

Questo 15 Read the following text: Pedro and Ana live near Brasilia with their children and 3 of Anas brothers. Pedro works as a cabinet maker and Ana works as a cleaning woman in an office. Theyve lived in Brasilia for 14 years and have been granted a government educational scholarship which enables children to stay in school instead of making them work early to help their families. In order to qualify for it, a family must have been a resident of Brasilia for at least 5 consecutive years and the children must be attending school. Bruno settled in Brasilia 3 years after his sister Ana arrived. He only has a one-month old baby. When Tiago, the other brother joined them, Ana had already been living here for 11 years. He had 3 children ages 8, 10 and 12. Leonardo, Anas youngest brother is 19 and single. He had been living with Ana for 4 years when Tiago joined them. Find out the sum of the numbers of years Ana and each of her brothers have lived in Brasilia.

447

INGLS
INSTRUO: As questes de nmeros 1 a 5 referem-se ao texto seguinte.
THE EARLY OLYMPIC GAMES

UNESP
F A S E

The celebrated games of the Greeks had their origin in the Heroic Age and were associated with religious festivals observed by an entire city or community, and were played near the oracle or altar of the god in whose honor they were instituted. By the sixth century B.C. they had lost their local character and assumed a national status. The Olympian games became the most famous of such festivals. In 776 B.C. Coroebus was victor in the foot-race at Olympia, and as from that time the names of the winners were carefully registered, that year came to be used by the Greeks as the starting-point in their chronology. The games were held every fourth year, and the intervals between two successive festivals was known as an Olympiad. The date of the occurrence was given by saying that it happened in the first, second, third, or fourth year of such an Olympiad the first, second, or third, etc. The contests consisted of foot-races, boxing, wrestling, and other athletic games. Later, chariot-racing was introduced, and became the most popular of all the contests. The competitors must be of the Hellenic race; must have undergone ten months training in the gymnasium; and must, moreover, be innocent of any crime against the state or sin against the gods. Spectators from all parts of the world crowded to the festival. The delegates of the different states competed with one another in the richness and splendor of their chariots and equipments, and in the magnificence of their attendants. The champion was crowned with a garland of wild olives; heralds proclaimed his name abroad; his native city received him as hero, sometimes through a hole made in the city walls; his statues, executed by eminent artists, were erected at Olympia and in his own city; sometimes even divine honor and worship were given to him; and poets and orators competed with the artist in perpetuating the name and deeds of him who had reflected undying honor upon his native state.

1. De acordo com o texto, indique a alternativa correta. a) Os jogos gregos, que se originaram na idade herica e faziam parte dos festivais religiosos at o sexto sculo a.C., no adquiriram status nacional porque as comunidades locais no queriam desagradar os seus deuses. b) Os festivais religiosos observados por cidades inteiras na Grcia antiga transformaram-se em jogos quando as comunidades resolveram agradar aos deuses perto dos seus altares. c) Por volta do sculo VI a.C., os jogos deixaram de ser uma instituio local para os gregos, pois estes acreditavam que os deuses sentiam-se homenageados quando lembrados nacionalmente. d) A poca do incio dos jogos da Grcia antiga passou a ser conhecida como a Idade Herica, em virtude dos festivais religiosos realizados em torno dos oratrios dos deuses como homenagem dos heris. e) Os jogos da Grcia antiga, associados aos festivais religiosos, eram realizados nas proximidades dos altares dos deuses que as comunidades queriam venerar e, por volta do sexto sculo a.C., passaram a ter mbito nacional.

448

INGLS
2. De acordo com o texto, indique a alternativa correta.

UNESP
F A S E

a) Os jogos olmpicos tornaram-se os mais famosos dos festivais da Grcia antiga quando os gregos passaram a utiliz-los como referncia cronolgica. b) O famoso corredor Coroebus esforou-se para vencer a prova olmpica porque esta fazia parte dos festivais que o associaram cronologia grega. c) Os jogos olmpicos, o mais famoso dos festivais religiosos da Grcia antiga, tornaram-se o ponto de referncia da cronologia grega a partir de 776 a.C., quando as vitrias passaram a ser cuidadosamente registradas. d) Os nomes dos vencedores eram registrados anualmente para que os gregos pudessem estabelecer o ponto de partida de sua cronologia, desconhecido antes de 776 a.C. e) Os jogos olmpicos tornaram-se os mais famosos festivais a partir de 776 a.C. por causa do registro da vitria de Coroebus, inventor da cronologia grega. 3. Segundo o texto, correto afirmar: a) As provas consistiam em corrida, luta de boxe, luta livre e outros jogos atlticos, mas quando a corrida de carruagens foi introduzida, esta tornou-se a mais popular de todas. b) A corrida de carruagens foi instituda para evitar a monotonia que ameaava a consolidao dos jogos olmpicos, cuja popularidade vinha diminuindo. c) A corrida de carruagens teve de ser introduzida nos jogos olmpicos para tambm ajudar no transporte de atletas e espectadores que residiam longe da sede dos jogos. d) Os donos das carruagens competiam uns com os outros quanto decorao das mesmas e quanto ao traje de seus acompanhantes, no intuito de agradar aos deuses. e) A corrida de carruagens tornou-se a prova mais popular porque era realizada fora do ginsio e podia ser vista pelas multides que vinham de outros pases. 4. Conforme o texto, assinale a alternativa correta. a) Os atletas culpados de crimes e pecados podiam disputar as olmpiadas desde que se submetessem a treinamento intensivo nas dependncias do ginsio. b) Os atletas culpados de crimes e pecados podiam disputar as olmpiadas desde que pertencessem raa helnica e freqentassem regularmente o ginsio. c) Atletas de quaisquer raas podiam disputar os jogos desde que fossem inocentes dos crimes e pecados da raa helnica. d) Os atletas tinham de ser da raa helnica, tinham de passar por treinamento intensivo e deviam ser inocentes de crimes e pecados. e) Os atletas que no eram da raa helnica podiam disputar os jogos olmpicos, mas no podiam fazer o treinamento intensivo no ginsio.

449

INGLS
5. Segundo o texto, correto afirmar:

UNESP
F A S E

a) Os campees olmpicos eram coroados com folhas de oliveira pelos arautos que proclamavam os seus feitos em sua terra natal e no exterior. b) Os campees dos jogos olmpicos chegavam a ser venerados como deuses, eram recebidos em suas cidades como heris, serviam de modelos para os artistas e eram exaltados pelos oradores. c) Os conterrneos dos campees s vezes faziam buracos nos muros das cidades, para transportarem as esttuas dos heris esculpidas por artistas eminentes. d) As esttuas dos campees eram erigidas em sua cidade natal e em Olmpia, para permitir a venerao do povo e dos artistas. e) Os poetas e os oradores invejavam os artistas por serem estes mais bem sucedidos na perpetuao dos feitos dos heris.

INSTRUO: Assinale a alternativa que preenche corretamente cada lacuna das questes de nmeros 6 a 12.

6. A lion is than a dog. a) strong b) strongest c) more strong d) stronger e) most strong

7. Those two women always help a) other each. b) each other. c) one other. d) other one. e) another each.

8. John me some money last week. a) sends b) send c) sent 9. The rain can our shoes. a) spoil b) spoils c) spoiled d) to spoil e) spoiling d) sending e) to send

450

INGLS
10. We stayed in Rome two months. a) since b) at c) in d) on e) for

UNESP
F A S E

11. Charles Mary are brother sister. a) or and b) and or c) and but 12. I have a friends. a) ten b) few c) some d) many e) several d) and and e) but and

451

MATEMTICA

CESGRANRIO

1. Uma torneira alimenta um reservatrio dgua cujo volume, em funo da altura que o nvel da gua atinge, registrado por um cientista, o qual, com os dados obtidos, constri o grfico abaixo.

Qual o percentual de aumento do volume de gua nesse reservatrio quando o nvel de gua varia de 6 cm para 10 cm? a) 15% b) 18% c) 20% d) 25% e) 35,8%

r 2. Os vetores u (x, 2x
a) 3/2 b) 2/3 c) 2/5

r 1) e v ( 2, 4) so ortogonais. Ento o valor de x igual a:


d) 2/3 e) 3/2

3. Sendo A

7 cos(5

8 sen 2

x)

3 cos(3 x

x)

, com x 2

k , k Z, ento :

a) A b) 2A c) 2A

1 1 1

d) 4A 5 0 e) 5A 4 0 0

4. Um projetor de slides, colocado a 4 metros de distncia de uma tela de cinema, projeta sobre ela um quadrado. Para que a rea desse quadrado aumente 20%, a que distncia da tela, em metros, deve ser colocado o projetor? a) 4,20 b) 4,50 c) 4,80 d) 5,60 e) 6,00

452

MATEMTICA
5. Se 0,6% de 3 1 3 a) 3,4% b) 9,8% c) 34%
3x 1, ento o valor de x :

CESGRANRIO

d) 54% e) 98%

6. Um fiscal do Ministrio do Trabalho faz uma visita mensal a cada uma das cinco empresas de construo civil existentes no municpio. Para evitar que os donos dessas empresas saibam quando o fiscal as inspecionar, ele varia a ordem de suas visitas. De quantas formas diferentes esse fiscal pode organizar o calendrio de visita mensal a essas empresas? a) 180 b) 120 c) 100 d) 48 e) 24

7. O grfico que melhor representa a funo real definida por f(x)

x2

2x

1 :

8. O nmero de assinantes de um jornal de grande circulao no estado aumentou, nos quatro primeiros meses do ano, em progresso geomtrica, segundo os dados de uma pesquisa constantes na tabela abaixo.

MS NMERO DE ASSINANTES

janeiro 5000

fevereiro

maro 6050

abril

Em relao ao ms de fevereiro, o nmero de assinantes desse jornal no ms de abril teve um aumento de: a) 1600 b) 1510 c) 1155 d) 1150 e) 1050

453

MATEMTICA

CESGRANRIO

9. O ponto de maior ordenada, pertencente ao grfico da funo real definida por f(x) (2x 1) (3 x), o par ordenado (a, b). Ento a b igual a: a) 39/8 b) 11/8 c) 3/8 d) 11/8 e) 39/8

10. Na figura ao lado, AB 8 cm, BC 10 cm, AD 4 cm e o ponto O o centro da circunferncia. O permetro do tringulo AOC mede, em cm: a) 36 b) 45 c) 48 d) 50 e) 54

11. Segundo dados de uma pesquisa, a populao de certa regio do pas vem decrescendo em relao ao tempo t, contado em anos, aproximadamente, segundo a relao P(t) P(o). 2 0,25t. Sendo P(o) uma constante que representa a populao inicial dessa regio e P(t) a populao t anos aps, determine quantos anos se passaro para que essa populao fique reduzida quarta parte da que era inicialmente. a) 6 b) 8 c) 10 d) 12 e) 15

12. Se a, b e c so as razes da equao x3 a2bc ab2c abc2 igual a: a) 400 13. b) 200 c)

10x2 100

2x d)

20 200

0, ento o valor da expresso e) 400

Uma barra de ferro com temperatura inicial de 10C foi aquecida at 30C. O grfico acima representa a variao da temperatura da barra em funo do tempo gasto nessa experincia. Calcule em quanto tempo, aps o incio da experincia, a temperatura da barra atingiu 0C. a) 1 min b) 1 min 5 seg c) 1 min 10 seg d) 1 min 15 seg e) 1 min 20 seg

454

MATEMTICA

CESGRANRIO

14. Um complexo z possui mdulo igual a 2 e argumento . Sendo z o conjugado de z, a forma 3 algbrica do complexo z : a) 1 b) c) d) 1 e) 2
i 3

3 3

i i 3

15. Uma folha de papel colorido, com a forma de um quadrado de 20 cm de lado, ser usada para cobrir todas as faces e a base de uma pirmide quadrangular regular com altura de 12 cm e aptema da base medindo 5 cm. Aps se ter concludo essa tarefa, e levando-se em conta que no houve desperdcio de papel, a frao percentual que sobrar dessa folha de papel corresponde a: a) 20% b) 16% c) 15% d) 12% e) 10%

x k 1 16. Para que valores de k existe uma nica matriz y , tal que 1

2 x k y

0 ? 0

a) b) c) d) e)

k 1 k 2 k 2 ou k 1 k 2 e k 1 k 2 e k 1

17. Se 2.log3 x a) 1/2

0, ento log1/9 x vale: c) 1/8 d) 1/4 e) 1/2

b) 1/4

18. O dispositivo que aciona a abertura do cofre de uma joalheria apresenta um teclado com nove teclas, sendo cinco algarismos (0, 1, 2, 3, 4) e quatro letras (x, y, z, w). O segredo do cofre uma seqncia de trs algarismos seguidos de duas letras. Qual a probabilidade de uma pessoa, numa nica tentativa, ao acaso, abrir o cofre? a) 1/7200 b) 1/2000 c) 1/1500 d) 1/720 e) 1/200

455

MATEMTICA
19.

CESGRANRIO

A equao da circunferncia cuja representao cartesiana est indicada pela figura acima : a) b) c) d) e) x2 x2 x2 x2 x2 y2 3x 4y 0 y2 6x 8y 0 y2 6x 8y 0 y2 8x 6y 0 y2 8x 6y 0

20. Um recipiente com a forma de um cilindro reto, cujo dimetro da base mede 40 cm e altura 100 cm, armazena um certo lquido, que ocupa 40% de sua capacidade. O volume do lquido contido nesse recipiente , em litros, aproximadamente, igual a: a) 16 b) 18 c) 20 d) 30 e) 40

456

MATEMTICA

FUVEST
F A S E

1. Que nmero deve ser somado ao numerador e ao denominador da frao tenha um aumento de 20%? a) 1 b) 2 x + 4z = 7 x 3y = 8 y+z=1

2 para que ela 3

c) 3

d) 4

e) 5

2.

Ento, x + y + z igual a a) 2 b) 1

c) 0

d) 1

e) 2

3. Durante uma viagem choveu 5 vezes. A chuva caia pela manh ou tarde, nunca o dia todo. Houve 6 manhs e 3 tardes sem chuva. Quantos dias durou a viagem? a) 6 b) 7 c) 8 d) 9 e) 10

4. No retngulo abaixo, o valor, em graus, de a + b a) 50 b) 90 c) 120 d) 130 e) 220

5. As retas r e s so perpendiculares e interceptam-se no ponto (2,4). A reta s passa pelo ponto (0,5). Uma equao da reta r a) 2y + x = 10 b) y = x + 2 c) 2y x = 6 d) 2x + y = 8 e) y = 2x

6. Numa primeira fase de um campeonato de xadrez cada jogador joga uma vez contra todos os demais. Nessa fase foram realizados 78 jogos. Quantos eram os jogadores? a) 10 b) 11 c) 12 d) 13 e) 14
3

7. O volume de um paraleleppedo reto retngulo 240 cm . As reas de duas de suas faces so 2 2 2 30 cm e 48 cm . A rea total do paraleleppedo, em cm , a) 96 b) 118 c) 236 d) 240 e) 472

457

MATEMTICA
8. Uma formiga resolveu andar de um vrtice a outro do prisma reto de bases triangulares ABC e DEG, seguindo um trajeto especial. Ela partiu do vrtice G, percorreu toda a aresta perpendicular base ABC, para em seguida caminhar toda a diagonal da face ADGC e, finalmente, completou seu passeio percorrendo a aresta reversa a CG. A formiga chegou ao vrtice a) A b) B c) C
2

FUVEST
F A S E

d) D

e) E

9. Para que a parbola y = 2x + mx + 5 no intercepte a reta y = 3, devemos ter a) 4 < m < 4 b) m < 3 ou m > 4 c) m > 5 ou m < 5

d) m = 5 ou m = 5 e) m 0

10. O conjunto das razes da equao log10(x ) = (log10x) a) {1} b) {1, 100} c) {10, 100} d) {1, 10} e) {x R | x > 0}

11. Na figura abaixo, AD = 2 cm, AB = 3 cm, a medida do ngulo BAC 30 e BD = DC , onde D ponto do lado AC. A medida do lado BC, em cm, a) b) 2 c) 5 3 d) e) 6 7

12. No papel quadriculado da figura abaixo, adota-se como unidade de comprimento o lado do quadrado hachurado. DE paralelo a BC. Para que a rea do tringulo ADE seja a metade da rea do tringulo ABC, a medida de AD, na unidade adotada,

a) 4 2 b) 4 c) 3 3 d) e) 8 3 3 7 3 2

458

MATEMTICA

FUVEST
F A S E

13. O valor, em reais, de uma pedra semipreciosa sempre numericamente igual ao quadrado de sua massa, em gramas. Infelizmente uma dessas pedras, de 8 gramas, caiu e se partiu em dois pedaos. O prejuzo foi o maior possvel. Em relao ao valor original, o prejuzo foi de a) 92% b) 80% c) 50% d) 20% e) 18%

14. O menor nmero natural n, diferente de zero, que torna o produto de 3888 por n um cubo perfeito a) 6 b) 12 c) 15 d) 18 e) 24

15. A, B, C e D so vrtices consecutivos de um hexgono regular. A medida, em graus, de um dos ngulos formados pelas diagonais AC e BD a) 90 b) 100 c) 110
2

d) 12

e) 150

16. Sendo i a unidade imaginria (i = 1) pergunta-se: quantos nmeros reais a existem para os 4 quais (a + i) um nmero real? a) 1 b) 2 c) 3 d) 4 e) infinitos

17. Sendo sen = a) sen < sen b) sen

9 , com 0 < < , tem-se 2 10

< sen 2 3

d) sen 2 < sen

< sen 3 3

< sen < sen 2 3 3

e) sen 2 < sen < sen

c) sen < sen 2 < sen

18. Um cubo de aresta m est inscrito em uma semi-esfera de raio R de tal modo que os vrtices de uma das faces pertencem ao plano equatorial da semi-esfera e os demais vrtices pertencem superfcie da semi-esfera. Ento, m igual a
a) R b) R 2 3

2 2 3 c) R 3 d) R e) R 3 2

459

MATEMTICA

FUVEST
F A S E

19. Na figura ao lado, A um ponto do plano cartesiano, com coordenadas (x, y). Sabendo que A est localizado abaixo da reta r e acima da reta s, tem-se x a) y < e y < x + 1 2 x b) y < ou y > x + 1 2 x c) < y e y > x + 1 2 20. P(x) um polinmio cujas razes formam uma progresso geomtrica de razo 2 e primeiro termo 2. O coeficiente do termo de mais alto grau de P(x) 1 e o termo independente igual a 221. O grau do polinmio a) 4 b) 5 c) 6 d) 7 e)8 x d) x + 1 < y < 2 x e) < y < x + 1 2

1. Considere a funo f dada por 12 x + 5 x+1 f(x) = 5 x+9 x+1 x a) Determine o domnio de f. b) Resolva a inequao f(x) > 0. 2. Os trabalhos da diretoria de um clube so realizados por seis comisses. Cada diretor participa exatamente de duas comisses e cada duas comisses tm exatamente um diretor comum. a) Quantos diretores tem o clube? b) Escolhendo-se, ao acaso, dois diretores, qual a probabilidade de que eles sejam de uma mesma comisso? 3. Considere um tringulo ABC tal que a altura BH seja interna ao tringulo e os ngulos BH e HBC sejam congruentes. a) Determine a medida do ngulo ABC. b) Calcule a medida de AC, sabendo que AB = 4 cm e a razo entre as reas dos tringulos ABH e BCH igual a 2.

F A S E

460

MATEMTICA
4. Seja o sistema

FUVEST
F A S E

x + 2y z = 0 x my 3z = 0 x + 3y + mz = m

a) Determine todos os valores de m para os quais o sistema admite soluo. b) Resolva o sistema, supondo m = 0.

5. No paraleleppedo reto retngulo mostrado na figura, AB = 2 cm e AD = AE = 1 cm. AB Seja X um ponto de AB e x a medida de AX. a) Para que valor de x, CX = XH? b) Para que valor de x, CXH reto?

6. ABC um tringulo retngulo em A e CX bissetriz do ngulo BCA, onde X ponto do lado AB. A medida de CX 4 cm e a de BC, 24 cm. Calcule a medida de AC.

7. Considere as circunferncias que passam pelos pontos (0,0) e (2,0) e que so tangentes reta y = x + 2. a) Determine as coordenadas dos centros dessas circunferncias. b) Determine os raios dessas circunferncias. 8. Suponha que o polinmio do 3 grau P(x) = x + x + mx + n, onde m e n so nmeros reais, seja divisvel por x 1. a) Determine n em funo de m. b) Determine m para que P(x) admita raiz dupla diferente de 1. c) Que condies m deve satisfazer para que P(x) admita trs razes reais e distintas? 9. Do conjunto de todos os nmeros naturais n, n 200, retiram-se os mltiplos de 5 e, em seguida, os mltiplos de 6. Calcule a soma dos nmeros que permanecem no conjunto. 10. a) Dados AB e um segmento de medida r, construa, usando rgua e compasso, um tringulo issceles sabendo que sua base AB e o raio da circunferncia inscrita nesse tringulo r. b) Descreva as construes feitas. r c) Justifique o porqu de cada construo A B
3 2

461

MATEMTICA
SMBOLO R R
* +

UFBA
SIGNIFICAO Conjunto dos nmeros reais Conjunto dos nmeros reais estritamente positivos Qualquer que seja; para todo {x R, a < x b} Unidade de comprimento

F A S E

]a,b] u. c.

QUESTES DE 1 a 8 INSTRUO:Assinale as proposies verdadeiras, some os nmeros a elas associadas e marque o resultado na Folha de Respostas. QUESTO 1 Numa escola, 45% dos estudantes so crianas, 35% so adolescentes, e os 600 alunos restantes so adultos. Nessas condies, pode-se afirmar: (1) A escola possui um total de 3000 alunos. (2) O nmero de adolescentes de 1250. (4) 270 crianas, menores de 5 anos, correspondem a 25% do total de crianas. (8) O percentual do nmero de adolescentes, em relao ao nmero de adultos, de 175%. (16) 420 adolescentes estudam tarde e correspondem a 2 do total de adolescentes. 5

462

MATEMTICA
QUESTO 2

UFBA
F A S E

Considerando-se as taxas de natalidade e mortalidade, a populao da cidade A apresenta crescimento de 3% ao ano, a populao da cidade B aumenta, a cada ano, 2000 habitantes em relao ao ano anterior. Se, em 1970, a populao da cidade A era de 200000 habitantes e a populao da cidade B era de 220000 habitantes, pode-se afirmar: (1) Em 1972, a populao de A era maior que a populao de B. (2) Em 1980, a populao de A cresceu aproximadamente 6 x 10 x (1,03) habitantes.
11 2 x 10 (4) Em 1980, a populao de A era aproximadamente [1 - (0,03) ] habitantes. 0, 03
5

(8) Em 1990, a populao de B era de 2,6 x 10 habitantes. (16) O grfico ao lado apresenta a variao da populao de B em relao ao tempo t, a partir de 1970.

QUESTO 3 Sobre equaes e inequaes, verdade que: (1) O produto de razes da equao (x - 6x + 8) (x + 3) = 0 -24 (2) A soluo da inequao x - 3x + 2 0 o conjunto (-, 1] [2, + ). (4) A equao |x| = -x no tem soluo. (8) Se |x + 2| < 1, ento x ] -3, -1[ (16) Se 2
x +y 2 2

= 16, ento x + y = 10

2 (32) Se log 1 (x + 1) < 1 ent o 1 < x < 3 3

463

MATEMTICA
QUESTO 4 Sobre funes reais, verdade que: (1) A funo f(x) = 1 - x decrescente.
1 2 (2) A funo f(x) = x - x + 1 tem valor mximo em x = 2 (4) Se f(x - 1) = x + 1, ento f(3) = 5

UFBA
F A S E

(8) Se f(x) = x - 1 e g(x) = x , ento gof(x) = x - 1 (16) Se f a funo inversa de g, ento f(Lg (x)) = x (34) Se f: R R uma funo crescente, ento f(3) > f(4) (64) Se f uma funo mpar e f(1) = 2, ento f(-1) = -2 QUESTO 5 Sobre exponenciais e logaritmos, verdade que: (1) log 0,1 + log
2

4=0

(2) O domnio da funo f(x) = log (1 -x) o conjunto R*


+

ab 2 * (4) log = log a + 2 log b log c, a, b, c R + c

(8) f(x) = (m - 1) uma funo exponencial crescente, para m > 2. (16) O grfico da funo f(x) = log2 (x + 2) intercepta o eixo OY no ponto (0,1).

(32) O grfico ao lado representa a 2 funo y = 1 + 2 .

464

MATEMTICA
QUESTO 6 Sendo P(x) = (m - 1) x + x + x - 1 um polinmio de grau 2 e 3 2 Q(x) = kx + x + 2x + 2 um polinmio que tem -1 como raiz. pode-se afirmar: (1) km = 1 (2) P(x) . Q(x) um polinmio de grau 6. (4) P(x) tem duas razes reais. (8) xP(x) - Q(x) = 2 + 3x 2 (16) O quociente da diviso de Q(x) por x + 1 x + 2 (32) O resto da diviso de Q(x) por P(x) 3x + 2 QUESTO 7 No semicrculo representado ao lado, considerem-se os tringulos retngulos CMO e MHO, sendo BM = 5 cm e AM = 3 cm. Nessas condies, pode-se afirmar: (1) OC = 3 cm (2) CM = 15 cm (4) O permetro do tringulo AMC igual a (5 6 + (8) A rea do tringulo MHO igual a
3 2

UFBA
F A S E

15 ) cm

15 2 cm . 2 2 (16) A rea do crculo de centro em O e raio OB igual a 16 cm .


(32) AB2 = AC 2 + CB2 QUESTO 8 Considerando-se, no sistema cartesiano XOY, o ponto P(-1, 1) e as retas r: y = 2x + 2 e s: x + 2y - 3 = 0, verdade: (1) O simtrico de P, em relao ao eixo OY, o ponto Q(-1, -1). (2) P s (4) A distncia de P origem de

2 u.c.

(8) As retas r e s so perpendiculares. (16) A reta que passa por P e paralela a r tem equao 2x - y + 1 = 0 (32) A reta r intercepta o eixo OX no ponto (-1,0)

465

MATEMTICA
QUESTES 9 E 10

UFBA
F A S E

INSTRUO: Efetue os clculos necessrios e marque o resultado na Folha de Respostas:

Questo 9 Dispondo-se de abacaxi, acerola, goiaba, laranja, ma, mamo e melo, calcule de quantos sabores diferentes pode-se preparar um suco, usando-se trs frutas distintas.

1 2 A

Questo 10 Uma estrada eleva-se 0,5m a cada com a horizontal. SMBOLO R ]a,b]
z

3 m . Calcule, em graus, o ngulo de inclinao da estrada 2


SIGNIFICAO Conjunto dos nmeros reais Qualquer que seja; para todo {x R, a < x b} Conjugado do complexo z Unidade de comprimento Unidade de rea Unidade de volume P R O V A

F A S E

u.c. u.a. u.v.

466

MATEMTICA
QUESTES DE 1 A 7

UFBA
F A S E

INSTRUO: Assinale as proposies verdadeiras, some os nmeros a elas associados e marque o resultado na Folha de Respostas. Questo 1 Sabendo-se que o complexo z1 =
6

2 A

x +1 , x R, um imaginrio puro e 1 + xi

z2 =

2 ( cos + i sen ) pode-se afirmar: 9 9


10 1

P R O V A

(1) z

. z2 = 1 + i 3
3 1 3 1 +i e +i 2 2 2 2

(2) As razes cbicas de z so -i,


1

(4) O simtrico de z2 em relao origem tem argumento igual a


1

. 3 2 2 (8) O lugar geomtrico dos complexos z tais que z - z = 1 tem equao x + y - 2x - 2y = 1.


2

(16) Os afixos das razes sextas de z formam um polgono de rea igual a 6 3 u.a. Questo 2

3 2 Sendo A = 3A , pode-se afirmar: n n


(1) Se
n

m! = 56, ento m > n. ( m 2)!

(2) P = 60 (4) C
3 2 = C n n

2 1 n (8) O termo independente de x, no desenvolvimento x 3 , igual a 10. x


(16) Com os elementos do conjunto A = {x N; 0 < x 9} , podem-se formar 84 produtos distintos com n algarismos. (32) Com 7 pessoas, podem-se tirar 2520 fotografias diferentes de n pessoas em fila.

467

MATEMTICA
QUESTO 3
x 2 x; se x < 0 Considerando-se a funo real f(x) = 2 1; se 0 x < 2, pode se afirmar : 9 3x; se x 2
1 2

UFBA
F A S E

2 A

(1) f assume valor mximo em x =

(2) A imagem de f o intervalo ] , 3] 1 , 2 (4) A funo f decresce no intervalo 2 (8) A reta y =

P R O V A

1 intercepta o grfico de f em quatro pontos. 8

(16) Se x > 4, ento f(x) < -3

(32) O grfico de f(|x|)

QUESTO 4 Sobre polimnios e equaes polinomiais, pode-se afirmar: (1) Se P(x) = x + 2x + ax + b divisvel por (x - 1) , ento ab = -28. (2) Se 0 raiz tripla e i raiz simples do polimnio P(x) = x + ax + bx + cx + dx + e de coeficientes reais, ento a + b + c + d + e = 0. (4) Se o polinmio P(x) tem coeficientes reais e admite 1, 1 + i e i como razes, ento P(x) tem, no mnimo, grau 3. (8) O polimnio P(x) = x + 2x + 3x + 1 no possui razes racionais. (16) A soma das razes da equao 2x - 6x + x + x + 1 igual a 3.
4 3 2 3 2 5 4 3 2 3 2 2

468

MATEMTICA
QUESTO 5
x + 2y + az = 0 Considere-se o sistema bx + 3z = 1 x + 3y 2z = 2

UFBA
F A S E

2 A

e sejam:

A: a matriz incompleta formada pelos coeficientes das incgnitas; B: a matriz completa associada ao sistema; C: a matriz dos termos independentes. P R O V A

Nessas condies, pode-se afirmar: (1) Sendo a = 1 e b = 2, A uma matriz simtrica. (2) Se a = b = -1, ento o determinante de A igual a -5.
1 2 (4) A matriz transporta de B a 0 b 1 0 3 3 2 1 2

(8) Para a = b = -1, a soma dos elementos da 3 coluna da matriz inversa de A igual a
2 ( a + 1) 6 (16) A.C. = 7

3 . 2

(32) Se S = (m, n, p) soluo do sistema para a = b = -1, ento m + n + p =

19 . 4

QUESTO 6 Considerando-se o trapzio retngulo representado na figura ao lado, pode-se afirmar: (1) A rea do trapzio 3,075 u.a. (2) O comprimento da circunferncia circunscrita ao tringulo BCD 1,5 u.c. (4) O volume do slido que se obtm, quando o trapzio d uma rotao completa em torno de AC, 2,736 u.v. (8) A rea lateral do slido que se obtm, quando apenas o retngulo BCD d uma rotao completa em torno de ED, 3,84 u.a. (16) A rea total do slido que se obtm, quando apenas o tringulo ABDE d uma rotao completa em torno de BD, 1,35 u.a. (32) O volume do prisma reto de base ABDE e altura igual a 2AC 9,6 u.v. (64) A diagonal do cubo de aresta AB 0,8

2 u.c.

469

MATEMTICA
QUESTO 7 Com base na trigonometria, verdade: (1) Se, no tringulo acutngulo ABC, sen =
2 3 e sen B = , ento sen C = 2 2

UFBA
F A S E

6+ 2 4

2 A

(2) Se, num paralelogramo, dois lados formam um ngulo de 120 e medem 6 cm e 8 cm, ento a diagonal maior mede 2 13 cm. (4) Na figura ao lado, a rea do tringulo ABC igual a 2 3 u.a.

P R O V A

(8) Na figura ao lado, tg =

6 . 17

(16) Na figura ao lado, sendo OA = 4 u.c. o raio da circunferncia e BC = 3 u.c., tem-se cos =
55 8

470

MATEMTICA
QUESTO 8 Considerando-se a funo f(x) = sen2x, pode-se afirmar: (1) f(x + 5) = -f(x), x R

UFBA
F A S E

2 (2) f(x + ) = 1 - 2sen x, x R 4


(4) f decrescente no intervalo , 0 4 (8) A soma dos valores de x, no intervalo [0, ], que satisfazem a relao f(x) = cosx igual a
3 . 2

2 A

P R O V A

(16) Existem quatro razes da equao f(x) =


3 1 = (32) f arc cos 2 2

2 , no intervalo [0, 2]. 2

x (64) O grfico ao lado representa a funo h(x) = 1 - 2f em [-2 , 2]. 2 4 QUESTES 9 E 10

INSTRUES: Efetue os clculos necessrios e marque o resultado na Folha de Respostas.

Questo 9 Sabendo-se que a e b, sendo a < b, so as razes da equao log (17 - 2


2 x +2

) = 2 - x, determine ab.

Questo 10 Na figura ao lado, uma circunferncia cujo arco de 45 mede 5 u.c.; ABC um tringulo equiltero; M ponto mdio de BC; CP // AB e MP // AC. Sendo MP = a 3 u.c., calcule a.

471

MATEMTICA
QUESTO DISCURSIVA Na figura ao lado, esto representadas uma parbola,
5 9 com vrtice no ponto , , e uma reta que forma 2 4 um ngulo obtuso com o eixo OX tal que 2 5 2 2 . Considerem-se x + y + ax + by + c = 0 5 a equao da circunferncia C, de centro em P1 e que passa por P2, e Ax + By + C = 0, A > 0, a equao da reta tangente a C, em P2.

UFBA
F A S E

2 A

sen =

P R O V A

Determine K = |a + b + c + A + B + C|, indicando, de modo completo, toda a resoluo do problema.

472

MATEMTICA

UFMG
F A S E

1. Durante o perodo de exibio de um filme, foram vendidos 2000 bilhetes, e a arrecadao foi de R$ 7.600,00. O preo do bilhete para adulto era de R$ 5,00 e, para criana, era de R$ 3,00. A razo entre o nmero de crianas e o de adultos que assistiram ao filme nesse perodo foi a) 1
3 b) 2

c) 8 5

d) 2

1 A

2. A diferena dos cubos de dois nmeros naturais consecutivos 91. Esses nmeros pertencem a a) {n IN: b) {n IN: 3 7 n n 3} 7} c) {n IN: 7 d) {n IN: n n 10} 10}

P R O V A

3. Uma criao de coelhos foi iniciada h extremamente um ano e, durante esse perodo, o nmero de coelhos duplicou a cada 4 meses. Hoje, parte dessa criao dever ser vendida para se ficar com a quantidade inicial de coelhos. Para que isso ocorra, a porcentagem da populao atual dessa criao de coelhos a ser vendida a) 75% b) 80% c) 83,33% d) 87,5%

4. A base de uma caixa retangular tem dimenses 2 cm e 3 cm. Colocam-se 21,6 gramas de um certo lquido nessa caixa. Se cada 0,9 grama desse lquido ocupa 1 cm3, o nvel do lquido na caixa a) 3,5 cm b) 4 cm c) 4,5 cm d) 5 cm

5. Seja m

2 1 3 1 1 5 4

O valor de m igual a
2 a) 15 4 b) 15 5 c) 9

d) 10 9

6. Em um treinamento numa pista circular, um ciclista gasta 21 minutos para completar cada volta, passando sempre pelos pontos A, B e C da pista e nessa ordem. Em cada volta, nos trechos entre A e B e entre B e C, ele gasta, respectivamente, o dobro e o triplo do tempo gasto no trecho entre C e A. Se esse ciclista passou por B s 16 horas, s 18 horas ele estar a) entre A e B. b) entre B e C. c) entre C e A. d) em A.

473

MATEMTICA
7. Observe este quadro. Posto A Carro 1 Carro 2 12 horas 12 horas e 15 minutos Posto B 13 horas 13 horas

UFMG
F A S E

1 A

Nesse quadro, esto registrados os horrios em que os carros 1 e 2, participantes de um rallye, passaram pelos postos A e B, em direo ao posto C. Os dois carros mantiveram constantes suas velocidades no percurso de A para C, e o mais veloz nesse percurso passou por C s 15 horas. O outro carro passou por C s a) 15 horas e 15 minutos. b) 15 horas e 20 minutos. c) 15 horas e 30 minutos. d) 15 horas e 40 minutos.

P R O V A

8. Considere o conjunto M

{n IN: 1

500}.

O nmero de elementos de M que no so mltiplos de 3 nem de 5 a) 234 b) 266 c) 267 d) 467

9. Observe a figura.

Nela, AB

8, BC

12 e BFDE um losango inscrito no tringulo ABC.

A medida do lado do losango a) 4 b) 4,8 c) 5 d) 5,2

474

MATEMTICA
10. Observe a figura.

UFMG
F A S E

Suponha que as medidas dos ngulos PSQ , QSR , SPR , assinalados na figura, sejam 45, 18 e 38, respectivamente. A medida do ngulo PQS , em graus,
a) 38 b) 63 c) 79 d) 87

1 A

11. Observe a figura. Se a medida de CE 80, o comprimento de BC a) 20 b) 10 c) 8 d) 5

P R O V A

12. Observe a figura. Nela, a circunferncia maior C tem raio 2, e cada uma das circunferncias menores, C1, C2, C3 e C4, tangente a C e a um lado do quadrado inscrito. Os centros de C1, C2, C3 e C4 esto em dimetros de C perpendiculares a lados do quadrado. A soma das reas limitadas por essas quatro cincunferncias menores a) 8 3 2 2 b) 3 2 2

c) 3 2 2 d) 2 3 2 2

13. Uma pirmide regular tem altura 6 e lado da base quadrada igual a 4. Ela deve ser cortada por um plano paralelo base, a uma distncia d dessa base, de forma a determinar dois slidos de mesmo volume. A distncia d deve ser a) 6

33 2

b) 3

33 4 2

c) 6

33 4

d) 6

23 2

475

MATEMTICA
14. O ponto de coordenadas (3, 4) pertence parbola de equao y A abscissa do vrtice dessa parbola a) 1 2 b) 1
3 c) 2

UFMG
ax2 bx 4.

d) 2

F A S E

1 A

15. Um terreno retangular, com rea de 800 m2 e frente maior que a lateral, foi cercado com um muro. O custo da obra era R$ 12,00 por metro linear construdo na frente, e de R$ 8,00 por metro linear construdo nas laterais e no fundo. Se foram gastos R$ 1.040,00 para cercar o terreno, o comprimento total do muro construdo, em metros, a) 114 b) 120 c) 132 ax d) 180 2 tal que f(f(1)) 3 P R O V A

16. Para um nmero real fixo a, a funo f(x) O valor de a a) 1 b) 2

c) 3

d) 4

17. O conjunto de todos os valores reais de x que satisfazem a equao 2 log10 x a) { 1, 11} 18. Observe a figura. Nessa figura, est representado o grfico de f(x) sendo k e a constante positivas. O valor de f(2)
3 a) 8

log10 x c) {10}

11 10 d) {11}

b) {5, 6}

kax,

b) 1 2

3 c) 4

d) 1

19. O lado BC de um ngulo reto ABC est sobre a reta de equao x coordenadas (2, 4) pertence reta que contm o lado BA. A equao da reta que contm o lado BA a) 4x 2y 5 0 b) x 2y 6 0 c) x 2y d) 2x y 10 0 8 0

2y

0, e o ponto de

476

MATEMTICA
20. Observe a figura. Nela, esto representadas as retas de equaes y ax b e y cx d. A alternativa que melhor representa o grfico de y (ax b) (cx d)

UFMG
F A S E

1 A

P R O V A a) c)

b)

d)

1. Uma conta de R$ 140,00 paga em cdulas de R$ 5,00 e R$ 10,00, num total de 18 cdulas. O nmero n de cdulas de R$ 5,00 usadas para o pagamento dessa conta tal que a) n b) 5 5 n 7 c) 7 d) n n 10 10

F A S E

1 B

2. Define-se a mdia aritmtica de n nmeros dados como o resultado da diviso por n da soma dos n nmeros dados. Sabe-se que 3,6 a mdia aritmtica de 2,7; 1,4; 5,2 e x. O nmero x igual a a) 2,325 b) 3,1 c) 3,6 d) 5,1 P R O V A

3. A diferena entre os quadrados de dois nmeros naturais 144, e a razo entre eles 3 . 5 A soma desses dois nmeros naturais a) 16 b) 24 c) 30 d) 34

477

MATEMTICA
3

UFMG
F A S E
2 3

4. Seja y

1 4

7
2

2 2
2

O valor de y igual a a)
8 3

b)

c) 1 2

1 B

d) 2

5. Num depsito, esto guardados 12 pacotes de 200 kg, 14 de 100 kg, 20 de 60 kg e 12 de 20 kg. Uma mquina, usada para transportar esses pacotes de um depsito para outro, carrega um por vez e gasta, para transportar cada um dos pacotes de 200 kg, 100 kg, 60 kg e 20 kg, respectivamente, 15 min, 10 min, 8 min e 8 min. O transporte feito levando-se sempre os mais pesados em primeiro lugar. Suponha que a mquina iniciou o transporte desses pacotes s 10 horas e s o interrompeu s 17 horas e 20 minutos. O nmero de pacotes transportados nesse perodo, por essa mquina, foi a) 20 b) 28 c) 41 d) 58

P R O V A

6. No ano passado, uma equipe de 13 professores, com um ritmo de trabalho suposto constante, corrigiu 3000 provas em 6 dias. Este ano, o nmero de provas aumentou para 5500 e a equipe foi ampliada para 15 professores. Para se obter uma estimativa do nmero n de dias necessrios para totalizar a correo, suponha que, durante todo o perodo de correo, o ritmo de trabalho da equipe deste ano ser o mesmo da equipe do ano passado. O nmero n satisfaz a condio a) n 8 c) 10 n b) 8 d) n n 10 12

12

478

MATEMTICA
7. Observe a tabela abaixo. Tabela do Imposto de Renda num determinado ms Rendimento para base de clculo do ms (R$) At 900,00 Acima de 900,00 at 1.800,00 Acima de 1.800,00 Alquota (%) 15 25 Parcela a deduzir (R$) Isento 135,00 315,00

UFMG
F A S E

1 B

P R O V A

Essa tabela utilizada para calcular o imposto de renda a ser pago Receita Federal por um trabalhador assalariado no ms em questo. Para se obter o rendimento para base de clculo, deve-se subtrair de seu rendimento bruto todas as dedues a que ele tem direito. Ao rendimento para base de clculo aplica-se a alquota correspondente e, em seguida, subtraise a parcela a deduzir, tambm correspondente, de acordo com a tabela, obtendo-se assim o valor do imposto de renda a ser pago. Nesse ms, um trabalhador, cujo rendimento bruto foi de R$ 2.000,00, teve direito somente s seguintes dedues: R$ 90,00 por dependente e R$ 200,00 pagos Previdncia. Nessas condies, sabendo-se que o valor do imposto pago por esse trabalhador, nesse ms, foi de R$ 108,00, o nmero de dependentes considerado foi a) 0 b) 1 c) 2 d) maior que 2

8. O nmero de mltiplos de 10, compreendidos entre 100 e 9999 e com todos os algarismos distintos, : a) 250 b) 321 c) 504 d) 576

9. Observe a figura. Nela, a, 2a, b, 2b e x representam as medidas, em graus, dos ngulos assinalados. O valor de x, em graus, a) 100 b) 110 c) 115 d) 120

479

MATEMTICA
10. Observe a figura. Nessa figura, ABCD representa um quadrado de lado 11 e AP AS CR CQ. O permetro do quadriltero PQRS a) 11 3 b) 22 3 11. Observe a figura. Nela, a circunferncia de centro O tem raio r e arcos AB, BC, CD, DE, EF, FG, GH e HA congruentes. O valor da rea sombreada, em funo de r, a) r2 ( 2) b) 2r2 ( 1) c) 2r2 d) r2 ( 1) c) 11 2 d) 22 2

UFMG
F A S E

1 B

P R O V A

12. Observe a figura. Nessa figura, tem-se: AB AC 6, BC BD


4 e CBQ QBD .

A tangente do ngulo CBQ


a)
2 4

b)

2 2

c) 1

2 2

d)

2 2

13. Observe a figura. Um prisma reto de base pentagonal foi desdobrado obtendo-se essa figura, na qual as linhas pontilhadas indicam as dobras. O volume desse prisma a) 6

9 3 4

c) 30 d) 30

9 3 4 45 3 4

c) 45 3 4

480

MATEMTICA

UFMG
F A S E

14. Um certo reservatrio, contendo 72 m3 de gua, deve ser drenado para limpeza. Decorridas t horas aps o incio da drenagem, o volume de gua que saiu do reservatrio, em m3, dado por V(t) 24t 2t2. Sabendo-se que a drenagem teve incio s 10 horas, o reservatrio estar completamente vazio s a) 14 horas. b) 16 horas. c) 19 horas. d) 22 horas.

1 B

15. Num cinema, ingressos so vendidos a R$ 10,00 para adultos e a R$ 5,00 para crianas. Num domingo, na sesso da tarde, o nmero de ingressos vendidos para crianas foi o dobro do nmero vendido para crianas na sesso da noite. A renda da sesso da tarde foi R$ 300,00 a menos que a da noite e, em ambas as sesses, foi vendido o mesmo nmero de ingressos. Nesse domingo, o nmero de ingressos vendidos para crianas, na sesso da noite, foi a) 50 b) 55 c) 60 d) 65

P R O V A

16. Para a funo f(x) O valor de b a) 1

5x

3 e um nmero b, tem-se f(f(b))

2.

b)

4 5

c)

17 25

d)

1 5

17. O valor de x que satisfaz a equao 24x a) 1 x 2 b) 2 x 3

6 (22x) c) 3 x

16 tal que 4 d) 4 x 5

18. Observe a figura. Nessa figura, est representado o grfico de f(x) loga x. O valor de f(128)
5 a) 2 b) 3 7 c) 2 d) 7

481

MATEMTICA
19. Sejam t e s as retas das equaes 2x A reta r contm o ponto A A equao de r a) 5x b) 5x y y 24 26 0 0 c) x d) x 5y 5y 10 0 0 y 3 0 e 3x 2y 1 0

UFMG
F A S E

(5,1) e o ponto de interseo de t e s.

1 B 2

20. Observe a figura. Nessa figura, esto representadas duas retas perpendiculares que so grficos de y f(x) e y g(x). O valor mximo da funo h(x)
5 a) 4

P R O V A

f(x) . g(x)

c) 3 d) 4

b) 9 4

QUESTO 01 Dadas duas funes f e g, pode-se calcular f(g(x)) desde que x pertena ao domnio de g, e g(x) pertena ao domnio de f. a) Sejam f(x) 5 e g(x) x 3. CALCULE g(f(x)) e f(g(x)). b) Sejam f(x)
x 1 3 e g(x) x 18 . 2

F A S E

DETERMINE o conjunto D de todos os valores de x para os quais f(g(x)) pode ser calculado e DETERMINE a expresso de f(g(x)).

QUESTO 02 Considere um retngulo de base 39. Suas dimenses so alteradas de modo que, a cada reduo de x em sua base, com x 0, obtm-se um novo retngulo de rea dada por A(x) 936 15x x2 a) MOSTRE que, para 0 x 39, a altura de cada um desses retngulos pode ser escrita na forma h(x) ax b e DETERMINE a e b. b) Dentre esses retngulos, DETERMINE as dimenses do que tem rea mxima.

482

MATEMTICA
QUESTO 03
log3 a Trs nmeros reais a, b e c satisfazem o sistema a b log3 b c 117 log3 c 9

UFMG
F A S E

Alm disso, eles esto em progresso geomtrica, isto , existe um nmero real r tal que b ar e c br. DETERMINE todos os possveis valores de r e os correspondentes valores de a, b e c. QUESTO 04 Os pontos A (2, 0) e B (0, 4) so extremos de um dimetro da circunferncia C.

a) DETERMINE a equao da circunferncia C. b) Seja r a reta que passa pelo centro da circunferncia C e que perpendicular reta AB. DETERMINE os pontos de interseo de C e r. QUESTO 05 Observe a figura. Nessa figura, ABCD um quadrado de lado a, e F o ponto de tangncia da diagonal BD com a semicircunferncia de centro E. CALCULE o raio da semicircunferncia em funo de a.

QUESTO 06 Seja S um slido que est contido entre dois planos paralelos, e , e que intercepta esses planos. Denote por A(x) a rea da seo obtida pela interseo de S com um plano paralelo a e cuja distncia a seja x. Se para cada x tem-se A(x) ax2 bx c, com 0 x h, em que h a distncia entre e , ento o volume V de S pode ser calculado usando-se a frmula

h A0 6 ( )

4A h 2

()

A( h )

a) DETERMINE a expresso de A(x) na situao em que S um cone circular reto de altura H e raio da base R. b) Usando a frmula dada no texto, CALCULE, em funo de H e R, o volume V do cone S do item a).

483

MATEMTICA
QUESTO 07 Nos tringulos ABC e DEF, AB DE c, AC DF tringulo ABC o dobro da rea do tringulo DEF. CALCULE o valor de cos . QUESTO 08 Observe a figura. Considere os caminhos ligando A a C, passando por B, traados a partir de A, deslocando-se sempre, ou 1 unidade para a direita, na horizontal, ou 1 unidade para cima, na vertical. DETERMINE o nmero total de caminhos distintos obtidos dessa forma. b, BAC , EDF

UFMG
F A S E

2 e a rea do

QUESTO 09 a1x Dado um sistema linear de trs equaes nas trs variveis x, y, z, a 2 x a x 3 associa-se a ele os seguintes determinantes:
a1 a2 a3 b1 b2 b3 c1 c2 ; c3 d1 d2 d3 b1 b2 b3 c1 c2 ; c3 a1 a2 a3 d1 d2 d3 c1 c2 : c3

b1y b2 y b3y

c1z c2z c 3z

d1 d2 d3

a1 a2 a3

b1 b2 b3

d1 d2 d3

Considere o sistema linear de trs equaes nas trs variveis x, y, z em que m e n so nmeros reais:
x y z 1 2x my 2z 4x ny ( m

2 2)z 3

a) RESPONDA quais so todos os possveis valores de m e n que anulam o determinante associado a esse sistema. VERIFIQUE que, para esses valores, os outros trs determinantes associados so nulos, ou seja, x 0, y 0 e z 0. b) Dentre todos os possveis valores de m e n para os quais DETERMINE aqueles que tornam o sistema impossvel. 0, x 0, y 0 e z 0,

484

MATEMTICA
a + b 2b x 3 = fazemos a = 2 e b = , temos a b + x igual a a 2x 2

UFPA
F A S E

1. Se na identidade a) 2

b) 0,5

c) 1,5

d) 3

e) 3,2 Se este retngulo tem

2. As dimenses de um retngulo so indicadas por x 2 e x + 2. 2 12m de rea, seu permetro , em metros, igual a a) 8 b) 10 c) 12 d) 14 e) 16

3. Na figura, o raio OA do crculo vale 6 e o segmento CB perpendicular a OA . A rea do tringulo ABC vale

a) b)

3 3 2
9 2

d)

9 3 2

e) 9 3

c) 3 3 4. Se um, em cada 3 candidatos ao vestibular 95 da UFPA, pediu iseno da taxa de inscrio e 2 dos pedidos foram deferidos, a porcentagem de candidatos isentos da taxa foi de 3 aproximadamente a) 18% b) 22% c) 33% d) 45% e) 66%

5. Uma pessoa fez no ms de julho uma aplicao de R$ 5.000,00 e aps 3 meses verificou que seu capital havia subido para R$ 5.480,00. Assim sendo, concluiu que a taxa mensal desta aplicao foi em mdia igual a a) 3,2% b) 2,7% c) 2,23% d) 1,68% e) 0,96%

485

MATEMTICA
6. Se A = x |x 1 0} e
2

UFPA
F A S E

B = {x | ento A B a) ] - 1, + ) b) [ - 1, + ) c) ] - 1, 1[

9 3
x

27 < 0}

d) [1, + ) e) ]1, + )

7. A parbola abaixo representa graficamente a funo quadrtica y = ax + bx + c

Assim sendo, podemos afirmar que a) a = b = c > 0 b) a > 0, b > 0 e c < 0 c) a > 0, b > 0 e c = 0 d) a > 0, b < 0 e c > 0 e) a > 0, b < 0 e c < 0

8. Mensalmente, pago pela prestao de minha casa 1/5 do meu salrio; metade do resto gasto em alimentao e 1/3 do que sobra coloco na poupana, restando-me ainda R$ 800,00 para gastos diversos. O valor colocado na poupana foi de: a) R$ 800,00 b) R$ 650,00 c) R$ 400,00 d) R$ 250,00 e) R$ 100,00

486

MATEMTICA
1. Seja f(x) =

UFPA
F A S E

x 5 2x , calcule:
c) f (x)

a) O domnio de f(x) b) Os zeros de f(x)


2

cos x cotg( + x) . cotg( x) 2 , 2. Dada a funo y = cos( x).tgx


a) Escreva sua expresso somente em termos de sen x b) Calcule lim
x 0

y x sec x
2 2

3. A reta da equao x + 2 y = 0 intercepta o crculo x + y + 2x + 4y 20 = 0 de centro C, nos pontos A e B. Determine: a) Os pontos A, B e C b) A rea do tringulo ABC 4. Considere a funo y = 2x + 4x + 1 a) Esboce o seu grfico b) Encontre a rea da regio limitada pelo grfico da funo, seu eixo de simetria, e os dois eixos coordenados. 5. Os nmeros log x , log (x + 2) e log 3x, esto em P.A. Encontre: a) O valor de x 12 b) O termo geral do desenvolvimento de (2 + x) 12 c) O valor numrico do terceiro termo do desenvolvimento de (2 + x) , considerando x encontrado no item a) 6. De um pedao quadrado de metal de lado L, corta-se uma pea circular de dimetro mximo, e, desta corta-se outro quadrado de lado mximo. Pergunta-se: a) Qual a relao entre as reas do segundo quadrado e do quadrado original? b) Se prosseguirmos sucessivamente com esses cortes, qual ser a soma das reas das peas cortadas em quadrado? 7. Entre os brindes distribudos num aniversrio de criana, esto um chapu cnico e uma bola. Uma das crianas colocou a bola sobre uma mesa e cobriu-a totalmente com o chapu. Se o 3 volume do chapu 96 cm , seu raio de 6 cm, e ele tangencia a bola, determine: a) A rea da superfcie da bola b) A superfcie lateral do chapu 8. Uma das razes do polinmio P(x) = x + 3x + K 1 i. Determine: a) O valor de K b) As solues da equao 10 sen(x + = | k + 5 2i | no intervalo [0,2 ] 3
3 2

487

MATEMTICA

UFPE
F A S E

1. Em um determinado dia a temperatura de Recife foi registrada no grfico abaixo, como funo do tempo. De acordo com este grfico, qual das afirmaes a seguir incorreta?
C

a) b) c) d) e)

A temperatura a partir das 18 h ficou entre 20C e 25C. A menor temperatura registrada neste dia foi superior a 15C. A temperatura mxima ocorreu antes das 9h. Das 2h at as 6h a temperatura ficou entre 15C e 25C. Entre 12h e 16h a temperatura ficou sempre acima dos 20C.

2. Um investidor decidiu aplicar certa quantia em aes de uma empresa. Aps um ms o valor destas aes subiu 5%. No segundo ms subiu 10% e no terceiro ms caiu 5%. A percentagem de ganho do investidor nestes trs meses foi: a) Maior do que 12% b) Entre 10% e 12% c) Igual a 10% d) Entre 8% e 10 e) Abaixo de 8%

3. Seja f: {1, 2, 3, 4, 5} {1, 2, 3, 4, 5} uma funo injetiva, satisfazendo f(1), f(2) {1, 2} f(3) {2, 4} f(4) {1, 4, 5}. Ento f(5) igual a: a) 1 b) 2 c) 3 d) 4 e) 5

4. O menor lado de um retngulo mede 20cm. Se uma diagonal deste retngulo forma um ngulo de 30 com um dos lados, quanto mede o maior lado deste retngulo? a) 30 cm b) 20 2 cm c) 20 3 cm d) 40 cm e) 30 2 cm

488

MATEMTICA

UFPE
F A S E

5. Em um certo pas o imposto (I) calculado em termos de renda (R) do cidado utilizando a equao I t R D onde a taxa (t) e a deduo (D) dependem da renda de acordo com a tabela abaixo:
renda (R) at 10.000 entre 10.000 e 40.000 a partir de 40.000 taxa (t) 0 0,2 0,4 deduo (D) 0 2.000 10.000

Qual dos grficos a seguir melhor expressa o imposto como funo de renda? Obs.: Os valores da renda e imposto nos grficos foram divididos por 1.000.

6. Sabendo que o polinmio X2 a) 1 b) 2

1 divide X4 c) 3

aX2 d) 0

b, assinale o valor de a e) 4

b:

489

MATEMTICA
7. Dados nmeros reais X e Y tais que
0 X Y 1 2

UFPE
F A S E
d) X
X

incorreto afirmar que: 1 a) y 1 X

b) X c) log2 X

Y
1

e) cos X

sen X

8. Numa cidade de 10.000 habitantes so consumidas cervejas de dois tipos A e B. Sabendo que 45% da populao tomam da cerveja A, 15% tomam os dois tipos de cerveja e 20% no tomam cerveja, quantos so os habitantes que tomam da cerveja B? a) 3.500 b) 5.000 c) 4.000 d) 4.500 e) 2.000

9. Qual das alternativas abaixo no verdadeira para algum nmero real t diferente de 0 e 2?

1 t2 a) t b) 4

1 4 2 t2

4 4t

t2
2

d) e)

(2

4t 2 t 2 2 4 c) 2 t 4t ( t 2)

t 2 4 t2 2 4t ( t 2)

t ) (2 t ) 4t ( t 2)
2

2 t 4t 2

2 t 4t 2

1 2t

(2

t ) (2 t ) 4t 2 ( t 2)

10. Qual a rea da regio, no plano cartesiano, determinada pelas seguintes desigualdades: Y 0 X Y 10 3X Y 6 a) 24 b) 30 c) 31 d) 35 e) 60

490

MATEMTICA

UFPE
F A S E

11. O que se conhece sobre a vida do maior algebrista grego, Diofanto, o problema a seguir:
Deus lhe deu um sexto da vida como infante. Somando uma duodcima parte a isto, cobriu-lhe as faces de barba abundante. E ainda uma stima parte antes do casamento. Cinco anos aps nasce-lhe vigoroso rebento. Lstima! Infeliz criana tardia. Depois de chegar metade da vida de seu pai o Destino frio o levou. Quatro anos mais de estudos consolam-no do pesar, para ento, deixando a terra, tambm ele, sua vida terminar.

Quantos anos viveu Diofanto? a) 72 b) 80 c) 86 d) 84 e) 78

12. Numa turma, com igual nmero de moas e rapazes, foi aplicada uma prova de matemtica. A mdia aritmtica das notas das moas foi 9,2 e a dos rapazes foi 8,8. Qual a mdia aritmtica de toda a turma nesta prova? a) 7 b) 8,9 c) 9 d) 9,1 e) 9,2

13. Considerando a, b, c e d nmeros reais positivos com b 1, assinale a alternativa incorreta: a) Se d ac ento logb d c/d ento ad logb a b c. logb c.

b) Se logb a d) Se b e) Se a2

c) Se a, b so racionais ento logb a racional. ad ento 1/d logb a. b3 ento logb a racional.

14. Um fazendeiro dispe de um terreno dividido em regies, como na figura abaixo, e pretende cultiv-las de forma que as regies com uma fronteira comum tenham plantios diferentes. De quantas formas ele pode fazer o plantio se pode optar entre milho, feijo, soja, arroz e trigo para cultivar? Scan com nmero errado (279)

a) 120

b) 24

c) 48

d) 64

e) 60

491

MATEMTICA

UFPE
F A S E

15. Dois planos distintos no espao ou so paralelos ou tm uma reta como interseo. Sobre as posies relativas de trs planos quaisquer , , no espao incorreto afirmar que: a) b) c) d) e) e podem ser paralelos e ambos perpendiculares a . , e podem ser retas paralelas. pode ser uma reta. pode ser um ponto. Se e no so vazios ento no vazio. Y2 1 e paralelas

16. Determine as equaes das retas tangentes circunferncia de equao X2 reta de equao X Y 0. a) Y b) Y c) Y d) Y e) Y 1 X X X X 1 2 2 2 e Y e Y e Y X X X 1 X 1 2 2 2 2 2

2 e Y 2 X e Y X

1. Um nutricionista pretende misturar trs tipos de alimentos (A, B e C) de forma que a mistura resultante contenha 3.600 unidades de vitaminas, 2.500 unidades de minerais e 2.700 unidades de gordura. As unidades por grama de vitaminas, minerais e gorduras dos alimentos constam da tabela abaixo:

F A S E

2 A

Quantas gramas do alimento C devem compor a mistura?

P R O V A

2. Uma loja de departamentos vende uma camisa por R$ 20,00 e 100 unidades desta camisa por ms. Observou-se que para cada real de desconto no pro da camisa as vendas aumentaram em 10 unidades por ms. Quanto deve ser o desconto em reais, de modo a se obter um faturamento mensal mximo na venda deste modelo de camisa?

3. Um fabricante de cadeiras produz os modelos A e B que lhe garantem um lucro por unidade de R$ 40,00 e R$ 50,00 respectivamente. A produo mensal do modelo A de no mximo 200 unidades e a do modelo B de 150 unidades. A produo mensal total dos modelos A e B no pode ultrapassar 300 unidades. Assinale a metade do nmero de modelos do tipo A, a serem fabricados mensalmente, de forma a maximizar o lucro.

492

MATEMTICA
4. Abaixo encontra-se a planta baixa de uma fazenda, situada em uma plancie:

UFPE
F A S E

2 A

O dono deseja cerc-la e o custo do metro de cerca de R$ 10,00. Quantos mil reais o dono ir gastar com a cerca?

P R O V A

5. Sabendo-se que na fazenda da questo anterior: 1) apenas a rea hachurada abaixo produtiva, 2) que o proprietrio declarou o valor do metro quadrado em R$ 1,00, 3) que o imposto pago na parte produtiva da propriedade foi de 2% de seu valor e na parte improdutiva foi de 20% de seu valor, determine o valor total do imposto pago, em R$, dividido por 3.000 3 .

6. Qual o maior coeficiente do polinmio de grau 4, com coeficiente lder igual a 1, que tem 2 3 como raiz?

7. Sobre a funo f(x)

3 x

podemos afirmar que:

0-0) Ela est definida para todos os reais exceto 0 e 1 1-1) f(x) 2-2) f(x) 3-3) f(x) 4-4) f(x) 0 para todo real x 1 e x 0 para todo real x 0 e x
3 5 3 5

5 para todo real x 0 e x 1 0 se x 0 ou 3 5 x 1

493

MATEMTICA

UFPE
F A S E

8. Uma pessoa alimenta seu co combinando o contedo de duas marcas de raes preparadas pelos fabricantes X e Y. A tabela abaixo discrimina a quantidade de unidades de vitaminas e de sais minerais em cada saco de rao e a quantidade mnima de unidades que o co deve consumir: RAO X RAO Y Vitaminas Sais minerais 40 20 20 40 MNIMO 200 200

2 A

Se o saco da rao X custa R$ 10,00 e o da Y custa R$ 15,00, determine o inteiro mais prximo do total de sacos a serem comprados de modo a minimizar os custos e satisfazer as quantidades mnimas requeridas.

P R O V A

9. Determine o maior valor de r de forma que as circunferncias (x 1)2 (y 1)2 1 (x 3)2 (y 3)2 r2 tenham um nico ponto de interseo. Indique o inteiro mais prximo de 10 r.

10. Sejam A e B matrizes 3 3 com coeficientes reais. Denote por I a matriz identidade 3 por det(X) o determinante da matriz X. Analise as afirmaes seguintes: 0-0) 1-1) 2-2) 3-3) 4-4) det(AB) (det A) (det B) Se B5 I ento B inversvel (A B)2 A2 2AB B2 det (2A) 8 det (A) det (A 1) det A 1

3e

11. De quantas formas podemos escolher, sem considerar a ordem, dois naturais distintos no conjunto {1, 2, 3, 4, ..., 20} de forma que sua soma seja mltiplo de 3?

12. Sejam f: R

{3} R

{ 2} e g: R

para todo x no domnio de g. Se g(x)

{ 2} R {3} das funes tais que f(g(x)) 3x 1 , calcule f(2). x 2

x,

13. Qual a maior soluo real positiva da equao x3x

3x2x

6xx

0?

494

MATEMTICA
14. Uma coleo de polgonos regulares congruentes um ladrilhamento do plano se:

UFPE
F A S E

1) Todo ponto no plano pertence a pelo menos um polgono na coleo. 2) A inteseo de dois polgonos na coleo vazia um vrtice ou um lado comum dos polgnos. Apenas alguns tipos de polgonos regulares ladrilham o plano. Indique a soma dos lados daqueles que possuem esta propriedade.

2 A

15. Uma epidemia de ratos propaga-se da seguinte forma: cada rato infectado contamina 3 (trs) outros ratos no perodo de uma semana. Quantas semanas, aps a contaminao do primeiro rato, sero necessrias para que uma populao de 220 ( 1.048.576) ratos esteja contaminada?

P R O V A

16. Uma pessoa aplicou R$ 100.000,00 na caderneta de poupana, que rendeu 1% ao ms ao longo de um ano (lembre que os juros so cumulativos). Ao fim deste ano, esta pessoa tinha x reais nesta caderneta de poupana, sem ter feito nenhum depsito alm do inicial nem ter realizado nenhum saque. Qual o inteiro mais prximo de log x?

17. Na figura ao lado, considerando que o tringulo ABC retngulo em A e ABFG, ACIH, BCDE so quadrados, pode-se afirmar que: 0-0) Os tringulos CBF e EBA so congruentes. 1-1) A rea de ABFG o dobro da rea de CBF. 2-2) A rea do retngulo BKJE o dobro da rea de EBA. 3-3) ABFG e BKJE tm mesma rea. 4-4) A rea de BCDE a soma das reas de ABFG e ACIH.

18. Aumentando-se o raio de um cilindro em 10% e diminuindo-se sua altura em 10%, podemos afirmar que: 0-0) 1-1) 2-2) 3-3) 4-4) A rea total do cilindro aumenta em 10,5%. O volume do cilindro aumenta em 33,1%. A rea de uma das bases do cilindro aumenta em 21%. A rea lateral do cilindro no varia. A soma do raio da base do cilindro com sua altura permanece inalterada.

495

MATEMTICA
19. Quantas so as solues da equao sen4 x cos3x 1 no intervalo [0,30]?

UFPE
F A S E

20. Considere E1 uma esfera de raio unitrio. Se, para cada i 1, Ci for o cubo inscrito na esfera Ei e Ei 1 a esfera inscrita no cubo Ci, qual o inteiro mais prximo da soma das reas das esferas E1, E2, E3, E4 ? 21. Considere a funo f : R R dada por f(x) 0-0) 1-1) 2-2) 3-3) 4-4)

2 A

log2 (3

cosx).

Esta funo no peridica. Esta funo tem como imagem o intervalo [1,2]. f(x) 2 quando x 2k , para k inteiro. Para todo x real, temos que f(x) f( x). f(2x) 1 log2 (1 cos2x), para todo x real.

P R O V A

22. Quais das identidades abaixo so verdadeiras para todo x real? 0-0) 1-1) 2-2) 3-3) 2 cos (2x 1) sen (2x 1) sen (4x 2) cos (2x 1) cos (2x 1) cos2 (4x2 1) sen (2x 1) sen (2x 1) 2 sen (2x) cos (1) cos (2x 1) cos (2x 1) cos ( 2)
sen(2x 1) 2 cos(1)

sen( 4x 2 1) 4-4) 2 cos(2x 1)

23. Seja i 0-0) 1-1) 2-2) 3-3) 4-4)

1 a unidade imaginria. Sobre a expanso de x

1 x

10

podemos afirmar:

A soma dos coeficientes 32 i. O termo independente de x tem coeficiente 252 i. Existem 5 termos com coeficientes imaginrios puros. Existem 4 termos com coeficientes reais positivos. Existem 3 termos com coeficientes reais negativos.

24. Seja A a rea da regio determinada pelas inequaes x2 x x Indique o inteiro mais prximo de 10 A. y2 4, y 2, 3y 0.

496

MATEMTICA

UFPE
F A S E

25. Seja C1 um crculo de centro O1 e raio 10 e C2 um outro de centro O2 e raio 9. Se C1 intercepta C2 em dois pontos P1 e P2, tais que O1P1 perpendicular a O2 P1 , calcule a rea do paralelogramo de vrtices O1, P1, O2 e P2.

2 A 2 B

26. Qual o inteiro mais prximo de 11

6 2

11

6 2?
P R O V A

27. Seja (a, b) o centro da circunferncia circunscrita ao tringulo com vrtices nos pontos (2, 3), (4, 0) e (1, 1). Assinale 7 (a b).

1. A figura abaixo representa um rio cujas margens so retas paralelas.

F A S E

Qual o inteiro mais prximo da largura do rio, quando medida em metros? 2. Na figura ao lado o retngulo ABCD de lados 4 cm e 5 cm foi dividido em quadrados de lado 1 cm. Qual a rea da regio hachurada?

P R O V A

3. A figura abaixo ilustra alguns degraus de uma escada de concreto. Cada degrau um prisma triangular reto de dimenses 15 cm, 30 cm e 60 cm. Se a escada tem 20 degraus, qual o volume, (em dm3) do concreto usado para construir a escada?

497

MATEMTICA
Observe o texto, para responder as questes 4 e 5.

UFPE
F A S E

Em uma lata na forma de cilindro circular reto de raio da base medindo r cm e altura 60 cm, deseja-se colocar um rtulo sem haver sobreposio de suas partes, de forma que o segmento AB , descrito a seguir, seja paralelo s suas bases. Este rtulo pode ser de cinco tipos: Tipo 1: um quadrado de lado igual a 30 cm, sendo AB um dos lados. Tipo 2: um quadrado de lado igual a 30 cm, sendo AB uma de suas diagonais. Tipo 3: um retngulo de lados medindo 20 cm e 40 cm, sendo AB um segmento paralelo ao maior lado. Tipo 4: um losango de lados medindo 16 cm e diagonal menor medindo 18 cm, sendo AB a diagonal maior. Tipo 5: uma circunferncia de raio igual a 20 cm, sendo AB um dos seus dimetros. 4. Quando r 0-0) Tipo 1 1-1) Tipo 2 2-2) Tipo 3 5, quais dos rtulos descritos anteriormente podem ser colados? 3-3) Tipo 4 4-4) Tipo 5

2 B

P R O V A

5. Sabendo-se que r inteiro, qual deve ser o menor valor de r para que todos rtulos possam ser colados?

6. Um hexgono regular ABCDEF est no primeiro quadrante de um sistema cartesiano XOY e possui o lado AB no eixo OX e o vrtice F no eixo OY. O vrtice C tem abscissa 4. Qual o permetro do hexgono?

7. Qual o nmero de lados das faces de um poliedro regular com 20 vrtices e 30 arestas?

8. Na figura ao lado os segmentos BC e DE so paralelos. Analise as proposies a seguir: 0-0) Os tringulos BCD e CBE tm mesma rea. 1-1) A razo entre as reas dos tringulos ADC e ABC igual a AD / AB . 2-2) Os tringulos ADC e AEB tm mesma rea.
AE 3-3) AD AB AC 4-4) As reas dos tringulos BDE e CBE so proporcionais DE e BC , respectivamente.

498

MATEMTICA
9. Uma cidade situada numa plancie possui todos os prdios na forma de prismas retangulares retos. Quando vista de cima, por um telescpio muito distante, os contornos dos seus prdios apresentam-se como na figura ao lado: O nmero no interior de cada retngulo indica a altura (em metros) do prdio do qual contorno. Quais das figuras abaixo podem representar a silhueta destes prdios, quando vistos lateralmente pelo mesmo telescpio?

UFPE
F A S E

2 B

P R O V A

10. Considere o tringulo de vrtices A(2, 5), B(6, 1) e C(4, 7). Refletindo-se A, B e C com respeito a reta x y 3 obtemos um novo tringulo de vrtices A, B e C. Determine o inteiro, mltiplo de 3, mais prximo da soma dos mdulos das coordenadas de A, B e C.

11. A reta r tangente a um crculo C de raio igual a 4 no ponto P. Seja O o ponto de C diametralmente oposto P e P um outro ponto de C distinto de P e O. A reta que passa por O e P intercepta r no ponto Q. Calcule o produto de OP por OQ .

12. Seja V um vrtice de um icosaedro regular. Para cada uma das arestas incidentes a V, considere o ponto que est a uma distncia de V igual a 1 do comprimento da aresta. Unindo-se estes 3 pontos obtm-se um pentgono, que a base de uma pirmide com vrtice em V. Para cada vrtice do icosaedro construa a pirmide como descrita acima. Retirando-se do icosaedro todas as pirmides obtemos um novo slido. Quantas arestas possui este slido?

499

MATEMTICA
13. Um retngulo ureo aquele cujo razo entre o maior e menor de seus lados Seja R um retngulo ureo.
1

UFPE
F A S E

5.

0-0) Retirando-se R o quadrado de lado igual ao menor dos seus lados obtemos um retngulo ureo. 1-1) O retngulo de lados 1 e 1 ureo. 2-2) O retngulo de lados 1 e 2 1 ureo. 3-3) O retngulo de lados 1 e 2 1 ureo. 4-4) A rea de um retngulo ureo proporcional ao quadrado de seu maior lado. 14. Seja (a,b) o centro da circunferncia circunscrita ao tringulo com vrtices nos pontos (2,3), (4,0) e (1,1). Assinale 7(a b).

2 B

P R O V A

15. Um cubo de lado 10 cm cortado por dois planos como ilustra a figura. Cada corte intercepta trs arestas do cubo em pontos distantes 3 cm do vrtice mais prximo. Se a distncia entre as faces triangulares do slido resultante x cm, calcule 3 x.

16. Qual o maior coeficiente do polinmio de grau 4, com coeficiente lder igual a 1, que tem 2 3 como raiz? 17. Os centros de dois crculos de raio 5 cm distam 5 cm entre si. Qual o valor de onde A cm a rea da unio dos dois crculos? 18. A figura ao lado ilustra um prisma triangular reto ABCDEF com bases ABC e DEF. Analise as afirmaes: 0-0) 1-1) 2-2) 3-3) Os tetraedros ACFB e ADFB tm o mesmo volume. Os tetraedros BCFA e BEFD tm o mesmo volume. Os tetraedros ADFB e BEFD tm o mesmo volume O volume do tetraedro ABCF um tero do volume do prisma triangular ABCDEF. 4-4) O volume do prisma triangular ABCDEF o produto da rea de ABC por CF.
2

12A , 3 3

500

MATEMTICA

UFPE
F A S E

19. Considere um tringulo equiltero ABC de lado 80 4 3 cm. Unindo-se os pontos mdios dos lados de DEF obtm-se um novo tringulo GHI. Repetindo-se este procedimento mais duas vezes, constri-se um tringulo NOP. Qual o inteiro mais prximo da rea deste tringulo, em cm3?

2 B

20. Na ilustrao a seguir, todos os crculos tem mesmo raio, o 3 tringulo ABC equiltero e seus lados medem 28 1 unidades de comprimento. Determine o raio dos crculos.

P R O V A

Observe o texto e a figura abaixo, para responder as questes 21 e 22. Ao lado encontra-se o esboo de uma regio que contm um lago. Os pontos da superfcie desta regio que esto a uma altura de 10 m, 20 m e 30 m da lmina dgua do lago so representados por curvas conforme a figura:

21. Um nadador noite est no ponto O. Quais as fontes de luz, que esto situadas no nvel do solo em A, B, C, D e E, seguramente no so vistas pelo nadador, independente da forma da superfcie deste terreno? 0-0) A 1-1) B 2-2) C 3-3) D 4-4) E

22. Se este terreno foi cortado perpendicularmente na linha pontilhada, quais das seguintes silhuetas podem ser vistas lateralmente?

501

MATEMTICA
NESTA PROVA SERO UTILIZADOS OS SEGUINTES SMBOLOS COM OS SIGNIFICADOS INDICADOS:

UFRGS

Z: conjunto dos nmeros inteiros R: conjunto dos nmeros reais (x, y): par ordenado de abscissa x e ordenada y. i: unidade imaginria z : conjugado do nmero complexo z (aij)nxn: matriz quadrada com n linhas e n colunas, onde, nos elementos aij, i indica a linha, e j indica a coluna log x: logaritmo de x na base 10 cos x: cosseno de x tan x: tangente de x AB: medida do segmento AB [a, b]: {x R / a x b}

1. A expresso
8 a) 15

3 5
b) 3 5

5 igual a 3
c) 1 d)

34 15

e) 8 15 15

2. Considerando A tais que y 2x

{x Z / 1 x 10}, e sendo R a relao em A formada pelos pares (x, y) 1, o domnio e a imagem dessa relao correspondem, respectivamente, a d) {1, 2, 3, 4, 5} e {1, 3, 5, 7, 9} e) {1, 2, 3, 4, 5} e {0, 2, 4, 6, 8}

a) {0, 1, 2, 3} e {1, 3, 5, 7} b) {1, 2, 3, 4} e {3, 5, 7, 9} c) {0, 1, 2, 3, 4} e {0, 2, 4, 6, 8}

3. Dentre os conjuntos de pontos do plano cartesiano, apresentados abaixo,

quais os que NO podem representar grficos de uma funo? a) Apenas I e III b) Apenas II e IV c) Apenas III e IV d) Apenas I, III e IV e) Apenas II, III e IV

502

MATEMTICA
4. Considerando a funo linear f(x) 2x e a funo quadrtica g(x) valores de x a funo g satisfaz as relaes g(x) f(x) e g(x) 1? x
2

UFRGS
4, para quais

3 x 1 5 a) b) 3 x 3 c) 1 5 x 1 5 5 x 3 d) 1 3 x 5 ou 3 e)
5. Seja a funo y

f(x) representada pelo grfico

Dentre os grficos abaixo, o que representa a funo y

f(x

2)

6. A equao 2mx a) m b) m c) m 0 0 4

mx

1 2

0 possui 2 razes reais distintas. Ento, d) m e) 0 0 ou m m 4 4

7. Considerando uma taxa mensal constante de 10% de inflao, o aumento de preos em 2 meses ser de a) 2% b) 4% c) 20% d) 21% e) 121%

503

MATEMTICA

UFRGS

8. Um grupo de estudantes dedicado confeco de produtos de artesanato gasta R$ 15,00 em material, por unidade produzida, e, alm disso, tem um gasto fixo de R$ 600,00. Cada unidade ser vendida por R$ 85,00. Quantas unidades tero de vender para obterem um lucro de R$ 800,00? a) 7 b) 10 c) 12 d) 15 e) 20

9. Uma pessoa gasta 1 do dinheiro que tem e, em seguida, 2 do que lhe resta, ficando com 4 3 R$ 350,00. Quanto tinha inicialmente? a) R$ 400,00 b) R$ 700,00 c) R$ 1400,00 d) R$ 2100,00 e) R$ 2800,00

10. O nmero z a) m b) m c) 3

(m

3) 3

(m2

9) i ser um nmero real no nulo para d) m e) m 3 0

3 3 ou m m 3

11. Considere z1 3 2i e z2 a) cos i sen 4 4 b) 2 cos i sen 4 4 i sen 3 c) cos 34 4

i . A representao trigonomtrica de z1 i sen 7 d) 2 cos 74 4 i sen 7 e) cos 74 4

z2

12. Considere as afirmaes: I - Se p(x) e q(x) so polimnios de grau n, ento p(x) q(x) um polimnio de grau 2n. 3 2 II - O resto na diviso de p(x) mx x x por q(x) x 1 igual a m. III - O produto de um polimnio de grau n por (x a) um polimnio de grau n 1. Quais esto corretas? a) Apenas I b) Apenas I e II c) Apenas III d) Apenas II e III e) I, II e III

13. Sendo A a) 3

(aij)nxn uma matriz onde n igual a 2 e aij b) 1 c) 0

j, o determinante da matriz A e) 3

d) 1

504

MATEMTICA
14. O determinante da matriz 1 a b nulo a) para quaisquer valores de a e b. b) apenas se a 0. c) apenas se b 0. 15. O sistema linear
x y 1 4x my 2

UFRGS

2 2a 1 b 2 b

3 3a 3

d) somente se a b. e) somente quando 1

2a

(b

3)

0.

possvel e determinado se e somente se a) m 2 b) m 4 c) m 4 d) m 1 e) 4m 1

16. As medidas dos trs lados de um tringulo retngulo so nmeros em progresso aritmtica. Qual o valor da rea do tringulo, sabendo-se que o menor lado mede 6? a) 12 2 b) 18 c) 20 2 d) 24 e) 30

17. A seqncia (x, xy, 2x), x 0 uma progresso geomtrica. Ento, necessariamente a) x um nmero irracional. b) x um nmero racional. c) y um nmero irracional. d) y um nmero racional. e) y/x um nmero irracional.

18. As substncias radioativas tm a tendncia natural a se desintegrarem. Considerando um caso em que a massa inicial da substncia seja 54 g, e t dias depois sua massa seja, aproximadamente, t 54 0,835 g, pergunta-se: em um dia, que porcentagem da massa desta substncia se desintegra? a) 83,5% b) 67,5% c) 16,5% d) 8,35% e) 6,75%

505

MATEMTICA
19. Os domnios das funes f(x) (log1/2 x) 2 ; g(x) log2 (2x 2 h(x) log3 (x 3x 2), em R, so respectivamente a) b) c) d) e) x 0 0 x x 4; x 1/2; 1 x 2 x 4; x 1/2; x 1 ou x x 4; x 1/2; 1 x 2 4; x 1/2; x 1 ou x 2 4; x 1/2; 1 x 2 log 0,001 2 2 1) e

UFRGS

20. Dada a expresso S a) 3 b)

log 100, o valor de S c) 1 ). Se 0 x d) 0 2 esta equao e) 1

21. Considere a equao cos x a) b) c) d) e)

cos (x

no tem soluo. tem apenas 1 soluo. tem somente as solues 0 e . tem somente as solues /2 e 3/2. tem infinitas solues.

22. O grfico abaixo representa a funo real f.

Esta funo dada por: a) f(x) b) f(x) c) f(x) 1 cos x 1 cos x cos (x 1) 1 d) f(x) e) f(x) cos (x cos (x 1) )

23. No intervalo [0, ] a equao tan x a) no possui razes. b) possui uma nica raiz. c) possui apenas 2 razes.

0 d) possui exatamente 4 razes. e) apresenta infinitas razes.

506

MATEMTICA

UFRGS

24. Um trem de passageiros constitudo de uma locomotiva e 6 vages distintos, sendo um deles restaurante. Sabendo-se que a locomotiva deve ir frente, e que o vago restaurante no pode ser colocado imeditamente aps a locomotiva, o nmero de modos diferentes de montar a composio a) 120 b) 320 c) 500 d) 600 e) 720

25. O nmero de mltiplos de trs, com quatro algarismos distintos, escolhidos entre 3, 4, 6, 8 e 9 a) 24 b) 36 c) 48 d) 72 e) 96

26. Dada a figura ao lado Qual o valor de x? a) 2,15 b) 2,35 c) 2,75 d) 3,15 e) 3,35

27. A altura de um tringulo equiltero inscrito numa circunferncia 2 3 cm. A razo entre a rea desse tringulo e a rea de um quadrado inscrito nessa mesma circunferncia a)
3 4

b) 3 3 4

3 c) 8

d)

3 8

e) 3 3 8

28. Seja a figura ao lado Sabendo-se que AD 12 cm; AE 15 cm e AB pode-se afirmar que a medida do raio do crculo a) 4 cm b) 4,5 cm c) 5 cm d) 5,5 cm e) 6 cm 8 cm;

29. Um pedao de cano de 30 cm de comprimento e 10 cm de dimetro interno encontra-se na posio vertical e possui a base inferior vedada. Colocando-se dois litros de gua em seu interior, a gua a) ultrapassa o meio do cano. b) transborda. c) no chega ao meio do cano. d) enche o cano at a borda. e) atinge exatamente o meio do cano.

30. Numa pirmide regular, a base um quadrado de lado a. Suas faces laterais so tringulos equilteros. O volume desta pirmide a)
2 a3 12

b)

2 a3 6

c)

2 a3 3

d)

3 a3 12

e)

3 a3 6

507

MATEMTICA

UFRGS

31. Um poliedro convexo de onze faces tem seis faces triangulares e cinco faces quadrangulares. O nmero de arestas e de vrtices do poliedro , respectivamente, a) 34 e 10 b) 19 e 10 c) 34 e 20 d) 12 e 10 e) 19 e 12

32. Considere a reta r passando em P (0,3). Duas retas p e q, paralelas ao eixo das ordenadas e distantes entre si 2 unidades, so interceptadas no 1 quadrante pela reta r em 2 pontos, cuja distncia 2 5 unidades. A equao de r a) y 3x b) y 2x c) 3x y 2 3 3 d) y e) 3x 0 y 0 determina no crculo 2x y 3 3 0

33. O comprimento da corda que a reta r definida pela equao 2x de centro no ponto C(2,0) e raio r 2 a) 0 b) 2 c) 5 d)

10 5

e) 4 5 5

34. Um ponto P(x, y) descreve uma trajetria no plano cartesiano, tendo sua posio a cada instante t(t 0) dada pelas equaes
x y

2t 3t

2 t 3 d) 3 13 e)
61

A distncia percorrida pelo ponto P(x,y) para 0 a) 2 35. A equao x a) m b) m c) m 0 0 13


2

b) 3 y
2

c) 4x 6y m

13

0 representa um crculo se e somente se d) m e) m 13 13

508

MATEMTICA
Nas questes de 1 a 9, marque:

UnB

itens CERTOS, na coluna I; itens ERRADOS, na coluna II. Use, para as devidas marcaes, a Folha de Rascunho e, posteriormente, a Folha de Respostas.

Questo 1 Um casal de operrios especializados trabalha no mesmo setor de uma fbrica. Em dezembro, a operria recebeu por dia de trabalho 3/4 do que recebeu o operrio, sendo que ela trabalhou 16 dias e ele 20 dias. No total, o casal recebeu a quantia de R$ 1.408,00. Analise essa situao e julgue os itens abaixo. (0) A mulher recebeu menos de R$ 32,00 por dia de trabalho. (1) O homem recebeu mais de 70% do total pago aos dois juntos, por dia de trabalho. (2) O casal teria recebido mais de R$ 1.600,00, se cada um tivesse trabalhado, no mnimo, 22 dias.

Questo 2 Considere a funo f definida no conjunto dos nmeros inteiros e dada pela seguinte expresso: f(n) Julgue os itens a seguir. (0) A soma dos nmeros inteiros para os quais f se anula igual a um. (1) Para todo n (2) Para todo n 3, vlida a igualdade f(n) 3, vlida a igualdade f(n n5 5n3 4n.

(n (n
1)

2)! . 3)!
f(n) n n 3. 2

(3) Para todo inteiro n, f(n) divisvel por 120.

Questo 3 Julgue os itens que se seguem. (0) Para todo nmero inteiro positivo n, tem-se 2 (1) Se o nmero real x tal que (2) Se y 0,01 x

( 1)

n 0,002, ento 0
y

2.
x
3

10 .

x 2 , em que x um nmero real, ento 0 1 x2

1/2.
2

x 1 1 , ento x 3. (3) Se o nmero real x 1 tal que x 1 (x (4) Existem exatamente trs valores reais de x que satisfazem equao x

5x

6)

1.

509

MATEMTICA
Questo 4 Julgue os itens a seguir.

UnB

(0) Em uma certa populao indgena, vive um total de M mulheres. Desse total, 47,5% adornam-se com um nico brinco. Do restante das mulheres, 50% usam dois brincos e as demais no usam brincos. Ento, o nmero total de brincos usados por todas as mulheres maior que M. (1) Uma secretria datilografa quatro cartas, destinadas a quatro pessoas diferentes, e escreve os endereos em quatro envelopes. Se ela colocar aleatoriamente as cartas nos envelopes, cada uma em um envelope diferente, ento a probabilidade de apenas uma carta ser endereada ao destinatrio errado de 1/4. (2) A figura seguinte ilustra a planta baixa de uma repartio pblica, com 36 salas internas que se comunicam por meio de portas. Essa repartio emite um documento extremamente importante. No entanto, para obt-lo, uma pessoa deve entrar na repartio, visitar obrigatoriamente cada uma das salas uma nica vez e depois sair. Nessas circunstncias, considerando a posio da entrada e a da sada da repartio, a pessoa poder obter o documento aps passar por 35 portas internas.

Questo 5 Considere P1 o pentgono regular cujos vrtices so determinados pelas razes complexas zk do polimnio z5 1, com k 0, 1, 2, 3 e 4 e P3 o pentgono regular cujos vrtices so determinados pelas razes complexas wk do polinmio w5 35, com k 0, 1, 2, 3 e 4, nos quais se supe que as razes estejam ordenadas por ordem crescente de seus argumentos. Julgue os seguintes itens. w (0) O nmero complexo z 3 tem parte imaginria no-nula. 3 (1) Para k 0, 1, 2 e 3, tem-se wk
1

w1z1. 310, ento

(2) Se D o decgono determinado pelas razes complexas do polinmio z10 todos os cinco vrtices de P3 coincidem com vrtices de D. (3) P1 e P3 so polgonos regulares semelhantes.

510

MATEMTICA
Questo 6

UnB

Em um tabuleiro quadrado, de 5 5, mostrado na figura abaixo, deseja-se ir do quadrado esquerdo superior (ES) ao quadrado direito inferior (DI).

Somente so permitidos os movimentos horizontal (H), vertical (V) e diagonal (D), conforme ilustrado nas representaes seguintes.

Com base nessa situao e com o auxlio dos princpios de anlise combinatria, julgue os itens que se seguem. (0) Se forem utilizados somente movimentos horizontais e verticais, ento o nmero de percursos possveis ser igual a 70. (1) Se forem utilizados movimentos horizontais, verticais e apenas um movimento diagonal, o nmero de percursos possveis ser igual a 140. (2) Utilizando movimentos horizontais, verticais e trs movimentos diagonais, o nmero de percursos possveis igual a 10.

511

MATEMTICA
Questo 7

UnB

O conceito de homotetia est relacionado com a mudana de escala, isto , a ampliao ou a reduo de figuras planas e de slidos. Considerando um ponto O do espao e um nmero real r 0, a homotetia h, de centro uur e O razo r, definida de modo que h(O) O e, para A O, h(A) o ponto A da semi-reta OA , tal que a medida de OA igual a r vezes a medida de OA .

A partir do enunciado, julgue os itens seguintes. (0) Se O, A e B so vrtices de um tringulo T e suas respectivas imagens O, A e B, pela homotetia h, so os vrtices do tringulo T, ento T e T tm um ngulo em comum, mas no so necessariamente semelhantes. (1) Na figura abaixo, P e P so planos paralelos, sendo d a distncia entre eles; F um polgono contido em P; F a imagem de F, por uma homotetia h, de centro O e razo r 0, com F contida em P. Ento, a rea de F igual a d (rea de F) r.

(2) Sabendo que o Estado do Colorado, nos Estados Unidos, tem a forma de um polgono e que, em um globo terrestre de raio igual a 20 cm, a sua rea de 2,64 cm2, e supondo que o raio da Terra seja igual a 6.400 km, correto concluir que a rea real daquele Estado maior que 269.000 km2.

512

MATEMTICA
Questo 8

UnB

Deseja-se construir uma estrada ligando as cidades A e B, que so separadas por um rio de margens paralelas. Em funo do custo, a ponte sobre o rio deve ser perpendicular s margens, e os trechos AC e DB devem ser segmentos de reta, como indica a figura abaixo. Suponha que, no sistema cartesiano da figura, o ponto A tenha coordenadas (0, 30), B tenha coordenadas (70, 41) e que o rio ocupe a faixa {(x, y): x R e 0 y 1}, em que x e y so medidos em quilmetros.

Com relao ao problema descrito, julgue os itens que se seguem. (0) Se C tem coordenadas (40, 0), ento a distncia entre as cidades A e B, medida no trajeto ACDB, menor que 100 km. (1) Se B uma cidade situada um quilmetro abaixo da cidade B, na direo vertical, ento os comprimentos dos trajetos ACBB e ACDB so iguais. (2) Se a ponte for construda de modo que o trajeto ACDB tenha comprimento mnimo, ento o ponto C dever ter coordenadas (30, 0).

513

MATEMTICA
Leia o texto a seguir para responder s questes 9 e 10.

UnB

Navegar preciso! A lgebra, a Geometria e a Trigonometria so utilizadas em navegao para se determinar a posio de embarcaes. Entre os mtodos existentes, a navegao astronmica o mtodo de o navegador determinar a posio da embarcao a partir da posio dos astros. Em linhas gerais, esse mtodo pode ser descrito como se segue. Usando um instrumento chamado sextante, o navegador mede, em um determinado momento, o ngulo entre a direo de um astro e a linha do horizonte, chamado altura angular, conforme ilustra a figura I. Registrando o ngulo e a hora em que foi feita a medio, o navegador consulta o Almanaque Nutico, que informa o ponto da Terra A mais prximo do astro. Explorando a geometria da situao, possvel calcular a distncia R, do navegador a esse ponto A. Assim, o lugar geomtrico das possveis posies do navio um crculo de raio R e centro A, chamado crculo de posio, conforme mostra a figura II. Sendo muito grande o raio de um crculo de posio, o pequeno segmento circular que interessa ao navegador, isto , o segmento circular prximo posio estimada por ele, pode ser aproximado pela reta tangente no crculo, denominada reta de altura.

Figura I

Figura II

45

30

15

15

30

45

Em navegao, os mapas utilizados so as Cartas de Mercator, cujas linhas verticais representam a direo norte, e cada grau corresponde a 60 milhas nuticas. A figura III uma Carta de Mercator, na qual est representado um crculo de posio C, para um astro com 40 de altura angular, s 12 horas. Para essa situao, o Almanaque Nutico forneceu o raio de C, igual 3.000 milhas nuticas, e as coordenadas de A: 28 de longitude oeste e 7 de latitude sul.

60 90 75 60 45 30 15 0 15 30

Figura III

514

MATEMTICA
Questo 9 Com o auxlio do texto e das figuras, julgue os itens a seguir.

UnB

(0) Nas Cartas de Mercator, devido ao fato de que as linhas verticais no convergem nos plos, as reas localizadas em latitudes mais altas ficam representadas em tamanhos maiores do que na verdade deveriam ser. 2 2 2 (1) A equao do crculo de posio C (x 28) (y 7) 50 . (2) Se o navegador estima estar na posio ( 58, 33) do crculo de posio C, ento a equao da reta de altura, nesse ponto, dada por 4y 3x 306. Questo 10 Um navio est situado no crculo de posio C indicado na figura III. possvel determinar as suas coordenadas utilizando o ngulo , indicado na figura abaixo, denominado azimute. Ele definido como o ngulo entre o norte da bssola e a linha imaginria que liga o navio ao centro do crculo de posio (A), medido a partir do Norte, no sentido do movimento dos ponteiros de um relgio. Se mede 135, ento a posio do navio o ponto de coordenadas (a, b). Calcule o mdulo da soma (a b), desconsiderando a parte fracionria do resultado, caso exista.

Questo 11 As revistas X, Y e Z so publicadas por uma mesma editora. A assinatura da revista Y custa o triplo da assinatura da revista X e a de Z custa 2/3 da assinatura de Y. Em uma promoo especial de assinaturas de suas revistas, com o objetivo de conquistar novos assinantes, a editora ofereceu 10%, 50% e 90% de desconto nos preos das assinaturas das revistas X, Y e Z, respectivamente, para aqueles que assinassem as trs revistas. Calcule o desconto, em porcentagem, obtido por uma pessoa que assinou as trs revistas, desprezando a parte fracionria de seu resultado, caso exista.

Questo 12 Considere um tringulo equiltero e um hexgono regular de permetros iguais. Determine a rea do hexgono, em centmetros quadrados, sabendo que o tringulo tem rea de 48 cm2. Desconsidere a parte fracionria de seu resultado, caso exista.

515

MATEMTICA
Questo 13

UnB

Durante o vero, quando h um aumento no consumo de refrigerantes, um grupamento de escoteiros decidiu coletar latas de alumnio para reciclagem, conseguindo recolher 300 latas por dia. A companhia de reciclagem pagava R$ 0,10 por lata, mas, a esse preo, as latas estavam se acumulando nos galpes, mais rapidamente do que poderiam ser recicladas. Assim, no dia em que os escoteiros iniciaram a coleta, a companhia mudou a sua estratgia e passou a pagar uma quantia menor por lata: houve uma reduo fixa e diria correspondente a 1,25% do preo inicial de R$ 0,10. Como as latas coletadas deveriam ser entregues de uma nica vez, devido aos custos de transporte, os escoteiros ficaram em um dilema: no incio, o preo estava melhor, mas eles tinham poucas latas; por outro lado, se esperassem muito, o preo ficaria significativamente menor. Determine o nmero de dias em que os escoteiros devem concluir a coleta e vender as latas, de modo que o grupamento receba a maior quantia possvel.

Questo 14 Os copos descartveis, em geral, tm a forma de um tronco de cone, cuja superfcie lateral pode ser planificada, dando origem a um setor de coroa circular, como ilustrado na figura abaixo.

Representando por V o volume, em cm3, do copo cujo setor de coroa circular tem ngulo interno de 2l6, raio menor medindo 5 cm e raio maior medindo 10 cm, calcule, em centmetros cbicos, o valor de V . Despreze a parte fracionria de seu resultado, caso exista.

516

MATEMTICA
Questo 15

UnB

O Nautilus, mostrado na figura I, um molusco que vive nas guas profundas do Pacfico e que tem uma estrutura composta por sucessivas cmaras, em forma de espiral logartimica, que pode ser obtida geometricamente, conforme representa a figura II.

Na figura II, o tringulo ABC issceles, com ngulos da base medindo 72. Esse tringulo dito ureo porque a razo r AB/BC satisfaz relao r2 r 1. O ponto D obtido pela interseo do segmento AC com a bissetriz do ngulo B. Considerando-se agora o tringulo BCD, obtm-se o ponto E pela interseo do segmento BD com a bissetriz de C. Do mesmo modo, procede-se com o tringulo CDE, para se obter o tringulo DEF, e assim por diante, at que o tringulo FGH seja obtido. A espiral passa, ento, sucessivamente e na mesma ordem, pelos trs vrtices de cada um desses tringulos. Supondo que a medida do segmento HG seja igual a 1 cm e escrevendo a soma dos comprimentos dos segmentos GF, FE e ED na forma nr m, na qual n e m so nmeros naturais, calcule, em centmetros, o valor da soma n m.

517

MATEMTICA

UNESP
F A S E

1. Por uma mensagem dos Estados Unidos para o Brasil, via fax, a Empresa de Correios e Telgrafos (ECT) cobra R$ 1,37 pela primeira pgina e R$ 0,67 por pgina que se segue, completa ou no. Qual o nmero mnimo de pginas de uma dessas mensagens para que seu preo ultrapasse o valor de R$ 10,00? a) 8 b) 10 c) 12 d) 14 e) 16

2. Suponhamos que nos vestibulares de 1996 uma universidade tivesse tido, para os seus diversos cursos, uma mdia de 3,60 candidatos por vaga oferecida. Se para os vestibulares 1997 o nmero de vagas for aumentado em 20% e o nmero de candidatos aumentar em 10%, qual a mdia de candidatos por vaga que essa universidade ter? a) 3,24 b) 3,30 c) 3,36 d) 3,40 e) 3,46

3. Segundo matria publicada em O Estado de So Paulo, 09/09/96, o Instituto Nacional de Seguridade Social (INSS) gasta atualmente 40 bilhes de reais por ano com o pagamento de aposentadorias e penses de 16 milhes de pessoas. A mesma matria informa que o Governo Federal gasta atualmente 20 bilhes de reais por ano com o pagamento de um milho de servidores pblicos federais aposentados. Indicando por x a remunerao anual mdia dos beneficirios do INSS e por y a remunerao anual mdia dos servidores federais aposentados, ento y igual a a) 2x b) 6x c) 8x d) 10x
2

e) 16x

4. A rea de um tringulo issceles 4 15 dm e a altura desse tringulo, relativa sua base, mede 2 15 dm. O permetro desse tringulo igual a a) 16 dm b) 18 dm c) 20 dm d) 22 dm e) 23 dm

5. Considere as seqncias (an) e (bn) definidas por an+1 = 2 e bn+1= 3 , n 0. Ento, o valor de a11. b6 : a) 2 . 3 b) (12) c) 5
15 5 11 6

d) 6 e) 6

15

30

518

MATEMTICA
6. O grfico mostra o resultado de uma experincia relativa absoro de potssio pelo tecido da folha de um certo vegetal, em funo do tempo e em condies diferentes de luminosidade. Nos dois casos, a funo linear y = mx ajustou-se razoavelmente bem aos dados, da a referncia a m como taxa de absoro (geralmente medida em moles por unidade de peso por hora). Com base no grfico, se m1 a taxa de absoro no claro e m2 a taxa de absoro no escuro, a relao entre essas duas taxas : a) m1 = m2 b) m = 2m1 2 c) m1 . m2 = 1 d) m1 . m2 = -1 e) m1 = 2m2

UNESP
F A S E

7. Na figura, B um ponto do segmento de reta AC e os ngulos DAB, DBE e BCE so retos. Se AD = 6 dm, AC = 11dm e EC = 3dm, as medidas possveis de AB, em dm, so: a) 4,5 e 6,5 b) 7,5 e 3,5 c) 8 e 3 d) 7 e 4 e) 9 e 2

8. A figura foi obtida mediante rotaes de 60, 120, 180, 240 e 300 aplicadas a um quadrado cujos lados medem 1 dm, em torno de um mesmo vrtice desse quadrado e num mesmo sentido. A rea da regio escura : a) 1 - 2tg (15) b) tg (30) c) 1 - 4tg (15) d) 1 - tg (30) e) 1 - tg (15)

9. Sejam a, b nmeros reais. Se a > 0, a 1 e loga 10 > loga (10) , ento: a) b < 0 b) b > 1 e a > 1 c) b < 1 e a < 1 d) b < 1 e a > 1 ou b > 1 e a < 1 e) b > 0

519

MATEMTICA

UNESP
F A S E

10. Indicando por m, n e p, respectivamente, o nmero de razes racionais, razes irracionais e razes no reais do polinmio p(x) = x - x + 2x - 2, temos: a) m = 1, n = 1 e p = 3 b) m = 1, n = 2 e p = 2 c) m = 2, n = 1 e p = 2 11. Considere as matrizes reais 3 x 3
a b c x y z 1 1 1 m n p x y z 1 1 1
5 3 2

d) m = 2, n = 2 e p = 1 e) m = 1, n = 3 e p = 1

Se indicarmos por A e B, respectivamente, os determinantes dessas matrizes, o determinante da matriz


a + m + 1 1 2x b + n +1 1 2y c + p + 1 1 2z

igual a a) -2A - 2B b) 2A + 2B + 1 c) 2A + 2B d) -2A - 2B -1 e) 2A - 2B -1

12. Uma circunferncia contida na superfcie de uma esfera diz-se circunferncia mxima da esfera se seu raio igual ao raio da esfera. Assim, pode-se afirmar que: a) Toda circunferncia contida na superfcie de uma esfera uma circunferncia mxima da esfera. b) Um plano e uma esfera que se cortam ou tm um nico ponto em comum ou sua interseo contm uma circunferncia mxima da esfera. c) Os planos determinados por duas circunferncias mximas distintas de uma mesma esfera so necessariamente secantes e sua interseo contm um dimetro comum s duas. d) Dadas duas esferas concntricas distintas, uma circunferncia mxima de uma e uma circunferncia mxima da outra so necessariamente circunferncias concntricas coplanares. e) Duas circunferncias mximas de uma mesma esfera esto necessariamente contidas em planos perpendiculares.

520

MATEMTICA

UNESP
F A S E

1. Suponhamos que, para uma dada eleio, uma cidade tivesse 18500 eleitores inscritos. Suponhamos ainda que, para essa eleio, no caso de se verificar um ndice de abstenes de 6% entre os homens e de 9% entre as mulheres, o nmero de votantes do sexo masculino ser exatamente igual ao de votantes do sexo feminino. Determine o nmero de eleitores inscritos de cada sexo.

2 A

2. Considere uma parbola de equao

y = ax + bx + c, em que

a + b + c= 0.

a) Mostre que o ponto (1,0) pertence a essa parbola. b) Mantida ainda a suposio inicial, prove que o ponto (0,0) pertence parbola se e somente se b = a.

P R O V A

3. Em uma semi-reta de origem A1 marcam-se os pontos A2, A3, de maneira que os segmentos A1A2,A2A3,sejam consecutivos e suas medidas formem, nessa ordem, uma progresso geomtrica de razo

1 , em que A1A2 = 1dm. Considere a seqncia de quadrados que tm 2

como diagonais os segmentos A1A2,A2A3, conforme a figura, desenhada sem escala.

a) Demonstre que as reas desses quadrados formam uma progresso geomtrica de razo

1 . 4 b) Determine a medida do lado do primeiro quadrado dessa seqncia cuja rea menor 1 2 que dm . 100

4. Dez rapazes, em frias no litoral, esto organizando um torneio de voleibol de praia. Cinco deles so selecionados para escolher os parceiros e capitanear as cinco equipes a serem formadas, cada uma com dois jogadores. a) Nessas condies, quantas possibilidades de formao de equipes eles tm? b) Uma vez formadas as cinco equipes, quantas partidas se realizaro, se cada uma das equipes dever enfrentar todas as outras uma nica vez?

521

MATEMTICA
5. Para calcular a distncia entre duas rvores situadas nas margens opostas de um rio, nos pontos A e B, um observador que se encontra junto a A afasta-se 20 m da margem, na direo da reta AB, at o ponto C e depois caminha em linha reta at o ponto D, a 40 m de C, do qual ainda pode ver as rvores. Tendo verificado que os ngulos DCB e BDC medem, respectivamente, cerca de 15 e 120, que valor ele encontrou para a distncia entre as rvores, se usou a aproximao 6 = 2,4?

UNESP
F A S E

2 A

P R O V A

6. Para que valores reais de a, b, c as funes polinomiais f e g, definidas por f(x) = x + x + x e g(x) = x + (a + b)x + (b +c) x + a - b - c, so iguais?
3 2 3 2

7. Sejam a, b, c nmeros reais positivos e diferentes de 1. Se x = logabc, y = logbac e z = logcab, demonstre que: (x + 1) (y + 1) (z + 1) = 0 se e somente se abc = 1.

8. O tetraedro VABC da figura regular e sua base encontra-se sobre um plano cartesiano, em relao ao qual seus vrtices tm coordenadas A (
1 3 , 0 ), B ( 1 , 0) e C (0, ). 2 2 2

Dando-se face ABV uma rotao em torno da aresta AB, no sentido indicado pela figura, at faz-la coincidir com o plano ABC da base, quais as coordenadoras do ponto P que o vrtice V ocupar aps a rotao?

522

MATEMTICA

UNESP
F A S E

9. Sabe-se que os pnaltis a favor de certa equipe de futebol so batidos pelos dois melhores cobradores da equipe, A e B, cujos ndices de aproveitamento (converso em gols) so, respectivamente, 85% e 90%. Sabe-se, ainda, que B cobra 75% dos pnaltis a favor da equipe. Acaba de ser marcado um pnalti a favor dessa equipe e, nesse momento, os jogadores A e B esto em campo. a) Qual a probabilidade de que o pnalti seja cobrado por B e no seja convertido em gol? b) Qual a probabilidade de o pnalti ser convertido em gol? 10. No cubo ABCDEFGH, sugerido pela figura, considere o ponto mdio, M, da aresta AE.

2 A

P R O V A

Se N o ponto em que o plano determinado por H, M e B corta a aresta CG, prove que: a) HMBN um paralelogramo; b) os lados de HMBN tm mesma medida.

1. Suponhamos que, com base nos dados do quadro, publicado pela Revista Veja de 17/07/96, um casal cujo marido 8 cm mais alto que a esposa e cuja mdia de idade 30 anos, tenha concludo que seu filho recm-nascido, do sexo masculino, dever ter aproximadamente 1,75 m de altura quando adulto.

F A S E

2 B
523

P R O V A

Calcule a altura de cada um deles.

MATEMTICA

UNESP
F A S E

2. O corpo de enfermeiros plantonistas de uma clnica compe-se de 6 homens e 4 mulheres. Isso posto, calcule: a) quantas equipes de 6 plantonistas possvel formar com os 10 enfermeiros, levando em conta que em nenhuma delas deve haver mais homens que mulheres; b) a probabilidade de que, escolhendo-se aleatoriamente uma dessas equipes, ela tenha nmero igual de homens e de mulheres. 3. Suponhamos que uma represa de rea igual a 128 km tenha sido infestada por uma vegetao aqutica. Suponhamos tambm que, por ocasio de um estudo sobre o problema, a rea tomada 2 pela vegetao fosse de 8 km e que esse estudo tivesse concludo que a taxa de aumento da rea cumulativamente infestada era de 50% ao ano. Nessas condies: a) qual seria a rea infestada n anos depois do estudo, caso no se tomasse nenhuma providncia? b) Com as mesmas hipteses, em quantos anos a vegetao tomaria conta de toda a represa? (Use os valores aproximados log10 2 = 0,3 e log103 = 0,48.) 4. Para facilitar a contagem de germes de uma determinada amostra de leite foram feitas duas diluies em gua destilada. Inicialmente, diluiu-se 1 cm3 de leite em 99 cm3 de gua que se encontrava num dado frasco. Em seguida, diluiu-se 1 cm3 da mistura obtida na primeira etapa em 9 cm3 de gua contida num segundo frasco. Calcule a razo entre a quantidade de gua e a de leite da mistura do segundo frasco, aps a segunda diluio.
2

2 B

P R O V A

524

PORTUGUS
REDAO

CESGRANRIO

No sois mquinas! Homens que sois! Lutemos por um mundo de razo, um mundo em que a cincia e o progresso conduzam ventura de todos ns
CHARLES CHAPLIN O ltimo discurso O Grande Ditador

A vida at parece uma festa Em certas horas isso o que nos resta No se esquece o preo que ela cobra Em certas horas isso o que nos sobra.
TITS Diverso

A vida sedutora e fascinante, mas as pessoas se esquecem disso com muita facilidade, porque os tempos modernos so muito esmagadores.
LYA LUFT , in O Globo, 17/8/96.

Voc concorda com a afirmativa de que os tempos modernos so muito esmagadores? Por qu? Ser este o preo que se paga pela modernidade?

Redija um texto dissertativo de aproximadamente 25 linhas no qual voc apresente, de forma clara, objetiva e coerente, argumentos que justifiquem seu ponto de vista a esse respeito. Seu texto dever ser estruturado em introduo, desenvolvimento e concluso e estar em conformidade com a norma culta da lngua. D um ttulo sua redao, que dever ser apresentada a tinta azul ou preta. Utilize os trechos acima apenas como apoio para o desenvolvimento de seu texto; no os copie. A redao dever ser feita no Caderno de Respostas entregue em anexo. O rascunho NO ser levado em considerao.

525

PORTUGUS
LNGUA PORTUGUESA E LITERATURA BRASILEIRA Texto I O Cortio fez grande sucesso e at hoje lido com gosto por aqueles que chegam voluntariamente s suas pginas. Apesar da distncia de mais de um sculo que nos separa do mundo descrito por Alusio de Azevedo, temos a impresso de que o Brasil, em alguns aspectos, continua sendo um imenso cortio, cuja populao sobrevive com dificuldade e observada com olhar divertido pelos vizinhos privilegiados do sobrado. Tambm a figura de Joo Romo continua tendo, desgraadamente, alguma atualidade. Quando algum egresso do universo popular consegue enriquecer (s custas de expedientes srdidos), rompe com os vnculos que teve em sua origem e trata de se deixar cooptar gostosamente pelos bares. O mais grave, porm, o que se passa com as variantes do movimento abolicionista. Ao longo da nossa Histria, os movimentos comprometidos com as causas populares se ressentem da falta de um apoio organizado por parte das massas. Assim como o movimento abolicionista no conseguia se apoiar na mobilizao dos grandes interessados na abolio (os escravos) e era levado a buscar aliados nas reas mais coniventes com os privilgios estabelecidos, os atuais movimentos empenhados em promover mudanas democratizadoras so levados a se aliar com foras de escassa (ou nenhuma!) vocao democrtica. Um exemplo claro de poder diluidor do conservadorismo sobre as iniciativas voltadas para reformas que correspondem aos interesses das camadas mais desfavorecidas da populao se acha na histria da luta pela reforma agrria. H cerca de meio sculo, temos tido dentro do aparelho do Estado rgos incumbidos de fazer uma reforma agrria, para atender a uma demanda da sociedade. Desconfio, no entanto, que na conduo da poltica agrria dos governos brasileiros dos ltimos 50 anos tem havido polticos distribundo diplomas de benemrito para muitos Joes Romes.
KONDER, Leandro in O Globo, 21/7/96 (com adaptaes)

CESGRANRIO

10

15

20

25

30

35

526

PORTUGUS

CESGRANRIO

1. Leandro Konder compara nossa sociedade atual quela apresentada em O Cortio no sculo XIX. Aps leitura atenta do trecho, marque (V) para o que for verdadeiro e (F) para o que for falso em relao ao que dito no texto I. ( ( ( ( ( ) ) ) ) ) A mesma agregao social observada em O Cortio se repete nas favelas urbanas. Os polticos estimulam a criao de cortios como garantia de dominao dos mais fracos. A pobreza dos menos favorecidos motivo de pilhria para os mais abastados. No se observa o engajamento do povo nas questes que so de seu prprio interesse. Os detentores do poder abraam as causas populares como resposta s prprias vocaes democrticas.

A seqncia correta : a) V - V - F - F - V b) V - F - F - V - V c) F - V - V - F - F d) F - F - V - V - F e) F - F - F - V - F

2. O texto estruturado a partir de processos analgicos. Das opes abaixo, assinale a nica que NO contm uma analogia presente no texto. a) b) c) d) e) Passado e presente. Joo Romo e ilustres personalidades. Abolicionismo e Reforma Agrria. Escravos e polticos. Moradores do cortio e populao de baixa renda.

3. Da leitura do primeiro perodo do texto I, podemos inferir que: a) o sucesso de O Cortio se deve precipuamente queles que chegam s suas pginas por interesse prprio. b) o sucesso de O Cortio se deve ao bom gosto daqueles que o elegeram como leitura prazerosa nos dias de hoje. c) o autor afirma que se frui melhor da leitura de O Cortio quando esta advm de um ato espontneo. d) o autor afirma que uma atitude coercitiva em relao leitura de O Cortio incutir o bom gosto em seus leitores. e) uma leitura de O Cortio baseada em vontade prpria que faz o sucesso dessa obra nos dias atuais. 4. No ltimo pargrafo do texto, ao se referir aos muitos Joes Romes, o autor quer salientar, nesse tipo de pessoa, o(a): a) b) c) d) e) oportunismo desmedido. carter empreendedor. abnegao por causas sociais. nobreza de esprito. alienao poltica.

527

PORTUGUS

CESGRANRIO

5. Tendo em vista o texto I, assinale a opo que contm informao correta quanto ao emprego da norma culta. a) O trecho que se passa... abolicionista (li. 15-16) exemplo de orao subordinada adjetiva restritiva. b) O trecho de um apoio organizado (li. 18) um exemplo de objeto indireto. c) Em se acha na histria da luta (li. 28-29), observa-se estrutura em voz ativa. d) Em demanda da sociedade (li. 32), o termo sublinhado exerce funo sinttica de complemento nominal. e) O trecho que na conduo... tem havido polticos (li. 33-35) exemplo de orao subordinada substantiva objetiva direta. 6. Em relao ao estilo do autor, assinale a opo INCORRETA acerca do texto I. a) O advrbio desgraadamente (li. 10) pode ser substitudo, sem alterao de sentido, por infelizmente. b) A ironia que se percebe no final do segundo pargrafo marcada pelo emprego do advrbio gostosamente (li. 13), entre travesses. c) A conjuno porm (li. 15) exprime uma ressalva ao que se disse anteriormente. d) A expresso entre parnteses (os escravos) (li. 21) vem acrescentar um esclarecimento ao enunciado anterior. e) Os parnteses da linha 25 (ou nenhuma) poderiam ser substitudos por vrgulas sem alterar o sentido que ali dado. Texto II Tomava caf, quando um empregado subiu para dizer que l embaixo estava um senhor, acompanhado de duas praas, e que desejava falar ao dono da casa. 5 Vou j, respondeu este. E acrescentou para Botelho: So eles! Deve ser, confirmou o velho. E desceram logo. Quem me procura?... exclamou Joo Romo com disfarce, chegando ao armazm. Um homem alto, com ar de estrina, adiantou-se e entregou-lhe uma folha de papel. Joo Romo, um pouco trmulo, abriu-a defronte dos olhos e leu-a demoradamente. Um silncio formou-se em torno dele: os caixeiros pararam em meio do servio, intimidados por aquela cena em que entrava a polcia. Est aqui com efeito... disse afinal o negociante. Pensei que fosse livre... minha escrava, afirmou o outro. Quer entregar-ma?... Mas imediatamente.

10

15

528

PORTUGUS
20 Onde est ela? Deve estar l dentro. Tenha a bondade de entrar... O sujeito fez sinal aos dois urbanos que o acompanharam logo, e encaminharam-se todos para o interior da casa. Botelho, frente deles, ensinava-lhes o caminho. Joo Romo ia atrs, plido, com as mos cruzadas nas costas. Atravessaram o armazm, depois um pequeno corredor que dava para um ptio calado, chegaram finalmente cozinha. Bertoleza, que havia j feito subir o jantar dos caixeiros, estava de ccoras, no cho, escamando peixe, para a ceia do seu homem, quando viu parar defronte dela aquele grupo sinistro. Reconheceu logo o filho mais velho do seu primitivo senhor, e um calafrio percorreu-lhe o corpo. Num relance de grande perigo compreendeu a situao; adivinhou tudo com a lucidez de quem se v perdido para sempre: adivinhou que tinha sido enganada; que a sua carta de alforria era uma mentira, e que o seu amante, no tendo coragem para matla, restitua-a ao cativeiro. Seu primeiro impulso foi o de fugir. Mal, porm, circunvagou os olhos em torno de si, procurando escapula, o senhor adiantou-se dela e segurou-lhe o ombro. esta! disse aos soldados que, com um gesto, intimaram a desgraada a segui-los. Prendam-na! escrava minha! 45 A negra, imvel, cercada de escamas e tripas de peixe, com uma das mos espalmada no cho e com a outra segurando a faca de cozinha, olhou aterrada para eles, sem pestanejar. Os policiais, vendo que ela se no despachava, desembainharam os sabres. Bertoleza ento, erguendo-se com mpeto de anta bravia, recuou de um salto, e antes que algum conseguisse alcan-la, j de um s golpe certeiro e fundo rasgara o ventre de lado a lado. E depois emborcou para a frente, rugindo e esfocinhando moribunda numa lameira de sangue. Joo Romo fugira at ao canto mais escuro do armazm, tapando o rosto com as mos. Nesse momento parava porta da rua uma carruagem. Era uma comisso de abolicionistas que vinha, de casaca, trazer-lhe respeitosamente o diploma de scio benemrito. Ele mandou que os conduzissem para a sala de visitas.
Alusio de Azevedo. O Cortio

CESGRANRIO

25

30

35

40

50

55

60

529

PORTUGUS

CESGRANRIO

7. Aps uma leitura atenta do texto II, percebe-se que o personagem Bertoleza : a) audaz e persuasivo. b) perspicaz e resoluto. c) sagaz e dissimulado. d) impassvel e tinhoso. e) loquaz e intempestivo.

8. Depreende-se do trecho no tendo coragem (li. 37) idia de: a) causa. b) conseqncia. c) finalidade. 9. O texto II s NO evidencia a: a) b) c) d) e) incoerncia nas atitudes de Joo Romo. superficialidade dos valores daquela sociedade. razo comandando o comportamento de Joo Romo. homenagem recebida pelo efetivo trabalho em prol da causa abolicionista. contribuio de Joo Romo na captura de escravos fugitivos. d) concesso. e) explicao.

10. Em relao predicao verbal, marque a opo em que a classificao apresentada corresponde ao verbo do respectivo exemplo. a) b) c) d) e) Verbo transitivo direto - entrava a polcia (li. 15) Verbo transitivo direto - Quer entregar-ma?... (li. 18) Verbo de ligao - Onde est ela? (li. 20) Verbo intransitivo - ia atrs, plido, com as mos cruzadas nas costas (li. 25) Verbo transitivo direto e indireto - parava porta da rua uma carruagem (li. 58)

11. Dentre as expresses abaixo, aquela que apresenta circunstncia adverbial diferente das demais : a) b) c) d) e) com disfarce (li. 8-9) com a lucidez de quem se v perdido (li. 34-35) com mpeto de anta bravia (li. 50-51) de um salto ( li. 51) com as mos (li. 57)

12. A opo que contm a classificao gramatical correta do vocbulo destacado : a) b) c) d) e) um pouco trmulo (li. 12) - pronome adjetivo indefinido. fez sinal aos dois urbanos (li. 22) - substantivo. que a sua carta de alforria (li. 36) - pronome relativo. Mal, porm, circunvagou (li. 39-40) - advrbio de modo. fugira at ao canto (li. 56) - palavra denotativa de incluso.

530

PORTUGUS
13. Dentre as opes abaixo, s NO se pode afirmar que: a) b) c) d) e)

CESGRANRIO

velho (li. 6) um exemplo de derivao imprpria. intimidados (li. 14-15) uma formao sufixal derivada do parassinttico intimidar. expresso havia j feito subir (li. 28) poderia ser substituda por j fizera subir. aterrada (li. 47) cognata de terra. o vocbulo os (li. 61), no texto, substitui abolicionistas.

14. Assinale a caracterstica do Naturalismo AUSENTE no texto em anlise. a) b) c) d) Enfatiza-se o aspecto fsico, no emocional dos personagens. Os fatos se desenrolam atravs da tica de um observador impessoal. O ser humano encarado sob um ponto de vista determinista e mecanicista. Enfocam-se os aspectos degradantes do ser humano em funo de seu comportamento animal. e) Apresenta-se uma viso objetiva dos fatos e do ambiente. 15. Em relao ao estilo do texto II, INCORRETO afirmar que no: a) reiterado emprego do discurso direto, no trecho compreendido entre as linhas 16 e 21, percebe-se que os personagens se expressam por si mesmos, livres da anlise do narrador. b) perodo Pensei que fosse livre... (li. 17), o emprego das reticncias sugere desfaatez e sarcasmo na atitude do personagem. c) 14 pargrafo, a descrio minuciosa do ambiente confere verossimilhana cena apresentada. d) perodo escrava minha! (li. 43-44), a ordem inversa procura enfatizar a posse do objeto referido. e) trecho Os policiais, vendo... lameira de sangue (li. 49-55), nota-se o emprego enftico de terminologia cientfica. 16. Marque a opo que est de acordo com a norma culta da lngua. a) b) c) d) e) No se distribui aleatoriamente ttulos de benemrito. J devem fazer muitos meses que eles voltaram. Joo Romo usou expresso similar de Botelho. Semelhante atitude implicar em sua excluso do projeto. Essa uma homenagem que no posso me privar.

531

PORTUGUS
Texto III Quando estou, quando estou apaixonado to fora de mim eu vivo que nem sei se vivo ou morto quando estou apaixonado. No pode a fera comigo quando estou, quando estou apaixonado, mas me derrota a formiga se que estou apaixonado. Estarei, quem, e entende, apaixonado neste arco de danao? Ou a morta paixo que me deixa, que me deixa neste estado?
Carlos Drummond de Andrade

CESGRANRIO

17. Feita a anlise do eu-lrico do poema (texto III), s NO vlido afirmar que ele: a) b) c) d) e) perde a noo de si mesmo quando se encontra apaixonado. perdendo seu senso crtico, fica impossibilitado de fazer uma auto-anlise. lamenta seu estado deplorvel e o associa perda do sentimento amoroso. apresenta uma valentia inconstante quando em estado de enlevo. na incerteza de estar apaixonado, sente-se dominado pelos estados de angstia e dvida.

18. Assinale a opo em que se encontra exemplo de elipse. a) b) c) d) e) to fora de mim eu vivo (v. 2) que nem sei se vivo ou morto (v. 3) No pode a fera comigo (v. 5) mas me derrota a formiga (v. 7) Ou a morta paixo (v. 11)

19. Entre as caractersticas da Esttica Modernista, apresentadas nas opes abaixo, apenas uma se encontra no texto III. Marque-a. a) b) c) d) e) Apelo ao inconsciente. Dimenso potica integrada ao cotidiano. Conscincia da fragilidade do eu. Uso de termos apoticos. Linguagem complexa e rebuscada.

532

PORTUGUS

CESGRANRIO

20. Apesar de modernista, esse poema de Drummond apresenta as caractersticas da Esttica do Romantismo relacionadas abaixo, com EXCEO de uma. Indique-a. a) b) c) d) e) Emprego de metforas e antteses. nfase na funo emotiva. Subjetivismo e passionalidade exacerbados. Liberdade dos modelos clssicos. Temtica da separao como origem do sofrimento.

533

PORTUGUS

FUVEST
F A S E

1. Os atuais simuladores de vo militares esto em condies no apenas de exibir uma imagem realista da paisagem sobrevoada, mas tambm de confront-la com a obtida dos radares. O termo que preenche adequadamente a lacuna no texto a) iconologia. b) iconoclastia. c) iconografia. d) iconofilia. e) iconolatria.

2. O diminutivo uma maneira ao mesmo tempo afetuosa e precavida de usar a linguagem. Afetuosa
porque geralmente o usamos para designar o que agradvel, aquelas coisas to afveis que se deixam diminuir sem perder o sentido. E precavida porque tambm o usamos para desarmar certas palavras que, por sua forma original, so ameaadoras demais. [Lus Fernando Verssimo, Diminutivos]

A alternativa inteiramente de acordo com a definio do autor sobre diminutivo : a) b) c) d) e) O iogurtinho que vale por um bifinho. Ser brotinho sorrir dos homens e rir interminavelmente das mulheres. Gosto muito de te ver, Leozinho. Essa menininha terrvel! Vamos bater um papinho.

3. Assinale a alternativa em que a correlao de tempos e modos verbais NO adequada ao contexto. a) Ainda aparecer no Congresso algum disposto a apresentar um projeto que fixe conseqncias para aqueles que enganem a sociedade. b) Tudo leva a crer que nesses cruzamentos de culturas a situao das reas coloniais apresente um convvio de extremos. c) No h dvida de que, nos traumas sociais, os sujeitos da cultura popular sofrem abalos graves. d) More algum nos bairros pobres da periferia de uma cidade grande e ver no que resultou essa condio do migrante. e) A sua conduta ser de inconformismo e violncia, at que um dia certas condies poderiam reconstituir sua vida familiar.

534

PORTUGUS
Texto para as questes 04, 05 e 06.

FUVEST
F A S E

Um historiador da nossa lngua, creio que Joo de Barros, pe na boca de um rei brbaro algumas palavras mansas, quando os portugueses lhe propunham estabelecer ali ao p uma fortaleza; dizia o rei que os bons amigos deviam ficar longe uns dos outros, no perto, para se no zangarem como as guas do mar que batiam furiosas no rochedo que eles viam dali. Que a sombra do escritor me perdoe, se eu duvido que o rei dissesse tal palavra nem que ela seja verdadeira. Provavelmente foi o mesmo escritor que a inventou para adornar o texto, e no fez mal, porque bonita; realmente, bonita. Eu creio que o mar ento batia na pedra, como seu costume, desde Ulisses e antes. Agora que a comparao seja verdadeira que no. Seguramente h inimigos contguos, mas tambm h amigos de perto e do peito. E o escritor esquecia (salvo se ainda no era do seu tempo) esquecia o adgio: longe dos olhos, longe do corao. [Machado de Assis, Dom Casmurro]

4. A frase que exprime o comentrio do narrador sobre a citao do historiador : a) b) c) d) e) As palavras mansas aproximam os amigos, defendendo-os das intempries. A vizinhana fortalece a amizade, ao contrrio do que ocorre entre o mar e o rochedo. Desde Ulisses, o mar batendo no rochedo representa os conflitos da amizade. A inveno dos escritores necessria para compreender melhor fatos ocorridos no passado. A constante proximidade entre amigos contribui para que a discrdia aparea.

5. No trecho, eu duvido que o rei dissesse tal palavra nem que ela seja verdadeira, o termo dissesse expressa uma a) continuidade. b) improbabilidade. c) simultaneidade. d) impossibilidade. e) alternncia.

6. Assinale a alternativa em que o termo sombra tem o mesmo sentido presente no trecho Que a sombra do escritor me perdoe. a) b) c) d) e) A sombra da alegria no lhe ocultava a preocupao. Sentiu invadi-lo a sombra do passado. Quem a v e ouve, hoje, no a conhece, uma sombra do que foi. Recebeu-me de boa sombra, fazendo as honras da casa. Das profundezas desse Reino, sobe a sombra ao meu encalo.

535

PORTUGUS
7. A nica frase inteiramente de acordo com as normas gramaticais do padro culto :

FUVEST
F A S E

a) A secretria pretende evitar que novos mandados de segurana ou liminares contra o decreto sejam expedidas. b) O CONTRU interditou vrias dependncias do prdio, inclusive o Salo Azul, cujo o madeiramento do forro foi atacado por cupins. c) O ministro da Agricultura da Inglaterra declarou que por hora no h motivo para sacrificar os animais. d) A poucos dias da eleio, os candidatos enfrentam agora uma verdadeira maratona. e) Posso venc-las, mesmo que usem drogas, pois no isso que as tornaro invencveis, declarou a nadadora. 8. Assinale a nica frase em que a ordem de colocao das palavras NO produz ambigidade. a) b) c) d) e) Rossi pede ao STF processo por calnia contra Motta. s colocar as moedas, girar a manivela e ter a escova j com pasta embalada nas mos. Casal procura filho seqestrado via Internet. Cmara torna crime porte ilegal de armas. Regressou a Braslia depois de uma cirurgia cardaca com cerimonial de chefe de Estado.

9. Quando eu estava na escola mdica de Boston, tive a sorte de fazer parte de um pequeno grupo de
estudantes que se reuniu informalmente com um clebre cardiologista, homem na casa dos oitenta anos. Num dado momento, um dos estudantes comentou com certa apreenso que as doenas cardacas eram a causa nmero um de morte nos Estados Unidos. O velho professor pensou sobre isso por alguns instantes e depois replicou: O que que voc preferia ter como causa nmero um de morte? [David Ehrenfeld, A Arrogncia do Humanismo]

A rplica do sutil professor ao estudante sugere que a) b) c) d) e) a especializao do saber aprimora a viso de conjunto. o avano tecnolgico nem sempre implica humanizao. a fixao no poder da cincia faz esquecer seus limites. a modernizao apressada costuma obrigar a recuos. a pesquisa perseverante contorna os maiores obstculos.

10. Pode ser substituda por dissenso, sem que se altere o sentido da frase apresentada, apenas a palavra destacada em: a) b) c) d) e) A discordncia entre as opinies inviabilizou qualquer acordo. A disparidade de oportunidades dificulta a concretizao de uma verdadeira democracia. Os que se envolveram na discusso do assunto no chegaram a um acordo. A mudana acelerada dos costumes levou dissoluo da famlia. A dissimulao das verdadeiras intenes no lhe garantiu chegar aos fins desejados.

536

PORTUGUS

FUVEST
F A S E

11. Conta Rubem Braga o conselho que um amigo lhe deu certa vez: Olhe, Rubem, faa como eu, no tope parada com a gramtica. Tratando Rubem por tu e respeitando o padro culto, o amigo deveria dizer: a) b) c) d) e) Olhai, Rubem, faz como eu, no enfrente a gramtica. Olhai, Rubem, faze como eu, no te vs atemorizar com a gramtica. Olha, Rubem, faas como eu, cuide de seguir a gramtica. Olhe, Rubem, faas como eu, evita fugir gramtica. Olha, Rubem, faz como eu, no desafies a gramtica.

12. Galileu duvidou tanto de Aristteles quanto das Escrituras. A mesma noo expressa pelo par destacado est tambm em: a) b) c) d) e) A criana tanto chorou que a me comprou o brinquedo. Quer voc queira, quer no, partimos amanh. No s o argumento falso, como o discurso todo mente. Ele apresentou de tal forma os fatos que convenceu a todos. Ele mais bradou que verdadeiramente lutou contra a opinio pblica.

13. A catacrese, figura que se observa na frase Montou a cavalo no burro bravo, ocorre em: a) b) c) d) e) Os tempos mudaram, no devagar depressa do tempo. ltima flor do Lcio, inculta e bela, s a um tempo esplendor e sepultura. Apressadamente, todos embarcaram no trem. mar salgado, quanto do teu sal so lgrimas de Portugal. Amanheceu, a luz tem cheiro.

Texto para as questes 14 e 15.


Navegar preciso, viver no preciso. Esta frase de antigos navegadores portugueses, retomada por Fernando Pessoa, por Caetano Veloso e sabe-se l por quantos mais citadores ou reinventores, ganha sua ltima verso no mbito da Informtica, em que o termo navegar adquire outro e preciso sentido. Na nova acepo, em tempos de Internet, o lema parece mais afirmativo do que nunca. Os olhos que hoje vagueiam pela tela iluminada do monitor j no precisam nem de velas, nem de versos, nem de fados: da vida s querem o cantinho de um quarto, de onde fazem o mundo flutuar em mares de virtualidades nunca dantes navegados.

537

PORTUGUS
14. Considere as seguintes afirmaes:

FUVEST
F A S E

I - A significao das palavras constitui um processo dinmico e supe o reconhecimento histrico de seu emprego. II - As expresses velas, fados e nunca dantes navegados ligam-se ao contexto primitivo do velho lema. III - Desligando-se de suas razes histricas, as palavras apresentam-se esvaziadas de qualquer sentido. Conforme se pode deduzir do texto, est correto o que se afirma a) apenas em I e II. b) apenas em I e III. c) apenas em II e III. d) apenas em I. e) em I, II e III.

15. Indique a afirmao correta em relao ao texto. a) O efeito sonoro explorado na seqncia de vagueiam, velas, versos, vida, virtualidade conhecido como rima interior. b) A construo Os olhos () j no precisam exemplo de metonmia. c) O termo vagueiam est empregado no sentido de norteiam e exemplo de personificao. d) Na frase Navegar preciso, viver no preciso h um pleonasmo. e) A construo nem de velas, nem de versos, nem de fados apia-se em antteses. 16. A casa que papai alugara no ficava na praia exatamente, mas numa das ruas que a ela davam e
onde uns operrios trabalhavam diariamente no alinhamento de um dos canais que carreavam o enxurro da cidade para o mar do golfo. [Mrio de Andrade]

No perodo acima, o segmento que a ela davam e onde pode ser substitudo, sem prejuzo para o sentido original do perodo, por a) b) c) d) e) para a cuja iam, nas quais que lhe conduziam, aonde qual cortavam, em cuja rua nela terminavam, s quais que nela desembocavam, rua em que

17. Rememorando o papel de sua gerao, dizia o artista: Ns no estvamos, apenas, na exposio; ns ramos a exposio. Alterando-se a ordem das palavras, a frase do artista NO tem seu sentido alterado em: a) b) c) d) e) Apenas, ns no estvamos na exposio; ns ramos a exposio. Ns no estvamos; apenas na exposio ns ramos a exposio. Ns no estvamos na exposio apenas ns; ramos a exposio. Ns no apenas estvamos na exposio; ns ramos a exposio. Ns no estvamos na exposio; ns ramos apenas a exposio.

538

PORTUGUS
18.
Finado Severino, quando passares em Jordo e os demnios te atacarem perguntando o que que levas Dize que levas somente coisas de no: fome, sede, privao.

FUVEST
F A S E

As coisas de sim esto, correspondentemente, em: a) b) c) d) e) saciedade fartura repleo satisfao vacuidade repleo carncia carncia saciedade fartura carncia. vacuidade. saciedade. fartura. repleo.

19. Indique a alternativa em que a aproximao estabelecida est correta. a) A terra paradisaca, em Gonalves Dias, projeo nacionalista; a Pasrgada, de Manuel Bandeira, anseio intimista. b) O lirismo de Gregrio de Matos conflitivo e confessional; o de Cludio Manuel da Costa sereno e impessoal. c) A fico regionalista, imatura no sculo XIX, ganhou fora ao abraar as teses do determinismo cientfico, no sculo XX. d) Jos de Alencar buscou expressar nossa diversidade cultural projeto que s a obra de Machado de Assis viria a realizar. e) A figura do malandro, positiva em Manuel Antnio de Almeida, o alvo de Mrio de Andrade em sua stira Macunama. 20. Indique a afirmao correta sobre o Auto da Barca do Inferno, de Gil Vicente: a) intrincada a estruturao de suas cenas, que surpreendem o pblico com o inesperado de cada situao. b) O moralismo vicentino localiza os vcios no nas instituies, mas nos indivduos que as fazem viciosas. c) complexa a crtica aos costumes da poca, j que o autor o primeiro a relativizar a distino entre o Bem e o Mal. d) A nfase desta stira recai sobre as personagens populares, as mais ridicularizadas e as mais severamente punidas. e) A stira aqui demolidora e indiscriminada, no fazendo referncia a qualquer exemplo de valor positivo.

539

PORTUGUS

FUVEST
F A S E

21. I - A manobra psicolgica do narrador-protagonista mostra que as lembranas obsessivas perdem a fora, quando o seu culto as eleva a uma respeitosa distncia. II - A paisagem paulistana percorrida pelo olhar ingnuo de quem busca reconhecimento e esbarra no oficialismo autoritrio. As afirmaes I e II referem-se, respectivamente, aos seguintes contos de Mrio de Andrade (Contos Novos): a) b) c) d) e) 22. Vestida de preto e Primeiro de maio. Peru de natal e Frederico Pacincia. O ladro e Frederico Pacincia. Peru de natal e Primeiro de maio. Vestida de preto e O ladro.

Em frente do meu leito, em negro quadro A minha amante dorme. uma estampa De bela adormecida. A rsea face Parece em visos de um amor lascivo De fogos vagabundos acender-se

Esses versos de lvares de Azevedo, da Lira dos Vinte Anos, apiam a seguinte afirmao sobre o conjunto Idias ntimas, de onde foram extrados: a) Em versos brancos e em ritmo fluente, o discurso potico combina notaes realistas e fantasias amorosas. b) A lascvia, combinada com a stira, elimina a possibilidade de lirismo amoroso, reservado para a segunda parte do livro. c) No espao do quarto, o poeta vinga-se das frustraes amorosas, satirizando a imagem de sua amada. d) Imaginando-se pintor, o poeta vai esboando num quadro as figuras da virgem romntica e da amante calorosa. e) Os decasslabos e o lirismo intimista so traos que j fazem antever as tendncias poticas da gerao seguinte. 23. Indique a alternativa que se refere corretamente ao protagonista de Memrias de um Sargento de Milcias, de Manuel Antnio de Almeida. a) Ele uma espcie de barro vital, ainda amorfo, a que o prazer e o medo vo mostrando os caminhos a seguir, at sua transformao final em smbolo sublimado. b) Enquanto cnico, calcula friamente o carreirismo matrimonial; mas o sujeito moral sempre emerge, condenando o prprio cinismo ao inferno da culpa, do remorso e da expiao. c) A personalidade assumida de stiro a mscara de seu fundo lrico, genuinamente puro, a ilustrar a tese da bondade natural, adotada pelo autor. d) Este heri de folhetim se d a conhecer sobretudo nos dilogos, nos quais revela ao mesmo tempo a malcia aprendida nas ruas e o idealismo romntico que busca ocultar. e) Nele, como tambm em personagens menores, h o contnuo e divertido esforo de driblar o acaso das condies adversas e a avidez de gozar os intervalos da boa sorte.

540

PORTUGUS

FUVEST
F A S E

24. Com essa histria enjoada de traiu ou no traiu, de Capitu ser anjo ou demnio, o leitor de Dom Casmurro acaba se esquecendo do fundamental: as memrias so do velho narrador, no da mulher, e o autor Machado de Assis, e no um escritor romntico dividido entre mistrios. Aceitas as observaes acima, o leitor de Dom Casmurro dever a) identificar o ponto de vista de Capitu, considerando ainda o universo prprio da fico naturalista. b) reconhecer os limites do tipo de narrador adotado, subordinando-o ao peculiar universo de valores do autor. c) aceitar os juzos do velho narrador, por meio de quem se representa a ndole confessional de Machado de Assis. d) rejeitar as acusaes do jovem Bentinho, preferindo-lhes a relativizao promovida pelo velho narrador. e) relativizar o ponto de vista da narrao, cuja ambigidade se deve personalidade oblqua de Capitu. 25. Quando o velho acabou de escrever a sua narrativa exclamei:
Tenho a impresso de que o senhor deixou as pernas embaixo de um automvel, Seu Ribeiro. Por que no andou mais depressa? o diabo.

A correta contextualizao da passagem acima no romance So Bernardo, de Graciliano Ramos, permite afirmar que a) a brutalidade de Paulo Honrio no exclui a modernizao e a tcnica, de que tambm se vale para triunfar sobre um mundo em que Seu Ribeiro j tivera prestgio. b) Seu Ribeiro buscara inutilmente acompanhar o ritmo do progresso, j que lhe faltava o instinto empreendedor capaz de realizar sua aspirao de ser um grande proprietrio. c) as lamrias do velho coronel prendem-se a um mundo mais autoritrio, vencido pelas idias modernas que deram origem e sustentao ao poder de Paulo Honrio. d) o oportunismo de Paulo Honrio leva-o a preterir os servios do antigo agregado em favor da jovem Madalena, a quem incumbe de equipar e modernizar a velha escola. e) o universo de valores do velho Major desorganizou-se em funo da velocidade do mundo moderno, ao qual tampouco se adapta o primitivismo de Paulo Honrio. 26. correto afirmar que no poema dramtico Morte e Vida Severina, de Joo Cabral de Melo Neto, a) a sucesso de frustraes vividas por Severino faz dele um exemplo tpico de heri moderno, cuja tragicidade se expressa na rejeio cultura a que pertence. b) a cena inicial e a final dialogam de modo a indicar que, no retorno terra de origem, o retirante estar munido das convices religiosas que adquiriu com o mestre carpina. c) o destino que as ciganas prevem para o recm-nascido o mesmo que Severino j cumprira ao longo de sua vida, marcada pela seca, pela falta de trabalho e pela retirada. d) o poeta buscou exprimir um aspecto da vida nordestina no estilo dos autos medievais, valendo-se da retrica e da moralidade religiosa que os caracterizavam. e) o auto de natal acaba por definir-se no exatamente num sentido religioso, mas enquanto reconhecimento da fora afirmativa e renovadora que est na prpria natureza.

541

PORTUGUS
LNGUA E LITERATURA

FUVEST
F A S E

1. Chamar o dicionrio de pai-dos-burros que burrice. Reconhecer um desconhecimento no uma virtude? Se a burrice costuma vir sempre acompanhada da insolncia, a inteligncia no dispensa a fora da humildade. a) Reescreva os dois primeiros perodos, substituindo os verbos chamar e reconhecer por substantivos que no sejam da mesma famlia desses verbos. Faa apenas as adaptaes necessrias, mantendo o sentido original. b) Reescreva o ltimo perodo do texto, utilizando agora as formas no costuma e dispensa. Faa apenas as alteraes necessrias, mantendo o sentido original. 2. Assim deveria ser a relao de autor para leitor: uma face nua num espelho lmpido. Mas to difcil...
Ou a face est mascarada ou o espelho embaciado [Mrio Quintana]

a) Explique como que Mrio Quintana caracteriza, em sua concepo idealizada, I - o autor; II - o leitor. b) Reescreva o ltimo perodo do texto, iniciando-o com Mesmo que a face.... Faa apenas as alteraes necessrias, preservando a idia original do autor.

3. Se, como diz o ditado, ter um pouco e dois que seria o bom, quando se trata do Trio Beaux Arts,
os trs so demais. No melhor sentido da palavra. [O Estado de So Paulo, 12/04/96, D4)

a) Qual o melhor sentido da palavra a que o autor se refere? b) Qual o contra-senso que ele evitou, ao acrescentar a ressalva no melhor sentido da palavra? 4. No dia 19, Juscelino registrou a amargura que lhe dominava: No estou bem por dentro, anotou. Uma
das razes que tornaram (sic) triste a longa permanncia na fazenda a ausncia de alguns amigos. [O Estado de S.Paulo, 14/03/96, A7]

Usa-se sic entre parnteses, numa citao, para indicar que o texto original aquele mesmo, por errado ou estranho que parea. a) Apresente uma justificativa para aceitar ou no o sic usado pelo autor do texto. b) H no texto uma construo que justifica o emprego do sic. Transcreva-a, aplicando o sic no lugar adequado.

542

PORTUGUS
5. AS COBRAS / Luis Fernando Verissimo

FUVEST
F A S E

[O Estado de S. Paulo, 18/04/96, D4] a) Explique o jogo de palavras que faz a graa da tira. b) Identifique a diferena no uso da linguagem em que se apoia esse jogo. 6.
Ser consciente talvez um esquecimento. Talvez pensar um sonho seja, ou um sono. Talvez dormir seja, um momento, Voltar o spirito nosso a ser dono. [Fernando Pessoa]

a) O trecho acima, do ponto de vista da composio, classifica-se como descritivo, narrativo ou dissertativo? b) Justifique sua resposta, transcrevendo pelo menos dois elementos do texto. 7. O fragmento abaixo da novela Campo geral (Miguilim), de Joo Guimares Rosa.
E o Dito mesmo gostava, pedia: Conta mais, conta mais... Miguilim contava, sem carecer de esforo, estrias compridas, que ningum nunca tinha sabido, no esbarrava de contar, estava to alegre nervoso, aquilo para ele era o entendimento maior.

a) As qualidades aqui atribudas ao Miguilim contador de histrias aproximam-no ou distanciam-no do modo de narrar que celebrizou Guimares Rosa? Justifique sua resposta. b) O desfecho da novela estaria a sugerir que Miguilim encontrar limitaes para desenvolver suas qualidades de contador de histrias? Justifique sua resposta. 8. Em O Poo, de CONTOS NOVOS, as personagens principais so de carne e osso, mas o silencioso protagonista do conto mesmo aquele Poder que vem de muito longe, do nosso passado colonial e das relaes escravistas, aqui encarnado no autoritarismo caprichoso e arbitrrio que se arroga todos os direitos, que se impe ao absolutamente fraco e submisso e que chega a se desnortear diante da dignidade custosa de um gesto de rebeldia. a) Destaque do texto, estritamente, uma expresso que se refira a cada uma das personagens: Albino (irmo mais novo), Jos (irmo mais velho), Joaquim Prestes. b) Alm de sua forte presena material, o poo que est no centro deste conto pode ser tomado como um smbolo expressivo. Procede a afirmao acima? Justifique sua resposta.

543

PORTUGUS

FUVEST
F A S E

9. Leia com ateno os seguintes versos de Joo Cabral de Melo Neto em Morte e vida severina:
E belo porque com o novo todo o velho contagia. Belo porque corrompe com sangue novo a anemia. Infecciona a misria com vida nova e sadia. Com osis, o deserto, com ventos, a calmaria.

a) Contextualize esses ventos no poema de Joo Cabral, indicando o evento a que se referem e a relao desse evento com a fala final de Seu Jos ao retirante. b) Que valor deu o poeta aos verbos contagiar, corromper e infeccionar no contexto da estrofe acima? Explique. 10. I - Como que sabes, Foi Blimunda que viu, o padre virou-se para ela, sorriu, olhou um e olhou
outro, e declarou, Tu s Sete-Sis porque vs s claras, tu sers Sete-Luas porque v as escuras...

II - E uma nuvem fechada est no centro de seu corpo. Ento Blimunda disse, Vem. Desprendeuse a vontade de Baltasar Sete-Sis, mas no subiu s estrelas, se terra pertencia e a Blimunda.

a) No contexto de Memorial do Convento, por qual razo o padre Bartolomeu Loureno atribui a alcunha a Blimunda (excerto I)? b) Levando em conta que o Sol e Lua pertencem a momentos diversos do espao de um dia, como interpretar a soluo que marca o final do romance (excerto II)? REDAO 1 Redija uma DISSERTAO em prosa, relacionando os trs textos abaixo. Texto 1 Na prova de Redao dos vestibulares, talvez a verdadeira questo seja sempre a mesma: Conseguirei?. Cada candidato aplica-se s reflexes e s frases na difcil tarefa de falar de um tema A proposto, com a preocupao em B Conseguirei? -, para convencer um leitor X. Texto 2 Ao escrever Lutar com palavras / a luta mais v. / Entanto lutamos / mal rompe a manh, Carlos Drummond de Andrade j era um poeta maior da nossa lngua. Texto 3
difcil defender, s com palavras, a vida [Joo Cabral de Melo Neto]

544

PORTUGUS
QUESTES DE 1 A 10

UFBA
F A S E

INSTRUO: Assinale as proposies verdadeiras, some os nmeros a elas associados e marque o resultado na Folha de Respostas. QUESTES DE 1 A 4 TEXTO:

O que faz o brasil, Brasil? A questo da identidade


Como se constri uma identidade social? Como um povo se transforma em Brasil? A pergunta, na sua discreta singeleza, permite descobrir algo muito importante. que no meio de uma multido de experincias dadas a todos os homens e sociedades, algumas necessrias prpria sobrevivncia, como comer, 5 dormir, morrer, reproduzir-se etc., outras acidentais ou superficiais: histricas, para ser mais preciso o Brasil foi descoberto por portugueses e no por chineses, a geografia do Brasil tem certas caractersticas como as montanhas na costa do Centro-Sul, sofremos presso de certas potncias europias e no de outras, falamos portugus e no francs, a famlia real transferiu-se para o Brasil no incio 10 do sculo XIX etc., cada sociedade (e cada ser humano) apenas se utiliza de um nmero limitado de coisas (e de experincias) para construir-se como algo nico, maravilhoso, divino e legal... Sei, ento, que sou brasileiro e no norte-americano, porque gosto de comer feijoada e no hambrguer, porque sou menos receptivo a coisas de outros pases, 15 sobretudo costumes e idias; (...) porque sei que no existe jamais um no diante de situaes formais e que todas admitem um jeitinho pela relao pessoal e pela amizade; porque entendo que ficar malandramente em cima do muro algo honesto, necessrio e prtico no caso do meu sistema; porque acredito em santos catlicos e tambm nos orixs africanos; porque sei que existe destino e, no entanto, tenho f no 20 estudo, na instruo e no futuro do Brasil; porque sou leal a meus amigos e nada posso negar a minha famlia; porque, finalmente, sei que tenho relaes pessoais que no me deixam caminhar sozinho neste mundo, como fazem os meus amigos americanos, que sempre se vem e existem como indivduos! Pois bem: somando esses traos, forma-se uma seqncia que permite dizer 25 quem sou, em contraste com o que seria um americano, aqui definido pelas ausncias ou negativas que a mesma lista efetivamente comporta. A construo de uma identidade social, ento, como a construo de uma sociedade, feita de afirmativas e de negativas diante de certas questes.Tome uma lista de tudo o que voc considera importante leis, idias relativas famlia, casamento e sexualidade; dinheiro; poder 30 poltico; religio e moralidade; artes; comida e prazer em geral e com ela voc poder saber quem quem. No de outro modo que se realizam as pesquisas antropolgicas e sociolgicas. Descobrindo como as pessoas se posicionam e atualizam as coisas desta lista, voc far um inventrio de identidades sociais e de sociedades. Isso lhe permitir descobrir o estilo e o jeito de cada sistema. Ou, como 35 se diz em linguagem antropolgica, a cultura ou ideologia de cada sociedade. Porque, para mim, a palavra cultura exprime precisamente um estilo, um modo e um jeito, repito, de fazer coisas.

545

PORTUGUS
Mas preciso no esquecer que essas escolhas seguem uma ordem. certo que eu inventei um brasileiro e um americano que o acompanhava por contraste linhas atrs, mas quem me garante que aquilo que disse convincente para definir um brasileiro foi a prpria sociedade brasileira. Ou seja: quando eu defini o brasileiro como sendo amante do futebol, da msica popular, do carnaval, da comida misturada, dos amigos e parentes, dos santos e orixs etc., usei uma frmula que me foi fornecida pelo Brasil.(...) Tudo isso nos leva a descobrir que existem dois modos bsicos de construir a identidade brasileira: o de fazer o brasil, Brasil... Num deles, utilizamos dados precisos: as estatsticas demogrficas e econmicas, os dados do PIB, PNB e os nmeros da renda per capita e da inflao, que sempre nos assusta e apavora. Falamos tambm dos dados relativos ao sistema poltico e educacional do pas, apenas para constatar que o Brasil no aquele pas que gostaramos que fosse. Essa classificao permite construir uma identidade social moderna, de acordo com os critrios estabelecidos pelo Ocidente europeu a partir da Revoluo Francesa e da Revoluo Industrial. Aqui, somos definidos por meio de critrios objetivos, quantitativos e claros. (...) Mas, no caso do Brasil e de outras sociedades, o problema que existe outro modo de classificao. A identidade se constri duplamente. Por meio dos dados quantitativos, pelos quais somos sempre uma coletividade que deixa a desejar, e por meio de dados sensveis e qualitativos pelos quais nos podemos ver a ns mesmos como algo que vale a pena. Aqui, o que faz o brasil, Brasil no mais a vergonha do regime ou a inflao galopante e sem vergonha, mas a comida deliciosa, a msica envolvente, a saudade que humaniza o tempo e a morte, e os amigos que permitem resistir a tudo. uma descoberta importante, creio, dizer que ns temos dado muito mais ateno a um s desses eixos classificatrios, querendo discutir o Brasil apenas como uma questo de modernidade e de economia e poltica; ou, ao contrrio, reduzindo sua realidade a um problema de famlia, de relaes pessoais e de cordialidade. Para mim, no se trata nem de uma coisa nem de outra, mas das duas que so dadas de modo simultneo e complexo. Nessa perspectiva, (...) a chave para entender a sociedade brasileira uma chave dupla. De um lado, ela moderna e eletrnica, mas de outro uma chave antiga e trabalhada pelos anos. tpica de nosso sistema essa capacidade de misturar e acasalar as coisas que tenho discutido no meu trabalho como uma atividade relacional, de ligar e descobrir um ponto central. (...) Sustento que, enquanto no formos capazes de discernir essas duas faces de uma mesma nao e sociedade, estaremos fadados a um jogo cujo resultado j se sabe de antemo. Pois, como ocorre com as moedas, ou teremos como jogada um brasil, pequeno e defasado das potncias mundiais, Brasil que nos leva a uma autoflagelao desanimadora; ou teremos como jogada o Brasil dos milagres e dos autoritarismos polticos e econmicos, que periodicamente entra numa crise. Ser preciso, portanto, discutir o Brasil como uma moeda. Como algo que tem dois lados. E mais: como uma realidade que nos tem iludido, precisamente porque nunca lhe propusemos esta questo relacional e reveladora: afinal de contas, como se ligam a duas faces de uma mesma moeda? O que faz o brasil, Brasil?

UFBA
F A S E

40

45

50

55

60

65

70

75

80

DAMATTA, Roberto. O que faz o brasil, Brasil? 7 ed. Rio de Janeiro: Rocco, 1994. p. 16-20 (texto adaptado).

546

PORTUGUS
Questo 1 De acordo com o texto, uma identidade social resulta:

UFBA
F A S E

(1) da sntese de todas as experincias de um povo no espao e no tempo de sua histria. (2) da assimilao e perpetuao de experincias exgenas importantes no processo de autodefinio de um povo. (4) da seleo de um nmero restrito de experincias de um determinado grupo, capazes de individualiz-lo. (8) de um processo de afirmao e negao de caractersticas, traos e idias a respeito do mundo, da vida e das coisas. (16) de uma ruptura com valores historicamente consagrados, seguida da construo de um novo sistema de valores. (32) de uma autodefinio referente a traos socioeconmicos e culturais diferenciadores e individualizadores de uma coletividade. Questo 2 Segundo o autor, uma compreenso profunda da identidade brasileira exige: (1) a opo por um determinado ngulo de anlise do pas, do seu povo e de suas caractersticas socioeconmicas e culturais. (2) a associao entre uma auto-imagem do pas e uma hetero-imagem, construda por naes mais desenvolvidas. (4) a descoberta das relaes entre uma imagem moderna, construda a partir de dados quantitativos, e uma outra histrica, formada a partir de dados antropolgicos. (8) a busca de razes histricas capazes de explicar o presente e determinar as bases de construo do futuro. (16) uma anlise que evite o vis de ou se considerar o pas a partir de parmetros que o depreciem, ou de exalt-lo nas suas peculiaridades. (32) um levantamento exaustivo dos diversos modos como os americanos vem e julgam os brasileiros. (64) uma interpretao da complexidade de traos e da simultaneidade de perspectivas analticas do que configura, hoje, a nao brasileira. Questo 3 Expresses cujo sentido se entrelaa, no contexto, em torno de uma mesma idia esto agrupadas em: (1) identidade social (li. 1); algo nico (li. 11); traos (li. 24). (2) jeitinho (li. 16); destino (li. 19); indivduos (li. 23). (4) estatsticas demogrficas e econmicas (li. 47-8); jogo (li. 73); potncias mundiais (li. 75). (8) identidade social moderna (li. 51-2); critrios objetivos (li. 54); autoflagelao (li. 75). (16) dados sensveis (li. 57); algo que vale a pena (li. 58); cordialidade (li.65). (32) eixos classificatrios (li. 63); chave dupla (li. 68); faces (li. 72). (64) atividade relacional (li. 71); nao e sociedade (li. 72-3); crise (li. 77).

547

PORTUGUS
Questo 4 O sentido do trecho transcrito est devidamente mantido em:

UFBA
F A S E

(1) ...sofremos presso de certas potncias europias e no de outras... (li. 8) em certas potncias europias sofremos presso e no em outras (2) ...cada sociedade (...) apenas se utiliza de um nmero limitado de coisas... (li. 10-1) cada sociedade utiliza apenas um nmero limitado de coisas. (4) ...quando eu defini o brasileiro como sendo amante do futebol, (...) usei uma frmula que me foi fornecida pelo Brasil. (li. 41-4) ...o Brasil forneceu a frmula por mim usada, ao definir o brasileiro como amante do futebol. (8) Tudo isso nos leva a descobrir que existem dois modos bsicos de construir a identidade brasileira... (li. 45-6) Tudo isso nos leva descoberta de que existem dois modos bsicos de construo da identidade brasileira... (16) De um lado, ela moderna e eletrnica, mas de outro uma chave antiga e trabalhada pelos anos. (li. 68-9) De um lado, ela moderna e eletrnica, embora, de outro, seja uma chave antiga trabalhada pelos anos. (32) enquanto no formos capazes de discernir essas duas faces... (li. 72) ...apesar de no sermos capazes para discernir essas duas faces... (64) ...o Brasil dos milagres e dos autoritarismos polticos e econmicos, que periodicamente entra numa crise. (li. 76-7) ...o Brasil dos milagres e dos autoritarismos polticos e econmicos, os quais periodicamente entram numa crise.

548

PORTUGUS
QUESTES 5 E 6 TEXTO:

UFBA
F A S E

AO ROMPER DALVA

Sigo s caminhando serra acima, E meu cavalo a galopar se anima Aos bafos da manh. A alvorada se eleva do levante. E, ao mirar na lagoa seu semblante, Julga ver sua irm.

25

30

Mas o que vejo? um sonho!... A barbaria Erguer-se neste sculo, luz do dia. Sem pejo se ostentar. E a escravido nojento crocodilo Da onda turva espulso l do Nilo Vir aqui se abrigar!... Oh! Deus! no ouves dentre a imensa orquestra Que a natureza virgem manda em festa Soberba, senhoril, Um grito que solua aflito, vivo, O retinir dos ferros do cativo, Um som discorde e vil? Senhor, no deixes que se manche a tela Onde traaste a criao mais bela De tua inspirao. O sol de tua glria foi toldado... Teu poema da Amrica manchado, Manchou-o a escravido.

10

Tudo luz, tudo aroma e murmrio. A barba branca da cascata o rio Faz orando tremer. No descampado o cedro curva a frente, Folhas e prece aos ps do Onipotente Manda a lufada erguer. Terra de Santa Cruz, sublime verso Da epopia gigante do universo, Da imensa criao. Com tuas matas, ciclopes de verdura, Onde o jaguar, que passa na espessura, Roja as folhas no cho; Como s bela, soberba, livre, ousada! Em tuas cordilheiras assentada A liberdade est. A prpura da bruma, a ventania Rasga, espedaa o cetro que serguia Do rijo piqui.

35

40

15

20

45

Oh! ver no posso este labu maldito! Quando dos livres ouvirei o grito? Sim... talvez amanh. Galopa, meu cavalo, serra acima! Arranca-me a este solo. Eia! te anima Aos bafos da manh!

CASTRO ALVES. Os escravos. In: . Obra completa. Rio de Janeiro: Jos Aguilar, 1960. p. 215-7.

549

PORTUGUS
Questo 5 Constitui sntese do poema:

UFBA
F A S E

(1) O poeta extravasa o seu sentir mais profundo ao constatar a presena da escravido em meio natureza harmoniosa e exuberante da sua terra natal. (2) O poeta, assumindo uma causa em favor da preservao da natureza, implora a ajuda divina e humana nessa rdua tarefa. (4) O poeta, diante da indiferena de seus compatriotas frente escravido e devastao da natureza, dominado pelo assombro e pela indignao. (8) O poeta considera a escravido uma mancha humana na obra divina da criao de seu territrio ptrio. (16) O poeta, face beleza e pujana de sua terra natal, revolta-se diante do atroz espetculo da escravido. (32) O poeta assume o papel de arauto dos brasileiros e promete engajar sua pena na luta contra a escravido. (64) O poeta blasfema contra Deus por ter criado to grande contraste entre uma natureza bela e uma humanidade injusta e cruel.

Questo 6 H um comentrio adequado aos recursos de linguagem usados pelo autor, em: (1) caminhando serra acima (v. 1) e se eleva do levante (v. 4) nivelam o homem e a natureza numa trajetria ascensional. (2) ciclopes de verdura (v. 16), jaguar (v. 17) e espessura (v. 17) transmitem sensao de agressividade. (4) No verso 19, bela, soberba, livre, ousada expressam atributos pertinentes liberdade. (8) Nos versos 19 e 25, a exclamao sugere embevecimento. (16) No verso 23, espedaa tem efeito sugestivo semelhante a Rasga. (32) Sem pejo (v. 27), nojento (v. 28) e turva (v. 29) expressam sentimentos contraditrios experimentados pelo poeta. (64) O emprego da 1 pessoa em posso (v. 43) e ouvirei (v. 44) indicador do envolvimento do eu do poeta.

550

PORTUGUS
QUESTES 7 E 8 As questes 7 e 8 baseiam-se no romance Clara dos Anjos de Lima Barreto. Questo 7

UFBA
F A S E

Nesse romance, um conjunto de situaes densamente articuladas conduzem a um desfecho dramtico. Essas situaes esto ilustradas em: (1) Rebelde, desde tenra idade, a doura para com ele, por parte de sua me, e os prejuzos dela impediram-na que o corrigisse convenientemente, assiduamente no tempo prprio. No ia ao colgio; fazia gazeta, correndo pelas matas das cercanias da residncia dos pais, ento em Itapiru, com outros garotos. O que faziam, pode-se bem adivinhar; mas a me fingia no perceber, passava a mo pela cabea do filho querido, nada dizia ao pai, que quase mourejava durante as vinte e quatro horas do dia. Cresceu assim, sem nenhuma fora moral que o comprimisse; e o pai seria a nica.
(LIMA BARRETO. Clara dos Anjos . 8 ed. So Paulo: tica, 1995. p. 33-4)

(2) Escolhia bem a vtima, simulava amor, escrevia detestavelmente cartas langorosas, fingia sofrer, empregava, enfim, todo o arsenal do amor antigo, que impressiona tanto a fraqueza de corao das pobres moas daquelas paragens, nas quais a pobreza, a estreiteza de inteligncia e a reduzida instruo concentram a esperana de felicidade num Amor, num grande e eterno Amor, na Paixo correspondida.
(Ibid. p. 35)

(4) Habituada s musicatas do pai e dos amigos, crescera cheia de vapores de modinhas e enfumaara a sua pequena alma de rapariga pobre e de cor com os dengues e o simplrio sentimentalismo amoroso dos descantes e cantarolas populares.
(Ibid. p. 42-3)

(8) Engrcia recebeu boa instruo, para a sua condio e sexo; mas, logo que se casou como em geral acontece com as nossas moas , tratou de esquecer o que tinha estudado. O seu consrcio com Joaquim, ela o efetuara na idade de dezoito anos. Fosse a educao mimosa que recebera, fosse uma fatalidade de sua compleio individual, o certo que, a no ser para os servios domsticos, Engrcia evitava todo o esforo de qualquer natureza.
(Ibid. p. 53)

(16) A gente pobre difcil de se suportar mutuamente; por qualquer ninharia, encontrando ponto de honra, brigando, especialmente as mulheres. (...) Uma diferena acidental de cor causa para que possa se julgar superior vizinha; o fato do marido desta ganhar mais do que o daquela outro. Um belchior de mesquinharias aula-lhes a vaidade e alimenta-lhes o despeito.
(Ibid. p. 73)

(32) At ali, ele contava com a benevolncia secreta de juzes e delegados, que, no ntimo, julgavam absurdo o casamento dele com as suas vtimas, devido diferena de educao, de nascimento, de cor, de instruo.
(Ibid. p.75-6)

(64) Ento, voc no me conhece mais, seu canaia? Ento voc no si lembra da Ins, aquela crioulinha que sua me criou e voc... Lembrou-se, ento, Cassi, de quem se tratava. Era a sua primeira vtima, que sua me, sem nenhuma considerao, tinha expulsado de casa em adiantado estado de gravidez. Reconhecendo-a e se lembrando disso, Cassi quis fugir.
(Ibid. p. 115)

551

PORTUGUS
Questo 8

UFBA
F A S E

Meu nome afinal soou, correu todo este Brasil ingrato e mesquinho; e eu fiquei cada vez mais pobre, a viver de uma aposentadoria miservel, com a cabea cheia de imagens de ouro e a alma iluminada pela luz imaterial dos espaos celestes. O fulgor do meu ideal me cegou; a vida, quando no me fosse traduzida em poesia, aborrecia-me. Pairei sempre no ideal; e se este me rebaixou aos olhos dos homens, por no compreender certos atos desarticulados da minha existncia; entretanto, elevou-me aos meus prprios, perante a minha conscincia, porque cumpri o meu dever, executei a minha misso: fui poeta! Para isto, fiz todo o sacrifcio. A Arte s ama a quem a ama inteiramente, s e unicamente; e eu precisava am-la, porque ela representava, no s a minha Redeno, mas toda a dos meus irmos, na mesma dor. Louco?! Haver cabea cujo maquinismo impunemente possa resistir a to inesperados embates, a to fortes conflitos, a colises com o meio to bruscas e imprevistas? Haver? LIMA BARRETO. Clara dos Anjos. 8 ed. So Paulo: tica, 1995. p. 87.

Essa uma fala do personagem Leonardo Flores. Relacionando-a ao contexto da narrativa, depreende-se: (1) O carter autobiogrfico do romance, no qual o autor, atravs dos personagens, revela dramas pessoais e sua forma de enfrent-los. (2) O compromisso do autor de denunciar preconceitos e injustias sociais de sua poca. (4) O determinismo social como definidor implacvel do destino dos personagens. (8) O patriotismo exacerbado do autor, que o impede de fazer uma anlise realista e precisa da situao do pas. (16) A necessidade de o autor projetar-se nacionalmente como escritor, que foi a motivao de toda a sua produo literria. (32) A dedicao arte como nica forma de sobrevivncia material do autor, dificultada pela sua situao econmica. (64) O sofrimento causado por problemas de viso, que gradualmente levam o autor ao desespero, por no poder escrever.

552

PORTUGUS
QUESTES 09 E 10 TEXTO:
Joo de Ado continou: No dia que tu quiser tu tem um lugar aqui nas docas. A gente tem um lugar guardado pra tu. Por qu? perguntou Boa-Vida, j que Pedro apenas olhava espantado. Porque o pai dele era Raimundo e morreu foi aqui mesmo lutando pela gente, pelo direito da gente. Era um homem e tanto. Valia dez destes que a gente encontra por a. Meu pai? fez Pedro Bala, que daquelas histrias s conhecia vagos rumores. Teu pai, era. A gente chamava ele de Loiro. Quando foi da greve fazia discurso pra gente, nem parecia um estivador. Foi pegado por uma bala. Mas tem um lugar pra tu nas docas. Pedro Bala riscava o asfalto com um graveto. Olhou Joo de Ado: Por que tu nunca me contou isso? Tu era pequeno para entender. Agora tu t ficando um homem e riu com satisfao. Pedro Bala riu tambm. Estava contente de saber a histria de seu pai, porque ele tinha sido um homem valente.

UFBA
F A S E

10

15

20

25

30

35

Pedro sorriu. Era outro que ia. No seriam meninos toda vida... Bem sabia que eles nunca tinham parecido crianas. Desde pequenos, na arriscada vida da rua, os Capites da Areia eram como homens, eram iguais a homens. Toda a diferena estava no tamanho. No mais eram iguais: amavam e derrubavam negras no areal desde cedo, furtavam para viver como os ladres da cidade. Quando eram presos apanhavam surras como os homens. Por vezes assaltavam de armas na mo como os mais temidos bandidos da Bahia. No tinham tambm conversas de meninos, conversavam como homens. (...) S Pedro Bala no sabe o que fazer. Dentro em pouco ser mais que um rapazola, ser um homem e ter que deixar para outro a chefia dos Capites da Areia. Para onde ir? No poder ser um intelectual como Professor, cujas mos s viviam para pintar, no nasceu para malandro, como Boa-Vida, que no sente o espetculo da luta diria dos homens, que s ama andar vagabundando pelas ruas, conversar acocorado nas docas, beber nas festas de morro. Pedro sente o espetculo dos homens, acha que aquela liberdade no suficiente para a sede de liberdade que tem dentro de si. Tampouco sente o chamado de Deus, como Pirulito o sentiu. Para ele as pregaes do padre Jos Pedro nunca disseram nada. Gostava do padre como de um homem bom. S as palavras de Joo de Ado encontravam acolhida no seu corao. Mas Joo de Ado mesmo sabe muito pouco. O que tem msculos potentes e voz autoritria, e no entanto amiga, para chefiar uma greve. Tampouco Pedra Bala quer ir como Gato enganar os coronis de Ilhus, arrancar o dinheiro deles. Quer qualquer coisa que no sabe ainda o que , e por isso se demora entre os Capites da Areia. AMADO, Jorge. Capites da areia. 83 ed. Rio de Janeiro: Record, 1996. p.76-7; 230-2.

553

PORTUGUS

UFBA
F A S E

Questo 9 Da leitura do romance Capites da Areia, especialmente do texto apresentado, depreende-se: (1) A inclinao natural para o delito iguala os Capites da Areia aos demais bandidos que assustam a cidade. (2) A relao conflitiva entre as crianas e o meio social resulta na perda da sensibilidade que as caracteriza, gerando violncia. (4) O ideal de impor sua individualidade leva os meninos a se insurgirem contra as leis e a moral do grupo, marginalizando-se. (8) A trajetria tumultuada dos Capites da Areia tem origem nos dramas pessoais dos meninos e na hostilidade da convivncia social, adversa a seus anseios. (16) A reflexo de Pedro Bala, na iminncia de deixar o grupo, revela uma tendncia pessoal para priorizar as questes coletivas, em detrimento dos seus prprios interesses. (32) A idia que Pedro Bala tem sobre liberdade desvia-o da lio deixada por seu pai, que morreu em defesa desse ideal. (64) A necessidade de lutar pela sobrevivncia explica a precocidade que identifica a maneira de ser, de pensar e de agir dos Capites da Areia. Questo 10 Comparado ao uso formal, o uso coloquial da linguagem, na expresso em destaque, est devidamente comentado em: (1) ... tu tem um lugar aqui nas docas. (li. 2) Flexo verbal de 3 pessoa acompanhando pronome pessoal de 2 pessoa, por influncia do pronome de tratamento voc. (2) A gente tem um lugar guardado... (li. 2-3) Desvio da concordncia formal, que exige o verbo na 1 pessoa do plural, coerente com a idia de pluralidade transmitida pelo sujeito. (4) ... um lugar guardado pra tu. (li. 2-3) Uso do pronome reto em situao de uso formal obrigatrio do pronome oblquo. (8) A gente chamava ele de Loiro. (li. 9) Uso de pronome reto na funo de sujeito posposto. (16) ... fazia discurso pra gente... (li. 9-10) Reduo silbica, normal no registro oral, mesmo em situaes formais. (32) Mas tem um lugar... (li. 10) Uso consagrado na comunicao oral: verbo ter, em lugar do verbo haver, com o sentido de existir. (64) Agora tu t ficando um homem... (li. 14) Forma arcaica do verbo ter, usada na oralidade com efeito onomatopaico.

554

PORTUGUS
QUESTES DE 1 A 15

UFBA
F A S E

INSTRUO: Assinale as proposies verdadeiras, some os nmeros a elas associados e marque o resultado na Folha de Respostas. QUESTES 1 E 2 TEXTO:

2 D

Parece-me gente de tal inocncia que, se ns entendssemos a sua fala e eles a nossa, seriam logo cristos, visto que no tm nem entendem crena alguma, segundo as aparncias. E, portanto, se os degredados que aqui ho de ficar aprenderem bem a sua fala e os entenderem, no duvido que eles, segundo a santa teno 5 de Vossa Alteza, se faro cristos e ho de crer na nossa santa f, qual praza a Nosso Senhor que os traga, porque certamente esta gente boa e de bela simplicidade. E imprimir-se- facilmente neles todo e qualquer cunho que lhes quiserem dar, uma vez que Nosso Senhor lhes deu bons corpos e bons rostos, como a homens bons. E o fato de Ele nos haver at aqui trazido, creio que no o foi sem causa. E 10 portanto Vossa Alteza, que tanto deseja acrescentar santa f catlica, deve cuidar da salvao deles. E aprazer a Deus que com pouco trabalho seja assim! Eles no lavram nem criam. Nem h aqui boi ou vaca, cabra, ovelha ou galinha, ou qualquer outro animal que esteja acostumado ao convvio com o homem. E no comem seno deste inhame, de que aqui h muito, e dessas sementes e frutos que a ter15 ra e as rvores de si deitam. E com isto andam tais e to rijos e to ndios que o no somos ns tanto, com quanto trigo e legumes comemos.

P R O V A

Esta terra, Senhor, parece-me que, da ponta que mais contra o sul vimos, at outra ponta que contra o norte vem, de que ns deste ponto temos vista, ser tamanha que haver nela bem vinte ou vinte e cinco lguas por costa. (...) De ponta a ponta 20 tudo praia redonda, muito ch e muito formosa. Pelo serto nos pareceu, vista do mar, muito grande, porque a estender dolhos no podamos ver seno terra com arvoredos, que nos parecia muito longa. Nela at agora no pudemos saber que haja ouro, nem prata, nem coisa alguma de metal ou ferro; nem o vimos. Porm a terra em si de muito bons ares, assim frios 25 e temperados como os de Entre-Douro e Minho, porque neste tempo de agora os achvamos como os de l.

CASTRO, Silvio. A carta de Pero Vaz de Caminha.

555

PORTUGUS

UFBA
Porto Alegre: L & PM, 1996. p. 94 e 97.

Questo 1 Caracteriza o primeiro olhar do europeu sobre a gente e a terra do Brasil: (1) A admirao extasiada pela terra, pelo clima, apesar de no haver constatao da existncia de ouro ou prata. (2) A conscincia da impossibilidade de compreenso entre povos de lugares e costumes to diferentes. (4) A crena de que a descoberta poderia ser atribuda ao desgnio divino, para possibilitar a expanso da f crist. (8) A convico de que os povos da nova terra no tinham f ou religio alguma, o que facilitaria o trabalho de catequese. (16) A avaliao da dificuldade de explorar a natureza selvagem, tendo em vista as barreiras naturais da geografia da terra descoberta. (32) O deslumbramento frente ao aspecto saudvel dos ndios, apesar de seus hbitos alimentares parecerem estranhos. (64) O prognstico da fertilidade da terra e da sua explorao futura em atividades agrcolas. Questo 2 Nas palavras do escrivo sobre a terra descoberta e sua gente, h assentamentos sobre fatos e sobre hipteses. Constitui expresso de uma hiptese: (1) (2) (4) (8) (16) (32) (64) ...sejam logo cristos...(li. 2) ...aprenderem bem a sua fala e os entenderem... (li. 3-4) ...ho de crer na nossa santa f... (li. 5) E aprazer a Deus... (li. 11) ...andam tais e to rijos e to ndios... (li. 15) ... tudo praia redonda, muito ch e muito formosa. (li. 19-20) ...neste tempo de agora os achvamos como os de l. (li. 25-6)

F A S E

2 D

P R O V A

556

PORTUGUS
QUESTES DE 3 A 7 TEXTO:
Ao longo das praias brasileiras de 1500, se defrontaram, pasmos de se verem uns aos outros tal qual eram, a selvageria e a civilizao. Suas concepes, no s diferentes mas opostas, do mundo, da vida, da morte, do amor, se chocaram cruamente. Os navegantes, barbudos, hirsutos, fedentos de meses de navegao ocenica, escalavrados de feridas do escorbuto, olhavam, em espanto, o que parecia ser a inocncia e a beleza encarnadas. Os ndios, vestidos da nudez emplumada, esplndidos de vigor e de beleza, tapando as ventas contra a pestilncia, viam, ainda mais pasmos, aqueles seres que saam do mar. Para os que chegavam, o mundo em que entravam era a arena dos seus ganhos, em ouros e glrias, ainda que estas fossem principalmente espirituais, ou parecessem ser, como ocorria com os missionrios. Para alcan-las, tudo lhes era concedido, uma vez que sua ao de alm-mar, por mais abjeta e brutal que chegasse a ser, estava previamente sacramentada pelas bulas e falas do papa e do rei. Eles eram, ou se viam, como novos cruzados destinados a assaltar e saquear tmulos e templos de hereges indianos. Mas aqui, o que viam, assombrados, era o que parecia ser uma humanidade ednica, anterior que havia sido expulsa do Paraso. Abre-se com esse encontro um tempo novo, em que nenhuma inocncia abrandaria sequer a sanha com que os invasores se lanavam sobre o gentio, prontos a subjug-los pela honra de Deus e pela prosperidade crist. S hoje, na esfera intelectual, repensando esse desencontro se pode alcanar seu real significado.

UFBA
F A S E

2 D

10

P R O V A

15

20

25

30

35

40

45

50

Aquele desencontro de gente ndia que enchia as praias, encantada de ver as velas enfunadas, e que era vista com fascnio pelos barbudos navegantes recm-chegados, era, tambm, o enfrentamento bitico mortal da higidez e da morbidade. A indiada no conhecia doenas, alm de coceiras e desvanecimentos por perda momentnea da alma. A branquitude trazia da crie dental bexiga, coqueluche, tuberculose e o sarampo. Desencandeia-se, ali, desde a primeira hora, uma guerra biolgica implacvel. De um lado, povos peneirados, nos sculos e milnios, por pestes a que sobreviveram e para as quais desenvolveram resistncia. Do outro lado, povos indenes, indefesos, que comeavam a morrer aos magotes. Assim que a civilizao se impe, primeiro, como uma epidemia de pestes mortais. Depois, pela dizimao atravs de guerras de extermnio e da escravizao. Entretanto, esses eram to-s os passos iniciais de uma escalada do calvrio das dores inenarrveis do extermnio genocida e etnocida. Para os ndios, a vida era uma tranqila fruio da existncia, num mundo dadivoso e numa sociedade solidria. Claro que tinham suas lutas, suas guerras. Mas todas concatenadas, como prlios, em que se exerciam, valentes. Um guerreiro lutava, bravo, para fazer prisioneiros, pela glria de alcanar um novo nome e uma nova marca tatuada cativando inimigos. Tambm servia para ofert-lo numa festana em que centenas de pessoas o comeriam convertido em paoca, num ato solene de comunho, para absorver sua valentia, que nos seus corpos continuaria viva. Uma mulher tecia uma rede ou tranava um cesto com a perfeio de que era capaz, pelo gosto de expressar-se em sua obra, como um fruto maduro de sua ingente vontade de beleza. Jovens, adornados de plumas sobre seus corpos escarlates de urucu, ou verde-azulados de jenipapo, engalfinhavam-se em lutas desportivas de corpo a corpo, em que punham a energia de batalhas na guerra para viver seu vigor e sua alegria.

557

PORTUGUS
Para os recm-chegados, muito ao contrrio, a vida era uma tarefa, uma sofrida obrigao, que a todos condenava ao trabalho e tudo subordinava ao lucro. Envoltos em panos, calados de botas e enchapelados, punham nessas peas seu luxo e vaidade, apesar de mais vezes as exibirem sujas e molambentas, do que pulcras e belas. Armados de chuos de ferro e de arcabuzes tonitroantes, eles se sabiam e se sentiam a flor da criao. Seu desejo, obsessivo, era multiplicar-se nos ventres das ndias e pr suas pernas e braos a seu servio, para plantar e colher suas roas, para caar e pescar o que comiam. Os homens serviam principalmente para tombar e juntar paus-de-tinta ou para produzir outra mercadoria para seu lucro e bem-estar. Esses ndios cativos, condenados tristeza mais vil, eram tambm os provedores de suas alegrias, sobretudo as mulheres, de sexo bom de fornicar, de brao bom de trabalhar, de ventre fecundo para prenhar. A vontade mais veemente daqueles heris dalm-mar era exercer-se sobre aquela gente vivente como seus duros senhores. Sua vocao era a de autoridades de mando e cutelo sobre bichos e matos e gentes, nas imensidades de terras de que iam se apropriando em nome de Deus e da Lei. O contraste no podia ser maior, nem mais infranquevel, em incompreenso recproca. Nada que os ndios tinham ou faziam foi visto com qualquer apreo, seno eles prprios, como objeto diverso de gozo e como fazedores do que no entendiam, produtores do que no consumiam. O invasor, ao contrrio, vinha com as mos cheias e as naus abarrotadas de machados, facas, faces, canivetes, tesouras, espelhos e, tambm, miangas cristalizadas em cores opalinas. Quanto ndio se desembestou, enlouquecido, contra outros ndios e at contra seu prprio povo, por amor dessas preciosidades! No podendo produzi-las, tiveram de encontrar e sofrer todos os modos de pagar seus preos, na medida em que elas se tornaram indispensveis. Elas eram, em essncia, a mercadoria que integrava o mundo ndio com o mercado, com a potncia prodigiosa de tudo subverter. Assim se desfez, uniformizado, o recmdescoberto Paraso Perdido.

UFBA
F A S E

55

2 D

60

65

P R O V A

RIBEIRO, Darcy. O povo brasileiro: a formao e o sentido do Brasil. 2 ed. So Paulo: Companhia das Letras, 1995. p. 44; 46-8.

Questo 3 Constitui evidncia do desencontro (li. 23): (1) O confronto entre dois grupos humanos belicosos, guerreiros implacveis na sua nsia de vitrias. (2) A perplexidade recproca frente a diferenas fsicas, de costumes e de atitudes perante a vida. (4) A dificuldade de compreenso entre ndio e colonizador, portadores de culturas construdas atravs de processos histricos distintos. (8) O difcil processo de assimilao gradual das diferenas, tendo em vista as inmeras reaes de resistncia de ambos os lados. (16) O contraste entre os interesses econmicos dos colonizadores e a forma e o objetivo dos processos de produo de bens materiais presentes na cultura indgena. (32) A inexplicvel assimilao passiva, por parte dos ndios, da cultura mais elaborada do colonizador, que no esperava essa reao. (64) A falta de condies fsicas e de defesas orgnicas apresentada pelo colonizador e pelo colonizado, impedindo um contato salutar entre eles.

558

PORTUGUS
Questo 4 O olhar do autor sobre o processo de colonizao destaca:

UFBA
F A S E

(1) o interesse econmico do colonizador, encoberto e justificado como necessidade de difuso da f e de salvao dos indgenas. (2) a inferioridade dos colonizadores, do ponto de vista da resistncia biolgica, por fora das sucessivas epidemias que assolaram a Europa. (4) a solidez dos princpios da f crist dos colonizadores, que, por ela, se submetiam a experincias desgastantes de catequese em mundos desconhecidos. (8) o sentido de pureza, sade e beleza do modo de vida do indgena, maculado pela introduo de valores alheios sua cultura. (16) o modo de vida dos colonizadores, que assentavam sua existncia no desejo do poder e na obteno do lucro nos seus empreendimentos de alm-mar. (32) o massacre cultural e fsico dos indgenas, em decorrncia do contato compulsrio com valores, costumes e interesses diversos dos seus. (64) os sacrifcios a que se impunham os portugueses na luta por uma ascenso econmica que os reabilitasse frente a outras naes da Europa. Questo 5 A expresso transcrita est devidamente explicada em: (1) ...vestidos da nudez emplumada... (li. 7) Expresso construda pela juno de idias que se opem. (2) ...a arena dos seus ganhos, em ouros e glrias... (li. 10-1) Imagem referente luta do povo da terra recm-descoberta, frente ambio do colonizador. (4) ...povos peneirados, nos sculos e milnios... (li. 29-30) Referncia ao processo de seleo e imunizao sofrido pelos povos da Europa. (8) uma escalada do calvrio das dores inenarrveis... (li. 35) Expresso que sugere o sofrimento dos ndios durante o processo de colonizao. (16) ...ato solene de comunho... (li. 43) Expresso referente possibilidade de os ndios virem a se converter f crist. (32) ...fruto maduro... (li. 46-7) Referncia plenitude da obra produzida a partir de um impulso criador e do prazer esttico. (64) ... Paraso Perdido. (li. 80) Aluso a outras terras colonizadas pelos portugueses e perdidas aps lutas de independncia.

2 D

P R O V A

559

PORTUGUS
Questo 6

UFBA
F A S E

Aquele desencontro de gente ndia que enchia as praias, encatada de ver as velas enfunadas, e que era vista com fascnio pelos barbudos navegantes recm-chegados, era, tambm, o enfrentamento bitico mortal da higidez e da morbidade. (li. 23-6) Com base no trecho acima transcrito, correto afirmar: (1) (2) (4) (8) (16) (32) (64) que (1 linha) tem como antecedente desencontro (1 linha). que (1 linha) tem funo idntica a que (2 linha). encantada (1 linha) e com fascnio (2 linha) expressam a mesma circunstncia. de (2 linha) introduz termo que complementa um nome antecendente. ver (2 linha) e era vista (2 linha) diferem quanto voz verbal. as velas enfunadas (2 linha) agente da ao verbal. era (3 linha) concorda com Aquele desencontro (1 linha).

2 D

P R O V A

Questo 7 Uma mulher tecia uma rede ou tranava um cesto com a perfeio de que era capaz, pelo gosto de expressar-se em sua obra, como um fruto maduro de sua ingente vontade de beleza. (li. 45-7) H equivalncia, na estruturao e no sentido, entre o perodo acima e: (1) Pelo gosto de expressar-se em sua obra, uma mulher, com a perfeio de que era capaz, tecia uma rede ou tranava um cesto, como fruto maduro de sua ingente vontade de beleza. (2) Como fruto maduro de sua ingente vontade de beleza, com a perfeio de que era capaz e pelo gosto de expressar-se em sua obra, uma mulher tecia uma rede ou tranava um cesto. (4) Pelo gosto de que era capaz e com a perfeio expressando-se em sua obra, uma mulher tecia uma rede ou tranava um cesto, para ser fruto maduro de sua ingente vontade de beleza. (8) Para expressar-se em sua obra com a perfeio de que era capaz, uma mulher, com o fruto maduro de sua ingente vontade de beleza, tecia uma rede ou tranava um cesto. (16) Tecendo uma rede ou tranando um cesto com o fruto maduro de sua ingente vontade de beleza, uma mulher expressava-se em sua obra com o gosto de que era capaz. (32) Com a perfeio de que era capaz e pelo gosto de expressar-se em sua obra, uma mulher tecia uma rede ou tranava um cesto, como fruto maduro de sua ingente vontade de beleza. (64) O gosto de expressar-se em sua obra com a perfeio de que era capaz era o fruto maduro da ingente vontade de beleza da mulher que tecia uma rede ou tranava um cesto.

560

PORTUGUS
QUESTES DE 8 A 10 TEXTO:
Quando o segundo pio da inhuma ressoou, Iracema corria na mata como a cora perseguida pelo caador. S respirou chegando campina, que recortava o bosque como um grande lago. 5 Quem seus olhos primeiro viram, Martim, estava traqilamente sentado em uma sapopema, olhando o que passava ali. Contra, cem guerreiros tabajaras, com Irapu frente, formavam arco. O bravo Caubi os afrontava a todos, com o olhar cheio de ira e as armas valentes empunhadas na mo robusta. O chefe exigira a entrega do estrangeiro, e o guia respondera simplesmente: Matai Caubi antes. 10 A filha do Paj passara como uma flecha; ei-la diante de Martim, opondo tambm seu corpo gentil aos golpes dos guerreiros. Irapu soltou o bramido da ona atacada na furna. Filha do Paj, disse Caubi em voz baixa, conduz o estrangeiro cabana; s Araqum pode salv-lo. 15 Iracema voltou-se para o guerreiro branco: Vem! Ele ficou imvel. Se tu no vens, disse a virgem, Iracema morrer contigo. 20 Martim ergueu-se; mas longe de seguir a virgem, caminhou direto a Irapu. A sua espada flamejou no ar. Os guerreiros de meu sangue, chefe, jamais recusaram combate. Se aquele que tu vs no foi o primeiro a provoc-lo, porque seus pais lhe ensinaram a no derramar sangue na terra hospedeira. 25 O chefe tabajara rugiu de alegria; sua mo possante brandiu o tacape. Mas os dois campees mal tiveram tempo de medir-se com os olhos; quando fendiam o primeiro golpe, j Caubi e Iracema estavam entre eles. A filha de Araqum debalde rogava ao cristo, debalde o cingia nos braos, buscando arranc-lo ao combate. De seu lado Caubi em vo provocava Irapu para atrair a si a raiva do chefe. 30 A um gesto de Irapu, os guerreiros afastaram os dois irmos; o combate prosseguiu. De repente o rouco som da inbia reboou pela mata; os filhos da serra estremeceram reconhecendo o estrdulo do bzio guerreiro dos pitiguaras, senhores das praias, ensombradas de coqueiros. O eco vinha da grande taba, que o inimigo talvez assaltava j. Os guerreiros precipitaram levando por diante o chefe. Com o estrangeiro s ficou a filha de Araqum.

UFBA
F A S E

2 D

P R O V A

35

ALENCAR, Jos de. Iracema. In: . Fico completa e outros escritos. 3 ed. Rio de Janeiro: Aguilar, 1964, v. 2. p. 1078-9 (ortografia atualizada).

561

PORTUGUS
Questo 8 A leitura da cena apresentada, associada trama do romance, permite inferir:

UFBA
F A S E

(1) Um elevado sentimento materno impulsiona Iracema na defesa do filho, cuja vida est em perigo. (2) Um dio insano o mvel da ao de Irapu contra Caubi, na inteno de faz-lo prisioneiro. (4) O chefe tabajara, movido pelo cime, investe contra o estrangeiro, embora este se encontre sob a proteo do paj. (8) A fidelidade aos princpios da hospitalidade impede que o guerreiro branco inicie o combate contra o guerreiro tabajara. (16) A atitude protetora assumida por Caubi revela a firmeza do personagem no cumprimento da misso de que estava encarregado. (32) A fria dos tabajaras encontra paralelo na fria dos guerreiros brancos, igualmente empenhados na defesa de sua gente. (64) A presteza da filha de Araqum em atender ao chamado de Martim leva-o a submeter-se aos caprichos amorosos da ndia tabajara. Questo 9 O enfrentamento entre o guerreiro branco e o chefe tabajara revela, respectivamente: (1) (2) (4) (8) (16) (32) (64) destemor e fria. arrogncia e insensibilidade. passividade e autoritarismo. submisso e revolta. patriotismo e lusofobia. altivez e indignao. nobreza e valentia.

2 D

P R O V A

Questo 10 No texto, a idia sugerida pela expresso est devidamente indicada em: (1) (2) (4) (8) Quando o segundo pio da inhuma ressoou... (li. 1) ...como a cora perseguida pelo caador. (li. 1-2) ...como um grande lago... (li. 3) ...como uma flecha... (li. 10) passagem de tempo. agilidade e rapidez. placidez, tranqilidade. determinao e sagacidade. extravasamento de dio contido. distanciamento emocional. desafio recproco.

(16) ...soltou o bramido da ona atacada na furna. (li. 11-2) (32) ...longe de seguir a virgem... (li. 19) (64) ...medir-se com os olhos... (li. 25)

562

PORTUGUS
QUESTO DE 11 A 13 TEXTO:

UFBA
F A S E

O ALIENISTA
Era assim que ele ia, o grande alienista, de um cabo a outra da vasta biblioteca, metido em si mesmo, estranho a todas as cousas que no fosse o tenebroso problema da patologia cerebral. Sbito, parou. Em p, diante de uma janela, com o cotovelo esquerdo apoiado na mo direita, aberta, e o queixo na mo esquerda, fechada,

2 D

perguntou ele a si: Mas deveras estariam eles doudos, e foram curados por mim, ou o que pareceu cura no foi mais do que a descoberta do perfeito desequilbrio do crebro? E cavando por a abaixo, eis o resultado a que chegou: os crebros bem organizados que ele acabava de curar eram to desequilibrados como os outros. Sim,

P R O V A

10

dizia ele consigo, eu no posso ter a pretenso de haver-lhes incutido um sentimento ou uma faculdade nova; uma e outra cousa existiam no estado latente, mas existiam. Chegado a esta concluso, o ilustre alienista teve duas sensaes contrrias, uma de gozo, outra de abatimento. A de gozo foi por ver que, ao cabo de longas e pacientes investigaes, constantes trabalhos, luta ingente com o povo, podia afirmar

15

esta verdade : no havia loucos em Itagua; Itagua no possua um s mentecapto. Mas to depressa esta idia lhe refrescara a alma, outra apareceu que neutralizou o primeiro efeito; foi a idia da dvida. Pois qu! Itagua no possuiria um nico crebro concertado? Esta concluso to absurda no seria por isso mesmo errnea, e no vinha, portanto, destruir o largo e majestoso edifcio da nova doutrina psicolgica?

20

A aflio do egrgio Simo Bacamarte definida pelos cronistas itaguaienses como uma das mais medonhas tempestades morais que tm desabado sobre o homem. Mas as tempestades s aterram os fracos; os fortes enrijam-se contra elas e fitam o trovo. Vinte minutos depois alumiou-se a fisionomia do alienista de uma suave claridade.

25

Sim, h de ser isso, pensou ele. Isso isto. Simo Bacamarte achou em si os caractersticos do perfeito equilbrio mental e moral; pareceu-lhe que possua a sagacidade, a pacincia, a perseverana, a tolerncia, a veracidade, o vigor moral, a lealdade, todas as qualidades enfim que podem formar um acabado mentecapto. Duvidou logo, certo, e chegou mesmo a

30

concluir que era iluso; mas sendo homem prudente, resolveu convocar um conselho de amigos, a quem interrogou com franqueza. A opinio foi afirmativa. Nenhum defeito? Nenhum, disse em coro a assemblia. Nenhum vcio?

35

Nada. Tudo perfeito? Tudo. No, impossvel, bradou o alienista. Digo que no sinto em mim essa superioridade que acabo de ver definir com tanta magnificncia. A simpatia que vos faz

40

falar. Estudo-me e nada acho que justifique os excessos da vossa bondade.

563

PORTUGUS
A assemblia insistiu; o alienista resistiu; finalmente o Padre Lopes explicou tudo com este conceito digno de um observador: Sabe a razo por que no v as suas elevadas qualidades, que alis todos ns admiramos? porque tem ainda uma qualidade que reala as outras: a modstia. 45 Era decisivo. Simo Bacamarte curvou a cabea, juntamente alegre e triste, e ainda mais alegre do que triste. Ato contnuo, recolheu-se Casa Verde. Em vo a mulher e os amigos lhe disseram que ficasse, que estava perfeitamente so e equilibrado: nem rogos nem sugestes nem lgrimas o detiveram um s instante. A questo cientfica, dizia ele; trata-se de uma doutrina nova, cujo primeiro 50 exemplo sou eu. Reno em mim mesmo a teoria e a prtica.

UFBA
F A S E

2 D

P R O V A

MACHADO DE ASSIS. Papis avulsos. In: . Obra completa. Rio de Janeiro: Aguilar, 1962. v. 2. p. 286-8. (ortografia atualizada)

Questo 11 Itagua no possua um s mentecapto. (li. 15) Ao longo do conto, o Dr. Bacamarte faz diversas constataes cientficas, como a citada acima. Outra dessas constataes est expressa em: (1) Loucos so todos os que apresentam qualquer desvio de conduta ou personalidade, por menor que seja, quando considerado determinado padro de comportamento. (2) Todas as pessoas que mantm em equilbrio as suas faculdades e so isentas de comportamento socialmente reprovveis devem ser consideradas loucas. (4) A loucura acomete freqentemente as mulheres, por serem elas passveis de distrbios emocionais e de comportamento, alm de cederem mais facilmente a ambies e vaidades. (8) H uma categoria de loucura que vitima os que, abondonando o mundo, dedicam-se a atividades religiosas com fanatismo. (16) No h ser humano, por mais perfeito moralmente, que possa resistir inoculao de um vcio ou pecado. (32) Todos os seres humanos so loucos, expressando ou no essa loucura, sendo impossvel estabelecerem-se tipologias ou categorizaes de nveis de insanidade. (64) A loucura no existe: essa apenas uma forma de designar aqueles cuja originalidade ou exotismo os afastam dos padres comuns.

Questo 12 H coerncia entre o trecho transcrito e a assero feita sobre o conto, em: (1) ...o tenebroso problema da patologia cerebral. (li. 2-3) O conto procura demonstrar a possibilidade de se assentarem as bases de uma cincia sem procedimentos cientficos rigorosos. (2) ...ou o que pareceu cura no foi mais do que a descoberta do perfeito desequilbrio do crebro? (li. 6-7) O personagem central do conto move suas aes no sentido de desmascarar os equvocos do conhecimento cientfico de sua poca.

564

PORTUGUS

UFBA
F A S E

(4) E cavando por a abaixo, eis o resultado a que chegou... (li. 8) H, no conto, uma crtica ao uso da razo como forma cientfica de deslindar os mistrios da mente humana. (8) ...o largo e majestoso edifcio da nova doutrina psicolgica? (li. 19) O autor do conto desenvolve a narrativa vertendo observaes irnicas para criticar os exageros do cientificismo de sua poca. (16) ...achou em si os caractersticos do perfeito equilbrio mental e moral... (li. 26-7) O desenho do personagem central do conto carregado de observaes crticas sobre a sua vaidade e irresponsabilidade profissional. (32) ...possua (...) todas as qualidades enfim que podem formar um acabado mentecapto. (li. 27-9) H, no conto, a indicao de que o mergulho em si mesmo, atravs da razo, pode gerar absurdos e contradies. (64) Reno em mim mesmo a teoria e a prtica. (li. 50) O conto leva o leitor a concluir que impossvel o avano de uma cincia que assente suas bases apenas em postulados tericos. Questo 13 H uso idntico de sinal de pontuao, em: (1) Era assim que ele ia, o grande alienista, de um cabo a outro (li. 1) No, impossvel, bradou o alienista (li. 38) (2) Sbito, parou (li. 3) Ato contnuo, recolheu-se Casa Verde (li. 46) (4) e o queixo na mo esquerda, fechada, perguntou ele a si (li. 4-5) possua a sagacidade, a pacincia, a perseverana (li. 27) (8) E cavando por a abaixo, eis o resultado (li. 8) Chegado a esta concluso, o ilustre alienista teve (li. 12) (16) eis o resultado a que chegou: os crebros bem organizados (li. l8-9) tem ainda uma qualidade que reala as outras: a modstia (li. 44) (32) Mas as tempestades s aterram os fracos; os fortes enrijam-se. (li. 22-3) A assemblia insistiu; o alienista resistiu (li. 41) (64) mas sendo homem prudente, resolveu convocar (li. 30) lhe disseram que ficasse, que estava perfeitamente so (li. 47)

2 D

P R O V A

565

PORTUGUS
QUESTES 14 e 15 TEXTO:
Z Padre, eu no tenho parte com o Diabo, tenho com Santa Brbara. PADRE (Agora para toda a praa) Estive o dia todo estudando este caso. Consultei livros, textos sagrados. Naquele burro est a explicao de tudo. Satans! S mesmo Satans podia levar algum a ridicularizar o sacrifcio de Jesus. ROSA No, Padre, no! PADRE Por que no? ROSA Porque eu conheo ele. um bom homem. At hoje s fez o bem.
Neste momento, entra Monsenhor. O Padre est no auge de sua clera. Ao ver Monsenhor, seu brao se imobiliza no ar, como ante uma apario sobrenatural.

UFBA
F A S E

2 D

10

P R O V A

15

PADRE Monsenhor! MONSENHOR Venho aqui a pedido de Monsenhor Arcebispo. S. Excia. est muito preocupado com o vulto que est tomando este incidente e incumbiu-me, pessoalmente, de resolver a questo. A fim de dar uma prova de tolerncia da igreja para com aqueles que se desviam dos cnones sagrados... Z (Interrompe) Padre, eu sou catlico. No entendo muita coisa do que dizem, mas queria que o senhor entendesse que eu sou catlico. Pode ser que eu tenha errado, mas sou catlico. MONSENHOR Pois bem. Vamos lhe dar uma oportunidade. Se catlico, renegue todos os atos que praticou por inspirao do Diabo e volte ao seio da Santa Madre Igreja. Z (Sem entender) Como, Padre? MONSENHOR Abjure a promessa que fez, reconhea que foi feita ao Demnio, atire fora essa cruz e venha, sozinho, pedir perdo a Deus. Z (Cai num terrvel conflito de conscincia) O senhor acha mesmo que eu devia fazer isso?... MONSENHOR sua nica maneira de salvar-se. A igreja catlica concede a ns, sacerdotes, o direito de trocar uma promessa por outra. Z

20

25

30

35

40

(Pausa) O senhor me liberta... mas no foi ao senhor que eu fiz a promessa, foi a Santa

Brbara. E quem me garante que como castigo, quando eu voltar pra minha roa no vou encontrar meu burro morto. MONSENHOR Decida! Renega ou no renega?

GOMES, Alfredo Dias. O pagador de promessas. 27 ed. Rio de Janeiro: Ediouro, 1996. p. 71-4.

566

PORTUGUS
Questo 14 Com base no texto e levando-se em conta o contexto geral da pea, pode-se afirmar:

UFBA
F A S E

(1) Z-do-Burro, em vista de um grave delito cometido contra sua santa protetora, empenhase obstinadamente em expiar sua culpa. (2) O Padre Olavo, imbudo de idias arraigadas sobre questes religiosas, assume uma atitude dogmtica, contrria ao livre arbtrio inerente pessoa de Z-do-Burro. (4) O Padre, diante da rejeio de Z-do-Burro aos princpios da igreja catlica, esfora-se pela converso do sertanejo, temporariamente revoltado com a religio. (8) A atitude intransigente demonstrada pelo pagador de promessas revela firmeza na inteno de preservar a honra de um compromisso assumido com a santa. (16) O Monsenhor e o Padre, apoiados em interpretao radical dos dogmas religiosos, contrapem-se com firmeza aos interesses de Z-do-Burro, que ignora inteiramente esses dogmas. (32) Com a interferncia do Monsenhor, a histria de Z-do-Burro tem o desfecho pretendido pela autoridade eclesistica em luta contra o sincretismo religioso. (64) A atuao da igreja sobre Z-do-Burro provoca-lhe um desgaste progressivo, que culmina com o seu desequilbrio emocional e com a sua morte. Questo 15 Constitui elemento da estrutura dramtica da pea: (1) Tempo com determinao imprecisa, pela ausncia de marcas referentes durao das cenas. (2) Ambiente claramente delimitado, configurado, simultaneamente, como espao de festa e espao de tragdia. (4) Desfecho tragicmico, resultante da ao de personagens secundrios na hierarquia dramtica. (8) Conflito gerado pela posio radical assumida pelo protagonista e pelo antagonista. (16) A existncia de personagens centrais simblicos, cujo conflito representa a coliso entre valores fundamentais de dois grupos social e hierarquicamente diferenciados. (32) Desfecho que representa a realizao do propsito do personagem central, em circunstncias trgicas. (64) Ao que se desenvolve a partir de sucessivas mudanas de atitude do personagem central, frente influncia de personagens secundrios.

2 D

P R O V A

567

PORTUGUS
QUESTO DISCURSIVA

UFBA
F A S E

CARACTERIZE A PRODUO SATRICA DE GREGRIO DE MATOS, CONSIDERANDO O CONTEDO, A FORMA E O CONTEXTO DESSA PRODUO. COMPROVE SUAS AFIRMAES CITANDO PASSAGENS DO TEXTO.

2 D

TEXTO:
Senhora Dona Bahia, nobre, e opulenta cidade, madrasta dos Naturais, e dos Estrangeiros madre. Dizei-me por vida vossa, em que fundais o ditame de exaltar, os que a vm, e abater, os que ali nascem?

ROMANCE
j tem tantos mil cruzados, segundo afirmam Pasguates. 35 Casa-se o meu matachim, pe duas Negras, e um Pajem, uma rede com dous Minas, chapu-de-sol, casas-grandes. Entra logo nos pilouros, e sai no primeiro lance Vereador da Bahia, que notvel dignidade. J temos o Canastreiro, que inda fede a seus beirames, metamorfosis da terra transformado em homem grande: e eis aqui a personagem.

P R O V A

10

15

Haver duzentos anos, (nem tantos podem contar-se) que reis uma aldeia pobre, e hoje sois rica cidade. Ento vos pisavam ndios, e vos habitavam cafres, hoje chispais fidalguias, arrojando personagens. A essas personagens vamos, sobre elas ser o debate.

20

Sai um pobrete de Cristo de Portugal, ou do Algarve cheio de drogas alheias para da tirar gages:

25

30

Entra pela barra dentro, d fundo, e logo a entonar-se comea a bordo da Nau cum vestidinho flamante. Salta em terra, toma casas, arma a botica dos trastes, em casa come Baleia, na rua entoja manjares. Vendendo gato por lebre, antes que quatro anos passem,

40

45

50

55

60

65

Vem um Clrigo idiota, desmaiado como um jalde, os vcios com seu bioco, com seu rebuo as maldades: Mais santo do que Mafoma na crena dos seus Arabes, Letrado como um Matulo, e velhaco como um Frade: Ontem simples Sacerdote, hoje uma gr dignidade, ontem salvage notrio, hoje encoberto ignorante. Ao tal Beato fingido fora, que o povo aclame, e os do governo se obriguem, pois edifica a cidade. Chovem uns, e chovem outros com ofcios e lugares, e o Beato tudo apanha por sua muita humildade. Cresce em dinheiro e respeito,

568

PORTUGUS
vai remetendo as fundagens, compra toda a sua terra, com que fica homem grande, e eis aqui a personagem. 80

UFBA
licena para queixar-se. Sempre vem, e sempre falam, at que Deus lhes depare, quem lhes faa de justia esta stira cidade.

70

F A S E

2 D

75

Vem isto os Filhos da terra, e entre tanta iniqidade so tais, que nem inda tomam

MATOS, Gregrio de. Obras completas. Salvador Bahia: Janana, 1968. v.2. p. 429-34.

P R O V A

GLOSSRIO: arrojar - produzir, projetar. barra - foz, entrada de porto. beirames - tecido de algodo fino, oriundo da ndia. bioco - capuz. botica - loja de venda a retalho. cafre - homem ignorante, rude. Canastreiro - mascate, vendedor ambulante. chispar - lanar de si. Clrigo - indivduo que pertence a ordem eclesistica. dar fundo - ancorar. ditame - preceito, sentena. entojar - sentir nojo, enjoar. entonar-se- ostentar luxo. entrar nos pilouros - concorrer eleio, candidatar-se. flamante - brilhante, vistoso. fundagem - o que sobra no fundo, borra (p. ext., lucro, economias). fundar - apoiar, fundamentar. gages - (fr. gage) - ganho, lucro. jalde - (jalne) - amarelo, cor de ouro. madre - me. Mafoma - Maom. matachim (machatim) - farsante. Matulo - vadio. Minas - escravo oriundo da Costa da Mina. ofcio - trabalho, cerimnia religiosa. Pasguates - idiota, palerma. rebuo - parte do vesturio destinada a cobrir o rosto. salvage - selvagem. trastes - mveis velhos, objetos diversos. velhaco - indivduo enganador, fraudulento, patife.

569

PORTUGUS
QUESTO DE REDAO

UFBA
F A S E

ESCOLHA UM DOS TEMAS (I OU II) E INDIQUE-O NA FOLHA DE REDAO, ANTES DE DESENVOLV-LO. TEMA I
Chora a nossa Ptria me gentil, choram marias e clarices, no solo do Brasil. Mas sei que uma dor assim pungente no h de ser inutilmente. A esperana dana na corda bamba de sombrinha e em cada passo dessa linha pode se machucar... (Joo Bosco) Gigante pela prpria natureza s belo, s forte, impvido colosso, e o teu futuro espelha essa grandeza. Terra adorada, Entre outras mil, s tu, Brasil, Ptria amada! (Joaquim Osrio Duque Estrada) Isso aqui, , , um pouquinho de Brasil Y Y desse Brasil que canta e feliz, feliz. tambm um pouco de uma raa que no tem medo de fumaa ai, ai, e no se entrega no... (Ary Barroso)

2 D

P R O V A

Eu, brasileiro, confesso minha culpa, meu pecado, Meu sonho desesperado, meu bem guardado segredo, Minha aflio.

Aqui o fim do mundo... Minha terra tem palmeiras onde sopra o vento forte Da fome, do medo e, muito principalmente, da morte. (Gilberto Gil)

Inspirando-se nos trechos acima, escreva um texto com o ttulo: POR QUE AMO (OU NO AMO) MEU PAS

570

PORTUGUS
TEMA II

UFBA
F A S E

2 D

P R O V A

Sem f no temos esperana, e sem esperana no temos propriamente vida. No tendo uma idia do futuro, tambm no temos uma idia de hoje, porque o hoje, para o homem de ao, no seno o prlogo do futuro. (Fernando Pessoa)

Com base na ilustrao e na reflexo de Fernando Pessoa, escreva um texto sobre o tema sugerido pela expresso: BRASILEIRO PROFISSO ESPERANA

571

PORTUGUS

UFMG
F A S E

INSTRUO: As questes de 1 a 10 baseiam-se no texto abaixo. Leia atentamente todo o texto antes de resolv-las.
No foi h tanto tempo assim. Cheguei praia com minhas filhas e encontrei um aglomerado de cidados. Eles montavam guarda num pequeno trecho da areia, caras alarmadas, pior: pungidas. No fui eu quem viu o grupo: foi o grupo que me viu e dois de seus membros vieram em minha direo, delicadamente me afastaram das meninas e comunicaram: Tire depressa suas filhas daqui! As palavras foram duras mas o tom era ameno, cmplice. Quis saber por qu. Em voz baixa, conspiratria, um dos cidados me comunicou que ali na arrebentao, boiando como uma anmona, alga desprendida das profundezas ocenicas, havia uma camisinha que na poca atendia pelo potico nome de camisa de Vnus. O grupo de cidados num tempo em que direitos e deveres da cidadania ainda esperavam pela epifania de Betinho ali estava desde cedo, alertando pais incautos, como se a camisinha fosse uma pastilha de material nuclear, uma cpsula de csio com prfidas e letais emanaes. No me lembro da reao que tive, possvel que tenha levado as meninas para outro canto, mas tenho certeza de que nem alarmado fiquei. Hoje, a camisinha aparece na televiso, banal e inocente como um par de patins, um aparelho de barba. Domingo ltimo, levando minhas setters unica praia em que so permitidos animais domsticos, encontrei um grupo de cidados em volta de uma coisa. No, no era aquele monstro marinho que Fellini colocou no final de um de seus filmes. Tampouco era uma camisinha - que as praias esto cheias delas, mais numerosas que as conchas e os tatus de antigamente. O motivo daquela expresso de cidadania era uma seringa que as guas despejaram na areia. Objeto na certa infectado, trazendo na ponta de sua agulha o vrus da Aids que algum viciado ali deixara, para contaminar inocentes e culpados. Daqui a dois, cinco anos, espero que a Aids no mais preocupe a humanidade. Mas os cidados continuaro alarmados, descobrindo novas misrias na efmera eternidade das espumas. Carlos Heitor Cony. Folha de So Paulo, p. 1-2, 09.01.1994.

1 A

P R O V A

1. Assinale a alternativa que apresenta o ttulo mais apropriado para o texto. a) b) c) d) A cam.ter efmero das preocupaes humanas. A importncia das descobertas da humanidade. O carter efmero das preocupaes humanas. O vrus da Aids: um perigo para a humanidade.

2. Todas as alternativas apresentam afirmaes que podem ser confirmadas pelo texto, EXCETO a) b) c) d) A humanidade encontra razo para alarme em todas as pocas. As preocupaes da humanidade mudam, mas existem sempre. As preocupaes do presente so mais srias do que as do passado. Os cidados de hoje esto preocupados com o vrus da Aids e suas conseqncias.

572

PORTUGUS
3. Todas as alternativas contm afirmaes corretas sobre o texto, EXCETO a) b) c) d)

UFMG
F A S E

O autor usa dois fatos, para fazer reflexes sobre o ser humano. O cronista tece consideraes sobre o futuro da humanidade. O narrador sugere que o sexo vai continuar sendo um tabu entre os homens. O texto contrape passado e presente, mostrando, no entanto, a semelhana entre essas pocas.

1 A

4. Todas as alternativas contm afirmaes corretas sobre a construo do texto, EXCETO a) O autor usa a exemplificao, para evidenciar diferentes posturas em relao ao conceito de cidadania. b) O autor usa o dilogo, para dar vivacidade cena narrada. c) O autor utiliza elementos temporais, para caracterizar diferentes momentos da transformao da sociedade. d) O autor utiliza uma linguagem concisa, para garantir objetividade nas suas avaliaes. 5. Em todas as alternativas, o significado das palavras destacadas est corretamente identificado, EXCETO em a) ...como se a camisinha fosse (...) uma cpsula de csio com prfidas e letais emanaes. (emanaes = contaminaes) b) ...como se a camisinha fosse (...) uma cpsula de csio com prfidas e letais emanaes. (letais = mortais) c) Eles montavam guarda num pequeno trecho da areia, caras alarmadas, pior: pungidas. (pungidas = atormentadas) d) O grupo de cidados (...) ali estava desde cedo, alertando pais incautos... (incautos = desavisados)

P R O V A

6. O motivo daquela expresso de cidadania era uma seringa que as guas despejaram na areia. Na passagem acima, o autor se mostra a) alarmado. b) deprimido. c) indignado. d) irnico.

7. Tampouco era uma camisinha que as praias esto cheias delas, mais numerosas que as conchas
e os tatus de antigamente.

A alternativa que apresenta a interpretao correta da palavra destacada a) b) c) d) Tampouco era uma camisinha cujas praias esto cheias delas... Tampouco era uma camisinha logo as praias esto cheias delas... Tampouco era uma camisinha no entanto as praias esto cheias delas... Tampouco era uma camisinha uma vez que as praias esto cheias delas...

573

PORTUGUS

UFMG
F A S E

8. Em todas as alternativas, a mudana de ordem das palavras destacadas no altera o significado bsico das passagens, EXCETO em a) Daqui a dois, cinco anos, espero que a Aids no mais preocupe a humanidade. Daqui a dois, cinco anos, espero que a Aids no preocupe mais a humanidade. b) Em voz baixa, conspiratria, um dos cidados me comunicou que ali (...) havia uma camisinha. Um dos cidados, em voz baixa, conspiratria, me comunicou que ali (...) havia uma camisinha. c) Hoje, a camisinha aparece na televiso, banal e inocente como um par de patins, um aparelho
de barba.

1 A

A camisinha, hoje, aparece na televiso, banal e inocente como um par de patins, um aparelho de barba. d) Objeto na certa infectado, trazendo na ponta de sua agulha o vrus da Aids, que algum viciado ali
deixara...

P R O V A

Objeto na certa infectado, trazendo na ponta de sua agulha o vrus da Aids, que viciado algum ali deixara...

9. Em todas as alternativas, est presente a relao de idias identificada entre parnteses, EXCETO em a) As palavras foram duras mas o tom era ameno, cmplice. (oposio) b) Levando minhas setters (...) praia (...), encontrei um grupo de cidados em volta de uma coisa. (temporalidade) c) No fui eu quem viu o grupo: foi o grupo que me viu. (comparao) d) Objeto na certa infectado, trazendo na ponta de sua agulha o vrus da Aids que algum viciado ali
deixara para contaminar inocentes e culpados.

(finalidade)

10. Em todas as alternativas, a palavra destacada est corretamente interpretada, EXCETO em a) Em voz baixa, conspiratria, um dos cidados me comunicou que ali na arrebentao (...) havia
uma camisinha.

(que = um dos cidados)

b) No me lembro da reao que tive, possvel que tenha levado as meninas para outro canto... (que = a reao) c) No, no era aquele monstro marinho que Fellini colocou no final de um de seus filmes. (que = aquele monstro marinho) d) O motivo daquela expresso de cidadania era uma seringa que as guas despejaram na areia. (que = uma seringa)

574

PORTUGUS

UFMG
F A S E

11. Todas as alternativas apresentam afirmaes verdadeiras em relao a Sargento Getlio, de Joo Ubaldo Ribeiro, EXCETO a) A obra destaca o herosmo e a resistncia do povo nordestino tirania. b) A obra incorpora elementos do imaginrio nordestino. c) A obra reproduz cenas comuns da poltica brasileira da dcada de 50. d) A obra retrata as relaes de poder e de mando no Nordeste brasileiro.

1 A

12. Leia com ateno o trecho que se segue, da obra Sargento Getlio.
Cheguei, como vai todo mundo, muito boa tarde, j vou indo, licena aqui. Quem se incomoda. Tudo s. Eu mesmo j pensei de outras maneiras. Ela estava de barriga na ocasio. Eu alisava a barriga, quando tinha tempo, quando vinha um sossego, quando quentava, quando deitava, quando estava neblina, quando aquietava. Parecia um cachorro, ficava ali, os olhos gazos midos me assuntando. O barrigo me trazia satisfao, j se adevinhava bem ali e o embigo bem que j saa um pouco para fora e se podia sentir passando a mo. Pois ficava alisando de um lado para o outro, numa banzeira, pensando no bicho l dentro. Quando matei, nem pensei mais em matar. Matei sem raiva. Pensei que no, antes da hora, pensei que ia com muita raiva, mas no fui. Cheguei, olhei, ela deitada assim e ainda perguntou: que que tem? Ela sabia, no sabia s disso, tinha certeza que no adiantava fugir, porque eu ia atrs. A dor de corno, uma dor funda na caixa, uma coisa tirando a fora de dentro. Nem sei. Uma mulher no como um homem.

P R O V A

O episdio narrado no ocorre na seqncia dos acontecimentos que se sucedem durante a jornada do Sargento Getlio com seu prisioneiro. Apesar de no se inserir na seqncia principal dos acontecimentos que compem a narrativa, esse episdio importante para a composio da personagem Sargento Getlio. Assinale a alternativa que apresenta uma situao em que o comportamento do Sargento Getlio pode ser explicado pelo trecho acima. a) A cena em que arranca, sem piedade, os dentes do prisioneiro que est conduzindo. b) A ocasio em que morre Luzinete, vtima das bombas de dinamite. c) A passagem em que se encontra com os emissrios enviados com o objetivo de faz-lo desistir de levar o preso. d) O episdio em que degola um tenente da fora pblica enviada para captur-lo.

13. A pea Vestido de noiva, de Nelson Rodrigues, representou um marco para o teatro brasileiro principalmente porque a) apresenta os processos inconscientes da personagem. b) atribui grande importncia s personagens femininas. c) incorpora um acidente de trnsito ao enredo. d) trata o erotismo de modo explcito.

575

PORTUGUS

UFMG
F A S E

14. A estrutura fragmentada e lacunar da pea Vestido de noiva, com sua desordem de planos, tem por finalidade a) fixar a vida contempornea num flagrante que a sintetiza. b) registrar as tragdias annimas que acontecem cotidianamente. c) relegar os acontecimentos exteriores insignificncia que devem ter. d) representar a desagregao mental de uma personagem que vai morrer.

1 A

15. caracterstica da Lira dos vinte anos, de Alvares de Azevedo, a) a apresentao do ndio como smbolo da nacionalidade brasileira. b) a manifestao do mal-do-sculo, pessimismo e desejo de morrer. c) a opo por uma poesia de denncia social, engajada na luta abolicionista. d) a representao de uma natureza brasileira, tropical e extica.

P R O V A

16. A alternativa que apresenta versos da Lira dos vinte anos em que a natureza se mostra solidria com o poeta a) Acorda! no durmas da cisma no vu!
Amemos, vivamos, que amor sonhar Um beijo, donzela! No ouves? no cu A brisa gemeu As vagas murmuram... As folhas sussurram: Amar!

b) Amo o vento da noite sussurrante


A tremer nos pinheiros E a cantiga do pobre caminhante No rancho dos tropeiros; (...)

c) No sabes o quanto di
Uma lembrana que ri A fibra que adormeceu?... Foi neste vale que amei, Que a primavera sonhei, Aqui minha alma viveu.

d) Que me resta, meu Deus?! aos meus suspiros


Nem geme a virao, E dentro no deserto do meu peito No dorme o corao!

576

PORTUGUS

UFMG
F A S E

17. Todas as alternativas apresentam afirmaes corretas sobre Memrias de um sargento de milcias, de Manuel Antnio de Almeida, EXCETO a) O romance denuncia a influncia do meio sobre o homem. b) O romance descreve tipos caractersticos do sculo XIX. c) O romance refere-se a festas e folguedos da cultura brasileira. d) O romance valoriza as classes populares urbanas brasileiras.

1 A

18. A personagem Leonardo Filho, do romance Memrias de um sargento de milcias, a) apresenta um forte moralismo que o faz traar um quadro crtico dos costumes e das classes sociais do incio do sculo XIX. b) classifica-se como um tpico heri romntico, por seus infortnios amorosos. c) mostra-se como um vadio, que vive ao sabor do acaso, nada aprendendo com a experincia. d) participa da condio servil, apresentando vises variadas da sociedade em que vive, a partir das diversas posies que nela ocupa.

P R O V A

19. As crnicas da obra Ai de ti, Copacabana!, de Rubem Braga, em seu conjunto, a) apresentam uma galeria de tipos populares do Rio de Janeiro. b) informam ao leitor da poca os acontecimentos daquele tempo. c) manifestam a viso subjetiva e lrica que o autor tinha de seu tempo. d) registram cronologicamente a histria brasileira da dcada de 50.

20. Leia com ateno o trecho seguinte, de Antnio Candido.


...vamos pensar um pouco na crnica como gnero. Lembrar, por exemplo, que o fato de ficar to

perto do dia-a-dia age como quebra do monumental e da nfase.

Em todas as alternativas que se seguem, o cronista Rubem Braga encontra-se perto do dia-adia, distante do monumental e da nfase, EXCETO em a) Ai de ti, Copacabana, porque eu j fiz o sinal bem claro de que chegada a vspera de teu dia, e tu no viste; porm minha voz te abalar at as entranhas. b) Chega o velho carteiro e me deixa uma carta. Quando se vai afastando eu o chamo: a carta no para mim. Aqui no mora ningum com este nome, explico-lhe. c) Ir praia cedo, como na infncia. As ilhas no horizonte ainda esto veladas pela nvoa da madrugada. O mar andou bravo esta noite, arrancando algas e mexilhes das pedras. d) Outro dia fui, noite, a Santa Tereza, e ontem, tarde, visitei um amigo na Clnica So Vicente. So raras, porm, minhas excurses pelas montanhas do Rio, por essa outra cidade...

577

PORTUGUS

UFMG
F A S E

INSTRUO: As questes de 01 a 10 baseiam-se no texto abaixo. Leia atentamente todo o texto antes de resolv-las.
A cara do mdico no boa, mas a cara dos mdicos, do outro lado da mesa, sempre enigmtica, faz parte da consulta, da profisso e dos honorrios: o jeito o paciente ficar paciente e aguardar os exames. Mas at os exames h os hierglifos que ele procura decifrar. H nomes com razes gregas e desinncias latinas, ele no entende nada, sabe apenas que um pedao de sua carne ser retirado e ir para os provetas, os reagentes, o diabo. Por falar no diabo, passa pela igreja e tem vontade de entrar, acender velas, pedir qualquer coisa. Mas pedir o qu, exatamente? Mesmo assim entra na igreja. Est escura, vazia, somente uma velha, l na frente, deve estar pedindo tambm alguma coisa. Pelo jeito, ela deve saber o que est pedindo o que no o caso dele. E vem de volta a cara do mdico: Se tudo correr bem, podemos salvar a vista. Sejamos otimistas, o senhor ficar bom! Ali na igreja a frase uma espcie de orao s avessas. O que significa ficar bom? Significa ser como antes, e ele nunca fora bom. Olhar as coisas, o mar, as crianas, a noite, a velha l na frente. Sim, o senhor ficar bom, mas pode haver razes gregas e declinaes latinas e tudo ficar complicado. No importa, agora. Est numa igreja onde se adora um Deus em que ele no acredita. Mas precisa acreditar, ao menos no laboratrio. Novamente na rua, confere o endereo, entra em nmeros errados, toma elevadores equivocados, desce em andares estranhos. At que v a porta de vidro com o nome gravado em azul: anlises clnicas. ali. A enfermeira comea a preparar as pinas, as placas de vidro. Em breve, uma gota de seu sangue ser uma pitanga muito vermelha pousada numa delas. A soluo no a salvao de todos os enigmas. Brevemente, o mundo acabar para seus olhos. E as mulheres, as crianas, o mar, os livros que gostaria de ler tudo ser a mancha to escura e estranha como a velha que rezava na igreja. Pela janela, v o nibus fazendo a curva na praa. Tem um pensamento idiota: ser essa a ltima imagem que ficar em seus olhos? De que adiantou ter visto a fachada de Santa Maria dei Fiori, as mulheres que amou? De que adiantou...? O pagamento adiantado. Seu nome no cheque o surpreende: no mais ele. Carlos Heitor Cony. Folha de So Paulo, p. 1-2, 28.01.1994.

1 B

P R O V A

1. So ttulos adequados para o texto, EXCETO a) Luzes que se apagam b) O despertar da f c) O itinerrio do fim d) Via crucis

2. Todas as alternativas apresentam afirmaes que podem ser confirmadas pelo texto, EXCETO a) b) c) d) A desordem aparente das idias revela o caos do pensamento do protagonista. A narrativa entremeada por lembranas e reflexes do protagonista. O texto, atravs do uso de interrogativas, mistura narrador e protagonista. O texto, sem limites ntidos entre partes, faz um recorte na vida do protagonista.

578

PORTUGUS

UFMG
F A S E

3. Todas as alternativas apresentam sentimentos do protagonista em relao ao seu problema, EXCETO a) Amargura c) Inquietude b) Desorientao d) Rebeldia

1 B

4. Em todas as alternativas, h pistas a respeito do problema de sade do protagonista, EXCETO em a) A cara do mdico no boa, mas a cara dos mdicos, do outro lado da mesa, sempre enigmtica... b) O que significa ficar bom? Significa ser como antes, e ele nunca fora bom. Olhar as coisas, o
mar, as crianas, a noite, a velha l na frente.

c) E as mulheres, as crianas, o mar, os livros que gostaria de ler tudo ser a mancha to escura
e estranha como a velha que rezava na igreja.

P R O V A

d) Pela janela, v o nibus fazendo a curva na praa. Tem um pensamento idiota: ser essa a ltima
imagem que ficar em seus olhos?

5. Todas as alternativas apresentam passagens que mostram a perturbao em que se encontra o protagonista, EXCETO a) b) c) d)
Em breve, uma gota de seu sangue ser uma pitanga muito vermelha pousada numa delas. ...entra em nmeros errados, toma elevadores equivocados, desce em andares estranhos. Pelo jeito, ela deve saber o que est pedindo: o que no o caso dele. ...tem vontade de entrar, acender velas, pedir qualquer coisa. Mas pedir o qu, exatamente?

6. Em todas as passagens, o autor joga com o duplo sentido de palavras, EXCETO em a) A cara do mdico no boa, mas a cara dos mdicos, do outro lado da mesa, sempre enigmtica... b) De que adiantou ter visto a fachada de Santa Maria dei Fiori (...)? De que adiantou...? O pagamento
adiantado.

c) ...o jeito o paciente ficar paciente e aguardar os exames. d) Sejamos otimistas, o senhor ficar bom. (...) O que significa ficar bom? Significa ser como antes
e ele nunca fora bom.

7. Em todas as alternativas, est presente a relao de idias identificadas entre parnteses, EXCETO em a) ... faz parte da consulta, da profisso e dos honorrios: o jeito o paciente ficar paciente... (concluso) b) Pelo jeito, ela deve saber o que est pedindo o que no o caso dele. (oposio) c) Se tudo correr bem, podemos salvar a vista. (condio) d) ...tudo ser a mancha to escura e estranha como a velha que rezava na igreja. (conseqncia)

579

PORTUGUS

UFMG
F A S E

8. Em todas as alternativas, o emprego das expresses ou termos destacados est corretamente identificado, EXCETO em a) ...ele no entende nada, sabe apenas que um pedao de sua carne ser retirado... (limitao) b) Est escura, vazia, somente uma velha, l na frente, deve estar pedindo tambm alguma coisa. (localizao) c) Mas at os exames h os hierglifos que ele procura decifrar. (incluso) d) Mas pedir o qu, exatamente? Mesmo assim entra na igreja. (concesso)

1 B

P R O V A

9. Em todas as alternativas, a palavra destacada explicita a relao entre as idias, mantendo o significado bsico das passagens do texto, EXCETO em a) ...a cara dos mdicos, do outro lado da mesa, sempre enigmtica, portanto faz parte da consulta, da profisso e dos honorrios... b) ...a cara dos mdicos (...) sempre enigmtica, faz parte da consulta, da profisso e dos honorrios: o jeito , ento, o paciente ficar paciente. c) O que significa ficar bom? Significa ser como antes, (...) mas ele nunca fora bom. d) Seu nome no cheque o surpreende, pois no mais ele. 10. Em todas as alternativas, a palavra destacada est corretamente interpretada, EXCETO em a) E as mulheres, as crianas, o mar, os livros que gostaria de ler tudo ser a mancha to escura
e estranha como a velha que rezava na igreja.

(que = os livros) b) Est numa igreja onde se adora um Deus em que ele no acredita. (que = Deus) c) Mas at os exames h os hierglifos que ele procura decifrar. (que = os hierglifos) d) Pelo jeito, ela deve saber o que est pedindo - o que no o caso dele. (que = ela) 11. O texto da obra Sargento Getlio, de Joo Ubaldo Ribeiro, desvia-se do portugus padro culto. Esteticamente esse procedimento se justifica porque a) a gerao a que pertence o autor utiliza erros de linguagem como provocao. b) a linguagem narrativa deve-se ajustar ao ponto de vista escolhido para a narrao. c) a lngua portuguesa falada no Brasil apresenta variaes regionais. d) o romance se enquadra nas tendncias regionalistas da literatura brasileira.

580

PORTUGUS

UFMG
F A S E

12. Embora o coronelismo seja entendido como uma forma especfica de poder poltico que floresceu durante a Primeira Repblica, traos dele permaneceram por muito tempo na histria poltica do Brasil. Na obra Sargento Getlio podem ser assinalados resqucios do coronelismo. Em todas as alternativas que se seguem manifesta-se diretamente o poder dos coronis, EXCETO em a) No gosto desse servio, no gosto de levar preso. Avexame. Depois que levar vosmec l, assento
os quartos num lugar e largo essa vida de cigano. S se doutor Z Antunes pedir muito.

1 B

b) Ningum entra numa usina para tirar um cabra. No gosto disso, contra a lei. Devia ser
contra a lei. Por que o homem tem o direito de passar a vida corrido, atocaiado numa usina? privilege.

P R O V A

c) Quando matei, nem pensei mais em matar. Matei sem raiva. Pensei que no, antes da hora,
pensei que ia com muita raiva, mas no fui. Cheguei, olhei, ela deitada assim e ainda perguntou: que que tem?

d) Vosmec no acredito que tenha visto um homem resistindo da morte, porque o que me dizem
que vosmec manda, no faz. Est direito, na sua posio.

13. Ao chamar de Tragdia sua pea Vestido de noiva, Nelson Rodrigues distinguiu-a do drama a) pela diviso do espao cnico em trs planos distintos. b) pela fatalidade cega que se abate sobre as personagens. c) pela influncia do teatro grego na concepo de sua estrutura. d) pelo uso de uma linguagem nobre, de tom grandiloqente.

14. Em Vestido de Noiva, os rudos indicativos de um acidente de trnsito, repetidos diversas vezes ao longo da pea, a) funcionam como um motivo condutor vinculado infncia da personagem central. b) pertencem ao plano da alucinao, em que o atropelamento da personagem principal insistentemente repetido. c) realizam um vnculo entre o plano da realidade e os planos da alucinao e da memria. d) sugerem os acontecimentos que esto por vir e que s ocorrem ao final da pea.

581

PORTUGUS

UFMG
F A S E

15. Considerando a obra Lira dos vinte anos, de lvares de Azevedo, pode-se dizer que em todas as alternativas a mulher aparece como referncia central da imagem, EXCETO em a) Meu desejo? era ser o sapatilho
Que teu mimoso p no baile encerra... A esperana que sonhas no futuro, As saudades que tens aqui na terra...

1 B

b) Acorda, minha donzela!


Foi-se a lua eis a manh E nos cus da primavera A aurora tua irm!

P R O V A

c) Na praia deserta que a lua branqueia,


Que mimo! que rosa, que filha de Deus! To plida ao v-la meu ser devaneia, Sufoco nos lbios os hlitos meus!

d) Se a vida lrio que a paixo desflora,


Meu lrio virginal eu conservei; Somente no passado tive sonhos E outrora nunca amei!

16. Em todas as alternativas, lvares de Azevedo mantm-se fiel proposta romntica de amor idealizado, EXCETO em a) Ah! vem plida virgem, se tens pena
De quem morre por ti, e morre amando, D vida em teu alento minha vida, Une nos lbios meus minhalma tua!

b) Namoro e sou feliz nos meus amores;


Sou garboso e rapaz... Uma criada Abrasada de amor por um soneto J um beijo me deu subindo a escada...

c) Plida luz da lmpada sombria,


Sobre o leito de flores reclinada, Como a lua por noite embalsamada, Entre as nuvens do amor ela dormia!

d) Um beijo divinal que acende as veias,


Que de encanto os olhos ilumina, Colhido a medo como flor da noite Do teu lbio na rosa purpurina, (...)

582

PORTUGUS

UFMG
F A S E

17. Todas as alternativas apresentam afirmaes verdadeiras sobre o narrador de Memrias de um sargento de milcias, de Manuel Antnio de Almeida, EXCETO a) Apresenta os acontecimentos na ordem temporal em que ocorreram. b) Busca trazer ao romance a fala coloquial do Brasil da poca. c) Sabe mais sobre os acontecimentos do que cada uma das personagens. d) Vale-se do humor para caricaturar hbitos da sociedade carioca em foco no romance.

1 B

18. A caracterstica de Memrias de um sargento de milcias que o aproxima dos ideais do Modernismo a) fazer uma crtica sociedade capitalista. b) libertar-se dos padres da fico romntica. c) recriar o ambiente social do tempo do rei. d) ter a narrao em terceira pessoa.

P R O V A

19. Leia o trecho seguinte, da crnica Romance policial carioca primeiro captulo, da obra Ai de ti, Copacabana!, de Rubem Braga.
Fritz Mller, alemo, divorciado, de 42 anos presumveis, importador, domiciliado Rua Cinco de Julho, 422, apt. 801, Copacabana, foi encontrado morto na cozinha de seu apartamento, com a cabea esmagada por um instrumento contundente. A vtima, que trajava apenas calo de banho, foi encontrada em decbito dorsal, com a cara voltada para a porta exterior da cozinha. O encontro foi feito pela domstica Severina de Arajo, de 27 anos, solteira, s 8h10m da manh de hoje, quando chegava ao apartamento para dar incio aos seus afazeres. Severina procurou o porteiro do edifcio, Aristbolo de Arajo, que comunicou o fato ao 2 Distrito Policial.

Pode-se afirmar que a passagem acima a) demonstra que o autor baseava suas crnicas na pgina policial dos jornais. b) exemplifica a objetividade dominante nas crnicas do autor. c) revela a preocupao do autor com a tradio do realismo social na lituratura. d) simula a linguagem jornalstica para fins de construo literria.

583

PORTUGUS
20. Leia com ateno a crnica que se segue, da obra Ai de ti, Copacabana! O PAVO
Eu considerei a glria de um pavo ostentando o esplendor de suas cores; um luxo imperial. Mas andei lendo livros, e descobri que aquelas cores todas no existem na pena do pavo. No h pigmentos. O que h so minsculas bolhas dgua em que a luz se fragmenta, como em um prisma. O pavo um arco-ris de plumas. Eu considerei que este o luxo do grande artista, atingir o mximo de matizes com o mnimo de elementos. De gua e luz ele faz seu esplendor; seu grande mistrio a simplicidade. Considerei, por fim, que assim o amor, oh! minha amada; de tudo que ele suscita e esplende e estremece e delira em mim existem apenas meus olhos recebendo a luz de teu olhar. Ele me cobre de glrias e me faz magnfico.
Rio, novembro, 1958.

UFMG
F A S E

1 B 2 A

P R O V A

Assinale a alternativa que contm uma afirmao correta sobre a crnica. a) b) c) d) A explicao cientfica da natureza considerada mais importante que a fantasia. A objetividade torna o texto voltado com exclusividade para o pavo. O autor utiliza o pavo para construir uma metfora da criao artstica. O cronista limita-se a registrar sua percepo sensvel do mundo exterior.

INSTRUO: As questes de 01 a 04 baseiam-se no texto abaixo. Leia atentamente todo o texto antes de resolv-las.
EDUCAO: AS PESSOAS E AS COISAS Tenho lido dezenas de estudos recheados de tabelas e grficos provando sociedade o que toda gente sabe: a educao bsica no Brasil vai muito mal. Na lista dos piores do mundo suplantamos s o Haiti e parece que tambm Moambique. A repetncia ao longo do primeiro ciclo altssima. Quem acompanhou as pesquisas do saudoso Srgio Costa Ribeiro, mestre em Estatstica, est consciente dessa verdadeira praga que vem sendo a reprovao sistemtica no ensino primrio. E a evaso fato notrio nas ltimas sries. Depois de muito bater a cabea procura das causas de uma situao to vexatria, os analistas foram descartando, uma a uma, as hipteses falsas ou frgeis. Lembro duas que ainda so correntes. Em primeiro lugar, no se trata de falta de vagas nem de prdios escolares. Pelo contrrio, h salas ociosas em algumas cidades do interior de So Paulo e dos estados sulinos. Afora os bolses de misria do Nordeste rural, o problema das vagas, outrora crnico, deixou de ser agudo, passou a conjuntural. Pode-se dizer, grosso modo, que as crianas brasileiras tm seu lugar nas escolas. O acesso ao primeiro grau foi razoavelmente ampliado nos ltimos anos. Logo, investir em construes no deve ser prioridade para nossos governos estaduais e municipais. O conbio de prefeito e empreiteira seria hoje um conluio. Outra hiptese aparentemente justa, mas ainda fora do alvo, atribua o fracasso escolar a fatores extra-educacionais, como a desnutrio dos alunos pobres, ou a baixa qualidade de vida das suas famlias de origem. Trata-se de males reais, sem dvida, mas, segundo fontes idneas, os casos de malogro no aprendizado devido m alimentao da criana no constituem regra geral. A administrao da merenda escolar estaria suprindo as deficincias mais graves, o que soa como

F A S E

P R O V A

584

PORTUGUS

UFMG
F A S E

uma boa nova em meio a tanta desolao. Igualmente a chamada carncia cultural do meio de onde provm o alunado j no o bode expiatrio que por tanto tempo serviu para justificar, no sem uma ponta de preconceito, a prtica da reprovao em massa. A causa das causas do nosso desastre pedaggico no se acharia portanto nem na falta de prdios e vagas, nem nas condies de sade, nem na extrao cultural das crianas e adolescentes do pas. Onde identificar o x da questo? Entre as quatro paredes da sala de aula. Na relao do professor com a sua profisso. Chegando mais perto: nas mentes e nos coraes dos nossos mestres. E para no cair na tentao fcil do idealismo: o n grdio a ser cortado o que amarra as condies que o professor leva dentro de si quando exerce o magistrio. Tanto a sociedade civil quanto os aparelhos estatais pensam e agem como se ignorassem este fato cotidiano mas espantoso: o nosso professor primrio remunerado como se fosse um operrio no qualificado. O seu salrio nada tem a ver com a importncia crucial da sua funo pblica, que a de primeiro motor do desenvolvimento, nem merece as cidas cobranas de eficincia que periodicamente lhe fazem a mesma sociedade e o mesmo estado, que o deixaram mngua. Tive o cuidado de comparar os vencimentos de professores da rede oficial em vrias unidades da federao. O docente de primeiro grau, aquele a quem a nao delega o encargo de ensinar a ler, escrever e contar, ganha, em mdia, dois reais por aula nas provncias mais bem aquinhoadas do Sudeste e do Sul. Faamos as contas, o que sempre mais honesto do que fazer-de-conta que tudo vai bem. Para receber cinco magros salrios mnimos, este nosso bia-fria do giz e da lousa teria que dar em torno de duzentas e cinqenta aulas por ms. Ou seja, teria que cumprir a faanha de trabalhar entre dez e doze horas por dia. Calculo por mdias, mas no escondo extremos afrontosos. Professores h, no Vale do Jequitinhonha, que amargam quarenta reais por ms, isto , cinqenta centavos por aula. Quanto ao limite superior da amostra: no Rio, em So Paulo e em Belo Horizonte, cidades onde melhor se paga ao mestre-escola, a hora-aula no vale mais que dois reais e meio. um salrio menos vil, relativamente, mas, absolutamente, sempre vil. Escolhi o teto de cinco salrios mnimos, pois no ousei tocar a soleira dos 763 reais, que representariam aquele quantum necessrio para que uma famlia de quatro pessoas pudesse viver hoje com alguma decncia. Louvo-me aqui na estimativa do Dieese, Departamento Intersindical de Estatstica e Estudos Scio-Econmicos. Temo que este cenrio de desvalorizao brutal do professor primrio no mude to cedo. Li com interesse o elenco das prioridades com que o governo pretende enfrentar o desafio da educao bsica. No discuto intenes. Apenas constato o primado das coisas sobre as pessoas. Computadores aos milhares sem professores prezados e estimulados so sucata virtual. Livros didticos sem mestres que os leiam e os interpretem com garra e entusiasmo so pilhas de papis destinados ao lixo do esquecimento. Quanto s avaliaes severas prometidas (tremam os relapsos afundados nos seus pingues proventos!), supem um longo tempo de experincia em condies humanas de trabalho. As pessoas, quando respeitadas no seu ofcio, produzem sentido e valor. Com ou sem as coisas. Mas as coisas sem as pessoas so letra morta. Preferir coisas a pessoas no realismo. apenas conformismo. Alfredo Bosi, Jornal do Brasil, Caderno Idias, p. 7, 10.02.96.

2 A

P R O V A

585

PORTUGUS
QUESTO 01 COMPLETE os espaos com informaes do texto. O fato: Estudos revelam que a educao bsica no Brasil vai muito mal. 1. Indicadores de tal fato: a) b) 2. Algumas causas aventadas: a) b) 3. Medidas governamentais j tomadas para tentar solucionar o problema: a) b) 4. Medidas governamentais anunciadas para tentar solucionar o problema: a) b)

UFMG
F A S E

2 A

P R O V A

QUESTO 02 Leia atentamente as passagens que se seguem.


A causa das causas do nosso desastre pedaggico no se acharia, portanto, nem na falta de prdios e vagas, nem nas condies de sade, nem na extrao cultural das crianas e adolescentes do pas.

A causa das causas do nosso desastre pedaggico no se acha, portanto, nem na falta de prdios e vagas, nem nas condies de sade, nem na extrao cultural das crianas e adolescentes do pas. REDIJA um texto, EXPLICANDO o efeito da alterao efetuada de uma passagem para outra.

QUESTO 03
Onde identificar o x da questo? Entre as quatro paredes da sala de aula. Na relao do professor com a sua profisso.

REDIJA um texto, JUSTIFICANDO essa afirmativa.

QUESTO 04
As pessoas, quando respeitadas no seu ofcio, produzem sentido e valor. Com ou sem as coisas. Mas as coisas sem as pessoas so letra morta.

REDIJA um texto, ARGUMENTANDO favoravelmente ou contrariamente afirmativa do autor.

586

PORTUGUS
QUESTO 05

UFMG
F A S E

REDIJA, um texto, EXPLICANDO a funo do plano da realidade, relacionando-o aos planos da memria e da alucinao, na pea Vestido de noiva, de Nelson Rodrigues.

2 A

QUESTO 06 Heris so personagens que se distinguem por suas virtudes, nobreza de alma e carter irrepreensvel. Levando em considerao a afirmativa acima, REDIJA um texto, DEMONSTRANDO que a personagem Leonardo Filho, em Memrias de um sargento de milcias, de Manuel Antnio de Almeida, pode ser caracterizada como um anti-heri, por suas atitudes e aes. P R O V A

QUESTO 07 Leia com ateno os versos que se seguem, dos poemas Soneto e ela! ela! ela! ela!, da obra Lira dos vinte anos, de lvares de Azevedo.

Soneto

ela! ela! ela! ela!

Plida luz da lmpada sombria, Sobre o leito de flores reclinada, Como a lua por noite embalsamada, Entre as nuvens do amor ela dormia! Era a virgem do mar, na escuma fria Pela mar das guas embalada! Era um anjo entre nuvens dalvorada Que em sonhos se banhava e se esquecia!

Como dormia! que profundo sono! Tinha na mo o ferro do engomado Como roncava maviosa e pura! Quase ca na rua desmaiado! Afastei a janela, entrei medroso: Palpitava-lhe o seio adormecido Fui beij-la roubei do seio dela Um bilhete que estava ali metido

REDIJA um texto, MOSTRANDO as diferentes percepes da mulher manifestadas nos versos dos dois poemas. QUESTO 08 Leia com ateno a afirmativa que se segue, de Davi Arrigucci Jr.
Muito prximo do evento mido do cotidiano, o cronista deve de algum modo dribl-lo, se no quiser naufragar agarrado ao efmero.

REDIJA um texto, DEMONSTRANDO que a afirmativa se aplica obra Ai de ti, Copacabana!, de Rubem Braga. UTILIZE, em sua resposta, um exemplo tomado a qualquer uma das crnicas do livro.

587

PORTUGUS

UFMG
F A S E

INSTRUO: As questes de 01 a 04 baseiam-se no texto abaixo. Leia atentamente todo o texto antes de resolv-las.
A EUTANSIA Os rpidos avanos verificados no campo da medicina esto aos poucos alterando os milenares padres ticos da arte de Hipcrates. Faz parte do juramento do pai da medicina a promessa de jamais, por motivo algum, tirar a vida do paciente. Se isso era compreensvel na Grcia Antiga e mesmo at poucas dcadas atrs, hoje torna-se cada vez mais discutvel. H pouco a Holanda baixou uma legislao considerada bastante liberal acerca da eutansia (termo que em grego significa literalmente boa morte). Esta semana, um tribunal federal de Nova York autorizou mdicos desse Estado, de Vermont e de Connecticut a prescrever drogas que acelerem a morte de pacientes terminais que queiram morrer e estejam lcidos. Se at h pouco entrar em estado de coma muitas vezes significava a morte pelo comprometimento das funes cerebrais que coordenam o funcionamento de rgos vitais, hoje aparelhos podem sustentar a vida do paciente por anos e anos a fio. Se at h pouco um diagnstico de cncer significava uma sentena de morte mais ou menos rpida, hoje terapias podem prolongar a vida do doente, mesmo que padea de dores atrozes suportveis apenas com o uso de altas doses de morfina. At que ponto o mdico deve preservar a vida do paciente? Agir deliberadamente para mat-lo uma atitude tica? Na impossibilidade de comunicao entre doente e mdico, a famlia tem o direito de decidir se ele vai viver ou morrer? E se esse paciente no tiver familiares? Ser o Estado que vai decidir a sua sorte? Essas so apenas algumas das novas questes que os avanos no campo da medicina comeam a suscitar. No Primeiro Mundo, o debate j est instalado, e muitos pases j esto modernizando suas legislaes. No Brasil, a situao da sade to precria que o acaso e no a vontade quem escolhe aqueles que vo sobreviver e aqueles que vo morrer. Folha de So Paulo, Editorial, p. 1-2, 05.04.1996.

2 B

P R O V A

QUESTO 01 REDIJA um texto, EXPLICANDO a passagem que se segue, no contexto social brasileiro de hoje.
No Brasil, a situao da sade to precria que o acaso e no a vontade quem escolhe aqueles que vo sobreviver e aqueles que vo morrer.

QUESTO 02
At que ponto o mdico deve preservar a vida do paciente? Agir deliberadamente para mat-lo uma atitude tica? Na impossibilidade de comunicao entre doente e mdico, a famlia tem o direito de decidir se ela vai viver ou morrer? E se esse paciente no tiver familiares? Ser o Estado que vai decidir a sua sorte?

REDIJA um texto, ARGUMENTANDO contrariamente ou favoravelmente eutansia.

588

PORTUGUS
QUESTO 03 Leia a passagem que se segue.
Se isso era compreensvel na Grcia Antiga e mesmo at poucas dcadas atrs, hoje torna-se cada vez mais discutvel.

UFMG
F A S E

2 B

a) DEFINA o tipo de relao que se estabelece entre as oraes no perodo acima. b) REESCREVA a passagem, MANTENDO seu sentido bsico e ELIMINANDO a palavra destacada. Se necessrio, faa adaptaes. P R O V A

QUESTO 04 IDENTIFIQUE a funo sinttica das oraes em destaque. a) hoje terapias podem prolongar a vida do doente, mesmo que padea de dores atrozes
suportveis apenas com o uso de altas doses de morfina.

b) Esta semana, um tribunal () autorizou mdicos () a prescrever drogas que acelerem a morte
de pacientes terminais

c) famlia tem o direito de decidir se ele vai viver ou morrer? d) Agir deliberadamente para mat-lo uma atitude tica? e) Agir deliberadamente para mat-lo uma atitude tica?

QUESTO 05 Leia com ateno o trecho que se segue, da obra Sargento Getlio, de Joo Ubaldo Ribeiro.
No sei, no sei, diz o padre, sacudindo a cabea e fazendo um bico com a boca. Por que vosmec no some? Eu sumir, eu sumir? Como que eu posso sumir, se primeiro eu sou eu e fico a me vendo sempre, no posso sumir de mim e eu estando a sempre estou, nunca que eu posso sumir. Quem some os outros, a gente nunca.

REDIJA um texto, RELACIONANDO esse trecho com o final do romance.

QUESTO 06 REDIJA um texto, JUSTIFICANDO o ttulo da obra Lira dos vinte anos de lvares de Azevedo.

589

PORTUGUS

UFMG
F A S E

QUESTO 07 Leia os dois trechos que se seguem, de Vestido de Noiva, de Nelson Rodrigues: o primeiro, do final do primeiro ato, e o segundo, do segundo ato.
TRECHO 1 (no plano da memria) (D. Laura parece ter notado a presena de uma pessoa que at ento no vira. Dirige-se a essa pessoa invisvel, beijando-a, presumivelmente, na testa.) D. LAURA Desculpe. Eu no tinha visto voc. (Pausa para uma resposta que ningum ouve.) D. LAURA (risonha) Quando o seu? (Pausa para outra resposta.) D. LAURA (maliciosa) Qual o qu? Est a, no acredito! To moa, to cheia de vida. PAI (para Alade, que est pronta) Ento vamos! (D. Laura faz um gesto qualquer para a invisvel pessoa e vai para junto de Alade.) D. LAURA Cuidado com a cauda! (D. Laura apanha a imaginria cauda e entrega-a a Alade.) ALADE (num ltimo olhar) No falta mais nada? ME (olhando tambm) Nada. Acho que no. PAI (impaciente) J tarde. Vamos descer. (Marcha nupcial. Trevas.) FIM DO PRIMEIRO ATO TRECHO 2 (no plano da memria) (D. Laura beija-a [a mulher de vu] na testa.) D. LAURA (risonha) Quando o seu? MULHER DE VU Tem tempo! (noutro tom) (com certa amargura) Nunca! D. LAURA (maliciosa) Qual o qu! Est a, no acredito! To moa, to cheia de vida! PAI (para Alade, que est pronta) Ento, vamos. (Som da Marcha nupcial.) (D. Laura faz um gesto qualquer para a mulher de vu e vai para junto de Alade.) D. LAURA (solcita) Cuidado com a cauda! (Apanha a cauda, que entrega a Alade.) ALADE (num ltimo olhar) No falta mais nada? (Todos olham, estando situados como no final do primeiro ato.) ME (olhando em torno) No. Acho que no. PAI (impaciente) J tarde! Vamos descer! (Ao som da Marcha nupcial, saem os personagens do casamento. Fica a mulher de vu, numa atitude pattica. Luz amortecida)

2 B

P R O V A

REDIJA um texto, EXPLICANDO a relao entre as duas cenas. QUESTO 08 REDIJA um texto, EXPLICANDO como o narrador se relaciona com seu virtual leitor no romance Memrias de um sargento de milcias, de Manuel Antnio de Almeida.

590

PORTUGUS
LNGUA PORTUGUESA
AS QUESTES DE 1 A 5 BASEIAM-SE NO TEXTO ABAIXO. ASSINALE A NICA OPO QUE COMPLETA CORRETAMENTE OS ENUNCIADOS.

UFPA
F A S E

Concordei, para dizer alguma coisa, para sair da espcie de sono magntico, ou o que quer que era que me tolhia a lngua e os sentidos. Queria e no queria acabar a conversao: fazia esforo para arredar os olhos dela, e arredava-os por um sentimento de respeito; mas a idia de parecer que era aborrecimento, quando no era, levava-me os olhos outra vez para Conceio. A conversa ia morrendo. Na rua, o silncio era completo. (Missa do Galo, Machado de Assis)

1. O termo mas pode ser substitudo, sem alterao de sentido por a) pois b) por conseguinte c) portanto d) conseqentemente e) contudo

2. A forma verbal mais usada (pretrito imperfeito do indicativo) traduz no texto a) incerteza b) fatos passados em relao ao momento da narrao e presentes em relao ao momento da conversao c) aes repetidas d) fatos presentes em relao ao momento da narrao, passados em relao a outros fatos e) fatos totalmente passados 3. Em ... levava-me os olhos outra vez para Conceio. a funo sinttica de me a) objeto direto b) adjunto adverbial c) objeto indireto d) adjunto adnominal e) agente da passiva

4. Na orao ... que me tolhia a lngua e os sentidos. h a) metonmia b) eufemismo c) silepse d) aliterao e) catacrese

5. A expresso ... arredar os olhos dela, equivale a a) tirar os olhos dela do lugar b) no olhar para mim c) no olhar para ela d) afastar os olhos dela de onde estavam e) seus olhos mudarem de direo

591

PORTUGUS
AS QUESTES DE 6 A 10 ESTO RELACIONADAS AOS FRAGMENTOS EXTRADOS DE LNGUA E PODRES PODERES, COMPOSIES DE CAETANO VELOSO. ASSINALE A NICA OPO QUE COMPLETA CORRETAMENTE OS ENUNCIADOS.

UFPA
F A S E

LNGUA Gosto de sentir a minha lngua roar A lngua de Lus de Cames Gosto de ser e de estar E quero me dedicar A criar confuses de prosdia E uma profuso de pardias Que encurtem dores E furtem cores como camalees Gosto de pessoa na pessoa Da rosa na rosa

PODRES PODERES Ser que apenas os hemisfrios Pascoais E os tons e os mil tons, seus sons e seus dons geniais Nos salvam, nos salvaro dessas trevas E nada mais?

6. Todas as afirmativas abaixo esto corretas EXCETO a a) No texto Lngua o verbo roar refora a conotao ertica da palavra lngua, que o antecede. b) Nos dois textos h referncias a compositores nacionais. c) Em ambos os textos percebe-se a reiveno da linguagem. d) Os dois textos apresentam a mesma temtica. e) O jogo de significaes fundindo o abstrato com o correto est presente em ambos os textos. 7. Em Lngua Caetano diz
E quero me dedicar A criar confuses de prosdia...

Em Podres Poderes aparece referncia a um elemento prosdico atravs da palavra a) sons b) tons c) hermetismos d) dons e) trevas

592

PORTUGUS

UFPA
F A S E

8. Reconhecendo as funes de linguagem nos dois textos, observa-se que, alm da funo potica, a) b) c) d) e) em Lngua ressalta-se a funo emotiva e em Podres Poderes a referencial. em Lngua revela-se a funo expressiva, em Podres Poderes a conativa. em Lngua nota-se a funo ftica, em Podres Poderes ressalta-se a funo emotiva. em Lngua destaca-se a funo referencial, em Podres Poderes a expressiva. em Lngua predominava a funo conativa, em Podres Poderes a referencial.

9. Nos versos
Que encurtem dores E furtem cores como camalees

A palavra que estabelece relao coesiva com outras de versos anteriores a) cores b) dores c) que d) e e) camalees

10. Na estrutura mrfica das formas verbais salvam e salvaro, o ltimo a em a) b) c) d) e) salvam vogal temtica, em salvaro faz parte da desinncia modo-temporal. salvam e em salvaro, vogal temtica. salvam e em salvaro, faz parte da desinncia modo-temporal. salvam e em salvaro, faz parte do sufixo. salvam desinncia modo-temporal, em salvaro faz parte do sufixo.

LITERATURA BRASILEIRA 1. Ao abordar a figura feminina atravs de uma perspectiva conflituosa, no soneto A Mesma D. ngela, Gregrio de Matos assim representa o ltimo terceto:
Mas vejo que por bela, e por galharda. Posto que os anjos nunca do pesares. Sois anjos que me tenta e no me guarda.

Da podemos concluir que a) os elementos da natureza refletem com exatido a beleza feminina. b) a figura feminina aparece etrea, divinizada, e, como tal, bem distante do eu lrico. c) a nica felicidade terrena est na realizao amorosa - o que prega tal terceto. d) coexistem a seduo mstica e carnal. e) a noo de efemeridade da vida ressalta neste terceto pelos jogos de metforas utilizados.

593

PORTUGUS

UFPA
F A S E

2. Leia o trecho abaixo, transcrito da Lira XV de Tomas Antnio Gonzaga, e assinale a alternativa que contm a frase que o relaciona corretamente com a poca em que foi composto.
Eu, Marlia, no fui algum vaqueiro. Fui honrado Pastor da tua aldeia Vestia finas ls, e tinha sempre A minha choa do preciso cheia.

a) Percebe-se, claramente, a preocupao do Arcadismo com imagens elaboradas. b) Nesse trecho perfeitamente perceptvel a tentativa de contornar a rusticidade prpria da vida pastoril, quando se refere ao pastor que vestia finas ls. c) A forma de imparcialidade e impessoalidade do texto rcade a se v pelo uso da 1 pessoa verbal. d) Evidencia-se que a natureza do problema tratado pertence a toda a comunidade dos pastores, logo atingiria um grande nmero de leitores. e) O carter autobiogrfico do trecho marcado pelo uso do verbo no pretrito. 3. No conjunto das obras literrias romnticas brasileiras percebe-se que houve uma inteno deliberada de construir uma literatura nacional: um levantamento de toda a vida brasileira feito por meio da obra de arte literria o que, por si s, suficiente para percebermos que os poetas tinham necessidade de buscar as suas razes nacionais. Tal necessidade est refletida, na poesia, principalmente, pela: a) b) c) d) tendncia indianista, tambm encarada como tentativa de volta Idade Mdia. viso pessimista da vida. preocupao de imitar a maneira francesa de fazer poesias atravs das obras de Victor Hugo. explorao da temtica amorosa, cheia de sentimentalismo, chegando, em alguns casos s raias do pieguismo. e) presena da natureza clssica e impassvel ante os dramas existenciais dos poetas. 4. Os contos Missa do Galo e Uns Braos de Machado de Assis apresentam em comum a) o foco narrativo de ambos, em 1 pessoa. b) a anlise do adultrio que colocado neles, explicitamente e cujo foco dirigido para as conseqncias do mesmo na formao da famlia. c) o registro da atrao que rapazes adolescentes sentiriam pela mulher madura, da a caracterizao dos personagens. d) o estudo das seqelas que uma paixo avassaladora deixa na alma feminina e seus reflexos no ambiente familiar. e) a constatao da dor moral em que se debate o homem, vtima do adultrio.

594

PORTUGUS

UFPA
F A S E

5. Sobre a narrativa curta de Maria Lcia Madeiros em Zeus ou A Menina e os culos e Rubem Fonseca em Lcia Mc Cartney, respectivamente, encaradas em conjunto, pode-se dizer que: a) desenvolvem regionalismos distintos explorando aspectos pitorescos tanto das criaturas como das situaes; b) no apresentam relatos onde os fatos e acontecimentos vm concatenados segundo lgica temporal rigorosa; c) abandonam a linguagem falada, dando nfase ao uso sofisticado da linguagem escrita; d) o mundo mgico que desenvolvem justifica plenamente o absurdo das situaes que abordam; e) protestam, em primeiro plano, em favor dos desvalidos sociais. 6. Olavo Bilac, em sua Profisso de F, ao tratar do fazer potico, elege a forma como sua deusa dizendo:
...Minha pena Segue esta norma, Por te servir, Deusa Serena Serena Forma!

e compara a tarefa do poeta com o lavor do ourives numa atitude tpica do parnasianismo, sobrepondo a forma ao contedo. Enquanto isso, Cruz e Souza, na Antfona, sob uma perspectiva simbolista a) prega uma forma fluida, evanescente como em: Formas alvas, brancas. Formas claras / de luares, de neves, de neblinas!... que mais se acentua quando se nota o descompasso entre substantivos e adjetivos. b) prefere extravasar, em primeiro plano, seu sentimento amoroso frustrado, metaforizado, em expresses do tipo: Flores negras do tdio e flores vagas / de amores vos, tantlicos, doentios. c) pretende atingir a perfeio formal absoluta, recomendando as rimas brilhantes e inflamadas. d) aconselha que a forma expresse, na poesia, carnes de mulher, delicadezas... para que se crie uma atmosfera marcadamente ertica. e) procura banir totalmente de seus versos a atmosfera encantatria do sonho, conforme diz na poesia referida: Do Sonho, as mais azuis diafanidades / que fuljam, que na Estrofe se levantem, resultante da preocupao formal.

595

PORTUGUS

UFPA
F A S E

7. Nos captulos iniciais de Lucola o personagem-narrador, ao conhecer Lcia, personagem principal do romance, atrapalha-se todo, o que o leva a confessar:
Compreendi e corei de minha simplicidade provinciana, que confundira a mscara hipcrita do vcio com o modesto recato da inocncia. S ento notei que aquela moa estava s, e que a ausncia de um pai, de um marido, ou de um irmo, devia-me ter feito suspeitar a verdade.

Pelo desenrolar do enredo fica evidente que a verdade a que se referia o personagem-narrador, e de que ele sequer suspeitava, era a) a condio de herdeira rica, que Lcia desfrutava na vida social do seu tempo. b) o fato de que a personagem trabalhava para se manter, confeccionado luxuosos trajes com que as mulheres desfilavam, tanto nas peras quanto nos saraus. c) que Lcia nascera no campo e viera cidade para tentar arranjar marido. d) extremamente chocante para os padres da poca: tratava-se de uma meretriz acostumada a passar as noites em meio a festas e bacanais. e) vista pelos demais personagens como problema fcil a ser resolvido: a condio virginal de Lcia. 8. Na ltima estrofe da Orao da Cabra Preta, Bruno de Menezes diz:
... E agora quando ela volta da cozinha dos ricos / mestre Desidrio fumando descansado, / est espera do quentinho dela / como se fosse sua companheira / para ambos gozarem o fastio do amor....

Se relacionarmos tal estrofe com as demais que constituem o poema, podemos afirmar que a) o personagem, Mestre Desidrio, teve o desprazer de ver seus desejos frustrados, por isso os versos seguem a maneira tradicional de poetar, tambm, a indiferena do personagem e o tdio da espera. b) todos os fatos e acontecimentos relatados atravs da poesia aconteceram apenas na imaginao do personagem, assim se justifica o verso: como se fosse sua companheira. c) o final feliz do poema: o uso do verso livre, da ausncia de rima e o tom prosaico da estrofe representam a adeso do autor a alguns dos ideais modernistas. d) os versos deste poema marcam, atravs de onomatopias, o ritmo das danas africanas o que combina perfeitamente com o contedo que desenvolvido nele. e) se refere concluso feliz de uma sesso realizada em um terreiro de umbanda onde no faltaram a cachaa, incorporaes e danas.

596

PORTUGUS
LNGUA PORTUGUESA OS SAPOS
(Manuel Bandeira) 1 Enfunando os papos, Saem da penumbra, Aos pulos, os sapos. A luz os deslumbra. Em ronco que aterra, Berra o sapo-boi: Meu pai foi guerra! No foi! Foi! No foi! O sapo-tanoeiro, Parnasiano aguado, Diz: Meu cancioneiro bem martelado. Vede como primo Em comer os hiatos! Que arte! E nunca rimo Os termos cognatos. O meu verso bom Frumento sem joio. Fao rimas com Consoantes de apoio. Vai por cinqenta anos Que lhes dei a norma: Reduzi sem danos A formas a forma. Calme a saparia Em crticas cticas: No h mais poesia Mas h artes poticas... 8 Urra o sapo-boi: Meu pai foi rei Foi! No foi! Foi! No foi! Brada em um assomo O sapo-tanoeiro: A grande arte como Lavor de joalheiro.

UFPA
F A S E

9 2

10 Ou bem de estatutrio. Tudo quanto belo. Tudo quanto vrio, Canta no martelo. 11 Outros, sapos-pipas (Um mal em si cabe), Falam pelas tripas: Sei! No sabe! Sabe!. 12 Longe dessa grita, L onde mais densa A noite infinita Verte a sombra imensa; 13 L, fugido ao mundo, Sem glria, sem f, No perau profundo E solitrio, 14 Que soluas tu, Transido de frio, Sapo cururu Da beira do rio...

Leia com ateno o poema Os Sapos, de Manuel Bandeira, e responda s questes propostas.

1. Quem so os sapos, referidos pelo poeta?

2. Explique, denotativamente, a frase encontrada na 3 estrofe: Meu cancioneiro bem martelado.

3. Justifique o plural da palavra sapo-pipa encontrado na 11 estrofe.

4. Explique o uso da vrgula, depois de Outros, na 11 estrofe (Outros, sapos-pipas (Um mal em si cabe), ).

597

PORTUGUS

UFPA
F A S E

5. Na 2 estrofe o poeta usou, no 1 verso, o demonstrativo dessa. O que justifica o uso de dessa em vez de desta? 6. Ainda na 12 estrofe, 2 verso, encontra-se o advrbio l. Qual ou quais o(s) termo(s) de referncia que o advrbio representa no poema? 7. Na 2 estrofe, ocorre uma orao adjetiva. Transcreva a substituio dessa orao pelo adjetivo correspondente.

8. Na 4 estrofe o poeta diz que nunca rima os termos cognatos. Para a gramtica normativa, que so termos cognatos? Exemplifique. LITERATURA LUSO-BRASILEIRA 1. Em certa altura do conto Cantiga de Esponsais h um corte no processo narrativo para que o narrador reflita, a propsito dos fatos e acontecimentos narrados; ento exclama:
Ah! se mestre Romo pudesse seria um grande compositor. Parece que h duas sortes de vocao, as que tem lngua e as que no tem. As primeiras realizam-se; as ltimas representam uma luta constante e estril entre o impulso interior e a ausncia de um modo de comunicao com os homens.

a) Relacione a atuao de mestre Romo, no conto em questo, com a teoria das vocaes expressa no trecho acima. b) Que situao nova inserida no desenvolvimento do enredo deste conto e que torna o desfecho mais dramtico? 2. Leia o trecho de Podres Poderes de Caetano Veloso que transcrevemos a seguir:
...Enquanto os homens exercem seus podres poderes ndios, padres e bichas negros e mulheres E adolescentes Fazem o carnaval Queria querer cantar afinado com eles Silenciar em respeito seu transe, num xtase Ser indecente Mas tudo muito mau...

Se considerarmos a mensagem da estrofe mencionada, podemos dividi-la em 2 segmentos: no primeiro constata-se um fato e no segundo, o eu-lrico se posiciona ante ele. Mediante as consideraes feitas, a) Identifique o fato abordado na estrofe e a posio que o eu-lrico assume. b) No que diz respeito prtica da evaso, que contraste se estabelece, por exemplo, entre esta estrofe e o poema Vou-me embora pra Pasrgada de Manoel Bandeira?

598

PORTUGUS

UFPA
F A S E

3. O poema narrativo Cobra Norato de Raul Bopp segue a estrutura das narrativas orais populares: h um heri que sai em busca de seu objetivo; no caminho, de um lado, encontra vrios obstculos que tentam lhe impedir de atingi-lo; de outro lado, encontra elementos que lhe ajudam a chegar a sua meta. Pergunta-se: a) Qual o objetivo do heri do poema em questo? Cite, pelo menos, dois obstculos que ele encontra pelo caminho. b) Qual a atmosfera bsica do poema e que j se evidencia no 1 Canto por meio dos seguintes versos:
Ento tem de fechar os olhos primeiro O sono escorregou nas plpebras pesadas Um cho de lama rouba as foras dos meus passos.

4.

Dois homens ergueram o morto ao alto sobre a amurada. Deram-lhe o balano para o arremessarem longe. E, antes que o baque do cadver se fizesse ouvir na gua, todos viram, e ningum j pde segurar Mariana, que se atirara ao mar. voz do comandante desamarraram rapidamente o bote, saltaram homens para salvar Mariana. Salv-la! Viram-na, um momento, bracejar, no para resistir morte, mas para abraar-se ao cadver de Simo, que uma onda lhe atirou aos braos. O comandante olhou para o stio donde Mariana se atirara, e viu, enleado no cordame, o avental, e flor dgua um rolo de papis que os marujos recolheram na lancha. Eram, como sabem, a correspondncia de Teresa e Simo. (Amor de Perdio - Camilo Castelo Branco)

O texto, em destaque, focaliza o eplogo do romance Amor de Perdio. Partindo do episdio apresentado, que relao voc faz entre o destino de Mariana e o de Lcia, protagonista de Lucola, de Jos de Alencar? Justifique sua resposta. 5. Ler com ateno os dois textos A
Lusa olhava-o. Achava-o mais varonil, mais trigueiro. No cabelo preto anelado havia agora alguns fios brancos; mas o bigode pequeno tinha o antigo ar moo, orgulhoso e intrpido; os olhos, quando ria, a mesma doura amolecida, banhada num fluido. Reparou na ferradura de prola da sua gravata de cetim preto, nas pequeninas estrelas brancas bordadas nas suas meias de seda. A Bahia no o vulgarizara. Voltava mais interessante! (O Primo Baslio - Ea de Queirs)

B
De repente supreendeu-se a dizer: Como deve ser bom o seu amor!... E ficou a cismar, a fazer conjeturas, a julg-lo minuciosamente da cabea aos ps. Parou nos olhos: Quantos tesouros de ternura no estariam neles escondidos? neles, do feitio de amndoas, banhados de bondade e cercados de pestanas crespas e negras, como os plos de um bicho venenoso; aquelas pestanas lembravam-lhe as sedas de uma aranha caranguejeira. Estremeceu, porm, vieram-lhe desejos de os apalpar com os lbios. Como devia ser bom ouvir dizer Eu te amo! por aquela boca e por aquela voz!... (O Mulato - Alusio de Azevedo)

Nos dois textos, aparecem registros das observaes de Lusa (O Primo Baslio) e de Ana Rosa (O Mulato), acerca de Baslio e de Raimundo, respectivamente. Pergunta-se: Como as duas heronas vem o homem amado a partir da esttica a que se filiaram os romancistas em questo.

599

PORTUGUS
6.
Pe-me onde se use toda a feridade, Entre lees e tigres, e verei Se neles achar posso a piedade Que entre feitos humanos no achei Ali, com amor intrseco e vontade Naquele por quem morro, criarei Estas relquias suas que aqui viste, Que refrigrio sejam da me triste.

UFPA
F A S E

(Os Lusadas - Cames)

Tomando por base as palavras de Ins de Castro expressas na estrofe acima transcrita, explique o significado dos quatro primeiros versos, considerando a relao possvel entre realidade e fico.

7.

Mar Portugus Mar salgado, quanto do teu sal So lgrimas de Portugal! Por te cruzarmos, quantos mes choraram, Quantos filhos em vo rezaram! Quantas noivas ficaram por casar Para que fosses nosso, mar! Valem a pena? Tudo vale a pena Se a alma no pequena. Quem quer passar alm do Bojador Tem que passar alm da dor. Deus ao mar o perigo e o abismo deu, Mas nele que espelhou o cu. (Fernando Pessoa)

O poema acima transcrito permite que se tenha uma idia da natureza contraditria do mar portugus, na viso do poeta Fernando Pessoa. Interprete o pensamento do escritor quanto s conseqncias do progresso, ilustrando sua resposta com exemplos retirados do prprio texto.

600

PORTUGUS
8. Ler com ateno o texto.
Num bairro moderno Dez horas da manh; os transparentes Matizam uma casa apalaada; Pelos jardins estancam-se nascentes, E fere a vista, com brancuras quentes, A larga rua macadamizada Rez-de-chausse repousam sossegados, Abriram-se, nalguns, as persianas, E dum ou doutro, em quartos estucados, Ou entre a rama dos papis pintados, Reluzem, num almoo, as porcelanas. Como saudvel ter o seu aconchego, E a sua vida fcil! Eu descia, Sem muita pressa, para o meu emprego, Aonde agora quase sempre chego Com as tonturas duma apoplexia. E rota, pequenina, azafamada, Notei de costas uma rapariga, Que no xadrez marmreo duma escada, Com um retalho de horta aglomerada, Pousava ajoelhada, a sua giga. E eu, apesar do sol, examinei-a: Ps-se de p; ressoam-lhe os tamancos; E abre-se-lhe o algodo azul da meia, Se ela se curva, esguedelhada, feia E pendurando os seus bracinhos brancos Do patamar responde-lhe um criado: Se te convm, despacha; no converses. Eu no dou mais. E muito descansado, Atira um cobre lvido, oxidado, Que vem bater nas faces duns alperces. (Cesrio Verde)

UFPA
F A S E

Com base nas idias apresentadas no texto, justifique o ttulo do mesmo.

601

PORTUGUS
REDAO

UFPA
F A S E
izem que a primeira e maior inveno foi o fogo. Seria? E a fala? No mais importante? Outros querem que a primeira inveno seja a roda. At pode ser. Mas aqui, nas Amricas, os incas e astecas no usavam roda e se davam muito bem.

1. O recorte acima foi extrado do livro Noes de Coisas do antroplogo Darcy Ribeiro.

2. Releia-o atentamente e observe que as palavras de Darcy Ribeiro nos levam a pensar no quanto o homem, ao longo de sua histria, vem criando e descobrindo, incessantemente na expectativa de superar problemas.

3. Mas, em plena passagem para o terceiro milnio, apesar de suas fabulosas invenes e descobertas, o homem ainda no alcanou condies ideais de vida.

4. Faa uma Redao que expresse o seguinte: Em sua opinio, dentre as tantas invenes e descobertas que nos maravilham a todo instante, qual delas foi um passo significativo em direo a um mundo melhor?

5. Para construir o seu texto, reflita e fundamente seu ponto de vista.

602

PORTUGUS
LNGUA E LITERATURA [TEXTO I]
Encostei-me a ti, sabendo que eras somente onda. Sabendo bem que eras nuvem, depus a minha vida em ti. Como sabia bem tudo isso, e dei-me ao teu destino frgil, fiquei sem poder chorar, quando ca.

UFPE
F A S E

1. Ceclia Meireles, autora do texto acima, estudada na Literatura Brasileira como integrante do Modernismo. Em relao autora e ao movimento do qual participou, incorreto afirmar: a) O Modernismo incentivou o emprego de versos livres. b) O tema da poesia acima, subjetivo e lrico, apresenta pontos de contato com o Romantismo. c) Com o movimento Parnasiano, os versos tm em comum a rigidez formal e o distanciamento do poeta em relao ao assunto. d) A corrente espiritualista da 2 fase do Modernismo teve em Ceclia Meireles a primeira voz potica feminina a ser respeitada na literatura brasileira. e) A poesia de Cecilia Meireles valoriza sobretudo a intuio e a emoo como formas de entender o mundo. 2. Observando as formas pronominais, ainda no TEXTO I, assinale a alternativa incorreta: a) O destinatrio expresso a 2 pessoa do singular, enquanto que o autor representado pelo eu (oculto). b) Em encostei-me temos um exemplo de nclise. c) A forma ti (tnica) poderia ser substituda por te (tona), sem alterar o sentido e a forma. d) A formas tnicas ti so exigidas pela presena das proposies a e em. e) Minha e teu so possessivos ligados s 1 e 2 pessoas do singular, respectivamente. 3. Os versos do TEXTO I expressam: a) A tristeza resignada de algum que, apesar de conhecer a fragilidade de outrem, depositou nele a sua confiana. b) A felicidade de quem sente a realizao de seu amor. c) A dificuldade de chorar diante dos acontecimentos quando as causas so passageiras. d) A confiana depositada na pessoa amada, contra as evidncias, tendo por isso alcanado a felicidade desejada. e) O lamento do poeta pelo destino frgil da pessoa amada.

603

PORTUGUS
4. Assinale a alternativa em que todas as palavras so acentuadas. a) b) c) d) e) Salario, urgencia, cinico, sabado, prejuizo Impossivel, comercio, apos, gramatical, economia Inteligencia, proposito, tambem, viavel, rubrica Apoio, ceus, pagina, fiel, hifen Ideias, minimo, comicio, eletrica, itens

UFPE
F A S E

5. Pertenceu, de incio, ao Parnasianismo: Posteriormente o criticou, satirizando a arte potica tradicional num clebre poema. Trata-se de: a) Ceclia Meireles
No... Neste espao que ainda resta, ponha uma cadeira vazia.

b) Manuel Bandeira
Enfunando os papos, Saem da penumbra Aos pulos, os sapos.

c) Joo Cabral de Melo Neto


Na paisagem do rio difcil saber onde comea o rio;

d) Carlos Drummond de Andrade


Chega mais perto e contempla as palavras cada uma tem mil faces secretas.

e) Alphonsus de Guimares
Ai, pobre amante solitrio, Que vens fazer no meu solar?

6. Relacione as frases cujos verbos destacados esto no mesmo tempo, modo e pessoa gramatical. 1. Que todo homem um diabo no h mulher que o negue. 2. Vem, eu te farei da minha vida participar. 3. Ide em paz, o Senhor vos acompanhe. 4. Estou preso vida e olho meus companheiros. 5. Tu no me tiraste a natureza... Tu mudaste a natureza. A seqncia correta : a) 3, 2, 4, 5 e 1 b) 4, 3, 2, 1 e 5 c) 5, 1, 4, 2 e 3 d) 1, 4, 5, 3 e 2 e) 2, 3, 4, 5 e 1 ( ) Cala essa cano soturna. ( ) Interrogaia-as agora que os reis tremem no seu trono. ( ) Debruo-me na grade da banca e respiro penosamente. ( ) Trouxeste-a para o p de mim. ( ) Mesmo assim elas procuram um diabo que as carregue.

604

PORTUGUS
[TEXTO II]
Torce, aprimora, alteia, lima A frase; e, enfim, Noverso de ouro engasta a rima, Como um rubim.

UFPE
F A S E

7. Olavo Bilac, representante mais famoso da poesia parnasiana, resume, nos versos acima, o ideal literrio do movimento. De que maneira? a) b) c) d) e) Pela valorizao excessiva do eu e do intimismo. Pela busca do inexplicvel e pelo pessimismo. Pela nfase no culto da forma e do virtuosismo. Pela viso generalizadora da realidade. Pelo retorno ao bucolismo.

8. Assinale a alternativa em que todas as palavras devem ser completas com a letra indicada entre parnteses: a) ave al cara arope b) pr vilgio - requ sito ntitular c) ma - exce o - ro a () d) ibia unco ria eito e) Pure a - portugue a - cort - anali enofobia (x) mpedimento (i) ente (j) ar (z)

9. Em qual das frases seguintes a regncia no est de acordo com a norma culta gramatical? a) b) c) d) e) O Sport joga contra o Santa Cruz pelo ttulo de campeo pernambucano. No foi por acaso que o Presidente fez aluso ao que acontecera na noite anterior. Amava-lhe com um amor tal, que seria capaz de lhe dar a sua prpria vida. Fizeram as pazes e logo esqueceram as desavenas antigas. Tenho certeza de que ele, se for consultado, preferir pagar a multa a ficar devendo favores a quem quer que seja.

TEXTO III
Os antitabagistas americanos lutam h anos para que o cigarro seja considerado droga e passe a ser controlado pelo rgo que regulamenta drogas e alimentos nos Estados Unidos. A indstria faz a presso contra. (Veja, 28/08/96)

10. Em relao ao TEXTO III, incorreto afirmar: a) b) c) d) e) Em h anos, o verbo haver deve ficar no singular porque indica tempo decorrido. Em antitabagistas h um prefixo de origem grega com sentido de oposio. Em rgo que regulamenta drogas e alimentos o que se refere a rgo. A indstria faz presso contra estabelece, com o resto do texto, uma idia de conseqncia. Para que, alm de indicar finalidade, obriga o verbo ser a estar no modo subjuntivo.

605

PORTUGUS
11. Assinale a alternativa que melhor expressa a idia principal do TEXTO III:

UFPE
F A S E

a) Americanos no-fumantes procuram restringir o uso do cigarro, enquanto a indstria do fumo procura vencer a presso. b) Todo americano no-fumante luta contra a indstria de cigarros. c) A indstria pressionou o governo americano para incluir o cigarro na lei que regulamenta alimentos e drogas. d) Fumar em lugares pblicos proibido pelo Departamento de Estado dos Estados Unidos. e) O cigarro j considerado droga nos Estados Unidos e por isso sujeito lei antidroga. 12. Assinale a alternativa em que as partes destacadas desempenham a mesma funo. a) 1. proibido pisar na grama. 2. Precisa-se de novos professores. b) 1. importante pedir a nota fiscal. 2. Aqui se vende picol. c) 1. H algo de novo no ar. 2. Todos ho de saber da novidade. d) 1. Os assaltantes foram presos pela polcia. 2. Ele entrou, de cabea erguida, pela porta da frente. e) 1. Alugam-se casas. 2. Disseram que ele viria. 13. O Barroco o primeiro movimento literrio a ter repercursso no Brasil. Qual das caractersticas abaixo no lhe prpria? a) b) c) d) e) Uso de metforas. Tema da fugacidade da vida. Jogo de contrastes. Seqncia cumulativa concluda com um termo sntese. Descrio da vida campestre.

14. Identifique a alternativa em que o uso da crase no atende norma padro: a) b) c) d) e) Ela chegou s presses para resolver o problema. Ela decidiu que todos iriam a p, mas somente noite. Refiro-me quelas normas de trnsito que havamos estudado juntos. Confesso a voc que nem sempre obedeci s ordens de meu pai. Em discurso, o candidato fez referncia decises do seu partido.

606

PORTUGUS
15. Relacione a 2 coluna de acordo com a 1. Movimento Literrio 1. Arcadismo Caractersticas

UFPE
F A S E

( ) Individualismo e subjetivismo. Culto natureza, valorizao do passado. No Brasil, surgem o regionalismo e o indianismo como procura de expresso literria brasileira, identificada com a independncia poltica. ( ) Volta aos modelos clssicos, imitao da natureza, ausncia de subjetividade. No Brasil, coincide com a formao de um pblico leitor e a efervescncia do movimento de emancipao poltica em Minas Gerais. ( ) Objetividade, impessoalidade e verossimilhana, tom pessimista e busca da perfeio formal. No Brasil, acrescenta-se a crtica sociedade agrria, idias positivistas e republicanas coincidindo com o final do sculo XIX. ( ) Arte cultiva apenas na poesia, tinha como trao marcante o espiritualismo, a sugesto, a musicalidade, o irracionalismo e o mistrio. Com origem na Frana, desenvolveu-se no Brasil, nos estados unidos do Sul e em Minas Gerais. ( ) Pregava a arte pela arte, com o culto da forma, dentro de uma temtica preponderantemente grecoromana. No Brasil, representou o desligamento da realidade local, no que esta tinha de feio, sujo e pobre. Foi uma poesia de elite no se estendeu prosa prestigiada e admirada. d) 5, 4, 1, 2, 3 e) 2, 1, 4, 5, 3

2. Romantismo

3. Parnasianismo

4. Realismo

5. Simbolismo

A seqncia correta a) 2, 3, 4, 5, 1 b) 3, 4, 5, 1, 2 c) 2, 3, 5, 4, 1

607

PORTUGUS
16. H desvio de concordncia verbal em:

UFPE
F A S E

a) Das milhares de urnas eletrnicas espalhadas pelo Brasil, estima-se que 30% apresentou algum tipo de problema. b) necessrio que nas relaes de aprendizagem se promovam o esprito criativo, a capacidade de sntese e a anlise lgica. c) O jogo pode ser um bom negcio para o Brasil se a legalizao for precedida de um cuidadoso estudo do impacto econmico e social nas regies onde sero instalados os cassinos. d) H quem afirme que se o voto no fosse obrigatrio no Brasil, a maior parte dos eleitores no compareceria s urnas. e) Apesar de previso pessimista de que a safra de feijo seja a menor dos ltimos tempos, os entendidos insistem em que no deve haver motivos para preocupaes. 17. Nas frases publicitrias abaixo, assinale a alternativa em que a palavra destacada introduz uma relao de oposio: a) b) c) d) e) Margarina M. Para quem no esqueceu o paladar. O refrigerante diettico R no tem concorrente, mas voc tem. Use sandlias S enquanto voc combina com ela. Ande devagar, Chapeuzinho, com sapatos C, para o lobo mau poder te pegar. Use Leite de Aveia D, porque sua pele merece.

TEXTO IV
O videotexto, conjugao do telefone com a televiso ou microcomputador, leva at voc, a qualquer hora do dia ou da noite, um mundo de servios, de negcios e de entretenimento. (Lista Telefnica n 800)

18. Sobre o TEXTO IV podemos afirmar: 1. As palavras telefone, televiso e microcomputadores tm como elemento de composio, respectivamente, os radicais gregos tele, que significa longe; e micro, que significa pequeno. 2. Entretenimento um substantivo formado por sufixao a partir do verbo entreter. 3. A palavra videotexto um exemplo do processo de formao de palavras chamado composio hbrida (latim + portugus). Esto corretas: a) 2, 3 apenas b) 1, 2, 3 c) 1, 3 apenas d) 1, 2 apenas e) 2 apenas

608

PORTUGUS
19. Assinale a alternativa correta no que se refere ao uso dos pronomes: a) b) c) d) e) No acredito que entre mim e voc surjam problemas deste tipo. Espere-me, pois estarei consigo na prxima semana. No h qualquer afinidade entre eu e eles. Estas flores chegaram para tu. Pedi que deixasse o documento para mim assinar.

UFPE
F A S E

20. O conceito de ptria varia atravs dos tempos e das ideologias vigentes, com reflexos na arte e na literatura. Observe este fato nos textos de duas escolas literrias diferentes:
Minha terra tem palmeiras onde canta o sabi as aves que aqui gorjeiam no gorjeiam como l (G. Dias) Que importa que uns falem mole, descansado Que os cariocas arranhem os erres na garganta Que os capixabas e paroaras escancarem as vogais? Juntos formamos este assombro de misrias e grandezas Brasil, nome de vegetal! (M. de Andrade)

Sobre os textos acima, incorreto afirmar: a) No texto de Gonalves Dias, o conceito de ptria est expresso com saudosismo e idealismo, prprio da poesia romntica. Junta-se a isto um tom buclico e pessoal na descrio da passagem. b) Mrio de Andrade, modernista, busca a verdadeira alma nacional, incluindo em seu texto o regional e o folclrico. c) Sendo romntico, Gonalves Dias adota a postura da arte pela arte e distancia-se do escrito com frieza e objetividade, sem emitir juizos de valor. d) Mrio de Andrade exalta as diversidades de falares regionais, aceitando os defeitos e qualidades do brasileiro. e) Os autores, embora de formas diferentes, tm em comum uma viso positiva da ptria-me. 21. Sobre o texto de Mrio de Andrade, incorreto afirmar: a) A idia principal a unidade do Brasil dentro da diversidade de caractersticas e de problemas. b) Como idia secundria surge a caracterizao dos falares brasileiros. c) Falar mole e descansado significa falar de forma incoerente. d) Assombro (substantivo) homnimo da 1 pessoa do singular do presente do indicativo do verbo assombrar. e) Misrias e grandezas expressam uma anttese e formam um par de antnimos.

609

PORTUGUS

UFPE
F A S E

22. Assinale a alternativa na qual as partculas de relao completam adequadamente o seguinte perodo: ideais, a) b) c) d) e) . . todos pensem o contrrio, saiba que lutarei . . . . . . . . . . alcanar meus neles eu acredito.

Embora - porquanto - porque Se bem que - a fim de - portanto Ainda que - para - pois Porque - a fim de - pois Contudo - para - porquanto

TEXTO V
Recife... Rua da Unio... A casa do meu av... Nunca pensei que ela acabasse! Tudo l parecia impregnado de eternidade Recife... Meu av morto. Recife morto, Recife bom, Recife brasileiro como a casa de meu av (Manuel Bandeira)

23. Em relao ao TEXTO V, assinale a alternativa incorreta: a) Os versos referem-se infncia do autor, e so organizados a partir de lembranas pessoais, sem cronologia nem construes frasais rgidas. b) O texto reflete um dos traos marcantes da poesia de Bandeira: a simplicidade. c) A viso do autor profundamente humana e marcada pela conscincia da fragilidade da vida. d) O texto escrito na base da desconstruo, criando um ambiente catico e contestador. e) O lirismo intenso, apresentando uma viso pessoal do passado. 24. Ainda no TEXTO V, pode-se observar que a pontuao usada de forma original, cabendo ao leitor a tarefa delicada de interpretar a inteno do autor. Tendo isto em mente, incorreto afirmar: a) b) c) d) e) As reticncias indicam supresso de pensamento provocada pela impreciso das lembranas. O ponto-de-exclamao traduz a emotividade do autor diante do fato recordado. O ponto final marca a concluso do pensamento em uma frase nominal. As vrgulas separam termos que exercem a mesma funo sinttica em uma enumerao. O excesso de reticncias proporciona uma compreenso objetiva, exata e precisa dos fatos.

610

PORTUGUS
LNGUA PORTUGUESA
Excertos do Documento Pacto pela Educao
Marco Maciel

UFPE
F A S E

Um sistema poltico eficaz aquele capaz de fazer dos postulados democrticos o compromisso cotidiano da cidadania. No apenas por sua garantia formal, mas por seu exerccio efetivo. Uma sociedade democrtica, por sua vez, no se esgota na proteo jurdica dos direitos e garantias individuais. Ela se consuma na efetivao dos direitos econmicos e sociais, sem os quais teremos sempre e fatalmente uma sociedade dualista. Considerados sob esse aspecto, Poltica e Educao esto submetidas aos mesmos princpios e exigem respeito liberdade individual, acatamento diversidade humana e preservao do pluralismo. Parodiando o velho dilema para saber se os pases so ricos porque so educados, ou so educados porque so ricos, existe a convico de que um bom sistema poltico depende essencialmente de um sistema educacional universalizado, eficiente e dinmico. Hoje, sabemos que no so apenas o crescimento material, o desenvolvimento econmico o aprimoramento social e o desfrute dos bens culturais e espirituais que levam uma sociedade adequadamente educada e apta a transformar em benefcios coletivos as conquistas da cincia e do conhecimento. Mais do que isso, temos conscincia de que a prpria sobrevivncia humana est condicionada pela possibilidade de acesso a todas as formas de conhecimento produzidas pelo homem. A manuteno e a expanso do emprego, uma fonte permanente de preocupao em todos os pases do mundo, esto associadas possibilidade de adquirirmos e aprimorarmos o conhecimento e as tcnicas que vm revolucionando as formas tradicionais de produo industrial, de intensificao do comrcio, de criao intelectual e do prprio lazer. Sociedades prsperas , portanto, so, necessriamente, no apenas sociedades educadas, mas aquelas capazes de se educarem permanentemente. Nenhuma fragilidade, por isso mesmo, mais cruel, nenhuma gera mais excluso e injustia do que a incapacidade de dar a todos a possibilidade de realizao de suas prprias potencialidades, por meio do conhecimento, da educao e do acesso aos bens culturais. Este o desafio que, neste fim de milnio, ainda estamos por vencer.

O dualismo que nos separa sobrevive, porque no fomos capazes de vencer o nico problema estrutural brasileiro, que o da educao. No me refiro aos aspectos formais, que incluem a diminuio das taxas de evaso e repetncia e a ampliao dos benefcios proporcionados pela qualidade de ensino, mas sim a algo mais abrangente e substantivo, que a educao como instrumento vital de preparao para a vida.

No basta alfabetizar. Educar muito mais do que isso. , sobretudo, instrumentalizar o ser humano como cidado, proporcionando-lhe, por meio de sistema educacional universalizado, eficiente e de alto padro de qualidade e rendimento, perspectivas de progresso pessoal e de mobilidade social. [...] A questo educacional , efetivamente, o verdadeiro desafio estrutural que estamos sendo chamados a vencer neste fim de sculo. Certamente h muitas e profundas razes para o nosso atraso. Uma de carter histrico, cultural e sociolgico, de que exemplo a circunstncia de termos sido o ltimo pas a abolir a chaga terrvel da escravido [...]. Da escravido, decorrem, em grande parte, o dualismo e a excluso social de que hoje somos uma das principais vtimas em todo o mundo, em razo da expresso poltica, econmica e demogrfica que atingimos no conceito universal. Outras razes so incontestavelmente polticas, como o modelo elitista que timbramos em no sepultar e que hesitamos muitas vezes em simplesmente reformar. Dele decorrem os males atvicos do Estado brasileiro, barreira e proteo para os privilgios que beneficiam a poucos em detrimento de quase todos.

necessrio [...] um pacto de Estado para termos uma sociedade mais justa, uma economia mais prspera e um sistema poltico que reflita as permanentes aspiraes nacionais por democracia, desenvolvimento e solidariedade social.

611

PORTUGUS
1. O texto: 0-0) 1-1) 2-2) 3-3) 4-4) Narra um acontecimento que obedece a uma ordenao temporal. Tenta captar o esttico da realidade, descrevendo um momento particular. Apresenta vrios argumentos em favor da proposta do autor. Descreve uma realidade idealizada. Faz juzos de valor sobre o tema a fim de convencer o interlocutor.

UFPE
F A S E

2 B

2. De acordo com o texto: 0-0) Investir na educao uma forma de transformar os seres humanos em cidados. 1-1) A superao dos desafios que caracterizam uma sociedade dualista dar-se- pelo enfrentamento dos problemas sociais. 2-2) Alfabetizar um ponto de partida que proporcionar ao ser humano o acesso aos bens culturais. 3-3) A sociedade brasileira venceu o desafio da educao. 4-4) Vivemos numa sociedade em que os direitos dos cidados foram praticamente efetivados. 3. Segundo o texto, a eficcia de um sistema poltico dentro de uma sociedade democrtica: 0-0) 1-1) 2-2) 3-3) 4-4) esgota-se na proteo jurdica dos indivduos. inclui apenas direitos econmicos e sociais. cria uma sociedade dualista. consolida-se com a universalizao do sistema educacional. exige respeito liberdade individual, aceitao da diversidade e do pluralismo.

P R O V A

4. Sobre a questo educacional: 0-0) 1-1) 2-2) 3-3) 4-4) O processo educativo compreendido unicamente nos seus aspectos formais. A alfabetizao em massa seria suficiente para vencer o desafio da educao. de ordem conjuntural. de ordem estrutural, sendo uma das razes para o nosso atraso. O sistema educativo adotado no pas precrio e ineficiente por razes sociais de carter histrico e de carter poltico.

612

PORTUGUS
5. De acordo com o texto:

UFPE
F A S E

0-0) Crescimento material e desenvolvimento econmico so suficientes para a estabilizao social. 1-1) Aprimoramento social e desfrute dos bens culturais e materiais so conquistas das sociedades desenvolvidas. 2-2) Crescimento material, desenvolvimento econmico, aprimoramento social, desfrute dos bens materiais e espirituais em uma sociedade esto interligados possibilidade de acesso a todas as formas de saber. 3-3) A sobrevivncia humana depende do conhecimento produzido pelo homem e compartilhado pelos seus semelhantes. 4-4) O desenvolvimento econmico por si s no suficiente para democratizar as conquistas da cincia e da tecnologia. 6. De acordo com o quarto pargrafo, a educao em uma sociedade prspera deve ser um processo: 0-0) estanque 1-1) contnuo 2-2) intermitente 3-3) permanente 4-4) efmero

2 B

P R O V A

7. Reforam os pontos de vista do texto os seguintes fragmentos: 0-0) [Em relao educao], ns temos uma questo, no Brasil, que estrutural, de desigualdade profunda, que vem de dcadas, de sculos, e cuja soluo depender de processo de transformao profunda...
(F.S.P. - 02/12/96)

1-1) Um breve vale a pena ver de novo nos leva ao incio deste ano, quando o grupo dos sete pases mais ricos do mundo, o G-7, encerrava sua conferncia em Lille, na Frana, sem uma soluo para combater o desemprego [...].
(JC - 03/12/96)

2-2) Um pas se faz a partir da erradicao do analfabetismo e de um sistema educacional dinmico, mas somente o empenho governamental no o bastante.
(Jornal do MEC - set/96)

3-3) Um ensino de qualidade eleva um pas condio de desenvolvido e coloca seu povo altura de enfrentar os grandes desafios do prximo sculo.
(Jornal do MEC - out/96)

4-4) O MEC analisou as instalaes de 2.800 escolas pblicas e privadas (...) e testou o desempenho de 90.000 alunos numa prova simulada. A pesquisa derruba alguns mitos, reafirma valores e deixa uma grande lio. Um mito derrubado: a escola privada no melhor do que a pblica. Uma suspeita confirmada: a educao dos filhos reflexo da formao dos pais.
(Revista Veja - 04/12/96)

613

PORTUGUS
8. Concluindo, o autor:

UFPE
F A S E

0-0) Sugere um pacto pela educao a fim de que a sociedade brasileira se torne mais justa. 1-1) Enfatiza que a eficincia e a qualidade do ensino determinam as perspectivas de progresso social. 2-2) Defende a universalizao da educao. 3-3) Adverte que a desigualdade e a excluso social no sero evitadas nem diminudas com o funcionamento de um sistema justo. 4-4) Ratifica a dependncia direta entre oportunidades iguais e justia social. 9. , sobretudo, instrumentalizar o ser humano como cidado, proporcionando-lhe [...] perspectivas de progresso pessoal e mobilidade social. Tomando como modelo a frase acima, o termo destacado pode ser substitudo por lhe(s) em: 0-0) A manuteno do emprego associa-se possibilidade de aquisio de conhecimentos. 1-1) No me refiro aos aspectos formais. 2-2) O educador recomendou aos leitores que assumissem um pacto pela educao. 3-3) O futuro dir aos cidados se o texto tinha fundamento. 4-4) A abolio da escravatura prometeu aos escravos liberdade e cidadania. 10. Quanto formao de palavras: 0-0) Preconceito formao prefixal. 1-1) Pluralismo e fragilidade so formaes sufixais. 2-2) Incontroverso, individual e interna so formadas com prefixo latino in, com sentido de negao. 3-3) Ampliao, repetncia, preparao e cidadania so substantivos formados a partir de formas verbais. 4-4) Em fragilizar, modernizar e democratizar o sufixo izar forma verbos a partir de adjetivos. 11. Quanto ao vocabulrio: 0-0) No texto, eficaz (linha 1) pode ser substitudo por eficiente. 1-1) Compromisso cotidiano significa obrigao habitual permanente. 2-2) Parodiar um verbo formado a partir de pardia. Significa imitar e em geral esta imitao tem um componente cmico. 3-3) So pares de antnimos: Elitista/populista; harmonia/atraso; ineficincia/obstinao. 4-4) Os termos destacados esto usados no sentido conotativo: Profundas razes; Desagregao econmica; Luta contra o analfabetismo; Sintonia com as aspiraes nacionais.

2 B

P R O V A

614

PORTUGUS

UFPE
F A S E

12. A forma reduzida de infinitivo pode ser substituda por uma orao desenvolvida (iniciada por que) nas seguintes frases: 0-0) 1-1) 2-2) 3-3) 4-4) Ser necessrio investirmos mais em cincia e tecnologia. Sociedades prsperas so aquelas capazes de se educarem permanentemente. Educar muito mais que isso. No basta alfabetizar. Parodiando o velho dilema para saber se os pases so ricos [...].

2 B

13. So exemplos de voz passiva: 0-0) 1-1) 2-2) 3-3) 4-4) A sobrevivncia humana condicionada pela possibilidade de acesso ao conhecimento. O modelo elitista no foi sepultado pelas reformas constitucionais. No me refiro aos seus aspectos formais [...] O dualismo e a excluso social decorrem da escravido. A manuteno e a expanso do emprego parecem estar diretamente ligadas possibilidade de adquirir conhecimentos.

P R O V A

14. Quanto regncia, esto corretas: 0-0) Em relao a manuteno e a expanso do emprego, as solues so viveis. 1-1) Obedecendo os padres de tica, os bens culturais devero ser para todos. 2-2) A educao visa o acesso de todas as formas de conhecimento. 3-3) O autor referiu-se aos erros do passado como sendo apenas de carter histrico. 4-4) Um sistema poltico eficaz deve preferir seguir postulados democrticos do que garantir o exerccio efetivo da cidadania. 15. A relao entre o conectivo utilizado e a circunstncia por ele indicada est correta em: 0-0) Na sociedade brasileira, o dualismo sobrevive porque no fomos capazes de vencer o desafio da educao. (conseqncia) 1-1) Sociedades prsperas, portanto, so sociedades educadas. (concluso) 2-2) Outras razes so polticas, como o modelo elitista que timbramos em no sepultar. (causa) 3-3) No me refiro aos seus aspectos formais, [...] mas sim a algo mais abrangente e substantivo. (oposio) 4-4) Nenhuma fragilidade, por isso mesmo, mais cruel [...] (condio) 16. Nenhuma fragilidade mais cruel [...] As frases apresentam estruturas comparativas em: 0-0) [...] outras razes so incontestavelmente polticas. 1-1) Filhos de pais educados recebem educao esmerada. 2-2) H um motivo superior aos demais. 3-3) O episdio da Abolio anterior ao da Repblica. 4-4) O modelo elitista to cruel quanto o imperialista.

615

PORTUGUS
LITERATURA

UFPE
F A S E

1. Recife/Pernambuco foi musa de muitos poetas, atravs dos tempos e das escolas literrias. H relao correta entre o texto e os comentrios em: 0-0) Por entre o Beberibe e o Oceano Em uma areia sfia e movedia Jaz o Recife, populao mestia Que o belga edificou, mpio tirano. Os versos no louvam a cidade, antes criticam a populao, o meio ambiente e at sua fundao. A forma ferina de escrever prpria de Gregrio de Matos, o Boca do Inferno. 1-1) Quero, na hora da morte, estar lcido para mandar a ti meu ltimo suspiro, Recife. O lirismo dos versos oscila entre a confisso pessoal e a vida cotidiana. Como recifense, Manuel Bandeira coloca uma forte dose de emoo, aliada a referncias bibliogrficas. 2-2) (Recife)
Retilnea Cana de acar Dobrada Para deixar mais alta Olinda Plantada sobre uma onda linda Do mar pernambucano.

2 B

P R O V A

Os versos livres e vanguardistas, pertencentes ao Modernismo, escapam do tom irnico e da forma destruidora, caracterstica do estilo de Oswald de Andrade. Paulista, ele descreve, em ritmo sincopado, a situao topogrfica do Recife. 3-3) Salve, terra formosa, Pernambuco!
Veneza Americana transportada, Boiando sobre as guas! Amigo gnio te formou na Europa, Gnio melhor te despertou sorrindo, sombra dos coqueiros.

Os versos laudatrios, cheios de exclamaes e vocativos, so prprios da linguagem romntica. Gonalves Dias, maranhense, romntico da 1 fase, cantou o Recife com admirao, falando de sua paisagem: uma natureza tropical e exuberante. 4-4) Pernambuco! Um dia, eu vi-te
Dormindo imenso ao luar Com os olhos quase cerrados, Com os lbios quase a falar Do brao, o clarim suspenso O punho no sabre extenso De, recife imenso Que rasga o peito do mar.

Na estrofe acima, podemos perceber o toque condoreiro e a adoo das causas libertrias. Castro Alves, romntico da 3 fase, que seguiu em Recife os cursos jurdicos, apresenta um Pernambuco guerreiro: usando clarim e sabre.

616

PORTUGUS
2. Largas dentaduras,
Vosso riso largo me consolar. No sei quantas fomes ferozes, secretas no fundo de mim. Dentaduras alvas admirveis presas, mastigando lestas e indiferentes a carne da Vida!

UFPE
F A S E

2 B

P R O V A

Os versos, de Carlos Drummond de Andrade, apresentam como caractersticas: 0-0) 1-1) 2-2) 3-3) 4-4) Poesia voltada para o cotidiano. Poesia centrada nos aspectos antilricos. Poesia direcionada para os problemas sociais. Ausncia de complacncias sentimentais. Fico potica de um mundo imaterial.

3. Machado de Assis, com Memrias Pstumas de Brs Cubas, torna-se o divisor de guas entre o Romantismo e o Realismo. A partir de ento, a ironia e o pessimismo tornam-se presentes na sua obra, dissecando o carter dos personagens. Estas duas caractersticas esto presentes em: 0-0) Marcela amou-me durante 15 dias e onze contos de ris. 1-1) No tive filhos, no transmiti a nenhuma criatura o legado da nossa misria. 2-2) Tinha 64 anos rijos e prsperos, era solteiro, possuia 300 contos e fui acompanhado ao cemitrio por onze amigos. Onze amigos! 3-3) Capitu deixou-se fitar e examinar. S me perguntava o que era, se nunca os vira. [seus olhos]: eu nada conclu de extraordinrio. A cor e a doura eram minhas conhecidas. 4-4) Ontem, logo que tive notcia da crise ministerial, recolhi-me a casa, para esperar os acontecimentos.

617

PORTUGUS

UFPE
F A S E

4. Alm, muito alm daquela serra que ainda azula no horizonte, nasceu Iracema. Iracema, a virgem dos lbios de mel, que tinha os cabelos mais negros que a asa da grana e mais longos que seu talhe de palmeira. O favo de jati era doce como seu sorriso. Jos de Alencar, autor do texto acima, o mais conhecido dos autores da prosa romntica. Sobre o texto, podemos afirmar: 0-0) Apresenta um perfil feminino com construo psicolgica elaborada a partir de cuidadosa observao da realidade. 1-1) O elemento humano representado de acordo com o capricho de sua fantasia, correspondendo a uma idealizao. 2-2) As linhas introdutrias confirmam a tendncia do autor do descritivismo com paisagens de um colorido intenso e traos de grandiosidade. 3-3) Uma lngua prpria brasileira representada aqui pelo uso de vocbulos indgenas. 4-4) A descrio da figura indgena representa uma aceitao de idias colonialistas em oposio a intuitos nativistas. 5. O Modernismo no Brasil subverteu as normas literrias e durou vrias dcadas. Este movimento: 0-0) Foi iniciado com a Semana de Arte Moderna em 1922, englobando vrias artes: literatura, msica, pintura e escultura. O plo principal desta fase foi So Paulo, na poca j um florescente parque industrial. 1-1) Embora adotando inovaes, prendeu-se concepo tradicional de literatura, esquecendo a histria da atualidade e fixando-se em valores do passado. 2-2) Em sua primeira fase, foi marcado pela desintegrao da linguagem tradicional com a busca da expresso nacional e a adoo das conquistas de vanguarda. 3-3) Ainda na 1 fase, teve em Oswald de Andrade, escritor e poeta paulista, um dos autores mais marcantes. Seu texto foi dos mais inovadores e corrosivos da esttica modernista. 4-4) Na segunda fase do movimento, surgiram grandes poetas, mas destaca-se especialmente o chamado. Romance de 30 ou Romance Regional.

2 B

P R O V A

618

PORTUGUS
6. Sobre Joo Cabral de Melo Neto, poeta da gerao de 45:

UFPE
F A S E

0-0) Em seu percurso potico manteve como marca registrada a conteno e a objetividade: No a forma encontrada como a concha perdida [...] Mas a forma atingida como uma ponta do novelo que a ateno, lenta, desenrola. 1-1) Sua obra mais conhecida Morte e Vida Severina ou Auto de Natal Pernambucano, que fala do imigrante nordestino: Somos muitos Severinos / iguais em tudo na vida / na mesma cabea grande / que a custo se equilibra. 2-2) Afastando-se do lirismo intimista, sua poesia baseia-se no cotidiano, na realidade, nos objetos: O lpis, o esquadro, o papel; / o desenho, o projeto, o nmero; / o engenheiro pensa o mundo justo / mundo que nenhum vu encobre. 3-3) Os versos acima ilustram a afirmao de que a poesia de Joo Cabral revela ao mesmo tempo rigor tcnico e preocupao social. 4-4) Cabral foi herdeiro da preocupao formal do Parnasianismo e de seus mtodos, com modelos pr-estabelecidos e formas a serem seguidas, rima e mtrica de padres clssicos como na estrofe abaixo:
E se encorpando em tela, entre todos Se erguendo tenda, onde entrem todos, Se entretendo para todos, no toldo (a manh) que plana livre da armao A manh, toldo de um tecido to areo Que, tecido, se eleva por si: luz balo.

2 B

P R O V A

7. Sobre o Parnasianismo: 0-0) Foi um movimento essencialmente potico que reagiu contra os exageros romnticos, no fim do sculo XIX. 1-1) Seu ideal era a arte pela arte, isto , artes sem finalidades prticas, o que culminou com o endeusamento dos processos formais. 2-2) A preocupao excessiva com a forma valeu fortes crticas, como a que se l abaixo, de Olavo Bilac: Invejo o ourives quando escrevo: Imito o amor Com que ele, em ouro, o alto relevo Faz de uma flor. 3-3) Para desvincular-se do mundo concreto, os poetas parnasianos escolheram como tema a Antigidade clssica, sua histria e sua mitologia, evitando as confisses e os extravasamentos subjetivos. 4-4) A preocupao com os problemas sociais e a luta pela consolidao das idias da independncia representaram um compromisso com a contemporaneidade, o que demonstrado em O Caador de Esmeraldas: Ferno Dias Paes Leme agoniza. Um lamento Chora longo a rolar na longa voz do vento.

619

PORTUGUS
8. Sobre o Romance Brasileiro:

UFPE
F A S E

0-0) Memrias Sentimentais de Joo Miramar considerada a primeira grande realizao da prosa modernista. De autoria de Oswald de Andrade, rompe com esquemas tradicionais da narrativa e construda a partir de fragmentos justapostos. 1-1) Jos Lins do Rego, com Menino do Engenho, Doidinho, Fogo Morto, Bangu, Usina, relata a decadncia da sociedade patriarcal nordestina no incio do sculo XX. Seus cinco primeiros livros constituem o ciclo da cana-de-acar e so pedaos da vida que ele queria contar, como menino criado em casa grande de engenho. Faz parte do chamado Romance de 30. 2-2) Ceclia Meireles representou no Romance Brasileiro a tica e o discurso femininos, o questionamento existencial e o medo e a angstia contemporneos. Em seu romance Uma Aprendizagem ou Livro dos Prazeres retrata a importncia da linguagem na percepo e na sensao sobrepondo-se ao mundo das aes. Pertenceu fase do Romance Ps-guerra. 3-3) Carlos Drummond de Andrade escreveu uma obra profundamente impregnada da realidade atual e humana, na qual esto presentes romances, contos, crnicas e poesias, em texto frio e despojado. Seu romance mais conhecido Sentimento do Mundo. Pertenceu primeira Fase do Movimento Modernista. 4-4) Alusio de Azevedo imps-se como iniciador do Romance Realista no Brasil, com O Mulato investida dramalhnica contra o escravismo. Em Casa de Penso os valores naturalistas j esto presentes, mas o romance em que o autor se realiza de maneira mais acabada O Cortio, cujo enredo se desenvolve em ambiente carioca.

2 B

P R O V A

620

PORTUGUS
9. Leia o texto a seguir:
Copo Vazio

UFPE
F A S E

sempre bom lembrar Que um copo vazio est cheio de ar sempre bom lembrar Que o ar sombrio de um rosto Est cheio de um ar vazio Vazio daquilo que no ar do copo Ocupa um lugar sempre bom lembrar, guardar de cor Que o ar vazio de um rosto sombrio Est cheio de dor sempre bom lembrar Que um copo vazio est cheio de ar

2 B

P R O V A

Os versos acima, de Gilberto Gil, apresentam caractersticas de diversas escolas literrias, no que se refere poesia: 0-0) Simbolismo: O poema parte de uma realidade material, tomando-a como smbolo, com uma certa ilogicidade na busca do eu profundo, do inconsciente. Como nos poemas simbolistas (msica, antes de tudo) a musicalidade dos versos acentuada pelas rimas internas (eco). 1-1) Barroco: A linguagem trabalha a oposio, na anttese cheio/vazio, relacionando-a, num elaborado jogo de imagens, com os sentimentos humanos. 2-2) Modernismo: Apesar do jogo de antteses, a linguagem despojada, estabelecendo a incorporao do cotidiano como tema potico. 3-3) Parnasianismo: A mtrica perfeita, nos padres tradicionais e clssicos, permite a captao da realidade de maneira neutra e objetiva. 4-4) Romantismo: A exacerbao sentimental centrada no subjetivismo e no individualismo. As emoes hipertrofiam-se, cantando o amor idealizado e platnico. A fantasia substitui a realidade.

621

PORTUGUS
10. Sobre Graciliano Ramos:

UFPE
F A S E

0-0) Metido no poro do navio, misturado aos criminosos na Correo, conduzido ao presdio da Ilha, tendo a cabea raspada, tornado farrapo humano pela misria fsica, assistindo aos espetculos mais srdidos, Graciliano Ramos retrata com impressionante nitidez, no apenas os quadros a que assistiu, mas, atravs destes quadros, uma fase da existncia do nosso pas. O livro que resultou da experincia de Graciliano descrita por Nelson Sodr foi Memrias do Crcere. 1-1) Este livro um impressionante texto autobiogrfico com as condies dramticas de sua priso durante o Estado Novo, no primeiro governo Vargas. 2-2) Angstia a continuao de Memrias do Crcere, e relata sua sada da priso, tendo sido escrito num tom de libelo contra a ditadura. 3-3) O procedimento constante de Graciliano em seu texto interrelacionar sociedade e psicologia de forma to contnua que difcil apontar em toda a sua obra situaes puramente psicolgicas com cenas meramente sociais. Realiza uma sntese sem similar na Literatura Brasileira. 4-4) As frases finais de Vidas Secas, obra prima de Graciliano, reafirmam sua grandeza que reside em dois elementos: Esmerada construo formal e viso realista do universo sertanejo. Chegariam a uma terra desconhecida e civilizada, ficariam presos nela. E o serto continuaria a mandar gente para l. O serto mandaria para a cidade homens fortes, brutos como Fabiano, Sinh Vitria e os dois meninos. 11. Sobre o Realismo / Naturalismo: 0-0) No Brasil, o Realismo representou uma reao ao Arcadismo, e foi introduzido numa poca de mudanas sociais logo aps a Independncia. 1-1) Tendo como pano de fundo a sociedade colonial, os autores realistas expressam com simplicidade a vida do homem no campo, ao lado da descrio da natureza exuberante que o envolve. 2-2) A anlise psicolgica e a tipificao social, presentes no Realismo, acentuam-se no Naturalismo, com a descrio de personagens patolgicos e com a crtica social explcita. 3-3) O Naturalismo um prolongamento do Realismo, acrescentando uma viso cientfica e determinista da existncia. Surge uma preocupao de descrio minuciosa, comprovada no texto abaixo. 4-4) Naquela terra encharcada e fumegante, naquela umidade quente e lodosa, comeou a minhocar, a esfervilhar, a crescer, um mundo, uma coisa viva, uma gerao que parecia brotar espontnea, ali mesmo, naquele lameiro, a multiplicar-se como larvas no esterco. O fragmento dO Cortio, de Aluzio de Azevedo, apresenta a caracterstica mais marcante do Naturalismo: compreenso biolgica do mundo.

2 B

P R O V A

622

PORTUGUS
12. bom, minha Marlia, bom ser dono
De um rebanho, que cubra monte e prado; Porm, gentil Pastora, o teu agrado Vale mais qum rebanho, e mais qum trono Graas, Marlia bela, Graas minha Estrela.
Tomas A. Gonzaga

UFPE
F A S E

2 B

No poema acima, as marcas do Arcadismo so: 0-0) Tema da vida pastoril no contato com a natureza idealizada. 1-1) Ideal de vida simples retratado atravs de uma linguagem despojada. Inverses e paradoxos, prprios do Barroco, caem em desuso. 2-2) Percepo do mundo terreno como efmero e vo, o que resulta no sentimento de nulidade diante do poder divino. 3-3) Descrio de um episdio pastoril num ambiente de harmonia, onde inexistem conflitos. 4-4) Tom pico, relatando grandes feitos hericos. 13. Tereza, se algum sujeito bancar
o sentimental em cima de voc e te jurar uma paixo do tamanho de um bonde Se ele chorar Se ele ajoelhar Se ele se rasgar todo No acredite no Tereza lgrima de cinema tapeao Mentira Cai fora
Manuel Bandeira

P R O V A

Numa noite, eu me lembro... Ela dormia Numa rede encostada molemente... Quase aberto o roupo... solto o cabelo E o p descalso do tapete rente Stava aberta a janela. Um cheiro agreste Exalavam as silvas da campina. E ao longe, num pedao de horizonte Via-se a noite plcida e divina Castro Alves

623

PORTUGUS
Em relao ao conceito de literatura, lendo os dois poemas podemos observar que:

UFPE
F A S E

0-0) A literatura a arte que imita a realidade pela palavra (Aristteles) ou a linguagem carregada de significado (Elza Pound). 1-1) O conceito de literatura mvel, fluido e inconstante porque a literatura e a realidade que lhe d forma muda com o tempo. 2-2) Existe uma essncia imutvel na literatura: os padres estticos permanecem atravs dos tempos. 3-3) A literatura utiliza um tipo especial de linguagem cotidiana. 4-4) O fato literrio, considerado arte, s pode estar associado ao belo, o que na poesia significa o uso excessivo de imagens, comparaes e metforas em mtrica e rima perfeitas. 14. Ainda em relao aos poemas da questo 13: 0-0) Do lirismo contraditrio de Castro Alves faz parte a sexualizao da natureza com a infiltrao do ertico, enquanto que Manuel Bandeira busca equilbrio entre a melancolia e o sentimentalismo com o desencanto e a amargura na viso de mundo. 1-1) H na lrica de Bandeira um tom coloquial e um senso de humor na abordagem dos sentimentos, tambm presentes nos versos de Castro Alves. 2-2) Poeta modernista, Bandeira revive pela lrica um peculiar estado de esprito prprio da poesia romntica brasileira, cantando a mulher como musa distante e inacessvel. 3-3) Com linguagens diferentes, os dois poetas tm em comum, nos poemas lidos, a abordagem do tema: a mulher na vivncia do cotidiano, como figura real. 4-4) A poesia lrica de Castro Alves essencialmente amorosa, quebrando a idealizao e o platonismo, herana clssica que o precedeu. uma lrica sensual.

2 B

P R O V A

624

PORTUGUS
15. Senhoras:

UFPE
F A S E

No pouco vos surpreender, por certo, o endereo e a literatura desta missiva. [...] bem verdade que na boa cidade de So Paulo a maior do universo, no dizer de seus prolixos habitantes no sois conhecidas por icamiabas, voz espria, sino que pelo apelativo de Amazonas; e de vs, se afirma, cavalgardes ginetes belgeros e virdes da Hlade clssica; e assim sois chamadas. [...]
Mrio de Andrade, Macunama: o heri sem nenhum carter.

2 B

O texto acima parte da obra mais conhecida de Mrio de Andrade. Sobre este autor e sua obra: 0-0) Em Paulicia Desvairada seus versos so livres e de mtrica informal, e focalizam aspectos humanos, sociais e polticos de So Paulo, subvertendo os valores estticos vigentes. Critica a burguesia paulista. 1-1) Em Macunama utilizada uma linguagem popular e espontnea, exceto na Carta a Icamiabas, na qual o autor, intencionalmente, utiliza um registro rebuscado. 2-2) Na tentativa de refletir, em sua obra, a alma nacional, o autor seleciona temas e cenrios que caracterizam o Brasil, mas d enfse cidade do Rio de Janeiro. 3-3) Em H uma Gota de Sangue em Cada Poema alcana o pice de sua fora potica, chegando perfeio formal. 4-4) Em 1928 publica Macunama: o heri sem nenhum carter, livro que chamou de rapsdia, considerando o fato de ter feito uma composio com fragmentos de assuntos variados e heterogneos: lendas, mitos, anedotas e elementos do folclore nacional. 16. Sobre o Romantismo: 0-0) Movimento literrio introduzido no Brasil no incio do sculo XIX, a primeira manifestao literria que de fato pode ser nomeada brasileira. Tem razes literrias mas tambm no-literrias: a asceno da burguesia, a partir da Revoluo Francesa e a implantao da sociedade capitalista com o incio da formao de um pblico leitor. 1-1) Tem como caractersticas literrias o subjetivismo e o individualismo, resultantes de uma viso de mundo centrada na individualidade. A libertao de valores preestabelecidos favorece o sentimentalismo e o amor natureza. 2-2) Do ponto de vista da escolha do tema e do uso da linguagem, o esprito de brasilidade se afirma com Jos de Alencar na prosa e Gonalves Dias na poesia. 3-3) O Romantismo Brasileiro no sofreu nenhuma influncia do Romantismo Europeu, resultando dessa absoluta originalidade o indianismo, adaptado ao ufanismo. 4-4) O movimento, no Brasil, dividido em trs momentos ou em trs geraes, que podem ser representadas respectivamente por Gonalves Dias, lvares de Azevedo e Castro Alves. Os autores continuam obedecendo s regras clssicas de definio rgida de gneros literrios em linguagem cientfica, impessoal, fria e objetiva. P R O V A

625

PORTUGUS
LNGUA PORTUGUESA Instruo: As questes de nmeros 1 a 10 referem-se ao texto abaixo:

UFRGS

(Adaptado de FLANDRIN, Jean-Louis. A distino pelo gosto. In: CHARTIER, Roger (Org.) Histria da vida privada 3: Da Renascena ao Sculo das Luzes. So Paulo: Companhia das Letras, 1991. p. 267-8)

1. 2. 3. 4. 5. 6. 7. 8. 9. 10. 11. 12. 13. 14. 15. 16. 17. 18. 19. 20. 21. 22. 23. 24. 25. 26. 27. 28. 29. 30. 31. 32. 33. 34. 35. 36. 37. 38. 39. 40. 41. 42.

Quando tratavam de maneiras mesa, os manuais de civilidade medievais ou talvez devamos dizer manuais de cor tesia, tendo em vista a poca condenavam as manifestaes de gula, a agitao, a sujeira, a falta de considerao pelos outros convivas. Tudo isso persiste nos sculos XVII e XVIII, porm novas prescries se acrescentam antigas. Em idia de geral, elas desenvolvem limpeza j presente na Idade Mdia , ordenando que se usem os novos utenslios de mesa: pratos, copos, facas, colheres e garfos individuais. O emprego dos dedos cada vez mais proscrito, bem como a transferncia dos alimentos diretamente da travessa comum para a boca. Isso evidencia no s uma obsesso pela limpeza, como ainda um progresso do individualismo: o prato, o copo, a faca, a colher e o garfo individuais na verdade erguem paredes invisveis entre os comensais. Na Idade Mdia, levava-se a mo ao prato comum, duas ou trs pessoas tomavam a sopa numa s escudela, todos comiam a carne na mesma travessa e bebiam de uma nica taa que circulava pela mesa; facas e colheres, ainda inadequadas, passavam de um conviva a outro; e cada qual mergulhava seu pedao de po ou de carne em saleiros e molheiras comuns. Nos sculos XVII e XVIII, ao contrrio, cada comensal dono de um prato, um copo, uma faca, uma colher, um garfo, um guardanapo e um pedao de po. Tudo que retirado das travessas, molheiras e saleiros comuns deve ser pego com os utenslios adequados e depositado no prato antes de ser tocado com os prprios talheres e levado boca. Cada conviva encerrado numa espcie de gaiola imaterial. Por que tais precaues, dois sculos antes de Pasteur descobrir a existncia dos micrbios? O que vem a ser essa sujeira que tanto se teme? No ser principalmente o medo do contato com o outro?

626

PORTUGUS

UFRGS

1. Assinale a alternativa que preenche corretamente as lacunas do texto na ordem em que aparecem. a) - s - - a b) a - a - - a c) - as - a - 2. De acordo com o texto, a) os manuais de cortesia da Idade Mdia toleravam a sujeira e a falta de educao que eram constantes nas refeies feitas em sociedade. b) nos sculos XVII e XVIII era considerado educado retirar a comida com os dedos diretamente das travessas comuns, desde que nos pratos individuais se utilizassem os utenslios adequados. c) o que parece ser uma bvia diferena de qualidade entre a etiqueta da Idade Mdia e dos sculos XVII-XVIII na verdade se trata de uma diferena cultural devida ao avano da idia de individualismo. d) as refeies comunais da Idade Mdia eram mais divertidas e animadas que os banquetes dos sculos XVII e XVIII, nos quais uma atmosfera de medo era instaurada pela sujeira constante e por uma rgida etiqueta. e) a etiqueta dos sculos XVII e XVIII determinava que as mesas fossem divididas em nichos individuais que isolassem os convivas de qualquer contato entre si. 3. Assinale a alternativa que identifica corretamente os ncleos dos sujeitos de, respectivamente, condenavam (li. 4), se acrescentam (li. 8), se usem (li. 11). a) b) c) d) e) maneiras (li. 1) - isso (li. 6) - utenslios (li. 11) maneiras (li. 1) - prescries (li. 8) - novos (li. 11) manuais (li. 2) - antigas (li. 8) - utenslios (li. 11) manuais (li. 2) - isso (li. 6) - novos (li. 11) manuais (li. 2) - prescries (li. 8) utenslios (li. 11) d) - s - a - e) a - as - -

4. Assinale a alternativa em que foi corretamente utilizada a palavra da mesma famlia ou de prescries (li. 8), ou de proscrito (li. 14). a) Repousei, tomei os remdios que o mdico proscreveu, e logo minha obedincia foi recompensada: recebi alta e pude voltar ao futebolzinho no quintal. b) A lei prescreve a pena de morte para o delito de que eu o incriminei; espero que seja cumprida, pois assim me apossarei de toda a sua fortuna. c) A volta do prescrito agitou a pequena cidadezinha do velho oeste; quereria ele vingar-se dos seus inimigos? d) A proscrio do padre confessor me apavorou: eu teria de rezar mil pais-nossos, mil avemarias e quinhentos credos, alm de acender uma vela para cada vez que estivera com a vizinha. e) O casamento no est de todo prescrito de nossa tribo, mas inegvel que ele contraria o que ordena nosso guru: Amem fisicamente a todos que puderem.

627

PORTUGUS

UFRGS

5. Supondo que a palavra escudela (li. 23) seja desconhecida para o leitor, so propostas as estratgias seguintes para determinar seu significado. I - Observar que o trecho do texto em que est inserida trata do uso de utenslios de mesa, e que a palavra escudela deve pertencer ao mesmo campo semntico. II - Pela anlise morfolgica da palavra, determinar que ela formada a partir de escudo e portanto remete guerra e s disputas da Idade Mdia. III - Pela anlise sinttica, determinar que, dada a funo que exerce na frase, a palavra no necessita concordar em nmero com o sujeito. Quais so adequadas para a finalidade acima expressa? a) Apenas I b) Apenas III c) Apenas I e II d) Apenas II e III e) I, II e III

6. As palavras molheira (linhas 29 e 33), saleiro (linhas 29 e 33) e sujeira (linhas 5 e 41) so formadas pela adio de um mesmo sufixo ao radical. Assinale a alternativa que NO apresenta o mesmo sufixo. a) roupeiro b) queira c) mosquiteiro d) fofoqueira e) lixeira

7. Considere as afirmativas seguintes sobre a pontuao do texto. I - As aspas da linha 3 indicam que se trata de palavras literais de outro autor, como se pode comprovar pela presena do verbo dizer introduzindo a citao. II - Os travesses da linha 10 poderiam ser substitudos por parnteses, sem que fosse alterado o sentido original da frase. III - No contexto, o trecho No ser (li. 41) poderia ser substitudo por , e a frase terminada com ponto final, uma vez que a ltima pergunta do texto retrica e disfara uma afirmao. Quais so corretas? a) Apenas I b) Apenas III c) Apenas I e II d) Apenas II e III e) I, II e III

628

PORTUGUS

UFRGS

8. Levando em conta as regras de pontuao do portugus culto, considere as possibilidades seguintes de substituio do dois-pontos da linha 19. I - Ponto, seguido de maiscula. II - Ponto-e-vrgula, seguido de minscula. III - Vrgula, seguida de minscula. Quais so corretas? a) Apenas I b) Apenas III c) Apenas I e II d) Apenas II e III e) I, II e III

9. Considere as possibilidades seguintes de reescrita da frase que inicia na linha 13. I - No se aceita o emprego nem dos dedos, nem a transferncia dos alimentos diretamente da travessa comum para a boca. II - No se aceita nem o emprego dos dedos, nem a transferncia dos alimentos diretamente da travessa comum para a boca. III - No se aceita o emprego dos dedos, nem a transferncia dos alimentos diretamente da travessa comum para a boca. Quais so corretas? a) Apenas I b) Apenas III c) Apenas I e II d) Apenas II e III e) I, II e III

10. Para evitar o uso de pego (li. 34), foram elaboradas as alternativas seguintes de reescrita para a frase em que se encontra a palavra. I - Tudo que retirado das travessas, molheiras e saleiros comuns deve ter sido feito com os utenslios adequados e depositar no prato antes de ser tocado com os prprios talheres boca. e levado II - Tudo que retirado das travessas, molheiras e saleiros comuns deve ser pegado com os utenslios adequados e depositado no prato antes de ser tocado com os prprios talheres e levado boca. III - Deve-se apanhar com os utenslios adequados o contedo das travessas, molheiras e saleiros comuns e deposit-lo no prato antes de toc-lo com os prprios talheres e lev-lo boca. Quais so corretas? a) Apenas I b) Apenas III c) Apenas I e II d) Apenas II e III e) I, II e III

629

PORTUGUS
Instruo: As questes de nmeros 11 a 17 referem-se ao texto abaixo:
1. 2. 3. 4. 5. 6. 7. 8. 9. 10. 11. 12. 13. 14. 15. 16. 17. 18. 19. 20. 21. 22. 23. 24. 25. Mas pra voc no uma pouca vergonha, ? ela disse, secando as lgrimas. Voltava ao desafio. Eu sou homem, ou voc no entende isto? No entendo. O Dr. Paranhos quedou-se quieto por um tempo marcado em sculos no relgio da agonia. Seus olhos baixaram para a mesa e assim ficaram, enquanto sua inteligncia buscava uma forma clara de ilustrar a madrinha. Por fim, suspirou e disse: Eu tenho um argumento que voc vai entender. Levantou-se com cer ta dificuldade e ausentou-se da cozinha, onde ficamos suspensos por um fio em pleno abismo. Logo, porm, ele retornou com o argumento na mo. Girou o tambor do argumento. Depois, com um gesto calmo, solene, botou o argumento na cintura. Sentou-se novamente e disse, pausadamente: Tem uma coisinha, Dirce: ou este tal de feminismo acaba hoje, ou o que acaba a tua mesada, se no acabar minha pacincia antes. Certo? Agora vamos jantar.

UFRGS

(SS, Ernani. O sempre lembrado. Porto Alegre: IGEL/IEL, 1989. p. 136-7)

11. Assinale a alternativa em que a palavra se (li. 6) exerce a mesma funo que no texto. a) b) c) d) e) Quem muito se queixa do casamento pouco faz para sair dele. Se eu fosse voc, reconsideraria o pedido de divrcio. Quanto vida em comum, sabe-se que no um mar de rosas. Nas piores brigas, os cnjuges se ofendem mutuamente. S vou se voc casar comigo depois.

630

PORTUGUS

UFRGS

12. A respeito do argumento a que se refere o texto a partir da linha 12, so feitas as afirmativas seguintes: I - O Dr. Paranhos utiliza-se de linguagem no verbal para apresentar seu argumento. II - Na atitude e na fala do Dr. Paranhos est implcita a ameaa do uso da fora. III - Pelos dados fornecidos nas seqncias retornou com o argumento na mo, Girou o tambor do argumento, botou o argumento na cintura, pode-se inferir que o Dr. Paranhos voltou para a cozinha com um revlver. Quais so corretas? a) Apenas I b) Apenas II c) Apenas I e III d) Apenas II e III e) I, II e III

13. Sobre o uso de nexos no texto, so feitas as afirmativas seguintes. I - A palavra onde (li. 15) refere-se palavra cozinha (li. 15). II - A palavra logo (li. 16) poderia ser substituda pela palavra portanto, sem alterao do significado original da frase. III - Se a palavra ou de ou este tal de feminismo (li. 22-23) fosse retirada, a frase no sofreria alterao do seu significado original. Quais so corretas? a) Apenas I b) Apenas II c) Apenas I e III d) Apenas II e III e) I, II e III

14. Dentre os marcadores de cronologia abaixo, o nico que NO situa a ao que se lhe segue no texto como anterior ou posterior a alguma outra a) enquanto (li. 9). b) por fim (li. 10). c) depois (li. 18). d) antes (li. 24). e) agora (li. 25).

15. Para substituir no texto o discurso direto pelo indireto, foram elaboradas as trs reescritas seguintes, uma para cada passagem do texto indicada entre parnteses. I - Secando as lgrimas, ela perguntou que para ele no era uma pouca vergonha. (li. 1-2) II - Ele disse que tinha um argumento que ela iria entender. (li. 12-13) III - O Dr. Paranhos disse a Dirce uma coisinha: ou este tal de feminismo acabava neste dia, ou o que acabava era a mesada dela, se no tivesse acabado a pacincia dele antes. (li. 22-24) Quais so corretas? a) Apenas I b) Apenas II c) Apenas I e III d) Apenas II e III e) I, II e III

631

PORTUGUS
16. Considere as afirmativas seguintes, sobre voz passiva.

UFRGS

I - O trecho um tempo marcado em sculos no relgio da agonia (li. 6-7) o agente da passiva da frase em que est inserido. II - Se a orao subordinada da frase Eu tenho um argumento que voc vai entender (li. 12-13) fosse passada para a voz passiva, o resultado seria Eu tenho um argumento que vai ser entendido por voc. III - A frase Girou o tambor do argumento (li. 18) pode ser passada para a voz passiva. Quais so corretas? a) Apenas I b) Apenas II c) Apenas I e III d) Apenas II e III e) I, II e III

17. A alternativa em que ambas as palavras ou expresses se referem a um mesmo personagem a) voc (li. 1), voc (li. 12). b) ela (li. 2), Dirce (li. 22). c) voc (li. 4), o Dr. Paranhos (li. 6). d) eu (li. 4), voc (li. 12). e) eu (li. 12), Dirce (li. 22).

632

PORTUGUS
Instruo: As questes de nmeros 18 a 26 referem-se ao texto abaixo:
1. 2. 3. 4. 5. 6. 7. 8. 9. 10. 11. 12. 13. 14. 15. 16. 17. 18. 19. 20. 21. Apesar de no termos iluses quanto ao carter das nossas elites, existia uma cer ta resistncia a essa espcie de niilismo a que o Brasil nos leva. Os escndalos na rea financeira esto acabando at com isso. Fica cada vez mais difcil espantar os burgueses. Os burgueses no se espantam com mais nada. Alguns talvez se surpreendam quando ouvem um filho pequeno ou um neto repetindo uma letra dos Mamonas, mas nestes casos o espanto diver tido, ou pelo menos resignado. A necessidade de se ser absolutamente claro sobre que tipos de atividade sexual causam AIDS e como fazer para preveni-la acabou com qualquer preocupao da imprensa e da propaganda com o pundonor (grande palavra) alheio embora ainda faam alguns rodeios. A linguagem ficou mais leve, ficamos menos hipcritas. Burgueses epatveis ainda existem, mas o acmulo de agresses a seus ouvidos e pruridos os insensibilizou e hoje, se reagem, no em pblico.

UFRGS

(VERISSIMO, L.F. Conluio. Porto Alegre: Extra Classe, junho/julho de 1996. p.3)

18. Segundo lemos no texto, a) b) c) d) e) os escndalos financeiros esto acabando com as elites brasileiras. os Mamonas acabaram de uma vez por todas com o espanto burgus. a imprensa no direta quando fala em AIDS. ns no mais somos hipcritas devido leveza da linguagem da mdia. os burgueses esto perdendo sua capacidade de espanto.

19. Considere as seguintes afirmativas sobre o significado de algumas expresses no texto. I - O uso de at na linha 5 indica que os escndalos financeiros no aniquilam apenas a resistncia ao niilismo. II - Os adjetivos certa (li. 2) e grande (li. 16) poderiam estar depois do substantivo que modificam, sem que houvesse alterao no significado das expresses. III - A palavra at poderia aparecer depois de com isso (li. 5), sem que houvesse qualquer alterao no significado da frase em que ocorre. Quais esto corretas? a) Apenas I b) Apenas II c) Apenas I e III d) Apenas II e III e) I, II e III

633

PORTUGUS

UFRGS

20. A palavra epatveis (l. 19) foi criada por Verissimo a partir do verbo pater, que no francs significa espantar. O neologismo do autor segue as regras de derivao da lngua portuguesa da seguinte forma: a partir do emprstimo da palavra francesa, obtm-se o verbo epatar; tomando tal verbo como radical possvel, ento, a formao de epatveis. Seguindo a sugesto do autor, foram criados os cinco neologismos que aparecem na coluna da esquerda; na coluna da direita, aparecem observaes sobre a estrutura ou o significado de alguns desses neologismos. 1. 2. 3. 4. 5. epatados epatabilidade inepatveis epatador epatadio ( ) substantivo formado a partir do adjetivo epatvel ( ) substantivo que designa o agente de uma ao, significando aquele que espanta ( ) palavra formada pela adio de prefixo e sufixo

Assinale a alternativa cuja numerao corresponde associao correta entre a coluna da esquerda e a da direita, de cima para baixo. a) 2 - 4 - 3 b) 1 - 4 - 3 c) 5 - 2 - 1 d) 1 - 5 - 2 e) 2 - 5 - 1

21. Observe as seguintes afirmaes sobre o perodo que comea com A necessidade (li. 11-12) e vai at rodeios. I - A reescrita do trecho A necessidade de se ser absolutamente claro como A necessidade de clareza absoluta no acarretaria alterao nem no significado do perodo, nem em sua estrutura. II - O uso de se (li. 12) indica que no so apenas a imprensa e a propaganda que precisam ser claras quando se referem AIDS. III - A substituio de qualquer (li. 14) por toda e a simultnea supresso de alguns (li. 17) no acarretariam alterao no significado do perodo. Quais esto corretas? a) Apenas I b) Apenas II c) Apenas III d) Apenas I e III e) I, II e III

22. Assinale a alternativa que apresenta a substituio que poderia ser feita no texto sem alterao de significado ou necessidade de qualquer ajuste na estrutura da frase. a) b) c) d) e) apesar de (li. 1) por ainda que pelo menos (li. 11) por quando muito embora (li. 16) por mesmo que mas (li. 19) por pois se (li. 21) por caso

634

PORTUGUS

UFRGS

23. Assinale a alternativa que apresenta as palavras que exigem, respectivamente, as duas ocorrncias da preposio a na linha 3. a) existia (li. 2), niilismo (li. 3) b) resistncia (li. 3), leva (li. 4) c) existia (li. 2), espcie (li. 3) d) resistncia (li. 3), niilismo (li. 3) e) espcie (li. 3), leva (li. 4)

24. Assinale a alternativa em que h uma associao correta entre o pronome e a palavra que ele substitui no texto. a) b) c) d) e) que (li. 3) - essa espcie de niilismo (li. 3) isso (li. 5) - niilismo (li. 3) la (li. 14) - atividade sexual (li. 13) seus (li. 20) - agresses (li. 20) os (li. 20) - ouvidos e pruridos (li. 20)

25. Considere as seguintes afirmaes. I - A palavra pundonor (li. 16) poderia ser substituda por pudor ou decoro, sem que o significado original do texto fosse alterado. II - O comentrio parenttico feito pelo autor logo aps a palavra pundonor (li. 16) indica uma reflexo sobre escolhas lexicais do texto. III - Com o comentrio parenttico que aparece logo aps a palavra pundonor (li. 16), o autor chama a ateno de seu leitor para um aspecto sinttico do texto. Quais esto corretas? a) Apenas I b) Apenas II c) Apenas III d) Apenas I e II e) I, II e III

26. Observe as seguintes afirmaes sobre concordncia. I - Caso a palavra alguns na linha 7 fosse substituda por algum, apenas dois verbos deveriam sofrer ajustes para fins de concordncia. II - Caso tivssemos O burgus ao invs de Burgueses na linha 18, quatro outras palavras deveriam sofrer ajustes para fins de concordncia. III - Caso a seqncia da imprensa e da propaganda (li. 15) fosse substituda por da mdia, o verbo faam (li. 17) deveria sofrer ajuste para fins de concordncia. Quais esto corretas? a) Apenas I b) Apenas I e II c) Apenas I e III d) Apenas II e III e) I, II e III

635

PORTUGUS
Instruo: As questes de nmeros 27 a 30 referem-se ao texto abaixo:
Que me? 1. 2. 3. 4. 5. 6. 7. 8. 9. 10. 11. 12. 13. 14. 15. 16. 17. 18. 19. 20. 21. 22. 23. 24. A me de hoje em dia uma mulher esbaforida, a representar papel avesso que lhe foi imposto at alguns anos atrs. Ela est s voltas com o problema de ter e criar filhos e de administrar uma carreira. Em muitos casos ela parte importante, quando no a nica fonte de renda da casa. Freqentemente ela a ponte para que a famlia de classe mdia classe mdia alta. [...] No limite do raciocnio, a expectativa sancionada pela sociedade repele uma mulher que possa abandonar o trabalho, que tantas abriu, e troc-lo pela possibilidades ateno aos filhos. A me tradicional, a antes louvada rainha do lar, por assim dizer, est rechaada. No ps-feminismo, a sensao de que todas as chances existem para uma mulher forte e competente chegar ao pice em qualquer terreno. [...] A me que todas as geraes conheceram aquela que ficava em casa e tratava das tarefas da famlia, especialmente dos filhos ser no futuro objeto de relato histrico a provocar estranheza nos ouvintes.

UFRGS

(Santos, Mrio Vtor. Que me? So Paulo: Revista da Folha de So Paulo, 12.05.96)

27. Assinale a alternativa que preenche corretamente as lacunas do texto (linhas 2, 9 e 13) na ordem em que aparecem. a) no - ascenda - lhe b) ao - ascenda - lhe c) no - ascenda - ela d) do - ascenda - lhe e) ao - ascenda - ela

28. Assinale a alternativa que faz uma afirmao correta sobre o significado do texto. a) Segundo lemos nas linhas 10 a 14, a sociedade no mais espera que as mes cuidem de seus filhos. b) De acordo com o que lemos nas linhas 16 a 19, o ps-feminismo trouxe a garantia de a mulher chegar ao topo em qualquer carreira que escolher, desde que seja forte e competente. c) De acordo com o primeiro pargrafo, a me da sociedade contempornea no cria seus filhos, mas administra uma carreira e permite que sua famlia mude de classe social. d) Segundo lemos na linha 10, o raciocnio segundo o qual a sociedade no espera que as mes abandonem seus empregos limitado. e) A me que fica em casa, segundo o ltimo pargrafo do texto, tende a desaparecer, mesmo que ainda possa ser conhecida pelas geraes atuais.

636

PORTUGUS

UFRGS

29. Assinale a alternativa que apresenta sinnimos adequados para as palavras sancionada (li. 11) sensao (li. 16) e provocar (li. 23), respectivamente, considerando o contexto em que tais palavras ocorrem. a) aprovada, impresso, suscitar b) criada, impresso, suscitar c) criada, intuio, suscitar d) aprovada, intuio, incitar e) aprovada, impresso, incitar

30. Assinale a nica alternativa que apresenta uma sugesto de reescrita que altera o significado original do trecho apontado entre parnteses. a) b) c) d) A me , hoje, uma mulher esbaforida, representando papel avesso... (li. 1-3) Ela , em muitos casos, parte importante... (li. 5-7) Ela a ponte para que a famlia freqentemente... (li. 7-9) ... todas as chances existem para, em qualquer terreno, uma mulher forte e decidida chegar ao pice. (li. 16-18) e) No futuro, a me que todas as geraes conheceram [...] ser objeto de relato histrico a provocar estranheza nos ouvintes. (li. 20-24) REDAO H muitas discusses sobre a reformulao do Concurso Vestibular. Alguns propem sua extino sumria; outros defendem que seja substitudo por concursos isolados para as diversas carreiras acadmicas; outros ainda advogam sua reformulao parcial, sugerindo pequenos ajustes na forma atual ou propondo a exigncia de questes apenas discursivas em todas as provas. Como membro da comunidade na qual esse concurso realizado, voc certamente tem muito a dizer sobre a adequao do vestibular ao tipo de candidato que normalmente presta os exames. Alm disso, ainda que jamais se tenha preocupado antes com o vestibular, voc est neste exato instante, envolvido com ele. Voc acha que o vestibular, na sua forma atual, seleciona de maneira eficaz os futuros alunos da UFRGS, valorizando adequadamente seus conhecimentos, habilidades e aptides? Ou voc acredita que possa haver uma maneira melhor de faz-lo? Lembre-se de que qualquer proposta relativa ao vestibular, se eventualmente adotada pela Universidade, teria reflexos extraordinrios na sociedade como um todo, e no ao seu bem individual que a Universidade visa, e sim ao da coletividade. Pois bem: sua redao dever desenvolver sua resposta questo que reformulaes deve haver no Concurso Vestibular da UFRGS?. Para isso, parta de sua experincia pessoal, enuncie a(s) reformulao(es) que considera necessria(s) e apresente motivos para sua proposta. Desde logo, fique claro que, para fins de atribuio de nota, no ser avaliado o mrito de suas opinies, mas sim sua capacidade de redigir um texto correto e articulado sobre o tema. Lembre-se de que voc est sendo solicitado a redigir uma dissertao, texto que se caracteriza por um esforo de reflexo racional em torno de um tema. Valha-se da experincia como ponto de partida, mas apresente-a articulada em um texto argumentativo, organizado dissertativamente. A dissertao deve ter a extenso mnima de 30 linhas e mxima de 60, considerando letra de tamanho regular. Inicialmente, utilize a folha de rascunho e, depois, passe a limpo na folha de redao, sem rasuras e com letra legvel, o que voc escreveu. Utilize caneta; lpis, apenas no rascunho.

637

PORTUGUS
LITERATURA BRASILEIRA 1. Sobre a poesia de Gregrio de Matos Guerra correto afirmar que

UFRGS

a) privilegia os cenrios buclicos percorridos por pastores e ninfas examinados de uma perspectiva satrica e irnica. b) expe em sintaxe simples o carter sereno e amoroso de um pastor que corteja sua amada com promessas de vida amena e burocrtica. c) expe em sintaxe complexa e com metforas antitticas os dilemas do amor e do esprito no quadro da Contra-Reforma. d) privilegia o cenrio urbano para denunciar as arbitrariedades da Inquisio e o racismo dos portugueses instalados na colnia. e) privilegia os cenrios palacianos em que ocorrem intrigas e conspiraes envolvendo nobres burocratas, monges e prostitutas. Instruo: O poema abaixo refere-se s questes de nmeros 2 e 3:
LIRA III Tu no vers, Marlia, cem cativos Tirarem o cascalho, e a rica terra, Ou dos cercos dos rios caudalosos, Ou da minada serra. No vers separar ao hbil negro Do pesado esmeril a grossa areia, E j brilharem os granetes de ouro No fundo da bateia. No vers derrubar os virgens matos; Queimar as capoeiras ainda novas; Servir de adubo terra a frtil cinza; Lanar os gros nas covas. No vers enrolar negros pacotes Das secas folhas do cheiroso fumo; Nem espremer entre as dentadas rodas Da doce cana o sumo. Vers em cima da espaosa mesa Altos volumes de enredados feitos: Ver-me-s folhear os grandes livros, E decidir os pleitos. Enquanto revolver os meus consultos, Tu me fars gostosa companhia, Lendo os fastos da sbia mestra histria, E os cantos da poesia. Lers em alta voz a imagem bela, E eu, vendo que lhe ds o justo apreo, Gostoso tornarei a ler de novo O cansado processo. (...)

NOTA: fastos = anais, registros.

638

PORTUGUS
2. O autor dos versos citados a) Alvarenga Peixoto. b) Toms Antnio Gonzaga. c) Cludio Manuel da Costa. 3. Considere as afirmativas seguintes. d) Silva Alvarenga. e) Baslio da Gama.

UFRGS

I - O poeta critica Marlia por no enxergar as atividades produtivas que provem o sustento e a riqueza do casal, mas elogia o senso esttico da amada, capaz de discernir entre boa e m poesia. II - Em estrofes de quatro versos, todas elas com rima entre o 2 e o 4 versos, o poeta contrasta a paisagem externa com o ambiente domstico em que ele e Marlia dividem tarefas. III - A cena domstica descrita pelo poeta demonstra a harmonia entre o casal que se dedica a atividades intelectuais, sendo que Marlia complementa e suaviza o cotidiano do marido. Quais esto corretas? a) Apenas I b) Apenas III c) Apenas I e II d) Apenas II e III e) I, II e III

4. Sobre a poesia de Gonalves Dias correto afirmar que a) cantou a natureza brasileira como cenrio das correrias e aventuras do indgena bravo e leal. b) denunciou a iniqidade da escravido em poemas altissonantes e repletos de metforas aladas. c) elogiou os esforos do colonizador portugus em suas campanhas militares. d) cantou a bondade da me e da irm, esteios femininos do ncleo familiar patriarcal. e) elogiou a dissipao e os excessos do vinho em orgias noturnas marcadas pela devassido e crueldade.

639

PORTUGUS
Instruo: Os poemas a seguir referem-se s questes de nmeros 5 a 7:
AMOR E MEDO Casimiro de Abreu Quando eu te fujo e me desvio cauto Da luz de fogo que te cerca, bela, Contigo dizes, suspirando amores: Meu Deus, que gelo, que frieza aquela! Como te enganas! meu amor chama, Que se alimenta no voraz segredo, E se te fujo que te adoro louco... s bela eu moo; tens amor, eu medo!... Tenho medo de mim, de ti, de tudo, Da luz, da sombra, do silncio ou vozes, Das folhas secas, do chorar das fontes, Das horas longas a correr velozes. (...) BOA-NOITE Castro Alves Boa-noite, Maria! Eu vou-me embora. A lua nas janelas bate em cheio. Boa-noite, Maria! tarde... tarde... No me apertes assim contra teu seio. Boa-noite!... E tu dizes Boa noite, Mas no digas assim por entre beijos... Mas no mo digas descobrindo o peito, Mar de amor onde vagam meus desejos. Julieta do cu! Ouve... a calhandra J rumoreja o canto da matina. Tu dizes que eu menti?... pois foi mentira... Quem cantou foi teu hlito, divina! (...)

UFRGS

5. Em Boa-Noite, Castro Alves a) apresenta uma cena de amor interrompida pelo canto de um pssaro que anuncia o surgimento de uma terceira pessoa na sala. b) despede-se da amada, que retribui o adeus demonstrando em sua atitude a vontade de que o amante permanea. c) despede-se da amada alegando que o luar, ao bater nas janelas, h de denunci-los aos demais moradores da casa. d) apresenta uma cena em que o amante amedrontado avalia depreciativamente os dotes fsicos da amada. e) despede-se da amada que se encontra no leito, desnuda e temerosa de que eles sejam surpreendidos.

640

PORTUGUS
6. Em Amor e medo, Casimiro de Abreu

UFRGS

a) recomenda cautela amada para que a luz de fogo que a cerca no revele a terceiros os segredos do casal. b) evita os encantos da amada justamente por desejar a moa em excesso, respondendo ao amor dela com seu medo. c) nota que a amada engana-se ao julg-lo ardente e amoroso, pois se trata apenas de uma impresso causada pela distncia que os separa. d) evita aproximar-se da amada porque as horas longas a correr velozes em breve prejudicaro a intensidade do desejo que os une. e) discorda da amada que afirma que ele foge dela para evitar a intensidade do amor que se alimenta no voraz segredo. 7. Considere as afirmativas seguintes. I - Os dois poemas apresentam mulheres envolvidas pela atrao amorosa: em Amor e medo a moa suspira e ressente-se da suposta indiferena do poeta; em Boa-noite ocorre o contato fsico entre o casal, e a mulher insinua com alguma veemncia que no deseja a partida do amante. II - Os dois poemas so constitudos de estrofes de quatro versos decasslabos dos quais o segundo e o quarto de cada estrofe rimam entre si. III - Nos dois poemas aparece a primeira pessoa do singular, que manifesta o eu-lrico do poeta em contato com a mulher amada; eu-lrico que, em Amor e medo, afirma amor ardente e medo enorme e, em Boa-noite, encontra-se envolvido em uma cena ntima e ertica. Quais esto corretas? a) Apenas I b) Apenas II c) Apenas I e III d) Apenas II e III e) I, II e III

641

PORTUGUS
Instruo: As questes de nmeros 8 e 9 referem-se ao texto abaixo:

UFRGS

A senhora estranhou, na ltima vez que estivemos juntos, a minha excessiva indulgncia pelas criaturas infelizes, que escandalizam a sociedade com a ostentao do seu luxo e extravagncias. Quis responder-lhe imediatamente, tanto o apreo em que tenho o tato sutil e esquisito da mulher superior para julgar de uma questo de sentimento. No o fiz porque vi sentada no sof, do outro lado da sala, sua neta, gentil menina de 16 anos, flor cndida e suave, que mal desabrocha sombra materna. Embora no pudesse ouvir-nos, a minha histria seria uma profanao na atmosfera que ela purificava com os perfumes de sua inocncia; e quem sabe talvez por ignota repercusso o melindre de seu pudor se arrufasse unicamente com os palpites de emoes que iam acordar em minha alma. [...] Desculpe, se alguma vez a fizer corar sob os seus cabelos brancos, pura e santa coroa de uma virtude que eu respeito. O rubor vexa em face de um homem; mas em face do papel, muda e impassvel testemunha, ele deve ser para aquelas que j imolaram velhice os ltimos desejos, uma como essncia de gozos extintos, ou extremo perfume que deixam nos espinhos as desfolhadas rosas. De resto, a senhora sabe que no possvel pintar sem que a luz projete claros e escuros. s sombras do meu quadro se esfumam traos carregados, contrastam debuxando o relevo colorido de lmpidos contornos.

8. Sobre o texto acima, trecho de abertura de um romance de Jos de Alencar, so feitas as afirmativas seguintes. I - O autor usa de um artifcio para disfarar o carter ficcional da narrativa e, ao mesmo tempo, estreitar os laos da interlocuo com o leitor: cria um personagem-narrador e uma personagem-leitora, esta com a funo de propiciar ao narrador a oportunidade de contar a sua histria. II - Pela referncia a criaturas infelizes que escandalizam a sociedade, profanao da atmosfera em que est envolta a neta da personagem qual se dirige o narrador, bem como pela cautela do narrador em abordar assunto que causa melindres, pode-se inferir que se trata da abertura do romance Lucola. III - De acordo com o terceiro pargrafo, a personagem-leitora no se deve preocupar caso leia algo que ofenda seu pudor, pois diante do texto escrito no so vexatrias, para a mulher, atitudes que seriam motivo de vergonha na presena de um homem. Quais so corretas? a) Apenas I b) Apenas II c) Apenas I e III d) Apenas II e III e) I, II e III

642

PORTUGUS
9. Alm do romance de que foi retirado o texto, Jos de Alencar escreveu

UFRGS

a) Inocncia, cuja ao se passa no serto, onde mora Pereira, que divide as mulheres entre prostitutas e famlias, estas as filhas e esposas mantidas isoladas em casa, pois, se vissem homem que no o pai ou marido, fatalmente se igualariam s prostitutas devido a sua condio de fmeas, por definio depravadas. b) O seminarista, romance passional que discute a instituio do celibato clerical: o Padre Eugnio, retornando cidade natal, sente reacender-se uma antiga paixo; apesar de tentar resistir, acaba por cair em tentao, e a quebra do voto de castidade o leva loucura. c) A moreninha, narrativa de intriga simples e temtica mundana, em que os sentimentos e motivaes jamais adquirem profundidades imprprias para a conversao em sociedade na presena de mocinhas e senhoras respeitveis. d) Ubirajara, um de seus romances voltados para a recriao do passado do Brasil, narrando os feitos hericos de um ndio capaz de permancer impassvel enquanto formigas savas lhe devoram a mo, numa das provas a que se submete para fazer jus mo de sua pretendida. e) Memrias de um sargento de milcias, que apresenta o quotidiano da arraia-mida do Rio de Janeiro, composta de personagens permanentemente em luta pela satisfao das necessidades humanas mais elementares e sempre vidos de gozar os momentos de sorte favorvel. 10. A respeito de Machado de Assis so feitas as afirmativas seguintes. I - Em seus primeiros romances denuncia a hipocrisia da sociedade da poca e apresenta solues originais para os conflitos dos protagonistas, como a morte de Helena. II - Em contos como Teoria do medalho, O espelho e Um homem clebre, ironiza a obsesso da elite brasileira pelas aparncias e sua incapacidade de ir alm da mediocridade. III - Em O Alienista, faz um discreto elogio cincia, vista como fonte do progresso e meio seguro de superar as contradies da sociedade brasileira no perodo imperial. Quais so corretas? a) Apenas I b) Apenas II c) Apenas I e III d) Apenas II e III e) I, II e III

11. Tomando em considerao seus romances, correto afirmar que Machado de Assis a) faz de Memrias pstumas de Brs Cubas o primeiro romance naturalista brasileiro, ao descrever em detalhes o processo de decomposio do corpo de Brs Cubas. b) explicita em Quincas Borba sua teoria do Humanitismo, a qual serve de justificativa para a atitude arrivista de Rubio e sua ascenso social baseada na trapaa e na seduo. c) estabelece ele prprio as bases da polmica sobre a fidelidade de Capitu a Bentinho ao adotar em Dom Casmurro um personagem-narrador parcial e desconfiado. d) discute em Esa e Jac os dilemas religiosos da sociedade brasileira do sculo passado, baseada numa populao majoritariamente umbandista, mas fiel herana judaico-crist. e) aproveita a aposentadoria do Conselheiro Aires, personagem de Esa e Jac, para, no Memorial de Aires faz-lo relembrar suas aventuras picarescas entre a senzala e a igreja.

643

PORTUGUS
12. A respeito do romance brasileiro do final do sculo XIX, pode-se afirmar que

UFRGS

a) o Impressionismo, com sua linguagem cientificista e a expresso de um determinismo mecanicista, buscava os aspectos mrbidos da atividade humana para demonstrar que o ser humano movido por condicionamentos de duas fontes: o meio e a herana gentica. b) o Realismo, movimento saudosista, buscava demonstrar como a vida era melhor no tempo da monarquia, opondo a estabilidade do regime sob a tutela de Dom Pedro II ao arrivismo de casais republicanos como Palha e Sofia. c) personagens como Pombinha, a moa decente que se torna homossexual e prostituta por influncia da madrinha, e Jernimo, o portugus trabalhador que larga a esposa por uma mulata fogosa, so ilustraes das teses naturalistas de Alusio de Azevedo. d) Adolfo Caminha autor de um romance de cunho memorialstico que narra as transformaes que se do na personalidade de um menino ao entrar em contato com o meio hipcrita e violento de um cortio do Rio de Janeiro no tempo do Imprio. e) O Ateneu narra a histria de um marinheiro mulato que, para tentar fugir ao preconceito que o isola dos prprios parentes, embarca no navio que d ttulo ao livro, sem saber que o capito e o imediato planejam vend-lo como escravo. 13. Sobre o Simbolismo brasileiro correto afirmar que a) reelabora a fala popular carioca em curtos poemas de temtica urbana repletos de elipses e trocadilhos bilnges. b) retoma a temtica romntica com nimo satrico e polmico, inclusive parodiando trechos de romances do sculo XIX. c) explora a mitologia greco-latina e episdios da histria antiga da Europa em sonetos descritivos com chave-de-ouro. d) explora a sugestividade dos sons da lngua em poemas que reportam sensaes indefinidas e sentimentos vagos. e) reelabora a musicalidade dos vocbulos com experincias em que as palavras so segmentadas e a frase parte-se em fragmentos. 14. correto afirmar que Augusto dos Anjos foi o poeta do a) pessimismo aliado cincia que acusava a degradao humana mediante associaes e comparaes com processos qumicos e biolgicos. b) cientificismo triunfante que, aliado idia de progresso, marcou boa parte da lrica contempornea aos primeiros anos da Repblica. c) pessimismo acusatrio que denunciou o latifndio e a poltica oligrquica, reproduzindo na poesia as preocupaes e temas de Lima Barreto. d) esteticismo que depurava a forma de seus sonetos perfeio, sem jamais fazer concesses a temas considerados prosaicos ou de mau gosto. e) cientificismo militante disposto a abranger temas como o clculo algbrico, a crtica literria e arquitetura para retirar o carter subjetivo da poesia.

644

PORTUGUS
15. Assinale a nica alternativa INCORRETA a respeito da obra de Euclides da Cunha.

UFRGS

a) Em Os Sertes, o autor se utiliza de uma linguagem preciosista e rebuscada, em que pontificam os termos emprestados cincia da poca, os adjetivos exagerados e os verbos cuidadosamente escolhidos. b) Os Sertes o produto final de um longo trabalho de pesquisa bibliogrfica, de uma engajada cobertura jornalstica realizada in loco pelo autor e da reviso de anotaes detalhadas feitas por Euclides da Cunha durante sua estada no serto baiano. c) A utilizao de personagens ficcionais lado a lado com as figuras histricas que protagonizam os acontecimentos de Os Sertes permite a Euclides da Cunha inserir suas impresses pessoais sobre o episdio e, alm disso, estabelece nexos entre as aes isoladas. d) Nas duas primeiras partes de Os Sertes o autor se mostra adepto das doutrinas cientficas da poca, que no viam com bons olhos a mestiagem e estabeleciam uma relao direta entre herana biolgica e progresso material dos povos. e) Na ltima parte de Os Sertes, A Luta, opera-se uma transformao do ponto de vista do autor, que, graas tcnica narrativa utilizada, contagia o leitor: inicialmente vistos como fanticos, os sertanejos de Canudos conquistam o respeito e a admirao.

Instruo: O poema abaixo refere-se s questes de nmeros 16 a 19:


CASO DO VESTIDO
Carlos Drummond de Andrade Nossa me, o que aquele vestido, naquele prego? Minhas filhas, o vestido de uma dona que passou. Passou quando, nossa me? Era nossa conhecida? Minhas filhas, boca presa. Vosso pai evm chegando. Nossa me, dizei depressa que vestido esse vestido. Minhas filhas, mas o corpo ficou frio e no o veste. O vestido, nesse prego, est morto, sossegado. Nossa me, esse vestido tanta renda, esse segredo! Minhas filhas, escutai palavras de minha boca. Era uma dona de longe, vosso pai enamorou-se. E ficou to transtornado, se perdeu tanto de ns, se afastou de toda vida, se fechou, se devorou, chorou no prato de carne, bebeu, brigou, me bateu, me deixou com vosso bero, foi para a dona de longe, mas a dona no ligou. Em vo o pai implorou. Dava aplice, fazenda, dava carro, dava ouro, beberia seu sobejo, lamberia seu sapato. Mas a dona nem ligou. Ento vosso pai, irado, me pediu que lhe pedisse, a essa dona to perversa, que tivesse pacincia e fosse dormir com ele... Nossa me, por que chorais? Nosso leno vos cedemos. Minhas filhas, vosso pai chega ao ptio. Disfarcemos. (...)

645

PORTUGUS
16. Em Caso do vestido, os episdios envolvendo a dona e o pai so narrados a) b) c) d) e) pela me. pelo filho, mediante a voz do poeta. pelo pai. por uma das filhas. pelas duas filhas.

UFRGS

17. Sobre as filhas curiosas, correto afirmar que a) b) c) d) e) lembram-se da dona, embora no tenham reconhecido o vestido guardado pela me. lembram-se do episdio, embora fossem crianas de bero quando tudo ocorreu. desconhecem o caso do vestido, embora se lembrem do sofrimento da me. querem conhecer os episdios envolvendo a runa pecuniria da famlia. insistem para que a me revele os episdios envolvendo o vestido e a dona.

18. Sobre Caso do vestido, correto afirmar que a) a denncia do condicionamento hereditrio da mulher demonstra o dbito do Modernismo para com o Naturalismo. b) as metforas envolvendo me e filhas exploram a musicalidade da lngua e sensaes de indefinido. c) a crtica ao patriarcalismo inclui comentrios sobre a profisso do pai e da me mencionados. d) a narrativa um tanto conformada e desapaixonada contrasta com a tenso do caso passional relatado. e) a denncia da arbitrariedade do pai entra em conflito com a perspectiva idealizada e romntica das filhas. 19. Considere as afirmativas seguintes. I - A me demonstra m vontade e s depois da insistncia das filhas comea a narrar o caso do vestido, inclusive o episdio em que se recusa a atender o pedido do marido e a implorar favores dona que o fascina. II - Segundo a me, o pai de suas filhas chegou ao ponto de embebedar-se, espanc-la e abandon-la, vindo a humilhar-se e a oferecer bens e riqueza para que a dona de longe o atendesse. III - Caso do vestido consiste em um dilogo entre a me e suas filhas, s quais a me se dirige usando a segunda pessoa do plural; o pai, cuja chegada anunciada, comparece principalmente como assunto da conversa. Quais esto corretas? a) Apenas I b) Apenas III c) Apenas I e II d) Apenas II e III e) I, II e III

646

PORTUGUS

UFRGS

Instruo: O trecho abaixo, extrado de Memrias sentimentais de Joo Miramar, de Oswald de Andrade, refere-se s questes de nmeros 20 e 21:
129. ATO III. CENA I. Na preguia solar da mesma sala grande onde framos felizes, Clia e a cadeira de balano choravam como um tango. J viu sua filha como est grandinha? J. Nem se importa mais com ela. Ela teve sarampo e gripe. Quase ficou com o olho torto. (Um silncio cheio de moscas.) Diga a verdade! Recebi uma carta annima contando tudo. No h nada mais triste do que ser enganada. Voc est apaixonado por essa atriz, Joozinho! Conte tudo. Acho voc envelhecido, preocupado, com cara de viciado, Joozinho!

20. Joo Miramar e sua esposa Clia atravessam uma crise conjugal em que Clia acusa o marido de a) haver abandonado sua esposa, que contrara sarampo e gripe e estava entregue s moscas. b) estar apaixonado por uma atriz e de no prestar a devida ateno sade de sua filha, que estivera doente. c) ter enviado uma carta annima revelando seu caso como uma atriz e, depois, tentar enganla com desculpas. d) ter-se tornado um viciado que dilapidara a riqueza da famlia e desmoralizara o nome dos antepassados. e) t-la trado com uma atriz na mesma sala grande da fazenda onde eles tinham sido felizes. 21. correto afirmar que o trecho a) b) c) d) e) reproduz diretamente a fala dos personagens, que no so descritos detalhadamente. explora dados do folclore e da cultura popular na caracterizao de Joozinho. apresenta Clia como uma sofisticada dama da sociedade indiferente ao marido. traz descries minuciosas do ambiente em que a conversa ocorre. inverte a ordem cronolgica dos episdios envolvendo Joozinho e a atriz.

22. Considere as afirmativas seguintes. I - Mrio Quintana usou desde o soneto at o poema em prosa para encarnar sua viso de mundo irnica e melanclica, em que as possibilidades formais aliam-se ao exame do dia-a-dia humilde e despretensioso do poeta e de seus personagens. II - Capaz de sonetos que honram a tradio camoniana, tais como o Soneto da separao e o Soneto da fidelidade, Vincius de Morais aproximou-se da bossa nova e da msica popular brasileira e veio a se tornar letrista de sambas e canes. III - Em vrios momentos da obra de Ceclia Meireles a sensao de vago e incorpreo associada temtica urbana revelada em cenas repletas de humor e crtica ao provincianismo da elite paulista, tudo expresso em versos cuja sonoridade explora as dissonncias e a fala popular. Quais esto corretas? a) Apenas I b) Apenas III c) Apenas I e II d) Apenas II e III e) I, II e III

647

PORTUGUS

UFRGS

23. A respeito da obra de trs romancistas sul-rio-grandenses so feitas as afirmativas seguintes. I - O passado herico e guerreiro do Rio Grande do Sul no se faz presente diretamente em boa parte da obra de Erico Verissimo, mas surge destacado e carregado de contradies em O tempo e o vento. II - O Naziazeno de Os ratos, buscando apenas o dinheiro suficiente para pagar o leiteiro, a prpria anttese do heri eqestre metido em guerras e revolues, tanto quanto o personagem-ttulo de O louco de Cati. III - Na trilogia do Gacho a p, Cyro Martins examina como a expulso dos trabalhadores do campo pelo avano do capitalismo confrontou-se com a tradio de bravura e fidelidade cultuada pelo gacho tradicional. Quais so corretas? a) Apenas I b) Apenas III c) Apenas I e II d) Apenas II e III e) I, II e III

24. Grande serto: veredas rompe com a narrativa conhecida como Romance de 30 e estabelece um novo padro para a narrativa longa brasileira. Entretanto, a obra de Guimares Rosa NO rompe com a) b) c) d) e) a ambientao preferencialmente rural. o foco narrativo na terceira pessoal. a crtica ao latifndio. a denncia social. a linguagem enxuta e discreta.

25. O personagem Riobaldo, de Grande serto: veredas, a) tem pssima pontaria, e por isso prefere lutar faca com seus inimigos, alcanando fama de degolador cruel e inclemente. b) desnuda sua sensibilidade feminina passo a passo, a partir da descoberta da atrao fsica pelos outros jagunos. c) indaga-se a fundo para saber se o pacto que fez com o Diabo tem valor ou no e se as guerras e os sofrimentos humanos tm origem na interveno do Demnio. d) ao derrotar Joca Ramiro e Medeiro Vaz, ganha a inimizade de Z Bebelo, aliado dos dois anteriores e scio de Hermgenes. e) para ser reconhecido como lder pelos jagunos, forado a lutar com seu melhor amigo, apesar de saber que Diadorim tem uma reza que lhe d corpo fechado. 26. Seus romances, como Perto do corao selvagem ou A paixo segundo G.H., tm em comum com a obra de Guimares Rosa a inovao formal e o cuidado com a linguagem. A afirmativa se refere a a) Raquel de Queirs. b) Clarice Lispector. c) Nlida Pion. d) Lya Luft. e) Ana Miranda.

648

PORTUGUS
Instruo: O texto abaixo refere-se s questes de nmeros 27 a 29:
Ela: Por que voc mentiu?

UFRGS

Eu: Achei que se fssemos para a cama juntos voc ia querer abandonar o marido e vir viver comigo. Retido. Ela: Mas acabamos indo para a cama e eu no abandonei o meu marido. Eu: Um erro de clculo meu, felizmente. Ela: Pois hoje decidi vir aqui, e tambm te dar uma nota. Uma nota alta significa que abandonarei o meu marido e virei morar com voc. Ela comea a se vestir enquanto eu protesto, que loucura, pensa bem no que vai fazer, eu no sirvo para casamento, meu gnio no presta, sou um velho solteiro, empedernido Calas, liga-cinta, meias, souti, angua, vestido, sapatos e ela vai para o banheiro Sou cheio de manias!, grito para ela ouvir e ela me solta uma gargalhada dessas que nos desenhos animados fazem partir os vidros das janelas. Fico preocupado: uma gargalhada daquelas feiticeiras de nariz curvo, assexuadas, que tramam a desgraa do mocinho, e o mocinho, bolas!, sou eu. Ela sai do banheiro e ambos estamos srios. Ela me olha de alto a baixo, o rosto impassvel. Um olhar inesperado, que me surpreende. Encho-me de coragem e pergunto: qual a minha nota? Zero, diz ela. Procuro no meu rosto algo que me diga que tudo no passa de uma brincadeira, mas nada consigo ver, num sentido ou no outro. Ela sai e me deixa sozinho, um velho.

27. Considere as afirmativas seguintes. I - Com a nota zero, a personagem feminina produz uma reverso nas expectativas do personagem masculino, que temia que ela propusesse abandonar o marido e ir morar com ele. II - Ela est terminando o relacionamento porque descobriu que ele mentiu com a inteno de lev-la para a cama, o que merece nota zero em termos de relacionamento afetivo. III - O narrador enumera as peas de roupa vestidas por ela para relacionar indumentria e comportamento: ela uma feiticeira, pois usa roupas sensuais e provocantes para enfeitiar os homens. Quais so corretas? a) Apenas I b) Apenas III c) Apenas I e II d) Apenas II e III e) I, II e III

649

PORTUGUS

UFRGS

28. As caractersticas apontadas abaixo esto presentes nos contos de Rubem Fonseca, autor do texto citado, EXCEO DE a) problematizao do homem urbano e da impossibilidade do relacionamento afetivo estvel. b) dissecao das causas da violncia urbana e os efeitos desta na vida e na psicologia do brasileiro. c) desvelamento das modificaes dos padres de comportamento sexual e dos conflitos da decorrentes. d) representao dos tipos sociais em suas prprias linguagens, respeitando estilo e cdigo adequados. e) utilizao de tcnicas ultramodernas de comunicao sem abrir mo de uma utopia comunitria. 29. Como faz Rubem Fonseca no texto, seguidamente apresenta dilogos ficcionais entre um homem e uma mulher que discutem seu relacionamento afetivo-sexual, com a diferena de que o vis adotado por este cronista o do humor. A alternativa que preenche corretamente a lacuna a) Rubem Braga. b) Fernando Sabino. c) Paulo Mendes Campos. d) Lus Fernando Verissimo. e) Paulo Sant Ana.

30. Sobre a obra contstica de autores da literatura brasileira so feitas as afirmativas seguintes. I - O pessimismo, os personagens da burguesia carioca e a crtica velada aos costumes esto presentes nos contos de Machado de Assis. II - Os contos de Monteiro Lobato focalizam uma problemtica urbana e cosmopolita, dando as costas ao atrasado Brasil rural. III - O tom neutro e a nfase na descrio presentes em O Homem que sabia javans, de Lima Barreto, so caractersticas modernistas que o autor precursoramente apontava. Quais so corretas? a) Apenas I b) Apenas III c) Apenas I e II d) Apenas II e III e) I, II e III

650

PORTUGUS
31. A respeito de Luiz Antnio de Assis Brasil e sua obra correto afirmar que

UFRGS

a) sua obra de estria, Um quarto de lgua em quadro, esboa um retrato pico da imigrao aoriana, destacando os atos de herosmo do protagonista, um campons veterano das guerras europias, na defesa das terras recebidas na medida de rea que d nome ao romance contra os ndios e castelhanos. b) em Manh transfigurada mostra a influncia do realismo mgico e do psicodelismo dos anos 70, visveis nas narrativas curtas marcadas pela alegoria, como alternativa para a denncia da desumanizao promovida pela ditadura militar, e por aluses s viagens provocadas pelo LSD. c) o dramaturgo Qorpo Santo enfrenta a mediocridade da Provncia na defesa de suas idias revolucionrias em As virtudes da casa, romance que inaugura uma linha satrica que viria a ser seguida em O homem amoroso. d) Videiras de cristal traz um retrato quase naturalista da imigrao italiana, tendo como fio condutor a investigao dirigida por um policial de Caxias do Sul sobre o assassinato de um proprietrio de vinhedos, Nann Tamanca, pela esposa do gerente da cooperativa agrcola da regio. e) na trilogia Um castelo no pampa, promove um acerto de contas com o passado oligrquico do Estado, desvendando as contradies do patriarca republicano e liberal que, paradoxalmente, mora num castelo de pedra erguido no meio do pampa e cultiva hbitos aristocrticos. 32. A respeito de trs romances de Tabajara Ruas so feitas as afirmativas seguintes. I - A Guerra do Paraguai o tema central de Os vares assinalados, que rene Bento Gonalves e outros veteranos da Guerra dos Farrapos mais uma vez defendendo a Repblica. II - A volta de um contrabandista a Uruguaiana, onde o espera a inevitvel morte em confronto com seus inimigos, o evento que catalisa as lembranas do narrador de Perseguio e cerco a Juvncio Gutierrez. III - Netto perde sua alma rene as reflexes de um colono sem terra que se encontra preso por ter matado um soldado da Brigada Militar com uma foice. Quais so corretas? a) Apenas I b) Apenas II c) Apenas I e III d) Apenas II e III e) I, II e III

651

PORTUGUS

UFRGS

33. Existe na literatura sul-rio-grandense uma tradio de narrar a participao de personagens ficcionais nas inmeras guerras que ocorreram na histria do Rio Grande do Sul. Relacione as guerras da coluna da esquerda com os personagens de Erico Verissimo da coluna da direita. 1. 2. 3. 4. Guerra dos Farrapos Conflito dos Muckers Revoluo Federalista Coluna Prestes ( ) Licurgo Cambar, em O continente ( ) Torbio Cambar, em O arquiplago ( ) Capito Rodrigo, em O continente

Assinale a alternativa que preenche, correta e respectivamente, de cima para baixo, a coluna da direita. a) 1, 2, 3. b) 1, 3, 4. c) 2, 1, 4. d) 3, 4, 1. e) 4, 3, 2.

Instruo: Para responder s questes de nmeros 14 e 15, utilize a chave abaixo: a) Srgio Faraco b) Charles Kiefer c) Simes Lopes Neto d) Caio Fernando Abreu e) Moacyr Scliar

34. Em Contos gauchescos, utiliza como narrador Blau Nunes, o velho e sbio peo que lutou ao lado de Bento Gonalves na Guerra dos Farrapos.

35. Nos contos escritos em sua juventude, traou um amplo painel da condio judaica em Porto Alegre, mesclando realismo mgico e descrio do quotidiano do bairro Bom Fim.

652

PORTUGUS
Nas questes de 1 a 15, marque: Itens CERTOS, na coluna I. Itens ERRADOS, na coluna II.

UnB

Use, para as devidas marcaes, a Folha de Rascunho e, posteriormente, a Folha de Respostas.

Leia o Texto I, a seguir, para responder s questes de 1 a 5.

MOS DADAS
1 No serei o poeta de um mundo caduco. Tambm no cantarei o mundo futuro. Estou preso vida e olho meus companheiros. Esto taciturnos mas nutrem grandes esperanas. Entre eles, considero a enorme realidade. O presente to grande, no nos afastemos. No nos afastemos muito, vamos de mos dadas. No serei o cantor de uma mulher, de uma histria, no direi os suspiros ao anoitecer, a paisagem vista da janela, no distribuirei entorpecentes ou cartas de suicida, no fugirei para as ilhas nem serei raptado por serafins. O tempo a minha matria, o tempo presente, os homens presentes, a vida presente.
Carlos Drummond de Andrade.

10

13

Questo 1 A compreenso de um texto passa, necessariamente, pela anlise das circunstncias que limitam o assunto em pauta. Com respeito s situaes evidenciadas no texto, julgue os itens abaixo. (0) A circunstncia temporal mais enfatizada do que a espacial. (1) So expresses utilizadas para destacar o mundo caduco (v. 1): preso vida (v. 3), taciturnos (v. 4), grandes esperanas (v. 4) e cartas de suicida (v. 10). (2) O espao tematizado no poema a realidade brasileira urbana, mais especificamente o Rio de Janeiro, local de balas perdidas, onde o poeta viveu seus ltimos anos. (3) As negaes expressas no texto referem-se, principalmente, s condutas consideradas pelo poeta inadequadas vida presente (v. 13).

653

PORTUGUS
Questo 2

UnB

Para apreender a idia principal de um texto necessrio perceber as idias secundrias que do densidade ao propsito fundamental do autor. Considerando que o ttulo resume a idia central do texto, julgue os seguintes itens. (0) (1) (2) (3) preciso esquecer o passado porque ele faz dos homens seres taciturnos. Para enfrentar os desafios do momento atual necessrio a unio de todos os homens. O tempo presente (v. 12) tal qual uma paisagem vista da janela (v. 9). Infere-se do texto um confronto entre duas concepes de poesia: uma poesia praticada como sonho e iluso e outra comprometida com a realidade.

Questo 3 Carlos Drummond de Andrade coloca-se como o poeta (v. 1) do presente (v. 12 e 13). A partir desse ponto de vista, ele faz uma avaliao dos estilos de poca anteriores. Em relao a esse assunto, o autor (0) confirma as tendncias contemporneas, quando diz: Entre eles, considero a enorme realidade (v. 5). (1) refere-se exclusivamente primeira fase do Modernismo, quando fala em no ser o cantor de uma mulher, de uma histria (v. 8). (2) posiciona-se contrrio a uma caracterstica do Romantismo, quando se refere a suspiros ao anoitecer (v. 9). (3) refere-se ao Parnasianismo ao falar em entorpecentes ou cartas de suicida (v. 10). (4) faz aluso ao Naturalismo, na passagem no fugirei para as ilhas nem serei raptado por serafins (v. 12-13). Questo 4 A obra literria em versos tem ritmo e metro distintos do texto em prosa. Com relao s caractersticas do texto literrio e tendo por base o poema Mos Dadas, julgue os itens a seguir. (0) (1) (2) (3) O poema composto por duas estrofes assimtricas. Os versos no obedecem a um esquema mtrico constante nem a um ritmo regular. As estrofes so constitudas por versos brancos. H um paralelismo rtmico, com uma gradao semntica, em o tempo presente, os homens presentes, a vida presente (v. 12-13).

Questo 5 A escolha vocabular, a organizao sinttica e o uso dos tempos e das flexes verbais constituem a estrutura textual do poema. Com o auxlio dessa informao, julgue os itens abaixo. (0) Na expresso enorme realidade (v. 5), o adjetivo reflete a complexidade da realidade brasileira, decorrente da extenso territorial do pas. (1) A expresso mundo caduco (v. 1) corresponde semanticamente a mundo futuro (v. 2). (2) Embora apresentando repetidamente formas verbais no futuro, o poema refere-se ao tempo presente. (3) As formas verbais subjuntivas empregadas nos versos 6 e 7 expressam apelos do poeta.

654

PORTUGUS
Leia o Texto II, a seguir, para responder s questes 6 e 7.
1 Voc no sente, no v Mas eu no posso deixar de dizer, meu amigo, Que uma nova mudana em breve vai acontecer: O que h algum tempo era jovem, novo, hoje antigo E precisamos todos rejuvenescer.
Belchior.

UnB

Questo 6 Uma letra de cano pode ser considerada um poema, na medida em que uma sucesso de idias, artisticamente estruturadas, que convergem para a compreenso de uma mensagem. A partir desse enunciado e segundo o texto de Belchior, julgue os itens abaixo. (0) necessrio o rejuvenescimento constante das pessoas, em face das mudanas que esto por vir. (1) Devido rapidez com que os fatos ocorrem, as pessoas no sofrem nem se deixam afetar pelas mudanas. (2) Diferentemente do texto I, de Drummond, o fragmento do poema de Belchior apresenta rimas ricas, toantes e dispostas de forma emparelhada. Questo 7 Enfocando a estrutura sinttica desse fragmento de letra potica, julgue os itens seguintes. (0) A construo eu no posso deixar de dizer (v. 2) engloba trs oraes porque apresenta trs verbos relacionados a um nico sujeito. (1) A expresso meu amigo (v. 2), que est entre vrgulas, um vocativo, ou seja, refere-se pessoa a quem o poeta se dirige. (2) A orao Que uma nova mudana em breve vai acontecer (v. 3) classifica-se como subordinada substantiva objetiva direta, por exercer a funo de objeto direto do verbo dizer (v. 2). (3) Aparecem quatro adjuntos adverbiais de tempo no quarto verso: algum tempo, jovem, novo e hoje. Questo 8 Com referncia ao sistema de formao vocabular da Lngua Portuguesa, julgue os itens que se seguem. (0) O processo que aproxima mudar de mudana anlogo ao empregado na relao entre andar e andana. (1) Nas relaes sentir / sentimento e antigo / antigamente, h identidade de sufixos. (2) So nomes cognatos de tempo: temporal, temporalidade, contemporneo e temporrio.

655

PORTUGUS
Leia o Texto III, a seguir, para responder s questes de 9 a 11. O SCULO
1 E vs, arcas do futuro, Crislias do porvir, Quando vosso brao ousado Legislaes construir Levantai um templo novo, Porm no que esmague o povo, Mas lhe seja pedestal. Que ao menino d escola, Ao veterano uma esmola... A todos luz e fanal. Basta!... Eu sei que a mocidade o Moiss no Sinai; Das mos do Eterno recebe As tbuas da lei! Marchai! Quem cai na luta com glria, Tomba nos braos da Histria, No corao do Brasil! Moos, do topo dos Andes, Pirmides vastas, grandes, Vos contemplam sculos mil!

UnB

10

14

17

20

Questo 9 Castro Alves, poeta da terceira fase do Romantismo brasileiro, h mais de cem anos comps o poema do qual duas das estrofes finais esto transcritas acima. A partir delas, julgue os itens que se seguem. (0) O texto uma conclamao aos jovens no sentido de que corrijam os erros que vm sendo cometidos ao longo dos sculos. (1) Os versos 1 e 2 apresentam construes metafricas associadas mocidade. (2) Percebe-se, nos versos 15, 16 e 17, um sentimento de desesperana na capacidade que os moos tm de mudar os rumos da histria do Brasil. (3) Na histria da literatura brasileira contempornea, a gerao Violo de Rua, da qual fizeram parte Ferreira Gullar, na poesia, e Geraldo Vandr, na msica popular, assumiu uma postura poltico-literria, no incio da dcada de sessenta, semelhante de Castro Alves, um sculo antes.

656

PORTUGUS
Questo 10

UnB

O fato de o texto haver sido escrito h mais de um sculo no impede o seu entendimento nos dias atuais. Com base na seleo vocabular e na estrutura sinttica, julgue os itens abaixo. (0) A escolha do vocbulo templo (v. 5) permite dupla leitura: Levantai um templo novo e Levantai um tempo novo. (1) Um sentimento de religiosidade perpassa esses versos, o que pode ser confirmado pelos exemplos: Moiss no Sinai (v. 12) e tbuas da lei (v. 14). (2) A grafia de Eterno com inicial maiscula, no verso 13, e a aluso aos Andes, no verso 18, so evidncias da filiao do poeta ao Simbolismo. (3) Os termos luz e fanal (v. 10) significam lume e guia, ambos com sentido de indicar um norte, um horizonte. (4) Os trs ltimos versos da segunda estrofe favorecem o seguinte entendimento: Moos, mil sculos vos contemplam do alto dos Andes, cordilheira que se assemelha a pirmides vastas e grandes. Questo 11 Analisando o texto quanto ao emprego dos sinais de pontuao, julgue os itens a seguir. (0) A pontuao do texto denota o predomnio da funo emotiva da linguagem. (1) O uso reiterado de pontos de exclamao contribui para reforar o tom hiperblico, exagerado, da poesia condoreira, caracterstica pela qual ficou conhecido Castro Alves. (2) O emprego das reticncias, no incio da segunda estrofe, em lugar de outro sinal de pontuao, indica a interrupo do pensamento do poeta. (3) O uso do travesso, nos dois ltimos versos da primeira estrofe, justifica-se apenas como recurso estilstico, pois contraria as regras gramaticais. Leia o Texto IV, a seguir, para responder s questes de 12 a 14. AONDE VOC VAI?
Para quem no sabe aonde vai qualquer caminho serve. S que o indeciso perde muito tempo. E tempo o bem mais escasso. Definir a rota de primeira ajuda a ganhar pontos. A rota o objetivo.
Dad Abi Chahine Squarisi. Correio Braziliense. Cidades, 18/12/96. p. 6 (com adaptaes).

657

PORTUGUS
Questo 12

UnB

Em uma dissertao, h idias que desenvolvem o tema proposto. O texto acima um pargrafo dissertativo. Considerando a adequao dos perodos abaixo para o desenvolvimento da idia central desse pargrafo, julgue os itens a seguir. (0) O mundo atual, marcado por contradies e injustias sociais, oferece mltiplas opes de escolha aos caminhantes. (1) Vencer a indeciso, aproveitar o tempo e acreditar na capacidade humana so maneiras de se aproximar do sucesso. (2) A fugacidade do tempo e a efemeridade da vida so temas atuais que preocupam a populao do planeta, s vsperas do terceiro milnio. (3) Para se alcanarem os objetivos, no h uma frmula predeterminada; enfrentando a caminhada que se aprende a caminhar. (4) No Natal e no Ano Novo, as pessoas tm pensamentos voltados para o futuro, ainda mais quando os dias de amanh se apresentam nebulosos. Questo 13 Tomando como referncia o texto da pgina anterior, quanto estrutura, formao e ao emprego do vocabulrio, julgue os itens seguintes. (0) Segundo as regras gramaticais, correto reescrever o primeiro perodo do texto assim: Para quem no sabe onde vai qualquer caminho serve. (1) S que o indeciso perde muito tempo (li. 2) equivale, semanticamente, a O indeciso s perde muito tempo. (2) Definir a rota de primeira ajuda a ganhar pontos (li. 3) e A definio da primeira rota ajuda a ganhar pontos so construes equivalentes. (3) Em A rota o objetivo (li. 4) e A folha est rota, os termos sublinhados tm estrutura, formao e significado idnticos. Questo 14 Considerando o pargrafo retirado do Correio Braziliense como parte de um texto dissertativo, que deve primar pela objetividade, pela clareza, pela coerncia e pelo tratamento consistente da idia principal escolhida, julgue os itens a seguir. (0) Por apresentar mais de trs idias diferentes, esse pargrafo faz parte do desenvolvimento de uma argumentao. (1) Nesse pargrafo, no h conectivo ou outro elemento de coeso. (2) A coerncia desse pargrafo baseia-se no processo de repetio de algumas palavraschave; por exemplo, tempo e rota.

658

PORTUGUS
Questo 15 Observe a ilustrao abaixo, associando-a com os textos apresentados anteriormente.

UnB

Composio 4 Antologia de textos e ilustraes Colgio Bandeirantes. So Paulo, 1984

Com referncia compreenso textual, julgue os itens a seguir. (0) A ilustrao refora a idia central do texto de Carlos Drummond de Andrade (texto I), devido presena da mo de um homem. (1) Os elementos da ilustrao reforam que uma mudana vai acontecer, aspecto destacado por Belchior (texto II) em sua letra potica. (2) A localizao de um homem no desenho adapta-se ao texto de Castro Alves (texto III), principalmente na passagem: Porm no que esmague o povo (v.6). (3) Relacionando o texto IV ilustrao, infere-se que o indivduo da gravura definiu a sua rota, mas vai demandar tempo o alcance de seu objetivo, porque existem obstculos a serem transpostos.

659

PORTUGUS
REDAO EM LNGUA PORTUGUESA

UnB

Leia os fragmentos abaixo, extrados do livro A vida de Galileu, de Bertolt Brecht, e observe as ilustraes. ... H dois mil anos a humanidade acreditou que o Sol e as estrelas do cu giravam em torno dela. O papa, os cardeais, os prncipes, os sbios, capites, comerciantes, peixeiros e crianas de escola, todos achando que estavam imveis nessa bola de cristal.

O tempo antigo acabou, e agora um tempo novo. J faz cem anos que a humanidade est esperando alguma coisa.

quinhentas mos se movem em conjunto, organizadas de maneira nova.

... agora, veja o que se diz: se as coisas so assim, assim no vo ficar. Tudo se move, meu amigo.

... J se descobriu muita coisa, mas h mais coisas ainda que podero ser descobertas. De modo que tambm as novas geraes tm o que fazer.

Valendo-se das sugestes acima, redija um texto dissertativo, com a extenso mnima de trinta e mxima de sessenta linhas, acerca do tema: A construo do futuro s possvel com a participao de todos.

660

PORTUGUS
LNGUA E LITERATURA

UNESP
F A S E

As questes de nmeros 1 a 3 tomam por base o poema-cano O Amor Velho, Menina, do poeta-msica Tom Z (Antnio Jos Santana Martins, 1936-), e um fragmento da Carta de Guia de Casados (1651), de Francisco Manuel de Melo (1608-1666), reconhecido como um dos maiores escritores portugueses de sua poca.
O AMOR VELHO, MENINA Tom Z O amor velho, velho, velho, velho E, menina, O amor trilha de lenis e culpa, Medo e maravilha. O tempo, a vida, a lida Andam pelo cho, O amor, aeroplanos. O amor zomba dos anos, O amor anda nos tangos, No rastro dos ciganos, No vo dos oceanos, O amor poo onde se despejam Lixo e brilhantes: Oraes, sacrifcios, traies. in: Tom Z. The Hips of Tradition. CD M945118-2, Warner, 1992. CARTA DE GUIA DE CASADOS fragmento Francisco Manuel de Melo Persuado-me, Senhor N., que esta coisa a que o mundo chama amor, no s uma coisa, porm muitas com um prprio nome. Poder bem ser que por isto os antigos fingissem haver tantos amores no mundo, a que davam diversos nascimentos; e tambm pode ser venha daqui que ao amor chamamos amores; pois se ele fora um s, grande impropriedade fora esta. Eu considero dois amores entre a gente: o primeiro aquele comum afeto com que, sem mais causa que a sua prpria violncia, nos movemos a amar, no sabendo o qu, nem o porque amamos. O segundo aquele com que prosseguimos em amar o que tratamos e conhecemos. O primeiro acaba na posse do que se desejou; o segundo comea nela: mas de tal sorte, que nem sempre o primeiro engendra o segundo, nem sempre o segundo procede do primeiro. Donde infiro que o amor que se produz do trato, familiaridade e f dos casados, para ser seguro e excelente, em nada depende do outro amor que se produziu do desejo do apetite, e desordem dos que se amaram antes desconcertadamente; a que, no sem erro, chamamos amores, que a muitos mais empeceram que aproveitaram. MELO, Francisco Manuel de. Carta de Guia de Casados (Coleo Portugal n 36). Porto: Ed. Domingos Barreira, s/d., p. 29.

661

PORTUGUS

UNESP
F A S E

1. Um dos recursos de expresso mais empregados em todas as pocas a anttese. Trata-se de uma figura de nfase estilstica que consiste em aproximar palavras ou expresses de sentido antagnico. Tendo por base as vrias construes antitticas presentes em ambos os textos, a) cite as duas antteses claramente evidenciadas na primeira e segunda estrofes da cano O Amor Velho, Menina, de Tom Z; b) interprete, conforme o pensamento contextualizado no texto de Francisco Manuel de Melo, a frase antittica O primeira [amor] acaba na posse do que se desejou; o segundo comea nela.

2. Na Carta de Guia de Casados, Francisco Manuel de Melo afirma j de incio que o amor no s uma coisa, porm muitas com um prprio nome. Releia atentamente os textos pautados e, a seguir, a) responda em que sentido esta mesma idia de pluralidade do amor est implcita no texto O Amor Velho, Menina; b) apresente um trecho do poema-cano que justifique sua resposta.

3. O estilo barroco de Francisco Manuel de Melo, caracterizado pela insistncia em enfatizar conceitos e argumentos, serve-se de procedimentos sintticos que o tornam um tanto repetitivo, redundante e, por vezes, obscuro, como exemplifica o seguinte perodo: Poder bem ser que por isto os antigos fingissem haver tantos amores no mundo, a que davam diversos nascimentos; e tambm pode ser venha daqui que ao amor chamamos amores; pois se ele fora um s, grande impropriedade fora esta. Examine com ateno o perodo citado e, a) indique a passagem na qual, no entanto, para evitar mais uma repetio, o autor praticou a elipse da conjuno subordinativa integrante; b) reescreva o perodo, evitando outras repeties de palavras ou locues, para torn-lo mais claro e conciso.

662

PORTUGUS

UNESP
F A S E

As questes de nmeros 4 a 6 tomam por base uma citao da Bblia Sagrada e o Soneto 88, de Lus Vaz de Cames (1524?-1580).
JAC ENCONTRA-SE COM RAQUEL Depois disse Labo a Jac: Acaso, por seres meu parente, irs servir-me de graa? Dize-me, qual ser o teu salrio? Ora Labo tinha duas filhas: Lia, a mais velha, e Raquel, a mais moa. Lia tinha olhos baos, porm Raquel era formosa de porte e de semblante. Jac amava a Raquel, e disse: Sete anos te servirei por tua filha mais moa, Raquel. Respondeu Labo: Melhor que eu ta d, em vez de d-la a outro homem; fica, pois, comigo. Assim, por amor a Raquel, serviu Jac sete anos; e estes lhe pareceram como poucos dias, pelo muito que a amava. Disse Jac a Labo: D-me minha mulher, pois j venceu o prazo, para que me case com ela. Reuniu, pois, Labo todos os homens do lugar, e deu um banquete. noite, conduziu a Lia, sua filha, e a entregou a Jac. E coabitaram. (...) Ao amanhecer, viu que era Lia, por isso disse Jac a Labo: Que isso que me fizeste? No te servi por amor a Raquel? Por que, pois, me enganaste? Respondeu Labo: No se faz assim em nossa terra, dar-se a mais nova antes da primognita. Decorrida a semana desta, dar-te-emos tambm a outra, pelo trabalho de mais sete anos que ainda me servirs. Concordou Jac, e se passou a semana desta; ento Labo lhe deu por mulher Raquel, sua filha. (...) E coabitaram. Mas Jac amava mais a Raquel do que a Lia; e continuou servindo a Labo por outros sete anos. (Gnesis, 29, 15-30) Bblia Sagrada (Trad. Joo Ferreira de Almeida.) Rio de Janeiro: Sociedade Bblica do Brasil, 1962.

SONETO 88 Sete anos de pastor Jac servia Labo, pai de Raquel, serrana bela; Mas no servia ao pai, servia a ela, Que a ela s por prmio pretendia. Os dias, na esperana de um s dia, Passava, contentando-se com v-la; Porm o pai, usando de cautela, Em lugar de Raquel lhe dava Lia. Vendo o triste pastor que com enganos Lhe fora assi(m) negada a sua pastora, Como se a no tivera merecida, Comea de servir outros sete anos, Dizendo: Mas servira, se no fora Pera to longo amor to curta a vida! CAMES. Obra Completa. Rio de Janeiro: Aguilar, 1963, p. 298.

4. O racionalismo uma das caractersticas mais freqentes da literatura clssica portuguesa. A logicidade do pensamento quinhentista repercutiu no rigor formal de seus escritores, e no culto expresso das verdades eternas, sem que isto implicasse tolhimento da liberdade imaginativa e potica. Com base nestas observaes, releia os dois textos apresentados e, a) aponte um procedimento literrio de Cames que comprove o rigor formal do classicismo; b) indique o dado da passagem bblica que, por ter sido omitido por Cames, revela a prtica da liberdade potica e confere maior carga sentimental ao seu modo de focalizar o mesmo episdio.

663

PORTUGUS

UNESP
F A S E

5. Em certos contextos, a anteposio do adjetivo ao substantivo costuma revelar traos de afetividade do emissor em relao aos objetos e seres referidos. Damos como exemplo o ttulo de um famoso romance de Lima Barreto: Triste fim de Policarpo Quaresma. Com base nestes comentrios, a) localize no poema de Cames um procedimento que se relacione ao mencionado fato estilstico; b) interprete o efeito semntico dado pela antecipao do adjetivo, no exemplo que voc localizou no item a. 6. Nos seis ltimos versos do poema, Cames, atendendo a necessidades de ritmo e rima, utilizase de variantes alternativas de emprego dos tempos e modos verbais. Com o refinamento de um poeta maior, alcana plena eficcia potica. Levando em considerao estes comentrios, a) aponte duas passagens, nos tercetos referidos, nas quais o poeta empregou o pretrito mais-que-perfeito do indicativo, quando poderia ter-se utilizado de forma verbal em outro tempo ou modo; b) reescreva essas passagens, empregando os verbos de acordo com o uso cotidiano da Lngua Portuguesa.

As questes de nmeros 7 e 8 tomam por base uma Tragdia em um Ato, assinada pelo escritor, tradutor e desenhista Millr Fernandes (1924-), e publicada pela primeira vez em O pif-paf (O Cruzeiro, 1945).
O CAPITALISMO MAIS REACIONRIO Tragdia em um ato Personagens - o patro e o empregado poca - atual ATO NICO

Empregado - Patro, eu queria lhe falar seriamente. H quarenta anos que trabalho na empresa e at hoje s cometi um erro. Patro - Est bem, meu filho, est bem. Mas de agora em diante tome mais cuidado. (Pano bem rpido)
in: FERNANDES, Millr. Trinta Anos de Mim Mesmo. Rio de Janeiro: Nrdica, 1974, p. 15.

7. Essa tragdia-relmpago de Millr Fernandes, denominada O Capitalismo Mais Reacionrio, mobiliza o campo da sugestividade e, num clima de aparente humorismo, libera todo um universo de conceitos, situaes e costumes subentendidos. Releia o texto apresentado e, na sua interpretao, responda: a) o que est subentendido no discurso direto do patro? b) por que este texto pode ser entendido como crtica de costumes?

664

PORTUGUS

UNESP
F A S E

8. A tragdia, no sentido clssico, uma obra fortemente dramtica, inspirada na lenda ou na histria, e que pe em cena personagens envolvidos em situaes que desencadeiam desgraas. Em sua funo potica, destina-se tambm a infundir o terror e a piedade. Considerando esta definio, releia o texto de Millr Fernandes e, a seguir, a) interprete por que apenas esse dilogo entre os dois personagens poderia caracterizar uma tragdia, segundo o autor; b) interprete um sentido conotativo da expresso meu filho, nas palavras do personagem patro.

As questes de nmeros 9 e 10 tomam por base um texto do poeta simbolista brasileiro Alphonsus de Guimaraens (1870-1921).
ERAS A SOMBRA DO POENTE Eras a sombra do poente Em calmarias bem calmas; E no ermo agreste, silente, Palmeira cheia de palmas. Eras a cano de outrora, Por entre nuvens de prece; Palidez que ao longe cora E beijo que aos lbios desce. Eras a harmonia esparsa Em violas e violoncelos: E como um vo de gara Em solitrios castelos. Eras tudo, tudo quanto De suave esperana existe; Manto dos pobres e manto Com que as chagas me cobriste. Eras o Cordeiro, a Pomba, A crena que o amor renova... s agora a cruz que tomba beira da tua cova. (Pastoral aos Crentes do Amor e da Morte, 1923.) in: GUIMARAENS, Alphonsus de. Poesia - I. Rio de Janeiro: Org. Simes, 1955, p. 284.

9. O texto em pauta, de Alphonsus de Guimaraens, apresenta ntidas caractersticas do simbolismo literrio brasileiro. Releia-o com ateno e, a seguir, a) aponte duas caractersticas tipicamente simbolistas do poema; b) com base em elementos do texto, comprove sua resposta.

665

PORTUGUS

UNESP
F A S E

10. A reiterao um procedimento que, aplicado a diferentes nveis do discurso, permite ao poeta obter efeitos de musicalidade e nfase semntica. Para tanto, o escritor pode reiterar fonemas (aliteraes, assonncias, rimas), vocbulos, versos, estrofes, ou, pelo processo denominado paralelismo, retomar as mesmas estruturas sintticas de frases, repetindo alguns elementos e fazendo vrios outros. Tendo em vista estas observaes, a) identifique no poema de Alphonsus um desses procedimentos. b) servindo-se de uma passagem do texto, demonstre o processo de reiterao que voc identificou no item a.

REDAO Para realizar sua redao, oferecemos dois textos de apoio sobre assunto de interesse social e que, no final do primeiro semestre do corrente ano, teve grande repercusso na imprensa brasileira.
JUSTIA MODERNA (Ministro do STF d a largada para desempoeirar o Cdigo Penal)

As pessoas envelhecem. Ser que as leis no? Faltava essa frase no chove no molha da reforma do Cdigo Penal brasileiro (que de 1942). O dono da frase o ministro do Supremo Tribunal Federal (STF), Marco Aurlio Mello. Faltava tambm um voto (de lucidez e modernidade) que a colocasse em prtica. Pois foi o mesmo ministro Marco Aurlio Mello quem assim votou na tera-feira 14: ele concedeu habeas-corpus em favor do mineiro Mrcio de Carvalho, acusado e condenado por ter mantido relaes sexuais com a menor M.A.N. (o fato ocorreu em 1991, quando Carvalho tinha 24 anos e a menina, 12). O Artigo 224 do Cdigo Penal diz que a violncia est sempre presumida quando a mulher tem menos de 14 anos ou seja, a culpa do homem dada a priori, ainda que a relao seja consentida, porque menor de 14 no sabe o que faz no campo sexual. Com olhos no tempo presente e no somente no Cdigo de meio sculo atrs que est caducando, o ministro Marco Aurlio votou de forma inovadora. Depois de uma sucesso vertiginosa de transformaes culturais, hoje podemos dizer que no h mais crianas de 12 anos, e sim moas de 12 anos, submetidas a uma enorme quantidade de informaes, conscientes da realidade e dos riscos que as cercam, disse a ISTO na quinta-feira 16. Mais: a prpria menina declarara na poca que transara com Mrcio porque pintara vontade. Mais ainda: a menina aparentava ter mais idade, j tivera namorados anteriores e concordou com a relao. No estamos fazendo Direito alternativo nem desprezando a ordem jurdica. O Cdigo Penal envelheceu. A norma morta, quem lhe d vida a aplicao prtica, diz o ministro. Essa aplicao faltava. Veio na tera-feira. A sociedade de 1996 agradece. in: Revista ISTO, n 1390, 22.maio/96, p. 12.

666

PORTUGUS
UMA SENHORA DE 12 ANOS Josias de Souza

UNESP
F A S E

H anos raiou no cu fluminense uma nova estrela. O uso do vocbulo fluminense, to fora de moda, denuncia que o Rio de que se falar a seguir um Rio remoto, distante. A frase abre um romance de Jos de Alencar, editado pela primeira vez em 1875. Chama-se Senhora. Suas pginas nos trazem um Rio da Corte, uma cidade de grandes bailes. A estrela que se insinua logo na primeira frase Aurlia Camargo, rf endinheirada busca de noivo. Tinha ela dezoito anos (...), introduziu o escritor. Era virgem e casta, dona de perfeio estaturia e talhe de slfide. O texto de Jos de Alencar segue tricotando adjetivos e despejando-os sobre Aurlia. Mas pararemos por aqui. O que nos importa reter a imagem da mulher desse Rio de 121 anos atrs, casta aos 18. Abra-se agora espao para um texto estalando de novo, escrito neste maio de 1996. Sendo irrestrito o acesso mdia, no se mostra incomum reparar-se a precocidade com que as crianas de hoje lidam, sem embaraos quaisquer, com assuntos concernentes sexualidade (...). Est-se reproduzindo parecer do ministro Marco Aurlio, do STF. Um parecer que libertou um homem, 24, que manteve relaes sexuais com criana de 12 anos, M.A.N. Nos nossos dias no h crianas, mas moas de 12 anos, prossegue o ministro. Precocemente amadurecidas, a maioria delas j conta com discernimento bastante para reagir ante eventuais adversidades. Tomadas assim, ao p da letra, as palavras revelam uma novidade abjeta. A infncia e a adolescncia perderam-se no intervalo de um sculo. A senhora do escritor rejuvenesceu. A menina de 12 uma utopia. Podemos viol-las saciedade. A TV, promscua, absolve-nos a todos. O que falta a Marco Aurlio uma filha, uma senhora de 12 anos, precocemente amadurecida. in: SOUZA, Josias de. Folha de S. Paulo [Cad. 1, p. 2 - Opinio], 26.maio/96.

Proposio Um jovem de doze anos, hoje em dia, acumula e domina muitssimo mais informaes do que, num passado no muito distante, uma pessoa de at mais idade. Por outro lado, em certos estratos sociais, as dificuldades da vida levam os indivduos cada vez mais cedo a assumir o trabalho e a enfrentar situaes caractersticas daquilo que se considera estado adulto. Os dois textos, ao focalizar um mesmo conflito real e polmico, posicionam-se de maneira diferente quanto deciso de um ministro do Supremo Tribunal Federal. O que est em jogo uma questo tica que se projeta no sistema judicirio brasileiro. Voc, certamente, tem opinio a respeito. Sendo assim, escreva uma redao em forma dissertativa sobre o seguinte tema:

O JOVEM DE DOZE ANOS, SEU NVEL DE CONSCINCIA E RESPONSABILIDADE.

667

PORTUGUS
LNGUA E LITERATURA

UNESP
F A S E

As questes de nmeros 1 e 2 baseiam-se no texto Mar Portuguez , do modernista Fernando Pessoa (1888-1935).
MAR PORTUGUEZ Fernando Pessoa mar salgado, quanto do teu sal So lgrimas de Portugual! Por te cruzarmos, quantas mes choraram, Quantos filhos em vo resaram! Quantas noivas ficaram por casar Para que fosses nosso, mar! Valeu a pena? Tudo vale a pena Se a alma no pequena. Quem quere passar alm do Bojador Tem que passar alm da dor. Deus ao mar o perigo e o abysmo deu, Mas nelle que espelhou o cu. (Mensagem, 1934) in: PESSOA, Fernando. Obra Potica, Rio de Janeiro: Aguilar, 1972, p. 82.

1. A vocao nutica dos portugueses e os grandes descobrimentos do passado tornaram o tema do mar bastante freqente na Literatura Portuguesa de todos os tempos. Fernando Pessoa, em Mar Portuguez, focaliza o custo que a aventura martima representou em termos de vidas humanas e sofrimentos ao povo de seu pas. Releia com ateno o poema pautado e, a seguir, a) identifique o recurso estilstico por meio do qual, ao operar escolhas nos planos grfico e morfolgico do discurso, o escritor sugere que a aventura nutica portuguesa refere-se ao passado longnquo; b) justifique sua resposta, apresentando dois exemplos dessa mudana empreendida na forma escrita. 2. Examinando cuidadosamente o poema, verifica-se que, em tom pico, grandiloqente e afetivo, a voz enunciadora inclui o prprio povo portugus em sua fala. Tendo em vista esta observao, a) aponte o verso em que, claramente, o eu-poemtico se manifesta como coletivo, e indique a forma pronominal que identifica o destinatrio dessa voz coletiva; b) a quem especificamente se dirige essa voz coletiva, e por meio de que recurso sinttico o faz?

668

PORTUGUS

UNESP
F A S E

As questes de nmeros 3 e 4 tomam por base um trecho do romance A Moreninha, do romntico Joaquim Manuel de Macedo (1820-1882).
CARTA DE FABRCIO A AUGUSTO fragmento Ah! maldito crioulo... estava-lhe o todo dizendo o para que servia!... Pinta na tua imaginao, Augusto, um crioulo de 16 anos, todo vestido de branco com uma cara mais negra e mais lustrosa do que um botim envernizado, tendo, alm disso, dois olhos belos, grandes, vivssimos e cuja esclertica era branca como o papel em que te escrevo, com lbios grossos e de ncar, ocultando duas ordens de finos e claros dentes, que fariam inveja a uma baiana; d-lhe a ligeireza, a inquietao e rapidez de movimentos de um macaco e ters feito idia desse diabo de azeviche, que se chama Tobias. No me foi preciso cham-lo: bastou um movimento de olhos para que o Tobias viesse a mim, rindo-se desavergonhadamente. Levei-o para um canto. Tu pertences quelas senhoras que esto no camarote, a cuja porta te encostavas? ... perguntei. Sim, senhor, me respondeu ele, e elas moram na rua de... n... ao lado esquerdo de quem vai para cima. E quem so?... So duas filhas de uma senhora viva, que tambm a est, e que se chama a Ilma. sra. d. Luiza. O meu defunto senhor era negociante e o pai de minha senhora padre. Como se chama a senhora que est vestida de branco? A sra. d. Joana... tem 17 anos, e morre por casar. Quem te disse isso?... Pelos olhos se conhece quem tem lombrigas, meu senhor!... Como te chamas? Tobias, escravo de meu senhor, crioulo de qualidade, fiel como um co e vivo como um gato. O maldito do crioulo era um clssico a falar portugus. Eu continuei. Hs de me levar um recado sra. d. Joana. Pronto, lesto e agudo, respondeu-me o moleque. Pois toma sentido. No precisa dizer duas vezes. Ouve. Das duas uma: ou poders falar com ela hoje, ou s amanh... Hoje... agora mesmo. Nestas coisas Tobias no cochila: com licena de meu senhor, eu c sou doutor nisto; meus parceiros me chamam orelha de cesto, p de coelho e boca de taramela. V dizendo o que quiser, que em menos de 10 minutos minha senhora saber tudo; o recado de meu senhor uma carambola que, batendo no meu ouvido vai logo bater no da senhora d. Joaninha. Pois dize-lhe que o moo que se sentar na ltima cadeira da 4 coluna da superior, que assoarse com um leno de seda verde, quando ela para ele olhar, se acha loucamente apaixonado de sua beleza etc., etc., etc. Sim, senhor, eu j sei o que se diz nessas ocasies: o discurso fica por minha conta. E amanh, ao anoitecer, espera-me na porta de tua casa. Pronto, lesto e agudo, repetiu de novo o crioulo. Eu recompensar-te-ei, se fores fiel. Mais pronto, mais lesto e mais agudo! Por agora toma estes cobres. Oh, meu senhor! prontssimo, lestssimo e agudssimo. MACEDO, Joaquim Manuel de. A Moreninha. 5a. ed., So Paulo: tica, 1973, p. 18-19.

669

PORTUGUS

UNESP
F A S E

3. Esta cena de A Moreninha descreve, por meio de uma carta, como Fabrcio conheceu Joana, no Teatro So Pedro de Alcntara. A fluncia e vivacidade de estilo de personagem Fabrcio, que pede a Augusto pinte a cena na imaginao, imprimem ao texto um conjunto de efeitos tpicos da prpria teatralidade. Com base neste comentrio, aponte e explique o funcionamento de a) pelo menos um elemento do enunciado que revela essa pintura teatralizada da cena; b) um procedimento da enunciao que revele a proximidade do texto com a linguagem especfica do teatro. 4. No fragmento que lhe apresentamos, Joaquim Manuel de Macedo transfere ao personagem Fabrcio a viso racista e o preconceito de cor vigente na sociedade brasileira de sua poca. Nessa linha depreciativa, so vrias as caracterizaes que definem Tobias em sua figura e personalidade. Releia o texto, e a) explique como se do as caractersticas de figura e personalidade do jovem escravo; b) justifique sua resposta, apontando pelo menos um exemplo de cada caso. 5. Tobias configura-se em cena como muito conversador e ladino, um misto de esperteza e submisso. Como frisara Fabrcio, era um clssico a falar portugus. A progressiva satisfao desse personagem, em relao recompensa almejada, se exprime com base numa seriao e gradao de formas adjetivas. Em vista do exposto, a) aponte trs passagens em que, manipulando repetidamente o uso de adjetivos, Tobias revelase interesseiro, e se entusiasma com as recompensas de Fabrcio; b) identifique o grau do adjetivo utilizado pelo escravo na ltima frase de seu dilogo.

A questo nmero 6 pe em pauta o poema Epitfio para um Banqueiro, do poeta contemporneo Jos Paulo Paes (1926-).
EPITFIO PARA UM BANQUEIRO negcio ego cio 0 in: PAES, Jos Paulo. Anatomias. So Paulo: Cultrix, 1967, p.-17.

6. Este Epitfio para um Banqueiro enfoca um tema literrio bastante atual: o egosmo, a solido do indivduo, a falta de comunicao que o leva a fechar-se nos limites de sua prpria existncia e, conseqentemente, ver o mundo sempre deformado por uma viso individualista. Tomando por base estas observaes, a) faa uma descrio do plano semntico-visual do texto, de modo a revelar sua compreenso do poema como um epitfio; b) aponte o signo que, numa das linhas do poema, demarca a caracterstica do indivduo como ser em si, exclusivista e isolado.

670

QUMICA

CESGRANRIO

1. Duas substncias de odores bem distintos curiosamente tm frmula molecular idntica C6H12O2 o que caracteriza o fenmeno da isomeria. Os odores e as substncias citadas so responsveis, respectivamente, pelo mau cheiro exalado pelas cabras: - CH3 - CH2 - CH2 CH2 - CH2 - COOH - e pela essncia do morango: - CH3 - COO - CH2 - CHCH3 - CH3. O tipo de isomeria que se verifica entre as duas substncias : a) de cadeia. b) de funo. c) de posio. d) de compensao. e) tautomeria.

2. Certa marca de vinagre indica em seu rtulo 6% em massa por volume de cido actico (etanico). Sabendo-se que ele se encontra 2% ionizado, o seu pH ser: (Dado: log 2 = 0,3) a) 0,7 b) 1,3 c) 1,7 d) 2,3 e) 5,7

3. A partir da dcada de 40, quando McMillan e Seaborg obtiveram em laboratrio os primeiros elementos transurnicos (NA > 92), o urnio natural foi usado algumas vezes para obter tais elementos. Para tanto, ele era bombardeado com ncleos de elementos leves. Na obteno do Plutnio, do Califrnio e do Frmio as tramutaes ocorreram da forma abaixo:
238 92

U + 2 He U+6 C U+
16 8 12

239 94

Pu + A Cf + B

238 92

245 98

238 92

250 100

Fm + C

() () ()
1 0

1 0

n
1 0

Sendo assim, os valores de A, B e C que indicam as quantidades de nutrons obtidas so, respectivamente: a) 1, 4 e 5. b) 1, 5 e 4. c) 2, 4 e 5. d) 3, 4 e 5. e) 3, 5 e 4.

4. De acordo com Linus Pauling, uma substncia considerada inica quando a diferena entre os valores das eletronegatividades de seus componentes igual ou superior a 1,7. No entanto, por outros motivos, algumas substncias no so inicas mesmo obedecendo a esta regra. Analise as substncias apresentadas abaixo. I. NaCl II. HF III. Fe2O3 IV. K3N V. WO3

Pode-se afirmar que no so inicas: a) a I e a II apenas. b) a I e a V apenas. c) a II e a IV apenas. d) a II e a V apenas. e) a III e a V apenas.

671

QUMICA

CESGRANRIO

5. Durante este ano, os jornais noticiaram que a populao de baixa renda fez uso de um leo encontrado em um depsito junto a transformadores de alta tenso. Este leo, denominado ASCAREL, uma mistura de compostos do tipo policloreto de bifenilo (PCB). Tais substncias sintticas contm entre 20% e 70% de cloro e, no homem, podem causar doenas irreversveis no fgado, bronquites crnicas e irritao da pele. Suas snteses podem ser feitas atravs da clorao do bifenilo, como demonstra a equao abaixo:

O nmero de tomos de cloro por molcula existente em um PCB de massa molecular = 361, que contenha 59% em massa de cloro, : a) 3 b) 4 c) 5 d) 6 e) 7

6. Considere as seguintes reaes orgnicas:

Podemos classific-las, respectivamente, como: a) b) c) d) e) adio, substituio, eliminao. adio, eliminao, substituio. eliminao, adio, substituio. eliminao, substituio, adio. substituio, adio, eliminao.

7. Tratando-se o fsforo branco (P4) com soluo aquosa de cido ntrico (HNO3) obtm-se cido fosfrico e monxido de nitrognio, segundo a equao qumica equilibrada: 3P4 + 20HNO3 + 8H2O 12 H3PO4 + 20NO Os agentes oxidante e redutor desta reao so, respectivamente: a) P4 e HNO3 b) P4 e H2O c) HNO3 e P4 d) H2O e HNO3 e) H2O e P4

672

QUMICA

CESGRANRIO

8. O fabricante de bebidas alcolicas obrigado a indicar, nos rtulos dos frascos, os teores do lcool nelas contido. Isso feito atravs de uma porcentagem de volume denominada Graus Gay-Lussac (GL). Por exemplo: 20 GL indica que a porcentagem de lcool de 20% em volume. Sabendo-se que o grau alcolico de um certo whisky de 46 GL, qual a massa, em gramas, de xido de clcio (CaO) necessria para retirar toda a gua de 1 (um) litro dessa bebida? (Considere a equao CaO + H2O Ca(OH)2, sendo a densidade da gua = 1,0 g/ml). a) 168 b) 336 c) 672 d) 840 e) 1.680

9. Um alceno X foi oxidado energicamente pela mistura sulfomangnica (KMnO4 + H2SO4). Os produtos da reao foram butanona e cido metilpropanico. Logo, o alceno X : a) b) c) d) e) 2-metil, 3-hexeno. 3-metil, 3-hexeno. 2,4 dimetil, 3-hexeno. 2,5 dimetil, 3-hexeno. 3,5 dimetil, 3-hexeno.

10. Associe os nomes comerciais de alguns compostos orgnicos e suas frmulas condensadas na coluna da esquerda com os nomes oficiais na coluna da direita. III III IV Formol (CH2O) Acetileno (CH CH) Vinagre (CH3 - COOH) Glicerina (CH2OH-CHOH-CH2OH) (P) Propano triol (Q) cido etanico (R) Metanal (S) Fenil amina (T) Etino

A associao correta entre as duas colunas : a) b) c) d) e) I - P, I - Q, I - Q, I - R, I - S, II - T, II - R, II - T, II - T, II - R, III - Q, III - T, III - P, III - Q, III - P, IV - R. IV - P. IV - R. IV - P. IV - Q.

11. De acordo com a teoria cido-base de Brnsted Lowry, cido toda substncia capaz de ceder prtons (H+). Assim, na srie de compostos orgnicos ao lado, a seqncia correta em ordem decrescente de acidez : a) b) c) d) e) I > II > III > IV II > I > IV > III III > IV > I > II IV > III > I > II IV > III > II > I

673

QUMICA

CESGRANRIO

12. Em laboratrio, um aluno misturou 10 ml de uma soluo de HCl 2N com 20 ml de uma soluo X N do mesmo cido em um balo volumtrico de 50 ml de capacidade. Em seguida, completou o volume do balo volumtrico com gua destilada. Na total neutralizao de 10 ml da soluo final obtida, foram consumidos 5 ml de soluo de NaOH 2N. Assim, o valor de X : a) 1,0 N b) 1,5 N c) 2,0 N d) 2,5 N e) 3,0 N

13. Uma das utilizaes da Classificao Peridica dos Elementos o estudo comparativo de suas propriedades. Dos elementos abaixo, aquele que, ao mesmo tempo, mais denso que o BROMO e tem maior potencial de ionizao do que o CHUMBO o: a) N b) O c) Ge d) Fe e) Kr

14. O gs hilariante (N2O) tem caractersticas anestsicas e age sobre o sistema nervoso central, fazendo com que as pessoas riam de forma histrica. Sua obteno feita a partir da decomposio trmica do nitrato de amnio (NH4NO3), que se inicia a 185C, de acordo com a seguinte equao: NH4NO3(s) N2O(g) + 2H2O(g) No entanto, o processo exotrmico e a temperatura fornecida age como energia de ativao. Sabe-se que as formaes das substncias N2O, H2O e NH4NO3 ocorrem atravs das seguintes equaes termoqumicas: N2(g) + 1/2O2(g) N2O(g) - 19,5 kcal H2(g) + 1/2O2(g) H2O(g) + 57,8 kcal N2(g) + 2H2(g) + 3/2O2(g) NH4NO3(s) + 87,3 kcal A quantidade de calor liberada, em Kcal, no processo de obteno do gs hilariante : a) 8,8 b) 17,6 c) 28,3 d) 125,6 e) 183,4

15. Os principais poluentes do ar nos grandes centros urbanos so o gs sulfuroso (SO2) e o monxido de carbono (CO). O SO2 proveniente das indstrias que queimam combustveis fsseis (carvo e petrleo). J o CO provm da combusto incompleta da gasolina em veculos automotivos desregulados. Sabendo-se que o SO2 (causador da chuva cida) e o CO (causador de inibio respiratria) so xidos, suas classificaes so, respectivamente: a) anftero e neutro. b) bsico e cido. c) cido e anftero. d) cido e bsico. e) cido e neutro.

16. Os elementos do grupo IV B da Classificao Peridica tm grande facilidade para atuar com nmeros de oxidao +3 e +4. Um destes elementos, o Titnio, forma xidos estveis com 3+ frmulas Ti2O3 (inico) e TiO2 (molecular). No xido inico, o on Ti tem como distribuio eletrnica, em nveis de energia: a) 2 - 8 - 10 - 5 b) 2 - 8 - 10 - 3 c) 2 - 8 - 10 - 2 d) 2 - 8 - 8 - 1 e) 2 - 8 - 9

674

QUMICA

CESGRANRIO

17. As pilhas alcalinas entraram em moda recentemente e so usadas em quase tudo que exige um trabalho contnuo e duradouro, desde relgios de pulso at calculadoras eletrnicas. Uma destas pilhas mais usadas a de nquel/cdmio, que chega a ter uma durao maior do que a da bateria de automvel e ainda pode ser recarregada vrias vezes. Ela constituda pelo metal cdmio, hidrxido de nquel III e uma pasta de hidrxido de potssio. Considere que os potenciais-padro de reduo so: Cd (s) + 2e Cd (s) - 0,4V +3 +2 Ni (s) + 1e Ni (s) + 1,0V Entre as opes abaixo, indique a que apresenta o sentido do fluxo de eltrons e a fora eletromotriz da pilha nquel-cdmio. a) b) c) d) e) Do eletrodo de cdmio para o eletrodo de hidrxido de nquel III + 1,4V Do eletrodo de cdmio para o eletrodo de hidrxido de nquel III + 1,6 V Do eletrodo de cdmio para o eletrodo de hidrxido de nquel III + 2,4 V Do eletrodo de hidrxido de nquel III para o eletrodo de cdmio + 1,4 V Do eletrodo de hidrxido de nquel III para o eletrodo de cdmio + 2,4 V
+2 O

18. Os gases provenientes da eletrlise da gua do mar foram recolhidos em um recipiente fechado de capacidade igual a 5 litros. A mistura recolhida apresentava 7,5 moles de hidrognio e 5 moles de cloro, que reagiram de acordo com a seguinte equao: H2(g) + Cl2(g) 2HCl(g). Sendo assim, a constante de equilbrio, em termos de concentrao molar (Kc) a uma dada temperatura em que 5 moles de HCl(g) foram obtidos, ser: a) 1,0 b) 2,0 c) 2,5 d) 4,0 e) 5,0

19. O gs de cozinha, tambm chamado de gs liquefeito de petrleo (GLP), formado por 50% de propano e 50% de butano. Sabendo-se que uma famlia domiciliada no Rio de Janeiro gastou 49,2 m3 de GLP no ms de setembro, o nmero de molculas de gs butano queimado nesse ms foi: (Dados: volume molar do butano a 27C e 1 Atm = 24,6 litros; temperatura mdia do ms de 23 Setembro = 27C; N de Avogadro = 6,0 x 10 ) a) 6,0 x 10 26 b) 6,0 x 10 25 c) 6,0 x 10
27

d) 3,0 x 10 25 e) 3,0 x 10

26

20. Aps uma aula, um professor lava as mos com gua e sabo, separando, com isto, todo o giz de suas mos. Tal processo de separao dos componentes da mistura denominado: (Dado: Giz basicamente formado por CaSO4 ) a) dissoluo. b) decantao. c) flotao. d) destilao. e) levigao.

675

QUMICA

FUVEST
F A S E

1. Em um frasco foram colocados soluo aquosa de HCl e raspas de zinco para gerar H2, gs pouco solvel em gua. Para se recolher esse gs, o melhor arranjo experimental :

2. Objetos de prata escurecidos (devido principalmente formao de Ag2S) podem ser limpos eletroquimicamente, sem perda da prata, mergulhando-os em um recipiente de alumnio contendo soluo quente de bicarbonato de sdio. Neste processo, a prata em contato com o Ag2S atua como catodo e o alumnio como anodo de uma pilha. A semi-reao que ocorre no catodo pode ser representada por: a) Ag2S > 2Ag + S 2 b) Ag2S > 2e > 2Ag + S 2 c) Ag2S > 2Ag + S + 2e
+ 2

d) Ag2S + 2e > 2Ag + S e) Ag2S > 2Ag + S

3. O estudo cintico, em fase gasosa, da reao representada por NO2 + CO > CO2 + NO mostrou que a velocidade da reao no depende da concentrao de CO, mas depende da concentrao de NO2 elevada ao quadrado. Esse resultado permite afirmar que a) b) c) d) e) o CO atua como catalisador. o CO desnecessrio para a converso de NO2 em NO. O NO2 atua como catalisador. a reao deve ocorrer em mais de uma etapa. a velocidade da reao dobra se a concentrao inicial de NO2 for duplicada.

4. Uma amostra de 0,212 g de uma haleto de alquila, quando vaporizada, apresentou um volume de 82 mL a 227C e 1 atm. Uma possvel frmula desse haleto
Volume molar de gs a 227C e 1 atm = 41 L/mol massas molares (g/mol) H=1 Cl = 35 C = 12 Br = 80

a) C3H7Cl b) C3H7Br c) C4H9Cl

d) C5H11Cl e) C5H11Br

676

QUMICA

FUVEST
F A S E

5. Em uma indstria um operrio misturou, inadvertidamente, polietileno (PE), poli (cloreto de vinila) (PVC) e poliestireno (PS), limpos e modos. Para recuperar cada um destes polmeros utilizou o seguinte mtodo de separao: jogou a mistura em um tanque contendo gua 3 (densidade = 1,00 g/cm ) separando, ento, a frao que flutuou (frao A) daquela que foi ao fundo (frao B). A seguir, recolheu a frao B, secou-a e a jogou em outro tanque contendo 3 soluo salina (densidade = 1,10 g/cm ), separando o material que flutuou (frao C) daquele que afundou (frao D). As fraes A, C e D eram, respectivamente, a) PE, PS e PVC b) PS, PE e PVC c) PVC, PS e PE d) PS, PVC e PE e) PE, PVC e PS

6. Em condies industrialmente apropriadas para se obter amnia, juntaram-se quantidades estequiomtricas dos gases N2 e H2 N2(g) + 3H2(g) 2NH3(g)

Depois de alcanado o equilbrio qumico, uma amostra da fase gasosa poderia ser representada corretamente por:

7. Uma mistura de xido de cobre (II) e carvo em p foi aquecida usando-se a aparelhagem esquematizada abaixo. Observou-se, aps algum tempo, que a gua de cal, inicialmente lmpida, apresentou slido branco em suspenso. No interior do tubo apareceram grnulos metlicos avermelhados. Qual a equao qumica que representa a transformao ocorrida nesse aquecimento? a) 2CuO + C > 2Cu + CO2 b) CuO + C > Cu + CO c) 2CuO + C > Cu2O + CO d) 2Cu2O + C > 4Cu + CO2 e) Cu2O + C > 2Cu + CO

677

QUMICA
8. Os pratos A e B de uma balana foram equilibrados com um pedao de papel em cada prato e efetuou-se a combusto apenas do material contido no prato A. Esse procedimento foi repetido com palha de ao em lugar de papel. Aps cada combusto observou-se

FUVEST
F A S E

com papel a) b) c) d) e) A abaixo de B A acima de B A acima de B A abaixo de B

com palha de ao A abaixo de B A acima de B A abaixo de B A e B no mesmo nvel

A e B no mesmo nvel A e B no mesmo nvel

9. A embalagem de um sal de cozinha comercial com reduzido teor de sdio, o chamado sal light, traz a seguinte informao: Cada 100 g contm 20 g de sdio . Isto significa que a porcentagem (em massa) de NaCl nesse sal aproximadamente igual a a) 20 b) 40 c) 50 d) 60 e) 80
massas molares (g/mol) Na = 23 NaCl = 58

10. A reduo da acidez de solos, imprprios para algumas culturas, pode ser feita tratando-os com a) gesso (CaSO4 1/2 H2O) b) salitre (NaNO3) c) calcrio (CaCO3) d) sal marinho (NaCl) e) slica (SiO2)

11. Sabes so usualmente obtidos pela reao de steres de cidos graxos com soda custica. As matrias-primas destas substncias so, respectivamente, a) petrleo e sal-gema. b) melao de cana e cal. c) gordura animal e gua mineral. d) leo vegetal e salmoura. e) gordura animal e cal.

678

QUMICA
12. Os seguintes dados foram obtidos analisando-se amostras de xidos de nitrognio.

FUVEST
F A S E

Amostra I II III

massa da amostra (g) 0,100 0,300 0,400

massa de nitrognio (g) 0,047 0,141 0,147

massa de oxignio 0,053 0,159 0,253

Pela anlise desses dados conclui-se que a) b) c) d) e) as amostras I, II e III so do mesmo xido. apenas as amostras I e II so do mesmo xido. apenas as amostras I e III so do mesmo xido. apenas as amostras II e III so do mesmo xido. as amostras I, II e III so de diferentes xidos.

13. Na tabela abaixo dada a composio aproximada de alguns constituintes de trs alimentos:

Composio (% em massa) Alimento I II III Protenas 12,5 3,1 10,3 Gorduras 8,2 2,5 1,0 Carboidratos 1,0 4,5 76,3

Os alimentos I, II e III podem ser, respectivamente, a) b) c) d) e) ovo de galinha, farinha de trigo e leite de vaca. ovo de galinha, leite de vaga e farinha de trigo. leite de vaca, ovo de galinha e farinha de trigo. leite de vaca, farinha de trigo e ovo de galinha. farinha de trigo, ovo de galinha e leite de vaca.

679

QUMICA
14.

FUVEST
F A S E

Alcano etano propano butano

Frmula C2H6 C3H8 C4H10

Calor de combusto * kJ/mol de alcano 1428 2044 2658

* reagentes e produtos gasosos a 25C e 1 atm

Determinou-se o calor de combusto* de um alcano obtendo-se o valor 3886 KJ/mol de alcano. Utilizando os dados da tabela, conclui-se que este alcano deve ser um a) pentano b) hexano c) heptano d) octano e) nonano

15. A figura abaixo representa trs etapas de uma experincia em que ocorre reao qumica entre dois gases incolores (NO e O2), mesma temperatura e presso e contidos em seringas separadas. Aps a mistura, houve consumo total dos reagentes com formao de um produto gasoso marrom, nas mesmas condies de presso e temperatura dos reagentes.

Se a reao qumica que ocorreu for representada por m NO + n O2 > p NxOy os coeficientes estequiomtricos, m, n e p devero ser, respectivamente, a) 2, 1, 1 b) 4, 1, 2 c) 1, 2, 1 d) 4, 3, 2 e) 2, 1, 2

680

QUMICA
16. Uma das reaes que ocorrem na obteno de ferro a partir da hematita : Fe2O3(s) + 3CO(g) > 3CO2(g) + 2Fe(s)

FUVEST
F A S E

O calor liberado por esta reao cerca de 29 kJ por mol de hematita consumida. Supondo que a reao se inicie temperatura ambiente (25C) e que todo esse calor seja absorvido pelo ferro formado (o qual no chega a fundir), a temperatura alcanada por este da ordem de a) 1 x 10 C 2 b) 2 x 10 C 2 c) 6 x 10 C
2

d) 1 x 10 C 3 e) 6 x 10 C

Calor requerido para elevar de 1C a temperatura de um mol de ferro = 25J/(mol C)

17. Em um acidente, um caminho carregado de soluo aquosa de cido fosfrico tombou derramando cerca de 24,5 toneladas dessa soluo no asfalto. Quantas toneladas de xido de clculo seriam necessrias para reagir totalmente com essa quantidade de cido? a) 7,5 b) 11,2 c) 16,8 d) 21,0 e) 22,9
Porcentagem em massa do H3PO4 na soluo = 80% Massas polares (g/mol): H3PO4 = 98 CaO = 56

18.

Um tubo de ensaio contm duas camadas lquidas, incolores e imiscveis; a superior gua e a inferior clorofrmio. Nesse tubo efetuam-se, seqencialmente, as operaes descritas abaixo: I - adio de pequenas quantidades de A e de hidrxido de sdio aquoso, agitao e repouso. II - adio de cido clordrico em quantidade suficiente para que, aps agitao e repouso, haja descoramento total de uma das camadas. III - adio de hidrxido de sdio aquoso em quantidade suficiente para que, aps agitao e repouso, haja descoramento total de uma das camadas. Ao final de cada uma das operaes I, II e III, a camada aquosa apresenta-se, respectivamente: a) azul, incolor e incolor. b) azul, vermelha e incolor. c) incolor, vermelha e incolor. d) vermelha, incolor e azul. e) azul, incolor e azul.

681

QUMICA
19.

FUVEST
F A S E

Na montagem acima, dependendo do metal (junto com seus ons) tem-se as seguintes pilhas, cujo catodo (onde ocorre reduo) o cobre: pilha cobre-alumnio cobre-chumbo cobre-magnsio cobre-nquel E* (volt) 2,00 0,47 2,71 0,59
* diferena de potencial eltrico nas condies padro

Nas condies padro e montagem anloga, a associao que representa uma pilha em que os eletrodos esto indicados corretamente a) b) c) d) e) catodo nquel magnsio magnsio alumnio chumbo anodo chumbo chumbo alumnio nquel alumnio

20. A substncia A, na presena de luz solar, transforma-se na substncia B que, por sua vez, no escuro se transforma em A.

Pelo esquema acima, pode-se afirmar que a) b) c) d) e) h uma interconverso de ismeros. a transformao de A em B libera energia. a luz converte uma acetona em um aldedo. na ausncia da luz, o carter aromtico destrudo. no escuro, um cido carboxlico reduzido a uma cetona.

682

QUMICA
1. O hidrognio usado na fabricao de inmeros produtos importantes: I - H2 + X metanol II - H2 + Y ciclo-hexano III - H2 + Z tripalmitato de glicerila (gordura saturada)

FUVEST
F A S E

H2C OOCC15H31 HC OOCC15H31 H2C OOCC15H31

a) Identifique X, Y e Z. b) Cite um uso para cada um dos produtos: metanol e gordura saturada. 2. O equipamento de proteo conhecido como air-bag, usado em automveis, contm substncias que se transformam, em determinadas condies, liberando N2 que infla um recipiente de plstico. As equaes das reaes envolvidas no processo so: 2NaN3 2 Na + 3 N2

azoteto de sdio 10 Na + 2 KNO3 K2O + 5 Na2O + N2 a) Considerando que N2 gerado nas duas reaes, calcule a massa de azoteto de sdio necessria para que sejam gerados 80 L de nitrognio, nas condies ambiente. b) Os xidos formados, em contato com a pele, podem provocar queimaduras. Escreva a equao da reao de um desses xidos com a gua contida na pele. Dados: Volume molar de gs nas condies ambiente: 25 L/mol massa molar do NaN3: 65 g/mol 3. Para estudar a velocidade da reao que ocorre entre magnsio e cido clordrico, foram feitos dois experimentos a 15C utilizando a mesma quantidade de magnsio e o mesmo volume de cido. Os dois experimentos diferiram apenas na concentrao do cido utilizado. O volume de hidrognio produzido em cada experimento, em diferentes tempos, foi medido a presso e temperatura ambientes. Os dados obtidos foram:
Experimento I II Tempo/min (vol.H2)/cm
3

0 0 0

1 18 28

2 33 49

3 48 60

4 60 62

5 63 63

6 63 63

7 63 63

(vol.H2))/cm

a) Em qual dos experimentos a velocidade da reao foi maior? Justifique com base nos dados experimentais. b) A curva obtida para o experimento I (15C) est no grfico ao lado. Neste mesmo grfico, represente a curva que seria obtida se o experimento I fosse realizado a uma temperatura mais alta. Explique.

683

QUMICA

FUVEST
F A S E

4. Quer-se distinguir uma amostra de p-clorofenol de uma o-nitrofenol, ambos slidos. a) Determinou-se o ponto de fuso de cada amostra, utilizando um termmetro que permite a leitura da temperatura com incerteza de 1C. Foi possvel, com esta medida experimental, distinguir essas amostras ? Explique. b) Em gua, tais fenis (ArOH) apresentam carter cido: ArOH ArO + H
+

Mostre com clculos que a determinao do pH de solues aquosas desses fenis, de concentrao 0,01 mol/L, serviria para identific-las. Dados:
Substncia p-clorofenol o-nitrofenol Ponto de Fuso (C) 43,5 45 Ka* 1 x 10 1 x 10
-9 -7

* Ka = constante de ionizao, em gua

5. Protenas so formadas por vrias cadeias peptdicas que se mantm unidas atravs de ligaes do tipo I, II e III, formando uma estrutura complexa, como a esquematizada abaixo:

a) Explique de que tipo so as ligaes I, II e III assinaladas no esquema da protena. b) Assinale, com um crculo, uma ligao peptdica na protena esquematizada acima. 6. O cido ntrico um importante produto industrial. Um dos processos de obteno fazer passar amnio (NH3) e ar, sob presso, por um catalisador a cerca de 850C, ocorrendo a formao de monxido de nitrognio e gua. O monxido de nitrognio em presena do oxignio do ar se transforma no dixido, que em gua forma cido ntrico (HNO3) e monxido de nitrognio (que reciclado no processo). a) Escreva as equaes balanceadas que representam as diferentes etapas da produo de cido ntrico atravs desse processo. b) O calor envolvido na primeira etapa, ou seja, a oxidao da amnia at o monxido de nitrognio, ajuda a manter o catalisador aquecido. Sendo assim, qual deve ser maior: a soma das energias de ligao dos reagentes ou a soma das energias de ligao dos produtos? Justifique.

684

QUMICA

FUVEST
F A S E

7. A 800C a constante de equilbrio, Kp (expressa em termos de presso parcial em atm), para o equilbrio representado abaixo vale 0,22. CaCO3 (s) CaO (s) + CO2 (g)
-4

Explique o que aconteceria se carbonato de clcio fosse aquecido, a 800C, em recipiente aberto a) na Terra onde a presso parcial do CO2 atmosfrico 3 x 10 atm. b) em Vnus onde a presso parcial do CO2 atmosfrico 87 atm. 8. Adicionando-se soluo aquosa de sal A a uma soluo aquosa de sal B, forma-se um precipitado em uma reao praticamente completa. Para se determinar os coeficientes estequiomtricos dos reagentes, na equao dessa reao, fez-se uma srie de 6 experimentos. Em cada um, a quantidade de A era fixa e igual a 4,0 x 10-3 mol. A quantidade de B era varivel. Os dados destes experimentos esto na tabela abaixo.

Experimento Volume (mL) da soluo do sal B, 0,10 mol/L massa (g) do precipitado formado

1 6,0 0,20

2 12,0 0,40

3 18,0 0,60

4 24,0 0,66

5 30,0 0,66

6 36,0 0,66

a) Calcule as quantidades, em mol, do sal B utilizadas nesses experimentos. b) No quadriculado ao lado, construa o grfico: massa de precipitado versus quantidade, em mol, de sal B. Atravs deste grfico justifique quais devem ser os coeficientes estequiomtricos de A e B.

685

QUMICA
9. Esto representados abaixo quatro esterides:

FUVEST
F A S E

a) Quais dentre eles so ismeros ? Explique. b) Considerando que o colesterol um composto insaturado, que reao poderia ocorrer, em condies apropriadas, se este fosse tratado com bromo (Br2) ?

10. E* (V) HCHO + 2H + 2e


+ + -

CH3OH
-

0,23 0,59 1,23

CH3OH + 2H + 2e O2 + 4H + 4e+

CH4 + H2O 2H2O

E* = potencial de reduo para a semi-reao nas condies padro. Com base nos dados acima, nestas condies, a) mostre que, em meio cido, seria possvel obter metanol a partir de metano e oxignio. b) Escreva a equao balanceada que representaria esta transformao.

686

QUMICA

UFBA
F A S E

QUESTES DE 1 A 7 INSTRUO: Assinale as proposies verdadeiras, some os nmeros a elas associados e marque o resultado na Folha de Respostas.

Questo 1 Alguns anticidos, usados comercialmente para combater a acidez estomacal, contm bicarbonato de sdio. Em relao a esses anticidos, considerando-se as reaes qumicas que ocorrem durante a dissoluo em gua, a ao sobre a acidez estomacal e os compostos envolvidos, pode-se afirmar: (1) (2) (4) (8) (16) (32) O gs liberado, durante a dissoluo, o hidrognio. O bicarbonato de sdio classificado como sal bsico. O bicarbonato de sdio, em soluo aquosa, se hidrolisa, produzindo ons OH (aq). Ocorre uma reao de neutralizao, quando o anticido atua sobre a acidez estomacal. O cido carbnico, em soluo aquosa, um cido forte. A ao do anticido eleva o pH no estmago.

Questo 2 Substncia madeira H2O(s) H2O(l) PVC* Au(s) * Policloreto de vinila Densidade (g/mL) 0,90 0,91 1,00 1,39 19,30

Com base na tabela acima e nos conhecimentos sobre as substncias apresentadas, pode-se afirmar: (1) (2) (4) (8) (16) (32) A densidade uma propriedade que caracteriza uma substncia pura. O PVC flutua na gua. A diferena de densidade entre H20(s) e H2O(l) devida s foras intermolecularees. A elevada temperatura de fuso do ouro justificada pela presena de ligaes covalentes. A gua uma substncia apolar. A celulose o principal componente da madeira.

687

QUMICA
Questo 3

UFBA
F A S E

O diagrama abaixo representa a preparao de 250 mL de caf pelo processo tradicional.

Com base no diagrama e nos conhecimentos sobre as substncias envolvidas, pode-se afirmar: (1) (2) (4) (8) (16) (32) (64) O caf uma substncia pura. Durante a adio de gua quente ao caf em p, ocorre uma extrao. A operao I de filtrao. A operao I classificada como um fenmeno qumico. O sistema C uma soluo. O acar dissociado, ao ser adicionado ao sistema B. Admitindo-se que o acar comercial contenha unicamente sacarose, C12H22O11, sua concentrao, no sistema C, aproximadamente 0,12 mol/L.

Questo 4 Os compostos NO2(g), SO3(g) e CO2(g) interagem com vapor de gua, na atmosfera. Em relao a esses compostos e sua ao sobre o meio ambiente, pode-se afirmar: So classificados como xidos bsicos. Reagem com gua, formando sais cidos. NO2(g) e SO3(g) reagem com gua, formando cido nitroso e cido sulfuroso, respectivamente. A presena de grandes concentraes de NO2(g) e SO3(g), na atmosfera, provoca o fenmeno das chuvas cidas. -8 (16) O sistema aquoso em que a concentrao molar de ons hidrognio 10 mol/L classificado como bsico. (32) CO2 liquefeito utilizado na produo de gelo-seco. (1) (2) (4) (8)

688

QUMICA
Questo 5

UFBA
F A S E

Composto I II III

Frmula molecular (C2H5)2O (C4H9)OH C4H8

Ponto de Ebulio (C) 34,6 117,7 130,0

Solubilidade (g/mL de H2O) 8,0 7,9 baixa solubilidade

As informaes apresentadas na tabela acima e os conhecimentos sobre propriedades dos compostos orgnicos permitem concluir: (1) (2) (4) (8) (16) (32) (64) I um ter. II o butanal. III apresenta, somente, tomos de carbono saturados. III pode ser obtido a partir da desidratao de II, em meio cido. II mais solvel em gua que III, devido diferena de polaridade. A presena de pontes de hidrognio justifica a diferena do ponto de ebulio entre I e II. I, II e III so compostos cclicos.

Questo 6 Numa lmina de zinco, em contacto com o ar atmosfrico, observou-se a formao de uma substncia A, de colorao branca. Em seguida, a lmina de zinco foi exposta a uma atmosfera de hidrognio. Com base nessas informaes e nos conhecimentos sobre propriedades peridicas dos elementos qumicos, pode-se afirmar: (1) O nmero de oxidao do zinco metlico 2 . (2) A substncia A xido de zinco. (4) A formao de A e sua reao com o hidrognio so representadas, respectivamente, por: 2Zn(s) + O2(g) 2ZnO(s) ZnO(s) + H2(g) Zn(s) + H2O(g) (8) O hidrognio atuou como agente oxidante. (16) Durante a formao de A, o zinco perdeu dois eltrons. (32) O primeiro potencial de ionizao do zinco maior que o segundo.
+

689

QUMICA
Questo 7 Composto CH4(g) H2O(g) CO(g)
1 CH4(g) + H2O(g) > CO(g) + 3H2(g)

UFBA
F A S E

Entalpia de 0 formao H f (kcal/mol) 17,9 57,8 26,4

Com base na reao acima, nas informaes da tabela e nos conhecimentos sobre compostos orgnicos e equilbrio qumico, pode-se afirmar: (1) CH4 apresenta estrutura geomtrica tetradrica. (2) A reao exotrmica. (4) Aumenta-se a converso do CH4(g), no equilbrio, elevando-se a temperatura do sistema reacional. (8) A quantidade de calor envolvida na reao igual a 102,1 kcal /mol de CH4(g). (16) A reao, em equilbrio, no sofre influncia da presso. (32) Adicionando-se vapor de gua ao sistema reacional, o equilbrio ser deslocado no sentido 2. (64) O calor envolvido numa reao qumica devido diferena entre as energias de formao e quebra de ligaes qumicas.

QUESTES DE 1 A 8 INSTRUO: Questo 1 A osteoporose, aumento anormal da porosidade dos ossos, uma doena comum em mulheres mais idosas, que apresentam deficincia de absoro de clcio pelo organismo. O clcio pode ser adicionado dieta, administrando-se comprimidos que contenham: carbonato de clcio, sulfato de clcio ou fosfato de clcio.
(BODNER, George M. PARDUE, Harry L. Chemistry: an experimental science. 2 ed. New York: John Wiley, 1995. p. 106. Traduzido e adaptado.)

Assinale as proposies verdadeiras, some os nmeros a elas associados e marque o resultado na Folha de Respostas.

Em relao a esses sais de clcio e compostos correlatos, correto afirmar: (1) (2) (4) (8) (16) Os carbonatos de metais alcalino-terrosos so muito solveis em gua. O carbonato supre o organismo com mais ons clcio do que o sulfato e o fosfato. Por decomposio trmica, o carbonato de clcio libera dixido de carbono. O raio inico do ction Ca2+ maior que o raio atmico do clcio. Massas iguais de carbonato de clcio e de sulfato de clcio possuem o mesmo nmero de moles. (32) As frmulas qumicas do fosfato de clcio e fosfito de clcio so, respectivamente, Ca(PO3)2 e Ca3(PO4)2.

>
2

F A S E

2 A

P R O V A

690

QUMICA
Questo 2 V2O5(s) + Cl3(g) VOCl3(s) + O2(g)

UFBA
F A S E

Considerando-se a reao qumica, no balanceada, representada acima e os compostos nela envolvidos, correto afirmar: (1) O Cl2 uma molcula polar. (2) A soma dos menores coeficientes estequiomtricos inteiros que balanceiam a reao igual a 15. (4) O Cl2 o agente redutor. (8) Com 4 moles de V2O5 obtm-se 6 moles de 02. (16) O VOCl3(s) apresenta retculo cristalino. (32) Toda reao qumica eletricamente neutra. Questo 3

2 A

P R O V A

A figura ao lado representa um experimento sobre difuso de gases em que dois recipientes esto interligados por um tubo de vidro com duas vlvulas. Durante a difuso dos gases, forma-se, no tubo, um anel branco de cloreto de amnio.

Em relao a esse experimento e s propriedades qumicas das substncias envolvidas, podese afirmar: (1) A velocidade de difuso dos gases diretamente proporcional ao seus pesos moleculares. (2) Abrindo-se simultaneamente as duas vlvulas, o anel de cloreto de amnio formar-se- a aproximadamente 28 cm do ponto A. (4) A amnia, em soluo aquosa, uma base, segundo o conceito de Brnsted-Lowry. (8) O HCl(aq) ioniza-se moderadamente. (16) O on amnio apresenta duas ligaes covalentes dativas. (32) As molculas de HCl apresentam interaes do tipo dipolo-dipolo.

691

QUMICA
Questo 4

UFBA
F A S E

O on bicarbonato, em soluo aquosa, estabelece simultaneamente os seguintes equilbrios: I. HCO3 (aq) + H2O(l) H3O (aq) + CO
+ 2(aq) -

Ka = 4,7 x 10 Kb = 2,2 x 10

-11

II. HCO3 (aq) + H2O(l) H2CO3(aq) + OH (aq)

-8

2 A

Com base na informao e equaes acima, correto afirmar: (1) (2) (4) (8) (16) O equilbrio I predominante. O on bicarbonato age como cido e base de Brnsted-Lowry. Em II, ocorre a hidrlise do on bicarbonato. A soluo aquosa de ons HCO 3 apresenta pH > 7. A adio de HCO3 (aq) ao sistema em equilbrio desloca-o, aumentando a concentrao de HCO 3 (aq). (32) O cido carbnico, em soluo aquosa, monoprtico. P R O V A

Questo 5 Presso de vapor (mmHg) (25C) 545 65 24

Com base nos dados apresentados na tabela acima e nos conhecimentos sobre presso de vapor, correto afirmar: (1) O ter etlico apresenta interaes moleculares mais intensas que o lcool etlico. (2) A 1 atm, a temperatura de ebulio da gua menor que a do lcool etlico. (4) A temperatura de ebulio do ter etlico maior do que 25C, presso atmosfrica de 600 mmHg. (8) A presso de vapor de um lquido, a temperatura constante, diminui com o aumento de volume do recipiente que o contm. (16) Duplicando-se a quantidade de lquido contido em um recipiente fechado, a temperatura constante, a presso de vapor desse lquido duplicar. (32) A presso de vapor de um lquido varia com a temperatura.

Composto (C2H5)2O C2H5OH H2O

692

QUMICA
Questo 6

UFBA
F A S E

A bateria chumbo/cido, utilizada na gerao de energia eltrica para automveis, pode ser recarregada pelo prprio dnamo do veculo.

2 A

P R O V A

Semi-reao
2 PbO2(s) + SO4(aq) + 4H (aq)+ 2e- +

Potencial padro de reduo E(V) PbSO4(s) + 2H2O(l) 0,36 1,69


-

PbSO4(s) + 2e- Pb + SO 4(aq)


2

(s)

Associando-se as informaes da tabela e da figura, correto afirmar: (1) (2) (4) (8) (16) O eletrodo de xido de chumbo o nodo da bateria. A diferena de potencial de 6 pilhas associadas em srie 12,30 V. 2 Uma semi-reao que ocorre na bateria Pb(s) + SO 4(aq) PbSO4(s) + 2eNo processo de recarga, a placa de chumbo o nodo da bateria. Quando ocorre a descarga da bateria, a densidade da soluo diminui, devido ao consumo de ons sulfato e formao de gua. (32) Durante o processo de descarga da bateria, so envolvidos 4 eltrons.

693

QUMICA
Questo 7

UFBA
F A S E

2 A

P R O V A Sobre a reao no balanceada, acima representada, e os compostos envolvidos, pode-se afirmar: (1) (2) (4) (8) (16) (32) (64) A reao de esterificao. I um ter e apresenta cadeia homognea. Se R possui uma ligao mltipla, o composto I um lipdio insaturado. II tem baixa solubilidade em gua e um trilcool primrio. II o propanotriol e possui ponto de ebulio superior ao do etanol. Em III, cada um dos sais apresenta uma extremidade polar e outra apolar. Em meio cido, o composto I no sofre hidrlise.

Questo 8 Em relao aos compostos orgnicos e suas reaes, pode-se afirmar: CH3CH2CONH2 uma amida. Na reao CH2CHCH2CH3 + HBr A, o composto A o 1bromo-butano. As protenas so constitudas por molculas de aminocidos. + Em CH3CH2CONH2 + H2O > B + NH3, B CH3CH2COO K , e a reao de KOH hidrlise em meio bsico. (16) CH3CH(NH2)COOH anftero. KMnO4 (32) Na reao no balanceada CH3CH(OH)CH2CH3 > C + H2O. C o butanal, H3O e a reao de adio. (64) CH3CH(NH2)CH3 uma amina terciria. (1) (2) (4) (8)

694

QUMICA
QUESTES 9 E 10

UFBA
F A S E

INSTRUO: Efetue os clculos necessrios e marque o resultado na Folha de Respostas. Questo 9 O metal X reage com cido clordrico de acordo com a equao balanceada: 3+ X(s) + 3HCl(aq) X (aq) + 3Cl (aq) + 3 H2(g) 2 Considerando-se que 500 mL de uma soluo 3M de cido clordrico reagem completamente com 26,0 g desse metal, calcule a massa atmica de X. Expresse o resultado com dois algarismos significativos.

2 A

P R O V A

Questo 10 Um botijo de gs contm 12,8 kg de uma mistura, composta de 53% de propano e 47% de butano em peso, a 25C e 7 atm de presso. Admitindo-se que 1L dessa mistura custe R$ 0,02, calcule, em real, o preo do contedo do botijo. Expresse o resultado com dois algarismos significativos.

695

QUMICA

UFMG
F A S E

1. Durante a preparao do popular cafezinho brasileiro, so utilizados alguns procedimentos de separao de misturas. A alternativa que apresenta corretamente a seqncia de operaes utilizadas a) destilao e decantao. b) destilao e filtrao. c) extrao e decantao. d) extrao e filtrao.

1 A

2. Ao resumir as caractersticas de cada um dos sucessivos modelos do tomo de hidrognio, um estudante elaborou o seguinte quadro:
MODELO ATMICO Dalton Thomson CARACTERSTICAS tomos macios e indivisveis. Eltron, de carga negativa, incrustado em uma esfera de carga positiva. A carga positiva est distribuda, homogeneamente, por toda a esfera. Eltron, de carga negativa, em rbita em torno de um ncleo central, de carga positiva. No h restrio quanto aos valores dos raios das rbitas e das energias do eltron. Eltron, de carga negativa, em rbita em torno de um ncleo central, de carga positiva. Apenas certos valores dos raios das rbitas e das energias do eltron so possveis.

P R O V A

Rutherford

Bohr

O nmero de ERROS cometidos pelo estudante a) 0 b) 1 c) 2 d) 3

3. Considere a abundncia relativa dos elementos na crosta terrestre representada no grfico ao lado. A anlise do grfico permite concluir que, em relao crosta terrestre, INCORRETO afirmar que a) apenas dois metais de transio esto entre os constituintes identificados por seus smbolos. b) entre os metais identificados por seus smbolos, os alcalinos so mais abundantes do que os alcalinos terrosos. c) menos da metade de sua massa constituda por metais. d) os halognios constituem menos de 1,1% de sua massa.

696

QUMICA

UFMG
F A S E

4. Existem algumas propriedades que so adequadas para caracterizar os slidos inicos, uma vez que a grande maioria desses slidos apresenta essas propriedades. Outras propriedades no so adequadas para esse fim, pois podem existir slidos inicos que no apresentem essas outras propriedades. Considere o conjunto dos slidos inicos. Entre os propriedades relacionadas, indique a que NO ser exibida por um grande nmero de slidos. a) Apresentar altas temperaturas de fuso. b) Conduzir corrente eltrica quando fundido. c) Ser isolante trmico e eltrico em estado slido. d) Ser solvel em gua.

1 A

P R O V A

5. O etanol (lcool etlico, CH3, CH2, OH) um lquido menos denso do que a gua. Ele usado na limpeza domstica porque dissolve gorduras, solvel em gua e mais voltil do que ela. O quadro abaixo representa cada uma dessas propriedades relacionadas a uma explicao com base nos modelos de interaes intermoleculares. Assinale a alternativa que contm uma explicao INADEQUADA para a propriedade relacionada.

PROPRIEDADE DO ETANOL

EXPLICAO a molcula de etanol tem uma parte pouco polar as interaes intermoleculares so mais fracas no etanol do que na gua a massa molar do etanol maior do que a da gua a molcula de etanol forma ligaes de hidrognio com a molcula de gua

a) b) c) d)

dissolver gorduras ser mais voltil do que a gua ser menos denso do que a gua ser solvel em gua

697

QUMICA

UFMG
F A S E

6. Um dos principais poluentes produzidos pelos automveis o monxido de carbono que resulta da queima parcial do combustvel. Uma proporo correta entre o combustvel e o ar, injetados no motor, fundamental no controle da emisso desse poluente. Em condies normais de uso do motor, a alternativa que apresenta, qualitativamente, a quantidade de CO produzida em funo da proporo ar/combustvel.

1 A

P R O V A

7. Um bom mtodo para a preparao controlada de oxignio muito puro a decomposio trmica de permanganato de potssio sob vcuo. Essa reao pode ser representada pela equao 2KMnO4(s) K2MnO4(s) + MnO2(s) + O2(g) Com relao decomposio completa de 2 mol de permanganato de potssio, INCORRETO afirmar que a) a massa de KMnO4(s) decomposta 316,0 g. b) a massa total dos produtos slidos 300,0 g. c) a quantidade de O2 (g) produzida 1 mol. d) as quantidades, em mol, de cada um dos produtos so iguais.

698

QUMICA

UFMG
F A S E

8. Precipitados podero ser formados quando, em soluo aquosa, ons que formem compostos pouco solveis forem colocados em contato. Brometo de prata um composto pouco solvel em gua. O mesmo ocorre com o sulfato de brio. Por outro lado, todos os sais comuns dos metais alcalinos so solveis em gua. Considere a tabela abaixo, referente formao ou no de precipitados pela mistura das solues indicadas, todas na concentrao de 0,1 mol/L. A formao de precipitado indicada pelo sinal + e a no-formao, pelo sinal ; um x indica que a experincia no foi realizada.
KBr(aq) AgNO3(aq) NaNO3(aq) KCl(aq) BaCl2(aq) + + x x Na2SO4(aq) x x

1 A

P R O V A

O nmero total de ERROS, contido na tabela, a) 0 b) 1 c) 2 d) 3

9. Estudaram-se as variaes das presses de vapor da gua pura e de uma soluo aquosa diluda de sacarose (acar de cana), em funo da temperatura. O grfico que descreve, qualitativamente, essas variaes

699

QUMICA

UFMG
F A S E

10. O rtulo de um produto usado como desinfetante apresenta, entre outras, a seguinte informao.
Cada 100 mL de desinfetante contm 10 mL de soluo de formaldedo 37% V/V (volume de formaldedo por volume de soluo).

1 A

A concentrao de formaldedo no desinfetante, em porcentagem volume por volume, a) 1,0% b) 3,7% c) 10% d) 37% P R O V A

11. Considere o seguinte diagrama de entalpia, envolvendo o dixido de carbono e as substncias elementares diamante, grafita e oxignio. Considerando esse diagrama, assinale a afirmativa FALSA. a) A transformao do diamante em grafita exotrmica. b) A variao de entalpia na combusto 1 de 1 mol de diamante igual a 392 kJ mol . c) A variao de entalpia na obteno de 1 mol de 1 CO2(g), a partir da grafita, igual a 394 kJ mol . d) A variao de entalpia na obteno de 1 mol de 1 diamante, a partir da grafita, igual a 2 kJ mol .

12. Em dois experimentos, solues de cido clordrico foram adicionadas a amostras idnticas de magnsio metlico. Em ambos os experimentos, o magnsio estava em excesso e a soluo recobria inteiramente esse metal. O grfico abaixo representa, para cada experimento, o volume total de hidrognio desprendido em funo do tempo. Com relao a esses experimentos, assinale a afirmativa FALSA. a) A concentrao do cido no experimento I igual a zero no tempo t = 80 s. b) A concentrao do cido usado no experimento I menor do que a do cido usado no experimento II. c) O volume de cido usado no experimento II maior do que o volume usado no experimento I. d) O volume total produzido de hidrognio, no final dos experimentos, maior no experimento II do que no I.

700

QUMICA

UFMG
F A S E

13. O rtulo de um medicamento utilizado no tratamento da azia e de outros transtornos digestivos indica que, em sua composio qumica, existem as seguintes substncias: cido acetilsaliclico, cido ctrico, carbonato cido de sdio e carbonato de sdio. Quando se coloca um comprimido desse medicamento em gua, observa-se uma efervescncia. Com relao ao exposto, assinale a afirmativa FALSA. a) A efervescncia devida liberao de CO2. b) As substncias presentes so compostos orgnicos. c) Os cidos reagem com os carbonatos em soluo aquosa. d) Os carbonatos presentes apresentam comportamento bsico. 14. Os metais possuem diferentes tendncias de sofrer corroso, um processo natural de oxidao. A corroso pode ser relacionada com a facilidade de obter os metais a partir de seus minrios. Essas informaes esto representadas no diagrama, para alguns metais:

1 A

P R O V A

Aumenta a facilidade de reduo dos ons

Zn

Fe

Ni

Cu

Ag

Pt

Au

Aumenta a facilidade de oxidao dos metais

Com relao ao exposto, assinale a afirmativa FALSA. a) A maior facilidade de um metal sofrer corroso corresponde a uma maior dificuldade para obt-lo a partir do seu minrio. b) A prata, a platina e o ouro so considerados metais nobres pela sua dificuldade de oxidar-se. c) Os metais com maior facilidade de oxidao so encontrados na natureza na forma de substncias simples. d) O zinco metlico o mais reativo entre os metais listados.

701

QUMICA

UFMG
F A S E

15. Os steres, assim como as cetonas, so responsveis pelo sabor e fragrncia de muitas frutas, flores e aromatizantes artificiais. O odor e o sabor do acetato de isopentila so semelhantes aos da banana. O = CH3C O CH2CH2CHCH3

1 A

Quando esse composto orgnico reage com hidrxido de sdio, numa reao de saponificao, os compostos obtidos so a) CH3C O = ONa e CH3CHCH2CH2OH CH3 CH3CHCH2CH2ONa CH3 CH3CHCH2CH2ONa CH3 e CH3CHCH2CH2OH CH3

CH3 acetato de isopentila

P R O V A

b) CH3C

c) CH3C

d) CH3C

16. O etileno, C2H4, pode ser obtido industrialmente pelo craqueamento de alcanos como, por exemplo, C8H18 CH4 + 2 C2H4 + C3H6 O etileno usado principalmente na fabricao do polietileno, um polmero dos mais utilizados no mundo. A reao de polimerizao pode ser representada por H H H H H H +C=C + C=C + C = C + - C-C-C-C-C-C- H H H H H H H H H H H H etileno polietileno
-

Com relao ao exposto, assinale a afirmativa FALSA. a) b) c) d) A polimerizao do etileno, envolve a formao de ligaes simples. O craqueamento de alcanos produz hidrocarbonetos de menor massa molar. O etileno o monmero da reao de polimerizao. O polietileno um alqueno de massa molar elevada.

H H H H H H

O = OH

O = ONa

O = OH

702

QUMICA

UFMG
F A S E

1. Um estudante listou os seguintes processos como exemplos de fenmenos que envolvem reaes qumicas: I II III IV V adio de lcool gasolina. fermentao da massa na fabricao de pes. obteno de sal por evaporao da gua do mar. precipitao da chuva. queima de uma vela.

1 B

O nmero de ERROS cometidos pelo estudante a) 0 b) 1 c) 2 d) 3

P R O V A

2. Na experincia de espalhamento de partculas alfa, conhecidas como experincia de Rutherford, um feixe de partculas alfa foi dirigido contra uma lmina finssima de ouro, e os experimentadores (Geiger e Marsden) observaram que um grande nmero dessas partculas atravessava a lmina sem sofrer desvios, mas que um pequeno nmero sofria desvios muito acentuados. Esse resultado levou Rutherford a modificar o modelo atmico de Thomson, propondo a existncia de um ncleo de carga positiva, de tamanho reduzido e com, praticamente, toda a massa do tomo. Assinale a alternativa que apresenta o resultado que era previsto para o experimento de acordo com o modelo de Thomson. a) A maioria das partculas atravessaria a lmina de ouro sem sofrer desvios e um pequeno nmero sofreria desvios muito pequenos. b) A maioria das partculas sofreria grandes desvios ao atravessar a lmina. c) A totalidade das partculas atravessaria a lmina de ouro sem sofrer nenhum desvio. d) A totalidade das partculas ricochetearia ao se chocar contra a lmina de ouro, sem conseguir atravess-la.

3. Considere o grfico ao lado, referente produo mundial, de 1980, dos metais mais comumente usados. A anlise do grfico permite concluir que todas as afirmativas esto corretas, EXCETO a) O metal mais produzido no mundo um metal de transio. b) O metal representativo menos produzido, entre os relacionados, tem massa molar igual a 82 g/mol. c) O segundo metal mais produzido pertence ao mesmo grupo do boro. d) Os metais de transio relacionados pertencem primeira srie de transio.

703

QUMICA

UFMG
F A S E

4. A curva abaixo mostra a variao da energia potencial Ep em funo da distncia entre os tomos, durante a formao da molcula H2 a partir de dois tomos de hidrognio, inicialmente a uma distncia infinita um do outro.

1 B

P R O V A

Em relao s informaes obtidas da anlise do grfico, assinale a afirmativa FALSA a) b) c) d) A energia potencial diminui na formao da ligao qumica. A quebra da ligao H-H consome 458 kJ/mol. -11 O comprimento de ligao da molcula H2 de 7,40 x 10 m. Os tomos separados por uma distncia infinita se atraem mutuamente.

5. Foram apresentadas a um estudante as frmulas de quatro pares de substncias. Foi pedido a ele que, considerando os modelos de ligaes qumicas e de interaes intermoleculares apropriados a cada caso, indicasse, em cada par, a substncia que tivesse a temperatura de fuso mais baixa. O estudante props o seguinte:

PARES DE SUBSTNCIAS CH4, CH3OH NaCl, HCl SiO2, H2O I2, Fe

SUBSTNCIAS DE TEMPERATURA DE FUSO MAIS BAIXA CH4 NaCl SiO2 I2

A alternativa que apresenta o nmero de previses corretas feitas pelo estudante a) 0 b) 1 c) 2 d) 3

704

QUMICA
6. Em 1937, o dirigvel Hindenburg, enchido de gs de hidrognio, explodiu e se incendiou ao aterrissar (fotografia). Desde ento, o gs hlio tem sido preferido para encher dirigveis e bales meteorolgicos. Com relao ao exposto, a afirmativa FALSA a) a exploso do dirigvel envolveu reaes exotrmicas. b) o gs carbnico foi o principal produto qumico da exploso. c) o gs hlio passou a ser preferido por ser inerte. d) o gs hidrognio era usado por causa de sua baixa densidade.

UFMG
F A S E

1 B

P R O V A

7. cido clordrico de alta pureza pode ser obtido pela reao entre cloro e hidrognio, seguida pela dissoluo do cloreto de hidrognio em gua. Esses processos podem ser representados pelas equaes: Cl2 (g) + H2 (g) 2 HCl (g) HCl (g) H2O HCl (aq)

Considere a situao em que 1,0 mol de Cl2 seja posto para reagir com 2,0 gramas de H2 e o cloreto de hidrognio obtido seja totalmente dissolvido em 1,0 L de gua. Nessa situao, a afirmativa FALSA a) a concentrao da soluo de cido obtida 2,0 mol/L. b) a massa de cloro que reage 71 g. c) a quantidade de HCl (g) produzida 73 g. d) o reagente em excesso H2(g).

8. Em um creme dental, encontra-se um teor de flor de 1,9 mg desse elemento por grama de dentifrcio. O flor adicionado est contido no composto monofluorfosfato de sdio Na2PO3F (massa molar: 144 g/mol). A quantidade de Na2PO3F utilizada na preparao de 100 g de creme dental a) 0,144 g b) 0,190 g c) 1,44 g d) 1,90 g

705

QUMICA

UFMG
F A S E

9. A queima de combustveis fsseis nos veculos automotores e nas indstriais e as grandes queimadas nas regies de florestas tropicais so duas das principais causas do aumento da concentrao de dixido de carbono na atmosfera. Esse aumento cerca de 11% nos ltimos trinta anos contribui para a elevao da temperatura mdia do globo terrestre, atravs do efeito estufa. Desse ponto de vista, o uso do lcool como combustvel em automveis interessante, porque no contribui, de forma permanente, para o aumento da concentrao atmosfrica de dixido de carbono. A alternativa que melhor explica essa vantagem do uso do lcool etlico a) b) c) d) a queima do etanol completa. a queima do etanol no produz CO2. o catalisador usado nos carros a etanol impede a formao de CO2. o replantio da cana-de-acar consome CO2.

1 B

P R O V A

10. Nos diagramas abaixo, as linhas horizontais correspondem a entalpias de substncias ou de misturas de substncias. O diagrama que, qualitativamente, indica as entalpias relativas de 1 mol de etanol lquido, 1 mol de etanol gasoso e dos produtos da combusto de 1 mol desse lcool, 2 CO2 + 3 H2O,

706

QUMICA

UFMG
F A S E

11. Uma das etapas na fabricao do cido sulfrico a conservao de SO2 a SO3, reao reversvel efetuada na presena de catalisador, que pode ser representada pela equao 2 SO2(g) + O2 (g) 2SO3(g) A reao direta exotrmica Considere um experimento em que a presso total dos gases seja mantida constante. O grfico que descreve, qualitativamente, a variao, no equilbrio, da concentrao de SO3 com a temperatura, nesse experimento,

1 B

12. A classificao de substncias como cidas ou bsicas e a distino entre cidos ou bases fortes e fracos ajudam a prever o seu comportamento qumico. Sabe-se, por exemplo, que o cido clordrico um cido forte e que o cido actico um cido fraco. Sabe-se, tambm, que o hidrxido de sdio uma base forte e que a amnia uma base fraca. Com relao s propriedades cido-bsicas dessas substncias, assinale a afirmativa FALSA. a) b) c) d) A gua um dos produtos da reao entre cido actico e hidrxido de sdio. A amnia, ao dissolver-se em gua, provoca um aumento na concentrao de ons hidrxido. A concentrao de ons acetato, em uma soluo aquosa 1 mol/L de cido actico, 1 mol/L. A concentrao de ons hidrognio, em uma soluo aquosa 0,01 mol/L de cido clordrico, 0,01 mol/L.

P R O V A

707

QUMICA

UFMG
F A S E

13. Na embalagem de um produto usado para desentupir pias e ralos, base de soda custica (hidrxido de sdio - NaOH), so encontradas, entre outras, as instrues:
Cuidado: Em caso de contato, lavar imeditamente os olhos ou a pele com gua em abundncia durante quinze minutos. Se ingerido, no provocar vmito. Dar grande quantidade de gua e tambm vinagre diludo em um copo de gua. A seguir, dar uma colher de leo comestvel. No reaproveitar a embalagem vazia. Lavar a colher utilizada como medida com bastante gua corrente antes de reutiliz-la. No adicionar gua embalagem com o produto.

1 B

O quadro abaixo relaciona algumas dessas instrues com as justificativas para o uso desses procedimentos, com base nas propriedades da soda custica e das outras espcies envolvidas. Assinale a alternativa que contm uma justificativa INCORRETA para a instruo relacionada. INSTRUO a) b) Dar vinagre diludo em um copo de gua Lavar a colher utilizada como medida com bastante gua corrente antes de reutiliz-la No adicionar gua embalagem com o produto No reaproveitar a embalagem vazia JUSTIFICATIVA O vinagre diludo neutraliza a soda custica atravs de reao cido-base A utilizao de grande quantidade de gua deve-se ao fato de a soda custica ser insolvel na gua A adio de gua embalagem com o produto provoca forte aquecimento A embalagem pode estar contaminada com resduos de soda custica

P R O V A

c) d)

14. O alumnio o segundo metal mais utilizado no mundo. Sua resistncia corroso devida camada aderente e impermevel de xido que se forma sobre a superfcie do metal. Essa camada protetora pode ser tornada mais espessa atravs de um processo denominado anodizao (figura abaixo). Nesse processo, oxignio gerado por eletrlise, segundo a semi-reao H2O (l)
+ 1 O2 (g) + 2 e + 2H (aq) 2

O oxignio reage, em seguida, com o alumnio, formando o xido correspondente. Com referncia ao exposto, a afirmativa FALSA a) a anodizao aumenta a resistncia do alumnio corroso. b) o fluxo de eltrons, pelo circuito externo, ocorre na direo do objeto de alumnio. c) o objeto de alumnio constitui o anodo da clula eletroqumica. d) o processo de anodizao consome energia eltrica.

708

QUMICA

UFMG
F A S E

15. Existem aparelhos que permitem estudar a variao da absoro de radiao eletromagntica por uma substncia, em funo do comprimento de onda da radiao. Nos grficos obtidos nesses aparelhos, absores intensas entre os comprimentos de onda 5,1 x 10-6 m e 6,1 x 10-6 m so caractersticas de ligaes duplas carbono-oxignio em compostos orgnicos. Entre os grficos abaixo, todos numa mesma escala, assinale aquele em que a absoro mais intensa observada pode ser associada com a funo orgnica indicada.

1 B

P R O V A

16. A celulose, um polmero natural, apresenta a estrutura:

Em relao celulose, assinale a alternativa correta. a) b) c) d) destruda na fabricao de carvo vegetal. solvel em gua. Tem a sacarose como monmero. Tem grupos fenlicos.

709

QUMICA
QUESTO 01

UFMG
F A S E

Os valores das sucessivas energias de ionizao de um tomo constituem uma evidncia emprica da existncia de nveis de energia. Os diagramas abaixo pretendem representar, qualitativamente, as quatro primeiras energias de ionizao de tomos, inicialmente neutros, do terceiro perodo da classificao peridica.

1. Um dos diagramas no corresponde a nenhum elemento. IDENTIFIQUE esse diagrama e JUSTIFIQUE sua resposta. DIAGRAMA: JUSTIFICATIVA:

2. INDIQUE qual o elemento que corresponde ao outro diagrama. ESCREVA a configurao eletrnica, por nveis, do tomo desse elemento. JUSTIFIQUE sua indicao. ELEMENTO: CONFIGURAO ELETRNICA POR NVEIS: JUSTIFICATIVA DA INDICAO:

QUESTO 02 Hidrxido de clcio, Ca(OH)2, e sulfato de alumnio, Al2(SO4)3, so utilizados no tratamento de guas para consumo pblico. Sob condies adequadas, forma-se um precipitado gelatinoso de hidrxido de alumnio, Al(OH)3, que arrasta consigo partculas de impureza. O precipitado e as impurezas so eliminados numa etapa posterior. 1. ESCREVA a equao qumica balanceada da reao entre hidrxido de clcio e sulfato de alumnio. 2. As quantidades de sulfato de alumnio e de hidrxido de clcio a serem adicionadas gua dependem da concentrao das impurezas presentes. Considere que seja adicionada uma certa quantidade de hidrxido de clcio a um tanque contendo gua no tratada. INDIQUE como varia o pH da gua e JUSTIFIQUE sua resposta.

710

QUMICA

UFMG
F A S E

3. Considere uma adio posterior, ao mesmo tanque, de sulfato de alumnio, causando precipitao de hidrxido de alumnio. INDIQUE o que acontece, nessa etapa, com o pH da gua e JUSTIFIQUE sua resposta.

4. O sulfato de alumnio comercial, cuja frmula Al2(SO4)3 14H2O, tem massa molar igual a 596 g/mol. Suponha terem sido adicionados 1192 g desse produto ao tanque mencionado nos itens anteriores. Considerando haver precipitao completa dos ons alumnio, CALCULE a massa, em gramas, de hidrxido de alumnio formada. Deixe indicadas todas as etapas de seus clculos, incluindo as unidades.

QUESTO 03 Em dois recipientes idnticos, foram colocados volumes iguais de gua pura (frasco A) e de uma soluo 0,2 mol/L de sacarose em gua (frasco B). Esses frascos foram colocados sob uma redoma de vidro completamente fechada e mantidos a temperatura constante.

1. COMPARE as temperaturas de ebulio dos lquidos colocados inicialmente nos frascos A e B. JUSTIFIQUE sua resposta.

2. Observa-se, com o transcorrer do tempo, que o volume do lquido do frasco A diminui e que, no frasco B, aumenta. JUSTIFIQUE essas observaes considerando os processos de evaporao e de condensao que ocorrem em cada frasco.

711

QUMICA
QUESTO 04

UFMG
F A S E

Um exemplo de uma reao para converso do metano, principal constituinte do gs natural, em outros hidrocarbonetos, a transformao desse composto em etano, que pode, em princpio, ser representada pela equao Reao I: 2 CH4 (g) > C2H6 (g) + H2 (g)

Na prtica, essa uma transformao cataltica, em que metano reage com oxignio, o que pode ser representado pela equao Reao II: 2 CH4 (g) + 0,5 O2 (g) > C2H6 (g) + H2O (g)

Considere o seguinte quadro de entalpias padro de formao ( f), a 25C, das substncias indicadas.

SUBSTNCIA Ch4 (g) C2H6 (g) H2O (g)

f/(kJ mol ) 75,0 85,0 242

-1

1. CALCULE a variao de entalpia padro a 25C das reaes I e II acima. Deixe indicadas todas as etapas dos clculos, incluindo as unidades. REAO I: REAO II: 2. Suponha que os recipientes em que ocorrem as reaes I e II sejam envolvidos por serpentinas em que circule gua, inicialmente a 25C. INDIQUE o que ocorre com a temperatura da gua em cada uma das reaes I e II e JUSTIFIQUE a sua resposta. REAO I: REAO II:

712

QUMICA
QUESTO 05

UFMG
F A S E

Quantidades equimolares de H2 e I2, em fase gasosa, a temperatura elevada, foram colocados em recipientes separados, mas unidos por uma vlvula que controla o fluxo dos gases. Aps a abertura da vlvula, esses gases se misturaram e reagiram de acordo com a equao H2 (g) + I2 (g) 2 HI (g) A figura abaixo ilustra a situao inicial das substncias H2 e I2, em que cada tomo de hidrognio foi representado por e cada tomo de iodo por .

1. CONSTRUA um grfico qualitativo mostrando as variaes das concentraes de H2, I2 e HI, desde o instante inicial, quando a vlvula foi aberta, at um certo tempo aps o equilbrio ter sido atingido.

2. COMPLETE a figura abaixo ilustrando a situao do sistema aps o estado de equilbrio ter sido atingido, estando aberta a vlvula. Use o mesmo nmero de tomos representados na primeira figura desta questo.

713

QUMICA
QUESTO 06

UFMG
F A S E

O cido ctrico o principal responsvel pelo sabor cido das frutas ctricas. Em reaes de neutralizao com NaOH 0,10 mol/L de sucos de dois tipos de laranja, foram anotados os seguintes resultados:
Tipo de laranja serra dgua pra Alquotas do suco / mL 5,0 5,0 Volume total de NaOH gasto na reao / mL 6,0 9,0

1. Considerando os dados do quadro, porm sem efetuar clculos, INDIQUE qual dos sucos de laranja o mais cido. JUSTIFIQUE a sua resposta. 2. O cido ctrico se dissocia em gua liberando trs ons H podendo, por isso, ser representado por H3Ci. Usando essa representao, ESCREVA a equao balanceada da reao de neutralizao total desse cido por hidrxido de sdio. 3. Admitindo que as propriedades cidas do suco de laranja provenham apenas do cido ctrico, CALCULE a concentrao, em mol/L, desse cido no suco de laranja-pra. Deixe indicadas todas as etapas dos clculos, incluindo as unidades.
+

QUESTO 07 Baterias de automvel so conjuntos de clulas eletroqumicas, cujos eletrodos esto imersos em soluo de cido sulfrico. Durante a descarga, as baterias geram energia eltrica. As semi-reaes que ocorrem durante a descarga e as suas respectivas foras eletromotrizes so Pb (s) + SO4 (aq) > PbSO4 (s) + 2 e
2 + 2 4

Eoxidao = 0,356 V Ereduo = 1,685 V

PbO2 (s) + SO4 (aq) + 4 H (aq) + 2 e > PbSO (s) + 2 H2O (l)

A reao global reversvel, o que permite que as baterias sejam recarregadas. 1. ESCREVA a equao balanceada da reao global que ocorre em uma bateria no processo de descarga. 2. CALCULE o valor da fora eletromotriz da reao de gerao de energia eltrica que ocorre na bateria. Deixe indicadas todas as etapas dos clculos, incluindo as unidades. 3. EXPLIQUE por que deve ser evitado o contato das mos e das roupas com a soluo da bateria. 4. comum o uso de um densmetro para se verificar se uma bateria est carregada ou no. Sabendo-se que o cido sulfrico mais denso do que a gua, INDIQUE se uma bateria descarregada tem soluo mais densa, menos densa ou se a sua densidade no varia em relao de uma bateria carregada. JUSTIFIQUE sua resposta.

714

QUMICA

UFMG
F A S E

QUESTO 08 O cloreto de polivinila, PVC, um dos polmeros mais utilizados no mundo. Em um dos processos para a sua fabricao, o etileno submetido a uma reao de adio com o cloro molecular. O produto obtido sofre, numa segunda etapa, eliminao de HCl e origina o cloreto de vinila. O HCl um importante subproduto dessa etapa. A ltima etapa a polimerizao do cloreto de vinila. 1. ESCREVA as equaes qumicas balanceadas das reaes que ocorrem nas duas primeiras etapas acima descritas. 2. CALCULE o volume de HCl(g), medido nas CNTP, que resulta como subproduto, quando so fabricados 625 quilogramas de cloreto de vinila. Nessas condies, admita que 1,0 mol de HCl(g) ocupe o volume de 22,4 L. Deixe indicadas todas as etapas de seus clculos, incluindo as unidades. 3. REPRESENTE, por sua frmula estrutural, uma parte da cadeia do PVC contendo, pelo menos, dois tomos de carbono. QUESTO 09 A tcnica chamada cromatografia gs-lquido pode ser usada para separar e identificar os componentes de diferentes tipos de gasolina. Um cromatgrafo gs-lquido constitudo por um tubo comprido a coluna que preenchido por partculas slidas inertes recobertas por uma pelcula de lquido. A amostra de gasolina injetada no cromatgrafo e vaporizada, sendo carregada ao longo da coluna por um gs inerte. A figura mostra um cromatograma, resultado da cromatografia de um tipo de gasolina especial. O cromatograma constitudo por uma srie de picos, cada um deles caracterstico de uma determinada substncia. Alguns dos picos esto identificados.

1. Existem duas classes de hidrocarbonetos entre as substncias identificadas nessa amostra de gasolina. INDIQUE quais so essas classes. 2. As diferentes substncias percorrem a coluna a diferentes velocidades, o que permite a sua separao e identificao. Essas velocidades dependem, entre outros fatores, da solubilidade das substncias no lquido da coluna. INDIQUE como o tempo que cada substncia leva para percorrer a coluna varia com o nmero de tomos de carbono. EXPLIQUE essa variao em termos de interaes intermoleculares entre as substncias identificadas e o lquido apolar da coluna. 3. Os picos A e B correspondem s substncias cujas molculas contm 4 tomos de carbono. O pico B est identificado como n-butano. INDIQUE a frmula estrutural de uma substncia que poderia ser responsvel pelo pico A e JUSTIFIQUE, em termos de sua estrutura e das interaes intermoleculares, o motivo pelo qual o seu pico aparece primeiro no cromatograma.

715

QUMICA

UFPA
F A S E

1. Quando um prisioneiro, condenado morte, executado em uma cmara de gs, a substncia letal, o gs HCN, produzida no ato da execuo atravs da reao: 2NaCN + H2SO4 2HCN + Na2SO4 Os reagentes e produtos desta reao pertencem s funes inorgnicas a) cido e base b) sal e xido c) sal e cido 2. Observe as reaes I e II abaixo: I - NH3 + HCl NH4Cl e II - 2HgO aquecimento 2Hg + O2 Podemos afirmar que I e II so, respectivamente, reaes de a) sntese e anlise b) simples troca e sntese c) dupla troca e anlise d) anlise e sntese e) dupla troca e simples troca d) base e sal e) xido e cido

3. Um tomo, cujo nmero atmico 18, est classificado na tabela peridica como a) metal alcalino b) metal alcalino terroso c) metal terroso d) ametal e) gs nobre

4. As substncias, cujas nomenclaturas segundo a IUPAC so: etanoato de metila; 2-metilpropanico; butanal; 3-metil-pentanol-3 e ciclohexanona, pertencem, respectivamente, s funes orgnicas a) b) c) d) e) cetona, ter, ster, amina e amida aldedo, ster, cetona, cido carboxlico e amida ster, aldedo, cido carboxlico, cetona e amina cido carboxlico, lcool, cetona, aldedo e hidrocarboneto ster, cido carboxlico, aldedo, lcool e cetona

716

QUMICA
5. Observe os tomos:
39

UFPA
F A S E

X , 40Y , 40Z e 39A

19

20

19

20

correto afirmar que a) X e Z so istonos b) Y e Z so istopos c) Z e A so istonos d) X e A so isbaros e) Y e A so isbaros

6. Em uma mistura homognea esto presentes gua (H2O), sal comum (NaCl) e cloreto de clcio (CaCl2). Estas substncias apresentam seus tomos unidos, respectivamente, por ligaes a) inicas, inicas e inicas b) covalentes, covalentes e covalentes c) inicas, covalentes e covalentes 7. Observe a reao: 6KI + 2KMnO4 + 4H2O 3I2 + 2MnO2 + 8KOH Os nmeros de oxidao dos elementos Mn (no KMnO4), I (no I2) e Mn (no Mn O2) so, respectivamente a) +7, 1, +4 b) +7, 0, +4 c) +4, 1, +4 d) +6, +1, +4 e) 4, 0, +4 d) covalentes, inicas e inicas e) covalentes, inicas e covalentes

8. A combusto total de lcoois (reao com o O2) apresenta como produtos gua e gs carbnico (CO2). Ao queimarmos 30g de propanol-2 obteremos uma quantidade de gs carbnico, em gramas, igual a a) 22 b) 66 c) 9 d) 63 e) 44

1. Considere a reao: H2(g) + 1/2 O2(g) a) b) c) d) H2O(l ) + 68,3 Kcal

Qual o valor da variao de entalpia? A reao endotrmica ou exotrmica? Represente a reao em um grfico de entalpia Represente graficamente a reao inversa.

F A S E

caminho da reao.

717

QUMICA

UFPA
F A S E

2. O isopropanol, muito utilizado comercialmente para a limpeza de cabeas magnticas, foi submetido a reao segundo o esquema: isopropanol X
[o]

1)CH3 Mg Br 2) H3O+

a) D nome, segundo a IUPAC, para X e Y. b) Diferencie as foras intermoleculares presentes em X com relao s presentes em Y. c) Compare a acidez entre Y e o isopropanol. 3. Para obter um determinado tipo de vidro uma indstria dispunha apenas das seguintes substncias Na2CO3, SiO2 e CaCl2, todas em grandes quantidades. A reao que envolve a fabricao do vidro : Na2CO3 + CaCO3 + 6 SiO2 Na2O.CaO.6SiO2 + 2CO2(g)
VIDRO

Como possvel notar no est disponvel inicialmente o reagente CaCO3, que pode ser produzido segundo a reao: Na2CO3 + CaCl2 CaCO3 + 2NaCl Considerando reagentes e produtos 100% puros e rendimentos das reaes de 100%, calcule a massa total de carbonato de sdio (em quilogramas), necessria para produzir 1000 frascos de 239 gramas cada.

4. Um halogeneto de alquila, com frmula molecular C4H9Br e que apresenta isomeria tica, foi submetido a reao em presena de KOH. a) Escreva a reao completa. b) Identifique os ismeros geomtricos obtidos e escreva suas frmulas estruturais. c) Explique porque o halogeneto de partida apresenta isomeria tica. 5. 116 gramas de um hidrocarboneto saturado gasoso, derivado de petrleo, esto confinados em um recipiente de 41 litros, a 23C e presso de 1 atm. Dado: R = 0,082 L.atm/K a) Escreva a frmula molecular deste gs. b) A volume constante, calcule a presso deste gs ao elevarmos a temperatura para 27C.

718

QUMICA
6. O processo de obteno do cido sulfrico se passa nas seguintes etapas: I - FeS(s) + O2(g) SO2(g) + Fe2O3(s) II - SO2(g) + 1/2 O2(g) SO3(g)
NO2 NO

UFPA
F A S E

III - SO3 + H2O H2SO4 a) Ajuste os coeficientes da equao I. b) A reao abaixo apresenta uma das utilizaes do cido sulfrico, escreva os produtos e classifique-a quanto ao tipo de reao. CaF2 + H2SO4 c) Indique a que funes inorgnicas pertencem os produtos da reao do item b). 7. Uma soluo de glicose (C6H12O6) foi preparada solubilizando-se 90 gramas deste soluto em gua suficiente para completar 250 ml de soluo (soluo 1). a) Que volume de uma soluo 0,5 M do mesmo soluto (soluo 2), dever ser adicionado soluo 1 para fornecer uma soluo final (soluo 3), 0,8 M. b) Considerando as solues 2 e 3, qual delas ter a menor presso de vapor. Explique. 8. Um istopo radioativo Y emite radiao e perde 87,5% de sua atividade em 24 dias, transformando-se em Xennio (Xe). a) Escreva a equao de decaimento de Y. b) Qual o elemento Y? c) Calcule o tempo de meia vida de Y. 9. Suponha que, em um determinado hospital, entregue no mesmo dia e hora, todas as semanas, uma mesma quantidade de material radioativo usado no tratamento de pacientes. Um tcnico em radioterapia encontra um frasco aberto do referido material sem a especificao do dia de chegada e observa usando um contador Geiger que so registradas 5,25 103 contagens por minuto. Usando o mesmo contador ele registra, para um frasco acabado de chegar, 84 103 contagens por minuto. Sabe-se que a meia-vida deste material 7 dias. Pede-se: a) O esboo do grfico do decaimento do nmero de tomos radioativos presentes no material (eixo das ordenadas) em funo do tempo (eixo das abscissas). b) Determinar, em dia, h quanto tempo j estava no hospital o material radioativo. c) Determinar, em dia, qual o tempo necessrio para que 75% dos tomos inicialmente presentes no material se desintegrem. Dado: n 0.75 = 0.287

719

QUMICA

UFPE
F A S E

1. Considere que os tomos do elemento 1 e os tomos do elemento 2 sero colocados num mesmo recipiente. As figuras I, II III e IV representam diferentes situaes que podem ocorrer. elem. 1 elem. 2

Escolha abaixo a alternativa que melhor representa as figuras: a) (I) - reagiu formando uma mistura; (II) - no houve reao; (III) - reao incompleta; (IV) - formou um nico composto. b) (I) - formou um nico composto; (II) - no houve reao; (III) - reagiu formando uma mistura; (IV) - reao incompleta. c) (I) no houve reao; (II) - reagiu formando uma mistura; (III) reao incompleta; (IV) formou um nico composto. d) (I) e (III) houve formao de mistura. e) (I) - ocorreu decomposio; (II) - reao incompleta; (III) reagiu formando uma mistura; (IV) formou uma substncia pura. 2. Considere as seguintes tarefas realizadas no dia-a-dia de uma cozinha e indique aquelas que envolvem transformaes qumicas. 1. 2. 3. 4. 5. Aquecer uma panela de alumnio. Acender um fsforo. Ferver gua. Queimar acar para fazer caramelo. Fazer gelo. d) 3 e 5 e) 2 e 5

a) 1, 3 e 4 b) 2 e 4 c) 1, 3 e 5

720

QUMICA

UFPE
F A S E

3. O tcnico de uma ourivesaria, diante de uma mistura de pequenos pedaos de ferro e ouro, deseja separar o ouro. Para realizar esta tarefa, que opo(es) o tcnico pode escolher? 1. 2. 3. 4. 5. Extrair o ouro com um solvente orgnico. Empregar um eletroim para remover o ferro. Dissolver o ferro com uma soluo de cido sulfrico. Realizar uma catao baseada na diferena de cor. Dissolver o ouro com uma soluo de soda custica.

A sua escolha : a) 1 b) 1, 2 e 4 c) 3 e 5 4. Faa a associao entre as duas colunas: 1. BF3 2. H2O 3) NH3 4) C6H6 5) NaF ( ( ( ( ( ) ) ) ) ) ligao base de Lewis ligao inica cido de Lewis dois pares de eltrons isolados d) 2, 3 e 4 e) 5

A seqncia correta de cima para baixo : a) 4, 3, 5, 1, e 2 b) 3, 1, 2, 5 e 4 c) 4, 2, 3, 1 e 5 d) 2, 3, 5, 4 e 1 e) 1, 2, 4, 5 e 3

5. A energia de estabilizao dos ncleos atmicos pode ser representada pela curva abaixo. Tomando-se a energia de estabilizao como um critrio de reatividade nuclear, pode-se dizer que: a) Os ncleos atmicos so muito instveis b) O processo de fuso nuclear ocorre com os ncleos mais leves porque d como produto ncleos mais pesados com energia de estabilizao maior. c) A fisso nuclear realizada com os ncleos mais leves porque estes tm o nmero de nutrons maior que o nmero de prtons. d) Quando a fuso nuclear envolve ncleos mais pesados resulta na formao de ncleos mais leves com maiores energias de estabilizao. e) A energia de estabilizao dos ncleos atmicos varia linearmente com o nmero de massa.

721

QUMICA

UFPE
F A S E

6. comum se ouvir a seguinte afirmativa: enfiando-se o cabo de uma colher e equilibrando-se a mesma na boca de uma garrafa de refrigerante, o gs no escapa. Com relao a esta afirmativa, qual das alternativas abaixo correta? a) b) c) d) e) O gs no escapa porque a colher obstrui parcialmente o gargalo da garrafa. As molculas do gs so atradas e retidas pelo metal da colher e depois retornam ao lquido. A presso de vapor do lquido diminui com a presena do metal e o gs no escapa. A presso do ar entre a superfcie do lquido e a boca da garrafa aumenta e o gs no escapa. A afirmativa no verdadeira, pois no h razo para o gs no escapar.

7. Os anticidos so substnciais utilizadas para diminuir a quantidade de cido clordrico do estmago. Vrias substncias podem ser utilizadas com essa finalidade. Uma delas o xido de magnsio MgO (uma base) que reage com o cido clordrico de acordo com a seguinte equao qumica: MgO + 2H
+

Mg + H2O

2+

A massa de xido de magnsio necessria para neutralizar todo o cido num estmago que contm 0,06 mol de cido clordrico : pesos atmicos: Mg = 24,30; O = 15,99; H = 1,00 a) 1,21 g b) 1,46 g c) 0,73 g d) 0,54 g e) 2,42 g

8. Os anticidos mais indicados devem ser aqueles que no reduzam demais a acidez no estmago. Quando a reduo da acidez muito grande, o estmago secreta excesso de cido. Este efeito conhecido como a revanche cida. Qual dos itens abaixo poderia ser associado a esse efeito? a) b) c) d) e) A Lei da Conservao da Energia. O Princpio da Excluso de Pauli. O Princpio de Le Chatelier. O Primeiro Princpio da Termodinmica. O Princpio da Incerteza de Heisenberg.

9. Grande parte da energia necessria vida humana provm da oxidao do acar no organismo gerando dixido de carbono e gua, conforme a reao a seguir: C6H12O6 + 6 O2 a CO2 + b H2O

G = - 686 kcal /mol Com relao a esta reao podemos afirmar que os coeficientes a e b so, respectivamente: a) 12 e 18 e a reao espontnea. b) 6 e 6 e a reao espontnea. c) 6 e 8 e a reao no espontnea. d) 4 e 6 e a reao espontnea. e) 6 e 12 e a reao no espontnea.

722

QUMICA
10. Relacione os itens abaixo com os conceitos: cido, bsico e neutro. 1. 2. 3. 4. a) b) c) d) e) Uma Coca-Cola tem um pH igual a 3. -5 Um tablete de um anticido dissolvido num copo dgua tem [OH-] = 10 M. + -5 Uma xcara de caf tem [H ] = 10 M. + Uma soluo em que [H ] = [OH-] . 1) bsico, 2) bsico, 3) cido, 4) neutro 1) cido, 2) bsico, 3) neutro, 4) neutro 1) neutro, 2) cido, 3) bsico, 4) cido 1) cido, 2) neutro, 3) bsico, 4) bsico 1) cido, 2) bsico, 3) cido, 4) neutro

UFPE
F A S E

11. Algumas reaes qumicas na natureza s ocorrem na presena de luz. Considere a seguinte reao e assinale a alternativa correta: A + luz A* + B A* AB + calor + calor

A + B + luz AB

a) A energia sob forma de luz absorvida pelo reagente A transformada em energia de ligao qumica e calor. b) A luz entra na reao induzindo a formao de calor e sai da reao sem sofrer nenhum alterao. c) A reao endotrmica. d) Os reagentes A e B so respectivamente um cido e uma base. e) A reao no uma reao fotoqumica. 12. Relacione os tipos de substncias orgnicas da coluna da esquerda com os compostos orgnicos contidos na coluna da direita: 1. aldedo 2. lcool 3. cido carboxlico 4. ter 5. cetona ( ) ( ) ( ) ( ) ( )

A seqncia correta, de cima para baixo, : a) 5, 3, 4, 1 e 2 b) 1, 5, 2, 4 e 3 c) 4, 5, 1, 2 e 3 d) 1, 2, 5, 3 e 4 e) 5, 3, 2, 4 e 1

723

QUMICA

UFPE
F A S E

13. Considere o sistema eletroqumico mostrado na figura e analise as proposies abaixo:

1. Em qualquer uma das clulas, no ctodo ocorre uma reao de reduo, enquanto que no nodo ocorre uma reao de oxidao. 2. Na clula eletroltica a variao de energia livre (G) negativa, enquanto que na clula galvnica ela positiva. 3. A eletrlise uma reao forada, pois s ocorre s custas de consumo de trabalho eltrico. Est(o) correta(s): a) 1 b) 1 e 3 c) 3 d) 2 e) 1, 2 e 3

14. Qual dos seguintes processos no espontneo? a) b) c) d) e) A dissoluo de acar numa xcara de caf. A reao de sdio metlico com cloro gasoso para formar cloreto de sdio. A colagem de um vaso quebrado. A transformao do papel em cinzas pela ao do fogo. A vaporizao de uma fragrncia de perfume numa sala.

724

QUMICA

UFPE
F A S E

15. Os compostos I e II estereoismeros da carvona, e possuem sabores diferentes. O estereoismero I o responsvel pelo sabor de hortel e o II pelo sabor de um condimento extico, a alcaravia. Em relao a estes compostos incorreto afirmar:

a) b) c) d) e)

Tanto o composto I como o II possuem uma ligao dupla conjugada carbonila. Os compostos I e II so oticamente ativos. O composto I a imagem especular de II. Os compostos I e II possuem propriedades qumicas diferentes. I e II constituem um par de enantimeros.

16. Considere os seguintes diagramas de energia de reao nas mesmas condies de temperatura e presso e em funo deles indique a alternativa correta:

a) Quando o equilbrio for atingido, as concentraes de C e D sero maiores no caso do diagrama 1. b) A reao A + B C + D endotrmica. c) A variao de entalpia padro da reao maior no caso do diagrama 1. d) No caso do diagrama 2 tem-se a presena de um catalisador. e) No caso do diagrama 1 a reao mais rpida.

725

QUMICA
1. Considere os dois eventos abaixo:

UFPE
F A S E

1. Uma lmpada de filamento de tungstnio, que por algum defeito de fabricao continha ar em seu interior, ao ser acesa tem o filamento oxidado e a lmpada queima. 2. Um filamento de tungstnio exposto ao ar, ao qual se aplica uma corrente eltrica at que o aquecimento resultante provoque a sua oxidao. A respeito desses eventos, pode-se afirmar que: 0-0) 1-1) 2-2) 3-3) 4-4) A massa de lmpada como um todo, maior aps a oxidao. As massas de ambos os filamentos so maiores aps a oxidao. O evento (1) impossvel pois viola a lei de conservao das massas. Deve existir algum gs oxidante no interior da lmpada do evento (1). Ocorre a mesma reao qumica nos dois filamentos.

2. Num certo dia um tanque para tratamento de resduos qumicos continha, quando cheio, 3 gramas de um dado sal numa concentrao de 0,5 M. Hoje a concentrao deste sal no tanque cheio de 2,5 M. Qual a massa do sal no tanque?

3. Por um acaso do destino, um nufrago em uma ilha deserta encontra uma maleta contendo solues aquosas de alguns produtos qumicos: cido clordrico, cloreto de brio e nitrato de prata. Curioso o nufrago derrama algumas gotas do cido sobre um fragmento de rocha da ilha e observa a evoluo de um gs incolor e inodoro. Em duas amostras de gua de uma lagoa, ele adiciona separadamente algumas gotas das solues dos sais encontrados na maleta e observa em ambos os casos a formao de um slido branco em suspenso. O nufrago poder concluir que: 0-0) 1-1) 2-2) 3-3) 4-4) A rocha no contm sulfatos. A gua da lagoa pode conter cloretos. A gua da lagoa pode conter nitratos. A rocha pode conter carbonatos. A gua da lagoa no contm potssio.

4. Uma soluo aquosa de um soluto desconhecido testada com papel indicador de tornassol que revelou ser esta uma substncia cida. Alm disso, a soluo fracamente condutora quando comparada com uma soluo de NaCl de mesma concentrao. A substncia desconhecida poderia ser: 0-0) NaOH 1-1) NH3 2-2) CH3COOH 3-3) HNO3 4-4) H3PO4

726

QUMICA

UFPE
F A S E

5. A vitamina A e a vitamina C possuem solubilidades diferentes em solventes polares e apolares. Estas diferenas podem ser entendidas em termos da estrutura de cada uma das molculas mostradas a seguir:

Com base nestas estruturas, podemos afirmar que: 0-0) A vitamina A um lcool de cadeia carbnica longa da ser quase apolar. 1-1) A vitamina C uma molcula menor contendo mais grupos hidroxlicos, sendo insolvel em gua. 2-2) A vitamina A mais solvel nas gorduras, podendo ser armazenada mais tempo no corpo. 3-3) A vitamina C no forma pontes de hidrognio com a gua. 4-4) A vitamina A mais solvel em solventes orgnicos que a vitamina C. 6. O nitrognio e o fsforo pertencem a mesma famlia de elementos qumicos. O nitrognio capaz de formar trs ligaes qumicas enquanto que o fsforo capaz de formar at 5 ligaes. Para explicar este comportamento podemos afirmar que: 0-0) O nitrognio mais reativo que o fsforo. 1-1) O fsforo tem mais eltrons de valncia que o nitrognio. 2-2) Os orbitais s, p e d se hibridizam muito mais facilmente no caso do fsforo que no caso do nitrognio. 3-3) O raio atmico do fsforo menor que o do nitrognio. 4-4) O fsforo possui orbitais d vazios na camada de valncia e o nitrognio no. 7. Os lcoois 1, 2, 3 e 4 apresentam em soluo aquosa os valores de pKa indicados ao lado. Os seus nions correspondentes (tambm chamados de alcxidos) podem ser utilizados como bases e como nuclefilos. Considere as seguintes afirmativas: 0-0) O lcool 4 forma o alcxido mais forte. 1-1) O lcool 3 o cido conjugado do alcxido mais fraco. 2-2) O alcxido de 1 o nuclefilo com maior impedimento estrico. 3-3) O lcool 2 o cido conjugado mais forte de todos. 4-4) Os compostos 3 e 4 so lcoois primrios.

727

QUMICA

UFPE
F A S E

8. A sacarose um acar que se obtm da cana-de-acar. No nosso organismo uma importante reao que ocorre com a sacarose a hidrlise que resulta na formao de glucose e frutose. Esta reao se encontra esquematizada abaixo.

Com respeito a este sistema podemos afirmar que: 0-0) 1-1) 2-2) 3-3) 4-4) Na presena de excesso de glucose, a hidrlise inibida. A frutose apresenta 6 carbonos assimtricos. O anel da glucose deve ser mais estvel que o da frutose. Somente a sacarose forma ligaes do tipo ponte de hidrognio com a gua. A reao no sentido inverso uma condensao.

9. No processo Hall para a obteno de alumnio a partir de seu xido purificado (esquematizado abaixo), bastes de grafite so ligados ao terminal positivo de uma fonte de corrente enquanto que o recipiente ligado ao terminal negativo. A eletrlise feita numa mistura de criolita (Na3AlF6) e Al2O3.

Com base nesse esquema podemos dizer que: 0-0) 1-1) 2-2) 3-3) O alumnio possui densidade menor que a mistura lquida de criolita e Al2O3. O alumnio oxidade durante este processo pois o produto final alumnio metlico. A grafite funciona como nodo e ali deve ocorrer a reao de oxidao. A criolita um composto covalente e por isso, quando fundida, um bom condutor de eletricidade. 4-4) 2700 kg de alumnio podem ser obtidos a partir de 7500 kg de seu xido.

728

QUMICA

UFPE
F A S E

10. Um istopo radioativo do iodo (t1/2 = 8,1 dias) foi utilizado em um teste para se determinar a absoro de ons iodeto por plantas aquticas. As plantas foram colocadas em um aqurio com gua contendo esse istopo na forma de ons iodeto. A atividade radioativa inicial de uma amostra de 1,00 l de gua foi de 80 cpm (contagens por minuto). Aps 16,2 dias a atividade de uma amostra de mesmo volume foi medida obtendo-se o valor de 11 cpm. Assumindo-se que os nicos processos responsveis pela queda na atividade sejam a absoro de iodeto pelas plantas e o decaimento radioativo, qual a porcentagem de iodo absorvido pelas plantas nesse perodo?

11. Um aluno de qumica estudando a cintica de uma dada reao, observou que esta depende + da concentrao de ons hidrognio [H ]. Ele verificou que as velocidades iniciais variaram + como uma funo de [H ] da seguinte forma: [H ] (M)
+

0,0500 10
-7

0,100 3,2 10
-7

0,200 1,6 10
-7

veloc. inicial(M/s) 6, 4

Baseado nestes dados o aluno concluiu que: 0-0) 1-1) 2-2) 3-3) 4-4) A ordem da reao com respeito ao on H 1. Ao adicionar uma base a reao foi acelerada. + Quando a concentrao de [H ] for 0,400M, a velocidade inicial ser de 0,8 Quanto menor o pH mais rpida ser a reao. A adio de gua no altera a velocidade da reao.
+

10 M/s.

-7

12. Considere a seguinte reao, representada pela equao qumica abaixo, no balanceada: Fe(s) + H2O(g) Fe3O4(s) + H2(g) H < 0

Em relao ao equilbrio deste sistema pode-se afirmar que: 0-0) Aps a adio de H2O(g) h um recipiente fechado, contendo somente Fe3O4 numa alta temperatura, possvel se atingir o equilbrio da reao. 1-1) Os coeficientes para a equao balanceada da esquerda para a direita so 3, 4, 1 e 4. 4 4 2-2) A expresso do equilbrio para esta reao Kc = [Fe3O4] [H2] / [H2O] . 3-3) O equilbrio no afetado ao se aumentar a presso total do sistema. 4-4) O aquecimento do sistema provoca um deslocamento do equilbrio para a direita.

729

QUMICA

UFPE
F A S E

13. A oxidao da glucose (C6H12O6) no corpo humano produz CO2 e H2O. No entanto, a fermentao anaerbica da glucose, produz etanol (C2H5OH) e CO2. As reaes de oxidao e fermentao encontram-se esquematizadas abaixo:
oxid.

(1) C6H12O6 (s) + 602 (g)


ferment.

6CO2 (g) + 6H2O(I) G = 2875 kJ

(2) C6H12O6 (s)

2C2H5OH (I) + 2CO2 (g) G = 226 kJ

correto afirmar que: 0-0) 1-1) 2-2) 3-3) 4-4) possvel obter mais trabalho a partir do processo (2). O processo (1) uma combusto. A constante de equilbrio do processo fermentativo maior que a do processo oxidativo. Ambos os processos so termodinamicamente muito favorveis. O processo (2) uma decomposio.

14. Os aminocidos so componentes importantes na construo das protenas. Os aminocidos possuem comportamento anfotrico por conterem grupamentos cido e base numa mesma molcula, podendo at formar um on hbrido chamado de zwitterion, como mostrado abaixo para o exemplo de glicina.

0-0) 1-1) 2-2) 3-3)

No zwitterion, o nitrognio realiza uma ligao coordenada (dativa) com um on H . + O carter bsico da glicina em gua, leva formao de [H3N-CH2-COOH] (aq)+OH(aq). O carbono carboxlico apresenta hibridizao do tipo sp3. Devido a sua forma zwitterinica os aminocidos so mais solveis em gua que em solventes orgnicos no polares. 4-4) A ligao C-N na glicina covalente apolar.

730

QUMICA

UFPE
F A S E

15. Um gs ideal aquele que no apresenta interaes entre suas partculas (tomos ou molculas) e cujas partculas possuem dimenses desprezveis. Esta idealidade pode ser atingida somente sob certas condies experimentais. Como base nestes comentrios analise as afirmativas abaixo: 0-0) A temperatura ambiente, o oxignio gasoso a 0,01 atm de presso se comporta menos idealmente que a 10 atm de presso. 1-1) Nas mesmas condies de temperatura e presso o hidrognio deve se comportar mais idealmente que o cloro. 2-2) Numa mesma presso, um mesmo gs deve ser mais ideal quanto maior for a sua temperatura. 3-3) Molculas polares devem se comportar mais idealmente do que molculas apolares. 4-4) Molculas de gua devem se comportar menos idealmente que molculas de dixido de carbono. 16. Em uma estao para tratamento de gua, muitas vezes realiza-se a remoo de ons Ca dissolvidos, que so responsveis pela dureza da gua. Esta remoo pode ser conseguida pela adio de carbonato de sdio que ir provocar a precipitao do carbonato de clcio. Se +2 100.000 litros de gua contendo 98,1 ppm (partes por milho) de ons Ca precisam ser tratados de acordo com o procedimento acima, quantos quilos de carbonato de sdio sero necessrios? Utilize nmeros inteiros para as massas atmicas em seu clculo. Assinale no carto o inteiro mais prximo de sua resposta.
+2

731

QUMICA
1. Considere as seguintes propriedades de trs substncias lquidas:

UFRGS

SUBSTNCIA

DENSIDADE (g/mL a 20C)

SOLUBILIDADE EM GUA

Hexano Tetracloreto de carbono gua

0,659 1,595 0,998

insolvel insolvel

Misturando-se volumes iguais de hexano, tetracloreto de carbono e gua, ser obtido um sistema a) b) c) d) e) monofsico. bifsico, no qual a fase sobrenadante o hexano. bifsico, no qual a fase sobrenadante o tetracloreto de carbono. trifsico, no qual a fase intermediria o tetracloreto de carbono. bifsico ou trifsico, dependendo da ordem de colocao das substncias durante a preparao da mistura.
-

2. Os smbolos Cl, Cl2 e Cl representam, respectivamente, a) o tomo do elemento cloro, a molcula da substncia simples cloro e o nion cloreto. b) a molcula da substncia simples cloro, o elemento cloro e o tomo de cloro eletronegativo. c) a molcula da substncia simples cloro, a molcula da substncia cloro e o tomo do elemento cloro. d) o tomo do elemento cloro, a molcula da substncia composta cloro e o ction cloreto. e) o elemento qumico cloro, tomos do elemento cloro e o tomo do elemento cloro eletronegativo.

732

QUMICA

UFRGS

3. O conhecimento sobre estrutura atmica evoluiu medida que determinados fatos experimentais eram observados, gerando a necessidade de proposio de modelos atmicos com caractersticas que os explicassem.
Fatos Observados
I - Investigaes sobre a natureza eltrica da matria e descargas eltricas em tubos de gases rarefeitos.

Caractersticas do Modelo Atmico 1 - tomos macios, indivisveis e indestrutveis.

II - Determinao das Leis Ponderais das Combinaes Qumicas. III - Anlise dos espectros atmicos (emisso de luz com cores caractersticas para cada elemento). IV - Estudos sobre radioatividade e disperso de partculas alfa.

2 - tomos com ncleo denso e positivo, rodeado pelos eltrons negativos. 3 - tomos como uma esfera positiva onde esto distribudas, uniformemente, as partculas negativas. 4 - tomos com eltrons, movimentando-se ao redor do ncleo em trajetrias circulares denominadas nveis com valor determinado de energia.

A associao correta entre o fato observado e o modelo atmico proposto, a partir deste subsdio, : a) I - 3; b) I - 1; c) I - 3; II - 1; II - 2; II - 1; III - 2; IV - 4. d) I - 4; e) I - 1; II - 2; II - 3; III - 1; III - 4; IV - 3. IV - 2.

III - 4 ; IV - 3. III - 4; IV - 2.

137 4. Em 1987, ocorreu, em Goinia-GO, um grave acidente por contaminao com material radioativo, 55 quando a blindagem de uma fonte de csio-137 foi destruda. Sobre o tomo de Cs correto afirmar que apresenta 137

a) nmero de prtons igual ao de um tomo de b) nmero de nutrons igual ao de um tomo 137 de c) nmero atmico igual ao de um tomo de
54

56 Ba. 138 56

Ba.

Xe. 137
53 55

d) distribuio eletrnica igual a de um tomo de133 I. e) nmero de nutrons igual ao de um tomo de Cs.

733

QUMICA
5. Sobre o elemento qumico hidrognio so feitas as seguintes afirmaes:

UFRGS

I - Apresenta apenas 1 eltron em sua camada de valncia; sendo, portanto, um metal alcalino. II - Ao ganhar 1 eltron, adquire configurao eletrnica semelhante do gs nobre hlio. III - Os tomos do istopo mais abundante no apresentam nutrons em seu ncleo. Quais esto corretas? a) Apenas II b) Apenas I e II c) Apenas I e III d) Apenas II e III e) I, II e III

6. O quadro abaixo apresenta valores comparativos de duas propriedades peridicas, medidos em unidades convenientes propriedade.

Elemento

Propriedade 1

Propriedade 2

Be

1,12

215

20

Ca

1,97

141

34

Se

1,40

225

As propriedades 1 e 2 podem ser, respectivamente, a) potencial de ionizao e eletropositividade. b) raio atmico e potencial de ionizao. c) afinidade eletrnica e raio atmico. d) eletronegatividade e potencial de ionizao. e) eletronegatividade e eletropositividade.

734

QUMICA

UFRGS

7. Considere a seguinte transformao que ocorre com uma amostra gasosa de massa m, apresentando comportamento de um gs ideal.

O grfico que melhor representa esta transformao :

8. A combinao entre o nitrognio e o oxignio pode originar diferentes xidos. Entre os xidos nitrogenados abaixo, aquele que apresenta, em sua composio, o maior teor ponderal de nitrognio o a) NO b) NO2 c) N2O d) N2O3 e) N2O5

735

QUMICA

UFRGS

9. Sabe-se que um radionucldeo, ao emitir uma partcula alfa 42 , tem seu nmero de massa diminudo de quatro unidades e seu nmero atmico diminudo de duas unidades. O urnio238, ao emitir uma partcula alfa, transforma-se no trio-234, conforme a equao
238 92 U
23

( )

4 2

234 90

Th

Se 3,01 10 tomos de urnio-238 emitirem partculas alfa, a massa aproximada de trio234 formada de a) 45 g b) 90 g c) 117 g d) 144 g e) 234 g

10. Misturam-se duas solues aquosas conforme o esquema abaixo. Aps a reao, observa-se que a soluo final a) neutra, pois no h reagente em excesso.

b) cida, devido a um excesso de 0,6 g de HNO3. c) cida, devido a um excesso de 0,3 g de HNO3. d) neutra, devido formao de Ca(NO3)2. e) bsica, devido a um excesso de 0,3 g de Ca(OH)2. 11. Os sistemas: I - Fio de cobre metlico (Cu(s)); II - Soluo aquosa de sulfato de cobre (CuSO4(aq)); III - Cloreto de sdio fundido (NaCl(l)); so condutores de eletricidade. As partculas responsveis pela conduo da corrente eltrica, em cada sistema, so, respectivamente, a) eltrons, ons e ons. b) eltrons, eltrons e eltrons. c) tomos, ons e molculas. d) ctions, nions e eltrons. e) tomos, ctions e nions.

736

QUMICA

UFRGS

12. O momento dipolar a medida quantitativa da polaridade de uma ligao. Em molculas apolares, a resultante dos momentos dipolares referentes a todas as ligaes apresenta valor igual a zero. Entre as substncias covalentes abaixo I - CH4 II - CS2 III - HBr IV - N2

quais as que apresentam a resultante do momento dipolar igual a zero? a) Apenas I e II b) Apenas II e III c) Apenas I, II e III d) Apenas I, II e IV e) I, II, III e IV

Instruo: O enunciado e o grfico abaixo referem-se questo de nmero 13: O grfico a seguir apresenta os dados de massa molecular (M) x ponto de ebulio (P.E.) para os quatro primeiros termos da srie homloga dos lcoois primrios.

13. Analisando-se os dados apresentados, verifica-se que os lcoois com massa molecular mais elevada apresentam a) maiores pontos de ebulio, devido formao de pontes de hidrognio intermoleculares. b) maiores pontos de ebulio, devido polaridade do grupo OH. c) maiores pontos de ebulio devido ao aumento do nmero de interaes intermoleculares do tipo Van der Waals. d) menores pontos de ebulio devido diminuio do nmero de interaes intermoleculares do tipo Van der Waals. e) menores pontos de ebulio, pois o aumento da cadeia carbnica diminui a polaridade do grupo OH.

737

QUMICA

UFRGS

14. Considerando-se que o elemento ferro pode formar diferentes compostos nos quais apresenta valores de nmero de oxidao +2 ou +3, as frmulas dos possveis sulfatos e hidrxidos de ferro so: a) b) c) d) e) Fe2SO4, Fe3SO4, Fe2OH, Fe3OH FeSO4, Fe2(SO4)3, Fe(OH)2, Fe(OH)3 Fe(SO4)2, Fe(SO4)3, FeO, Fe2O3 FeSO3, Fe2(SO3)3, FeOH, Fe(OH)3 FeS, Fe2S3, Fe2O3, Fe3O4

15. Os aminocidos formam sais internos devido presena dos grupos -NH2 e -COOH em sua estrutura. Este fenmeno ocorre pela transferncia de um prton do -COOH para o -NH2, conforme o esquema

Nesse caso o -NH2 e o -COOH comportam-se, respectivamente, como a) b) c) d) e) base de Arrhenius e cido de Arrhenius cido de Brnsted-Lowry e base de Arrhenius. cido de Brnsted-Lowry e base de Lewis. cido de Lewis e base de Lewis. base de Brnsted-Lowry e cido de Brnsted-Lowry.

16. Entre as equaes qumicas abaixo, aquela que no representa uma reao qumica de acordo com a Lei da Conservao da Massa : a) (NH4)2Cr2O7 N2 + Cr2O3 + 4 H2O b) Al2(SO4)3 + 6 NaOH c) 3 P + 5 HNO3 + 2 HCl H2SO4 2 Al(OH)3 + 3 Na2SO4 + 5 NO + 2 H2O 3 H3PO4 MnCl2 + CaSO4

d) MnO2

2 H2O + Cl2 + 2 H2 O

e) Ca(OH)2 +

738

QUMICA

UFRGS

17. O ferro galvanizado apresenta-se revestido por uma camada de zinco. Se um objeto desse material for riscado, o ferro ficar exposto s condies do meio ambiente e poder formar o hidrxido ferroso. Neste caso, o zinco, por ser mais reativo, regenera o ferro, conforme a reao representada abaixo. Fe(OH)2 + Zn Zn(OH)2 + Fe Sobre essa reao pode-se afirmar que a) b) c) d) e) o ferro sofre oxidao, pois perder eltrons. o zinco sofre oxidao, pois perder eltrons. o ferro sofre reduo, pois perder eltrons. o zinco sofre reduo, pois ganhar eltrons. o ferro sofre oxidao, pois ganhar eltrons.

18. O hidrocarboneto que apresenta todos os tomos de carbono com orientao espacial tetradrica. a) H2C = CH2 b) c) HC CH d) H2C = C = CH2 e) H3C CH CH3 CH3 CH3

19. Na anlise de uma amostra de manteiga ranosa, foram encontrados compostos com frmulas moleculares C3H6O e C4H8O2. Sabendo-se que esses compostos apresentam cadeias carbnicas acclicas, normais e saturadas, possvel prever que se trata de a) b) c) d) e) aldedos, cetonas e cidos carboxlicos. aldedos, lcoois e teres. lcoois, teres e steres. cidos carboxlicos, cetonas e lcoois. steres, aldedos e teres.

0. Ortocresol, presente na creolina, resulta da substituio de um tomo de hidrognio do hidroxibenzeno por um radical metila. A frmula molecular do ortocresol a) C7H8O b) C7H9O c) C6H7O d) C6H8O e) C6H9O

739

QUMICA
21. Na reao de esterificao:

UFRGS

os nomes dos compostos I, II e III so, respectivamente, a) b) c) d) e) cido etanico, propanal e metanoato de isopropila. etanal, propanol-1 e propanoato de etila. cido etanico, propanol-1, etanoato de n-propila. etanal, cido propanico, metanoato de n-propila. cido metanico, propanal e etanoato de n-propila.

22. Com a frmula molecular C4H11N, so representados os seguintes pares de compostos:

Os pares I, II e III so, respectivamente, a) b) c) d) e) ismeros de posio, metmeros e ismeros de cadeia. ismeros de posio, tautmeros e ismeros funcionais. ismeros de cadeia, metmeros e ismeros de posio. ismeros funcionais, ismeros de posio e ismeros de cadeia. ismeros de cadeia, ismeros de posio e metmeros.

23. Em 1887, Wislicenus determinou experimentalmente a existncia de dois ismeros com frmula estrutural plana HOOC - CH = CH - COOH. O ismero cis foi denominado de cido malico e o ismero trans de cido fumrico. Sobre estes cidos correto afirmar que a) b) c) d) e) ambos desviam a luz polarizada, porm em sentidos opostos. ambos tm o mesmo ponto de fuso. os cidos malico e fumrico no sofrem reao de desidratao. formam um par de antpodas pticos. por adio de H2, em presena de catalisador, ambos produzem o cido butanodiico.

740

QUMICA

UFRGS

24. A anilina, matria-prima importante na obteno de corantes, pode ser preparada, sinteticamente, de acordo com a seqncia

As reaes I e II so, respectivamente, a) adio e oxidao. b) substituio e reduo. c) adio e reduo.

d) substituio e oxidao. e) eliminao e reduo.

25. As funes predominantes no leo de soja, leo diesel e leo de xisto betuminoso so, respectivamente, a) b) c) d) e) hidrocarbonetos, hidrocarbonetos e hidrocarbonetos. estres, steres e steres. hidrocarbonetos, steres e hidrocarbonetos. teres, hidrocarbonetos e steres. steres, hidrocarbonetos e hidrocarbonetos.

26. O formol uma soluo aquosa de metanal (HCHO) a 40%, em massa, e possui densidade de 0,92 g/mL. Essa soluo apresenta a) b) c) d) e) 920 g de metanal em 1 L de gua. 40 g de metanal em 100 mL de gua. 4 g de metanal em 920 g de soluo. 4 g de metanal em 10 g de soluo. 9,2 g de metanal em 100 mL de gua.

27. Uma soluo aquosa de cido sulfrico (H2SO4), para ser utilizada em baterias de chumbo de veculos automotivos, deve apresentar concentrao igual a 4 mol/L. O volume total de uma soluo adequada para se utilizar nestas baterias, que pode ser obtido a partir de 500 mL de soluo de H2SO4 de concentrao 18 mol/L, igual a a) 0,50 L b) 2,00 L c) 2,25 L d) 4,50 L e) 9,00 L

741

QUMICA

UFRGS

28. Considere o grfico abaixo que representa as variaes das presses mximas de vapor da gua pura (A.P.) e duas amostras lquidas A e B, em funo da temperatura.

Pode-se concluir que, em temperaturas iguais, a) b) c) d) e) a amostra A constitui-se de um lquido menos voltil que a gua pura. a amostra B pode ser constituda de uma soluo aquosa de cloreto de sdio. a amostra B constitui-se de um lquido que evapora mais rapidamente que a gua pura. a amostra A pode ser constituda de soluo aquosa de sacarose. as amostras A e B constituem-se de solues aquosas preparadas com solutos diferentes.

29. Dadas as equaes termoqumicas, a 1 atm e 25C. I - CH4(g) + 2 O2(g) CO2(g) + 2 H2O(l) H I = -888 kJ/mol II - C2H6O(l) + 3 O2(g) 2 CO2(g) + 3 H2O(l) H II = - 1373 kJ/mol III - C8H18(l) + 25/2 O2(g) 8 CO2(g) + 9 H2O(g) H III = - 5110 kJ/mol IV - H2(g) +
1/ 2

O2(g)

H2O(l) H IV 286 kJ/mol


=

V - C(grafite)

O2(g)

CO2(g) H V = 393,5 kJ/mol

o combustvel que libera a maior quantidade de calor, por grama consumido, a) CH4(g) b) C2H6O(l) c) C H18(l)
8

d) H2(g) e) C(grafite)

742

QUMICA

UFRGS

30. A reao de neutralizao entre um cido forte e uma base forte libera uma quantidade de calor constante e igual a 58 kJ por mol de H2O formado. Das reaes representadas abaixo, apresenta H = - 58 kJ/mol a reao a) NH4OH + HCl NH4Cl + H2O b) H2S + NaOH NaHS + H2O c) KOH + HCl KCl + H2O d) H2SO4 + Ca(OH)2 CaSO4 + 2 H2O e) HCN + CuOH CuCN + H2O 31. O carvo um combustvel constitudo de uma mistura de compostos ricos em carbono. A situao em que a forma de apresentao do combustvel, do comburente e a temperatura utilizada favorecero a combusto do carbono com maior velocidade : Combustvel a) b) c) d) e) carvo em pedaos carvo pulverizado carvo em pedaos carvo pulverizado carvo em pedaos Comburente ar atmosfrico ar atmosfrico oxignio puro oxignio puro oxignio liquefeito Temperatura (C) 0 30 20 100 50 Situao I

32. As figuras ao lado representam as colises entre as molculas reagentes de uma mesma reao em trs situaes. Pode-se afirmar que a) na situao I, as molculas reagentes apresentam energia maior que a energia de ativao, mas a geometria da coliso no favorece a formao dos produtos. b) na situao II, ocorreu uma coliso com geometria favorvel e energia suficiente para formar os produtos. c) na situao III, as molculas reagentes foram completamente transformadas em produtos. d) nas situaes I e III, ocorreram reaes qumicas, pois as colises foram eficazes. e) nas situaes I, II e III, ocorreu a formao do complexo ativado, produzindo novas substncias.

Situao II

Situao III

743

QUMICA

UFRGS

33. Conhecendo-se o valor da constante de equilbrio (Kc), pode-se determinar a probabilidade de uma dada reao ocorrer na natureza. Considere duas reaes possveis para a fixao do nitrognio atmosfrico, convertendo-o em compostos utilizveis pelos seres vivos. I. II. N2(g) + N2(g) + O2(g) 3 H2(g) NO(g) Kc = 1x10
-30

a 25C
8

2 NH3(g) Kc = 5x10

a 25C

Para uma fixao eficiente N2, correto afirmar-se que a) a reao I no seria um processo adequado, pois a concentrao de NO, no equilbrio, seria extremamente pequena. b) a reao II no seria um processo adequado, pois o sistema em equilbrio consistiria, praticamente, do produto NH 3. c) a reao I seria um processo adequado, pois esta reao tende a se processar no sentido da formao de NO. d) a reao II seria um processo adequado, pois no equilbrio so grandes as concentraes do N2 e do H2. e) as duas reaes so processos adequados, pois quando o equilbrio atingido nas duas reaes, grande a concentrao de produtos. 34. Certas plantas apresentam flores cujas cores so influenciadas pelo pH do solo. Uma destas plantas apresenta florao rosa quando o solo tem caractersticas bsicas e florao azul quando o solo tem caractersticas cidas. Em funo disso, pode-se concluir que em solos onde a) [H ] > 10 a cor azul. -7 b) [OH ] < 10 a cor rosa. + c) [H ] > [OH ] a cor rosa.
+ -7

d) [H ] = 10 a cor azul. -6 e) [OH ] = 10 a cor azul.

-8

35. Aparelhos eletrnicos, como telefones sem fio e calculadoras, podem utilizar baterias recarregveis de nquel-cdmio. As reaes que ocorrem nesta bateria podem ser representadas por: A. Cd Cd B. Cd
2+ 2+

+ 2e
-

+ 2 OH Cd(OH)2
-

C. NiO2 + 2 H2O + 2 e Ni(OH)2 + 2 OH Sobre estas reaes so feitas as afirmaes:

I - A reao A ocorre no nodo da bateria. II - A reao C ocorre no ctodo da bateria. III - O cdmio o agente oxidante e o xido de nquel o agente redutor. Quais esto corretas? a) Apenas I b) Apenas II c) Apenas III d) Apenas I e II e) I, II e III

744

QUMICA

UnB

Nas questes de 1 a 3, marque: itens CERTOS, na coluna I; itens ERRADOS, na coluna II. Use, para as devidas marcaes, a Folha de Rascunho e, posteriormene, a Folha de Respostas. Questo 1 A Qumica est to presente na vida do homem, que difcil imaginar a vida sem a Qumica. Os produtos qumicos tm inmeras aplicaes, entre as quais ressalta-se a fabricao dos computadores, que constituem a revoluo deste final de sculo. Considerando a presena da Qumica no cotidiano, julgue os itens abaixo. (0) Apesar dos benefcios que os produtos qumicos trazem para a indstria, deve-se evitar a ingesto de quaisquer desses produtos. (1) A gua do mar uma substncia composta formada por gua (H2O) e cloreto de sdio (NaCl). (2) Um aqurio com muitos peixes deve ter sua gua borbulhada com ar para repor o oxignio que os peixes consomem das molculas de gua (H2O) durante a respirao. (3) O eventual processo de enferrujamento de componentes de um computador, confeccionados com determinado metal, um exemplo de transformao qumica. Questo 2 O entendimento da estrutura dos tomos no importante apenas para satisfazer curiosidade dos cientistas; possibilita a produo de novas tecnologias. Um exemplo disso a descoberta dos raios catdicos, feita pelo fsico William Crookes, enquanto estudava as propriedades da eletricidade. Tal descoberta, alm de ter contribudo para um melhor entendimento a respeito da constituio da matria, deu origem aos tubos de imagem dos televisores e dos monitores dos computadores. Alguns grandes cientistas que contriburam para o entendimento da estrutura do tomo foram: Dalton (1766-1844), Rutherford (1871-1937), Bohr (1885-1962) e Linus Pauling (1901-1994). Com relao estrutura da matria, julgue os itens seguintes. (0) Ao passar entre duas placas eletricamente carregadas, uma positivamente e outra negativamente, as partculas alfa desviam-se para o lado da placa negativa. (1) O tomo a menor partcula que constitui a matria. (2) Cada tipo de elemento qumico caracterizado por um determinado nmero de massa. (3) O modelo atmico que representa exatamente o comportamento do eltron o modelo de Rutherford-Bohr.

745

QUMICA
Questo 3

UnB

Para produzir a grande quantidade de materiais de que necessita, o homem tem ao seu dispor cerca de 90 diferentes tipos de tomos, sem contar com os elementos artificiais que no so utilizados pela indstria. Para melhor organizar as informaes sobre os elementos qumicos, o cientista russo Mendeleyev props a utilizao de uma tabela peridica similar que se utiliza hoje. Na indstria de computadores, dois elementos importantssimos so o silcio (elemento que constitui aproximadamente 27,2% da massa da crosta terrestre) e o germnio, que so utilizados para a confeco dos chips. Estes, cada vez menores, mais complexos e eficientes, j so feitos tambm a partir de diamantes (sintticos), que so uma forma alotrpica do carbono. No caso da memria de um computador, os chips possuem a seguinte estrutura:

camada de silcio camada de dixido de silcio camada de silcio contaminada (dopada)


com tomos de fsforo

Com o auxlio das informaes contidas no enunciado e na tabela fornecida nesta prova, julgue os itens que se seguem. (0) Os smbolos qumicos do silcio e do fsforo so, respectivamente, S e F. (1) Alguns elementos que constam da tabela peridica recebem o nome de gases nobres porque no reagem com nenhuma substncia. (2) O silcio e o fsforo so usados na fabricao dos chips porque pertencem mesma famlia na tabela peridica. (3) A utilizao do germnio, em vez do silcio, para a confeco de chips, pode ser entendida a partir de uma anlise da tabela peridica. Questo 4 Os polmeros, sintetizados pela primeira vez em 1862 por Alexander Parkes, so utilizados na produo de inmeros materiais, entre eles, os teclados, as capas dos fios e todos os componentes plsticos dos computadores. O que permite essa diversidade de materiais so as ligaes existentes entre os tomos, resultando em diferentes propriedades das substncias. Com relao s ligaes qumicas e s estruturas das substncias, julgue os itens a seguir. (0) Os metais so bons condutores de eletricidade devido s ligaes metlicas, nas quais os eltrons esto livres, podendo mover-se entre os ncleos. (1) A substncia bromo (Br2), formada por um elemento muito eletronegativo, mais solvel em hexano (solvente de baixa polaridade) do que em gua. + + (2) Sabendo-se que o HCl dissolvido em gua forma as espcies inicas H (ou H3O ) e Cl , conclui-se que suas molculas so formadas por ligaes inicas.

746

QUMICA
Questo 5

UnB

Os microprocessadores atuais so muito pequenos e substituram enormes placas contendo inmeras vlvulas. Eles so organizados de forma que apresentem determinadas respostas ao serem percorridos por um impulso eltrico. S possvel a construo de dispositivos to pequenos devido ao diminuto tamanho dos tomos. Sendo estes muito pequenos, impossvel cont-los. A constante de Avogadro e no o nmero de Avogadro permite que se calcule o nmero de entidades tomos, molculas, frmulas unitrias etc. presentes em uma dada amostra de substncia. O valor dessa constante, medido experimentalmente, igual a 23 -1 6,02 x 10 mol . Com relao ao assunto, julgue os seguintes itens. (0) A constante de Avogadro uma grandeza, sendo, portanto, um nmero (6,02 x 1023) multiplicado por uma unidade de medida (mol-1). (1) A constante de Avogadro, por ser uma grandeza determinada experimentalmente, pode ter seu valor alterado em funo do avano tecnolgico. (2) Massas iguais de diferentes elementos qumicos contm o mesmo nmero de tomos. (3) Entre os elementos qumicos, o nico que, em princpio, no est sujeito a uma variao de massa atmica o istopo do carbono de massa 12,00 u. Questo 6 Nas lojas de informtica, possvel comprar frascos contendo um lquido para limpeza dos CDs dos kits de multimdia. Julgue os itens seguintes, relativos a esse lquido, que o 2-propanol (isopropanol). (0) (1) (2) (3) 0 2-propanol mais voltil que o metanol. O 1-propanol e o 2-propanol so ismeros geomtricos. O 2-propanol mais solvel em gua que o metanol. A oxidao do 2-propanol no ambiente produz a 2-propanona.

Questo 7 Um estudante estava digitando o seu trabalho didtico de Qumica e, inadvertidamente, comia um sanduche, cujo molho continha vinagre. Acidentalmente, o molho caiu no teclado. Sabendo -5 que a constante de dissociao do cido actico igual a 1,8 x 10 mol/L, julgue os itens a seguir, a respeito dos conceitos relacionados s conseqncias qumicas provocadas pelo acidente. (0) A concentrao hidrogeninica da soluo derramada sobre o teclado provavelmente deve -7 ser maior do que 1 x 10 mol/L. (1) Se um aluno fizesse a limpeza do teclado com algum material contendo cido, provavelmente deslocaria o equilbrio de dissociao do cido actico para o lado de sua + forma no-ionizada (CH3COOH = CH3COO +H ). (2) Aps o cido actico atingir o seu estado de equilbrio na soluo derramada, a concentrao de acetato permanece inalterada, apesar de ons acetato continuarem reagindo + com ons H .

747

QUMICA
Questo 8

UnB

As atividades do qumico incluem identificar a composio das substncias e determinar a sua concentrao nos materiais. Para a realizao de tais atividades, so utilizados atualmente equipamentos analticos, entre os quais os instrumentos espectrofotomtricos, de alta preciso e sensibilidade. Esses equipamentos possuem um sistema computacional acoplado que processa as informaes obtidas pelo instrumento, fornecendo ao analista a identificao dos elementos qumicos presentes na substncia, bem como a sua concentrao. A instalao e a manuteno desses equipamentos em laboratrio exigem alguns cuidados bsicos, em funo da existncia de sistemas eletrnicos de microprocessamento. Julgue os itens que se seguem, relativos ao problema da conservao desses instrumentos. (0) A necessidade de manter esses equipamentos em compartimento fechado, anexo ao laboratrio, pode ser justificada pela utilizao de substncias com baixo ponto de ebulio e que contaminam o ambiente. (1) A teoria cintico-molecular demonstra que, em dias quentes, os vapores e gases emitidos no laboratrio podero atacar o sistema eletrnico dos equipamentos com maior intensidade do que em dias frios. (2) Em laboratrios situados em regies geogrficas de elevada altitude, a vaporizao de substncias volteis ser mais rpida do que em laboratrios localizados em regies prximas ao nvel do mar. Questo 9 Fitas de vdeo, fitas K-7 e disquetes de computadores so confeccionados misturando-se um material magntico (xido de ferro) a um material polimrico. O teclado, o gabinete e o monitor so tambm confeccionados usando-se diferentes tipos de plsticos. Ao lado esto representadas as estruturas de alguns plsticos comuns. Com o auxlio das representaes das estruturas, julgue os itens seguintes. (0) No PVC, a estrutura que se repete (- CH - CH - CH -). 2 Cl Cl (1) O poliestireno um polmero contendo anis aromticos. (2) A borracha uma poliolefina (polieno). (3) O polietilenotereftalato um polister. (4) Os polmeros so tambm chamados de macromolculas. Faltou uma parte do scan

748

QUMICA
Questo 10

UnB

Ao acessar a rede Internet, procurando algum texto a respeito do tema radioatividade no Cade? (http://www.cade.com.br), um jovem deparou-se com a seguinte figura, representativa do poder de penetrao de diferentes tipos de radiao.

Com o auxlio da figura, julgue os itens abaixo. (0) (1) (2) (3) A radiao esquematizada em II representa o poder de penetrao das partculas beta. A radiao esquematizada em III representa o poder de penetrao das partculas alfa. As partculas alfa e beta so neutras. Quando um ncleo radioativo emite uma radiao do tipo I, o nmero atmico fica inalterado.

Questo 11 A explicao da utilizao de ouro no recobrimento dos contatos eletrnicos das placas dos computadores est relacionada com o potencial eletroqumico dos metais.
SEMI-REAO 3e - Al Al
3-

E (volts) -1,66 -0,44 -0,34 -1,50

2e - Fe Fe
222-

2e - Cu Cu
-

3e - Au Au

Analisando a tabela de potencial de reduo dos metais mostrada acima e utilizando os conceitos relativos oxirreduo, julgue os itens seguintes. (0) O estado de oxidao do ction Al3+ indica que esse ction possui trs prtons a mais do que seu nmero de eltrons. (1) Em um composto covalente, se o nmero de oxidao de um tomo positivo, ele indica a quantidade de eltrons que esse tomo efetivamente perdeu na ligao qumica. (2) Na reao Al + Au3 Al3+ + Au, o on ouro o agente redutor. (3) Fios de cobre so mais facilmente oxidados do que fios de alumnio.

749

QUMICA
Questo 12

UnB

Um estudante, consultando um livro didtico de Qumica, encontrou uma experincia que lhe chamou a ateno. No dispondo dos reagentes em sua escola, ele solicitou, por meio da Internet, no site http://www.unb.br/qui/lpeq/, informaes sobre os resultados que poderiam ser obtidos na experincia. Tendo recebido, via Internet, a tabela de dados, o aluno elaborou o seguinte relatrio. Experincia: estudo sobre a velocidade de reao. Procedimento: preparando-se tubos de ensaio em diferentes concentraes de soluo de tiossulfato de sdio e em diferentes condies de temperatura, conforme especificado na tabela de dados, adicionaram-se quatro gotas de cido sulfrico em cada tubo, medindo-se imediatamente o tempo durante o qual a soluo ficou turva, no permitindo a visualizao de um trao feito a lpis em uma tira de papel que estava atrs do tubo. Tabela de dados:
TUBO 1 2 3 4 5 6 CONCENTRAO DA SOLUO DE NA2S2O3 (MOL/L) 0,5 0,5 0,5 0,4 0,3 0,2 CONDIO DE TEMPERATURA DA SOLUO gelada quente ambiente ambiente ambiente ambiente TEMPO (S) 20 5 10 15 20 30

Anlise de dados: o tiossulfato reage com o cido sulfrico, produzindo um precipitado enxofre o qual turva a soluo. A variao das condies da reao afeta a velocidade de formao do produto. Equao: Na2S2O3 + H2SO4 Na2SO4 + H2O + SO2 + S H < 0 Com o auxlio das informaes acima, julgue os itens a seguir. (0) O aluno pode concluir corretamente que a concentrao e a temperatura afetam a velocidade da reao. (1) A entalpia da reao variou nos primeiros trs tubos. (2) A energia de ligao dos produtos maior que a energia de ligao dos reagentes. (3) No tubo 1, a energia cintica dos reagentes foi maior do que no tubo 2; por isso, o tempo da reao foi maior.

750

QUMICA
Leia o texto a seguir para responder s questes 13 e 14.

UnB

O processo de fabricao dos circuitos integrados impressos, usados na construo de microcomputadores, emprega o cido sulfrico de alta pureza. Sendo ele um cido muito forte, o resduo industrial do processo necessita ser tratado antes de ser lanado no meio ambiente.

Questo 13 Com o auxlio do texto e considerando que o resduo , geralmente, tratado com hidrxido de sdio, julgue os itens abaixo. (0) Na reao do cido sulfrico com o hidrxido de sdio, um dos produtos o Na2SO4. (1) Se o resduo industrial for lanado em um rio antes do tratamento com hidrxido de sdio, o pH das guas desse rio ser aumentado. (2) A reao entre o cido sulfrico e o hidrxido de sdio conhecida como reao de neutralizao. (3) cido sulfrico diludo encontrado nas chuvas cidas que ocorrem em plos industriais. Nas questes 14 e 15, marque: o algarismo das DEZENAS, na coluna I; o algarismo das UNIDADES, na coluna II. O algarismo das DEZENAS deve ser obrigatoriamente marcado, mesmo que seja igual a zero. Use, para as devidas marcaes, a Folha de Rascunho e, posteriormente, a Folha de Respostas. Questo 14 Uma indstria resolveu alterar o procedimento normal e tratar seu resduo com cal hidratada (hidrxido de clcio). A partir dos dados apresentados na tabela peridica, calcule, em toneladas, a quantidade de sulfato de clcio que ser produzida ao se neutralizarem 49 toneladas de cido sulfrico. Desconsidere a parte fracionria do resultado, caso exista.

Questo 15 Consultando a tabela de potencial de reduo dos metais apresentada na questo 11, calcule, em volts, a diferena de potencial (tenso eltrica) produzida em uma pilha que tenha eletrodos de ferro e de cobre. Para marcar a sua resposta, multiplique o valor calculado por 100 e despreze a parte fracionria do resultado, caso exista.

751

QUMICA
1. O rtulo de uma gar rafa de gua miner al est r eproduzido a seguir. Composio Qumica Provvel: Sulfato de clcio Bicabornato de clcio 0,0038 mg/L 0,0167 mg/L

UNESP
F A S E

Com base nestas informaes, podemos classificar a gua miner al como a) substncia pura. b) substncia simples. c) mistura heterognea. d) mistura homognea. e) suspenso coloidal.

2. Considere as seguintes afirmaes sobre tomos e molculas. I - No modelo proposto por Rutherford, o tomo tem praticamente toda sua massa concentrada num ncleo pequeno e os eltrons esto a uma grande distncia do ncleo. II - No modelo proposto por Bohr par a o tomo de hidrognio, os eltrons se movem em rbitas cir culares, cujas energias podem assumir quaisquer valores. III - Molcula a menor poro de uma substncia co valente que mantm sua composio. Est(o) correta(s): a) apenas I. b) apenas II. c) apenas III. d) apenas I e II. e) apenas I e III.

3. O ciclo do nitrognio na natureza pode ser representado pelo esquema

Neste ciclo, o nitrognio sofre um processo de a) oxidao na etapa (V). b) oxidao em todas as etapas. c) reduo na etapa (I). d) reduo na etapa (VI). e) reduo em todas as etapas.

752

QUMICA

UNESP
F A S E

4. Um composto de carbono, hidrognio e oxignio apresenta na sua constituio 40,0% de carbono e 6,6% de hidr ognio. A sua frmula mnima a) CHO. b) CH2O. c) CHO2. d) C2HO. e) C2H2O.

Massas molares, em g/mol: H=1; C=12; O=16.

5. Os elementos X eY tm,respectivamente, 2 e 6 eltrons na camada de v alncia. Quando X e Y reagem, forma-se um composto a) covalente, de frmula XY . b) covalente, de frmula XY . 2 c) covalente, de frmula X2Y3. d) inico, de frmula X Y . + 2e) inico, de frmula X 2Y .
2+ 2-

6. Tm-se trs solues aquosas distintas, respectivamente de NaCl, NaCN e sacarose (C12H22O11), todas de concentrao 0,1 mol/L. Elas poderiam ser distinguidas pelo sabor, o que no deve ser feito, porque NaCN um veneno violento. Estas solues podem ser identificadas por a) b) c) d) e) medidas de presso osmtica e ponto de ebulio. cor e medidas de ponto de congelamento. medidas de condutividade eltrica e de pH. precipitao com bases diludas. precipitao com sulfato de potssio.

7. O equilbrio gasoso representado pela equao N2 + O 2 2NO deslocado no sentido de formao de NO, se a) b) c) d) e) a presso for abaixada. N2 for retirado do sistema. a temperatura for aumentada. for adicionado um catalisador slido ao sistema. o volume do recipiente for diminudo. H = + 88 kJ

753

QUMICA

UNESP
F A S E

8. O acidente do reator nuclear de Chernobyl, em 1986, lanou para a atmosfera grande quantidade de
90 38 Sr radioativo, cuja meia-vida de 28 anos. Supondo ser este istopo a nica contaminao 90 38Sr

radioativa e sabendo que o local poder ser considerado seguro quando a quantidade de reduzir, por desintegrao, a

se

1 da quantidade inicialmente pr esente, o local poder ser 16

habitado novamente a par tir do ano de a) 2014. b) 2098. c) 2266. d) 2986. e) 3000.

9. O solo do cerrado cido. Para diminuir a acidez, deve-se adicionar ao solo a) CaSO4. b) KCl. c) NH4NO3. d) (H2N)2CO. e) CaCO 3.

10. Na obteno de margar ina a par tir da hidr ogenao de leos vegetais, uma das reaes que ocorre representada por H3C - (CH2)7 - CH = CH - (CH 2) - COOH + H 2
7

cido olico H3C - (CH2)16 - COOH cido esterico A respeito deste processo, so feitas as trs se guintes afirmaes. I - A transformao de cido olico em ester ico envolve uma reao de adio. II - Dos dois cidos, somente o olico apresenta isomeria cis-trans. III - O cido esterico mais resistente oxidao pelo oxignio do ar que o cido olico. Est(o) correta(s): a) apenas I. b) apenas II. c) apenas I e III. d) apenas II e III. e) I, II e III.

754

QUMICA
11. O amino-cido tr iptofano, de frmula estr utural

UNESP
F A S E

se transforma no neurotransmissor serotonina aps sofrer duas reaes, uma de hidroxilao do anel benznico e outra de descarbo xilao (perda de CO 2). A molcula de serotonina apresenta as funes qumicas a) lcool e amida. b) lcool e cetona. c) cido e amida. d) fenol e amina. e) fenol e cido.

12. Assinale a alter nativa que a presenta um composto orgnico que descor a rapidamente uma soluo de bromo em tetracloreto de carbono. a) Butino-2. b) Butano. c) ter etlico. d) Benzeno. e) Clorofrmio.

1. Nquel metlico puro pode ser obtido pela decomposio trmica do tetracarbonilnquel, Ni(CO)4. No processo liberado o mesmo gs txico que usualmente provm do escapamento de automveis. a) Escreva a equao qumica balanceada da decomposio do Ni(CO) 4. b) Calcule a massa de nquel metlico pur o, expressa em gramas, que pode ser obtida pela decomposio estequiomtrica de 3,414 kg de Ni(CO)4. Massas molares, em g/mol: Ni=58,7; C=12,0;O=16,0.

F A S E

2 A

2. A obteno de ferro metlico a par tir de um minrio envolve a etapa de equilbr io representada pela equao Fe 2O3 (s) + 3 CO (g) 2Fe (s) + 3 CO 2 (g)

P R O V A

a) Escreva a expresso da constante de equilbr io da reao. b) Discuta o efeito da retirada de ferro metlico sobre a posio do equilbrio, quando a reao r ealizada em condies de temperatura e volume constantes.

755

QUMICA
3. Considere a substncia natural de frmula estrutural

UNESP
F A S E

2 A

P R O V A

a) Quais so os g rupos funcionais existentes nessa molcula? b) Indique os tomos de carbono assimtricos dessa molcula. 4. Objetos de prata escurecem, principalmente pela formao de uma camada de sulfeto de prata (Ag2S) em sua superfcie. Uma forma de se limpar um objeto de prata escurecido consiste em envolv-lo em folha de papel alumnio e mergulh-lo em soluo aquosa de carbonato de sdio. a) Explique o princpio deste mtodo de limpeza. b) Ocorre diminuio da massa de prata do objeto nesse processo de limpeza? Justifique. 5. A reao entr e alumnio e xido de ferro (III) pulverizados exotrmica, e fornece como produtos ferro metlico e xido de alumnio (III) slidos. a) Escreva a equao balanceada da r eao, indicando os estados de a gregao de r eagentes e produtos. b) Calcule a var iao de entalpia deste pr ocesso qumico, a par tir das entalpias de r eao dadas a se guir: 2Fe(s) + 3/2 O 2(g) Fe 2O3(s) 2Al(s) + 3/2 O2(g) Al2O3(s) H1= 824,2 kJ/mol H2= 1676 kJ/mol

6. lcoois podem ser obtidos pela hidratao de alcenos, catalisada por cido sulfr ico. A reao de adio segue a regra de Mar konikov, que prev a adio do tomo de hidr ognio da gua ao tomo de carbono mais hidrogenado do alceno. Escreva: a) a equao qumica balanceada da r eao de hidr atao ca talisada do b uteno-1; b) o nome oficial do produto formado na reao indicada no item a.

756

QUMICA

UNESP
F A S E

1. Chuva cida resulta da combinao de gua a tmosfrica com dixido de enxofre ou com trixido de enxofre. Escreva: a) as equaes qumicas balanceadas das reaes de cada um dos dois xidos com gua; b) os nomes oficiais dos produtos das reaes dos dois xidos com gua. 2. 80,0 mL de uma soluo aquosa de hidrxido de potssio de concentrao 0,250 mol/L so parcialmente neutralizados por 20,0 mL de uma soluo aquosa de cido ntrico de concentrao 0,500 mol/L. a) Escreva a equao qumica da r eao de neutr alizao. b) Sabendo que pOH = - log[OH ] e que pH + pOH = 14, calcule o pH da soluo aps a adio do cido.
-

2 B

P R O V A

3. Esta questo se r efere a der ivados do cido pr opinico. a) Escreva as fr mulas estr uturais dos ismeros que podem ser obtidos a par tir do cido propinico, substituindo-se um tomo de hidrognio ligado a carbono por um g rupo -OH. b) Um dos ismeros obtidos no item a apresenta tambm atividade ptica, e conhecido como cido ltico. Com base nestas informaes, escreva o nome oficial do cido ltico. 4. Silcio elementar, na forma slida, pode ser obtido pela r eao entr e dixido de silcio pulverizado e magnsio metlico. a) Escreva a equao balanceada da r eao, indicando os estados de a gregao de r eagentes e produtos. b) Calcule a var iao de entalpia deste pr ocesso qumico a par tir das entalpias de r eao dadas a se guir: Si (s) + O 2 (g) SiO2(s) H1 = - 910,9 kJ/mol H2 = -601,8 kJ/mol

Mg (s) + 1/2 O 2(g) MgO (s)

757

QUMICA

UNESP
F A S E

5. O grfico mostra as cur vas de solubilidade em gua, em funo da temperatura, dos sais KNO3 e MnSO4.

2 B

P R O V A

Com base neste grfico, discuta se as afirmaes a e b so verdadeiras ou falsas. a) O processo de dissoluo dos dois sais endotrmico. b) 100 mL de soluo saturada a 56 C contm aproximadamente 10 g de KNO3. 6. O equilbrio gasoso N2O4 2NO2

apresenta a uma dada temperatura, constante de equilbrio K c = 2. Nesta temperatura foram feitas duas misturas diferentes, A e B, cada uma acondicionada em r ecipiente fechado, isolado e distinto. As condies iniciais esto mostradas na ta bela.
MISTURA A B [NO2] / mol/L 2 x 10 -1 2 x 10
-2

[N2O4]/mol/L 2 x 10 -3 1 x 10
-4

a) Efetue os clculos necessr ios e conclua se a mistura A se encontra ou no em situao de equilbrio. b) Efetue os clculos necessrios e conclua se a mistura B se encontra ou no em situao de equilbrio.

758

S-ar putea să vă placă și